{ "cells": [ { "cell_type": "markdown", "metadata": {}, "source": [ "7. Multi-variable Calculus\n", "===\n", "[13.1 Functions of Several Variables](6%20Multi-variable%20Calculus-Differentiation-1.ipynb#Functions-of-Several-Variables)
\n", "[13.2 Limits and Continuuity](6%20Multi-variable%20Calculus-Differentiation-1.ipynb#Limit-and-Continuity)
\n", "[13.3 Partial Differentiation](6%20Multi-variable%20Calculus-Differentiation-2.ipynb#Partial-Differentiation)
\n", "[13.4 Chain Rule](6%20Multi-variable%20Calculus-Differentiation-2.ipynb#Chain-Rule)
\n", "[13.5 Tangent Plane](6%20Multi-variable%20Calculus-Differentiation-2.ipynb#Tangent-Plane)
\n", "[13.8 Relative Extrema](#Relative-Maxima-and-Minima)
\n", "[13.9 Lagrange Multiplier](#Optimization-Problem-with-Constraints)
\n", "[13.8 Method of Least Squares](#The-Method-of-Least-Squares)" ] }, { "cell_type": "code", "execution_count": 2, "metadata": {}, "outputs": [ { "data": { "text/html": [ "\n", "\n", "\n", "\n", "\n", "\n", "\n" ], "text/plain": [ "" ] }, "execution_count": 2, "metadata": {}, "output_type": "execute_result" } ], "source": [ "from IPython.core.display import HTML\n", "css_file = 'css/ngcmstyle.css'\n", "HTML(open(css_file, \"r\").read())" ] }, { "cell_type": "code", "execution_count": 4, "metadata": { "collapsed": true }, "outputs": [], "source": [ "%matplotlib inline\n", "\n", "#rcParams['figure.figsize'] = (10,3) #wide graphs by default\n", "import scipy\n", "import numpy as np\n", "import time\n", "from mpl_toolkits.mplot3d import Axes3D\n", "from IPython.display import clear_output,display\n", "import matplotlib.pylab as plt\n", "from matplotlib import cm\n", "plt.style.use('ggplot')" ] }, { "cell_type": "code", "execution_count": 5, "metadata": { "collapsed": true }, "outputs": [], "source": [ "from sympy import hessian,symbols,solve,diff,sin,cos,pi\n", "\n", "grad = lambda func, vars :[diff(func,var) for var in vars]\n", "\n", "x,y,z=symbols(\"x y z\",real=True)" ] }, { "cell_type": "code", "execution_count": 3, "metadata": {}, "outputs": [ { "data": { "text/html": [ "" ], "text/plain": [ "" ] }, "metadata": {}, "output_type": "display_data" } ], "source": [ "import plotly.plotly as py\n", "from plotly.offline import download_plotlyjs, init_notebook_mode, plot, iplot,iplot_mpl\n", "init_notebook_mode()" ] }, { "cell_type": "markdown", "metadata": {}, "source": [ "Relative Maxima and Minima\n", "---\n", "\n", "From now on, let $\\vec{\\mathbf{x}} = (x^1, \\cdots, x^n) \\in \\mathbb{R}^n$ and\n", "$\\overrightarrow{\\mathbf{x}_0} = (x_0^1, \\cdots, x^n_0) \\in \\mathbb{R}^n$.\n", "Also let $f_i (\\vec{x}) = \\frac{\\partial f}{\\partial x^i}$ be the $i$-th\n", "partial derivative.\n", "\n", "Definition\n", "---\n", "\n", "$z_0 = f (\\vec{\\mathbf{x\\!}}_0)$ is called a relative maximum if there is $r> 0$ such that we have \n", "$f (\\vec{\\mathbf{\\!x}}) \\le f(\\vec{\\mathbf{x}}_0) $\n", "for $| \\vec{\\mathbf{x}} - \\vec{\\mathbf{x}}_0 |< r$. $z_0 = f (\\vec{\\mathbf{x}}_0)$ \n", "is called a relative minimum if there is $r > 0$ such that we have \n", "$f (\\vec{\\mathbf{x}}) \\ge f(\\vec{\\mathbf{x}}_0) $ for $| \\vec{\\mathbf{x}} - \\vec{\\mathbf{x}}_0 |< r$.\n", "\n", "\n", "Theorem\n", "---\n", "\n", "If $z_0 = f (\\vec{\\mathbf{x}}_0)$ is a relative extremum then $f_i(\\vec{\\mathbf{x}}_0) = 0$ for all $i = 1, \\cdots, n$ if they exist. All such points are called **critical points** or\n", "**stationary points**.\n", "\n", "\n", "Although, the theorem can not state at which function attains its extrema, but\n", "we can still use this theorem to find out all the places at which extrema of\n", "functions attain." ] }, { "cell_type": "markdown", "metadata": {}, "source": [ "Example\n", "---\n", " Find all the critical points of the following functions:\n", "\n", "**1.** $f (x, y) = x^2 + 4 y^2$. (reference the following picture at left) \n", "\n", "$f_x = 2 x = 0, f_y = 8 y = 0$ implies $(x, y) = (0, 0)$ is the only\n", " critical point of $f (x, y)$. Since $f (x, y) \\ge0 = f (0, 0)$,\n", " $f (x, y)$ attains its minimum at $(0, 0)$.\n", "\n", "**2.** For $f (x, y) = x^2 - 4 y^2$ (reference the following picture at right) ,\n", "$f_x = 2 x = 0, f_y = - 8 y = 0$ implies $(x, y) = (0,0)$ which is also the only one critical point of $f (x, y)$. But $f (0, 0)$ can not be any extremum since\n", "\n", "$$ f (\\delta, 0) \\geqslant f (0, 0) \\ge f (0, \\delta) \\text{ for any } \\delta > 0 $$\n", "\n", "**3.** From the condition,\n", "\n", "$$ f_x = 4 x - 1 = 0, f_y = 2 y - 2 = 0, f_z = 8 z = 0 $$\n", "the critical point is $(\\frac{1}{4}, 1, 0)$ and $f (x, y, z)$ attainsits relative minimum at this critical point.\n", "\n", "**4.** $f(x,y)=x^2+y^2-4x-6y+17=(x-2)^2+(y-3)^2+4\\ge4$: this implies only critical point at $(x,y)=(2,3)$ which attains its minimuum, 4.\n", "\n", "**5.** $f(x,y)=3-\\sqrt{x^2+y^2}\\le3$: critical point $(0,0)$ and maximum is 3:\n", "$$\\nabla f=\\left[\\frac{-x}{\\sqrt{x^2+y^2}},\\frac{-y}{\\sqrt{x^2+y^2}}\\right]=(0,0)$$\n", "Note: $\\nabla f $ fails to exit at $(0,0)$." ] }, { "cell_type": "code", "execution_count": 4, "metadata": {}, "outputs": [ { "data": { "text/plain": [ "" ] }, "execution_count": 4, "metadata": {}, "output_type": "execute_result" }, { "data": { "image/png": "iVBORw0KGgoAAAANSUhEUgAAArMAAAI1CAYAAADFDnLeAAAABHNCSVQICAgIfAhkiAAAAAlwSFlz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\nhUKhUCiUsIUmsxQKhUKhUCiUsOX/AwQ1JrMA+QsPAAAAAElFTkSuQmCC\n", "text/plain": [ "" ] }, "metadata": {}, "output_type": "display_data" } ], "source": [ "fig = plt.figure(figsize=(12,10))\n", "ax1 = fig.add_subplot(2, 2, 1, projection='3d')\n", "#ax1 = fig.gca(projection='3d')\n", "X = np.arange(-1.2, 1.2, 0.04)\n", "Y = np.arange(-1.2, 1.2, 0.04)\n", "X, Y = np.meshgrid(X, Y)\n", "g= X**2 + 4 *Y**2\n", "ax1.plot_surface(X, Y, g)\n", "ax1.set_title(\"$f(x,y)=x^2+4y^2$\")\n", "\n", "ax = fig.add_subplot(2, 2, 2, projection='3d')\n", "X = np.arange(-1.2, 1.2, 0.04)\n", "Y = np.arange(-1.2, 1.2, 0.04)\n", "X, Y = np.meshgrid(X, Y)\n", "f= X**2 - 4 *Y**2\n", "ax.plot_wireframe(X, Y, f)\n", "ax.set_title(\"$f(x,y)=x^2-4y^2$\")\n", "\n", "\n", "ax1 = fig.add_subplot(2, 2, 3, projection='3d')\n", "#ax1 = fig.gca(projection='3d')\n", "X = np.arange(1, 3, 0.04)\n", "Y = np.arange(2, 4, 0.04)\n", "X, Y = np.meshgrid(X, Y)\n", "g= (X-2)**2 + (Y-3)**2+4\n", "ax1.plot_surface(X, Y, g)\n", "ax1.set_title(\"$f(x,y)=(x-2)^2+(y-3)^2+4$\")\n", "\n", "\n", "from numpy import sqrt,pi\n", "ax = fig.add_subplot(2, 2, 4, projection='3d')\n", "X = np.arange(-2, 2, 0.04)\n", "Y = np.arange(-2, 2, 0.04)\n", "X, Y = np.meshgrid(X, Y)\n", "f= 3-sqrt(X**2 + Y**2)\n", "ax.plot_wireframe(X, Y, f)\n", "ax.set_title(\"$3-\\sqrt{x^2+y^2}$\")\n" ] }, { "cell_type": "code", "execution_count": 25, "metadata": {}, "outputs": [ { "data": { "text/plain": [ "{x: 1/4, z: 0, y: 1}" ] }, "execution_count": 25, "metadata": {}, "output_type": "execute_result" } ], "source": [ "from sympy import solve\n", "x,y,z=symbols(\"x y z\")\n", "f=2*x*x+y*y+4*z*z-x-2*y\n", "df=grad(f,[x,y,z])\n", "solve(df,[x,y,z])" ] }, { "cell_type": "markdown", "metadata": {}, "source": [ "Example\n", "---\n", "\n", "Suppose that $x^i, i = 1, 2, \\cdots, n$ satisfies\n", "\n", "$$ f_{\\mu, \\sigma} (x^i) = \\frac{1}{\\sqrt{2 \\pi} \\sigma} \\exp \\left( -\n", " \\frac{(x^i - \\mu)^2}{2 \\sigma^2} \\right) $$\n", "Define $L (\\mu, \\sigma^2)$ as follows:\n", "\n", "\\begin{eqnarray*}\n", " L (\\mu, \\sigma^2) & = & \\prod_{i = 1}^n f_{\\mu, \\sigma} (x^i)\\\\\n", " & = & \\prod_{i = 1}^n \\left[ \\frac{1}{\\sqrt{2 \\pi} \\sigma} \\exp \\left( -\n", " \\frac{(x^i - \\mu)^2}{2 \\sigma^2} \\right) \\right]\\\\\n", " & = & \\frac{1}{(2 \\pi \\sigma^2)^{n / 2}} \\exp \\left( - \\sum_{i = 1}^n\n", " \\frac{(x^i - \\mu)^2}{2 \\sigma^2} \\right)\n", " \\end{eqnarray*}\n", " What values of $(\\mu, \\sigma^2)$ will makes $L (\\mu, \\sigma^2)$ attains its\n", " maximum? Before answering this question, note that\n", "\n", "- assume $(\\hat{\\mu}, \\hat{\\sigma}^2)$ is the value such that\n", " maximizes $L (\\mu, \\sigma^2)$;\n", "- Suppose that the value of $(\\hat{\\mu}, \\hat{\\sigma}^2)$ maximizes $L\n", " (\\mu, \\sigma^2)$. This it also maximizes $\\ln L (\\mu, \\sigma^2)$. This\n", " means that we have to find the value of $(\\hat{\\mu}, \\hat{\\sigma}^2)$\n", " that maximizes the following:\n", "\n", "\\begin{eqnarray*}\n", " \\ln L (\\mu, \\sigma^2) & = & \\ln \\left[ \\frac{1}{(2 \\pi \\sigma^2)^{n /\n", " 2}} \\exp \\left( - \\sum_{i = 1}^n \\frac{(x^i - \\mu)^2}{2 \\sigma^2}\n", " \\right) \\right]\\\\\n", " & = & - \\frac{n}{2} (\\ln 2 \\pi + \\ln \\sigma^2) - \\left( \\frac{1}{2\n", " \\sigma^2} \\sum_{i = 1}^n (x^i - \\mu)^2 \\right)\n", " \\end{eqnarray*}\n", " \n", "Then the critical value $(\\hat{\\mu}, \\hat{\\sigma}^2)$ satisfies:\n", "\n", "\\begin{eqnarray*}\n", " 0 & = & \\frac{\\partial}{\\partial \\mu} \\ln L (\\mu, \\sigma^2)\\\\\n", " & = & \\frac{1}{\\sigma^2} \\sum_{i = 1}^n (x^i - \\mu)\\\\\n", " 0 & = & \\frac{\\partial}{\\partial \\sigma^2} \\ln L (\\mu, \\sigma)\\\\\n", " & = & - \\frac{n}{2 \\sigma^2} + \\frac{1}{2 (\\sigma^2)^2} \\sum_{i = 1}^n\n", " (x^i - \\mu)^2\n", " \\end{eqnarray*}\n", " From the first result, we can get the value, $\\hat{\\mu}$, as:\n", "\n", "\\begin{eqnarray*}\n", " & \\frac{1}{\\sigma^2} \\sum_{i = 1}^n (x^i - \\mu) = 0 & \\\\\n", " \\Longrightarrow & \\sum_{i = 1}^n (x^i - \\mu) = 0 & \\\\\n", " \\Longrightarrow & n \\mu = \\sum_{i = 1}^n x^i & \\\\\n", " \\Longrightarrow & \\hat{\\mu} = \\sum_{i = 1}^n x^i / n = \\bar{x} & \n", " \\end{eqnarray*}\n", " i.e. $\\hat{\\mu}$ is the mean of sum of $x^i, i = 1, 2, \\cdots, n$,\n", " called **sample mean**. \n", "\n", "$$ - \\frac{n}{2 \\sigma^2} + \\frac{1}{2 (\\sigma^2)^2} \\sum_{i = 1}^n (x^i -\n", " \\mu)^2 = 0 $$\n", "we have:\n", "\n", "\\begin{eqnarray*}\n", " \\widehat{\\sigma^2} & = & \\frac{1}{n} \\sum_{i = 1}^n (x^i - \\hat{\\mu})^2\\\\\n", " & = & \\frac{1}{n} \\sum_{i = 1}^n (x^i - \\bar{x})^2\\\\\n", " & = & \\frac{1}{n} \\sum_{i = 1}^n (x^i)^2 - \\frac{2 \\bar{x}}{n} \\sum_{i =\n", " 1}^n x^i + \\frac{1}{n} \\sum_{i = 1}^n (\\bar{x})^2\\\\\n", " & = & \\frac{1}{n} \\sum_{i = 1}^n (x^i)^2 - 2 (\\bar{x})^2 + (\\bar{x})^2 =\n", " \\frac{1}{n} \\sum_{i = 1}^n (x^i)^2 - (\\bar{x})^2\n", " \\end{eqnarray*}\n", " so called the **sample variance**.\n", " \n", "\n", "Note that the term in last result, $\\color{red}{- mx_0 - mx_1 + m^2}$, is\n", " equal to $\\color{red}{- m^2}$ since$\\sum x^i = n \\cdot \\bar{x}$\n", " \n", "Note\n", "---\n", "However, $\\widehat{\\sigma^2}$ is so-called biased estimator, since\n", "\n", "\\begin{eqnarray*}\n", " E \\widehat{\\sigma^2} & = & E \\left( \\frac{1}{n} \\sum_{i = 1}^n (x^i -\n", " \\bar{x})^2 \\right)\\\\\n", " & = & \\frac{1}{n} E \\sum_{i = 1}^n (x^i - \\mu - (\\bar{x} - \\mu))^2\\\\\n", " & = & \\frac{1}{n} \\sum_{i = 1}^n E \\left( \\frac{n - 1}{n} (x^i - \\mu)\n", " - \\frac{1}{n} \\sum^n_{j = 1 j \\neq i} (x^j - \\mu) \\right)^2\\\\\n", " & = & \\frac{1}{n} \\sum_{i = 1}^n \\left[ \\left( \\frac{n - 1}{n}\n", " \\right)^2 \\sigma^2 + \\frac{n - 1}{n^2} \\sigma^2 \\right]\\\\\n", " & = & \\frac{1}{n} \\sum_{i = 1}^n \\frac{n - 1}{n} \\sigma^2\\\\\n", " & = & \\frac{n - 1}{n} \\sigma^2 \\neq \\sigma^2\n", " \\end{eqnarray*}\n", "And \n", "\n", "$$n \\widehat{\\sigma^2} / (n - 1)=\\frac{1}{n - 1}\n", " \\sum\\limits^n_{i = 1} (x^i - \\bar{x})^2$$ \n", "is called the **unbiased estimator** for $\\sigma^2$ since\n", "\n", "$$ E \\left( \\frac{1}{n - 1} \\sum^n_{i = 1} (x^i -\n", " \\bar{x})^2 \\right) = \\sigma^2 . $$\n", "\n" ] }, { "cell_type": "code", "execution_count": 71, "metadata": {}, "outputs": [ { "data": { "text/plain": [ "[x_0/2 + x_1/2]" ] }, "execution_count": 71, "metadata": {}, "output_type": "execute_result" } ], "source": [ "\n", "from sympy import Matrix,symarray,log,symbols,pi,simplify\n", "\n", "x,mu,sigma=symbols(\"x m s\")\n", "n=2\n", "X=symarray('x',n)\n", "Xn=Matrix(X).transpose()\n", "M=Matrix(1,n,lambda i,j: mu )\n", "XX=(Xn-M)*(Xn-M).transpose()\n", "f=1/sqrt(2*pi*sigma)**n*exp(-XX[0]/(2*sigma))\n", "L=log(f)\n", "dL=grad(L,[mu,sigma])\n", "\n", "m_opt=solve(dL[0],mu)\n", "m_opt" ] }, { "cell_type": "code", "execution_count": 79, "metadata": {}, "outputs": [ { "data": { "text/plain": [ "2*pi*s*(-exp((-(-m + x_0)**2 - (-m + x_1)**2)/(2*s))/(2*pi*s**2) - (-(-m + x_0)**2 - (-m + x_1)**2)*exp((-(-m + x_0)**2 - (-m + x_1)**2)/(2*s))/(4*pi*s**3))*exp(-(-(-m + x_0)**2 - (-m + x_1)**2)/(2*s))" ] }, "execution_count": 79, "metadata": {}, "output_type": "execute_result" } ], "source": [ "# not completed\n", "mu,m_opt\n", "dL[1]\n", "#eq_s=sigma*(dL[1]*sigma).subs(mu,m_opt)\n", "#solve(eq_s,sigma)" ] }, { "cell_type": "markdown", "metadata": {}, "source": [ "Example\n", "---\n", "\n", "(Multi-product Monopoly) A company produces t$\\text{wo} \\text{kinds}$ of\n", " goods, $A$ and $B$, with relative demand functions, $p_1$ and $p_2$. Suppose\n", " the functions satisfy:\n", "\n", "\\begin{eqnarray*}\n", " A & = & 100 - 2 p_1 + p_2\\\\\n", " B & = & 120 + 3 p_1 - 5 p_2\n", " \\end{eqnarray*}\n", " Assume the cost function for producing $A$ and $B$ has been estimated as\n", " $$ C = 50 + 10 A + 20 B $$\n", " Then the profit function is\n", "\n", "\\begin{eqnarray*}\n", " P (p_1, p_2) & = & p_1 A + p_2 B - C\\\\\n", " & = & p_1 (100 - 2 p_1 + p_2) + p_2 (120 + 3 p_1 - 5 p_2) - (50 + 10 (100 - 2 p_1 + p_2) + 20 (120 + 3 p_1 - 5 p_2) \\\\\n", " & = & 1350 + 60 p_1 + 210 p_2 - 2 p_{1^{}}^2 - 5 p_2^2 + 4 p_1 p_2\n", "\\end{eqnarray*}\n", "\n", "The critical point of profit function can be obtained by\n", "\n", "\\begin{eqnarray*}\n", " & P_{p_1} = P_{p_2} = 0 & \\\\\n", " \\Longrightarrow & p_1 = 60 & p_2 = 45\n", " \\end{eqnarray*}\n", "\n", "And these imply $A = 25$ and $B = 75$. Here, it is difficulty to check\n", " whether profit is an extremum at this critical point. Later, we will give\n", " some conditions to confirm whether the function value is an extremum at\n", " critical points.\n", " " ] }, { "cell_type": "code", "execution_count": 81, "metadata": {}, "outputs": [ { "data": { "text/plain": [ "{q: 45, p: 60}" ] }, "execution_count": 81, "metadata": {}, "output_type": "execute_result" } ], "source": [ "A,B,C,p,q=symbols(\"A B C p q\")\n", "P=A*p+B*q-C\n", "PP=P.subs(C,50+10*A+20*B).subs([(A,100-2*p+q),(B,120+3*p-5*q)])\n", "solve(grad(PP,[p,q]),[p,q])" ] }, { "cell_type": "markdown", "metadata": {}, "source": [ "Example\n", "---\n", " Certain company sells its goods in two different brands, $A$ and $B$.\n", " Suppose that the relations between price function, $p_i$, and demand\n", " function, $x_{i,}$ $i = 1, 2$ in different sub-markets are:\n", "\n", "\\begin{eqnarray*}\n", " p_1 = 100 - x_1 & & \\\\\n", " p_2 = 120 - 2 x_2 & & \n", " \\end{eqnarray*}\n", " with the total cost function:\n", " $$ C = 20 (x_1 + x_2) $$\n", " Then profit function can be obtained by:\n", "\n", "\\begin{eqnarray*}\n", " P (x_1, x_2) & = & p_1 x_1 + p_2 x_2 - C\\\\\n", " & = & 80 x_1 - x_1^2 + 100 x_2 - 2 x_2^2\n", " \\end{eqnarray*}\n", " It is trivial that the maximum exists. From the first partial derivatives\n", " properties, the relative extrema must occurs at\n", "\n", "\\begin{eqnarray*}\n", " P_{x_1} & = & 80 - 2 x_1 = 0\\\\\n", " P_{x_2} & = & 100 - 4 x_2 = 0\n", " \\end{eqnarray*}\n", " And this implies that there exists only one stationary point, $(x_1, x_2) =\n", " (40, 25)$, and at which the maximum attains." ] }, { "cell_type": "markdown", "metadata": {}, "source": [ "Example\n", "---\n", " Two firms produce identical products with same price:\n", "\n", "$$ p = 150 - x_1 - x_2 $$\n", " and with zero cost. For each firms, their profits are in the following\n", " manners respectively:\n", "\n", "\\begin{eqnarray*}\n", " P_1 & = & p x_1 = (150 - x_1 - x_2) x_1\\\\\n", " P_2 & = & p x_2 = (150 - x_1 - x_2) x_2\n", " \\end{eqnarray*}\n", " Then if they want to get maximum profits, the first partial derivatives have\n", " to be held:\n", "\n", "\\begin{eqnarray*}\n", " \\frac{\\partial P_1}{\\partial x_1} & = & 0\\\\\n", " \\frac{\\partial P_2}{\\partial x_2} & = & 0\n", " \\end{eqnarray*}\n", " And these imply\n", "\n", "\\begin{eqnarray*}\n", " \\widehat{x_1} = \\frac{150 - x_2}{2} & & \\\\\n", " \\widehat{x_2} = \\frac{150 - x_1}{2} & & \n", " \\end{eqnarray*}\n", " From above results, the maximum occurs dependent on other stationary point.\n", " Therefore, two results have to be held simultaneously. In other words, we\n", " have\n", "\n", "$$ x_1 = x_2 = \\frac{150}{3} = 50 $$\n", " From the graphs, the demand functions are intersected at this point,\n", " $\\left(\\frac{100}{3}, \\frac{100}{3}\\right)$:\n", " \n", " \n", "This point is called **Cournot equilibrium**.\n", "\n" ] }, { "cell_type": "markdown", "metadata": {}, "source": [ "Example\n", "---\n", " Suppose that there are $n$ little firms produce identical products now. The\n", " price function is\n", " $$ p = 150 - \\sum_{i = 1}^n x_i $$\n", " with zero cost for simplicity. Then at stationary points, the following\n", " relation holds for any $k = 1, \\cdots, n$:\n", "\n", "\\begin{eqnarray*}\n", " & P_k = p x_k & = x_k \\left(150 - \\sum_{i = 1}^n x_i\\right)\\\\\n", " \\Longrightarrow & 0 = \\frac{\\partial P_k}{\\partial x_k} & = \\left(150 - \\sum_{i= 1}^n x_i\\right) - x_k\n", " \\end{eqnarray*}\n", "Since the equation still remains unchanged even by interchanging indexes\n", " with any two different variables, $x_i$ and $x_j$, it is obvious that\n", " all the $x_i$'s are equal. Thus replace all $x_k$'s with $\\widehat{x}$,\n", " then:\n", "\n", "\\begin{eqnarray*}\n", " & 0 =\\left(150 - \\sum_{i = 1}^n \\widehat{x_{}}\\right) - \\widehat{x_{}} & \\\\\n", " \\Longrightarrow & \\widehat{x_{}} = \\frac{150}{n + 1} & \\text{ for } k = 1,\n", " \\cdots, n\n", " \\end{eqnarray*}\n", " The last example is just in the case, $n = 2$." ] }, { "cell_type": "markdown", "metadata": {}, "source": [ "In above examples, the first derivative properties do not assure at which\n", "critical points are extrema. Some strongly conditions are needed to confirm\n", "whether there are extrema at critical point.\n", "\n", "Definition\n", "---\n", "Suppose that $M = (m_{i j})_{n \\times n}$ is a $n \\times n$ square matrix\n", " and $\\vec{h} = \\left(\n", " h_1,h_2,\\cdots,h_n\\right)_{1 \\times n}$ is any $1 \\times n$ vector. Then $M$ is\n", " called **positive definite** \n", "if $\\vec{h}^t M \\vec{h} = \\sum\\limits^n_{i,\n", " j = 1} h_i m_{i j} h_j > 0$ and called **negative definite** if\n", " $\\vec{h}^t M \\vec{h} < 0$.\n", "\n", "\n", "Define the Hessian matrix of function $f (\\vec{x})$ with its minor matrices\n", "as:\n", "\n", "\\begin{eqnarray*}\n", " H & = & (f_{i j})_{n \\times n}\\\\\n", " H_1 = (f_{11}) & \\cdots & H_k = (f_{i j})_{k \\times k}, k = 1, \\cdots, n - 1\n", "\\end{eqnarray*}\n", "and let $|H_i |$ be the determinant of $H_i$. For multi-variable functions, we\n", "have the following theorem:\n", "\n", "\n", "\n", "The definite properties of Hessian matrices can be determined by calculating\n", "determinants of Hessian matrices and their minor matrices.\n", "\n" ] }, { "cell_type": "markdown", "metadata": {}, "source": [ "Theorem\n", "---\n", "Let $$A = \\frac{\\partial^2 f}{\\partial x^2} (x_0, y_0), B =\n", " \\frac{\\partial^2 f}{\\partial y \\partial x} (x_0, y_0) = \\frac{\\partial^2\n", " f}{\\partial x \\partial y} (x_0, y_0),C = \\frac{\\partial^2 f}{\\partial\n", " y^2} (x_0, y_0) \\text{ and }\\frac{\\partial f}{\\partial x} (x_0, y_0) =\n", " \\frac{\\partial f}{\\partial y} (x_0, y_0) = 0$$ where $(x_0, y_0)$ is the\n", " critical point of $f (x, y)$, $D = A C - B^2$, then\n", "1. if $D > 0$ and $A < 0$ , $f (x_0, y_0)$ is a relative maximum,\n", "- if $D > 0$ and $A > 0$ , $f (x_0, y_0)$ is a relative minimum,\n", "- if $D < 0$ , $(x_0, y_0, f (x_0, y_0))$ is a saddle point,\n", "- if $D = 0$, no conclusion.\n", "\n", "Here the **Hessian matrix** for function $f (x, y)$ is defined as\n", " follows:\n", "\n", "$$ H = \\left(\\begin{array}{cc}\n", " \\frac{\\partial^2 f}{\\partial x^2} & \\frac{\\partial^2 f}{\\partial y\n", " \\partial x}\\\\\n", " \\frac{\\partial^2 f}{\\partial x \\partial y} & \\frac{\\partial^2\n", " f}{\\partial y^2}\n", " \\end{array}\\right) = \\left(\\begin{array}{cc}\n", " f_{11} & f_{12}\\\\\n", " f_{21} & f_{22}\n", " \\end{array}\\right) $$\n", " and with determinant $D = |H|$. For higher dimension case, $n \\ge 2$,\n", " then:\n", "1. if $|H_k | > 0$, for $k = 1, \\cdots, n$, $f (\\vec{x}_0)$ is a relative minimum,\n", "- if $|H_1 | < 0, |H_2 | > 0, \\cdots$, $f (\\vec{x}_0)$ is a relative maximum,\n", "\n", " where $H_k$ is the submatrix of $H$ and defineds as follows:\n", "\n", " " ] }, { "cell_type": "code", "execution_count": 5, "metadata": {}, "outputs": [ { "data": { "text/plain": [ "" ] }, "execution_count": 5, "metadata": {}, "output_type": "execute_result" }, { "data": { "image/png": "iVBORw0KGgoAAAANSUhEUgAAArMAAAFbCAYAAADGGAZpAAAABHNCSVQICAgIfAhkiAAAAAlwSFlz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\nYII=\n", "text/plain": [ "" ] }, "metadata": {}, "output_type": "display_data" } ], "source": [ "fig = plt.figure(figsize=(12,6))\n", "ax1 = fig.add_subplot(1, 1, 1, projection='3d')\n", "#ax1 = fig.gca(projection='3d')\n", "X = np.arange(-2,2.5, 0.04)\n", "Y = np.arange(2, 7)\n", "X, Y = np.meshgrid(X, Y)\n", "g= X**3+Y**2-2*X*Y+7*X-8*Y+2\n", "ax1.plot_surface(X, Y, g)\n", "ax1.set_title(\"$f(x,y)=x^3+y^2-2xy+7x-8y+2$\")\n" ] }, { "cell_type": "markdown", "metadata": {}, "source": [ "Example\n", "---\n", " Suppose that $f (x, y) =x^3+y^2-2xy+7x-8y+2.$ Then the\n", " critical value comes from the following relations:\n", "\n", "\\begin{eqnarray*}\n", " & f_x = 3x^2-2y+7 = 0 \\text{ and } f_y = 2y-2x-8 = 0 & \\\\\n", " \\Longrightarrow & (x, y) = (3, 1) & \n", " \\end{eqnarray*}\n", " i.e. there are two critical values, $(x, y) = P_1=(1,5), (x, y) = P_2=\\left(-1/3,11/3\\right)$. By the way, we also have\n", "\n", "\\begin{eqnarray*}\n", " H_2 &=& \\left(\\begin{array}{cc}\n", " f_{11} & f_{12}\\\\\n", " f_{21} & f_{22}\n", " \\end{array}\\right) & \\\\\n", " & = &12x-4\n", " \\end{eqnarray*}\n", "1. $P_1$, $f_{xx}(P_1)=6>0,H(P_1)=8>0$ imply $f(P_1)$ is local minimum;\n", "- $P_2$, $H(P_2)=-8<0$ imply $f(P_2)$ is saddle point;\n" ] }, { "cell_type": "markdown", "metadata": {}, "source": [ "Definition\n", "---\n", "$f(\\mathbf{x_0})$ is called absolute maximum of $f(\\mathbf{x})$ if $f(\\mathbf{x_0})\\ge f(\\mathbf{x})$ for any $\\mathbf{x}$ in its domain; and is called absolute minimum if $f(\\mathbf{x_0})\\le f(\\mathbf{x})$.\n", "\n", "Theorem\n", "---\n", "Continuous function $f(\\mathbf{x})$ should attain its both absolute maximum and minimum in closed domain. This means that the absolute extrema of $f(\\mathbf{x})$ could be found from the following steps:\n", "1. find out all critical values.\n", "- evaluate the function values at the points of (1) and the boundary of $D$,\n", "- absolute maximum is the largest of (2) and the absolute minimum is smallest of (2).\n", "\n", "Note\n", "---\n", "Extrema on the boundary could be evaluated by the **Lagrange's method** or called **mountain-pass** theorem, introduced later. " ] }, { "cell_type": "code", "execution_count": 6, "metadata": {}, "outputs": [ { "data": { "application/vnd.plotly.v1+json": { "data": [ { "opacity": 0.8, "type": "surface", "x": [ [ -1, -0.955, -0.91, -0.865, -0.8200000000000001, -0.775, -0.73, -0.685, -0.64, -0.595, -0.55, -0.505, -0.45999999999999996, -0.41500000000000004, -0.37, -0.32500000000000007, -0.28, -0.235, -0.19000000000000006, -0.14500000000000002, -0.10000000000000009, -0.05500000000000005, -0.010000000000000009, 0.03499999999999992, 0.08000000000000007, 0.125, 0.16999999999999993, 0.21499999999999986, 0.26, 0.30499999999999994, 0.34999999999999987, 0.395, 0.43999999999999995, 0.4849999999999999, 0.53, 0.575, 0.6199999999999999, 0.665, 0.71, 0.7549999999999999, 0.7999999999999998, 0.845, 0.8899999999999999, 0.9349999999999998, 0.98, 1.025, 1.0699999999999998, 1.1149999999999998, 1.1600000000000001, 1.205, 1.25, 1.295, 1.3399999999999999, 1.3849999999999998, 1.4299999999999997, 1.475, 1.52, 1.565, 1.6099999999999999, 1.6549999999999998, 1.6999999999999997, 1.745, 1.79, 1.835, 1.88, 1.9249999999999998, 1.9699999999999998, 2.0149999999999997, 2.06, 2.105, 2.15, 2.195, 2.2399999999999998, 2.2849999999999997, 2.33, 2.375, 2.42, 2.465, 2.51, 2.5549999999999997, 2.5999999999999996, 2.645, 2.69, 2.735, 2.78, 2.8249999999999997, 2.8699999999999997, 2.915, 2.96, 3.005, 3.05, 3.0949999999999998, 3.1399999999999997, 3.1849999999999996, 3.2299999999999995, 3.2749999999999995, 3.3200000000000003, 3.365, 3.41, 3.455, 3.5 ], [ -1, -0.955, -0.91, -0.865, -0.8200000000000001, -0.775, -0.73, -0.685, -0.64, -0.595, -0.55, -0.505, -0.45999999999999996, -0.41500000000000004, -0.37, -0.32500000000000007, -0.28, -0.235, -0.19000000000000006, -0.14500000000000002, -0.10000000000000009, -0.05500000000000005, -0.010000000000000009, 0.03499999999999992, 0.08000000000000007, 0.125, 0.16999999999999993, 0.21499999999999986, 0.26, 0.30499999999999994, 0.34999999999999987, 0.395, 0.43999999999999995, 0.4849999999999999, 0.53, 0.575, 0.6199999999999999, 0.665, 0.71, 0.7549999999999999, 0.7999999999999998, 0.845, 0.8899999999999999, 0.9349999999999998, 0.98, 1.025, 1.0699999999999998, 1.1149999999999998, 1.1600000000000001, 1.205, 1.25, 1.295, 1.3399999999999999, 1.3849999999999998, 1.4299999999999997, 1.475, 1.52, 1.565, 1.6099999999999999, 1.6549999999999998, 1.6999999999999997, 1.745, 1.79, 1.835, 1.88, 1.9249999999999998, 1.9699999999999998, 2.0149999999999997, 2.06, 2.105, 2.15, 2.195, 2.2399999999999998, 2.2849999999999997, 2.33, 2.375, 2.42, 2.465, 2.51, 2.5549999999999997, 2.5999999999999996, 2.645, 2.69, 2.735, 2.78, 2.8249999999999997, 2.8699999999999997, 2.915, 2.96, 3.005, 3.05, 3.0949999999999998, 3.1399999999999997, 3.1849999999999996, 3.2299999999999995, 3.2749999999999995, 3.3200000000000003, 3.365, 3.41, 3.455, 3.5 ], [ -1, -0.955, -0.91, -0.865, -0.8200000000000001, -0.775, -0.73, -0.685, -0.64, -0.595, -0.55, -0.505, -0.45999999999999996, -0.41500000000000004, -0.37, -0.32500000000000007, -0.28, -0.235, -0.19000000000000006, -0.14500000000000002, -0.10000000000000009, -0.05500000000000005, -0.010000000000000009, 0.03499999999999992, 0.08000000000000007, 0.125, 0.16999999999999993, 0.21499999999999986, 0.26, 0.30499999999999994, 0.34999999999999987, 0.395, 0.43999999999999995, 0.4849999999999999, 0.53, 0.575, 0.6199999999999999, 0.665, 0.71, 0.7549999999999999, 0.7999999999999998, 0.845, 0.8899999999999999, 0.9349999999999998, 0.98, 1.025, 1.0699999999999998, 1.1149999999999998, 1.1600000000000001, 1.205, 1.25, 1.295, 1.3399999999999999, 1.3849999999999998, 1.4299999999999997, 1.475, 1.52, 1.565, 1.6099999999999999, 1.6549999999999998, 1.6999999999999997, 1.745, 1.79, 1.835, 1.88, 1.9249999999999998, 1.9699999999999998, 2.0149999999999997, 2.06, 2.105, 2.15, 2.195, 2.2399999999999998, 2.2849999999999997, 2.33, 2.375, 2.42, 2.465, 2.51, 2.5549999999999997, 2.5999999999999996, 2.645, 2.69, 2.735, 2.78, 2.8249999999999997, 2.8699999999999997, 2.915, 2.96, 3.005, 3.05, 3.0949999999999998, 3.1399999999999997, 3.1849999999999996, 3.2299999999999995, 3.2749999999999995, 3.3200000000000003, 3.365, 3.41, 3.455, 3.5 ], [ -1, -0.955, -0.91, -0.865, -0.8200000000000001, -0.775, -0.73, -0.685, -0.64, -0.595, -0.55, -0.505, -0.45999999999999996, -0.41500000000000004, -0.37, -0.32500000000000007, -0.28, -0.235, -0.19000000000000006, -0.14500000000000002, -0.10000000000000009, -0.05500000000000005, -0.010000000000000009, 0.03499999999999992, 0.08000000000000007, 0.125, 0.16999999999999993, 0.21499999999999986, 0.26, 0.30499999999999994, 0.34999999999999987, 0.395, 0.43999999999999995, 0.4849999999999999, 0.53, 0.575, 0.6199999999999999, 0.665, 0.71, 0.7549999999999999, 0.7999999999999998, 0.845, 0.8899999999999999, 0.9349999999999998, 0.98, 1.025, 1.0699999999999998, 1.1149999999999998, 1.1600000000000001, 1.205, 1.25, 1.295, 1.3399999999999999, 1.3849999999999998, 1.4299999999999997, 1.475, 1.52, 1.565, 1.6099999999999999, 1.6549999999999998, 1.6999999999999997, 1.745, 1.79, 1.835, 1.88, 1.9249999999999998, 1.9699999999999998, 2.0149999999999997, 2.06, 2.105, 2.15, 2.195, 2.2399999999999998, 2.2849999999999997, 2.33, 2.375, 2.42, 2.465, 2.51, 2.5549999999999997, 2.5999999999999996, 2.645, 2.69, 2.735, 2.78, 2.8249999999999997, 2.8699999999999997, 2.915, 2.96, 3.005, 3.05, 3.0949999999999998, 3.1399999999999997, 3.1849999999999996, 3.2299999999999995, 3.2749999999999995, 3.3200000000000003, 3.365, 3.41, 3.455, 3.5 ], [ -1, -0.955, -0.91, -0.865, -0.8200000000000001, -0.775, -0.73, -0.685, -0.64, -0.595, -0.55, -0.505, -0.45999999999999996, -0.41500000000000004, -0.37, -0.32500000000000007, -0.28, -0.235, -0.19000000000000006, -0.14500000000000002, -0.10000000000000009, -0.05500000000000005, -0.010000000000000009, 0.03499999999999992, 0.08000000000000007, 0.125, 0.16999999999999993, 0.21499999999999986, 0.26, 0.30499999999999994, 0.34999999999999987, 0.395, 0.43999999999999995, 0.4849999999999999, 0.53, 0.575, 0.6199999999999999, 0.665, 0.71, 0.7549999999999999, 0.7999999999999998, 0.845, 0.8899999999999999, 0.9349999999999998, 0.98, 1.025, 1.0699999999999998, 1.1149999999999998, 1.1600000000000001, 1.205, 1.25, 1.295, 1.3399999999999999, 1.3849999999999998, 1.4299999999999997, 1.475, 1.52, 1.565, 1.6099999999999999, 1.6549999999999998, 1.6999999999999997, 1.745, 1.79, 1.835, 1.88, 1.9249999999999998, 1.9699999999999998, 2.0149999999999997, 2.06, 2.105, 2.15, 2.195, 2.2399999999999998, 2.2849999999999997, 2.33, 2.375, 2.42, 2.465, 2.51, 2.5549999999999997, 2.5999999999999996, 2.645, 2.69, 2.735, 2.78, 2.8249999999999997, 2.8699999999999997, 2.915, 2.96, 3.005, 3.05, 3.0949999999999998, 3.1399999999999997, 3.1849999999999996, 3.2299999999999995, 3.2749999999999995, 3.3200000000000003, 3.365, 3.41, 3.455, 3.5 ], [ -1, -0.955, -0.91, -0.865, -0.8200000000000001, -0.775, -0.73, -0.685, -0.64, -0.595, -0.55, -0.505, -0.45999999999999996, -0.41500000000000004, -0.37, -0.32500000000000007, -0.28, -0.235, -0.19000000000000006, -0.14500000000000002, -0.10000000000000009, -0.05500000000000005, -0.010000000000000009, 0.03499999999999992, 0.08000000000000007, 0.125, 0.16999999999999993, 0.21499999999999986, 0.26, 0.30499999999999994, 0.34999999999999987, 0.395, 0.43999999999999995, 0.4849999999999999, 0.53, 0.575, 0.6199999999999999, 0.665, 0.71, 0.7549999999999999, 0.7999999999999998, 0.845, 0.8899999999999999, 0.9349999999999998, 0.98, 1.025, 1.0699999999999998, 1.1149999999999998, 1.1600000000000001, 1.205, 1.25, 1.295, 1.3399999999999999, 1.3849999999999998, 1.4299999999999997, 1.475, 1.52, 1.565, 1.6099999999999999, 1.6549999999999998, 1.6999999999999997, 1.745, 1.79, 1.835, 1.88, 1.9249999999999998, 1.9699999999999998, 2.0149999999999997, 2.06, 2.105, 2.15, 2.195, 2.2399999999999998, 2.2849999999999997, 2.33, 2.375, 2.42, 2.465, 2.51, 2.5549999999999997, 2.5999999999999996, 2.645, 2.69, 2.735, 2.78, 2.8249999999999997, 2.8699999999999997, 2.915, 2.96, 3.005, 3.05, 3.0949999999999998, 3.1399999999999997, 3.1849999999999996, 3.2299999999999995, 3.2749999999999995, 3.3200000000000003, 3.365, 3.41, 3.455, 3.5 ], [ -1, -0.955, -0.91, -0.865, -0.8200000000000001, -0.775, -0.73, -0.685, -0.64, -0.595, -0.55, -0.505, -0.45999999999999996, -0.41500000000000004, -0.37, -0.32500000000000007, -0.28, -0.235, -0.19000000000000006, -0.14500000000000002, -0.10000000000000009, -0.05500000000000005, -0.010000000000000009, 0.03499999999999992, 0.08000000000000007, 0.125, 0.16999999999999993, 0.21499999999999986, 0.26, 0.30499999999999994, 0.34999999999999987, 0.395, 0.43999999999999995, 0.4849999999999999, 0.53, 0.575, 0.6199999999999999, 0.665, 0.71, 0.7549999999999999, 0.7999999999999998, 0.845, 0.8899999999999999, 0.9349999999999998, 0.98, 1.025, 1.0699999999999998, 1.1149999999999998, 1.1600000000000001, 1.205, 1.25, 1.295, 1.3399999999999999, 1.3849999999999998, 1.4299999999999997, 1.475, 1.52, 1.565, 1.6099999999999999, 1.6549999999999998, 1.6999999999999997, 1.745, 1.79, 1.835, 1.88, 1.9249999999999998, 1.9699999999999998, 2.0149999999999997, 2.06, 2.105, 2.15, 2.195, 2.2399999999999998, 2.2849999999999997, 2.33, 2.375, 2.42, 2.465, 2.51, 2.5549999999999997, 2.5999999999999996, 2.645, 2.69, 2.735, 2.78, 2.8249999999999997, 2.8699999999999997, 2.915, 2.96, 3.005, 3.05, 3.0949999999999998, 3.1399999999999997, 3.1849999999999996, 3.2299999999999995, 3.2749999999999995, 3.3200000000000003, 3.365, 3.41, 3.455, 3.5 ], [ -1, -0.955, -0.91, -0.865, -0.8200000000000001, -0.775, -0.73, -0.685, -0.64, -0.595, -0.55, -0.505, -0.45999999999999996, -0.41500000000000004, -0.37, -0.32500000000000007, -0.28, -0.235, -0.19000000000000006, -0.14500000000000002, -0.10000000000000009, -0.05500000000000005, -0.010000000000000009, 0.03499999999999992, 0.08000000000000007, 0.125, 0.16999999999999993, 0.21499999999999986, 0.26, 0.30499999999999994, 0.34999999999999987, 0.395, 0.43999999999999995, 0.4849999999999999, 0.53, 0.575, 0.6199999999999999, 0.665, 0.71, 0.7549999999999999, 0.7999999999999998, 0.845, 0.8899999999999999, 0.9349999999999998, 0.98, 1.025, 1.0699999999999998, 1.1149999999999998, 1.1600000000000001, 1.205, 1.25, 1.295, 1.3399999999999999, 1.3849999999999998, 1.4299999999999997, 1.475, 1.52, 1.565, 1.6099999999999999, 1.6549999999999998, 1.6999999999999997, 1.745, 1.79, 1.835, 1.88, 1.9249999999999998, 1.9699999999999998, 2.0149999999999997, 2.06, 2.105, 2.15, 2.195, 2.2399999999999998, 2.2849999999999997, 2.33, 2.375, 2.42, 2.465, 2.51, 2.5549999999999997, 2.5999999999999996, 2.645, 2.69, 2.735, 2.78, 2.8249999999999997, 2.8699999999999997, 2.915, 2.96, 3.005, 3.05, 3.0949999999999998, 3.1399999999999997, 3.1849999999999996, 3.2299999999999995, 3.2749999999999995, 3.3200000000000003, 3.365, 3.41, 3.455, 3.5 ], [ -1, -0.955, -0.91, -0.865, -0.8200000000000001, -0.775, -0.73, -0.685, -0.64, -0.595, -0.55, -0.505, -0.45999999999999996, -0.41500000000000004, -0.37, -0.32500000000000007, -0.28, -0.235, -0.19000000000000006, -0.14500000000000002, -0.10000000000000009, -0.05500000000000005, -0.010000000000000009, 0.03499999999999992, 0.08000000000000007, 0.125, 0.16999999999999993, 0.21499999999999986, 0.26, 0.30499999999999994, 0.34999999999999987, 0.395, 0.43999999999999995, 0.4849999999999999, 0.53, 0.575, 0.6199999999999999, 0.665, 0.71, 0.7549999999999999, 0.7999999999999998, 0.845, 0.8899999999999999, 0.9349999999999998, 0.98, 1.025, 1.0699999999999998, 1.1149999999999998, 1.1600000000000001, 1.205, 1.25, 1.295, 1.3399999999999999, 1.3849999999999998, 1.4299999999999997, 1.475, 1.52, 1.565, 1.6099999999999999, 1.6549999999999998, 1.6999999999999997, 1.745, 1.79, 1.835, 1.88, 1.9249999999999998, 1.9699999999999998, 2.0149999999999997, 2.06, 2.105, 2.15, 2.195, 2.2399999999999998, 2.2849999999999997, 2.33, 2.375, 2.42, 2.465, 2.51, 2.5549999999999997, 2.5999999999999996, 2.645, 2.69, 2.735, 2.78, 2.8249999999999997, 2.8699999999999997, 2.915, 2.96, 3.005, 3.05, 3.0949999999999998, 3.1399999999999997, 3.1849999999999996, 3.2299999999999995, 3.2749999999999995, 3.3200000000000003, 3.365, 3.41, 3.455, 3.5 ], [ -1, -0.955, -0.91, -0.865, -0.8200000000000001, -0.775, -0.73, -0.685, -0.64, -0.595, -0.55, -0.505, -0.45999999999999996, -0.41500000000000004, -0.37, -0.32500000000000007, -0.28, -0.235, -0.19000000000000006, -0.14500000000000002, -0.10000000000000009, -0.05500000000000005, -0.010000000000000009, 0.03499999999999992, 0.08000000000000007, 0.125, 0.16999999999999993, 0.21499999999999986, 0.26, 0.30499999999999994, 0.34999999999999987, 0.395, 0.43999999999999995, 0.4849999999999999, 0.53, 0.575, 0.6199999999999999, 0.665, 0.71, 0.7549999999999999, 0.7999999999999998, 0.845, 0.8899999999999999, 0.9349999999999998, 0.98, 1.025, 1.0699999999999998, 1.1149999999999998, 1.1600000000000001, 1.205, 1.25, 1.295, 1.3399999999999999, 1.3849999999999998, 1.4299999999999997, 1.475, 1.52, 1.565, 1.6099999999999999, 1.6549999999999998, 1.6999999999999997, 1.745, 1.79, 1.835, 1.88, 1.9249999999999998, 1.9699999999999998, 2.0149999999999997, 2.06, 2.105, 2.15, 2.195, 2.2399999999999998, 2.2849999999999997, 2.33, 2.375, 2.42, 2.465, 2.51, 2.5549999999999997, 2.5999999999999996, 2.645, 2.69, 2.735, 2.78, 2.8249999999999997, 2.8699999999999997, 2.915, 2.96, 3.005, 3.05, 3.0949999999999998, 3.1399999999999997, 3.1849999999999996, 3.2299999999999995, 3.2749999999999995, 3.3200000000000003, 3.365, 3.41, 3.455, 3.5 ], [ -1, -0.955, -0.91, -0.865, -0.8200000000000001, -0.775, -0.73, -0.685, -0.64, -0.595, -0.55, -0.505, -0.45999999999999996, -0.41500000000000004, -0.37, -0.32500000000000007, -0.28, -0.235, -0.19000000000000006, -0.14500000000000002, -0.10000000000000009, -0.05500000000000005, -0.010000000000000009, 0.03499999999999992, 0.08000000000000007, 0.125, 0.16999999999999993, 0.21499999999999986, 0.26, 0.30499999999999994, 0.34999999999999987, 0.395, 0.43999999999999995, 0.4849999999999999, 0.53, 0.575, 0.6199999999999999, 0.665, 0.71, 0.7549999999999999, 0.7999999999999998, 0.845, 0.8899999999999999, 0.9349999999999998, 0.98, 1.025, 1.0699999999999998, 1.1149999999999998, 1.1600000000000001, 1.205, 1.25, 1.295, 1.3399999999999999, 1.3849999999999998, 1.4299999999999997, 1.475, 1.52, 1.565, 1.6099999999999999, 1.6549999999999998, 1.6999999999999997, 1.745, 1.79, 1.835, 1.88, 1.9249999999999998, 1.9699999999999998, 2.0149999999999997, 2.06, 2.105, 2.15, 2.195, 2.2399999999999998, 2.2849999999999997, 2.33, 2.375, 2.42, 2.465, 2.51, 2.5549999999999997, 2.5999999999999996, 2.645, 2.69, 2.735, 2.78, 2.8249999999999997, 2.8699999999999997, 2.915, 2.96, 3.005, 3.05, 3.0949999999999998, 3.1399999999999997, 3.1849999999999996, 3.2299999999999995, 3.2749999999999995, 3.3200000000000003, 3.365, 3.41, 3.455, 3.5 ], [ -1, -0.955, -0.91, -0.865, -0.8200000000000001, -0.775, -0.73, -0.685, -0.64, -0.595, -0.55, -0.505, -0.45999999999999996, -0.41500000000000004, -0.37, -0.32500000000000007, -0.28, -0.235, -0.19000000000000006, -0.14500000000000002, -0.10000000000000009, -0.05500000000000005, -0.010000000000000009, 0.03499999999999992, 0.08000000000000007, 0.125, 0.16999999999999993, 0.21499999999999986, 0.26, 0.30499999999999994, 0.34999999999999987, 0.395, 0.43999999999999995, 0.4849999999999999, 0.53, 0.575, 0.6199999999999999, 0.665, 0.71, 0.7549999999999999, 0.7999999999999998, 0.845, 0.8899999999999999, 0.9349999999999998, 0.98, 1.025, 1.0699999999999998, 1.1149999999999998, 1.1600000000000001, 1.205, 1.25, 1.295, 1.3399999999999999, 1.3849999999999998, 1.4299999999999997, 1.475, 1.52, 1.565, 1.6099999999999999, 1.6549999999999998, 1.6999999999999997, 1.745, 1.79, 1.835, 1.88, 1.9249999999999998, 1.9699999999999998, 2.0149999999999997, 2.06, 2.105, 2.15, 2.195, 2.2399999999999998, 2.2849999999999997, 2.33, 2.375, 2.42, 2.465, 2.51, 2.5549999999999997, 2.5999999999999996, 2.645, 2.69, 2.735, 2.78, 2.8249999999999997, 2.8699999999999997, 2.915, 2.96, 3.005, 3.05, 3.0949999999999998, 3.1399999999999997, 3.1849999999999996, 3.2299999999999995, 3.2749999999999995, 3.3200000000000003, 3.365, 3.41, 3.455, 3.5 ], [ -1, -0.955, -0.91, -0.865, -0.8200000000000001, -0.775, -0.73, -0.685, -0.64, -0.595, -0.55, -0.505, -0.45999999999999996, -0.41500000000000004, -0.37, -0.32500000000000007, -0.28, -0.235, -0.19000000000000006, -0.14500000000000002, -0.10000000000000009, -0.05500000000000005, -0.010000000000000009, 0.03499999999999992, 0.08000000000000007, 0.125, 0.16999999999999993, 0.21499999999999986, 0.26, 0.30499999999999994, 0.34999999999999987, 0.395, 0.43999999999999995, 0.4849999999999999, 0.53, 0.575, 0.6199999999999999, 0.665, 0.71, 0.7549999999999999, 0.7999999999999998, 0.845, 0.8899999999999999, 0.9349999999999998, 0.98, 1.025, 1.0699999999999998, 1.1149999999999998, 1.1600000000000001, 1.205, 1.25, 1.295, 1.3399999999999999, 1.3849999999999998, 1.4299999999999997, 1.475, 1.52, 1.565, 1.6099999999999999, 1.6549999999999998, 1.6999999999999997, 1.745, 1.79, 1.835, 1.88, 1.9249999999999998, 1.9699999999999998, 2.0149999999999997, 2.06, 2.105, 2.15, 2.195, 2.2399999999999998, 2.2849999999999997, 2.33, 2.375, 2.42, 2.465, 2.51, 2.5549999999999997, 2.5999999999999996, 2.645, 2.69, 2.735, 2.78, 2.8249999999999997, 2.8699999999999997, 2.915, 2.96, 3.005, 3.05, 3.0949999999999998, 3.1399999999999997, 3.1849999999999996, 3.2299999999999995, 3.2749999999999995, 3.3200000000000003, 3.365, 3.41, 3.455, 3.5 ], [ -1, -0.955, -0.91, -0.865, -0.8200000000000001, -0.775, -0.73, -0.685, -0.64, -0.595, -0.55, -0.505, -0.45999999999999996, -0.41500000000000004, -0.37, -0.32500000000000007, -0.28, -0.235, -0.19000000000000006, -0.14500000000000002, -0.10000000000000009, -0.05500000000000005, -0.010000000000000009, 0.03499999999999992, 0.08000000000000007, 0.125, 0.16999999999999993, 0.21499999999999986, 0.26, 0.30499999999999994, 0.34999999999999987, 0.395, 0.43999999999999995, 0.4849999999999999, 0.53, 0.575, 0.6199999999999999, 0.665, 0.71, 0.7549999999999999, 0.7999999999999998, 0.845, 0.8899999999999999, 0.9349999999999998, 0.98, 1.025, 1.0699999999999998, 1.1149999999999998, 1.1600000000000001, 1.205, 1.25, 1.295, 1.3399999999999999, 1.3849999999999998, 1.4299999999999997, 1.475, 1.52, 1.565, 1.6099999999999999, 1.6549999999999998, 1.6999999999999997, 1.745, 1.79, 1.835, 1.88, 1.9249999999999998, 1.9699999999999998, 2.0149999999999997, 2.06, 2.105, 2.15, 2.195, 2.2399999999999998, 2.2849999999999997, 2.33, 2.375, 2.42, 2.465, 2.51, 2.5549999999999997, 2.5999999999999996, 2.645, 2.69, 2.735, 2.78, 2.8249999999999997, 2.8699999999999997, 2.915, 2.96, 3.005, 3.05, 3.0949999999999998, 3.1399999999999997, 3.1849999999999996, 3.2299999999999995, 3.2749999999999995, 3.3200000000000003, 3.365, 3.41, 3.455, 3.5 ], [ -1, -0.955, -0.91, -0.865, -0.8200000000000001, -0.775, -0.73, -0.685, -0.64, -0.595, -0.55, -0.505, -0.45999999999999996, -0.41500000000000004, -0.37, -0.32500000000000007, -0.28, -0.235, -0.19000000000000006, -0.14500000000000002, -0.10000000000000009, -0.05500000000000005, -0.010000000000000009, 0.03499999999999992, 0.08000000000000007, 0.125, 0.16999999999999993, 0.21499999999999986, 0.26, 0.30499999999999994, 0.34999999999999987, 0.395, 0.43999999999999995, 0.4849999999999999, 0.53, 0.575, 0.6199999999999999, 0.665, 0.71, 0.7549999999999999, 0.7999999999999998, 0.845, 0.8899999999999999, 0.9349999999999998, 0.98, 1.025, 1.0699999999999998, 1.1149999999999998, 1.1600000000000001, 1.205, 1.25, 1.295, 1.3399999999999999, 1.3849999999999998, 1.4299999999999997, 1.475, 1.52, 1.565, 1.6099999999999999, 1.6549999999999998, 1.6999999999999997, 1.745, 1.79, 1.835, 1.88, 1.9249999999999998, 1.9699999999999998, 2.0149999999999997, 2.06, 2.105, 2.15, 2.195, 2.2399999999999998, 2.2849999999999997, 2.33, 2.375, 2.42, 2.465, 2.51, 2.5549999999999997, 2.5999999999999996, 2.645, 2.69, 2.735, 2.78, 2.8249999999999997, 2.8699999999999997, 2.915, 2.96, 3.005, 3.05, 3.0949999999999998, 3.1399999999999997, 3.1849999999999996, 3.2299999999999995, 3.2749999999999995, 3.3200000000000003, 3.365, 3.41, 3.455, 3.5 ], [ -1, -0.955, -0.91, -0.865, -0.8200000000000001, -0.775, -0.73, -0.685, -0.64, -0.595, -0.55, -0.505, -0.45999999999999996, -0.41500000000000004, -0.37, -0.32500000000000007, -0.28, -0.235, -0.19000000000000006, -0.14500000000000002, -0.10000000000000009, -0.05500000000000005, -0.010000000000000009, 0.03499999999999992, 0.08000000000000007, 0.125, 0.16999999999999993, 0.21499999999999986, 0.26, 0.30499999999999994, 0.34999999999999987, 0.395, 0.43999999999999995, 0.4849999999999999, 0.53, 0.575, 0.6199999999999999, 0.665, 0.71, 0.7549999999999999, 0.7999999999999998, 0.845, 0.8899999999999999, 0.9349999999999998, 0.98, 1.025, 1.0699999999999998, 1.1149999999999998, 1.1600000000000001, 1.205, 1.25, 1.295, 1.3399999999999999, 1.3849999999999998, 1.4299999999999997, 1.475, 1.52, 1.565, 1.6099999999999999, 1.6549999999999998, 1.6999999999999997, 1.745, 1.79, 1.835, 1.88, 1.9249999999999998, 1.9699999999999998, 2.0149999999999997, 2.06, 2.105, 2.15, 2.195, 2.2399999999999998, 2.2849999999999997, 2.33, 2.375, 2.42, 2.465, 2.51, 2.5549999999999997, 2.5999999999999996, 2.645, 2.69, 2.735, 2.78, 2.8249999999999997, 2.8699999999999997, 2.915, 2.96, 3.005, 3.05, 3.0949999999999998, 3.1399999999999997, 3.1849999999999996, 3.2299999999999995, 3.2749999999999995, 3.3200000000000003, 3.365, 3.41, 3.455, 3.5 ], [ -1, -0.955, -0.91, -0.865, -0.8200000000000001, -0.775, -0.73, -0.685, -0.64, -0.595, -0.55, -0.505, -0.45999999999999996, -0.41500000000000004, -0.37, -0.32500000000000007, -0.28, -0.235, -0.19000000000000006, -0.14500000000000002, -0.10000000000000009, -0.05500000000000005, -0.010000000000000009, 0.03499999999999992, 0.08000000000000007, 0.125, 0.16999999999999993, 0.21499999999999986, 0.26, 0.30499999999999994, 0.34999999999999987, 0.395, 0.43999999999999995, 0.4849999999999999, 0.53, 0.575, 0.6199999999999999, 0.665, 0.71, 0.7549999999999999, 0.7999999999999998, 0.845, 0.8899999999999999, 0.9349999999999998, 0.98, 1.025, 1.0699999999999998, 1.1149999999999998, 1.1600000000000001, 1.205, 1.25, 1.295, 1.3399999999999999, 1.3849999999999998, 1.4299999999999997, 1.475, 1.52, 1.565, 1.6099999999999999, 1.6549999999999998, 1.6999999999999997, 1.745, 1.79, 1.835, 1.88, 1.9249999999999998, 1.9699999999999998, 2.0149999999999997, 2.06, 2.105, 2.15, 2.195, 2.2399999999999998, 2.2849999999999997, 2.33, 2.375, 2.42, 2.465, 2.51, 2.5549999999999997, 2.5999999999999996, 2.645, 2.69, 2.735, 2.78, 2.8249999999999997, 2.8699999999999997, 2.915, 2.96, 3.005, 3.05, 3.0949999999999998, 3.1399999999999997, 3.1849999999999996, 3.2299999999999995, 3.2749999999999995, 3.3200000000000003, 3.365, 3.41, 3.455, 3.5 ], [ -1, -0.955, -0.91, -0.865, -0.8200000000000001, -0.775, -0.73, -0.685, -0.64, -0.595, -0.55, -0.505, -0.45999999999999996, -0.41500000000000004, -0.37, -0.32500000000000007, -0.28, -0.235, -0.19000000000000006, -0.14500000000000002, -0.10000000000000009, -0.05500000000000005, -0.010000000000000009, 0.03499999999999992, 0.08000000000000007, 0.125, 0.16999999999999993, 0.21499999999999986, 0.26, 0.30499999999999994, 0.34999999999999987, 0.395, 0.43999999999999995, 0.4849999999999999, 0.53, 0.575, 0.6199999999999999, 0.665, 0.71, 0.7549999999999999, 0.7999999999999998, 0.845, 0.8899999999999999, 0.9349999999999998, 0.98, 1.025, 1.0699999999999998, 1.1149999999999998, 1.1600000000000001, 1.205, 1.25, 1.295, 1.3399999999999999, 1.3849999999999998, 1.4299999999999997, 1.475, 1.52, 1.565, 1.6099999999999999, 1.6549999999999998, 1.6999999999999997, 1.745, 1.79, 1.835, 1.88, 1.9249999999999998, 1.9699999999999998, 2.0149999999999997, 2.06, 2.105, 2.15, 2.195, 2.2399999999999998, 2.2849999999999997, 2.33, 2.375, 2.42, 2.465, 2.51, 2.5549999999999997, 2.5999999999999996, 2.645, 2.69, 2.735, 2.78, 2.8249999999999997, 2.8699999999999997, 2.915, 2.96, 3.005, 3.05, 3.0949999999999998, 3.1399999999999997, 3.1849999999999996, 3.2299999999999995, 3.2749999999999995, 3.3200000000000003, 3.365, 3.41, 3.455, 3.5 ], [ -1, -0.955, -0.91, -0.865, -0.8200000000000001, -0.775, -0.73, -0.685, -0.64, -0.595, -0.55, -0.505, -0.45999999999999996, -0.41500000000000004, -0.37, -0.32500000000000007, -0.28, -0.235, -0.19000000000000006, -0.14500000000000002, -0.10000000000000009, -0.05500000000000005, -0.010000000000000009, 0.03499999999999992, 0.08000000000000007, 0.125, 0.16999999999999993, 0.21499999999999986, 0.26, 0.30499999999999994, 0.34999999999999987, 0.395, 0.43999999999999995, 0.4849999999999999, 0.53, 0.575, 0.6199999999999999, 0.665, 0.71, 0.7549999999999999, 0.7999999999999998, 0.845, 0.8899999999999999, 0.9349999999999998, 0.98, 1.025, 1.0699999999999998, 1.1149999999999998, 1.1600000000000001, 1.205, 1.25, 1.295, 1.3399999999999999, 1.3849999999999998, 1.4299999999999997, 1.475, 1.52, 1.565, 1.6099999999999999, 1.6549999999999998, 1.6999999999999997, 1.745, 1.79, 1.835, 1.88, 1.9249999999999998, 1.9699999999999998, 2.0149999999999997, 2.06, 2.105, 2.15, 2.195, 2.2399999999999998, 2.2849999999999997, 2.33, 2.375, 2.42, 2.465, 2.51, 2.5549999999999997, 2.5999999999999996, 2.645, 2.69, 2.735, 2.78, 2.8249999999999997, 2.8699999999999997, 2.915, 2.96, 3.005, 3.05, 3.0949999999999998, 3.1399999999999997, 3.1849999999999996, 3.2299999999999995, 3.2749999999999995, 3.3200000000000003, 3.365, 3.41, 3.455, 3.5 ], [ -1, -0.955, -0.91, -0.865, -0.8200000000000001, -0.775, -0.73, -0.685, -0.64, -0.595, -0.55, -0.505, -0.45999999999999996, -0.41500000000000004, -0.37, -0.32500000000000007, -0.28, -0.235, -0.19000000000000006, -0.14500000000000002, -0.10000000000000009, -0.05500000000000005, -0.010000000000000009, 0.03499999999999992, 0.08000000000000007, 0.125, 0.16999999999999993, 0.21499999999999986, 0.26, 0.30499999999999994, 0.34999999999999987, 0.395, 0.43999999999999995, 0.4849999999999999, 0.53, 0.575, 0.6199999999999999, 0.665, 0.71, 0.7549999999999999, 0.7999999999999998, 0.845, 0.8899999999999999, 0.9349999999999998, 0.98, 1.025, 1.0699999999999998, 1.1149999999999998, 1.1600000000000001, 1.205, 1.25, 1.295, 1.3399999999999999, 1.3849999999999998, 1.4299999999999997, 1.475, 1.52, 1.565, 1.6099999999999999, 1.6549999999999998, 1.6999999999999997, 1.745, 1.79, 1.835, 1.88, 1.9249999999999998, 1.9699999999999998, 2.0149999999999997, 2.06, 2.105, 2.15, 2.195, 2.2399999999999998, 2.2849999999999997, 2.33, 2.375, 2.42, 2.465, 2.51, 2.5549999999999997, 2.5999999999999996, 2.645, 2.69, 2.735, 2.78, 2.8249999999999997, 2.8699999999999997, 2.915, 2.96, 3.005, 3.05, 3.0949999999999998, 3.1399999999999997, 3.1849999999999996, 3.2299999999999995, 3.2749999999999995, 3.3200000000000003, 3.365, 3.41, 3.455, 3.5 ], [ -1, -0.955, -0.91, -0.865, -0.8200000000000001, -0.775, -0.73, -0.685, -0.64, -0.595, -0.55, -0.505, -0.45999999999999996, -0.41500000000000004, -0.37, -0.32500000000000007, -0.28, -0.235, -0.19000000000000006, -0.14500000000000002, -0.10000000000000009, -0.05500000000000005, -0.010000000000000009, 0.03499999999999992, 0.08000000000000007, 0.125, 0.16999999999999993, 0.21499999999999986, 0.26, 0.30499999999999994, 0.34999999999999987, 0.395, 0.43999999999999995, 0.4849999999999999, 0.53, 0.575, 0.6199999999999999, 0.665, 0.71, 0.7549999999999999, 0.7999999999999998, 0.845, 0.8899999999999999, 0.9349999999999998, 0.98, 1.025, 1.0699999999999998, 1.1149999999999998, 1.1600000000000001, 1.205, 1.25, 1.295, 1.3399999999999999, 1.3849999999999998, 1.4299999999999997, 1.475, 1.52, 1.565, 1.6099999999999999, 1.6549999999999998, 1.6999999999999997, 1.745, 1.79, 1.835, 1.88, 1.9249999999999998, 1.9699999999999998, 2.0149999999999997, 2.06, 2.105, 2.15, 2.195, 2.2399999999999998, 2.2849999999999997, 2.33, 2.375, 2.42, 2.465, 2.51, 2.5549999999999997, 2.5999999999999996, 2.645, 2.69, 2.735, 2.78, 2.8249999999999997, 2.8699999999999997, 2.915, 2.96, 3.005, 3.05, 3.0949999999999998, 3.1399999999999997, 3.1849999999999996, 3.2299999999999995, 3.2749999999999995, 3.3200000000000003, 3.365, 3.41, 3.455, 3.5 ], [ -1, -0.955, -0.91, -0.865, -0.8200000000000001, -0.775, -0.73, -0.685, -0.64, -0.595, -0.55, -0.505, -0.45999999999999996, -0.41500000000000004, -0.37, -0.32500000000000007, -0.28, -0.235, -0.19000000000000006, -0.14500000000000002, -0.10000000000000009, -0.05500000000000005, -0.010000000000000009, 0.03499999999999992, 0.08000000000000007, 0.125, 0.16999999999999993, 0.21499999999999986, 0.26, 0.30499999999999994, 0.34999999999999987, 0.395, 0.43999999999999995, 0.4849999999999999, 0.53, 0.575, 0.6199999999999999, 0.665, 0.71, 0.7549999999999999, 0.7999999999999998, 0.845, 0.8899999999999999, 0.9349999999999998, 0.98, 1.025, 1.0699999999999998, 1.1149999999999998, 1.1600000000000001, 1.205, 1.25, 1.295, 1.3399999999999999, 1.3849999999999998, 1.4299999999999997, 1.475, 1.52, 1.565, 1.6099999999999999, 1.6549999999999998, 1.6999999999999997, 1.745, 1.79, 1.835, 1.88, 1.9249999999999998, 1.9699999999999998, 2.0149999999999997, 2.06, 2.105, 2.15, 2.195, 2.2399999999999998, 2.2849999999999997, 2.33, 2.375, 2.42, 2.465, 2.51, 2.5549999999999997, 2.5999999999999996, 2.645, 2.69, 2.735, 2.78, 2.8249999999999997, 2.8699999999999997, 2.915, 2.96, 3.005, 3.05, 3.0949999999999998, 3.1399999999999997, 3.1849999999999996, 3.2299999999999995, 3.2749999999999995, 3.3200000000000003, 3.365, 3.41, 3.455, 3.5 ], [ -1, -0.955, -0.91, -0.865, -0.8200000000000001, -0.775, -0.73, -0.685, -0.64, -0.595, -0.55, -0.505, -0.45999999999999996, -0.41500000000000004, -0.37, -0.32500000000000007, -0.28, -0.235, -0.19000000000000006, -0.14500000000000002, -0.10000000000000009, -0.05500000000000005, -0.010000000000000009, 0.03499999999999992, 0.08000000000000007, 0.125, 0.16999999999999993, 0.21499999999999986, 0.26, 0.30499999999999994, 0.34999999999999987, 0.395, 0.43999999999999995, 0.4849999999999999, 0.53, 0.575, 0.6199999999999999, 0.665, 0.71, 0.7549999999999999, 0.7999999999999998, 0.845, 0.8899999999999999, 0.9349999999999998, 0.98, 1.025, 1.0699999999999998, 1.1149999999999998, 1.1600000000000001, 1.205, 1.25, 1.295, 1.3399999999999999, 1.3849999999999998, 1.4299999999999997, 1.475, 1.52, 1.565, 1.6099999999999999, 1.6549999999999998, 1.6999999999999997, 1.745, 1.79, 1.835, 1.88, 1.9249999999999998, 1.9699999999999998, 2.0149999999999997, 2.06, 2.105, 2.15, 2.195, 2.2399999999999998, 2.2849999999999997, 2.33, 2.375, 2.42, 2.465, 2.51, 2.5549999999999997, 2.5999999999999996, 2.645, 2.69, 2.735, 2.78, 2.8249999999999997, 2.8699999999999997, 2.915, 2.96, 3.005, 3.05, 3.0949999999999998, 3.1399999999999997, 3.1849999999999996, 3.2299999999999995, 3.2749999999999995, 3.3200000000000003, 3.365, 3.41, 3.455, 3.5 ], [ -1, -0.955, -0.91, -0.865, -0.8200000000000001, -0.775, -0.73, -0.685, -0.64, -0.595, -0.55, -0.505, -0.45999999999999996, -0.41500000000000004, -0.37, -0.32500000000000007, -0.28, -0.235, -0.19000000000000006, -0.14500000000000002, -0.10000000000000009, -0.05500000000000005, -0.010000000000000009, 0.03499999999999992, 0.08000000000000007, 0.125, 0.16999999999999993, 0.21499999999999986, 0.26, 0.30499999999999994, 0.34999999999999987, 0.395, 0.43999999999999995, 0.4849999999999999, 0.53, 0.575, 0.6199999999999999, 0.665, 0.71, 0.7549999999999999, 0.7999999999999998, 0.845, 0.8899999999999999, 0.9349999999999998, 0.98, 1.025, 1.0699999999999998, 1.1149999999999998, 1.1600000000000001, 1.205, 1.25, 1.295, 1.3399999999999999, 1.3849999999999998, 1.4299999999999997, 1.475, 1.52, 1.565, 1.6099999999999999, 1.6549999999999998, 1.6999999999999997, 1.745, 1.79, 1.835, 1.88, 1.9249999999999998, 1.9699999999999998, 2.0149999999999997, 2.06, 2.105, 2.15, 2.195, 2.2399999999999998, 2.2849999999999997, 2.33, 2.375, 2.42, 2.465, 2.51, 2.5549999999999997, 2.5999999999999996, 2.645, 2.69, 2.735, 2.78, 2.8249999999999997, 2.8699999999999997, 2.915, 2.96, 3.005, 3.05, 3.0949999999999998, 3.1399999999999997, 3.1849999999999996, 3.2299999999999995, 3.2749999999999995, 3.3200000000000003, 3.365, 3.41, 3.455, 3.5 ], [ -1, -0.955, -0.91, -0.865, -0.8200000000000001, -0.775, -0.73, -0.685, -0.64, -0.595, -0.55, -0.505, -0.45999999999999996, -0.41500000000000004, -0.37, -0.32500000000000007, -0.28, -0.235, -0.19000000000000006, -0.14500000000000002, -0.10000000000000009, -0.05500000000000005, -0.010000000000000009, 0.03499999999999992, 0.08000000000000007, 0.125, 0.16999999999999993, 0.21499999999999986, 0.26, 0.30499999999999994, 0.34999999999999987, 0.395, 0.43999999999999995, 0.4849999999999999, 0.53, 0.575, 0.6199999999999999, 0.665, 0.71, 0.7549999999999999, 0.7999999999999998, 0.845, 0.8899999999999999, 0.9349999999999998, 0.98, 1.025, 1.0699999999999998, 1.1149999999999998, 1.1600000000000001, 1.205, 1.25, 1.295, 1.3399999999999999, 1.3849999999999998, 1.4299999999999997, 1.475, 1.52, 1.565, 1.6099999999999999, 1.6549999999999998, 1.6999999999999997, 1.745, 1.79, 1.835, 1.88, 1.9249999999999998, 1.9699999999999998, 2.0149999999999997, 2.06, 2.105, 2.15, 2.195, 2.2399999999999998, 2.2849999999999997, 2.33, 2.375, 2.42, 2.465, 2.51, 2.5549999999999997, 2.5999999999999996, 2.645, 2.69, 2.735, 2.78, 2.8249999999999997, 2.8699999999999997, 2.915, 2.96, 3.005, 3.05, 3.0949999999999998, 3.1399999999999997, 3.1849999999999996, 3.2299999999999995, 3.2749999999999995, 3.3200000000000003, 3.365, 3.41, 3.455, 3.5 ], [ -1, -0.955, -0.91, -0.865, -0.8200000000000001, -0.775, -0.73, -0.685, -0.64, -0.595, -0.55, -0.505, -0.45999999999999996, -0.41500000000000004, -0.37, -0.32500000000000007, -0.28, -0.235, -0.19000000000000006, -0.14500000000000002, -0.10000000000000009, -0.05500000000000005, -0.010000000000000009, 0.03499999999999992, 0.08000000000000007, 0.125, 0.16999999999999993, 0.21499999999999986, 0.26, 0.30499999999999994, 0.34999999999999987, 0.395, 0.43999999999999995, 0.4849999999999999, 0.53, 0.575, 0.6199999999999999, 0.665, 0.71, 0.7549999999999999, 0.7999999999999998, 0.845, 0.8899999999999999, 0.9349999999999998, 0.98, 1.025, 1.0699999999999998, 1.1149999999999998, 1.1600000000000001, 1.205, 1.25, 1.295, 1.3399999999999999, 1.3849999999999998, 1.4299999999999997, 1.475, 1.52, 1.565, 1.6099999999999999, 1.6549999999999998, 1.6999999999999997, 1.745, 1.79, 1.835, 1.88, 1.9249999999999998, 1.9699999999999998, 2.0149999999999997, 2.06, 2.105, 2.15, 2.195, 2.2399999999999998, 2.2849999999999997, 2.33, 2.375, 2.42, 2.465, 2.51, 2.5549999999999997, 2.5999999999999996, 2.645, 2.69, 2.735, 2.78, 2.8249999999999997, 2.8699999999999997, 2.915, 2.96, 3.005, 3.05, 3.0949999999999998, 3.1399999999999997, 3.1849999999999996, 3.2299999999999995, 3.2749999999999995, 3.3200000000000003, 3.365, 3.41, 3.455, 3.5 ], [ -1, -0.955, -0.91, -0.865, -0.8200000000000001, -0.775, -0.73, -0.685, -0.64, -0.595, -0.55, -0.505, -0.45999999999999996, -0.41500000000000004, -0.37, -0.32500000000000007, -0.28, -0.235, -0.19000000000000006, -0.14500000000000002, -0.10000000000000009, -0.05500000000000005, -0.010000000000000009, 0.03499999999999992, 0.08000000000000007, 0.125, 0.16999999999999993, 0.21499999999999986, 0.26, 0.30499999999999994, 0.34999999999999987, 0.395, 0.43999999999999995, 0.4849999999999999, 0.53, 0.575, 0.6199999999999999, 0.665, 0.71, 0.7549999999999999, 0.7999999999999998, 0.845, 0.8899999999999999, 0.9349999999999998, 0.98, 1.025, 1.0699999999999998, 1.1149999999999998, 1.1600000000000001, 1.205, 1.25, 1.295, 1.3399999999999999, 1.3849999999999998, 1.4299999999999997, 1.475, 1.52, 1.565, 1.6099999999999999, 1.6549999999999998, 1.6999999999999997, 1.745, 1.79, 1.835, 1.88, 1.9249999999999998, 1.9699999999999998, 2.0149999999999997, 2.06, 2.105, 2.15, 2.195, 2.2399999999999998, 2.2849999999999997, 2.33, 2.375, 2.42, 2.465, 2.51, 2.5549999999999997, 2.5999999999999996, 2.645, 2.69, 2.735, 2.78, 2.8249999999999997, 2.8699999999999997, 2.915, 2.96, 3.005, 3.05, 3.0949999999999998, 3.1399999999999997, 3.1849999999999996, 3.2299999999999995, 3.2749999999999995, 3.3200000000000003, 3.365, 3.41, 3.455, 3.5 ], [ -1, -0.955, -0.91, -0.865, -0.8200000000000001, -0.775, -0.73, -0.685, -0.64, -0.595, -0.55, -0.505, -0.45999999999999996, -0.41500000000000004, -0.37, -0.32500000000000007, -0.28, -0.235, -0.19000000000000006, -0.14500000000000002, -0.10000000000000009, -0.05500000000000005, -0.010000000000000009, 0.03499999999999992, 0.08000000000000007, 0.125, 0.16999999999999993, 0.21499999999999986, 0.26, 0.30499999999999994, 0.34999999999999987, 0.395, 0.43999999999999995, 0.4849999999999999, 0.53, 0.575, 0.6199999999999999, 0.665, 0.71, 0.7549999999999999, 0.7999999999999998, 0.845, 0.8899999999999999, 0.9349999999999998, 0.98, 1.025, 1.0699999999999998, 1.1149999999999998, 1.1600000000000001, 1.205, 1.25, 1.295, 1.3399999999999999, 1.3849999999999998, 1.4299999999999997, 1.475, 1.52, 1.565, 1.6099999999999999, 1.6549999999999998, 1.6999999999999997, 1.745, 1.79, 1.835, 1.88, 1.9249999999999998, 1.9699999999999998, 2.0149999999999997, 2.06, 2.105, 2.15, 2.195, 2.2399999999999998, 2.2849999999999997, 2.33, 2.375, 2.42, 2.465, 2.51, 2.5549999999999997, 2.5999999999999996, 2.645, 2.69, 2.735, 2.78, 2.8249999999999997, 2.8699999999999997, 2.915, 2.96, 3.005, 3.05, 3.0949999999999998, 3.1399999999999997, 3.1849999999999996, 3.2299999999999995, 3.2749999999999995, 3.3200000000000003, 3.365, 3.41, 3.455, 3.5 ], [ -1, -0.955, -0.91, -0.865, -0.8200000000000001, -0.775, -0.73, -0.685, -0.64, -0.595, -0.55, -0.505, -0.45999999999999996, -0.41500000000000004, -0.37, -0.32500000000000007, -0.28, -0.235, -0.19000000000000006, -0.14500000000000002, -0.10000000000000009, -0.05500000000000005, -0.010000000000000009, 0.03499999999999992, 0.08000000000000007, 0.125, 0.16999999999999993, 0.21499999999999986, 0.26, 0.30499999999999994, 0.34999999999999987, 0.395, 0.43999999999999995, 0.4849999999999999, 0.53, 0.575, 0.6199999999999999, 0.665, 0.71, 0.7549999999999999, 0.7999999999999998, 0.845, 0.8899999999999999, 0.9349999999999998, 0.98, 1.025, 1.0699999999999998, 1.1149999999999998, 1.1600000000000001, 1.205, 1.25, 1.295, 1.3399999999999999, 1.3849999999999998, 1.4299999999999997, 1.475, 1.52, 1.565, 1.6099999999999999, 1.6549999999999998, 1.6999999999999997, 1.745, 1.79, 1.835, 1.88, 1.9249999999999998, 1.9699999999999998, 2.0149999999999997, 2.06, 2.105, 2.15, 2.195, 2.2399999999999998, 2.2849999999999997, 2.33, 2.375, 2.42, 2.465, 2.51, 2.5549999999999997, 2.5999999999999996, 2.645, 2.69, 2.735, 2.78, 2.8249999999999997, 2.8699999999999997, 2.915, 2.96, 3.005, 3.05, 3.0949999999999998, 3.1399999999999997, 3.1849999999999996, 3.2299999999999995, 3.2749999999999995, 3.3200000000000003, 3.365, 3.41, 3.455, 3.5 ], [ -1, -0.955, -0.91, -0.865, -0.8200000000000001, -0.775, -0.73, -0.685, -0.64, -0.595, -0.55, -0.505, -0.45999999999999996, -0.41500000000000004, -0.37, -0.32500000000000007, -0.28, -0.235, -0.19000000000000006, -0.14500000000000002, -0.10000000000000009, -0.05500000000000005, -0.010000000000000009, 0.03499999999999992, 0.08000000000000007, 0.125, 0.16999999999999993, 0.21499999999999986, 0.26, 0.30499999999999994, 0.34999999999999987, 0.395, 0.43999999999999995, 0.4849999999999999, 0.53, 0.575, 0.6199999999999999, 0.665, 0.71, 0.7549999999999999, 0.7999999999999998, 0.845, 0.8899999999999999, 0.9349999999999998, 0.98, 1.025, 1.0699999999999998, 1.1149999999999998, 1.1600000000000001, 1.205, 1.25, 1.295, 1.3399999999999999, 1.3849999999999998, 1.4299999999999997, 1.475, 1.52, 1.565, 1.6099999999999999, 1.6549999999999998, 1.6999999999999997, 1.745, 1.79, 1.835, 1.88, 1.9249999999999998, 1.9699999999999998, 2.0149999999999997, 2.06, 2.105, 2.15, 2.195, 2.2399999999999998, 2.2849999999999997, 2.33, 2.375, 2.42, 2.465, 2.51, 2.5549999999999997, 2.5999999999999996, 2.645, 2.69, 2.735, 2.78, 2.8249999999999997, 2.8699999999999997, 2.915, 2.96, 3.005, 3.05, 3.0949999999999998, 3.1399999999999997, 3.1849999999999996, 3.2299999999999995, 3.2749999999999995, 3.3200000000000003, 3.365, 3.41, 3.455, 3.5 ], [ -1, -0.955, -0.91, -0.865, -0.8200000000000001, -0.775, -0.73, -0.685, -0.64, -0.595, -0.55, -0.505, -0.45999999999999996, -0.41500000000000004, -0.37, -0.32500000000000007, -0.28, -0.235, -0.19000000000000006, -0.14500000000000002, -0.10000000000000009, -0.05500000000000005, -0.010000000000000009, 0.03499999999999992, 0.08000000000000007, 0.125, 0.16999999999999993, 0.21499999999999986, 0.26, 0.30499999999999994, 0.34999999999999987, 0.395, 0.43999999999999995, 0.4849999999999999, 0.53, 0.575, 0.6199999999999999, 0.665, 0.71, 0.7549999999999999, 0.7999999999999998, 0.845, 0.8899999999999999, 0.9349999999999998, 0.98, 1.025, 1.0699999999999998, 1.1149999999999998, 1.1600000000000001, 1.205, 1.25, 1.295, 1.3399999999999999, 1.3849999999999998, 1.4299999999999997, 1.475, 1.52, 1.565, 1.6099999999999999, 1.6549999999999998, 1.6999999999999997, 1.745, 1.79, 1.835, 1.88, 1.9249999999999998, 1.9699999999999998, 2.0149999999999997, 2.06, 2.105, 2.15, 2.195, 2.2399999999999998, 2.2849999999999997, 2.33, 2.375, 2.42, 2.465, 2.51, 2.5549999999999997, 2.5999999999999996, 2.645, 2.69, 2.735, 2.78, 2.8249999999999997, 2.8699999999999997, 2.915, 2.96, 3.005, 3.05, 3.0949999999999998, 3.1399999999999997, 3.1849999999999996, 3.2299999999999995, 3.2749999999999995, 3.3200000000000003, 3.365, 3.41, 3.455, 3.5 ], [ -1, -0.955, -0.91, -0.865, -0.8200000000000001, -0.775, -0.73, -0.685, -0.64, -0.595, -0.55, -0.505, -0.45999999999999996, -0.41500000000000004, -0.37, -0.32500000000000007, -0.28, -0.235, -0.19000000000000006, -0.14500000000000002, -0.10000000000000009, -0.05500000000000005, -0.010000000000000009, 0.03499999999999992, 0.08000000000000007, 0.125, 0.16999999999999993, 0.21499999999999986, 0.26, 0.30499999999999994, 0.34999999999999987, 0.395, 0.43999999999999995, 0.4849999999999999, 0.53, 0.575, 0.6199999999999999, 0.665, 0.71, 0.7549999999999999, 0.7999999999999998, 0.845, 0.8899999999999999, 0.9349999999999998, 0.98, 1.025, 1.0699999999999998, 1.1149999999999998, 1.1600000000000001, 1.205, 1.25, 1.295, 1.3399999999999999, 1.3849999999999998, 1.4299999999999997, 1.475, 1.52, 1.565, 1.6099999999999999, 1.6549999999999998, 1.6999999999999997, 1.745, 1.79, 1.835, 1.88, 1.9249999999999998, 1.9699999999999998, 2.0149999999999997, 2.06, 2.105, 2.15, 2.195, 2.2399999999999998, 2.2849999999999997, 2.33, 2.375, 2.42, 2.465, 2.51, 2.5549999999999997, 2.5999999999999996, 2.645, 2.69, 2.735, 2.78, 2.8249999999999997, 2.8699999999999997, 2.915, 2.96, 3.005, 3.05, 3.0949999999999998, 3.1399999999999997, 3.1849999999999996, 3.2299999999999995, 3.2749999999999995, 3.3200000000000003, 3.365, 3.41, 3.455, 3.5 ], [ -1, -0.955, -0.91, -0.865, -0.8200000000000001, -0.775, -0.73, -0.685, -0.64, -0.595, -0.55, -0.505, -0.45999999999999996, -0.41500000000000004, -0.37, -0.32500000000000007, -0.28, -0.235, -0.19000000000000006, -0.14500000000000002, -0.10000000000000009, -0.05500000000000005, -0.010000000000000009, 0.03499999999999992, 0.08000000000000007, 0.125, 0.16999999999999993, 0.21499999999999986, 0.26, 0.30499999999999994, 0.34999999999999987, 0.395, 0.43999999999999995, 0.4849999999999999, 0.53, 0.575, 0.6199999999999999, 0.665, 0.71, 0.7549999999999999, 0.7999999999999998, 0.845, 0.8899999999999999, 0.9349999999999998, 0.98, 1.025, 1.0699999999999998, 1.1149999999999998, 1.1600000000000001, 1.205, 1.25, 1.295, 1.3399999999999999, 1.3849999999999998, 1.4299999999999997, 1.475, 1.52, 1.565, 1.6099999999999999, 1.6549999999999998, 1.6999999999999997, 1.745, 1.79, 1.835, 1.88, 1.9249999999999998, 1.9699999999999998, 2.0149999999999997, 2.06, 2.105, 2.15, 2.195, 2.2399999999999998, 2.2849999999999997, 2.33, 2.375, 2.42, 2.465, 2.51, 2.5549999999999997, 2.5999999999999996, 2.645, 2.69, 2.735, 2.78, 2.8249999999999997, 2.8699999999999997, 2.915, 2.96, 3.005, 3.05, 3.0949999999999998, 3.1399999999999997, 3.1849999999999996, 3.2299999999999995, 3.2749999999999995, 3.3200000000000003, 3.365, 3.41, 3.455, 3.5 ], [ -1, -0.955, -0.91, -0.865, -0.8200000000000001, -0.775, -0.73, -0.685, -0.64, -0.595, -0.55, -0.505, -0.45999999999999996, -0.41500000000000004, -0.37, -0.32500000000000007, -0.28, -0.235, -0.19000000000000006, -0.14500000000000002, -0.10000000000000009, -0.05500000000000005, -0.010000000000000009, 0.03499999999999992, 0.08000000000000007, 0.125, 0.16999999999999993, 0.21499999999999986, 0.26, 0.30499999999999994, 0.34999999999999987, 0.395, 0.43999999999999995, 0.4849999999999999, 0.53, 0.575, 0.6199999999999999, 0.665, 0.71, 0.7549999999999999, 0.7999999999999998, 0.845, 0.8899999999999999, 0.9349999999999998, 0.98, 1.025, 1.0699999999999998, 1.1149999999999998, 1.1600000000000001, 1.205, 1.25, 1.295, 1.3399999999999999, 1.3849999999999998, 1.4299999999999997, 1.475, 1.52, 1.565, 1.6099999999999999, 1.6549999999999998, 1.6999999999999997, 1.745, 1.79, 1.835, 1.88, 1.9249999999999998, 1.9699999999999998, 2.0149999999999997, 2.06, 2.105, 2.15, 2.195, 2.2399999999999998, 2.2849999999999997, 2.33, 2.375, 2.42, 2.465, 2.51, 2.5549999999999997, 2.5999999999999996, 2.645, 2.69, 2.735, 2.78, 2.8249999999999997, 2.8699999999999997, 2.915, 2.96, 3.005, 3.05, 3.0949999999999998, 3.1399999999999997, 3.1849999999999996, 3.2299999999999995, 3.2749999999999995, 3.3200000000000003, 3.365, 3.41, 3.455, 3.5 ], [ -1, -0.955, -0.91, -0.865, -0.8200000000000001, -0.775, -0.73, -0.685, -0.64, -0.595, -0.55, -0.505, -0.45999999999999996, -0.41500000000000004, -0.37, -0.32500000000000007, -0.28, -0.235, -0.19000000000000006, -0.14500000000000002, -0.10000000000000009, -0.05500000000000005, -0.010000000000000009, 0.03499999999999992, 0.08000000000000007, 0.125, 0.16999999999999993, 0.21499999999999986, 0.26, 0.30499999999999994, 0.34999999999999987, 0.395, 0.43999999999999995, 0.4849999999999999, 0.53, 0.575, 0.6199999999999999, 0.665, 0.71, 0.7549999999999999, 0.7999999999999998, 0.845, 0.8899999999999999, 0.9349999999999998, 0.98, 1.025, 1.0699999999999998, 1.1149999999999998, 1.1600000000000001, 1.205, 1.25, 1.295, 1.3399999999999999, 1.3849999999999998, 1.4299999999999997, 1.475, 1.52, 1.565, 1.6099999999999999, 1.6549999999999998, 1.6999999999999997, 1.745, 1.79, 1.835, 1.88, 1.9249999999999998, 1.9699999999999998, 2.0149999999999997, 2.06, 2.105, 2.15, 2.195, 2.2399999999999998, 2.2849999999999997, 2.33, 2.375, 2.42, 2.465, 2.51, 2.5549999999999997, 2.5999999999999996, 2.645, 2.69, 2.735, 2.78, 2.8249999999999997, 2.8699999999999997, 2.915, 2.96, 3.005, 3.05, 3.0949999999999998, 3.1399999999999997, 3.1849999999999996, 3.2299999999999995, 3.2749999999999995, 3.3200000000000003, 3.365, 3.41, 3.455, 3.5 ], [ -1, -0.955, -0.91, -0.865, -0.8200000000000001, -0.775, -0.73, -0.685, -0.64, -0.595, -0.55, -0.505, -0.45999999999999996, -0.41500000000000004, -0.37, -0.32500000000000007, -0.28, -0.235, -0.19000000000000006, -0.14500000000000002, -0.10000000000000009, -0.05500000000000005, -0.010000000000000009, 0.03499999999999992, 0.08000000000000007, 0.125, 0.16999999999999993, 0.21499999999999986, 0.26, 0.30499999999999994, 0.34999999999999987, 0.395, 0.43999999999999995, 0.4849999999999999, 0.53, 0.575, 0.6199999999999999, 0.665, 0.71, 0.7549999999999999, 0.7999999999999998, 0.845, 0.8899999999999999, 0.9349999999999998, 0.98, 1.025, 1.0699999999999998, 1.1149999999999998, 1.1600000000000001, 1.205, 1.25, 1.295, 1.3399999999999999, 1.3849999999999998, 1.4299999999999997, 1.475, 1.52, 1.565, 1.6099999999999999, 1.6549999999999998, 1.6999999999999997, 1.745, 1.79, 1.835, 1.88, 1.9249999999999998, 1.9699999999999998, 2.0149999999999997, 2.06, 2.105, 2.15, 2.195, 2.2399999999999998, 2.2849999999999997, 2.33, 2.375, 2.42, 2.465, 2.51, 2.5549999999999997, 2.5999999999999996, 2.645, 2.69, 2.735, 2.78, 2.8249999999999997, 2.8699999999999997, 2.915, 2.96, 3.005, 3.05, 3.0949999999999998, 3.1399999999999997, 3.1849999999999996, 3.2299999999999995, 3.2749999999999995, 3.3200000000000003, 3.365, 3.41, 3.455, 3.5 ], [ -1, -0.955, -0.91, -0.865, -0.8200000000000001, -0.775, -0.73, -0.685, -0.64, -0.595, -0.55, -0.505, -0.45999999999999996, -0.41500000000000004, -0.37, -0.32500000000000007, -0.28, -0.235, -0.19000000000000006, -0.14500000000000002, -0.10000000000000009, -0.05500000000000005, -0.010000000000000009, 0.03499999999999992, 0.08000000000000007, 0.125, 0.16999999999999993, 0.21499999999999986, 0.26, 0.30499999999999994, 0.34999999999999987, 0.395, 0.43999999999999995, 0.4849999999999999, 0.53, 0.575, 0.6199999999999999, 0.665, 0.71, 0.7549999999999999, 0.7999999999999998, 0.845, 0.8899999999999999, 0.9349999999999998, 0.98, 1.025, 1.0699999999999998, 1.1149999999999998, 1.1600000000000001, 1.205, 1.25, 1.295, 1.3399999999999999, 1.3849999999999998, 1.4299999999999997, 1.475, 1.52, 1.565, 1.6099999999999999, 1.6549999999999998, 1.6999999999999997, 1.745, 1.79, 1.835, 1.88, 1.9249999999999998, 1.9699999999999998, 2.0149999999999997, 2.06, 2.105, 2.15, 2.195, 2.2399999999999998, 2.2849999999999997, 2.33, 2.375, 2.42, 2.465, 2.51, 2.5549999999999997, 2.5999999999999996, 2.645, 2.69, 2.735, 2.78, 2.8249999999999997, 2.8699999999999997, 2.915, 2.96, 3.005, 3.05, 3.0949999999999998, 3.1399999999999997, 3.1849999999999996, 3.2299999999999995, 3.2749999999999995, 3.3200000000000003, 3.365, 3.41, 3.455, 3.5 ], [ -1, -0.955, -0.91, -0.865, -0.8200000000000001, -0.775, -0.73, -0.685, -0.64, -0.595, -0.55, -0.505, -0.45999999999999996, -0.41500000000000004, -0.37, -0.32500000000000007, -0.28, -0.235, -0.19000000000000006, -0.14500000000000002, -0.10000000000000009, -0.05500000000000005, -0.010000000000000009, 0.03499999999999992, 0.08000000000000007, 0.125, 0.16999999999999993, 0.21499999999999986, 0.26, 0.30499999999999994, 0.34999999999999987, 0.395, 0.43999999999999995, 0.4849999999999999, 0.53, 0.575, 0.6199999999999999, 0.665, 0.71, 0.7549999999999999, 0.7999999999999998, 0.845, 0.8899999999999999, 0.9349999999999998, 0.98, 1.025, 1.0699999999999998, 1.1149999999999998, 1.1600000000000001, 1.205, 1.25, 1.295, 1.3399999999999999, 1.3849999999999998, 1.4299999999999997, 1.475, 1.52, 1.565, 1.6099999999999999, 1.6549999999999998, 1.6999999999999997, 1.745, 1.79, 1.835, 1.88, 1.9249999999999998, 1.9699999999999998, 2.0149999999999997, 2.06, 2.105, 2.15, 2.195, 2.2399999999999998, 2.2849999999999997, 2.33, 2.375, 2.42, 2.465, 2.51, 2.5549999999999997, 2.5999999999999996, 2.645, 2.69, 2.735, 2.78, 2.8249999999999997, 2.8699999999999997, 2.915, 2.96, 3.005, 3.05, 3.0949999999999998, 3.1399999999999997, 3.1849999999999996, 3.2299999999999995, 3.2749999999999995, 3.3200000000000003, 3.365, 3.41, 3.455, 3.5 ], [ -1, -0.955, -0.91, -0.865, -0.8200000000000001, -0.775, -0.73, -0.685, -0.64, -0.595, -0.55, -0.505, -0.45999999999999996, -0.41500000000000004, -0.37, -0.32500000000000007, -0.28, -0.235, -0.19000000000000006, -0.14500000000000002, -0.10000000000000009, -0.05500000000000005, -0.010000000000000009, 0.03499999999999992, 0.08000000000000007, 0.125, 0.16999999999999993, 0.21499999999999986, 0.26, 0.30499999999999994, 0.34999999999999987, 0.395, 0.43999999999999995, 0.4849999999999999, 0.53, 0.575, 0.6199999999999999, 0.665, 0.71, 0.7549999999999999, 0.7999999999999998, 0.845, 0.8899999999999999, 0.9349999999999998, 0.98, 1.025, 1.0699999999999998, 1.1149999999999998, 1.1600000000000001, 1.205, 1.25, 1.295, 1.3399999999999999, 1.3849999999999998, 1.4299999999999997, 1.475, 1.52, 1.565, 1.6099999999999999, 1.6549999999999998, 1.6999999999999997, 1.745, 1.79, 1.835, 1.88, 1.9249999999999998, 1.9699999999999998, 2.0149999999999997, 2.06, 2.105, 2.15, 2.195, 2.2399999999999998, 2.2849999999999997, 2.33, 2.375, 2.42, 2.465, 2.51, 2.5549999999999997, 2.5999999999999996, 2.645, 2.69, 2.735, 2.78, 2.8249999999999997, 2.8699999999999997, 2.915, 2.96, 3.005, 3.05, 3.0949999999999998, 3.1399999999999997, 3.1849999999999996, 3.2299999999999995, 3.2749999999999995, 3.3200000000000003, 3.365, 3.41, 3.455, 3.5 ], [ -1, -0.955, -0.91, -0.865, -0.8200000000000001, -0.775, -0.73, -0.685, -0.64, -0.595, -0.55, -0.505, -0.45999999999999996, -0.41500000000000004, -0.37, -0.32500000000000007, -0.28, -0.235, -0.19000000000000006, -0.14500000000000002, -0.10000000000000009, -0.05500000000000005, -0.010000000000000009, 0.03499999999999992, 0.08000000000000007, 0.125, 0.16999999999999993, 0.21499999999999986, 0.26, 0.30499999999999994, 0.34999999999999987, 0.395, 0.43999999999999995, 0.4849999999999999, 0.53, 0.575, 0.6199999999999999, 0.665, 0.71, 0.7549999999999999, 0.7999999999999998, 0.845, 0.8899999999999999, 0.9349999999999998, 0.98, 1.025, 1.0699999999999998, 1.1149999999999998, 1.1600000000000001, 1.205, 1.25, 1.295, 1.3399999999999999, 1.3849999999999998, 1.4299999999999997, 1.475, 1.52, 1.565, 1.6099999999999999, 1.6549999999999998, 1.6999999999999997, 1.745, 1.79, 1.835, 1.88, 1.9249999999999998, 1.9699999999999998, 2.0149999999999997, 2.06, 2.105, 2.15, 2.195, 2.2399999999999998, 2.2849999999999997, 2.33, 2.375, 2.42, 2.465, 2.51, 2.5549999999999997, 2.5999999999999996, 2.645, 2.69, 2.735, 2.78, 2.8249999999999997, 2.8699999999999997, 2.915, 2.96, 3.005, 3.05, 3.0949999999999998, 3.1399999999999997, 3.1849999999999996, 3.2299999999999995, 3.2749999999999995, 3.3200000000000003, 3.365, 3.41, 3.455, 3.5 ], [ -1, -0.955, -0.91, -0.865, -0.8200000000000001, -0.775, -0.73, -0.685, -0.64, -0.595, -0.55, -0.505, -0.45999999999999996, -0.41500000000000004, -0.37, -0.32500000000000007, -0.28, -0.235, -0.19000000000000006, -0.14500000000000002, -0.10000000000000009, -0.05500000000000005, -0.010000000000000009, 0.03499999999999992, 0.08000000000000007, 0.125, 0.16999999999999993, 0.21499999999999986, 0.26, 0.30499999999999994, 0.34999999999999987, 0.395, 0.43999999999999995, 0.4849999999999999, 0.53, 0.575, 0.6199999999999999, 0.665, 0.71, 0.7549999999999999, 0.7999999999999998, 0.845, 0.8899999999999999, 0.9349999999999998, 0.98, 1.025, 1.0699999999999998, 1.1149999999999998, 1.1600000000000001, 1.205, 1.25, 1.295, 1.3399999999999999, 1.3849999999999998, 1.4299999999999997, 1.475, 1.52, 1.565, 1.6099999999999999, 1.6549999999999998, 1.6999999999999997, 1.745, 1.79, 1.835, 1.88, 1.9249999999999998, 1.9699999999999998, 2.0149999999999997, 2.06, 2.105, 2.15, 2.195, 2.2399999999999998, 2.2849999999999997, 2.33, 2.375, 2.42, 2.465, 2.51, 2.5549999999999997, 2.5999999999999996, 2.645, 2.69, 2.735, 2.78, 2.8249999999999997, 2.8699999999999997, 2.915, 2.96, 3.005, 3.05, 3.0949999999999998, 3.1399999999999997, 3.1849999999999996, 3.2299999999999995, 3.2749999999999995, 3.3200000000000003, 3.365, 3.41, 3.455, 3.5 ], [ -1, -0.955, -0.91, -0.865, -0.8200000000000001, -0.775, -0.73, -0.685, -0.64, -0.595, -0.55, -0.505, -0.45999999999999996, -0.41500000000000004, -0.37, -0.32500000000000007, -0.28, -0.235, -0.19000000000000006, -0.14500000000000002, -0.10000000000000009, -0.05500000000000005, -0.010000000000000009, 0.03499999999999992, 0.08000000000000007, 0.125, 0.16999999999999993, 0.21499999999999986, 0.26, 0.30499999999999994, 0.34999999999999987, 0.395, 0.43999999999999995, 0.4849999999999999, 0.53, 0.575, 0.6199999999999999, 0.665, 0.71, 0.7549999999999999, 0.7999999999999998, 0.845, 0.8899999999999999, 0.9349999999999998, 0.98, 1.025, 1.0699999999999998, 1.1149999999999998, 1.1600000000000001, 1.205, 1.25, 1.295, 1.3399999999999999, 1.3849999999999998, 1.4299999999999997, 1.475, 1.52, 1.565, 1.6099999999999999, 1.6549999999999998, 1.6999999999999997, 1.745, 1.79, 1.835, 1.88, 1.9249999999999998, 1.9699999999999998, 2.0149999999999997, 2.06, 2.105, 2.15, 2.195, 2.2399999999999998, 2.2849999999999997, 2.33, 2.375, 2.42, 2.465, 2.51, 2.5549999999999997, 2.5999999999999996, 2.645, 2.69, 2.735, 2.78, 2.8249999999999997, 2.8699999999999997, 2.915, 2.96, 3.005, 3.05, 3.0949999999999998, 3.1399999999999997, 3.1849999999999996, 3.2299999999999995, 3.2749999999999995, 3.3200000000000003, 3.365, 3.41, 3.455, 3.5 ], [ -1, -0.955, -0.91, -0.865, -0.8200000000000001, -0.775, -0.73, -0.685, -0.64, -0.595, -0.55, -0.505, -0.45999999999999996, -0.41500000000000004, -0.37, -0.32500000000000007, -0.28, -0.235, -0.19000000000000006, -0.14500000000000002, -0.10000000000000009, -0.05500000000000005, -0.010000000000000009, 0.03499999999999992, 0.08000000000000007, 0.125, 0.16999999999999993, 0.21499999999999986, 0.26, 0.30499999999999994, 0.34999999999999987, 0.395, 0.43999999999999995, 0.4849999999999999, 0.53, 0.575, 0.6199999999999999, 0.665, 0.71, 0.7549999999999999, 0.7999999999999998, 0.845, 0.8899999999999999, 0.9349999999999998, 0.98, 1.025, 1.0699999999999998, 1.1149999999999998, 1.1600000000000001, 1.205, 1.25, 1.295, 1.3399999999999999, 1.3849999999999998, 1.4299999999999997, 1.475, 1.52, 1.565, 1.6099999999999999, 1.6549999999999998, 1.6999999999999997, 1.745, 1.79, 1.835, 1.88, 1.9249999999999998, 1.9699999999999998, 2.0149999999999997, 2.06, 2.105, 2.15, 2.195, 2.2399999999999998, 2.2849999999999997, 2.33, 2.375, 2.42, 2.465, 2.51, 2.5549999999999997, 2.5999999999999996, 2.645, 2.69, 2.735, 2.78, 2.8249999999999997, 2.8699999999999997, 2.915, 2.96, 3.005, 3.05, 3.0949999999999998, 3.1399999999999997, 3.1849999999999996, 3.2299999999999995, 3.2749999999999995, 3.3200000000000003, 3.365, 3.41, 3.455, 3.5 ], [ -1, -0.955, -0.91, -0.865, -0.8200000000000001, -0.775, -0.73, -0.685, -0.64, -0.595, -0.55, -0.505, -0.45999999999999996, -0.41500000000000004, -0.37, -0.32500000000000007, -0.28, -0.235, -0.19000000000000006, -0.14500000000000002, -0.10000000000000009, -0.05500000000000005, -0.010000000000000009, 0.03499999999999992, 0.08000000000000007, 0.125, 0.16999999999999993, 0.21499999999999986, 0.26, 0.30499999999999994, 0.34999999999999987, 0.395, 0.43999999999999995, 0.4849999999999999, 0.53, 0.575, 0.6199999999999999, 0.665, 0.71, 0.7549999999999999, 0.7999999999999998, 0.845, 0.8899999999999999, 0.9349999999999998, 0.98, 1.025, 1.0699999999999998, 1.1149999999999998, 1.1600000000000001, 1.205, 1.25, 1.295, 1.3399999999999999, 1.3849999999999998, 1.4299999999999997, 1.475, 1.52, 1.565, 1.6099999999999999, 1.6549999999999998, 1.6999999999999997, 1.745, 1.79, 1.835, 1.88, 1.9249999999999998, 1.9699999999999998, 2.0149999999999997, 2.06, 2.105, 2.15, 2.195, 2.2399999999999998, 2.2849999999999997, 2.33, 2.375, 2.42, 2.465, 2.51, 2.5549999999999997, 2.5999999999999996, 2.645, 2.69, 2.735, 2.78, 2.8249999999999997, 2.8699999999999997, 2.915, 2.96, 3.005, 3.05, 3.0949999999999998, 3.1399999999999997, 3.1849999999999996, 3.2299999999999995, 3.2749999999999995, 3.3200000000000003, 3.365, 3.41, 3.455, 3.5 ], [ -1, -0.955, -0.91, -0.865, -0.8200000000000001, -0.775, -0.73, -0.685, -0.64, -0.595, -0.55, -0.505, -0.45999999999999996, -0.41500000000000004, -0.37, -0.32500000000000007, -0.28, -0.235, -0.19000000000000006, -0.14500000000000002, -0.10000000000000009, -0.05500000000000005, -0.010000000000000009, 0.03499999999999992, 0.08000000000000007, 0.125, 0.16999999999999993, 0.21499999999999986, 0.26, 0.30499999999999994, 0.34999999999999987, 0.395, 0.43999999999999995, 0.4849999999999999, 0.53, 0.575, 0.6199999999999999, 0.665, 0.71, 0.7549999999999999, 0.7999999999999998, 0.845, 0.8899999999999999, 0.9349999999999998, 0.98, 1.025, 1.0699999999999998, 1.1149999999999998, 1.1600000000000001, 1.205, 1.25, 1.295, 1.3399999999999999, 1.3849999999999998, 1.4299999999999997, 1.475, 1.52, 1.565, 1.6099999999999999, 1.6549999999999998, 1.6999999999999997, 1.745, 1.79, 1.835, 1.88, 1.9249999999999998, 1.9699999999999998, 2.0149999999999997, 2.06, 2.105, 2.15, 2.195, 2.2399999999999998, 2.2849999999999997, 2.33, 2.375, 2.42, 2.465, 2.51, 2.5549999999999997, 2.5999999999999996, 2.645, 2.69, 2.735, 2.78, 2.8249999999999997, 2.8699999999999997, 2.915, 2.96, 3.005, 3.05, 3.0949999999999998, 3.1399999999999997, 3.1849999999999996, 3.2299999999999995, 3.2749999999999995, 3.3200000000000003, 3.365, 3.41, 3.455, 3.5 ], [ -1, -0.955, -0.91, -0.865, -0.8200000000000001, -0.775, -0.73, -0.685, -0.64, -0.595, -0.55, -0.505, -0.45999999999999996, -0.41500000000000004, -0.37, -0.32500000000000007, -0.28, -0.235, -0.19000000000000006, -0.14500000000000002, -0.10000000000000009, -0.05500000000000005, -0.010000000000000009, 0.03499999999999992, 0.08000000000000007, 0.125, 0.16999999999999993, 0.21499999999999986, 0.26, 0.30499999999999994, 0.34999999999999987, 0.395, 0.43999999999999995, 0.4849999999999999, 0.53, 0.575, 0.6199999999999999, 0.665, 0.71, 0.7549999999999999, 0.7999999999999998, 0.845, 0.8899999999999999, 0.9349999999999998, 0.98, 1.025, 1.0699999999999998, 1.1149999999999998, 1.1600000000000001, 1.205, 1.25, 1.295, 1.3399999999999999, 1.3849999999999998, 1.4299999999999997, 1.475, 1.52, 1.565, 1.6099999999999999, 1.6549999999999998, 1.6999999999999997, 1.745, 1.79, 1.835, 1.88, 1.9249999999999998, 1.9699999999999998, 2.0149999999999997, 2.06, 2.105, 2.15, 2.195, 2.2399999999999998, 2.2849999999999997, 2.33, 2.375, 2.42, 2.465, 2.51, 2.5549999999999997, 2.5999999999999996, 2.645, 2.69, 2.735, 2.78, 2.8249999999999997, 2.8699999999999997, 2.915, 2.96, 3.005, 3.05, 3.0949999999999998, 3.1399999999999997, 3.1849999999999996, 3.2299999999999995, 3.2749999999999995, 3.3200000000000003, 3.365, 3.41, 3.455, 3.5 ], [ -1, -0.955, -0.91, -0.865, -0.8200000000000001, -0.775, -0.73, -0.685, -0.64, -0.595, -0.55, -0.505, -0.45999999999999996, -0.41500000000000004, -0.37, -0.32500000000000007, -0.28, -0.235, -0.19000000000000006, -0.14500000000000002, -0.10000000000000009, -0.05500000000000005, -0.010000000000000009, 0.03499999999999992, 0.08000000000000007, 0.125, 0.16999999999999993, 0.21499999999999986, 0.26, 0.30499999999999994, 0.34999999999999987, 0.395, 0.43999999999999995, 0.4849999999999999, 0.53, 0.575, 0.6199999999999999, 0.665, 0.71, 0.7549999999999999, 0.7999999999999998, 0.845, 0.8899999999999999, 0.9349999999999998, 0.98, 1.025, 1.0699999999999998, 1.1149999999999998, 1.1600000000000001, 1.205, 1.25, 1.295, 1.3399999999999999, 1.3849999999999998, 1.4299999999999997, 1.475, 1.52, 1.565, 1.6099999999999999, 1.6549999999999998, 1.6999999999999997, 1.745, 1.79, 1.835, 1.88, 1.9249999999999998, 1.9699999999999998, 2.0149999999999997, 2.06, 2.105, 2.15, 2.195, 2.2399999999999998, 2.2849999999999997, 2.33, 2.375, 2.42, 2.465, 2.51, 2.5549999999999997, 2.5999999999999996, 2.645, 2.69, 2.735, 2.78, 2.8249999999999997, 2.8699999999999997, 2.915, 2.96, 3.005, 3.05, 3.0949999999999998, 3.1399999999999997, 3.1849999999999996, 3.2299999999999995, 3.2749999999999995, 3.3200000000000003, 3.365, 3.41, 3.455, 3.5 ], [ -1, -0.955, -0.91, -0.865, -0.8200000000000001, -0.775, -0.73, -0.685, -0.64, -0.595, -0.55, -0.505, -0.45999999999999996, -0.41500000000000004, -0.37, -0.32500000000000007, -0.28, -0.235, -0.19000000000000006, -0.14500000000000002, -0.10000000000000009, -0.05500000000000005, -0.010000000000000009, 0.03499999999999992, 0.08000000000000007, 0.125, 0.16999999999999993, 0.21499999999999986, 0.26, 0.30499999999999994, 0.34999999999999987, 0.395, 0.43999999999999995, 0.4849999999999999, 0.53, 0.575, 0.6199999999999999, 0.665, 0.71, 0.7549999999999999, 0.7999999999999998, 0.845, 0.8899999999999999, 0.9349999999999998, 0.98, 1.025, 1.0699999999999998, 1.1149999999999998, 1.1600000000000001, 1.205, 1.25, 1.295, 1.3399999999999999, 1.3849999999999998, 1.4299999999999997, 1.475, 1.52, 1.565, 1.6099999999999999, 1.6549999999999998, 1.6999999999999997, 1.745, 1.79, 1.835, 1.88, 1.9249999999999998, 1.9699999999999998, 2.0149999999999997, 2.06, 2.105, 2.15, 2.195, 2.2399999999999998, 2.2849999999999997, 2.33, 2.375, 2.42, 2.465, 2.51, 2.5549999999999997, 2.5999999999999996, 2.645, 2.69, 2.735, 2.78, 2.8249999999999997, 2.8699999999999997, 2.915, 2.96, 3.005, 3.05, 3.0949999999999998, 3.1399999999999997, 3.1849999999999996, 3.2299999999999995, 3.2749999999999995, 3.3200000000000003, 3.365, 3.41, 3.455, 3.5 ], [ -1, -0.955, -0.91, -0.865, -0.8200000000000001, -0.775, -0.73, -0.685, -0.64, -0.595, -0.55, -0.505, -0.45999999999999996, -0.41500000000000004, -0.37, -0.32500000000000007, -0.28, -0.235, -0.19000000000000006, -0.14500000000000002, -0.10000000000000009, -0.05500000000000005, -0.010000000000000009, 0.03499999999999992, 0.08000000000000007, 0.125, 0.16999999999999993, 0.21499999999999986, 0.26, 0.30499999999999994, 0.34999999999999987, 0.395, 0.43999999999999995, 0.4849999999999999, 0.53, 0.575, 0.6199999999999999, 0.665, 0.71, 0.7549999999999999, 0.7999999999999998, 0.845, 0.8899999999999999, 0.9349999999999998, 0.98, 1.025, 1.0699999999999998, 1.1149999999999998, 1.1600000000000001, 1.205, 1.25, 1.295, 1.3399999999999999, 1.3849999999999998, 1.4299999999999997, 1.475, 1.52, 1.565, 1.6099999999999999, 1.6549999999999998, 1.6999999999999997, 1.745, 1.79, 1.835, 1.88, 1.9249999999999998, 1.9699999999999998, 2.0149999999999997, 2.06, 2.105, 2.15, 2.195, 2.2399999999999998, 2.2849999999999997, 2.33, 2.375, 2.42, 2.465, 2.51, 2.5549999999999997, 2.5999999999999996, 2.645, 2.69, 2.735, 2.78, 2.8249999999999997, 2.8699999999999997, 2.915, 2.96, 3.005, 3.05, 3.0949999999999998, 3.1399999999999997, 3.1849999999999996, 3.2299999999999995, 3.2749999999999995, 3.3200000000000003, 3.365, 3.41, 3.455, 3.5 ], [ -1, -0.955, -0.91, -0.865, -0.8200000000000001, -0.775, -0.73, -0.685, -0.64, -0.595, -0.55, -0.505, -0.45999999999999996, -0.41500000000000004, -0.37, -0.32500000000000007, -0.28, -0.235, -0.19000000000000006, -0.14500000000000002, -0.10000000000000009, -0.05500000000000005, -0.010000000000000009, 0.03499999999999992, 0.08000000000000007, 0.125, 0.16999999999999993, 0.21499999999999986, 0.26, 0.30499999999999994, 0.34999999999999987, 0.395, 0.43999999999999995, 0.4849999999999999, 0.53, 0.575, 0.6199999999999999, 0.665, 0.71, 0.7549999999999999, 0.7999999999999998, 0.845, 0.8899999999999999, 0.9349999999999998, 0.98, 1.025, 1.0699999999999998, 1.1149999999999998, 1.1600000000000001, 1.205, 1.25, 1.295, 1.3399999999999999, 1.3849999999999998, 1.4299999999999997, 1.475, 1.52, 1.565, 1.6099999999999999, 1.6549999999999998, 1.6999999999999997, 1.745, 1.79, 1.835, 1.88, 1.9249999999999998, 1.9699999999999998, 2.0149999999999997, 2.06, 2.105, 2.15, 2.195, 2.2399999999999998, 2.2849999999999997, 2.33, 2.375, 2.42, 2.465, 2.51, 2.5549999999999997, 2.5999999999999996, 2.645, 2.69, 2.735, 2.78, 2.8249999999999997, 2.8699999999999997, 2.915, 2.96, 3.005, 3.05, 3.0949999999999998, 3.1399999999999997, 3.1849999999999996, 3.2299999999999995, 3.2749999999999995, 3.3200000000000003, 3.365, 3.41, 3.455, 3.5 ], [ -1, -0.955, -0.91, -0.865, -0.8200000000000001, -0.775, -0.73, -0.685, -0.64, -0.595, -0.55, -0.505, -0.45999999999999996, -0.41500000000000004, -0.37, -0.32500000000000007, -0.28, -0.235, -0.19000000000000006, -0.14500000000000002, -0.10000000000000009, -0.05500000000000005, -0.010000000000000009, 0.03499999999999992, 0.08000000000000007, 0.125, 0.16999999999999993, 0.21499999999999986, 0.26, 0.30499999999999994, 0.34999999999999987, 0.395, 0.43999999999999995, 0.4849999999999999, 0.53, 0.575, 0.6199999999999999, 0.665, 0.71, 0.7549999999999999, 0.7999999999999998, 0.845, 0.8899999999999999, 0.9349999999999998, 0.98, 1.025, 1.0699999999999998, 1.1149999999999998, 1.1600000000000001, 1.205, 1.25, 1.295, 1.3399999999999999, 1.3849999999999998, 1.4299999999999997, 1.475, 1.52, 1.565, 1.6099999999999999, 1.6549999999999998, 1.6999999999999997, 1.745, 1.79, 1.835, 1.88, 1.9249999999999998, 1.9699999999999998, 2.0149999999999997, 2.06, 2.105, 2.15, 2.195, 2.2399999999999998, 2.2849999999999997, 2.33, 2.375, 2.42, 2.465, 2.51, 2.5549999999999997, 2.5999999999999996, 2.645, 2.69, 2.735, 2.78, 2.8249999999999997, 2.8699999999999997, 2.915, 2.96, 3.005, 3.05, 3.0949999999999998, 3.1399999999999997, 3.1849999999999996, 3.2299999999999995, 3.2749999999999995, 3.3200000000000003, 3.365, 3.41, 3.455, 3.5 ], [ -1, -0.955, -0.91, -0.865, -0.8200000000000001, -0.775, -0.73, -0.685, -0.64, -0.595, -0.55, -0.505, -0.45999999999999996, -0.41500000000000004, -0.37, -0.32500000000000007, -0.28, -0.235, -0.19000000000000006, -0.14500000000000002, -0.10000000000000009, -0.05500000000000005, -0.010000000000000009, 0.03499999999999992, 0.08000000000000007, 0.125, 0.16999999999999993, 0.21499999999999986, 0.26, 0.30499999999999994, 0.34999999999999987, 0.395, 0.43999999999999995, 0.4849999999999999, 0.53, 0.575, 0.6199999999999999, 0.665, 0.71, 0.7549999999999999, 0.7999999999999998, 0.845, 0.8899999999999999, 0.9349999999999998, 0.98, 1.025, 1.0699999999999998, 1.1149999999999998, 1.1600000000000001, 1.205, 1.25, 1.295, 1.3399999999999999, 1.3849999999999998, 1.4299999999999997, 1.475, 1.52, 1.565, 1.6099999999999999, 1.6549999999999998, 1.6999999999999997, 1.745, 1.79, 1.835, 1.88, 1.9249999999999998, 1.9699999999999998, 2.0149999999999997, 2.06, 2.105, 2.15, 2.195, 2.2399999999999998, 2.2849999999999997, 2.33, 2.375, 2.42, 2.465, 2.51, 2.5549999999999997, 2.5999999999999996, 2.645, 2.69, 2.735, 2.78, 2.8249999999999997, 2.8699999999999997, 2.915, 2.96, 3.005, 3.05, 3.0949999999999998, 3.1399999999999997, 3.1849999999999996, 3.2299999999999995, 3.2749999999999995, 3.3200000000000003, 3.365, 3.41, 3.455, 3.5 ], [ -1, -0.955, -0.91, -0.865, -0.8200000000000001, -0.775, -0.73, -0.685, -0.64, -0.595, -0.55, -0.505, -0.45999999999999996, -0.41500000000000004, -0.37, -0.32500000000000007, -0.28, -0.235, -0.19000000000000006, -0.14500000000000002, -0.10000000000000009, -0.05500000000000005, -0.010000000000000009, 0.03499999999999992, 0.08000000000000007, 0.125, 0.16999999999999993, 0.21499999999999986, 0.26, 0.30499999999999994, 0.34999999999999987, 0.395, 0.43999999999999995, 0.4849999999999999, 0.53, 0.575, 0.6199999999999999, 0.665, 0.71, 0.7549999999999999, 0.7999999999999998, 0.845, 0.8899999999999999, 0.9349999999999998, 0.98, 1.025, 1.0699999999999998, 1.1149999999999998, 1.1600000000000001, 1.205, 1.25, 1.295, 1.3399999999999999, 1.3849999999999998, 1.4299999999999997, 1.475, 1.52, 1.565, 1.6099999999999999, 1.6549999999999998, 1.6999999999999997, 1.745, 1.79, 1.835, 1.88, 1.9249999999999998, 1.9699999999999998, 2.0149999999999997, 2.06, 2.105, 2.15, 2.195, 2.2399999999999998, 2.2849999999999997, 2.33, 2.375, 2.42, 2.465, 2.51, 2.5549999999999997, 2.5999999999999996, 2.645, 2.69, 2.735, 2.78, 2.8249999999999997, 2.8699999999999997, 2.915, 2.96, 3.005, 3.05, 3.0949999999999998, 3.1399999999999997, 3.1849999999999996, 3.2299999999999995, 3.2749999999999995, 3.3200000000000003, 3.365, 3.41, 3.455, 3.5 ], [ -1, -0.955, -0.91, -0.865, -0.8200000000000001, -0.775, -0.73, -0.685, -0.64, -0.595, -0.55, -0.505, -0.45999999999999996, -0.41500000000000004, -0.37, -0.32500000000000007, -0.28, -0.235, -0.19000000000000006, -0.14500000000000002, -0.10000000000000009, -0.05500000000000005, -0.010000000000000009, 0.03499999999999992, 0.08000000000000007, 0.125, 0.16999999999999993, 0.21499999999999986, 0.26, 0.30499999999999994, 0.34999999999999987, 0.395, 0.43999999999999995, 0.4849999999999999, 0.53, 0.575, 0.6199999999999999, 0.665, 0.71, 0.7549999999999999, 0.7999999999999998, 0.845, 0.8899999999999999, 0.9349999999999998, 0.98, 1.025, 1.0699999999999998, 1.1149999999999998, 1.1600000000000001, 1.205, 1.25, 1.295, 1.3399999999999999, 1.3849999999999998, 1.4299999999999997, 1.475, 1.52, 1.565, 1.6099999999999999, 1.6549999999999998, 1.6999999999999997, 1.745, 1.79, 1.835, 1.88, 1.9249999999999998, 1.9699999999999998, 2.0149999999999997, 2.06, 2.105, 2.15, 2.195, 2.2399999999999998, 2.2849999999999997, 2.33, 2.375, 2.42, 2.465, 2.51, 2.5549999999999997, 2.5999999999999996, 2.645, 2.69, 2.735, 2.78, 2.8249999999999997, 2.8699999999999997, 2.915, 2.96, 3.005, 3.05, 3.0949999999999998, 3.1399999999999997, 3.1849999999999996, 3.2299999999999995, 3.2749999999999995, 3.3200000000000003, 3.365, 3.41, 3.455, 3.5 ], [ -1, -0.955, -0.91, -0.865, -0.8200000000000001, -0.775, -0.73, -0.685, -0.64, -0.595, -0.55, -0.505, -0.45999999999999996, -0.41500000000000004, -0.37, -0.32500000000000007, -0.28, -0.235, -0.19000000000000006, -0.14500000000000002, -0.10000000000000009, -0.05500000000000005, -0.010000000000000009, 0.03499999999999992, 0.08000000000000007, 0.125, 0.16999999999999993, 0.21499999999999986, 0.26, 0.30499999999999994, 0.34999999999999987, 0.395, 0.43999999999999995, 0.4849999999999999, 0.53, 0.575, 0.6199999999999999, 0.665, 0.71, 0.7549999999999999, 0.7999999999999998, 0.845, 0.8899999999999999, 0.9349999999999998, 0.98, 1.025, 1.0699999999999998, 1.1149999999999998, 1.1600000000000001, 1.205, 1.25, 1.295, 1.3399999999999999, 1.3849999999999998, 1.4299999999999997, 1.475, 1.52, 1.565, 1.6099999999999999, 1.6549999999999998, 1.6999999999999997, 1.745, 1.79, 1.835, 1.88, 1.9249999999999998, 1.9699999999999998, 2.0149999999999997, 2.06, 2.105, 2.15, 2.195, 2.2399999999999998, 2.2849999999999997, 2.33, 2.375, 2.42, 2.465, 2.51, 2.5549999999999997, 2.5999999999999996, 2.645, 2.69, 2.735, 2.78, 2.8249999999999997, 2.8699999999999997, 2.915, 2.96, 3.005, 3.05, 3.0949999999999998, 3.1399999999999997, 3.1849999999999996, 3.2299999999999995, 3.2749999999999995, 3.3200000000000003, 3.365, 3.41, 3.455, 3.5 ], [ -1, -0.955, -0.91, -0.865, -0.8200000000000001, -0.775, -0.73, -0.685, -0.64, -0.595, -0.55, -0.505, -0.45999999999999996, -0.41500000000000004, -0.37, -0.32500000000000007, -0.28, -0.235, -0.19000000000000006, -0.14500000000000002, -0.10000000000000009, -0.05500000000000005, -0.010000000000000009, 0.03499999999999992, 0.08000000000000007, 0.125, 0.16999999999999993, 0.21499999999999986, 0.26, 0.30499999999999994, 0.34999999999999987, 0.395, 0.43999999999999995, 0.4849999999999999, 0.53, 0.575, 0.6199999999999999, 0.665, 0.71, 0.7549999999999999, 0.7999999999999998, 0.845, 0.8899999999999999, 0.9349999999999998, 0.98, 1.025, 1.0699999999999998, 1.1149999999999998, 1.1600000000000001, 1.205, 1.25, 1.295, 1.3399999999999999, 1.3849999999999998, 1.4299999999999997, 1.475, 1.52, 1.565, 1.6099999999999999, 1.6549999999999998, 1.6999999999999997, 1.745, 1.79, 1.835, 1.88, 1.9249999999999998, 1.9699999999999998, 2.0149999999999997, 2.06, 2.105, 2.15, 2.195, 2.2399999999999998, 2.2849999999999997, 2.33, 2.375, 2.42, 2.465, 2.51, 2.5549999999999997, 2.5999999999999996, 2.645, 2.69, 2.735, 2.78, 2.8249999999999997, 2.8699999999999997, 2.915, 2.96, 3.005, 3.05, 3.0949999999999998, 3.1399999999999997, 3.1849999999999996, 3.2299999999999995, 3.2749999999999995, 3.3200000000000003, 3.365, 3.41, 3.455, 3.5 ], [ -1, -0.955, -0.91, -0.865, -0.8200000000000001, -0.775, -0.73, -0.685, -0.64, -0.595, -0.55, -0.505, -0.45999999999999996, -0.41500000000000004, -0.37, -0.32500000000000007, -0.28, -0.235, -0.19000000000000006, -0.14500000000000002, -0.10000000000000009, -0.05500000000000005, -0.010000000000000009, 0.03499999999999992, 0.08000000000000007, 0.125, 0.16999999999999993, 0.21499999999999986, 0.26, 0.30499999999999994, 0.34999999999999987, 0.395, 0.43999999999999995, 0.4849999999999999, 0.53, 0.575, 0.6199999999999999, 0.665, 0.71, 0.7549999999999999, 0.7999999999999998, 0.845, 0.8899999999999999, 0.9349999999999998, 0.98, 1.025, 1.0699999999999998, 1.1149999999999998, 1.1600000000000001, 1.205, 1.25, 1.295, 1.3399999999999999, 1.3849999999999998, 1.4299999999999997, 1.475, 1.52, 1.565, 1.6099999999999999, 1.6549999999999998, 1.6999999999999997, 1.745, 1.79, 1.835, 1.88, 1.9249999999999998, 1.9699999999999998, 2.0149999999999997, 2.06, 2.105, 2.15, 2.195, 2.2399999999999998, 2.2849999999999997, 2.33, 2.375, 2.42, 2.465, 2.51, 2.5549999999999997, 2.5999999999999996, 2.645, 2.69, 2.735, 2.78, 2.8249999999999997, 2.8699999999999997, 2.915, 2.96, 3.005, 3.05, 3.0949999999999998, 3.1399999999999997, 3.1849999999999996, 3.2299999999999995, 3.2749999999999995, 3.3200000000000003, 3.365, 3.41, 3.455, 3.5 ], [ -1, -0.955, -0.91, -0.865, -0.8200000000000001, -0.775, -0.73, -0.685, -0.64, -0.595, -0.55, -0.505, -0.45999999999999996, -0.41500000000000004, -0.37, -0.32500000000000007, -0.28, -0.235, -0.19000000000000006, -0.14500000000000002, -0.10000000000000009, -0.05500000000000005, -0.010000000000000009, 0.03499999999999992, 0.08000000000000007, 0.125, 0.16999999999999993, 0.21499999999999986, 0.26, 0.30499999999999994, 0.34999999999999987, 0.395, 0.43999999999999995, 0.4849999999999999, 0.53, 0.575, 0.6199999999999999, 0.665, 0.71, 0.7549999999999999, 0.7999999999999998, 0.845, 0.8899999999999999, 0.9349999999999998, 0.98, 1.025, 1.0699999999999998, 1.1149999999999998, 1.1600000000000001, 1.205, 1.25, 1.295, 1.3399999999999999, 1.3849999999999998, 1.4299999999999997, 1.475, 1.52, 1.565, 1.6099999999999999, 1.6549999999999998, 1.6999999999999997, 1.745, 1.79, 1.835, 1.88, 1.9249999999999998, 1.9699999999999998, 2.0149999999999997, 2.06, 2.105, 2.15, 2.195, 2.2399999999999998, 2.2849999999999997, 2.33, 2.375, 2.42, 2.465, 2.51, 2.5549999999999997, 2.5999999999999996, 2.645, 2.69, 2.735, 2.78, 2.8249999999999997, 2.8699999999999997, 2.915, 2.96, 3.005, 3.05, 3.0949999999999998, 3.1399999999999997, 3.1849999999999996, 3.2299999999999995, 3.2749999999999995, 3.3200000000000003, 3.365, 3.41, 3.455, 3.5 ], [ -1, -0.955, -0.91, -0.865, -0.8200000000000001, -0.775, -0.73, -0.685, -0.64, -0.595, -0.55, -0.505, -0.45999999999999996, -0.41500000000000004, -0.37, -0.32500000000000007, -0.28, -0.235, -0.19000000000000006, -0.14500000000000002, -0.10000000000000009, -0.05500000000000005, -0.010000000000000009, 0.03499999999999992, 0.08000000000000007, 0.125, 0.16999999999999993, 0.21499999999999986, 0.26, 0.30499999999999994, 0.34999999999999987, 0.395, 0.43999999999999995, 0.4849999999999999, 0.53, 0.575, 0.6199999999999999, 0.665, 0.71, 0.7549999999999999, 0.7999999999999998, 0.845, 0.8899999999999999, 0.9349999999999998, 0.98, 1.025, 1.0699999999999998, 1.1149999999999998, 1.1600000000000001, 1.205, 1.25, 1.295, 1.3399999999999999, 1.3849999999999998, 1.4299999999999997, 1.475, 1.52, 1.565, 1.6099999999999999, 1.6549999999999998, 1.6999999999999997, 1.745, 1.79, 1.835, 1.88, 1.9249999999999998, 1.9699999999999998, 2.0149999999999997, 2.06, 2.105, 2.15, 2.195, 2.2399999999999998, 2.2849999999999997, 2.33, 2.375, 2.42, 2.465, 2.51, 2.5549999999999997, 2.5999999999999996, 2.645, 2.69, 2.735, 2.78, 2.8249999999999997, 2.8699999999999997, 2.915, 2.96, 3.005, 3.05, 3.0949999999999998, 3.1399999999999997, 3.1849999999999996, 3.2299999999999995, 3.2749999999999995, 3.3200000000000003, 3.365, 3.41, 3.455, 3.5 ], [ -1, -0.955, -0.91, -0.865, -0.8200000000000001, -0.775, -0.73, -0.685, -0.64, -0.595, -0.55, -0.505, -0.45999999999999996, -0.41500000000000004, -0.37, -0.32500000000000007, -0.28, -0.235, -0.19000000000000006, -0.14500000000000002, -0.10000000000000009, -0.05500000000000005, -0.010000000000000009, 0.03499999999999992, 0.08000000000000007, 0.125, 0.16999999999999993, 0.21499999999999986, 0.26, 0.30499999999999994, 0.34999999999999987, 0.395, 0.43999999999999995, 0.4849999999999999, 0.53, 0.575, 0.6199999999999999, 0.665, 0.71, 0.7549999999999999, 0.7999999999999998, 0.845, 0.8899999999999999, 0.9349999999999998, 0.98, 1.025, 1.0699999999999998, 1.1149999999999998, 1.1600000000000001, 1.205, 1.25, 1.295, 1.3399999999999999, 1.3849999999999998, 1.4299999999999997, 1.475, 1.52, 1.565, 1.6099999999999999, 1.6549999999999998, 1.6999999999999997, 1.745, 1.79, 1.835, 1.88, 1.9249999999999998, 1.9699999999999998, 2.0149999999999997, 2.06, 2.105, 2.15, 2.195, 2.2399999999999998, 2.2849999999999997, 2.33, 2.375, 2.42, 2.465, 2.51, 2.5549999999999997, 2.5999999999999996, 2.645, 2.69, 2.735, 2.78, 2.8249999999999997, 2.8699999999999997, 2.915, 2.96, 3.005, 3.05, 3.0949999999999998, 3.1399999999999997, 3.1849999999999996, 3.2299999999999995, 3.2749999999999995, 3.3200000000000003, 3.365, 3.41, 3.455, 3.5 ], [ -1, -0.955, -0.91, -0.865, -0.8200000000000001, -0.775, -0.73, -0.685, -0.64, -0.595, -0.55, -0.505, -0.45999999999999996, -0.41500000000000004, -0.37, -0.32500000000000007, -0.28, -0.235, -0.19000000000000006, -0.14500000000000002, -0.10000000000000009, -0.05500000000000005, -0.010000000000000009, 0.03499999999999992, 0.08000000000000007, 0.125, 0.16999999999999993, 0.21499999999999986, 0.26, 0.30499999999999994, 0.34999999999999987, 0.395, 0.43999999999999995, 0.4849999999999999, 0.53, 0.575, 0.6199999999999999, 0.665, 0.71, 0.7549999999999999, 0.7999999999999998, 0.845, 0.8899999999999999, 0.9349999999999998, 0.98, 1.025, 1.0699999999999998, 1.1149999999999998, 1.1600000000000001, 1.205, 1.25, 1.295, 1.3399999999999999, 1.3849999999999998, 1.4299999999999997, 1.475, 1.52, 1.565, 1.6099999999999999, 1.6549999999999998, 1.6999999999999997, 1.745, 1.79, 1.835, 1.88, 1.9249999999999998, 1.9699999999999998, 2.0149999999999997, 2.06, 2.105, 2.15, 2.195, 2.2399999999999998, 2.2849999999999997, 2.33, 2.375, 2.42, 2.465, 2.51, 2.5549999999999997, 2.5999999999999996, 2.645, 2.69, 2.735, 2.78, 2.8249999999999997, 2.8699999999999997, 2.915, 2.96, 3.005, 3.05, 3.0949999999999998, 3.1399999999999997, 3.1849999999999996, 3.2299999999999995, 3.2749999999999995, 3.3200000000000003, 3.365, 3.41, 3.455, 3.5 ], [ -1, -0.955, -0.91, -0.865, -0.8200000000000001, -0.775, -0.73, -0.685, -0.64, -0.595, -0.55, -0.505, -0.45999999999999996, -0.41500000000000004, -0.37, -0.32500000000000007, -0.28, -0.235, -0.19000000000000006, -0.14500000000000002, -0.10000000000000009, -0.05500000000000005, -0.010000000000000009, 0.03499999999999992, 0.08000000000000007, 0.125, 0.16999999999999993, 0.21499999999999986, 0.26, 0.30499999999999994, 0.34999999999999987, 0.395, 0.43999999999999995, 0.4849999999999999, 0.53, 0.575, 0.6199999999999999, 0.665, 0.71, 0.7549999999999999, 0.7999999999999998, 0.845, 0.8899999999999999, 0.9349999999999998, 0.98, 1.025, 1.0699999999999998, 1.1149999999999998, 1.1600000000000001, 1.205, 1.25, 1.295, 1.3399999999999999, 1.3849999999999998, 1.4299999999999997, 1.475, 1.52, 1.565, 1.6099999999999999, 1.6549999999999998, 1.6999999999999997, 1.745, 1.79, 1.835, 1.88, 1.9249999999999998, 1.9699999999999998, 2.0149999999999997, 2.06, 2.105, 2.15, 2.195, 2.2399999999999998, 2.2849999999999997, 2.33, 2.375, 2.42, 2.465, 2.51, 2.5549999999999997, 2.5999999999999996, 2.645, 2.69, 2.735, 2.78, 2.8249999999999997, 2.8699999999999997, 2.915, 2.96, 3.005, 3.05, 3.0949999999999998, 3.1399999999999997, 3.1849999999999996, 3.2299999999999995, 3.2749999999999995, 3.3200000000000003, 3.365, 3.41, 3.455, 3.5 ], [ -1, -0.955, -0.91, -0.865, -0.8200000000000001, -0.775, -0.73, -0.685, -0.64, -0.595, -0.55, -0.505, -0.45999999999999996, -0.41500000000000004, -0.37, -0.32500000000000007, -0.28, -0.235, -0.19000000000000006, -0.14500000000000002, -0.10000000000000009, -0.05500000000000005, -0.010000000000000009, 0.03499999999999992, 0.08000000000000007, 0.125, 0.16999999999999993, 0.21499999999999986, 0.26, 0.30499999999999994, 0.34999999999999987, 0.395, 0.43999999999999995, 0.4849999999999999, 0.53, 0.575, 0.6199999999999999, 0.665, 0.71, 0.7549999999999999, 0.7999999999999998, 0.845, 0.8899999999999999, 0.9349999999999998, 0.98, 1.025, 1.0699999999999998, 1.1149999999999998, 1.1600000000000001, 1.205, 1.25, 1.295, 1.3399999999999999, 1.3849999999999998, 1.4299999999999997, 1.475, 1.52, 1.565, 1.6099999999999999, 1.6549999999999998, 1.6999999999999997, 1.745, 1.79, 1.835, 1.88, 1.9249999999999998, 1.9699999999999998, 2.0149999999999997, 2.06, 2.105, 2.15, 2.195, 2.2399999999999998, 2.2849999999999997, 2.33, 2.375, 2.42, 2.465, 2.51, 2.5549999999999997, 2.5999999999999996, 2.645, 2.69, 2.735, 2.78, 2.8249999999999997, 2.8699999999999997, 2.915, 2.96, 3.005, 3.05, 3.0949999999999998, 3.1399999999999997, 3.1849999999999996, 3.2299999999999995, 3.2749999999999995, 3.3200000000000003, 3.365, 3.41, 3.455, 3.5 ], [ -1, -0.955, -0.91, -0.865, -0.8200000000000001, -0.775, -0.73, -0.685, -0.64, -0.595, -0.55, -0.505, -0.45999999999999996, -0.41500000000000004, -0.37, -0.32500000000000007, -0.28, -0.235, -0.19000000000000006, -0.14500000000000002, -0.10000000000000009, -0.05500000000000005, -0.010000000000000009, 0.03499999999999992, 0.08000000000000007, 0.125, 0.16999999999999993, 0.21499999999999986, 0.26, 0.30499999999999994, 0.34999999999999987, 0.395, 0.43999999999999995, 0.4849999999999999, 0.53, 0.575, 0.6199999999999999, 0.665, 0.71, 0.7549999999999999, 0.7999999999999998, 0.845, 0.8899999999999999, 0.9349999999999998, 0.98, 1.025, 1.0699999999999998, 1.1149999999999998, 1.1600000000000001, 1.205, 1.25, 1.295, 1.3399999999999999, 1.3849999999999998, 1.4299999999999997, 1.475, 1.52, 1.565, 1.6099999999999999, 1.6549999999999998, 1.6999999999999997, 1.745, 1.79, 1.835, 1.88, 1.9249999999999998, 1.9699999999999998, 2.0149999999999997, 2.06, 2.105, 2.15, 2.195, 2.2399999999999998, 2.2849999999999997, 2.33, 2.375, 2.42, 2.465, 2.51, 2.5549999999999997, 2.5999999999999996, 2.645, 2.69, 2.735, 2.78, 2.8249999999999997, 2.8699999999999997, 2.915, 2.96, 3.005, 3.05, 3.0949999999999998, 3.1399999999999997, 3.1849999999999996, 3.2299999999999995, 3.2749999999999995, 3.3200000000000003, 3.365, 3.41, 3.455, 3.5 ], [ -1, -0.955, -0.91, -0.865, -0.8200000000000001, -0.775, -0.73, -0.685, -0.64, -0.595, -0.55, -0.505, -0.45999999999999996, -0.41500000000000004, -0.37, -0.32500000000000007, -0.28, -0.235, -0.19000000000000006, -0.14500000000000002, -0.10000000000000009, -0.05500000000000005, -0.010000000000000009, 0.03499999999999992, 0.08000000000000007, 0.125, 0.16999999999999993, 0.21499999999999986, 0.26, 0.30499999999999994, 0.34999999999999987, 0.395, 0.43999999999999995, 0.4849999999999999, 0.53, 0.575, 0.6199999999999999, 0.665, 0.71, 0.7549999999999999, 0.7999999999999998, 0.845, 0.8899999999999999, 0.9349999999999998, 0.98, 1.025, 1.0699999999999998, 1.1149999999999998, 1.1600000000000001, 1.205, 1.25, 1.295, 1.3399999999999999, 1.3849999999999998, 1.4299999999999997, 1.475, 1.52, 1.565, 1.6099999999999999, 1.6549999999999998, 1.6999999999999997, 1.745, 1.79, 1.835, 1.88, 1.9249999999999998, 1.9699999999999998, 2.0149999999999997, 2.06, 2.105, 2.15, 2.195, 2.2399999999999998, 2.2849999999999997, 2.33, 2.375, 2.42, 2.465, 2.51, 2.5549999999999997, 2.5999999999999996, 2.645, 2.69, 2.735, 2.78, 2.8249999999999997, 2.8699999999999997, 2.915, 2.96, 3.005, 3.05, 3.0949999999999998, 3.1399999999999997, 3.1849999999999996, 3.2299999999999995, 3.2749999999999995, 3.3200000000000003, 3.365, 3.41, 3.455, 3.5 ], [ -1, -0.955, -0.91, -0.865, -0.8200000000000001, -0.775, -0.73, -0.685, -0.64, -0.595, -0.55, -0.505, -0.45999999999999996, -0.41500000000000004, -0.37, -0.32500000000000007, -0.28, -0.235, -0.19000000000000006, -0.14500000000000002, -0.10000000000000009, -0.05500000000000005, -0.010000000000000009, 0.03499999999999992, 0.08000000000000007, 0.125, 0.16999999999999993, 0.21499999999999986, 0.26, 0.30499999999999994, 0.34999999999999987, 0.395, 0.43999999999999995, 0.4849999999999999, 0.53, 0.575, 0.6199999999999999, 0.665, 0.71, 0.7549999999999999, 0.7999999999999998, 0.845, 0.8899999999999999, 0.9349999999999998, 0.98, 1.025, 1.0699999999999998, 1.1149999999999998, 1.1600000000000001, 1.205, 1.25, 1.295, 1.3399999999999999, 1.3849999999999998, 1.4299999999999997, 1.475, 1.52, 1.565, 1.6099999999999999, 1.6549999999999998, 1.6999999999999997, 1.745, 1.79, 1.835, 1.88, 1.9249999999999998, 1.9699999999999998, 2.0149999999999997, 2.06, 2.105, 2.15, 2.195, 2.2399999999999998, 2.2849999999999997, 2.33, 2.375, 2.42, 2.465, 2.51, 2.5549999999999997, 2.5999999999999996, 2.645, 2.69, 2.735, 2.78, 2.8249999999999997, 2.8699999999999997, 2.915, 2.96, 3.005, 3.05, 3.0949999999999998, 3.1399999999999997, 3.1849999999999996, 3.2299999999999995, 3.2749999999999995, 3.3200000000000003, 3.365, 3.41, 3.455, 3.5 ], [ -1, -0.955, -0.91, -0.865, -0.8200000000000001, -0.775, -0.73, -0.685, -0.64, -0.595, -0.55, -0.505, -0.45999999999999996, -0.41500000000000004, -0.37, -0.32500000000000007, -0.28, -0.235, -0.19000000000000006, -0.14500000000000002, -0.10000000000000009, -0.05500000000000005, -0.010000000000000009, 0.03499999999999992, 0.08000000000000007, 0.125, 0.16999999999999993, 0.21499999999999986, 0.26, 0.30499999999999994, 0.34999999999999987, 0.395, 0.43999999999999995, 0.4849999999999999, 0.53, 0.575, 0.6199999999999999, 0.665, 0.71, 0.7549999999999999, 0.7999999999999998, 0.845, 0.8899999999999999, 0.9349999999999998, 0.98, 1.025, 1.0699999999999998, 1.1149999999999998, 1.1600000000000001, 1.205, 1.25, 1.295, 1.3399999999999999, 1.3849999999999998, 1.4299999999999997, 1.475, 1.52, 1.565, 1.6099999999999999, 1.6549999999999998, 1.6999999999999997, 1.745, 1.79, 1.835, 1.88, 1.9249999999999998, 1.9699999999999998, 2.0149999999999997, 2.06, 2.105, 2.15, 2.195, 2.2399999999999998, 2.2849999999999997, 2.33, 2.375, 2.42, 2.465, 2.51, 2.5549999999999997, 2.5999999999999996, 2.645, 2.69, 2.735, 2.78, 2.8249999999999997, 2.8699999999999997, 2.915, 2.96, 3.005, 3.05, 3.0949999999999998, 3.1399999999999997, 3.1849999999999996, 3.2299999999999995, 3.2749999999999995, 3.3200000000000003, 3.365, 3.41, 3.455, 3.5 ], [ -1, -0.955, -0.91, -0.865, -0.8200000000000001, -0.775, -0.73, -0.685, -0.64, -0.595, -0.55, -0.505, -0.45999999999999996, -0.41500000000000004, -0.37, -0.32500000000000007, -0.28, -0.235, -0.19000000000000006, -0.14500000000000002, -0.10000000000000009, -0.05500000000000005, -0.010000000000000009, 0.03499999999999992, 0.08000000000000007, 0.125, 0.16999999999999993, 0.21499999999999986, 0.26, 0.30499999999999994, 0.34999999999999987, 0.395, 0.43999999999999995, 0.4849999999999999, 0.53, 0.575, 0.6199999999999999, 0.665, 0.71, 0.7549999999999999, 0.7999999999999998, 0.845, 0.8899999999999999, 0.9349999999999998, 0.98, 1.025, 1.0699999999999998, 1.1149999999999998, 1.1600000000000001, 1.205, 1.25, 1.295, 1.3399999999999999, 1.3849999999999998, 1.4299999999999997, 1.475, 1.52, 1.565, 1.6099999999999999, 1.6549999999999998, 1.6999999999999997, 1.745, 1.79, 1.835, 1.88, 1.9249999999999998, 1.9699999999999998, 2.0149999999999997, 2.06, 2.105, 2.15, 2.195, 2.2399999999999998, 2.2849999999999997, 2.33, 2.375, 2.42, 2.465, 2.51, 2.5549999999999997, 2.5999999999999996, 2.645, 2.69, 2.735, 2.78, 2.8249999999999997, 2.8699999999999997, 2.915, 2.96, 3.005, 3.05, 3.0949999999999998, 3.1399999999999997, 3.1849999999999996, 3.2299999999999995, 3.2749999999999995, 3.3200000000000003, 3.365, 3.41, 3.455, 3.5 ], [ -1, -0.955, -0.91, -0.865, -0.8200000000000001, -0.775, -0.73, -0.685, -0.64, -0.595, -0.55, -0.505, -0.45999999999999996, -0.41500000000000004, -0.37, -0.32500000000000007, -0.28, -0.235, -0.19000000000000006, -0.14500000000000002, -0.10000000000000009, -0.05500000000000005, -0.010000000000000009, 0.03499999999999992, 0.08000000000000007, 0.125, 0.16999999999999993, 0.21499999999999986, 0.26, 0.30499999999999994, 0.34999999999999987, 0.395, 0.43999999999999995, 0.4849999999999999, 0.53, 0.575, 0.6199999999999999, 0.665, 0.71, 0.7549999999999999, 0.7999999999999998, 0.845, 0.8899999999999999, 0.9349999999999998, 0.98, 1.025, 1.0699999999999998, 1.1149999999999998, 1.1600000000000001, 1.205, 1.25, 1.295, 1.3399999999999999, 1.3849999999999998, 1.4299999999999997, 1.475, 1.52, 1.565, 1.6099999999999999, 1.6549999999999998, 1.6999999999999997, 1.745, 1.79, 1.835, 1.88, 1.9249999999999998, 1.9699999999999998, 2.0149999999999997, 2.06, 2.105, 2.15, 2.195, 2.2399999999999998, 2.2849999999999997, 2.33, 2.375, 2.42, 2.465, 2.51, 2.5549999999999997, 2.5999999999999996, 2.645, 2.69, 2.735, 2.78, 2.8249999999999997, 2.8699999999999997, 2.915, 2.96, 3.005, 3.05, 3.0949999999999998, 3.1399999999999997, 3.1849999999999996, 3.2299999999999995, 3.2749999999999995, 3.3200000000000003, 3.365, 3.41, 3.455, 3.5 ], [ -1, -0.955, -0.91, -0.865, -0.8200000000000001, -0.775, -0.73, -0.685, -0.64, -0.595, -0.55, -0.505, -0.45999999999999996, -0.41500000000000004, -0.37, -0.32500000000000007, -0.28, -0.235, -0.19000000000000006, -0.14500000000000002, -0.10000000000000009, -0.05500000000000005, -0.010000000000000009, 0.03499999999999992, 0.08000000000000007, 0.125, 0.16999999999999993, 0.21499999999999986, 0.26, 0.30499999999999994, 0.34999999999999987, 0.395, 0.43999999999999995, 0.4849999999999999, 0.53, 0.575, 0.6199999999999999, 0.665, 0.71, 0.7549999999999999, 0.7999999999999998, 0.845, 0.8899999999999999, 0.9349999999999998, 0.98, 1.025, 1.0699999999999998, 1.1149999999999998, 1.1600000000000001, 1.205, 1.25, 1.295, 1.3399999999999999, 1.3849999999999998, 1.4299999999999997, 1.475, 1.52, 1.565, 1.6099999999999999, 1.6549999999999998, 1.6999999999999997, 1.745, 1.79, 1.835, 1.88, 1.9249999999999998, 1.9699999999999998, 2.0149999999999997, 2.06, 2.105, 2.15, 2.195, 2.2399999999999998, 2.2849999999999997, 2.33, 2.375, 2.42, 2.465, 2.51, 2.5549999999999997, 2.5999999999999996, 2.645, 2.69, 2.735, 2.78, 2.8249999999999997, 2.8699999999999997, 2.915, 2.96, 3.005, 3.05, 3.0949999999999998, 3.1399999999999997, 3.1849999999999996, 3.2299999999999995, 3.2749999999999995, 3.3200000000000003, 3.365, 3.41, 3.455, 3.5 ], [ -1, -0.955, -0.91, -0.865, -0.8200000000000001, -0.775, -0.73, -0.685, -0.64, -0.595, -0.55, -0.505, -0.45999999999999996, -0.41500000000000004, -0.37, -0.32500000000000007, -0.28, -0.235, -0.19000000000000006, -0.14500000000000002, -0.10000000000000009, -0.05500000000000005, -0.010000000000000009, 0.03499999999999992, 0.08000000000000007, 0.125, 0.16999999999999993, 0.21499999999999986, 0.26, 0.30499999999999994, 0.34999999999999987, 0.395, 0.43999999999999995, 0.4849999999999999, 0.53, 0.575, 0.6199999999999999, 0.665, 0.71, 0.7549999999999999, 0.7999999999999998, 0.845, 0.8899999999999999, 0.9349999999999998, 0.98, 1.025, 1.0699999999999998, 1.1149999999999998, 1.1600000000000001, 1.205, 1.25, 1.295, 1.3399999999999999, 1.3849999999999998, 1.4299999999999997, 1.475, 1.52, 1.565, 1.6099999999999999, 1.6549999999999998, 1.6999999999999997, 1.745, 1.79, 1.835, 1.88, 1.9249999999999998, 1.9699999999999998, 2.0149999999999997, 2.06, 2.105, 2.15, 2.195, 2.2399999999999998, 2.2849999999999997, 2.33, 2.375, 2.42, 2.465, 2.51, 2.5549999999999997, 2.5999999999999996, 2.645, 2.69, 2.735, 2.78, 2.8249999999999997, 2.8699999999999997, 2.915, 2.96, 3.005, 3.05, 3.0949999999999998, 3.1399999999999997, 3.1849999999999996, 3.2299999999999995, 3.2749999999999995, 3.3200000000000003, 3.365, 3.41, 3.455, 3.5 ], [ -1, -0.955, -0.91, -0.865, -0.8200000000000001, -0.775, -0.73, -0.685, -0.64, -0.595, -0.55, -0.505, -0.45999999999999996, -0.41500000000000004, -0.37, -0.32500000000000007, -0.28, -0.235, -0.19000000000000006, -0.14500000000000002, -0.10000000000000009, -0.05500000000000005, -0.010000000000000009, 0.03499999999999992, 0.08000000000000007, 0.125, 0.16999999999999993, 0.21499999999999986, 0.26, 0.30499999999999994, 0.34999999999999987, 0.395, 0.43999999999999995, 0.4849999999999999, 0.53, 0.575, 0.6199999999999999, 0.665, 0.71, 0.7549999999999999, 0.7999999999999998, 0.845, 0.8899999999999999, 0.9349999999999998, 0.98, 1.025, 1.0699999999999998, 1.1149999999999998, 1.1600000000000001, 1.205, 1.25, 1.295, 1.3399999999999999, 1.3849999999999998, 1.4299999999999997, 1.475, 1.52, 1.565, 1.6099999999999999, 1.6549999999999998, 1.6999999999999997, 1.745, 1.79, 1.835, 1.88, 1.9249999999999998, 1.9699999999999998, 2.0149999999999997, 2.06, 2.105, 2.15, 2.195, 2.2399999999999998, 2.2849999999999997, 2.33, 2.375, 2.42, 2.465, 2.51, 2.5549999999999997, 2.5999999999999996, 2.645, 2.69, 2.735, 2.78, 2.8249999999999997, 2.8699999999999997, 2.915, 2.96, 3.005, 3.05, 3.0949999999999998, 3.1399999999999997, 3.1849999999999996, 3.2299999999999995, 3.2749999999999995, 3.3200000000000003, 3.365, 3.41, 3.455, 3.5 ], [ -1, -0.955, -0.91, -0.865, -0.8200000000000001, -0.775, -0.73, -0.685, -0.64, -0.595, -0.55, -0.505, -0.45999999999999996, -0.41500000000000004, -0.37, -0.32500000000000007, -0.28, -0.235, -0.19000000000000006, -0.14500000000000002, -0.10000000000000009, -0.05500000000000005, -0.010000000000000009, 0.03499999999999992, 0.08000000000000007, 0.125, 0.16999999999999993, 0.21499999999999986, 0.26, 0.30499999999999994, 0.34999999999999987, 0.395, 0.43999999999999995, 0.4849999999999999, 0.53, 0.575, 0.6199999999999999, 0.665, 0.71, 0.7549999999999999, 0.7999999999999998, 0.845, 0.8899999999999999, 0.9349999999999998, 0.98, 1.025, 1.0699999999999998, 1.1149999999999998, 1.1600000000000001, 1.205, 1.25, 1.295, 1.3399999999999999, 1.3849999999999998, 1.4299999999999997, 1.475, 1.52, 1.565, 1.6099999999999999, 1.6549999999999998, 1.6999999999999997, 1.745, 1.79, 1.835, 1.88, 1.9249999999999998, 1.9699999999999998, 2.0149999999999997, 2.06, 2.105, 2.15, 2.195, 2.2399999999999998, 2.2849999999999997, 2.33, 2.375, 2.42, 2.465, 2.51, 2.5549999999999997, 2.5999999999999996, 2.645, 2.69, 2.735, 2.78, 2.8249999999999997, 2.8699999999999997, 2.915, 2.96, 3.005, 3.05, 3.0949999999999998, 3.1399999999999997, 3.1849999999999996, 3.2299999999999995, 3.2749999999999995, 3.3200000000000003, 3.365, 3.41, 3.455, 3.5 ], [ -1, -0.955, -0.91, -0.865, -0.8200000000000001, -0.775, -0.73, -0.685, -0.64, -0.595, -0.55, -0.505, -0.45999999999999996, -0.41500000000000004, -0.37, -0.32500000000000007, -0.28, -0.235, -0.19000000000000006, -0.14500000000000002, -0.10000000000000009, -0.05500000000000005, -0.010000000000000009, 0.03499999999999992, 0.08000000000000007, 0.125, 0.16999999999999993, 0.21499999999999986, 0.26, 0.30499999999999994, 0.34999999999999987, 0.395, 0.43999999999999995, 0.4849999999999999, 0.53, 0.575, 0.6199999999999999, 0.665, 0.71, 0.7549999999999999, 0.7999999999999998, 0.845, 0.8899999999999999, 0.9349999999999998, 0.98, 1.025, 1.0699999999999998, 1.1149999999999998, 1.1600000000000001, 1.205, 1.25, 1.295, 1.3399999999999999, 1.3849999999999998, 1.4299999999999997, 1.475, 1.52, 1.565, 1.6099999999999999, 1.6549999999999998, 1.6999999999999997, 1.745, 1.79, 1.835, 1.88, 1.9249999999999998, 1.9699999999999998, 2.0149999999999997, 2.06, 2.105, 2.15, 2.195, 2.2399999999999998, 2.2849999999999997, 2.33, 2.375, 2.42, 2.465, 2.51, 2.5549999999999997, 2.5999999999999996, 2.645, 2.69, 2.735, 2.78, 2.8249999999999997, 2.8699999999999997, 2.915, 2.96, 3.005, 3.05, 3.0949999999999998, 3.1399999999999997, 3.1849999999999996, 3.2299999999999995, 3.2749999999999995, 3.3200000000000003, 3.365, 3.41, 3.455, 3.5 ], [ -1, -0.955, -0.91, -0.865, -0.8200000000000001, -0.775, -0.73, -0.685, -0.64, -0.595, -0.55, -0.505, -0.45999999999999996, -0.41500000000000004, -0.37, -0.32500000000000007, -0.28, -0.235, -0.19000000000000006, -0.14500000000000002, -0.10000000000000009, -0.05500000000000005, -0.010000000000000009, 0.03499999999999992, 0.08000000000000007, 0.125, 0.16999999999999993, 0.21499999999999986, 0.26, 0.30499999999999994, 0.34999999999999987, 0.395, 0.43999999999999995, 0.4849999999999999, 0.53, 0.575, 0.6199999999999999, 0.665, 0.71, 0.7549999999999999, 0.7999999999999998, 0.845, 0.8899999999999999, 0.9349999999999998, 0.98, 1.025, 1.0699999999999998, 1.1149999999999998, 1.1600000000000001, 1.205, 1.25, 1.295, 1.3399999999999999, 1.3849999999999998, 1.4299999999999997, 1.475, 1.52, 1.565, 1.6099999999999999, 1.6549999999999998, 1.6999999999999997, 1.745, 1.79, 1.835, 1.88, 1.9249999999999998, 1.9699999999999998, 2.0149999999999997, 2.06, 2.105, 2.15, 2.195, 2.2399999999999998, 2.2849999999999997, 2.33, 2.375, 2.42, 2.465, 2.51, 2.5549999999999997, 2.5999999999999996, 2.645, 2.69, 2.735, 2.78, 2.8249999999999997, 2.8699999999999997, 2.915, 2.96, 3.005, 3.05, 3.0949999999999998, 3.1399999999999997, 3.1849999999999996, 3.2299999999999995, 3.2749999999999995, 3.3200000000000003, 3.365, 3.41, 3.455, 3.5 ], [ -1, -0.955, -0.91, -0.865, -0.8200000000000001, -0.775, -0.73, -0.685, -0.64, -0.595, -0.55, -0.505, -0.45999999999999996, -0.41500000000000004, -0.37, -0.32500000000000007, -0.28, -0.235, -0.19000000000000006, -0.14500000000000002, -0.10000000000000009, -0.05500000000000005, -0.010000000000000009, 0.03499999999999992, 0.08000000000000007, 0.125, 0.16999999999999993, 0.21499999999999986, 0.26, 0.30499999999999994, 0.34999999999999987, 0.395, 0.43999999999999995, 0.4849999999999999, 0.53, 0.575, 0.6199999999999999, 0.665, 0.71, 0.7549999999999999, 0.7999999999999998, 0.845, 0.8899999999999999, 0.9349999999999998, 0.98, 1.025, 1.0699999999999998, 1.1149999999999998, 1.1600000000000001, 1.205, 1.25, 1.295, 1.3399999999999999, 1.3849999999999998, 1.4299999999999997, 1.475, 1.52, 1.565, 1.6099999999999999, 1.6549999999999998, 1.6999999999999997, 1.745, 1.79, 1.835, 1.88, 1.9249999999999998, 1.9699999999999998, 2.0149999999999997, 2.06, 2.105, 2.15, 2.195, 2.2399999999999998, 2.2849999999999997, 2.33, 2.375, 2.42, 2.465, 2.51, 2.5549999999999997, 2.5999999999999996, 2.645, 2.69, 2.735, 2.78, 2.8249999999999997, 2.8699999999999997, 2.915, 2.96, 3.005, 3.05, 3.0949999999999998, 3.1399999999999997, 3.1849999999999996, 3.2299999999999995, 3.2749999999999995, 3.3200000000000003, 3.365, 3.41, 3.455, 3.5 ], [ -1, -0.955, -0.91, -0.865, -0.8200000000000001, -0.775, -0.73, -0.685, -0.64, -0.595, -0.55, -0.505, -0.45999999999999996, -0.41500000000000004, -0.37, -0.32500000000000007, -0.28, -0.235, -0.19000000000000006, -0.14500000000000002, -0.10000000000000009, -0.05500000000000005, -0.010000000000000009, 0.03499999999999992, 0.08000000000000007, 0.125, 0.16999999999999993, 0.21499999999999986, 0.26, 0.30499999999999994, 0.34999999999999987, 0.395, 0.43999999999999995, 0.4849999999999999, 0.53, 0.575, 0.6199999999999999, 0.665, 0.71, 0.7549999999999999, 0.7999999999999998, 0.845, 0.8899999999999999, 0.9349999999999998, 0.98, 1.025, 1.0699999999999998, 1.1149999999999998, 1.1600000000000001, 1.205, 1.25, 1.295, 1.3399999999999999, 1.3849999999999998, 1.4299999999999997, 1.475, 1.52, 1.565, 1.6099999999999999, 1.6549999999999998, 1.6999999999999997, 1.745, 1.79, 1.835, 1.88, 1.9249999999999998, 1.9699999999999998, 2.0149999999999997, 2.06, 2.105, 2.15, 2.195, 2.2399999999999998, 2.2849999999999997, 2.33, 2.375, 2.42, 2.465, 2.51, 2.5549999999999997, 2.5999999999999996, 2.645, 2.69, 2.735, 2.78, 2.8249999999999997, 2.8699999999999997, 2.915, 2.96, 3.005, 3.05, 3.0949999999999998, 3.1399999999999997, 3.1849999999999996, 3.2299999999999995, 3.2749999999999995, 3.3200000000000003, 3.365, 3.41, 3.455, 3.5 ], [ -1, -0.955, -0.91, -0.865, -0.8200000000000001, -0.775, -0.73, -0.685, -0.64, -0.595, -0.55, -0.505, -0.45999999999999996, -0.41500000000000004, -0.37, -0.32500000000000007, -0.28, -0.235, -0.19000000000000006, -0.14500000000000002, -0.10000000000000009, -0.05500000000000005, -0.010000000000000009, 0.03499999999999992, 0.08000000000000007, 0.125, 0.16999999999999993, 0.21499999999999986, 0.26, 0.30499999999999994, 0.34999999999999987, 0.395, 0.43999999999999995, 0.4849999999999999, 0.53, 0.575, 0.6199999999999999, 0.665, 0.71, 0.7549999999999999, 0.7999999999999998, 0.845, 0.8899999999999999, 0.9349999999999998, 0.98, 1.025, 1.0699999999999998, 1.1149999999999998, 1.1600000000000001, 1.205, 1.25, 1.295, 1.3399999999999999, 1.3849999999999998, 1.4299999999999997, 1.475, 1.52, 1.565, 1.6099999999999999, 1.6549999999999998, 1.6999999999999997, 1.745, 1.79, 1.835, 1.88, 1.9249999999999998, 1.9699999999999998, 2.0149999999999997, 2.06, 2.105, 2.15, 2.195, 2.2399999999999998, 2.2849999999999997, 2.33, 2.375, 2.42, 2.465, 2.51, 2.5549999999999997, 2.5999999999999996, 2.645, 2.69, 2.735, 2.78, 2.8249999999999997, 2.8699999999999997, 2.915, 2.96, 3.005, 3.05, 3.0949999999999998, 3.1399999999999997, 3.1849999999999996, 3.2299999999999995, 3.2749999999999995, 3.3200000000000003, 3.365, 3.41, 3.455, 3.5 ], [ -1, -0.955, -0.91, -0.865, -0.8200000000000001, -0.775, -0.73, -0.685, -0.64, -0.595, -0.55, -0.505, -0.45999999999999996, -0.41500000000000004, -0.37, -0.32500000000000007, -0.28, -0.235, -0.19000000000000006, -0.14500000000000002, -0.10000000000000009, -0.05500000000000005, -0.010000000000000009, 0.03499999999999992, 0.08000000000000007, 0.125, 0.16999999999999993, 0.21499999999999986, 0.26, 0.30499999999999994, 0.34999999999999987, 0.395, 0.43999999999999995, 0.4849999999999999, 0.53, 0.575, 0.6199999999999999, 0.665, 0.71, 0.7549999999999999, 0.7999999999999998, 0.845, 0.8899999999999999, 0.9349999999999998, 0.98, 1.025, 1.0699999999999998, 1.1149999999999998, 1.1600000000000001, 1.205, 1.25, 1.295, 1.3399999999999999, 1.3849999999999998, 1.4299999999999997, 1.475, 1.52, 1.565, 1.6099999999999999, 1.6549999999999998, 1.6999999999999997, 1.745, 1.79, 1.835, 1.88, 1.9249999999999998, 1.9699999999999998, 2.0149999999999997, 2.06, 2.105, 2.15, 2.195, 2.2399999999999998, 2.2849999999999997, 2.33, 2.375, 2.42, 2.465, 2.51, 2.5549999999999997, 2.5999999999999996, 2.645, 2.69, 2.735, 2.78, 2.8249999999999997, 2.8699999999999997, 2.915, 2.96, 3.005, 3.05, 3.0949999999999998, 3.1399999999999997, 3.1849999999999996, 3.2299999999999995, 3.2749999999999995, 3.3200000000000003, 3.365, 3.41, 3.455, 3.5 ], [ -1, -0.955, -0.91, -0.865, -0.8200000000000001, -0.775, -0.73, -0.685, -0.64, -0.595, -0.55, -0.505, -0.45999999999999996, -0.41500000000000004, -0.37, -0.32500000000000007, -0.28, -0.235, -0.19000000000000006, -0.14500000000000002, -0.10000000000000009, -0.05500000000000005, -0.010000000000000009, 0.03499999999999992, 0.08000000000000007, 0.125, 0.16999999999999993, 0.21499999999999986, 0.26, 0.30499999999999994, 0.34999999999999987, 0.395, 0.43999999999999995, 0.4849999999999999, 0.53, 0.575, 0.6199999999999999, 0.665, 0.71, 0.7549999999999999, 0.7999999999999998, 0.845, 0.8899999999999999, 0.9349999999999998, 0.98, 1.025, 1.0699999999999998, 1.1149999999999998, 1.1600000000000001, 1.205, 1.25, 1.295, 1.3399999999999999, 1.3849999999999998, 1.4299999999999997, 1.475, 1.52, 1.565, 1.6099999999999999, 1.6549999999999998, 1.6999999999999997, 1.745, 1.79, 1.835, 1.88, 1.9249999999999998, 1.9699999999999998, 2.0149999999999997, 2.06, 2.105, 2.15, 2.195, 2.2399999999999998, 2.2849999999999997, 2.33, 2.375, 2.42, 2.465, 2.51, 2.5549999999999997, 2.5999999999999996, 2.645, 2.69, 2.735, 2.78, 2.8249999999999997, 2.8699999999999997, 2.915, 2.96, 3.005, 3.05, 3.0949999999999998, 3.1399999999999997, 3.1849999999999996, 3.2299999999999995, 3.2749999999999995, 3.3200000000000003, 3.365, 3.41, 3.455, 3.5 ], [ -1, -0.955, -0.91, -0.865, -0.8200000000000001, -0.775, -0.73, -0.685, -0.64, -0.595, -0.55, -0.505, -0.45999999999999996, -0.41500000000000004, -0.37, -0.32500000000000007, -0.28, -0.235, -0.19000000000000006, -0.14500000000000002, -0.10000000000000009, -0.05500000000000005, -0.010000000000000009, 0.03499999999999992, 0.08000000000000007, 0.125, 0.16999999999999993, 0.21499999999999986, 0.26, 0.30499999999999994, 0.34999999999999987, 0.395, 0.43999999999999995, 0.4849999999999999, 0.53, 0.575, 0.6199999999999999, 0.665, 0.71, 0.7549999999999999, 0.7999999999999998, 0.845, 0.8899999999999999, 0.9349999999999998, 0.98, 1.025, 1.0699999999999998, 1.1149999999999998, 1.1600000000000001, 1.205, 1.25, 1.295, 1.3399999999999999, 1.3849999999999998, 1.4299999999999997, 1.475, 1.52, 1.565, 1.6099999999999999, 1.6549999999999998, 1.6999999999999997, 1.745, 1.79, 1.835, 1.88, 1.9249999999999998, 1.9699999999999998, 2.0149999999999997, 2.06, 2.105, 2.15, 2.195, 2.2399999999999998, 2.2849999999999997, 2.33, 2.375, 2.42, 2.465, 2.51, 2.5549999999999997, 2.5999999999999996, 2.645, 2.69, 2.735, 2.78, 2.8249999999999997, 2.8699999999999997, 2.915, 2.96, 3.005, 3.05, 3.0949999999999998, 3.1399999999999997, 3.1849999999999996, 3.2299999999999995, 3.2749999999999995, 3.3200000000000003, 3.365, 3.41, 3.455, 3.5 ], [ -1, -0.955, -0.91, -0.865, -0.8200000000000001, -0.775, -0.73, -0.685, -0.64, -0.595, -0.55, -0.505, -0.45999999999999996, -0.41500000000000004, -0.37, -0.32500000000000007, -0.28, -0.235, -0.19000000000000006, -0.14500000000000002, -0.10000000000000009, -0.05500000000000005, -0.010000000000000009, 0.03499999999999992, 0.08000000000000007, 0.125, 0.16999999999999993, 0.21499999999999986, 0.26, 0.30499999999999994, 0.34999999999999987, 0.395, 0.43999999999999995, 0.4849999999999999, 0.53, 0.575, 0.6199999999999999, 0.665, 0.71, 0.7549999999999999, 0.7999999999999998, 0.845, 0.8899999999999999, 0.9349999999999998, 0.98, 1.025, 1.0699999999999998, 1.1149999999999998, 1.1600000000000001, 1.205, 1.25, 1.295, 1.3399999999999999, 1.3849999999999998, 1.4299999999999997, 1.475, 1.52, 1.565, 1.6099999999999999, 1.6549999999999998, 1.6999999999999997, 1.745, 1.79, 1.835, 1.88, 1.9249999999999998, 1.9699999999999998, 2.0149999999999997, 2.06, 2.105, 2.15, 2.195, 2.2399999999999998, 2.2849999999999997, 2.33, 2.375, 2.42, 2.465, 2.51, 2.5549999999999997, 2.5999999999999996, 2.645, 2.69, 2.735, 2.78, 2.8249999999999997, 2.8699999999999997, 2.915, 2.96, 3.005, 3.05, 3.0949999999999998, 3.1399999999999997, 3.1849999999999996, 3.2299999999999995, 3.2749999999999995, 3.3200000000000003, 3.365, 3.41, 3.455, 3.5 ], [ -1, -0.955, -0.91, -0.865, -0.8200000000000001, -0.775, -0.73, -0.685, -0.64, -0.595, -0.55, -0.505, -0.45999999999999996, -0.41500000000000004, -0.37, -0.32500000000000007, -0.28, -0.235, -0.19000000000000006, -0.14500000000000002, -0.10000000000000009, -0.05500000000000005, -0.010000000000000009, 0.03499999999999992, 0.08000000000000007, 0.125, 0.16999999999999993, 0.21499999999999986, 0.26, 0.30499999999999994, 0.34999999999999987, 0.395, 0.43999999999999995, 0.4849999999999999, 0.53, 0.575, 0.6199999999999999, 0.665, 0.71, 0.7549999999999999, 0.7999999999999998, 0.845, 0.8899999999999999, 0.9349999999999998, 0.98, 1.025, 1.0699999999999998, 1.1149999999999998, 1.1600000000000001, 1.205, 1.25, 1.295, 1.3399999999999999, 1.3849999999999998, 1.4299999999999997, 1.475, 1.52, 1.565, 1.6099999999999999, 1.6549999999999998, 1.6999999999999997, 1.745, 1.79, 1.835, 1.88, 1.9249999999999998, 1.9699999999999998, 2.0149999999999997, 2.06, 2.105, 2.15, 2.195, 2.2399999999999998, 2.2849999999999997, 2.33, 2.375, 2.42, 2.465, 2.51, 2.5549999999999997, 2.5999999999999996, 2.645, 2.69, 2.735, 2.78, 2.8249999999999997, 2.8699999999999997, 2.915, 2.96, 3.005, 3.05, 3.0949999999999998, 3.1399999999999997, 3.1849999999999996, 3.2299999999999995, 3.2749999999999995, 3.3200000000000003, 3.365, 3.41, 3.455, 3.5 ], [ -1, -0.955, -0.91, -0.865, -0.8200000000000001, -0.775, -0.73, -0.685, -0.64, -0.595, -0.55, -0.505, -0.45999999999999996, -0.41500000000000004, -0.37, -0.32500000000000007, -0.28, -0.235, -0.19000000000000006, -0.14500000000000002, -0.10000000000000009, -0.05500000000000005, -0.010000000000000009, 0.03499999999999992, 0.08000000000000007, 0.125, 0.16999999999999993, 0.21499999999999986, 0.26, 0.30499999999999994, 0.34999999999999987, 0.395, 0.43999999999999995, 0.4849999999999999, 0.53, 0.575, 0.6199999999999999, 0.665, 0.71, 0.7549999999999999, 0.7999999999999998, 0.845, 0.8899999999999999, 0.9349999999999998, 0.98, 1.025, 1.0699999999999998, 1.1149999999999998, 1.1600000000000001, 1.205, 1.25, 1.295, 1.3399999999999999, 1.3849999999999998, 1.4299999999999997, 1.475, 1.52, 1.565, 1.6099999999999999, 1.6549999999999998, 1.6999999999999997, 1.745, 1.79, 1.835, 1.88, 1.9249999999999998, 1.9699999999999998, 2.0149999999999997, 2.06, 2.105, 2.15, 2.195, 2.2399999999999998, 2.2849999999999997, 2.33, 2.375, 2.42, 2.465, 2.51, 2.5549999999999997, 2.5999999999999996, 2.645, 2.69, 2.735, 2.78, 2.8249999999999997, 2.8699999999999997, 2.915, 2.96, 3.005, 3.05, 3.0949999999999998, 3.1399999999999997, 3.1849999999999996, 3.2299999999999995, 3.2749999999999995, 3.3200000000000003, 3.365, 3.41, 3.455, 3.5 ], [ -1, -0.955, -0.91, -0.865, -0.8200000000000001, -0.775, -0.73, -0.685, -0.64, -0.595, -0.55, -0.505, -0.45999999999999996, -0.41500000000000004, -0.37, -0.32500000000000007, -0.28, -0.235, -0.19000000000000006, -0.14500000000000002, -0.10000000000000009, -0.05500000000000005, -0.010000000000000009, 0.03499999999999992, 0.08000000000000007, 0.125, 0.16999999999999993, 0.21499999999999986, 0.26, 0.30499999999999994, 0.34999999999999987, 0.395, 0.43999999999999995, 0.4849999999999999, 0.53, 0.575, 0.6199999999999999, 0.665, 0.71, 0.7549999999999999, 0.7999999999999998, 0.845, 0.8899999999999999, 0.9349999999999998, 0.98, 1.025, 1.0699999999999998, 1.1149999999999998, 1.1600000000000001, 1.205, 1.25, 1.295, 1.3399999999999999, 1.3849999999999998, 1.4299999999999997, 1.475, 1.52, 1.565, 1.6099999999999999, 1.6549999999999998, 1.6999999999999997, 1.745, 1.79, 1.835, 1.88, 1.9249999999999998, 1.9699999999999998, 2.0149999999999997, 2.06, 2.105, 2.15, 2.195, 2.2399999999999998, 2.2849999999999997, 2.33, 2.375, 2.42, 2.465, 2.51, 2.5549999999999997, 2.5999999999999996, 2.645, 2.69, 2.735, 2.78, 2.8249999999999997, 2.8699999999999997, 2.915, 2.96, 3.005, 3.05, 3.0949999999999998, 3.1399999999999997, 3.1849999999999996, 3.2299999999999995, 3.2749999999999995, 3.3200000000000003, 3.365, 3.41, 3.455, 3.5 ], [ -1, -0.955, -0.91, -0.865, -0.8200000000000001, -0.775, -0.73, -0.685, -0.64, -0.595, -0.55, -0.505, -0.45999999999999996, -0.41500000000000004, -0.37, -0.32500000000000007, -0.28, -0.235, -0.19000000000000006, -0.14500000000000002, -0.10000000000000009, -0.05500000000000005, -0.010000000000000009, 0.03499999999999992, 0.08000000000000007, 0.125, 0.16999999999999993, 0.21499999999999986, 0.26, 0.30499999999999994, 0.34999999999999987, 0.395, 0.43999999999999995, 0.4849999999999999, 0.53, 0.575, 0.6199999999999999, 0.665, 0.71, 0.7549999999999999, 0.7999999999999998, 0.845, 0.8899999999999999, 0.9349999999999998, 0.98, 1.025, 1.0699999999999998, 1.1149999999999998, 1.1600000000000001, 1.205, 1.25, 1.295, 1.3399999999999999, 1.3849999999999998, 1.4299999999999997, 1.475, 1.52, 1.565, 1.6099999999999999, 1.6549999999999998, 1.6999999999999997, 1.745, 1.79, 1.835, 1.88, 1.9249999999999998, 1.9699999999999998, 2.0149999999999997, 2.06, 2.105, 2.15, 2.195, 2.2399999999999998, 2.2849999999999997, 2.33, 2.375, 2.42, 2.465, 2.51, 2.5549999999999997, 2.5999999999999996, 2.645, 2.69, 2.735, 2.78, 2.8249999999999997, 2.8699999999999997, 2.915, 2.96, 3.005, 3.05, 3.0949999999999998, 3.1399999999999997, 3.1849999999999996, 3.2299999999999995, 3.2749999999999995, 3.3200000000000003, 3.365, 3.41, 3.455, 3.5 ], [ -1, -0.955, -0.91, -0.865, -0.8200000000000001, -0.775, -0.73, -0.685, -0.64, -0.595, -0.55, -0.505, -0.45999999999999996, -0.41500000000000004, -0.37, -0.32500000000000007, -0.28, -0.235, -0.19000000000000006, -0.14500000000000002, -0.10000000000000009, -0.05500000000000005, -0.010000000000000009, 0.03499999999999992, 0.08000000000000007, 0.125, 0.16999999999999993, 0.21499999999999986, 0.26, 0.30499999999999994, 0.34999999999999987, 0.395, 0.43999999999999995, 0.4849999999999999, 0.53, 0.575, 0.6199999999999999, 0.665, 0.71, 0.7549999999999999, 0.7999999999999998, 0.845, 0.8899999999999999, 0.9349999999999998, 0.98, 1.025, 1.0699999999999998, 1.1149999999999998, 1.1600000000000001, 1.205, 1.25, 1.295, 1.3399999999999999, 1.3849999999999998, 1.4299999999999997, 1.475, 1.52, 1.565, 1.6099999999999999, 1.6549999999999998, 1.6999999999999997, 1.745, 1.79, 1.835, 1.88, 1.9249999999999998, 1.9699999999999998, 2.0149999999999997, 2.06, 2.105, 2.15, 2.195, 2.2399999999999998, 2.2849999999999997, 2.33, 2.375, 2.42, 2.465, 2.51, 2.5549999999999997, 2.5999999999999996, 2.645, 2.69, 2.735, 2.78, 2.8249999999999997, 2.8699999999999997, 2.915, 2.96, 3.005, 3.05, 3.0949999999999998, 3.1399999999999997, 3.1849999999999996, 3.2299999999999995, 3.2749999999999995, 3.3200000000000003, 3.365, 3.41, 3.455, 3.5 ], [ -1, -0.955, -0.91, -0.865, -0.8200000000000001, -0.775, -0.73, -0.685, -0.64, -0.595, -0.55, -0.505, -0.45999999999999996, -0.41500000000000004, -0.37, -0.32500000000000007, -0.28, -0.235, -0.19000000000000006, -0.14500000000000002, -0.10000000000000009, -0.05500000000000005, -0.010000000000000009, 0.03499999999999992, 0.08000000000000007, 0.125, 0.16999999999999993, 0.21499999999999986, 0.26, 0.30499999999999994, 0.34999999999999987, 0.395, 0.43999999999999995, 0.4849999999999999, 0.53, 0.575, 0.6199999999999999, 0.665, 0.71, 0.7549999999999999, 0.7999999999999998, 0.845, 0.8899999999999999, 0.9349999999999998, 0.98, 1.025, 1.0699999999999998, 1.1149999999999998, 1.1600000000000001, 1.205, 1.25, 1.295, 1.3399999999999999, 1.3849999999999998, 1.4299999999999997, 1.475, 1.52, 1.565, 1.6099999999999999, 1.6549999999999998, 1.6999999999999997, 1.745, 1.79, 1.835, 1.88, 1.9249999999999998, 1.9699999999999998, 2.0149999999999997, 2.06, 2.105, 2.15, 2.195, 2.2399999999999998, 2.2849999999999997, 2.33, 2.375, 2.42, 2.465, 2.51, 2.5549999999999997, 2.5999999999999996, 2.645, 2.69, 2.735, 2.78, 2.8249999999999997, 2.8699999999999997, 2.915, 2.96, 3.005, 3.05, 3.0949999999999998, 3.1399999999999997, 3.1849999999999996, 3.2299999999999995, 3.2749999999999995, 3.3200000000000003, 3.365, 3.41, 3.455, 3.5 ], [ -1, -0.955, -0.91, -0.865, -0.8200000000000001, -0.775, -0.73, -0.685, -0.64, -0.595, -0.55, -0.505, -0.45999999999999996, -0.41500000000000004, -0.37, -0.32500000000000007, -0.28, -0.235, -0.19000000000000006, -0.14500000000000002, -0.10000000000000009, -0.05500000000000005, -0.010000000000000009, 0.03499999999999992, 0.08000000000000007, 0.125, 0.16999999999999993, 0.21499999999999986, 0.26, 0.30499999999999994, 0.34999999999999987, 0.395, 0.43999999999999995, 0.4849999999999999, 0.53, 0.575, 0.6199999999999999, 0.665, 0.71, 0.7549999999999999, 0.7999999999999998, 0.845, 0.8899999999999999, 0.9349999999999998, 0.98, 1.025, 1.0699999999999998, 1.1149999999999998, 1.1600000000000001, 1.205, 1.25, 1.295, 1.3399999999999999, 1.3849999999999998, 1.4299999999999997, 1.475, 1.52, 1.565, 1.6099999999999999, 1.6549999999999998, 1.6999999999999997, 1.745, 1.79, 1.835, 1.88, 1.9249999999999998, 1.9699999999999998, 2.0149999999999997, 2.06, 2.105, 2.15, 2.195, 2.2399999999999998, 2.2849999999999997, 2.33, 2.375, 2.42, 2.465, 2.51, 2.5549999999999997, 2.5999999999999996, 2.645, 2.69, 2.735, 2.78, 2.8249999999999997, 2.8699999999999997, 2.915, 2.96, 3.005, 3.05, 3.0949999999999998, 3.1399999999999997, 3.1849999999999996, 3.2299999999999995, 3.2749999999999995, 3.3200000000000003, 3.365, 3.41, 3.455, 3.5 ], [ -1, -0.955, -0.91, -0.865, -0.8200000000000001, -0.775, -0.73, -0.685, -0.64, -0.595, -0.55, -0.505, -0.45999999999999996, -0.41500000000000004, -0.37, -0.32500000000000007, -0.28, -0.235, -0.19000000000000006, -0.14500000000000002, -0.10000000000000009, -0.05500000000000005, -0.010000000000000009, 0.03499999999999992, 0.08000000000000007, 0.125, 0.16999999999999993, 0.21499999999999986, 0.26, 0.30499999999999994, 0.34999999999999987, 0.395, 0.43999999999999995, 0.4849999999999999, 0.53, 0.575, 0.6199999999999999, 0.665, 0.71, 0.7549999999999999, 0.7999999999999998, 0.845, 0.8899999999999999, 0.9349999999999998, 0.98, 1.025, 1.0699999999999998, 1.1149999999999998, 1.1600000000000001, 1.205, 1.25, 1.295, 1.3399999999999999, 1.3849999999999998, 1.4299999999999997, 1.475, 1.52, 1.565, 1.6099999999999999, 1.6549999999999998, 1.6999999999999997, 1.745, 1.79, 1.835, 1.88, 1.9249999999999998, 1.9699999999999998, 2.0149999999999997, 2.06, 2.105, 2.15, 2.195, 2.2399999999999998, 2.2849999999999997, 2.33, 2.375, 2.42, 2.465, 2.51, 2.5549999999999997, 2.5999999999999996, 2.645, 2.69, 2.735, 2.78, 2.8249999999999997, 2.8699999999999997, 2.915, 2.96, 3.005, 3.05, 3.0949999999999998, 3.1399999999999997, 3.1849999999999996, 3.2299999999999995, 3.2749999999999995, 3.3200000000000003, 3.365, 3.41, 3.455, 3.5 ], [ -1, -0.955, -0.91, -0.865, -0.8200000000000001, -0.775, -0.73, -0.685, -0.64, -0.595, -0.55, -0.505, -0.45999999999999996, -0.41500000000000004, -0.37, -0.32500000000000007, -0.28, -0.235, -0.19000000000000006, -0.14500000000000002, -0.10000000000000009, -0.05500000000000005, -0.010000000000000009, 0.03499999999999992, 0.08000000000000007, 0.125, 0.16999999999999993, 0.21499999999999986, 0.26, 0.30499999999999994, 0.34999999999999987, 0.395, 0.43999999999999995, 0.4849999999999999, 0.53, 0.575, 0.6199999999999999, 0.665, 0.71, 0.7549999999999999, 0.7999999999999998, 0.845, 0.8899999999999999, 0.9349999999999998, 0.98, 1.025, 1.0699999999999998, 1.1149999999999998, 1.1600000000000001, 1.205, 1.25, 1.295, 1.3399999999999999, 1.3849999999999998, 1.4299999999999997, 1.475, 1.52, 1.565, 1.6099999999999999, 1.6549999999999998, 1.6999999999999997, 1.745, 1.79, 1.835, 1.88, 1.9249999999999998, 1.9699999999999998, 2.0149999999999997, 2.06, 2.105, 2.15, 2.195, 2.2399999999999998, 2.2849999999999997, 2.33, 2.375, 2.42, 2.465, 2.51, 2.5549999999999997, 2.5999999999999996, 2.645, 2.69, 2.735, 2.78, 2.8249999999999997, 2.8699999999999997, 2.915, 2.96, 3.005, 3.05, 3.0949999999999998, 3.1399999999999997, 3.1849999999999996, 3.2299999999999995, 3.2749999999999995, 3.3200000000000003, 3.365, 3.41, 3.455, 3.5 ], [ -1, -0.955, -0.91, -0.865, -0.8200000000000001, -0.775, -0.73, -0.685, -0.64, -0.595, -0.55, -0.505, -0.45999999999999996, -0.41500000000000004, -0.37, -0.32500000000000007, -0.28, -0.235, -0.19000000000000006, -0.14500000000000002, -0.10000000000000009, -0.05500000000000005, -0.010000000000000009, 0.03499999999999992, 0.08000000000000007, 0.125, 0.16999999999999993, 0.21499999999999986, 0.26, 0.30499999999999994, 0.34999999999999987, 0.395, 0.43999999999999995, 0.4849999999999999, 0.53, 0.575, 0.6199999999999999, 0.665, 0.71, 0.7549999999999999, 0.7999999999999998, 0.845, 0.8899999999999999, 0.9349999999999998, 0.98, 1.025, 1.0699999999999998, 1.1149999999999998, 1.1600000000000001, 1.205, 1.25, 1.295, 1.3399999999999999, 1.3849999999999998, 1.4299999999999997, 1.475, 1.52, 1.565, 1.6099999999999999, 1.6549999999999998, 1.6999999999999997, 1.745, 1.79, 1.835, 1.88, 1.9249999999999998, 1.9699999999999998, 2.0149999999999997, 2.06, 2.105, 2.15, 2.195, 2.2399999999999998, 2.2849999999999997, 2.33, 2.375, 2.42, 2.465, 2.51, 2.5549999999999997, 2.5999999999999996, 2.645, 2.69, 2.735, 2.78, 2.8249999999999997, 2.8699999999999997, 2.915, 2.96, 3.005, 3.05, 3.0949999999999998, 3.1399999999999997, 3.1849999999999996, 3.2299999999999995, 3.2749999999999995, 3.3200000000000003, 3.365, 3.41, 3.455, 3.5 ], [ -1, -0.955, -0.91, -0.865, -0.8200000000000001, -0.775, -0.73, -0.685, -0.64, -0.595, -0.55, -0.505, -0.45999999999999996, -0.41500000000000004, -0.37, -0.32500000000000007, -0.28, -0.235, -0.19000000000000006, -0.14500000000000002, -0.10000000000000009, -0.05500000000000005, -0.010000000000000009, 0.03499999999999992, 0.08000000000000007, 0.125, 0.16999999999999993, 0.21499999999999986, 0.26, 0.30499999999999994, 0.34999999999999987, 0.395, 0.43999999999999995, 0.4849999999999999, 0.53, 0.575, 0.6199999999999999, 0.665, 0.71, 0.7549999999999999, 0.7999999999999998, 0.845, 0.8899999999999999, 0.9349999999999998, 0.98, 1.025, 1.0699999999999998, 1.1149999999999998, 1.1600000000000001, 1.205, 1.25, 1.295, 1.3399999999999999, 1.3849999999999998, 1.4299999999999997, 1.475, 1.52, 1.565, 1.6099999999999999, 1.6549999999999998, 1.6999999999999997, 1.745, 1.79, 1.835, 1.88, 1.9249999999999998, 1.9699999999999998, 2.0149999999999997, 2.06, 2.105, 2.15, 2.195, 2.2399999999999998, 2.2849999999999997, 2.33, 2.375, 2.42, 2.465, 2.51, 2.5549999999999997, 2.5999999999999996, 2.645, 2.69, 2.735, 2.78, 2.8249999999999997, 2.8699999999999997, 2.915, 2.96, 3.005, 3.05, 3.0949999999999998, 3.1399999999999997, 3.1849999999999996, 3.2299999999999995, 3.2749999999999995, 3.3200000000000003, 3.365, 3.41, 3.455, 3.5 ], [ -1, -0.955, -0.91, -0.865, -0.8200000000000001, -0.775, -0.73, -0.685, -0.64, -0.595, -0.55, -0.505, -0.45999999999999996, -0.41500000000000004, -0.37, -0.32500000000000007, -0.28, -0.235, -0.19000000000000006, -0.14500000000000002, -0.10000000000000009, -0.05500000000000005, -0.010000000000000009, 0.03499999999999992, 0.08000000000000007, 0.125, 0.16999999999999993, 0.21499999999999986, 0.26, 0.30499999999999994, 0.34999999999999987, 0.395, 0.43999999999999995, 0.4849999999999999, 0.53, 0.575, 0.6199999999999999, 0.665, 0.71, 0.7549999999999999, 0.7999999999999998, 0.845, 0.8899999999999999, 0.9349999999999998, 0.98, 1.025, 1.0699999999999998, 1.1149999999999998, 1.1600000000000001, 1.205, 1.25, 1.295, 1.3399999999999999, 1.3849999999999998, 1.4299999999999997, 1.475, 1.52, 1.565, 1.6099999999999999, 1.6549999999999998, 1.6999999999999997, 1.745, 1.79, 1.835, 1.88, 1.9249999999999998, 1.9699999999999998, 2.0149999999999997, 2.06, 2.105, 2.15, 2.195, 2.2399999999999998, 2.2849999999999997, 2.33, 2.375, 2.42, 2.465, 2.51, 2.5549999999999997, 2.5999999999999996, 2.645, 2.69, 2.735, 2.78, 2.8249999999999997, 2.8699999999999997, 2.915, 2.96, 3.005, 3.05, 3.0949999999999998, 3.1399999999999997, 3.1849999999999996, 3.2299999999999995, 3.2749999999999995, 3.3200000000000003, 3.365, 3.41, 3.455, 3.5 ], [ -1, -0.955, -0.91, -0.865, -0.8200000000000001, -0.775, -0.73, -0.685, -0.64, -0.595, -0.55, -0.505, -0.45999999999999996, -0.41500000000000004, -0.37, -0.32500000000000007, -0.28, -0.235, -0.19000000000000006, -0.14500000000000002, -0.10000000000000009, -0.05500000000000005, -0.010000000000000009, 0.03499999999999992, 0.08000000000000007, 0.125, 0.16999999999999993, 0.21499999999999986, 0.26, 0.30499999999999994, 0.34999999999999987, 0.395, 0.43999999999999995, 0.4849999999999999, 0.53, 0.575, 0.6199999999999999, 0.665, 0.71, 0.7549999999999999, 0.7999999999999998, 0.845, 0.8899999999999999, 0.9349999999999998, 0.98, 1.025, 1.0699999999999998, 1.1149999999999998, 1.1600000000000001, 1.205, 1.25, 1.295, 1.3399999999999999, 1.3849999999999998, 1.4299999999999997, 1.475, 1.52, 1.565, 1.6099999999999999, 1.6549999999999998, 1.6999999999999997, 1.745, 1.79, 1.835, 1.88, 1.9249999999999998, 1.9699999999999998, 2.0149999999999997, 2.06, 2.105, 2.15, 2.195, 2.2399999999999998, 2.2849999999999997, 2.33, 2.375, 2.42, 2.465, 2.51, 2.5549999999999997, 2.5999999999999996, 2.645, 2.69, 2.735, 2.78, 2.8249999999999997, 2.8699999999999997, 2.915, 2.96, 3.005, 3.05, 3.0949999999999998, 3.1399999999999997, 3.1849999999999996, 3.2299999999999995, 3.2749999999999995, 3.3200000000000003, 3.365, 3.41, 3.455, 3.5 ], [ -1, -0.955, -0.91, -0.865, -0.8200000000000001, -0.775, -0.73, -0.685, -0.64, -0.595, -0.55, -0.505, -0.45999999999999996, -0.41500000000000004, -0.37, -0.32500000000000007, -0.28, -0.235, -0.19000000000000006, -0.14500000000000002, -0.10000000000000009, -0.05500000000000005, -0.010000000000000009, 0.03499999999999992, 0.08000000000000007, 0.125, 0.16999999999999993, 0.21499999999999986, 0.26, 0.30499999999999994, 0.34999999999999987, 0.395, 0.43999999999999995, 0.4849999999999999, 0.53, 0.575, 0.6199999999999999, 0.665, 0.71, 0.7549999999999999, 0.7999999999999998, 0.845, 0.8899999999999999, 0.9349999999999998, 0.98, 1.025, 1.0699999999999998, 1.1149999999999998, 1.1600000000000001, 1.205, 1.25, 1.295, 1.3399999999999999, 1.3849999999999998, 1.4299999999999997, 1.475, 1.52, 1.565, 1.6099999999999999, 1.6549999999999998, 1.6999999999999997, 1.745, 1.79, 1.835, 1.88, 1.9249999999999998, 1.9699999999999998, 2.0149999999999997, 2.06, 2.105, 2.15, 2.195, 2.2399999999999998, 2.2849999999999997, 2.33, 2.375, 2.42, 2.465, 2.51, 2.5549999999999997, 2.5999999999999996, 2.645, 2.69, 2.735, 2.78, 2.8249999999999997, 2.8699999999999997, 2.915, 2.96, 3.005, 3.05, 3.0949999999999998, 3.1399999999999997, 3.1849999999999996, 3.2299999999999995, 3.2749999999999995, 3.3200000000000003, 3.365, 3.41, 3.455, 3.5 ], [ -1, -0.955, -0.91, -0.865, -0.8200000000000001, -0.775, -0.73, -0.685, -0.64, -0.595, -0.55, -0.505, -0.45999999999999996, -0.41500000000000004, -0.37, -0.32500000000000007, -0.28, -0.235, -0.19000000000000006, -0.14500000000000002, -0.10000000000000009, -0.05500000000000005, -0.010000000000000009, 0.03499999999999992, 0.08000000000000007, 0.125, 0.16999999999999993, 0.21499999999999986, 0.26, 0.30499999999999994, 0.34999999999999987, 0.395, 0.43999999999999995, 0.4849999999999999, 0.53, 0.575, 0.6199999999999999, 0.665, 0.71, 0.7549999999999999, 0.7999999999999998, 0.845, 0.8899999999999999, 0.9349999999999998, 0.98, 1.025, 1.0699999999999998, 1.1149999999999998, 1.1600000000000001, 1.205, 1.25, 1.295, 1.3399999999999999, 1.3849999999999998, 1.4299999999999997, 1.475, 1.52, 1.565, 1.6099999999999999, 1.6549999999999998, 1.6999999999999997, 1.745, 1.79, 1.835, 1.88, 1.9249999999999998, 1.9699999999999998, 2.0149999999999997, 2.06, 2.105, 2.15, 2.195, 2.2399999999999998, 2.2849999999999997, 2.33, 2.375, 2.42, 2.465, 2.51, 2.5549999999999997, 2.5999999999999996, 2.645, 2.69, 2.735, 2.78, 2.8249999999999997, 2.8699999999999997, 2.915, 2.96, 3.005, 3.05, 3.0949999999999998, 3.1399999999999997, 3.1849999999999996, 3.2299999999999995, 3.2749999999999995, 3.3200000000000003, 3.365, 3.41, 3.455, 3.5 ], [ -1, -0.955, -0.91, -0.865, -0.8200000000000001, -0.775, -0.73, -0.685, -0.64, -0.595, -0.55, -0.505, -0.45999999999999996, -0.41500000000000004, -0.37, -0.32500000000000007, -0.28, -0.235, -0.19000000000000006, -0.14500000000000002, -0.10000000000000009, -0.05500000000000005, -0.010000000000000009, 0.03499999999999992, 0.08000000000000007, 0.125, 0.16999999999999993, 0.21499999999999986, 0.26, 0.30499999999999994, 0.34999999999999987, 0.395, 0.43999999999999995, 0.4849999999999999, 0.53, 0.575, 0.6199999999999999, 0.665, 0.71, 0.7549999999999999, 0.7999999999999998, 0.845, 0.8899999999999999, 0.9349999999999998, 0.98, 1.025, 1.0699999999999998, 1.1149999999999998, 1.1600000000000001, 1.205, 1.25, 1.295, 1.3399999999999999, 1.3849999999999998, 1.4299999999999997, 1.475, 1.52, 1.565, 1.6099999999999999, 1.6549999999999998, 1.6999999999999997, 1.745, 1.79, 1.835, 1.88, 1.9249999999999998, 1.9699999999999998, 2.0149999999999997, 2.06, 2.105, 2.15, 2.195, 2.2399999999999998, 2.2849999999999997, 2.33, 2.375, 2.42, 2.465, 2.51, 2.5549999999999997, 2.5999999999999996, 2.645, 2.69, 2.735, 2.78, 2.8249999999999997, 2.8699999999999997, 2.915, 2.96, 3.005, 3.05, 3.0949999999999998, 3.1399999999999997, 3.1849999999999996, 3.2299999999999995, 3.2749999999999995, 3.3200000000000003, 3.365, 3.41, 3.455, 3.5 ], [ -1, -0.955, -0.91, -0.865, -0.8200000000000001, -0.775, -0.73, -0.685, -0.64, -0.595, -0.55, -0.505, -0.45999999999999996, -0.41500000000000004, -0.37, -0.32500000000000007, -0.28, -0.235, -0.19000000000000006, -0.14500000000000002, -0.10000000000000009, -0.05500000000000005, -0.010000000000000009, 0.03499999999999992, 0.08000000000000007, 0.125, 0.16999999999999993, 0.21499999999999986, 0.26, 0.30499999999999994, 0.34999999999999987, 0.395, 0.43999999999999995, 0.4849999999999999, 0.53, 0.575, 0.6199999999999999, 0.665, 0.71, 0.7549999999999999, 0.7999999999999998, 0.845, 0.8899999999999999, 0.9349999999999998, 0.98, 1.025, 1.0699999999999998, 1.1149999999999998, 1.1600000000000001, 1.205, 1.25, 1.295, 1.3399999999999999, 1.3849999999999998, 1.4299999999999997, 1.475, 1.52, 1.565, 1.6099999999999999, 1.6549999999999998, 1.6999999999999997, 1.745, 1.79, 1.835, 1.88, 1.9249999999999998, 1.9699999999999998, 2.0149999999999997, 2.06, 2.105, 2.15, 2.195, 2.2399999999999998, 2.2849999999999997, 2.33, 2.375, 2.42, 2.465, 2.51, 2.5549999999999997, 2.5999999999999996, 2.645, 2.69, 2.735, 2.78, 2.8249999999999997, 2.8699999999999997, 2.915, 2.96, 3.005, 3.05, 3.0949999999999998, 3.1399999999999997, 3.1849999999999996, 3.2299999999999995, 3.2749999999999995, 3.3200000000000003, 3.365, 3.41, 3.455, 3.5 ], [ -1, -0.955, -0.91, -0.865, -0.8200000000000001, -0.775, -0.73, -0.685, -0.64, -0.595, -0.55, -0.505, -0.45999999999999996, -0.41500000000000004, -0.37, -0.32500000000000007, -0.28, -0.235, -0.19000000000000006, -0.14500000000000002, -0.10000000000000009, -0.05500000000000005, -0.010000000000000009, 0.03499999999999992, 0.08000000000000007, 0.125, 0.16999999999999993, 0.21499999999999986, 0.26, 0.30499999999999994, 0.34999999999999987, 0.395, 0.43999999999999995, 0.4849999999999999, 0.53, 0.575, 0.6199999999999999, 0.665, 0.71, 0.7549999999999999, 0.7999999999999998, 0.845, 0.8899999999999999, 0.9349999999999998, 0.98, 1.025, 1.0699999999999998, 1.1149999999999998, 1.1600000000000001, 1.205, 1.25, 1.295, 1.3399999999999999, 1.3849999999999998, 1.4299999999999997, 1.475, 1.52, 1.565, 1.6099999999999999, 1.6549999999999998, 1.6999999999999997, 1.745, 1.79, 1.835, 1.88, 1.9249999999999998, 1.9699999999999998, 2.0149999999999997, 2.06, 2.105, 2.15, 2.195, 2.2399999999999998, 2.2849999999999997, 2.33, 2.375, 2.42, 2.465, 2.51, 2.5549999999999997, 2.5999999999999996, 2.645, 2.69, 2.735, 2.78, 2.8249999999999997, 2.8699999999999997, 2.915, 2.96, 3.005, 3.05, 3.0949999999999998, 3.1399999999999997, 3.1849999999999996, 3.2299999999999995, 3.2749999999999995, 3.3200000000000003, 3.365, 3.41, 3.455, 3.5 ], [ -1, -0.955, -0.91, -0.865, -0.8200000000000001, -0.775, -0.73, -0.685, -0.64, -0.595, -0.55, -0.505, -0.45999999999999996, -0.41500000000000004, -0.37, -0.32500000000000007, -0.28, -0.235, -0.19000000000000006, -0.14500000000000002, -0.10000000000000009, -0.05500000000000005, -0.010000000000000009, 0.03499999999999992, 0.08000000000000007, 0.125, 0.16999999999999993, 0.21499999999999986, 0.26, 0.30499999999999994, 0.34999999999999987, 0.395, 0.43999999999999995, 0.4849999999999999, 0.53, 0.575, 0.6199999999999999, 0.665, 0.71, 0.7549999999999999, 0.7999999999999998, 0.845, 0.8899999999999999, 0.9349999999999998, 0.98, 1.025, 1.0699999999999998, 1.1149999999999998, 1.1600000000000001, 1.205, 1.25, 1.295, 1.3399999999999999, 1.3849999999999998, 1.4299999999999997, 1.475, 1.52, 1.565, 1.6099999999999999, 1.6549999999999998, 1.6999999999999997, 1.745, 1.79, 1.835, 1.88, 1.9249999999999998, 1.9699999999999998, 2.0149999999999997, 2.06, 2.105, 2.15, 2.195, 2.2399999999999998, 2.2849999999999997, 2.33, 2.375, 2.42, 2.465, 2.51, 2.5549999999999997, 2.5999999999999996, 2.645, 2.69, 2.735, 2.78, 2.8249999999999997, 2.8699999999999997, 2.915, 2.96, 3.005, 3.05, 3.0949999999999998, 3.1399999999999997, 3.1849999999999996, 3.2299999999999995, 3.2749999999999995, 3.3200000000000003, 3.365, 3.41, 3.455, 3.5 ] ], "y": [ [ -1, -1, -1, -1, -1, -1, -1, -1, -1, -1, -1, -1, -1, -1, -1, -1, -1, -1, -1, -1, -1, -1, -1, -1, -1, -1, -1, -1, -1, -1, -1, -1, -1, -1, -1, -1, -1, -1, -1, -1, -1, -1, -1, -1, -1, -1, -1, -1, -1, -1, -1, -1, -1, -1, -1, -1, -1, -1, -1, -1, -1, -1, -1, -1, -1, -1, -1, -1, -1, -1, -1, -1, -1, -1, -1, -1, -1, -1, -1, -1, -1, -1, -1, -1, -1, -1, -1, -1, -1, -1, -1, -1, -1, -1, -1, -1, -1, -1, -1, -1, -1 ], [ -0.965, -0.965, -0.965, -0.965, -0.965, -0.965, -0.965, -0.965, -0.965, -0.965, -0.965, -0.965, -0.965, -0.965, -0.965, -0.965, -0.965, -0.965, -0.965, -0.965, -0.965, -0.965, -0.965, -0.965, -0.965, -0.965, -0.965, -0.965, -0.965, -0.965, -0.965, -0.965, -0.965, -0.965, -0.965, -0.965, -0.965, -0.965, -0.965, -0.965, -0.965, -0.965, -0.965, -0.965, -0.965, -0.965, -0.965, -0.965, -0.965, -0.965, -0.965, -0.965, -0.965, -0.965, -0.965, -0.965, -0.965, -0.965, -0.965, -0.965, -0.965, -0.965, -0.965, -0.965, -0.965, -0.965, -0.965, -0.965, -0.965, -0.965, -0.965, -0.965, -0.965, -0.965, -0.965, -0.965, -0.965, -0.965, -0.965, -0.965, -0.965, -0.965, -0.965, -0.965, -0.965, -0.965, -0.965, -0.965, -0.965, -0.965, -0.965, -0.965, -0.965, -0.965, -0.965, -0.965, -0.965, -0.965, -0.965, -0.965, -0.965 ], [ -0.9299999999999999, -0.9299999999999999, -0.9299999999999999, -0.9299999999999999, -0.9299999999999999, -0.9299999999999999, -0.9299999999999999, -0.9299999999999999, -0.9299999999999999, -0.9299999999999999, -0.9299999999999999, -0.9299999999999999, -0.9299999999999999, -0.9299999999999999, -0.9299999999999999, -0.9299999999999999, -0.9299999999999999, -0.9299999999999999, -0.9299999999999999, -0.9299999999999999, -0.9299999999999999, -0.9299999999999999, -0.9299999999999999, -0.9299999999999999, -0.9299999999999999, -0.9299999999999999, -0.9299999999999999, -0.9299999999999999, -0.9299999999999999, -0.9299999999999999, -0.9299999999999999, -0.9299999999999999, -0.9299999999999999, -0.9299999999999999, -0.9299999999999999, -0.9299999999999999, -0.9299999999999999, -0.9299999999999999, -0.9299999999999999, -0.9299999999999999, -0.9299999999999999, -0.9299999999999999, -0.9299999999999999, -0.9299999999999999, -0.9299999999999999, -0.9299999999999999, -0.9299999999999999, -0.9299999999999999, -0.9299999999999999, -0.9299999999999999, -0.9299999999999999, -0.9299999999999999, -0.9299999999999999, -0.9299999999999999, -0.9299999999999999, -0.9299999999999999, -0.9299999999999999, -0.9299999999999999, -0.9299999999999999, -0.9299999999999999, -0.9299999999999999, -0.9299999999999999, -0.9299999999999999, -0.9299999999999999, -0.9299999999999999, -0.9299999999999999, -0.9299999999999999, -0.9299999999999999, -0.9299999999999999, -0.9299999999999999, -0.9299999999999999, -0.9299999999999999, -0.9299999999999999, -0.9299999999999999, -0.9299999999999999, -0.9299999999999999, -0.9299999999999999, -0.9299999999999999, -0.9299999999999999, -0.9299999999999999, -0.9299999999999999, -0.9299999999999999, -0.9299999999999999, -0.9299999999999999, -0.9299999999999999, -0.9299999999999999, -0.9299999999999999, -0.9299999999999999, -0.9299999999999999, -0.9299999999999999, -0.9299999999999999, -0.9299999999999999, -0.9299999999999999, -0.9299999999999999, -0.9299999999999999, -0.9299999999999999, -0.9299999999999999, -0.9299999999999999, -0.9299999999999999, -0.9299999999999999, -0.9299999999999999 ], [ -0.895, -0.895, -0.895, -0.895, -0.895, -0.895, -0.895, -0.895, -0.895, -0.895, -0.895, -0.895, -0.895, -0.895, -0.895, -0.895, -0.895, -0.895, -0.895, -0.895, -0.895, -0.895, -0.895, -0.895, -0.895, -0.895, -0.895, -0.895, -0.895, -0.895, -0.895, -0.895, -0.895, -0.895, -0.895, -0.895, -0.895, -0.895, -0.895, -0.895, -0.895, -0.895, -0.895, -0.895, -0.895, -0.895, -0.895, -0.895, -0.895, -0.895, -0.895, -0.895, -0.895, -0.895, -0.895, -0.895, -0.895, -0.895, -0.895, -0.895, -0.895, -0.895, -0.895, -0.895, -0.895, -0.895, -0.895, -0.895, -0.895, -0.895, -0.895, -0.895, -0.895, -0.895, -0.895, -0.895, -0.895, -0.895, -0.895, -0.895, -0.895, -0.895, -0.895, -0.895, -0.895, -0.895, -0.895, -0.895, -0.895, -0.895, -0.895, -0.895, -0.895, -0.895, -0.895, -0.895, -0.895, -0.895, -0.895, -0.895, -0.895 ], [ -0.86, -0.86, -0.86, -0.86, -0.86, -0.86, -0.86, -0.86, -0.86, -0.86, -0.86, -0.86, -0.86, -0.86, -0.86, -0.86, -0.86, -0.86, -0.86, -0.86, -0.86, -0.86, -0.86, -0.86, -0.86, -0.86, -0.86, -0.86, -0.86, -0.86, -0.86, -0.86, -0.86, -0.86, -0.86, -0.86, -0.86, -0.86, -0.86, -0.86, -0.86, -0.86, -0.86, -0.86, -0.86, -0.86, -0.86, -0.86, -0.86, -0.86, -0.86, -0.86, -0.86, -0.86, -0.86, -0.86, -0.86, -0.86, -0.86, -0.86, -0.86, -0.86, -0.86, -0.86, -0.86, -0.86, -0.86, -0.86, -0.86, -0.86, -0.86, -0.86, -0.86, -0.86, -0.86, -0.86, -0.86, -0.86, -0.86, -0.86, -0.86, -0.86, -0.86, -0.86, -0.86, -0.86, -0.86, -0.86, -0.86, -0.86, -0.86, -0.86, -0.86, -0.86, -0.86, -0.86, -0.86, -0.86, -0.86, -0.86, -0.86 ], [ -0.825, -0.825, -0.825, -0.825, -0.825, -0.825, -0.825, -0.825, -0.825, -0.825, -0.825, -0.825, -0.825, -0.825, -0.825, -0.825, -0.825, -0.825, -0.825, -0.825, -0.825, -0.825, -0.825, -0.825, -0.825, -0.825, -0.825, -0.825, -0.825, -0.825, -0.825, -0.825, -0.825, -0.825, -0.825, -0.825, -0.825, -0.825, -0.825, -0.825, -0.825, -0.825, -0.825, -0.825, -0.825, -0.825, -0.825, -0.825, -0.825, -0.825, -0.825, -0.825, -0.825, -0.825, -0.825, -0.825, -0.825, -0.825, -0.825, -0.825, -0.825, -0.825, -0.825, -0.825, -0.825, -0.825, -0.825, -0.825, -0.825, -0.825, -0.825, -0.825, -0.825, -0.825, -0.825, -0.825, -0.825, -0.825, -0.825, -0.825, -0.825, -0.825, -0.825, -0.825, -0.825, -0.825, -0.825, -0.825, -0.825, -0.825, -0.825, -0.825, -0.825, -0.825, -0.825, -0.825, -0.825, -0.825, -0.825, -0.825, -0.825 ], [ -0.79, -0.79, -0.79, -0.79, -0.79, -0.79, -0.79, -0.79, -0.79, -0.79, -0.79, -0.79, -0.79, -0.79, -0.79, -0.79, -0.79, -0.79, -0.79, -0.79, -0.79, -0.79, -0.79, -0.79, -0.79, -0.79, -0.79, -0.79, -0.79, -0.79, -0.79, -0.79, -0.79, -0.79, -0.79, -0.79, -0.79, -0.79, -0.79, -0.79, -0.79, -0.79, -0.79, -0.79, -0.79, -0.79, -0.79, -0.79, -0.79, -0.79, -0.79, -0.79, -0.79, -0.79, -0.79, -0.79, -0.79, -0.79, -0.79, -0.79, -0.79, -0.79, -0.79, -0.79, -0.79, -0.79, -0.79, -0.79, -0.79, -0.79, -0.79, -0.79, -0.79, -0.79, -0.79, -0.79, -0.79, -0.79, -0.79, -0.79, -0.79, -0.79, -0.79, -0.79, -0.79, -0.79, -0.79, -0.79, -0.79, -0.79, -0.79, -0.79, -0.79, -0.79, -0.79, -0.79, -0.79, -0.79, -0.79, -0.79, -0.79 ], [ -0.755, -0.755, -0.755, -0.755, -0.755, -0.755, -0.755, -0.755, -0.755, -0.755, -0.755, -0.755, -0.755, -0.755, -0.755, -0.755, -0.755, -0.755, -0.755, -0.755, -0.755, -0.755, -0.755, -0.755, -0.755, -0.755, -0.755, -0.755, -0.755, -0.755, -0.755, -0.755, -0.755, -0.755, -0.755, -0.755, -0.755, -0.755, -0.755, -0.755, -0.755, -0.755, -0.755, -0.755, -0.755, -0.755, -0.755, -0.755, -0.755, -0.755, -0.755, -0.755, -0.755, -0.755, -0.755, -0.755, -0.755, -0.755, -0.755, -0.755, -0.755, -0.755, -0.755, -0.755, -0.755, -0.755, -0.755, -0.755, -0.755, -0.755, -0.755, -0.755, -0.755, -0.755, -0.755, -0.755, -0.755, -0.755, -0.755, -0.755, -0.755, -0.755, -0.755, -0.755, -0.755, -0.755, -0.755, -0.755, -0.755, -0.755, -0.755, -0.755, -0.755, -0.755, -0.755, -0.755, -0.755, -0.755, -0.755, -0.755, -0.755 ], [ -0.72, -0.72, -0.72, -0.72, -0.72, -0.72, -0.72, -0.72, -0.72, -0.72, -0.72, -0.72, -0.72, -0.72, -0.72, -0.72, -0.72, -0.72, -0.72, -0.72, -0.72, -0.72, -0.72, -0.72, -0.72, -0.72, -0.72, -0.72, -0.72, -0.72, -0.72, -0.72, -0.72, -0.72, -0.72, -0.72, -0.72, -0.72, -0.72, -0.72, -0.72, -0.72, -0.72, -0.72, -0.72, -0.72, -0.72, -0.72, -0.72, -0.72, -0.72, -0.72, -0.72, -0.72, -0.72, -0.72, -0.72, -0.72, -0.72, -0.72, -0.72, -0.72, -0.72, -0.72, -0.72, -0.72, -0.72, -0.72, -0.72, -0.72, -0.72, -0.72, -0.72, -0.72, -0.72, -0.72, -0.72, -0.72, -0.72, -0.72, -0.72, -0.72, -0.72, -0.72, -0.72, -0.72, -0.72, -0.72, -0.72, -0.72, -0.72, -0.72, -0.72, -0.72, -0.72, -0.72, -0.72, -0.72, -0.72, -0.72, -0.72 ], [ -0.6849999999999999, -0.6849999999999999, -0.6849999999999999, -0.6849999999999999, -0.6849999999999999, -0.6849999999999999, -0.6849999999999999, -0.6849999999999999, -0.6849999999999999, -0.6849999999999999, -0.6849999999999999, -0.6849999999999999, -0.6849999999999999, -0.6849999999999999, -0.6849999999999999, -0.6849999999999999, -0.6849999999999999, -0.6849999999999999, -0.6849999999999999, -0.6849999999999999, -0.6849999999999999, -0.6849999999999999, -0.6849999999999999, -0.6849999999999999, -0.6849999999999999, -0.6849999999999999, -0.6849999999999999, -0.6849999999999999, -0.6849999999999999, -0.6849999999999999, -0.6849999999999999, -0.6849999999999999, -0.6849999999999999, -0.6849999999999999, -0.6849999999999999, -0.6849999999999999, -0.6849999999999999, -0.6849999999999999, -0.6849999999999999, -0.6849999999999999, -0.6849999999999999, -0.6849999999999999, -0.6849999999999999, -0.6849999999999999, -0.6849999999999999, -0.6849999999999999, -0.6849999999999999, -0.6849999999999999, -0.6849999999999999, -0.6849999999999999, -0.6849999999999999, -0.6849999999999999, -0.6849999999999999, -0.6849999999999999, -0.6849999999999999, -0.6849999999999999, -0.6849999999999999, -0.6849999999999999, -0.6849999999999999, -0.6849999999999999, -0.6849999999999999, -0.6849999999999999, -0.6849999999999999, -0.6849999999999999, -0.6849999999999999, -0.6849999999999999, -0.6849999999999999, -0.6849999999999999, -0.6849999999999999, -0.6849999999999999, -0.6849999999999999, -0.6849999999999999, -0.6849999999999999, -0.6849999999999999, -0.6849999999999999, -0.6849999999999999, -0.6849999999999999, -0.6849999999999999, -0.6849999999999999, -0.6849999999999999, -0.6849999999999999, -0.6849999999999999, -0.6849999999999999, -0.6849999999999999, -0.6849999999999999, -0.6849999999999999, -0.6849999999999999, -0.6849999999999999, -0.6849999999999999, -0.6849999999999999, -0.6849999999999999, -0.6849999999999999, -0.6849999999999999, -0.6849999999999999, -0.6849999999999999, -0.6849999999999999, -0.6849999999999999, -0.6849999999999999, -0.6849999999999999, -0.6849999999999999, -0.6849999999999999 ], [ -0.6499999999999999, -0.6499999999999999, -0.6499999999999999, -0.6499999999999999, -0.6499999999999999, -0.6499999999999999, -0.6499999999999999, -0.6499999999999999, -0.6499999999999999, -0.6499999999999999, -0.6499999999999999, -0.6499999999999999, -0.6499999999999999, -0.6499999999999999, -0.6499999999999999, -0.6499999999999999, -0.6499999999999999, -0.6499999999999999, -0.6499999999999999, -0.6499999999999999, -0.6499999999999999, -0.6499999999999999, -0.6499999999999999, -0.6499999999999999, -0.6499999999999999, -0.6499999999999999, -0.6499999999999999, -0.6499999999999999, -0.6499999999999999, -0.6499999999999999, -0.6499999999999999, -0.6499999999999999, -0.6499999999999999, -0.6499999999999999, -0.6499999999999999, -0.6499999999999999, -0.6499999999999999, -0.6499999999999999, -0.6499999999999999, -0.6499999999999999, -0.6499999999999999, -0.6499999999999999, -0.6499999999999999, -0.6499999999999999, -0.6499999999999999, -0.6499999999999999, -0.6499999999999999, -0.6499999999999999, -0.6499999999999999, -0.6499999999999999, -0.6499999999999999, -0.6499999999999999, -0.6499999999999999, -0.6499999999999999, -0.6499999999999999, -0.6499999999999999, -0.6499999999999999, -0.6499999999999999, -0.6499999999999999, -0.6499999999999999, -0.6499999999999999, -0.6499999999999999, -0.6499999999999999, -0.6499999999999999, -0.6499999999999999, -0.6499999999999999, -0.6499999999999999, -0.6499999999999999, -0.6499999999999999, -0.6499999999999999, -0.6499999999999999, -0.6499999999999999, -0.6499999999999999, -0.6499999999999999, -0.6499999999999999, -0.6499999999999999, -0.6499999999999999, -0.6499999999999999, -0.6499999999999999, -0.6499999999999999, -0.6499999999999999, -0.6499999999999999, -0.6499999999999999, -0.6499999999999999, -0.6499999999999999, -0.6499999999999999, -0.6499999999999999, -0.6499999999999999, -0.6499999999999999, -0.6499999999999999, -0.6499999999999999, -0.6499999999999999, -0.6499999999999999, -0.6499999999999999, -0.6499999999999999, -0.6499999999999999, -0.6499999999999999, -0.6499999999999999, -0.6499999999999999, -0.6499999999999999, -0.6499999999999999 ], [ -0.615, -0.615, -0.615, -0.615, -0.615, -0.615, -0.615, -0.615, -0.615, -0.615, -0.615, -0.615, -0.615, -0.615, -0.615, -0.615, -0.615, -0.615, -0.615, -0.615, -0.615, -0.615, -0.615, -0.615, -0.615, -0.615, -0.615, -0.615, -0.615, -0.615, -0.615, -0.615, -0.615, -0.615, -0.615, -0.615, -0.615, -0.615, -0.615, -0.615, -0.615, -0.615, -0.615, -0.615, -0.615, -0.615, -0.615, -0.615, -0.615, -0.615, -0.615, -0.615, -0.615, -0.615, -0.615, -0.615, -0.615, -0.615, -0.615, -0.615, -0.615, -0.615, -0.615, -0.615, -0.615, -0.615, -0.615, -0.615, -0.615, -0.615, -0.615, -0.615, -0.615, -0.615, -0.615, -0.615, -0.615, -0.615, -0.615, -0.615, -0.615, -0.615, -0.615, -0.615, -0.615, -0.615, -0.615, -0.615, -0.615, -0.615, -0.615, -0.615, -0.615, -0.615, -0.615, -0.615, -0.615, -0.615, -0.615, -0.615, -0.615 ], [ -0.58, -0.58, -0.58, -0.58, -0.58, -0.58, -0.58, -0.58, -0.58, -0.58, -0.58, -0.58, -0.58, -0.58, -0.58, -0.58, -0.58, -0.58, -0.58, -0.58, -0.58, -0.58, -0.58, -0.58, -0.58, -0.58, -0.58, -0.58, -0.58, -0.58, -0.58, -0.58, -0.58, -0.58, -0.58, -0.58, -0.58, -0.58, -0.58, -0.58, -0.58, -0.58, -0.58, -0.58, -0.58, -0.58, -0.58, -0.58, -0.58, -0.58, -0.58, -0.58, -0.58, -0.58, -0.58, -0.58, -0.58, -0.58, -0.58, -0.58, -0.58, -0.58, -0.58, -0.58, -0.58, -0.58, -0.58, -0.58, -0.58, -0.58, -0.58, -0.58, -0.58, -0.58, -0.58, -0.58, -0.58, -0.58, -0.58, -0.58, -0.58, -0.58, -0.58, -0.58, -0.58, -0.58, -0.58, -0.58, -0.58, -0.58, -0.58, -0.58, -0.58, -0.58, -0.58, -0.58, -0.58, -0.58, -0.58, -0.58, -0.58 ], [ -0.5449999999999999, -0.5449999999999999, -0.5449999999999999, -0.5449999999999999, -0.5449999999999999, -0.5449999999999999, -0.5449999999999999, -0.5449999999999999, -0.5449999999999999, -0.5449999999999999, -0.5449999999999999, -0.5449999999999999, -0.5449999999999999, -0.5449999999999999, -0.5449999999999999, -0.5449999999999999, -0.5449999999999999, -0.5449999999999999, -0.5449999999999999, -0.5449999999999999, -0.5449999999999999, -0.5449999999999999, -0.5449999999999999, -0.5449999999999999, -0.5449999999999999, -0.5449999999999999, -0.5449999999999999, -0.5449999999999999, -0.5449999999999999, -0.5449999999999999, -0.5449999999999999, -0.5449999999999999, -0.5449999999999999, -0.5449999999999999, -0.5449999999999999, -0.5449999999999999, -0.5449999999999999, -0.5449999999999999, -0.5449999999999999, -0.5449999999999999, -0.5449999999999999, -0.5449999999999999, -0.5449999999999999, -0.5449999999999999, -0.5449999999999999, -0.5449999999999999, -0.5449999999999999, -0.5449999999999999, -0.5449999999999999, -0.5449999999999999, -0.5449999999999999, -0.5449999999999999, -0.5449999999999999, -0.5449999999999999, -0.5449999999999999, -0.5449999999999999, -0.5449999999999999, -0.5449999999999999, -0.5449999999999999, -0.5449999999999999, -0.5449999999999999, -0.5449999999999999, -0.5449999999999999, -0.5449999999999999, -0.5449999999999999, -0.5449999999999999, -0.5449999999999999, -0.5449999999999999, -0.5449999999999999, -0.5449999999999999, -0.5449999999999999, -0.5449999999999999, -0.5449999999999999, -0.5449999999999999, -0.5449999999999999, -0.5449999999999999, -0.5449999999999999, -0.5449999999999999, -0.5449999999999999, -0.5449999999999999, -0.5449999999999999, -0.5449999999999999, -0.5449999999999999, -0.5449999999999999, -0.5449999999999999, -0.5449999999999999, -0.5449999999999999, -0.5449999999999999, -0.5449999999999999, -0.5449999999999999, -0.5449999999999999, -0.5449999999999999, -0.5449999999999999, -0.5449999999999999, -0.5449999999999999, -0.5449999999999999, -0.5449999999999999, -0.5449999999999999, -0.5449999999999999, -0.5449999999999999, -0.5449999999999999 ], [ -0.51, -0.51, -0.51, -0.51, -0.51, -0.51, -0.51, -0.51, -0.51, -0.51, -0.51, -0.51, -0.51, -0.51, -0.51, -0.51, -0.51, -0.51, -0.51, -0.51, -0.51, -0.51, -0.51, -0.51, -0.51, -0.51, -0.51, -0.51, -0.51, -0.51, -0.51, -0.51, -0.51, -0.51, -0.51, -0.51, -0.51, -0.51, -0.51, -0.51, -0.51, -0.51, -0.51, -0.51, -0.51, -0.51, -0.51, -0.51, -0.51, -0.51, -0.51, -0.51, -0.51, -0.51, -0.51, -0.51, -0.51, -0.51, -0.51, -0.51, -0.51, -0.51, -0.51, -0.51, -0.51, -0.51, -0.51, -0.51, -0.51, -0.51, -0.51, -0.51, -0.51, -0.51, -0.51, -0.51, -0.51, -0.51, -0.51, -0.51, -0.51, -0.51, -0.51, -0.51, -0.51, -0.51, -0.51, -0.51, -0.51, -0.51, -0.51, -0.51, -0.51, -0.51, -0.51, -0.51, -0.51, -0.51, -0.51, -0.51, -0.51 ], [ -0.475, -0.475, -0.475, -0.475, -0.475, -0.475, -0.475, -0.475, -0.475, -0.475, -0.475, -0.475, -0.475, -0.475, -0.475, -0.475, -0.475, -0.475, -0.475, -0.475, -0.475, -0.475, -0.475, -0.475, -0.475, -0.475, -0.475, -0.475, -0.475, -0.475, -0.475, -0.475, -0.475, -0.475, -0.475, -0.475, -0.475, -0.475, -0.475, -0.475, -0.475, -0.475, -0.475, -0.475, -0.475, -0.475, -0.475, -0.475, -0.475, -0.475, -0.475, -0.475, -0.475, -0.475, -0.475, -0.475, -0.475, -0.475, -0.475, -0.475, -0.475, -0.475, -0.475, -0.475, -0.475, -0.475, -0.475, -0.475, -0.475, -0.475, -0.475, -0.475, -0.475, -0.475, -0.475, -0.475, -0.475, -0.475, -0.475, -0.475, -0.475, -0.475, -0.475, -0.475, -0.475, -0.475, -0.475, -0.475, -0.475, -0.475, -0.475, -0.475, -0.475, -0.475, -0.475, -0.475, -0.475, -0.475, -0.475, -0.475, -0.475 ], [ -0.43999999999999995, -0.43999999999999995, -0.43999999999999995, -0.43999999999999995, -0.43999999999999995, -0.43999999999999995, -0.43999999999999995, -0.43999999999999995, -0.43999999999999995, -0.43999999999999995, -0.43999999999999995, -0.43999999999999995, -0.43999999999999995, -0.43999999999999995, -0.43999999999999995, -0.43999999999999995, -0.43999999999999995, -0.43999999999999995, -0.43999999999999995, -0.43999999999999995, -0.43999999999999995, -0.43999999999999995, -0.43999999999999995, -0.43999999999999995, -0.43999999999999995, -0.43999999999999995, -0.43999999999999995, -0.43999999999999995, -0.43999999999999995, -0.43999999999999995, -0.43999999999999995, -0.43999999999999995, -0.43999999999999995, -0.43999999999999995, -0.43999999999999995, -0.43999999999999995, -0.43999999999999995, -0.43999999999999995, -0.43999999999999995, -0.43999999999999995, -0.43999999999999995, -0.43999999999999995, -0.43999999999999995, -0.43999999999999995, -0.43999999999999995, -0.43999999999999995, -0.43999999999999995, -0.43999999999999995, -0.43999999999999995, -0.43999999999999995, -0.43999999999999995, -0.43999999999999995, -0.43999999999999995, -0.43999999999999995, -0.43999999999999995, -0.43999999999999995, -0.43999999999999995, -0.43999999999999995, -0.43999999999999995, -0.43999999999999995, -0.43999999999999995, -0.43999999999999995, -0.43999999999999995, -0.43999999999999995, -0.43999999999999995, -0.43999999999999995, -0.43999999999999995, -0.43999999999999995, -0.43999999999999995, -0.43999999999999995, -0.43999999999999995, -0.43999999999999995, -0.43999999999999995, -0.43999999999999995, -0.43999999999999995, -0.43999999999999995, -0.43999999999999995, -0.43999999999999995, -0.43999999999999995, -0.43999999999999995, -0.43999999999999995, -0.43999999999999995, -0.43999999999999995, -0.43999999999999995, -0.43999999999999995, -0.43999999999999995, -0.43999999999999995, -0.43999999999999995, -0.43999999999999995, -0.43999999999999995, -0.43999999999999995, -0.43999999999999995, -0.43999999999999995, -0.43999999999999995, -0.43999999999999995, -0.43999999999999995, -0.43999999999999995, -0.43999999999999995, -0.43999999999999995, -0.43999999999999995, -0.43999999999999995 ], [ -0.4049999999999999, -0.4049999999999999, -0.4049999999999999, -0.4049999999999999, -0.4049999999999999, -0.4049999999999999, -0.4049999999999999, -0.4049999999999999, -0.4049999999999999, -0.4049999999999999, -0.4049999999999999, -0.4049999999999999, -0.4049999999999999, -0.4049999999999999, -0.4049999999999999, -0.4049999999999999, -0.4049999999999999, -0.4049999999999999, -0.4049999999999999, -0.4049999999999999, -0.4049999999999999, -0.4049999999999999, -0.4049999999999999, -0.4049999999999999, -0.4049999999999999, -0.4049999999999999, -0.4049999999999999, -0.4049999999999999, -0.4049999999999999, -0.4049999999999999, -0.4049999999999999, -0.4049999999999999, -0.4049999999999999, -0.4049999999999999, -0.4049999999999999, -0.4049999999999999, -0.4049999999999999, -0.4049999999999999, -0.4049999999999999, -0.4049999999999999, -0.4049999999999999, -0.4049999999999999, -0.4049999999999999, -0.4049999999999999, -0.4049999999999999, -0.4049999999999999, -0.4049999999999999, -0.4049999999999999, -0.4049999999999999, -0.4049999999999999, -0.4049999999999999, -0.4049999999999999, -0.4049999999999999, -0.4049999999999999, -0.4049999999999999, -0.4049999999999999, -0.4049999999999999, -0.4049999999999999, -0.4049999999999999, -0.4049999999999999, -0.4049999999999999, -0.4049999999999999, -0.4049999999999999, -0.4049999999999999, -0.4049999999999999, -0.4049999999999999, -0.4049999999999999, -0.4049999999999999, -0.4049999999999999, -0.4049999999999999, -0.4049999999999999, -0.4049999999999999, -0.4049999999999999, -0.4049999999999999, -0.4049999999999999, -0.4049999999999999, -0.4049999999999999, -0.4049999999999999, -0.4049999999999999, -0.4049999999999999, -0.4049999999999999, -0.4049999999999999, -0.4049999999999999, -0.4049999999999999, -0.4049999999999999, -0.4049999999999999, -0.4049999999999999, -0.4049999999999999, -0.4049999999999999, -0.4049999999999999, -0.4049999999999999, -0.4049999999999999, -0.4049999999999999, -0.4049999999999999, -0.4049999999999999, -0.4049999999999999, -0.4049999999999999, -0.4049999999999999, -0.4049999999999999, -0.4049999999999999, -0.4049999999999999 ], [ -0.3699999999999999, -0.3699999999999999, -0.3699999999999999, -0.3699999999999999, -0.3699999999999999, -0.3699999999999999, -0.3699999999999999, -0.3699999999999999, -0.3699999999999999, -0.3699999999999999, -0.3699999999999999, -0.3699999999999999, -0.3699999999999999, -0.3699999999999999, -0.3699999999999999, -0.3699999999999999, -0.3699999999999999, -0.3699999999999999, -0.3699999999999999, -0.3699999999999999, -0.3699999999999999, -0.3699999999999999, -0.3699999999999999, -0.3699999999999999, -0.3699999999999999, -0.3699999999999999, -0.3699999999999999, -0.3699999999999999, -0.3699999999999999, -0.3699999999999999, -0.3699999999999999, -0.3699999999999999, -0.3699999999999999, -0.3699999999999999, -0.3699999999999999, -0.3699999999999999, -0.3699999999999999, -0.3699999999999999, -0.3699999999999999, -0.3699999999999999, -0.3699999999999999, -0.3699999999999999, -0.3699999999999999, -0.3699999999999999, -0.3699999999999999, -0.3699999999999999, -0.3699999999999999, -0.3699999999999999, -0.3699999999999999, -0.3699999999999999, -0.3699999999999999, -0.3699999999999999, -0.3699999999999999, -0.3699999999999999, -0.3699999999999999, -0.3699999999999999, -0.3699999999999999, -0.3699999999999999, -0.3699999999999999, -0.3699999999999999, -0.3699999999999999, -0.3699999999999999, -0.3699999999999999, -0.3699999999999999, -0.3699999999999999, -0.3699999999999999, -0.3699999999999999, -0.3699999999999999, -0.3699999999999999, -0.3699999999999999, -0.3699999999999999, -0.3699999999999999, -0.3699999999999999, -0.3699999999999999, -0.3699999999999999, -0.3699999999999999, -0.3699999999999999, -0.3699999999999999, -0.3699999999999999, -0.3699999999999999, -0.3699999999999999, -0.3699999999999999, -0.3699999999999999, -0.3699999999999999, -0.3699999999999999, -0.3699999999999999, -0.3699999999999999, -0.3699999999999999, -0.3699999999999999, -0.3699999999999999, -0.3699999999999999, -0.3699999999999999, -0.3699999999999999, -0.3699999999999999, -0.3699999999999999, -0.3699999999999999, -0.3699999999999999, -0.3699999999999999, -0.3699999999999999, -0.3699999999999999, -0.3699999999999999 ], [ -0.33499999999999996, -0.33499999999999996, -0.33499999999999996, -0.33499999999999996, -0.33499999999999996, -0.33499999999999996, -0.33499999999999996, -0.33499999999999996, -0.33499999999999996, -0.33499999999999996, -0.33499999999999996, -0.33499999999999996, -0.33499999999999996, -0.33499999999999996, -0.33499999999999996, -0.33499999999999996, -0.33499999999999996, -0.33499999999999996, -0.33499999999999996, -0.33499999999999996, -0.33499999999999996, -0.33499999999999996, -0.33499999999999996, -0.33499999999999996, -0.33499999999999996, -0.33499999999999996, -0.33499999999999996, -0.33499999999999996, -0.33499999999999996, -0.33499999999999996, -0.33499999999999996, -0.33499999999999996, -0.33499999999999996, -0.33499999999999996, -0.33499999999999996, -0.33499999999999996, -0.33499999999999996, -0.33499999999999996, -0.33499999999999996, -0.33499999999999996, -0.33499999999999996, -0.33499999999999996, -0.33499999999999996, -0.33499999999999996, -0.33499999999999996, -0.33499999999999996, -0.33499999999999996, -0.33499999999999996, -0.33499999999999996, -0.33499999999999996, -0.33499999999999996, -0.33499999999999996, -0.33499999999999996, -0.33499999999999996, -0.33499999999999996, -0.33499999999999996, -0.33499999999999996, -0.33499999999999996, -0.33499999999999996, -0.33499999999999996, -0.33499999999999996, -0.33499999999999996, -0.33499999999999996, -0.33499999999999996, -0.33499999999999996, -0.33499999999999996, -0.33499999999999996, -0.33499999999999996, -0.33499999999999996, -0.33499999999999996, -0.33499999999999996, -0.33499999999999996, -0.33499999999999996, -0.33499999999999996, -0.33499999999999996, -0.33499999999999996, -0.33499999999999996, -0.33499999999999996, -0.33499999999999996, -0.33499999999999996, -0.33499999999999996, -0.33499999999999996, -0.33499999999999996, -0.33499999999999996, -0.33499999999999996, -0.33499999999999996, -0.33499999999999996, -0.33499999999999996, -0.33499999999999996, -0.33499999999999996, -0.33499999999999996, -0.33499999999999996, -0.33499999999999996, -0.33499999999999996, -0.33499999999999996, -0.33499999999999996, -0.33499999999999996, -0.33499999999999996, -0.33499999999999996, -0.33499999999999996, -0.33499999999999996 ], [ -0.29999999999999993, -0.29999999999999993, -0.29999999999999993, -0.29999999999999993, -0.29999999999999993, -0.29999999999999993, -0.29999999999999993, -0.29999999999999993, -0.29999999999999993, -0.29999999999999993, -0.29999999999999993, -0.29999999999999993, -0.29999999999999993, -0.29999999999999993, -0.29999999999999993, -0.29999999999999993, -0.29999999999999993, -0.29999999999999993, -0.29999999999999993, -0.29999999999999993, -0.29999999999999993, -0.29999999999999993, -0.29999999999999993, -0.29999999999999993, -0.29999999999999993, -0.29999999999999993, -0.29999999999999993, -0.29999999999999993, -0.29999999999999993, -0.29999999999999993, -0.29999999999999993, -0.29999999999999993, -0.29999999999999993, -0.29999999999999993, -0.29999999999999993, -0.29999999999999993, -0.29999999999999993, -0.29999999999999993, -0.29999999999999993, -0.29999999999999993, -0.29999999999999993, -0.29999999999999993, -0.29999999999999993, -0.29999999999999993, -0.29999999999999993, -0.29999999999999993, -0.29999999999999993, -0.29999999999999993, -0.29999999999999993, -0.29999999999999993, -0.29999999999999993, -0.29999999999999993, -0.29999999999999993, -0.29999999999999993, -0.29999999999999993, -0.29999999999999993, -0.29999999999999993, -0.29999999999999993, -0.29999999999999993, -0.29999999999999993, -0.29999999999999993, -0.29999999999999993, -0.29999999999999993, -0.29999999999999993, -0.29999999999999993, -0.29999999999999993, -0.29999999999999993, -0.29999999999999993, -0.29999999999999993, -0.29999999999999993, -0.29999999999999993, -0.29999999999999993, -0.29999999999999993, -0.29999999999999993, -0.29999999999999993, -0.29999999999999993, -0.29999999999999993, -0.29999999999999993, -0.29999999999999993, -0.29999999999999993, -0.29999999999999993, -0.29999999999999993, -0.29999999999999993, -0.29999999999999993, -0.29999999999999993, -0.29999999999999993, -0.29999999999999993, -0.29999999999999993, -0.29999999999999993, -0.29999999999999993, -0.29999999999999993, -0.29999999999999993, -0.29999999999999993, -0.29999999999999993, -0.29999999999999993, -0.29999999999999993, -0.29999999999999993, -0.29999999999999993, -0.29999999999999993, -0.29999999999999993, -0.29999999999999993 ], [ -0.2649999999999999, -0.2649999999999999, -0.2649999999999999, -0.2649999999999999, -0.2649999999999999, -0.2649999999999999, -0.2649999999999999, -0.2649999999999999, -0.2649999999999999, -0.2649999999999999, -0.2649999999999999, -0.2649999999999999, -0.2649999999999999, -0.2649999999999999, -0.2649999999999999, -0.2649999999999999, -0.2649999999999999, -0.2649999999999999, -0.2649999999999999, -0.2649999999999999, -0.2649999999999999, -0.2649999999999999, -0.2649999999999999, -0.2649999999999999, -0.2649999999999999, -0.2649999999999999, -0.2649999999999999, -0.2649999999999999, -0.2649999999999999, -0.2649999999999999, -0.2649999999999999, -0.2649999999999999, -0.2649999999999999, -0.2649999999999999, -0.2649999999999999, -0.2649999999999999, -0.2649999999999999, -0.2649999999999999, -0.2649999999999999, -0.2649999999999999, -0.2649999999999999, -0.2649999999999999, -0.2649999999999999, -0.2649999999999999, -0.2649999999999999, -0.2649999999999999, -0.2649999999999999, -0.2649999999999999, -0.2649999999999999, -0.2649999999999999, -0.2649999999999999, -0.2649999999999999, -0.2649999999999999, -0.2649999999999999, -0.2649999999999999, -0.2649999999999999, -0.2649999999999999, -0.2649999999999999, -0.2649999999999999, -0.2649999999999999, -0.2649999999999999, -0.2649999999999999, -0.2649999999999999, -0.2649999999999999, -0.2649999999999999, -0.2649999999999999, -0.2649999999999999, -0.2649999999999999, -0.2649999999999999, -0.2649999999999999, -0.2649999999999999, -0.2649999999999999, -0.2649999999999999, -0.2649999999999999, -0.2649999999999999, -0.2649999999999999, -0.2649999999999999, -0.2649999999999999, -0.2649999999999999, -0.2649999999999999, -0.2649999999999999, -0.2649999999999999, -0.2649999999999999, -0.2649999999999999, -0.2649999999999999, -0.2649999999999999, -0.2649999999999999, -0.2649999999999999, -0.2649999999999999, -0.2649999999999999, -0.2649999999999999, -0.2649999999999999, -0.2649999999999999, -0.2649999999999999, -0.2649999999999999, -0.2649999999999999, -0.2649999999999999, -0.2649999999999999, -0.2649999999999999, -0.2649999999999999, -0.2649999999999999 ], [ -0.22999999999999998, -0.22999999999999998, -0.22999999999999998, -0.22999999999999998, -0.22999999999999998, -0.22999999999999998, -0.22999999999999998, -0.22999999999999998, -0.22999999999999998, -0.22999999999999998, -0.22999999999999998, -0.22999999999999998, -0.22999999999999998, -0.22999999999999998, -0.22999999999999998, -0.22999999999999998, -0.22999999999999998, -0.22999999999999998, -0.22999999999999998, -0.22999999999999998, -0.22999999999999998, -0.22999999999999998, -0.22999999999999998, -0.22999999999999998, -0.22999999999999998, -0.22999999999999998, -0.22999999999999998, -0.22999999999999998, -0.22999999999999998, -0.22999999999999998, -0.22999999999999998, -0.22999999999999998, -0.22999999999999998, -0.22999999999999998, -0.22999999999999998, -0.22999999999999998, -0.22999999999999998, -0.22999999999999998, -0.22999999999999998, -0.22999999999999998, -0.22999999999999998, -0.22999999999999998, -0.22999999999999998, -0.22999999999999998, -0.22999999999999998, -0.22999999999999998, -0.22999999999999998, -0.22999999999999998, -0.22999999999999998, -0.22999999999999998, -0.22999999999999998, -0.22999999999999998, -0.22999999999999998, -0.22999999999999998, -0.22999999999999998, -0.22999999999999998, -0.22999999999999998, -0.22999999999999998, -0.22999999999999998, -0.22999999999999998, -0.22999999999999998, -0.22999999999999998, -0.22999999999999998, -0.22999999999999998, -0.22999999999999998, -0.22999999999999998, -0.22999999999999998, -0.22999999999999998, -0.22999999999999998, -0.22999999999999998, -0.22999999999999998, -0.22999999999999998, -0.22999999999999998, -0.22999999999999998, -0.22999999999999998, -0.22999999999999998, -0.22999999999999998, -0.22999999999999998, -0.22999999999999998, -0.22999999999999998, -0.22999999999999998, -0.22999999999999998, -0.22999999999999998, -0.22999999999999998, -0.22999999999999998, -0.22999999999999998, -0.22999999999999998, -0.22999999999999998, -0.22999999999999998, -0.22999999999999998, -0.22999999999999998, -0.22999999999999998, -0.22999999999999998, -0.22999999999999998, -0.22999999999999998, -0.22999999999999998, -0.22999999999999998, -0.22999999999999998, -0.22999999999999998, -0.22999999999999998, -0.22999999999999998 ], [ -0.19499999999999995, -0.19499999999999995, -0.19499999999999995, -0.19499999999999995, -0.19499999999999995, -0.19499999999999995, -0.19499999999999995, -0.19499999999999995, -0.19499999999999995, -0.19499999999999995, -0.19499999999999995, -0.19499999999999995, -0.19499999999999995, -0.19499999999999995, -0.19499999999999995, -0.19499999999999995, -0.19499999999999995, -0.19499999999999995, -0.19499999999999995, -0.19499999999999995, -0.19499999999999995, -0.19499999999999995, -0.19499999999999995, -0.19499999999999995, -0.19499999999999995, -0.19499999999999995, -0.19499999999999995, -0.19499999999999995, -0.19499999999999995, -0.19499999999999995, -0.19499999999999995, -0.19499999999999995, -0.19499999999999995, -0.19499999999999995, -0.19499999999999995, -0.19499999999999995, -0.19499999999999995, -0.19499999999999995, -0.19499999999999995, -0.19499999999999995, -0.19499999999999995, -0.19499999999999995, -0.19499999999999995, -0.19499999999999995, -0.19499999999999995, -0.19499999999999995, -0.19499999999999995, -0.19499999999999995, -0.19499999999999995, -0.19499999999999995, -0.19499999999999995, -0.19499999999999995, -0.19499999999999995, -0.19499999999999995, -0.19499999999999995, -0.19499999999999995, -0.19499999999999995, -0.19499999999999995, -0.19499999999999995, -0.19499999999999995, -0.19499999999999995, -0.19499999999999995, -0.19499999999999995, -0.19499999999999995, -0.19499999999999995, -0.19499999999999995, -0.19499999999999995, -0.19499999999999995, -0.19499999999999995, -0.19499999999999995, -0.19499999999999995, -0.19499999999999995, -0.19499999999999995, -0.19499999999999995, -0.19499999999999995, -0.19499999999999995, -0.19499999999999995, -0.19499999999999995, -0.19499999999999995, -0.19499999999999995, -0.19499999999999995, -0.19499999999999995, -0.19499999999999995, -0.19499999999999995, -0.19499999999999995, -0.19499999999999995, -0.19499999999999995, -0.19499999999999995, -0.19499999999999995, -0.19499999999999995, -0.19499999999999995, -0.19499999999999995, -0.19499999999999995, -0.19499999999999995, -0.19499999999999995, -0.19499999999999995, -0.19499999999999995, -0.19499999999999995, -0.19499999999999995, -0.19499999999999995, -0.19499999999999995 ], [ -0.15999999999999992, -0.15999999999999992, -0.15999999999999992, -0.15999999999999992, -0.15999999999999992, -0.15999999999999992, -0.15999999999999992, -0.15999999999999992, -0.15999999999999992, -0.15999999999999992, -0.15999999999999992, -0.15999999999999992, -0.15999999999999992, -0.15999999999999992, -0.15999999999999992, -0.15999999999999992, -0.15999999999999992, -0.15999999999999992, -0.15999999999999992, -0.15999999999999992, -0.15999999999999992, -0.15999999999999992, -0.15999999999999992, -0.15999999999999992, -0.15999999999999992, -0.15999999999999992, -0.15999999999999992, -0.15999999999999992, -0.15999999999999992, -0.15999999999999992, -0.15999999999999992, -0.15999999999999992, -0.15999999999999992, -0.15999999999999992, -0.15999999999999992, -0.15999999999999992, -0.15999999999999992, -0.15999999999999992, -0.15999999999999992, -0.15999999999999992, -0.15999999999999992, -0.15999999999999992, -0.15999999999999992, -0.15999999999999992, -0.15999999999999992, -0.15999999999999992, -0.15999999999999992, -0.15999999999999992, -0.15999999999999992, -0.15999999999999992, -0.15999999999999992, -0.15999999999999992, -0.15999999999999992, -0.15999999999999992, -0.15999999999999992, -0.15999999999999992, -0.15999999999999992, -0.15999999999999992, -0.15999999999999992, -0.15999999999999992, -0.15999999999999992, -0.15999999999999992, -0.15999999999999992, -0.15999999999999992, -0.15999999999999992, -0.15999999999999992, -0.15999999999999992, -0.15999999999999992, -0.15999999999999992, -0.15999999999999992, -0.15999999999999992, -0.15999999999999992, -0.15999999999999992, -0.15999999999999992, -0.15999999999999992, -0.15999999999999992, -0.15999999999999992, -0.15999999999999992, -0.15999999999999992, -0.15999999999999992, -0.15999999999999992, -0.15999999999999992, -0.15999999999999992, -0.15999999999999992, -0.15999999999999992, -0.15999999999999992, -0.15999999999999992, -0.15999999999999992, -0.15999999999999992, -0.15999999999999992, -0.15999999999999992, -0.15999999999999992, -0.15999999999999992, -0.15999999999999992, -0.15999999999999992, -0.15999999999999992, -0.15999999999999992, -0.15999999999999992, -0.15999999999999992, -0.15999999999999992, -0.15999999999999992 ], [ -0.12499999999999989, -0.12499999999999989, -0.12499999999999989, -0.12499999999999989, -0.12499999999999989, -0.12499999999999989, -0.12499999999999989, -0.12499999999999989, -0.12499999999999989, -0.12499999999999989, -0.12499999999999989, -0.12499999999999989, -0.12499999999999989, -0.12499999999999989, -0.12499999999999989, -0.12499999999999989, -0.12499999999999989, -0.12499999999999989, -0.12499999999999989, -0.12499999999999989, -0.12499999999999989, -0.12499999999999989, -0.12499999999999989, -0.12499999999999989, -0.12499999999999989, -0.12499999999999989, -0.12499999999999989, -0.12499999999999989, -0.12499999999999989, -0.12499999999999989, -0.12499999999999989, -0.12499999999999989, -0.12499999999999989, -0.12499999999999989, -0.12499999999999989, -0.12499999999999989, -0.12499999999999989, -0.12499999999999989, -0.12499999999999989, -0.12499999999999989, -0.12499999999999989, -0.12499999999999989, -0.12499999999999989, -0.12499999999999989, -0.12499999999999989, -0.12499999999999989, -0.12499999999999989, -0.12499999999999989, -0.12499999999999989, -0.12499999999999989, -0.12499999999999989, -0.12499999999999989, -0.12499999999999989, -0.12499999999999989, -0.12499999999999989, -0.12499999999999989, -0.12499999999999989, -0.12499999999999989, -0.12499999999999989, -0.12499999999999989, -0.12499999999999989, -0.12499999999999989, -0.12499999999999989, -0.12499999999999989, -0.12499999999999989, -0.12499999999999989, -0.12499999999999989, -0.12499999999999989, -0.12499999999999989, -0.12499999999999989, -0.12499999999999989, -0.12499999999999989, -0.12499999999999989, -0.12499999999999989, -0.12499999999999989, -0.12499999999999989, -0.12499999999999989, -0.12499999999999989, -0.12499999999999989, -0.12499999999999989, -0.12499999999999989, -0.12499999999999989, -0.12499999999999989, -0.12499999999999989, -0.12499999999999989, -0.12499999999999989, -0.12499999999999989, -0.12499999999999989, -0.12499999999999989, -0.12499999999999989, -0.12499999999999989, -0.12499999999999989, -0.12499999999999989, -0.12499999999999989, -0.12499999999999989, -0.12499999999999989, -0.12499999999999989, -0.12499999999999989, -0.12499999999999989, -0.12499999999999989, -0.12499999999999989 ], [ -0.08999999999999986, -0.08999999999999986, -0.08999999999999986, -0.08999999999999986, -0.08999999999999986, -0.08999999999999986, -0.08999999999999986, -0.08999999999999986, -0.08999999999999986, -0.08999999999999986, -0.08999999999999986, -0.08999999999999986, -0.08999999999999986, -0.08999999999999986, -0.08999999999999986, -0.08999999999999986, -0.08999999999999986, -0.08999999999999986, -0.08999999999999986, -0.08999999999999986, -0.08999999999999986, -0.08999999999999986, -0.08999999999999986, -0.08999999999999986, -0.08999999999999986, -0.08999999999999986, -0.08999999999999986, -0.08999999999999986, -0.08999999999999986, -0.08999999999999986, -0.08999999999999986, -0.08999999999999986, -0.08999999999999986, -0.08999999999999986, -0.08999999999999986, -0.08999999999999986, -0.08999999999999986, -0.08999999999999986, -0.08999999999999986, -0.08999999999999986, -0.08999999999999986, -0.08999999999999986, -0.08999999999999986, -0.08999999999999986, -0.08999999999999986, -0.08999999999999986, -0.08999999999999986, -0.08999999999999986, -0.08999999999999986, -0.08999999999999986, -0.08999999999999986, -0.08999999999999986, -0.08999999999999986, -0.08999999999999986, -0.08999999999999986, -0.08999999999999986, -0.08999999999999986, -0.08999999999999986, -0.08999999999999986, -0.08999999999999986, -0.08999999999999986, -0.08999999999999986, -0.08999999999999986, -0.08999999999999986, -0.08999999999999986, -0.08999999999999986, -0.08999999999999986, -0.08999999999999986, -0.08999999999999986, -0.08999999999999986, -0.08999999999999986, -0.08999999999999986, -0.08999999999999986, -0.08999999999999986, -0.08999999999999986, -0.08999999999999986, -0.08999999999999986, -0.08999999999999986, -0.08999999999999986, -0.08999999999999986, -0.08999999999999986, -0.08999999999999986, -0.08999999999999986, -0.08999999999999986, -0.08999999999999986, -0.08999999999999986, -0.08999999999999986, -0.08999999999999986, -0.08999999999999986, -0.08999999999999986, -0.08999999999999986, -0.08999999999999986, -0.08999999999999986, -0.08999999999999986, -0.08999999999999986, -0.08999999999999986, -0.08999999999999986, -0.08999999999999986, -0.08999999999999986, -0.08999999999999986, -0.08999999999999986 ], [ -0.05499999999999994, -0.05499999999999994, -0.05499999999999994, -0.05499999999999994, -0.05499999999999994, -0.05499999999999994, -0.05499999999999994, -0.05499999999999994, -0.05499999999999994, -0.05499999999999994, -0.05499999999999994, -0.05499999999999994, -0.05499999999999994, -0.05499999999999994, -0.05499999999999994, -0.05499999999999994, -0.05499999999999994, -0.05499999999999994, -0.05499999999999994, -0.05499999999999994, -0.05499999999999994, -0.05499999999999994, -0.05499999999999994, -0.05499999999999994, -0.05499999999999994, -0.05499999999999994, -0.05499999999999994, -0.05499999999999994, -0.05499999999999994, -0.05499999999999994, -0.05499999999999994, -0.05499999999999994, -0.05499999999999994, -0.05499999999999994, -0.05499999999999994, -0.05499999999999994, -0.05499999999999994, -0.05499999999999994, -0.05499999999999994, -0.05499999999999994, -0.05499999999999994, -0.05499999999999994, -0.05499999999999994, -0.05499999999999994, -0.05499999999999994, -0.05499999999999994, -0.05499999999999994, -0.05499999999999994, -0.05499999999999994, -0.05499999999999994, -0.05499999999999994, -0.05499999999999994, -0.05499999999999994, -0.05499999999999994, -0.05499999999999994, -0.05499999999999994, -0.05499999999999994, -0.05499999999999994, -0.05499999999999994, -0.05499999999999994, -0.05499999999999994, -0.05499999999999994, -0.05499999999999994, -0.05499999999999994, -0.05499999999999994, -0.05499999999999994, -0.05499999999999994, -0.05499999999999994, -0.05499999999999994, -0.05499999999999994, -0.05499999999999994, -0.05499999999999994, -0.05499999999999994, -0.05499999999999994, -0.05499999999999994, -0.05499999999999994, -0.05499999999999994, -0.05499999999999994, -0.05499999999999994, -0.05499999999999994, -0.05499999999999994, -0.05499999999999994, -0.05499999999999994, -0.05499999999999994, -0.05499999999999994, -0.05499999999999994, -0.05499999999999994, -0.05499999999999994, -0.05499999999999994, -0.05499999999999994, -0.05499999999999994, -0.05499999999999994, -0.05499999999999994, -0.05499999999999994, -0.05499999999999994, -0.05499999999999994, -0.05499999999999994, -0.05499999999999994, -0.05499999999999994, -0.05499999999999994, -0.05499999999999994 ], [ -0.019999999999999907, -0.019999999999999907, -0.019999999999999907, -0.019999999999999907, -0.019999999999999907, -0.019999999999999907, -0.019999999999999907, -0.019999999999999907, -0.019999999999999907, -0.019999999999999907, -0.019999999999999907, -0.019999999999999907, -0.019999999999999907, -0.019999999999999907, -0.019999999999999907, -0.019999999999999907, -0.019999999999999907, -0.019999999999999907, -0.019999999999999907, -0.019999999999999907, -0.019999999999999907, -0.019999999999999907, -0.019999999999999907, -0.019999999999999907, -0.019999999999999907, -0.019999999999999907, -0.019999999999999907, -0.019999999999999907, -0.019999999999999907, -0.019999999999999907, -0.019999999999999907, -0.019999999999999907, -0.019999999999999907, -0.019999999999999907, -0.019999999999999907, -0.019999999999999907, -0.019999999999999907, -0.019999999999999907, -0.019999999999999907, -0.019999999999999907, -0.019999999999999907, -0.019999999999999907, -0.019999999999999907, -0.019999999999999907, -0.019999999999999907, -0.019999999999999907, -0.019999999999999907, -0.019999999999999907, -0.019999999999999907, -0.019999999999999907, -0.019999999999999907, -0.019999999999999907, -0.019999999999999907, -0.019999999999999907, -0.019999999999999907, -0.019999999999999907, -0.019999999999999907, -0.019999999999999907, -0.019999999999999907, -0.019999999999999907, -0.019999999999999907, -0.019999999999999907, -0.019999999999999907, -0.019999999999999907, -0.019999999999999907, -0.019999999999999907, -0.019999999999999907, -0.019999999999999907, -0.019999999999999907, -0.019999999999999907, -0.019999999999999907, -0.019999999999999907, -0.019999999999999907, -0.019999999999999907, -0.019999999999999907, -0.019999999999999907, -0.019999999999999907, -0.019999999999999907, -0.019999999999999907, -0.019999999999999907, -0.019999999999999907, -0.019999999999999907, -0.019999999999999907, -0.019999999999999907, -0.019999999999999907, -0.019999999999999907, -0.019999999999999907, -0.019999999999999907, -0.019999999999999907, -0.019999999999999907, -0.019999999999999907, -0.019999999999999907, -0.019999999999999907, -0.019999999999999907, -0.019999999999999907, -0.019999999999999907, -0.019999999999999907, -0.019999999999999907, -0.019999999999999907, -0.019999999999999907, -0.019999999999999907 ], [ 0.015000000000000124, 0.015000000000000124, 0.015000000000000124, 0.015000000000000124, 0.015000000000000124, 0.015000000000000124, 0.015000000000000124, 0.015000000000000124, 0.015000000000000124, 0.015000000000000124, 0.015000000000000124, 0.015000000000000124, 0.015000000000000124, 0.015000000000000124, 0.015000000000000124, 0.015000000000000124, 0.015000000000000124, 0.015000000000000124, 0.015000000000000124, 0.015000000000000124, 0.015000000000000124, 0.015000000000000124, 0.015000000000000124, 0.015000000000000124, 0.015000000000000124, 0.015000000000000124, 0.015000000000000124, 0.015000000000000124, 0.015000000000000124, 0.015000000000000124, 0.015000000000000124, 0.015000000000000124, 0.015000000000000124, 0.015000000000000124, 0.015000000000000124, 0.015000000000000124, 0.015000000000000124, 0.015000000000000124, 0.015000000000000124, 0.015000000000000124, 0.015000000000000124, 0.015000000000000124, 0.015000000000000124, 0.015000000000000124, 0.015000000000000124, 0.015000000000000124, 0.015000000000000124, 0.015000000000000124, 0.015000000000000124, 0.015000000000000124, 0.015000000000000124, 0.015000000000000124, 0.015000000000000124, 0.015000000000000124, 0.015000000000000124, 0.015000000000000124, 0.015000000000000124, 0.015000000000000124, 0.015000000000000124, 0.015000000000000124, 0.015000000000000124, 0.015000000000000124, 0.015000000000000124, 0.015000000000000124, 0.015000000000000124, 0.015000000000000124, 0.015000000000000124, 0.015000000000000124, 0.015000000000000124, 0.015000000000000124, 0.015000000000000124, 0.015000000000000124, 0.015000000000000124, 0.015000000000000124, 0.015000000000000124, 0.015000000000000124, 0.015000000000000124, 0.015000000000000124, 0.015000000000000124, 0.015000000000000124, 0.015000000000000124, 0.015000000000000124, 0.015000000000000124, 0.015000000000000124, 0.015000000000000124, 0.015000000000000124, 0.015000000000000124, 0.015000000000000124, 0.015000000000000124, 0.015000000000000124, 0.015000000000000124, 0.015000000000000124, 0.015000000000000124, 0.015000000000000124, 0.015000000000000124, 0.015000000000000124, 0.015000000000000124, 0.015000000000000124, 0.015000000000000124, 0.015000000000000124, 0.015000000000000124 ], [ 0.050000000000000044, 0.050000000000000044, 0.050000000000000044, 0.050000000000000044, 0.050000000000000044, 0.050000000000000044, 0.050000000000000044, 0.050000000000000044, 0.050000000000000044, 0.050000000000000044, 0.050000000000000044, 0.050000000000000044, 0.050000000000000044, 0.050000000000000044, 0.050000000000000044, 0.050000000000000044, 0.050000000000000044, 0.050000000000000044, 0.050000000000000044, 0.050000000000000044, 0.050000000000000044, 0.050000000000000044, 0.050000000000000044, 0.050000000000000044, 0.050000000000000044, 0.050000000000000044, 0.050000000000000044, 0.050000000000000044, 0.050000000000000044, 0.050000000000000044, 0.050000000000000044, 0.050000000000000044, 0.050000000000000044, 0.050000000000000044, 0.050000000000000044, 0.050000000000000044, 0.050000000000000044, 0.050000000000000044, 0.050000000000000044, 0.050000000000000044, 0.050000000000000044, 0.050000000000000044, 0.050000000000000044, 0.050000000000000044, 0.050000000000000044, 0.050000000000000044, 0.050000000000000044, 0.050000000000000044, 0.050000000000000044, 0.050000000000000044, 0.050000000000000044, 0.050000000000000044, 0.050000000000000044, 0.050000000000000044, 0.050000000000000044, 0.050000000000000044, 0.050000000000000044, 0.050000000000000044, 0.050000000000000044, 0.050000000000000044, 0.050000000000000044, 0.050000000000000044, 0.050000000000000044, 0.050000000000000044, 0.050000000000000044, 0.050000000000000044, 0.050000000000000044, 0.050000000000000044, 0.050000000000000044, 0.050000000000000044, 0.050000000000000044, 0.050000000000000044, 0.050000000000000044, 0.050000000000000044, 0.050000000000000044, 0.050000000000000044, 0.050000000000000044, 0.050000000000000044, 0.050000000000000044, 0.050000000000000044, 0.050000000000000044, 0.050000000000000044, 0.050000000000000044, 0.050000000000000044, 0.050000000000000044, 0.050000000000000044, 0.050000000000000044, 0.050000000000000044, 0.050000000000000044, 0.050000000000000044, 0.050000000000000044, 0.050000000000000044, 0.050000000000000044, 0.050000000000000044, 0.050000000000000044, 0.050000000000000044, 0.050000000000000044, 0.050000000000000044, 0.050000000000000044, 0.050000000000000044, 0.050000000000000044 ], [ 0.08500000000000019, 0.08500000000000019, 0.08500000000000019, 0.08500000000000019, 0.08500000000000019, 0.08500000000000019, 0.08500000000000019, 0.08500000000000019, 0.08500000000000019, 0.08500000000000019, 0.08500000000000019, 0.08500000000000019, 0.08500000000000019, 0.08500000000000019, 0.08500000000000019, 0.08500000000000019, 0.08500000000000019, 0.08500000000000019, 0.08500000000000019, 0.08500000000000019, 0.08500000000000019, 0.08500000000000019, 0.08500000000000019, 0.08500000000000019, 0.08500000000000019, 0.08500000000000019, 0.08500000000000019, 0.08500000000000019, 0.08500000000000019, 0.08500000000000019, 0.08500000000000019, 0.08500000000000019, 0.08500000000000019, 0.08500000000000019, 0.08500000000000019, 0.08500000000000019, 0.08500000000000019, 0.08500000000000019, 0.08500000000000019, 0.08500000000000019, 0.08500000000000019, 0.08500000000000019, 0.08500000000000019, 0.08500000000000019, 0.08500000000000019, 0.08500000000000019, 0.08500000000000019, 0.08500000000000019, 0.08500000000000019, 0.08500000000000019, 0.08500000000000019, 0.08500000000000019, 0.08500000000000019, 0.08500000000000019, 0.08500000000000019, 0.08500000000000019, 0.08500000000000019, 0.08500000000000019, 0.08500000000000019, 0.08500000000000019, 0.08500000000000019, 0.08500000000000019, 0.08500000000000019, 0.08500000000000019, 0.08500000000000019, 0.08500000000000019, 0.08500000000000019, 0.08500000000000019, 0.08500000000000019, 0.08500000000000019, 0.08500000000000019, 0.08500000000000019, 0.08500000000000019, 0.08500000000000019, 0.08500000000000019, 0.08500000000000019, 0.08500000000000019, 0.08500000000000019, 0.08500000000000019, 0.08500000000000019, 0.08500000000000019, 0.08500000000000019, 0.08500000000000019, 0.08500000000000019, 0.08500000000000019, 0.08500000000000019, 0.08500000000000019, 0.08500000000000019, 0.08500000000000019, 0.08500000000000019, 0.08500000000000019, 0.08500000000000019, 0.08500000000000019, 0.08500000000000019, 0.08500000000000019, 0.08500000000000019, 0.08500000000000019, 0.08500000000000019, 0.08500000000000019, 0.08500000000000019, 0.08500000000000019 ], [ 0.1200000000000001, 0.1200000000000001, 0.1200000000000001, 0.1200000000000001, 0.1200000000000001, 0.1200000000000001, 0.1200000000000001, 0.1200000000000001, 0.1200000000000001, 0.1200000000000001, 0.1200000000000001, 0.1200000000000001, 0.1200000000000001, 0.1200000000000001, 0.1200000000000001, 0.1200000000000001, 0.1200000000000001, 0.1200000000000001, 0.1200000000000001, 0.1200000000000001, 0.1200000000000001, 0.1200000000000001, 0.1200000000000001, 0.1200000000000001, 0.1200000000000001, 0.1200000000000001, 0.1200000000000001, 0.1200000000000001, 0.1200000000000001, 0.1200000000000001, 0.1200000000000001, 0.1200000000000001, 0.1200000000000001, 0.1200000000000001, 0.1200000000000001, 0.1200000000000001, 0.1200000000000001, 0.1200000000000001, 0.1200000000000001, 0.1200000000000001, 0.1200000000000001, 0.1200000000000001, 0.1200000000000001, 0.1200000000000001, 0.1200000000000001, 0.1200000000000001, 0.1200000000000001, 0.1200000000000001, 0.1200000000000001, 0.1200000000000001, 0.1200000000000001, 0.1200000000000001, 0.1200000000000001, 0.1200000000000001, 0.1200000000000001, 0.1200000000000001, 0.1200000000000001, 0.1200000000000001, 0.1200000000000001, 0.1200000000000001, 0.1200000000000001, 0.1200000000000001, 0.1200000000000001, 0.1200000000000001, 0.1200000000000001, 0.1200000000000001, 0.1200000000000001, 0.1200000000000001, 0.1200000000000001, 0.1200000000000001, 0.1200000000000001, 0.1200000000000001, 0.1200000000000001, 0.1200000000000001, 0.1200000000000001, 0.1200000000000001, 0.1200000000000001, 0.1200000000000001, 0.1200000000000001, 0.1200000000000001, 0.1200000000000001, 0.1200000000000001, 0.1200000000000001, 0.1200000000000001, 0.1200000000000001, 0.1200000000000001, 0.1200000000000001, 0.1200000000000001, 0.1200000000000001, 0.1200000000000001, 0.1200000000000001, 0.1200000000000001, 0.1200000000000001, 0.1200000000000001, 0.1200000000000001, 0.1200000000000001, 0.1200000000000001, 0.1200000000000001, 0.1200000000000001, 0.1200000000000001, 0.1200000000000001 ], [ 0.15500000000000003, 0.15500000000000003, 0.15500000000000003, 0.15500000000000003, 0.15500000000000003, 0.15500000000000003, 0.15500000000000003, 0.15500000000000003, 0.15500000000000003, 0.15500000000000003, 0.15500000000000003, 0.15500000000000003, 0.15500000000000003, 0.15500000000000003, 0.15500000000000003, 0.15500000000000003, 0.15500000000000003, 0.15500000000000003, 0.15500000000000003, 0.15500000000000003, 0.15500000000000003, 0.15500000000000003, 0.15500000000000003, 0.15500000000000003, 0.15500000000000003, 0.15500000000000003, 0.15500000000000003, 0.15500000000000003, 0.15500000000000003, 0.15500000000000003, 0.15500000000000003, 0.15500000000000003, 0.15500000000000003, 0.15500000000000003, 0.15500000000000003, 0.15500000000000003, 0.15500000000000003, 0.15500000000000003, 0.15500000000000003, 0.15500000000000003, 0.15500000000000003, 0.15500000000000003, 0.15500000000000003, 0.15500000000000003, 0.15500000000000003, 0.15500000000000003, 0.15500000000000003, 0.15500000000000003, 0.15500000000000003, 0.15500000000000003, 0.15500000000000003, 0.15500000000000003, 0.15500000000000003, 0.15500000000000003, 0.15500000000000003, 0.15500000000000003, 0.15500000000000003, 0.15500000000000003, 0.15500000000000003, 0.15500000000000003, 0.15500000000000003, 0.15500000000000003, 0.15500000000000003, 0.15500000000000003, 0.15500000000000003, 0.15500000000000003, 0.15500000000000003, 0.15500000000000003, 0.15500000000000003, 0.15500000000000003, 0.15500000000000003, 0.15500000000000003, 0.15500000000000003, 0.15500000000000003, 0.15500000000000003, 0.15500000000000003, 0.15500000000000003, 0.15500000000000003, 0.15500000000000003, 0.15500000000000003, 0.15500000000000003, 0.15500000000000003, 0.15500000000000003, 0.15500000000000003, 0.15500000000000003, 0.15500000000000003, 0.15500000000000003, 0.15500000000000003, 0.15500000000000003, 0.15500000000000003, 0.15500000000000003, 0.15500000000000003, 0.15500000000000003, 0.15500000000000003, 0.15500000000000003, 0.15500000000000003, 0.15500000000000003, 0.15500000000000003, 0.15500000000000003, 0.15500000000000003, 0.15500000000000003 ], [ 0.19000000000000017, 0.19000000000000017, 0.19000000000000017, 0.19000000000000017, 0.19000000000000017, 0.19000000000000017, 0.19000000000000017, 0.19000000000000017, 0.19000000000000017, 0.19000000000000017, 0.19000000000000017, 0.19000000000000017, 0.19000000000000017, 0.19000000000000017, 0.19000000000000017, 0.19000000000000017, 0.19000000000000017, 0.19000000000000017, 0.19000000000000017, 0.19000000000000017, 0.19000000000000017, 0.19000000000000017, 0.19000000000000017, 0.19000000000000017, 0.19000000000000017, 0.19000000000000017, 0.19000000000000017, 0.19000000000000017, 0.19000000000000017, 0.19000000000000017, 0.19000000000000017, 0.19000000000000017, 0.19000000000000017, 0.19000000000000017, 0.19000000000000017, 0.19000000000000017, 0.19000000000000017, 0.19000000000000017, 0.19000000000000017, 0.19000000000000017, 0.19000000000000017, 0.19000000000000017, 0.19000000000000017, 0.19000000000000017, 0.19000000000000017, 0.19000000000000017, 0.19000000000000017, 0.19000000000000017, 0.19000000000000017, 0.19000000000000017, 0.19000000000000017, 0.19000000000000017, 0.19000000000000017, 0.19000000000000017, 0.19000000000000017, 0.19000000000000017, 0.19000000000000017, 0.19000000000000017, 0.19000000000000017, 0.19000000000000017, 0.19000000000000017, 0.19000000000000017, 0.19000000000000017, 0.19000000000000017, 0.19000000000000017, 0.19000000000000017, 0.19000000000000017, 0.19000000000000017, 0.19000000000000017, 0.19000000000000017, 0.19000000000000017, 0.19000000000000017, 0.19000000000000017, 0.19000000000000017, 0.19000000000000017, 0.19000000000000017, 0.19000000000000017, 0.19000000000000017, 0.19000000000000017, 0.19000000000000017, 0.19000000000000017, 0.19000000000000017, 0.19000000000000017, 0.19000000000000017, 0.19000000000000017, 0.19000000000000017, 0.19000000000000017, 0.19000000000000017, 0.19000000000000017, 0.19000000000000017, 0.19000000000000017, 0.19000000000000017, 0.19000000000000017, 0.19000000000000017, 0.19000000000000017, 0.19000000000000017, 0.19000000000000017, 0.19000000000000017, 0.19000000000000017, 0.19000000000000017, 0.19000000000000017 ], [ 0.2250000000000001, 0.2250000000000001, 0.2250000000000001, 0.2250000000000001, 0.2250000000000001, 0.2250000000000001, 0.2250000000000001, 0.2250000000000001, 0.2250000000000001, 0.2250000000000001, 0.2250000000000001, 0.2250000000000001, 0.2250000000000001, 0.2250000000000001, 0.2250000000000001, 0.2250000000000001, 0.2250000000000001, 0.2250000000000001, 0.2250000000000001, 0.2250000000000001, 0.2250000000000001, 0.2250000000000001, 0.2250000000000001, 0.2250000000000001, 0.2250000000000001, 0.2250000000000001, 0.2250000000000001, 0.2250000000000001, 0.2250000000000001, 0.2250000000000001, 0.2250000000000001, 0.2250000000000001, 0.2250000000000001, 0.2250000000000001, 0.2250000000000001, 0.2250000000000001, 0.2250000000000001, 0.2250000000000001, 0.2250000000000001, 0.2250000000000001, 0.2250000000000001, 0.2250000000000001, 0.2250000000000001, 0.2250000000000001, 0.2250000000000001, 0.2250000000000001, 0.2250000000000001, 0.2250000000000001, 0.2250000000000001, 0.2250000000000001, 0.2250000000000001, 0.2250000000000001, 0.2250000000000001, 0.2250000000000001, 0.2250000000000001, 0.2250000000000001, 0.2250000000000001, 0.2250000000000001, 0.2250000000000001, 0.2250000000000001, 0.2250000000000001, 0.2250000000000001, 0.2250000000000001, 0.2250000000000001, 0.2250000000000001, 0.2250000000000001, 0.2250000000000001, 0.2250000000000001, 0.2250000000000001, 0.2250000000000001, 0.2250000000000001, 0.2250000000000001, 0.2250000000000001, 0.2250000000000001, 0.2250000000000001, 0.2250000000000001, 0.2250000000000001, 0.2250000000000001, 0.2250000000000001, 0.2250000000000001, 0.2250000000000001, 0.2250000000000001, 0.2250000000000001, 0.2250000000000001, 0.2250000000000001, 0.2250000000000001, 0.2250000000000001, 0.2250000000000001, 0.2250000000000001, 0.2250000000000001, 0.2250000000000001, 0.2250000000000001, 0.2250000000000001, 0.2250000000000001, 0.2250000000000001, 0.2250000000000001, 0.2250000000000001, 0.2250000000000001, 0.2250000000000001, 0.2250000000000001, 0.2250000000000001 ], [ 0.26000000000000023, 0.26000000000000023, 0.26000000000000023, 0.26000000000000023, 0.26000000000000023, 0.26000000000000023, 0.26000000000000023, 0.26000000000000023, 0.26000000000000023, 0.26000000000000023, 0.26000000000000023, 0.26000000000000023, 0.26000000000000023, 0.26000000000000023, 0.26000000000000023, 0.26000000000000023, 0.26000000000000023, 0.26000000000000023, 0.26000000000000023, 0.26000000000000023, 0.26000000000000023, 0.26000000000000023, 0.26000000000000023, 0.26000000000000023, 0.26000000000000023, 0.26000000000000023, 0.26000000000000023, 0.26000000000000023, 0.26000000000000023, 0.26000000000000023, 0.26000000000000023, 0.26000000000000023, 0.26000000000000023, 0.26000000000000023, 0.26000000000000023, 0.26000000000000023, 0.26000000000000023, 0.26000000000000023, 0.26000000000000023, 0.26000000000000023, 0.26000000000000023, 0.26000000000000023, 0.26000000000000023, 0.26000000000000023, 0.26000000000000023, 0.26000000000000023, 0.26000000000000023, 0.26000000000000023, 0.26000000000000023, 0.26000000000000023, 0.26000000000000023, 0.26000000000000023, 0.26000000000000023, 0.26000000000000023, 0.26000000000000023, 0.26000000000000023, 0.26000000000000023, 0.26000000000000023, 0.26000000000000023, 0.26000000000000023, 0.26000000000000023, 0.26000000000000023, 0.26000000000000023, 0.26000000000000023, 0.26000000000000023, 0.26000000000000023, 0.26000000000000023, 0.26000000000000023, 0.26000000000000023, 0.26000000000000023, 0.26000000000000023, 0.26000000000000023, 0.26000000000000023, 0.26000000000000023, 0.26000000000000023, 0.26000000000000023, 0.26000000000000023, 0.26000000000000023, 0.26000000000000023, 0.26000000000000023, 0.26000000000000023, 0.26000000000000023, 0.26000000000000023, 0.26000000000000023, 0.26000000000000023, 0.26000000000000023, 0.26000000000000023, 0.26000000000000023, 0.26000000000000023, 0.26000000000000023, 0.26000000000000023, 0.26000000000000023, 0.26000000000000023, 0.26000000000000023, 0.26000000000000023, 0.26000000000000023, 0.26000000000000023, 0.26000000000000023, 0.26000000000000023, 0.26000000000000023, 0.26000000000000023 ], [ 0.29500000000000015, 0.29500000000000015, 0.29500000000000015, 0.29500000000000015, 0.29500000000000015, 0.29500000000000015, 0.29500000000000015, 0.29500000000000015, 0.29500000000000015, 0.29500000000000015, 0.29500000000000015, 0.29500000000000015, 0.29500000000000015, 0.29500000000000015, 0.29500000000000015, 0.29500000000000015, 0.29500000000000015, 0.29500000000000015, 0.29500000000000015, 0.29500000000000015, 0.29500000000000015, 0.29500000000000015, 0.29500000000000015, 0.29500000000000015, 0.29500000000000015, 0.29500000000000015, 0.29500000000000015, 0.29500000000000015, 0.29500000000000015, 0.29500000000000015, 0.29500000000000015, 0.29500000000000015, 0.29500000000000015, 0.29500000000000015, 0.29500000000000015, 0.29500000000000015, 0.29500000000000015, 0.29500000000000015, 0.29500000000000015, 0.29500000000000015, 0.29500000000000015, 0.29500000000000015, 0.29500000000000015, 0.29500000000000015, 0.29500000000000015, 0.29500000000000015, 0.29500000000000015, 0.29500000000000015, 0.29500000000000015, 0.29500000000000015, 0.29500000000000015, 0.29500000000000015, 0.29500000000000015, 0.29500000000000015, 0.29500000000000015, 0.29500000000000015, 0.29500000000000015, 0.29500000000000015, 0.29500000000000015, 0.29500000000000015, 0.29500000000000015, 0.29500000000000015, 0.29500000000000015, 0.29500000000000015, 0.29500000000000015, 0.29500000000000015, 0.29500000000000015, 0.29500000000000015, 0.29500000000000015, 0.29500000000000015, 0.29500000000000015, 0.29500000000000015, 0.29500000000000015, 0.29500000000000015, 0.29500000000000015, 0.29500000000000015, 0.29500000000000015, 0.29500000000000015, 0.29500000000000015, 0.29500000000000015, 0.29500000000000015, 0.29500000000000015, 0.29500000000000015, 0.29500000000000015, 0.29500000000000015, 0.29500000000000015, 0.29500000000000015, 0.29500000000000015, 0.29500000000000015, 0.29500000000000015, 0.29500000000000015, 0.29500000000000015, 0.29500000000000015, 0.29500000000000015, 0.29500000000000015, 0.29500000000000015, 0.29500000000000015, 0.29500000000000015, 0.29500000000000015, 0.29500000000000015, 0.29500000000000015 ], [ 0.33000000000000007, 0.33000000000000007, 0.33000000000000007, 0.33000000000000007, 0.33000000000000007, 0.33000000000000007, 0.33000000000000007, 0.33000000000000007, 0.33000000000000007, 0.33000000000000007, 0.33000000000000007, 0.33000000000000007, 0.33000000000000007, 0.33000000000000007, 0.33000000000000007, 0.33000000000000007, 0.33000000000000007, 0.33000000000000007, 0.33000000000000007, 0.33000000000000007, 0.33000000000000007, 0.33000000000000007, 0.33000000000000007, 0.33000000000000007, 0.33000000000000007, 0.33000000000000007, 0.33000000000000007, 0.33000000000000007, 0.33000000000000007, 0.33000000000000007, 0.33000000000000007, 0.33000000000000007, 0.33000000000000007, 0.33000000000000007, 0.33000000000000007, 0.33000000000000007, 0.33000000000000007, 0.33000000000000007, 0.33000000000000007, 0.33000000000000007, 0.33000000000000007, 0.33000000000000007, 0.33000000000000007, 0.33000000000000007, 0.33000000000000007, 0.33000000000000007, 0.33000000000000007, 0.33000000000000007, 0.33000000000000007, 0.33000000000000007, 0.33000000000000007, 0.33000000000000007, 0.33000000000000007, 0.33000000000000007, 0.33000000000000007, 0.33000000000000007, 0.33000000000000007, 0.33000000000000007, 0.33000000000000007, 0.33000000000000007, 0.33000000000000007, 0.33000000000000007, 0.33000000000000007, 0.33000000000000007, 0.33000000000000007, 0.33000000000000007, 0.33000000000000007, 0.33000000000000007, 0.33000000000000007, 0.33000000000000007, 0.33000000000000007, 0.33000000000000007, 0.33000000000000007, 0.33000000000000007, 0.33000000000000007, 0.33000000000000007, 0.33000000000000007, 0.33000000000000007, 0.33000000000000007, 0.33000000000000007, 0.33000000000000007, 0.33000000000000007, 0.33000000000000007, 0.33000000000000007, 0.33000000000000007, 0.33000000000000007, 0.33000000000000007, 0.33000000000000007, 0.33000000000000007, 0.33000000000000007, 0.33000000000000007, 0.33000000000000007, 0.33000000000000007, 0.33000000000000007, 0.33000000000000007, 0.33000000000000007, 0.33000000000000007, 0.33000000000000007, 0.33000000000000007, 0.33000000000000007, 0.33000000000000007 ], [ 0.3650000000000002, 0.3650000000000002, 0.3650000000000002, 0.3650000000000002, 0.3650000000000002, 0.3650000000000002, 0.3650000000000002, 0.3650000000000002, 0.3650000000000002, 0.3650000000000002, 0.3650000000000002, 0.3650000000000002, 0.3650000000000002, 0.3650000000000002, 0.3650000000000002, 0.3650000000000002, 0.3650000000000002, 0.3650000000000002, 0.3650000000000002, 0.3650000000000002, 0.3650000000000002, 0.3650000000000002, 0.3650000000000002, 0.3650000000000002, 0.3650000000000002, 0.3650000000000002, 0.3650000000000002, 0.3650000000000002, 0.3650000000000002, 0.3650000000000002, 0.3650000000000002, 0.3650000000000002, 0.3650000000000002, 0.3650000000000002, 0.3650000000000002, 0.3650000000000002, 0.3650000000000002, 0.3650000000000002, 0.3650000000000002, 0.3650000000000002, 0.3650000000000002, 0.3650000000000002, 0.3650000000000002, 0.3650000000000002, 0.3650000000000002, 0.3650000000000002, 0.3650000000000002, 0.3650000000000002, 0.3650000000000002, 0.3650000000000002, 0.3650000000000002, 0.3650000000000002, 0.3650000000000002, 0.3650000000000002, 0.3650000000000002, 0.3650000000000002, 0.3650000000000002, 0.3650000000000002, 0.3650000000000002, 0.3650000000000002, 0.3650000000000002, 0.3650000000000002, 0.3650000000000002, 0.3650000000000002, 0.3650000000000002, 0.3650000000000002, 0.3650000000000002, 0.3650000000000002, 0.3650000000000002, 0.3650000000000002, 0.3650000000000002, 0.3650000000000002, 0.3650000000000002, 0.3650000000000002, 0.3650000000000002, 0.3650000000000002, 0.3650000000000002, 0.3650000000000002, 0.3650000000000002, 0.3650000000000002, 0.3650000000000002, 0.3650000000000002, 0.3650000000000002, 0.3650000000000002, 0.3650000000000002, 0.3650000000000002, 0.3650000000000002, 0.3650000000000002, 0.3650000000000002, 0.3650000000000002, 0.3650000000000002, 0.3650000000000002, 0.3650000000000002, 0.3650000000000002, 0.3650000000000002, 0.3650000000000002, 0.3650000000000002, 0.3650000000000002, 0.3650000000000002, 0.3650000000000002, 0.3650000000000002 ], [ 0.40000000000000013, 0.40000000000000013, 0.40000000000000013, 0.40000000000000013, 0.40000000000000013, 0.40000000000000013, 0.40000000000000013, 0.40000000000000013, 0.40000000000000013, 0.40000000000000013, 0.40000000000000013, 0.40000000000000013, 0.40000000000000013, 0.40000000000000013, 0.40000000000000013, 0.40000000000000013, 0.40000000000000013, 0.40000000000000013, 0.40000000000000013, 0.40000000000000013, 0.40000000000000013, 0.40000000000000013, 0.40000000000000013, 0.40000000000000013, 0.40000000000000013, 0.40000000000000013, 0.40000000000000013, 0.40000000000000013, 0.40000000000000013, 0.40000000000000013, 0.40000000000000013, 0.40000000000000013, 0.40000000000000013, 0.40000000000000013, 0.40000000000000013, 0.40000000000000013, 0.40000000000000013, 0.40000000000000013, 0.40000000000000013, 0.40000000000000013, 0.40000000000000013, 0.40000000000000013, 0.40000000000000013, 0.40000000000000013, 0.40000000000000013, 0.40000000000000013, 0.40000000000000013, 0.40000000000000013, 0.40000000000000013, 0.40000000000000013, 0.40000000000000013, 0.40000000000000013, 0.40000000000000013, 0.40000000000000013, 0.40000000000000013, 0.40000000000000013, 0.40000000000000013, 0.40000000000000013, 0.40000000000000013, 0.40000000000000013, 0.40000000000000013, 0.40000000000000013, 0.40000000000000013, 0.40000000000000013, 0.40000000000000013, 0.40000000000000013, 0.40000000000000013, 0.40000000000000013, 0.40000000000000013, 0.40000000000000013, 0.40000000000000013, 0.40000000000000013, 0.40000000000000013, 0.40000000000000013, 0.40000000000000013, 0.40000000000000013, 0.40000000000000013, 0.40000000000000013, 0.40000000000000013, 0.40000000000000013, 0.40000000000000013, 0.40000000000000013, 0.40000000000000013, 0.40000000000000013, 0.40000000000000013, 0.40000000000000013, 0.40000000000000013, 0.40000000000000013, 0.40000000000000013, 0.40000000000000013, 0.40000000000000013, 0.40000000000000013, 0.40000000000000013, 0.40000000000000013, 0.40000000000000013, 0.40000000000000013, 0.40000000000000013, 0.40000000000000013, 0.40000000000000013, 0.40000000000000013, 0.40000000000000013 ], [ 0.43500000000000005, 0.43500000000000005, 0.43500000000000005, 0.43500000000000005, 0.43500000000000005, 0.43500000000000005, 0.43500000000000005, 0.43500000000000005, 0.43500000000000005, 0.43500000000000005, 0.43500000000000005, 0.43500000000000005, 0.43500000000000005, 0.43500000000000005, 0.43500000000000005, 0.43500000000000005, 0.43500000000000005, 0.43500000000000005, 0.43500000000000005, 0.43500000000000005, 0.43500000000000005, 0.43500000000000005, 0.43500000000000005, 0.43500000000000005, 0.43500000000000005, 0.43500000000000005, 0.43500000000000005, 0.43500000000000005, 0.43500000000000005, 0.43500000000000005, 0.43500000000000005, 0.43500000000000005, 0.43500000000000005, 0.43500000000000005, 0.43500000000000005, 0.43500000000000005, 0.43500000000000005, 0.43500000000000005, 0.43500000000000005, 0.43500000000000005, 0.43500000000000005, 0.43500000000000005, 0.43500000000000005, 0.43500000000000005, 0.43500000000000005, 0.43500000000000005, 0.43500000000000005, 0.43500000000000005, 0.43500000000000005, 0.43500000000000005, 0.43500000000000005, 0.43500000000000005, 0.43500000000000005, 0.43500000000000005, 0.43500000000000005, 0.43500000000000005, 0.43500000000000005, 0.43500000000000005, 0.43500000000000005, 0.43500000000000005, 0.43500000000000005, 0.43500000000000005, 0.43500000000000005, 0.43500000000000005, 0.43500000000000005, 0.43500000000000005, 0.43500000000000005, 0.43500000000000005, 0.43500000000000005, 0.43500000000000005, 0.43500000000000005, 0.43500000000000005, 0.43500000000000005, 0.43500000000000005, 0.43500000000000005, 0.43500000000000005, 0.43500000000000005, 0.43500000000000005, 0.43500000000000005, 0.43500000000000005, 0.43500000000000005, 0.43500000000000005, 0.43500000000000005, 0.43500000000000005, 0.43500000000000005, 0.43500000000000005, 0.43500000000000005, 0.43500000000000005, 0.43500000000000005, 0.43500000000000005, 0.43500000000000005, 0.43500000000000005, 0.43500000000000005, 0.43500000000000005, 0.43500000000000005, 0.43500000000000005, 0.43500000000000005, 0.43500000000000005, 0.43500000000000005, 0.43500000000000005, 0.43500000000000005 ], [ 0.4700000000000002, 0.4700000000000002, 0.4700000000000002, 0.4700000000000002, 0.4700000000000002, 0.4700000000000002, 0.4700000000000002, 0.4700000000000002, 0.4700000000000002, 0.4700000000000002, 0.4700000000000002, 0.4700000000000002, 0.4700000000000002, 0.4700000000000002, 0.4700000000000002, 0.4700000000000002, 0.4700000000000002, 0.4700000000000002, 0.4700000000000002, 0.4700000000000002, 0.4700000000000002, 0.4700000000000002, 0.4700000000000002, 0.4700000000000002, 0.4700000000000002, 0.4700000000000002, 0.4700000000000002, 0.4700000000000002, 0.4700000000000002, 0.4700000000000002, 0.4700000000000002, 0.4700000000000002, 0.4700000000000002, 0.4700000000000002, 0.4700000000000002, 0.4700000000000002, 0.4700000000000002, 0.4700000000000002, 0.4700000000000002, 0.4700000000000002, 0.4700000000000002, 0.4700000000000002, 0.4700000000000002, 0.4700000000000002, 0.4700000000000002, 0.4700000000000002, 0.4700000000000002, 0.4700000000000002, 0.4700000000000002, 0.4700000000000002, 0.4700000000000002, 0.4700000000000002, 0.4700000000000002, 0.4700000000000002, 0.4700000000000002, 0.4700000000000002, 0.4700000000000002, 0.4700000000000002, 0.4700000000000002, 0.4700000000000002, 0.4700000000000002, 0.4700000000000002, 0.4700000000000002, 0.4700000000000002, 0.4700000000000002, 0.4700000000000002, 0.4700000000000002, 0.4700000000000002, 0.4700000000000002, 0.4700000000000002, 0.4700000000000002, 0.4700000000000002, 0.4700000000000002, 0.4700000000000002, 0.4700000000000002, 0.4700000000000002, 0.4700000000000002, 0.4700000000000002, 0.4700000000000002, 0.4700000000000002, 0.4700000000000002, 0.4700000000000002, 0.4700000000000002, 0.4700000000000002, 0.4700000000000002, 0.4700000000000002, 0.4700000000000002, 0.4700000000000002, 0.4700000000000002, 0.4700000000000002, 0.4700000000000002, 0.4700000000000002, 0.4700000000000002, 0.4700000000000002, 0.4700000000000002, 0.4700000000000002, 0.4700000000000002, 0.4700000000000002, 0.4700000000000002, 0.4700000000000002, 0.4700000000000002 ], [ 0.5050000000000001, 0.5050000000000001, 0.5050000000000001, 0.5050000000000001, 0.5050000000000001, 0.5050000000000001, 0.5050000000000001, 0.5050000000000001, 0.5050000000000001, 0.5050000000000001, 0.5050000000000001, 0.5050000000000001, 0.5050000000000001, 0.5050000000000001, 0.5050000000000001, 0.5050000000000001, 0.5050000000000001, 0.5050000000000001, 0.5050000000000001, 0.5050000000000001, 0.5050000000000001, 0.5050000000000001, 0.5050000000000001, 0.5050000000000001, 0.5050000000000001, 0.5050000000000001, 0.5050000000000001, 0.5050000000000001, 0.5050000000000001, 0.5050000000000001, 0.5050000000000001, 0.5050000000000001, 0.5050000000000001, 0.5050000000000001, 0.5050000000000001, 0.5050000000000001, 0.5050000000000001, 0.5050000000000001, 0.5050000000000001, 0.5050000000000001, 0.5050000000000001, 0.5050000000000001, 0.5050000000000001, 0.5050000000000001, 0.5050000000000001, 0.5050000000000001, 0.5050000000000001, 0.5050000000000001, 0.5050000000000001, 0.5050000000000001, 0.5050000000000001, 0.5050000000000001, 0.5050000000000001, 0.5050000000000001, 0.5050000000000001, 0.5050000000000001, 0.5050000000000001, 0.5050000000000001, 0.5050000000000001, 0.5050000000000001, 0.5050000000000001, 0.5050000000000001, 0.5050000000000001, 0.5050000000000001, 0.5050000000000001, 0.5050000000000001, 0.5050000000000001, 0.5050000000000001, 0.5050000000000001, 0.5050000000000001, 0.5050000000000001, 0.5050000000000001, 0.5050000000000001, 0.5050000000000001, 0.5050000000000001, 0.5050000000000001, 0.5050000000000001, 0.5050000000000001, 0.5050000000000001, 0.5050000000000001, 0.5050000000000001, 0.5050000000000001, 0.5050000000000001, 0.5050000000000001, 0.5050000000000001, 0.5050000000000001, 0.5050000000000001, 0.5050000000000001, 0.5050000000000001, 0.5050000000000001, 0.5050000000000001, 0.5050000000000001, 0.5050000000000001, 0.5050000000000001, 0.5050000000000001, 0.5050000000000001, 0.5050000000000001, 0.5050000000000001, 0.5050000000000001, 0.5050000000000001, 0.5050000000000001 ], [ 0.54, 0.54, 0.54, 0.54, 0.54, 0.54, 0.54, 0.54, 0.54, 0.54, 0.54, 0.54, 0.54, 0.54, 0.54, 0.54, 0.54, 0.54, 0.54, 0.54, 0.54, 0.54, 0.54, 0.54, 0.54, 0.54, 0.54, 0.54, 0.54, 0.54, 0.54, 0.54, 0.54, 0.54, 0.54, 0.54, 0.54, 0.54, 0.54, 0.54, 0.54, 0.54, 0.54, 0.54, 0.54, 0.54, 0.54, 0.54, 0.54, 0.54, 0.54, 0.54, 0.54, 0.54, 0.54, 0.54, 0.54, 0.54, 0.54, 0.54, 0.54, 0.54, 0.54, 0.54, 0.54, 0.54, 0.54, 0.54, 0.54, 0.54, 0.54, 0.54, 0.54, 0.54, 0.54, 0.54, 0.54, 0.54, 0.54, 0.54, 0.54, 0.54, 0.54, 0.54, 0.54, 0.54, 0.54, 0.54, 0.54, 0.54, 0.54, 0.54, 0.54, 0.54, 0.54, 0.54, 0.54, 0.54, 0.54, 0.54, 0.54 ], [ 0.5750000000000002, 0.5750000000000002, 0.5750000000000002, 0.5750000000000002, 0.5750000000000002, 0.5750000000000002, 0.5750000000000002, 0.5750000000000002, 0.5750000000000002, 0.5750000000000002, 0.5750000000000002, 0.5750000000000002, 0.5750000000000002, 0.5750000000000002, 0.5750000000000002, 0.5750000000000002, 0.5750000000000002, 0.5750000000000002, 0.5750000000000002, 0.5750000000000002, 0.5750000000000002, 0.5750000000000002, 0.5750000000000002, 0.5750000000000002, 0.5750000000000002, 0.5750000000000002, 0.5750000000000002, 0.5750000000000002, 0.5750000000000002, 0.5750000000000002, 0.5750000000000002, 0.5750000000000002, 0.5750000000000002, 0.5750000000000002, 0.5750000000000002, 0.5750000000000002, 0.5750000000000002, 0.5750000000000002, 0.5750000000000002, 0.5750000000000002, 0.5750000000000002, 0.5750000000000002, 0.5750000000000002, 0.5750000000000002, 0.5750000000000002, 0.5750000000000002, 0.5750000000000002, 0.5750000000000002, 0.5750000000000002, 0.5750000000000002, 0.5750000000000002, 0.5750000000000002, 0.5750000000000002, 0.5750000000000002, 0.5750000000000002, 0.5750000000000002, 0.5750000000000002, 0.5750000000000002, 0.5750000000000002, 0.5750000000000002, 0.5750000000000002, 0.5750000000000002, 0.5750000000000002, 0.5750000000000002, 0.5750000000000002, 0.5750000000000002, 0.5750000000000002, 0.5750000000000002, 0.5750000000000002, 0.5750000000000002, 0.5750000000000002, 0.5750000000000002, 0.5750000000000002, 0.5750000000000002, 0.5750000000000002, 0.5750000000000002, 0.5750000000000002, 0.5750000000000002, 0.5750000000000002, 0.5750000000000002, 0.5750000000000002, 0.5750000000000002, 0.5750000000000002, 0.5750000000000002, 0.5750000000000002, 0.5750000000000002, 0.5750000000000002, 0.5750000000000002, 0.5750000000000002, 0.5750000000000002, 0.5750000000000002, 0.5750000000000002, 0.5750000000000002, 0.5750000000000002, 0.5750000000000002, 0.5750000000000002, 0.5750000000000002, 0.5750000000000002, 0.5750000000000002, 0.5750000000000002, 0.5750000000000002 ], [ 0.6100000000000001, 0.6100000000000001, 0.6100000000000001, 0.6100000000000001, 0.6100000000000001, 0.6100000000000001, 0.6100000000000001, 0.6100000000000001, 0.6100000000000001, 0.6100000000000001, 0.6100000000000001, 0.6100000000000001, 0.6100000000000001, 0.6100000000000001, 0.6100000000000001, 0.6100000000000001, 0.6100000000000001, 0.6100000000000001, 0.6100000000000001, 0.6100000000000001, 0.6100000000000001, 0.6100000000000001, 0.6100000000000001, 0.6100000000000001, 0.6100000000000001, 0.6100000000000001, 0.6100000000000001, 0.6100000000000001, 0.6100000000000001, 0.6100000000000001, 0.6100000000000001, 0.6100000000000001, 0.6100000000000001, 0.6100000000000001, 0.6100000000000001, 0.6100000000000001, 0.6100000000000001, 0.6100000000000001, 0.6100000000000001, 0.6100000000000001, 0.6100000000000001, 0.6100000000000001, 0.6100000000000001, 0.6100000000000001, 0.6100000000000001, 0.6100000000000001, 0.6100000000000001, 0.6100000000000001, 0.6100000000000001, 0.6100000000000001, 0.6100000000000001, 0.6100000000000001, 0.6100000000000001, 0.6100000000000001, 0.6100000000000001, 0.6100000000000001, 0.6100000000000001, 0.6100000000000001, 0.6100000000000001, 0.6100000000000001, 0.6100000000000001, 0.6100000000000001, 0.6100000000000001, 0.6100000000000001, 0.6100000000000001, 0.6100000000000001, 0.6100000000000001, 0.6100000000000001, 0.6100000000000001, 0.6100000000000001, 0.6100000000000001, 0.6100000000000001, 0.6100000000000001, 0.6100000000000001, 0.6100000000000001, 0.6100000000000001, 0.6100000000000001, 0.6100000000000001, 0.6100000000000001, 0.6100000000000001, 0.6100000000000001, 0.6100000000000001, 0.6100000000000001, 0.6100000000000001, 0.6100000000000001, 0.6100000000000001, 0.6100000000000001, 0.6100000000000001, 0.6100000000000001, 0.6100000000000001, 0.6100000000000001, 0.6100000000000001, 0.6100000000000001, 0.6100000000000001, 0.6100000000000001, 0.6100000000000001, 0.6100000000000001, 0.6100000000000001, 0.6100000000000001, 0.6100000000000001, 0.6100000000000001 ], [ 0.6450000000000002, 0.6450000000000002, 0.6450000000000002, 0.6450000000000002, 0.6450000000000002, 0.6450000000000002, 0.6450000000000002, 0.6450000000000002, 0.6450000000000002, 0.6450000000000002, 0.6450000000000002, 0.6450000000000002, 0.6450000000000002, 0.6450000000000002, 0.6450000000000002, 0.6450000000000002, 0.6450000000000002, 0.6450000000000002, 0.6450000000000002, 0.6450000000000002, 0.6450000000000002, 0.6450000000000002, 0.6450000000000002, 0.6450000000000002, 0.6450000000000002, 0.6450000000000002, 0.6450000000000002, 0.6450000000000002, 0.6450000000000002, 0.6450000000000002, 0.6450000000000002, 0.6450000000000002, 0.6450000000000002, 0.6450000000000002, 0.6450000000000002, 0.6450000000000002, 0.6450000000000002, 0.6450000000000002, 0.6450000000000002, 0.6450000000000002, 0.6450000000000002, 0.6450000000000002, 0.6450000000000002, 0.6450000000000002, 0.6450000000000002, 0.6450000000000002, 0.6450000000000002, 0.6450000000000002, 0.6450000000000002, 0.6450000000000002, 0.6450000000000002, 0.6450000000000002, 0.6450000000000002, 0.6450000000000002, 0.6450000000000002, 0.6450000000000002, 0.6450000000000002, 0.6450000000000002, 0.6450000000000002, 0.6450000000000002, 0.6450000000000002, 0.6450000000000002, 0.6450000000000002, 0.6450000000000002, 0.6450000000000002, 0.6450000000000002, 0.6450000000000002, 0.6450000000000002, 0.6450000000000002, 0.6450000000000002, 0.6450000000000002, 0.6450000000000002, 0.6450000000000002, 0.6450000000000002, 0.6450000000000002, 0.6450000000000002, 0.6450000000000002, 0.6450000000000002, 0.6450000000000002, 0.6450000000000002, 0.6450000000000002, 0.6450000000000002, 0.6450000000000002, 0.6450000000000002, 0.6450000000000002, 0.6450000000000002, 0.6450000000000002, 0.6450000000000002, 0.6450000000000002, 0.6450000000000002, 0.6450000000000002, 0.6450000000000002, 0.6450000000000002, 0.6450000000000002, 0.6450000000000002, 0.6450000000000002, 0.6450000000000002, 0.6450000000000002, 0.6450000000000002, 0.6450000000000002, 0.6450000000000002 ], [ 0.6800000000000002, 0.6800000000000002, 0.6800000000000002, 0.6800000000000002, 0.6800000000000002, 0.6800000000000002, 0.6800000000000002, 0.6800000000000002, 0.6800000000000002, 0.6800000000000002, 0.6800000000000002, 0.6800000000000002, 0.6800000000000002, 0.6800000000000002, 0.6800000000000002, 0.6800000000000002, 0.6800000000000002, 0.6800000000000002, 0.6800000000000002, 0.6800000000000002, 0.6800000000000002, 0.6800000000000002, 0.6800000000000002, 0.6800000000000002, 0.6800000000000002, 0.6800000000000002, 0.6800000000000002, 0.6800000000000002, 0.6800000000000002, 0.6800000000000002, 0.6800000000000002, 0.6800000000000002, 0.6800000000000002, 0.6800000000000002, 0.6800000000000002, 0.6800000000000002, 0.6800000000000002, 0.6800000000000002, 0.6800000000000002, 0.6800000000000002, 0.6800000000000002, 0.6800000000000002, 0.6800000000000002, 0.6800000000000002, 0.6800000000000002, 0.6800000000000002, 0.6800000000000002, 0.6800000000000002, 0.6800000000000002, 0.6800000000000002, 0.6800000000000002, 0.6800000000000002, 0.6800000000000002, 0.6800000000000002, 0.6800000000000002, 0.6800000000000002, 0.6800000000000002, 0.6800000000000002, 0.6800000000000002, 0.6800000000000002, 0.6800000000000002, 0.6800000000000002, 0.6800000000000002, 0.6800000000000002, 0.6800000000000002, 0.6800000000000002, 0.6800000000000002, 0.6800000000000002, 0.6800000000000002, 0.6800000000000002, 0.6800000000000002, 0.6800000000000002, 0.6800000000000002, 0.6800000000000002, 0.6800000000000002, 0.6800000000000002, 0.6800000000000002, 0.6800000000000002, 0.6800000000000002, 0.6800000000000002, 0.6800000000000002, 0.6800000000000002, 0.6800000000000002, 0.6800000000000002, 0.6800000000000002, 0.6800000000000002, 0.6800000000000002, 0.6800000000000002, 0.6800000000000002, 0.6800000000000002, 0.6800000000000002, 0.6800000000000002, 0.6800000000000002, 0.6800000000000002, 0.6800000000000002, 0.6800000000000002, 0.6800000000000002, 0.6800000000000002, 0.6800000000000002, 0.6800000000000002, 0.6800000000000002 ], [ 0.7150000000000001, 0.7150000000000001, 0.7150000000000001, 0.7150000000000001, 0.7150000000000001, 0.7150000000000001, 0.7150000000000001, 0.7150000000000001, 0.7150000000000001, 0.7150000000000001, 0.7150000000000001, 0.7150000000000001, 0.7150000000000001, 0.7150000000000001, 0.7150000000000001, 0.7150000000000001, 0.7150000000000001, 0.7150000000000001, 0.7150000000000001, 0.7150000000000001, 0.7150000000000001, 0.7150000000000001, 0.7150000000000001, 0.7150000000000001, 0.7150000000000001, 0.7150000000000001, 0.7150000000000001, 0.7150000000000001, 0.7150000000000001, 0.7150000000000001, 0.7150000000000001, 0.7150000000000001, 0.7150000000000001, 0.7150000000000001, 0.7150000000000001, 0.7150000000000001, 0.7150000000000001, 0.7150000000000001, 0.7150000000000001, 0.7150000000000001, 0.7150000000000001, 0.7150000000000001, 0.7150000000000001, 0.7150000000000001, 0.7150000000000001, 0.7150000000000001, 0.7150000000000001, 0.7150000000000001, 0.7150000000000001, 0.7150000000000001, 0.7150000000000001, 0.7150000000000001, 0.7150000000000001, 0.7150000000000001, 0.7150000000000001, 0.7150000000000001, 0.7150000000000001, 0.7150000000000001, 0.7150000000000001, 0.7150000000000001, 0.7150000000000001, 0.7150000000000001, 0.7150000000000001, 0.7150000000000001, 0.7150000000000001, 0.7150000000000001, 0.7150000000000001, 0.7150000000000001, 0.7150000000000001, 0.7150000000000001, 0.7150000000000001, 0.7150000000000001, 0.7150000000000001, 0.7150000000000001, 0.7150000000000001, 0.7150000000000001, 0.7150000000000001, 0.7150000000000001, 0.7150000000000001, 0.7150000000000001, 0.7150000000000001, 0.7150000000000001, 0.7150000000000001, 0.7150000000000001, 0.7150000000000001, 0.7150000000000001, 0.7150000000000001, 0.7150000000000001, 0.7150000000000001, 0.7150000000000001, 0.7150000000000001, 0.7150000000000001, 0.7150000000000001, 0.7150000000000001, 0.7150000000000001, 0.7150000000000001, 0.7150000000000001, 0.7150000000000001, 0.7150000000000001, 0.7150000000000001, 0.7150000000000001 ], [ 0.7500000000000002, 0.7500000000000002, 0.7500000000000002, 0.7500000000000002, 0.7500000000000002, 0.7500000000000002, 0.7500000000000002, 0.7500000000000002, 0.7500000000000002, 0.7500000000000002, 0.7500000000000002, 0.7500000000000002, 0.7500000000000002, 0.7500000000000002, 0.7500000000000002, 0.7500000000000002, 0.7500000000000002, 0.7500000000000002, 0.7500000000000002, 0.7500000000000002, 0.7500000000000002, 0.7500000000000002, 0.7500000000000002, 0.7500000000000002, 0.7500000000000002, 0.7500000000000002, 0.7500000000000002, 0.7500000000000002, 0.7500000000000002, 0.7500000000000002, 0.7500000000000002, 0.7500000000000002, 0.7500000000000002, 0.7500000000000002, 0.7500000000000002, 0.7500000000000002, 0.7500000000000002, 0.7500000000000002, 0.7500000000000002, 0.7500000000000002, 0.7500000000000002, 0.7500000000000002, 0.7500000000000002, 0.7500000000000002, 0.7500000000000002, 0.7500000000000002, 0.7500000000000002, 0.7500000000000002, 0.7500000000000002, 0.7500000000000002, 0.7500000000000002, 0.7500000000000002, 0.7500000000000002, 0.7500000000000002, 0.7500000000000002, 0.7500000000000002, 0.7500000000000002, 0.7500000000000002, 0.7500000000000002, 0.7500000000000002, 0.7500000000000002, 0.7500000000000002, 0.7500000000000002, 0.7500000000000002, 0.7500000000000002, 0.7500000000000002, 0.7500000000000002, 0.7500000000000002, 0.7500000000000002, 0.7500000000000002, 0.7500000000000002, 0.7500000000000002, 0.7500000000000002, 0.7500000000000002, 0.7500000000000002, 0.7500000000000002, 0.7500000000000002, 0.7500000000000002, 0.7500000000000002, 0.7500000000000002, 0.7500000000000002, 0.7500000000000002, 0.7500000000000002, 0.7500000000000002, 0.7500000000000002, 0.7500000000000002, 0.7500000000000002, 0.7500000000000002, 0.7500000000000002, 0.7500000000000002, 0.7500000000000002, 0.7500000000000002, 0.7500000000000002, 0.7500000000000002, 0.7500000000000002, 0.7500000000000002, 0.7500000000000002, 0.7500000000000002, 0.7500000000000002, 0.7500000000000002, 0.7500000000000002 ], [ 0.7850000000000001, 0.7850000000000001, 0.7850000000000001, 0.7850000000000001, 0.7850000000000001, 0.7850000000000001, 0.7850000000000001, 0.7850000000000001, 0.7850000000000001, 0.7850000000000001, 0.7850000000000001, 0.7850000000000001, 0.7850000000000001, 0.7850000000000001, 0.7850000000000001, 0.7850000000000001, 0.7850000000000001, 0.7850000000000001, 0.7850000000000001, 0.7850000000000001, 0.7850000000000001, 0.7850000000000001, 0.7850000000000001, 0.7850000000000001, 0.7850000000000001, 0.7850000000000001, 0.7850000000000001, 0.7850000000000001, 0.7850000000000001, 0.7850000000000001, 0.7850000000000001, 0.7850000000000001, 0.7850000000000001, 0.7850000000000001, 0.7850000000000001, 0.7850000000000001, 0.7850000000000001, 0.7850000000000001, 0.7850000000000001, 0.7850000000000001, 0.7850000000000001, 0.7850000000000001, 0.7850000000000001, 0.7850000000000001, 0.7850000000000001, 0.7850000000000001, 0.7850000000000001, 0.7850000000000001, 0.7850000000000001, 0.7850000000000001, 0.7850000000000001, 0.7850000000000001, 0.7850000000000001, 0.7850000000000001, 0.7850000000000001, 0.7850000000000001, 0.7850000000000001, 0.7850000000000001, 0.7850000000000001, 0.7850000000000001, 0.7850000000000001, 0.7850000000000001, 0.7850000000000001, 0.7850000000000001, 0.7850000000000001, 0.7850000000000001, 0.7850000000000001, 0.7850000000000001, 0.7850000000000001, 0.7850000000000001, 0.7850000000000001, 0.7850000000000001, 0.7850000000000001, 0.7850000000000001, 0.7850000000000001, 0.7850000000000001, 0.7850000000000001, 0.7850000000000001, 0.7850000000000001, 0.7850000000000001, 0.7850000000000001, 0.7850000000000001, 0.7850000000000001, 0.7850000000000001, 0.7850000000000001, 0.7850000000000001, 0.7850000000000001, 0.7850000000000001, 0.7850000000000001, 0.7850000000000001, 0.7850000000000001, 0.7850000000000001, 0.7850000000000001, 0.7850000000000001, 0.7850000000000001, 0.7850000000000001, 0.7850000000000001, 0.7850000000000001, 0.7850000000000001, 0.7850000000000001, 0.7850000000000001 ], [ 0.8200000000000003, 0.8200000000000003, 0.8200000000000003, 0.8200000000000003, 0.8200000000000003, 0.8200000000000003, 0.8200000000000003, 0.8200000000000003, 0.8200000000000003, 0.8200000000000003, 0.8200000000000003, 0.8200000000000003, 0.8200000000000003, 0.8200000000000003, 0.8200000000000003, 0.8200000000000003, 0.8200000000000003, 0.8200000000000003, 0.8200000000000003, 0.8200000000000003, 0.8200000000000003, 0.8200000000000003, 0.8200000000000003, 0.8200000000000003, 0.8200000000000003, 0.8200000000000003, 0.8200000000000003, 0.8200000000000003, 0.8200000000000003, 0.8200000000000003, 0.8200000000000003, 0.8200000000000003, 0.8200000000000003, 0.8200000000000003, 0.8200000000000003, 0.8200000000000003, 0.8200000000000003, 0.8200000000000003, 0.8200000000000003, 0.8200000000000003, 0.8200000000000003, 0.8200000000000003, 0.8200000000000003, 0.8200000000000003, 0.8200000000000003, 0.8200000000000003, 0.8200000000000003, 0.8200000000000003, 0.8200000000000003, 0.8200000000000003, 0.8200000000000003, 0.8200000000000003, 0.8200000000000003, 0.8200000000000003, 0.8200000000000003, 0.8200000000000003, 0.8200000000000003, 0.8200000000000003, 0.8200000000000003, 0.8200000000000003, 0.8200000000000003, 0.8200000000000003, 0.8200000000000003, 0.8200000000000003, 0.8200000000000003, 0.8200000000000003, 0.8200000000000003, 0.8200000000000003, 0.8200000000000003, 0.8200000000000003, 0.8200000000000003, 0.8200000000000003, 0.8200000000000003, 0.8200000000000003, 0.8200000000000003, 0.8200000000000003, 0.8200000000000003, 0.8200000000000003, 0.8200000000000003, 0.8200000000000003, 0.8200000000000003, 0.8200000000000003, 0.8200000000000003, 0.8200000000000003, 0.8200000000000003, 0.8200000000000003, 0.8200000000000003, 0.8200000000000003, 0.8200000000000003, 0.8200000000000003, 0.8200000000000003, 0.8200000000000003, 0.8200000000000003, 0.8200000000000003, 0.8200000000000003, 0.8200000000000003, 0.8200000000000003, 0.8200000000000003, 0.8200000000000003, 0.8200000000000003, 0.8200000000000003 ], [ 0.8550000000000002, 0.8550000000000002, 0.8550000000000002, 0.8550000000000002, 0.8550000000000002, 0.8550000000000002, 0.8550000000000002, 0.8550000000000002, 0.8550000000000002, 0.8550000000000002, 0.8550000000000002, 0.8550000000000002, 0.8550000000000002, 0.8550000000000002, 0.8550000000000002, 0.8550000000000002, 0.8550000000000002, 0.8550000000000002, 0.8550000000000002, 0.8550000000000002, 0.8550000000000002, 0.8550000000000002, 0.8550000000000002, 0.8550000000000002, 0.8550000000000002, 0.8550000000000002, 0.8550000000000002, 0.8550000000000002, 0.8550000000000002, 0.8550000000000002, 0.8550000000000002, 0.8550000000000002, 0.8550000000000002, 0.8550000000000002, 0.8550000000000002, 0.8550000000000002, 0.8550000000000002, 0.8550000000000002, 0.8550000000000002, 0.8550000000000002, 0.8550000000000002, 0.8550000000000002, 0.8550000000000002, 0.8550000000000002, 0.8550000000000002, 0.8550000000000002, 0.8550000000000002, 0.8550000000000002, 0.8550000000000002, 0.8550000000000002, 0.8550000000000002, 0.8550000000000002, 0.8550000000000002, 0.8550000000000002, 0.8550000000000002, 0.8550000000000002, 0.8550000000000002, 0.8550000000000002, 0.8550000000000002, 0.8550000000000002, 0.8550000000000002, 0.8550000000000002, 0.8550000000000002, 0.8550000000000002, 0.8550000000000002, 0.8550000000000002, 0.8550000000000002, 0.8550000000000002, 0.8550000000000002, 0.8550000000000002, 0.8550000000000002, 0.8550000000000002, 0.8550000000000002, 0.8550000000000002, 0.8550000000000002, 0.8550000000000002, 0.8550000000000002, 0.8550000000000002, 0.8550000000000002, 0.8550000000000002, 0.8550000000000002, 0.8550000000000002, 0.8550000000000002, 0.8550000000000002, 0.8550000000000002, 0.8550000000000002, 0.8550000000000002, 0.8550000000000002, 0.8550000000000002, 0.8550000000000002, 0.8550000000000002, 0.8550000000000002, 0.8550000000000002, 0.8550000000000002, 0.8550000000000002, 0.8550000000000002, 0.8550000000000002, 0.8550000000000002, 0.8550000000000002, 0.8550000000000002, 0.8550000000000002 ], [ 0.8900000000000001, 0.8900000000000001, 0.8900000000000001, 0.8900000000000001, 0.8900000000000001, 0.8900000000000001, 0.8900000000000001, 0.8900000000000001, 0.8900000000000001, 0.8900000000000001, 0.8900000000000001, 0.8900000000000001, 0.8900000000000001, 0.8900000000000001, 0.8900000000000001, 0.8900000000000001, 0.8900000000000001, 0.8900000000000001, 0.8900000000000001, 0.8900000000000001, 0.8900000000000001, 0.8900000000000001, 0.8900000000000001, 0.8900000000000001, 0.8900000000000001, 0.8900000000000001, 0.8900000000000001, 0.8900000000000001, 0.8900000000000001, 0.8900000000000001, 0.8900000000000001, 0.8900000000000001, 0.8900000000000001, 0.8900000000000001, 0.8900000000000001, 0.8900000000000001, 0.8900000000000001, 0.8900000000000001, 0.8900000000000001, 0.8900000000000001, 0.8900000000000001, 0.8900000000000001, 0.8900000000000001, 0.8900000000000001, 0.8900000000000001, 0.8900000000000001, 0.8900000000000001, 0.8900000000000001, 0.8900000000000001, 0.8900000000000001, 0.8900000000000001, 0.8900000000000001, 0.8900000000000001, 0.8900000000000001, 0.8900000000000001, 0.8900000000000001, 0.8900000000000001, 0.8900000000000001, 0.8900000000000001, 0.8900000000000001, 0.8900000000000001, 0.8900000000000001, 0.8900000000000001, 0.8900000000000001, 0.8900000000000001, 0.8900000000000001, 0.8900000000000001, 0.8900000000000001, 0.8900000000000001, 0.8900000000000001, 0.8900000000000001, 0.8900000000000001, 0.8900000000000001, 0.8900000000000001, 0.8900000000000001, 0.8900000000000001, 0.8900000000000001, 0.8900000000000001, 0.8900000000000001, 0.8900000000000001, 0.8900000000000001, 0.8900000000000001, 0.8900000000000001, 0.8900000000000001, 0.8900000000000001, 0.8900000000000001, 0.8900000000000001, 0.8900000000000001, 0.8900000000000001, 0.8900000000000001, 0.8900000000000001, 0.8900000000000001, 0.8900000000000001, 0.8900000000000001, 0.8900000000000001, 0.8900000000000001, 0.8900000000000001, 0.8900000000000001, 0.8900000000000001, 0.8900000000000001, 0.8900000000000001 ], [ 0.9250000000000003, 0.9250000000000003, 0.9250000000000003, 0.9250000000000003, 0.9250000000000003, 0.9250000000000003, 0.9250000000000003, 0.9250000000000003, 0.9250000000000003, 0.9250000000000003, 0.9250000000000003, 0.9250000000000003, 0.9250000000000003, 0.9250000000000003, 0.9250000000000003, 0.9250000000000003, 0.9250000000000003, 0.9250000000000003, 0.9250000000000003, 0.9250000000000003, 0.9250000000000003, 0.9250000000000003, 0.9250000000000003, 0.9250000000000003, 0.9250000000000003, 0.9250000000000003, 0.9250000000000003, 0.9250000000000003, 0.9250000000000003, 0.9250000000000003, 0.9250000000000003, 0.9250000000000003, 0.9250000000000003, 0.9250000000000003, 0.9250000000000003, 0.9250000000000003, 0.9250000000000003, 0.9250000000000003, 0.9250000000000003, 0.9250000000000003, 0.9250000000000003, 0.9250000000000003, 0.9250000000000003, 0.9250000000000003, 0.9250000000000003, 0.9250000000000003, 0.9250000000000003, 0.9250000000000003, 0.9250000000000003, 0.9250000000000003, 0.9250000000000003, 0.9250000000000003, 0.9250000000000003, 0.9250000000000003, 0.9250000000000003, 0.9250000000000003, 0.9250000000000003, 0.9250000000000003, 0.9250000000000003, 0.9250000000000003, 0.9250000000000003, 0.9250000000000003, 0.9250000000000003, 0.9250000000000003, 0.9250000000000003, 0.9250000000000003, 0.9250000000000003, 0.9250000000000003, 0.9250000000000003, 0.9250000000000003, 0.9250000000000003, 0.9250000000000003, 0.9250000000000003, 0.9250000000000003, 0.9250000000000003, 0.9250000000000003, 0.9250000000000003, 0.9250000000000003, 0.9250000000000003, 0.9250000000000003, 0.9250000000000003, 0.9250000000000003, 0.9250000000000003, 0.9250000000000003, 0.9250000000000003, 0.9250000000000003, 0.9250000000000003, 0.9250000000000003, 0.9250000000000003, 0.9250000000000003, 0.9250000000000003, 0.9250000000000003, 0.9250000000000003, 0.9250000000000003, 0.9250000000000003, 0.9250000000000003, 0.9250000000000003, 0.9250000000000003, 0.9250000000000003, 0.9250000000000003, 0.9250000000000003 ], [ 0.9600000000000002, 0.9600000000000002, 0.9600000000000002, 0.9600000000000002, 0.9600000000000002, 0.9600000000000002, 0.9600000000000002, 0.9600000000000002, 0.9600000000000002, 0.9600000000000002, 0.9600000000000002, 0.9600000000000002, 0.9600000000000002, 0.9600000000000002, 0.9600000000000002, 0.9600000000000002, 0.9600000000000002, 0.9600000000000002, 0.9600000000000002, 0.9600000000000002, 0.9600000000000002, 0.9600000000000002, 0.9600000000000002, 0.9600000000000002, 0.9600000000000002, 0.9600000000000002, 0.9600000000000002, 0.9600000000000002, 0.9600000000000002, 0.9600000000000002, 0.9600000000000002, 0.9600000000000002, 0.9600000000000002, 0.9600000000000002, 0.9600000000000002, 0.9600000000000002, 0.9600000000000002, 0.9600000000000002, 0.9600000000000002, 0.9600000000000002, 0.9600000000000002, 0.9600000000000002, 0.9600000000000002, 0.9600000000000002, 0.9600000000000002, 0.9600000000000002, 0.9600000000000002, 0.9600000000000002, 0.9600000000000002, 0.9600000000000002, 0.9600000000000002, 0.9600000000000002, 0.9600000000000002, 0.9600000000000002, 0.9600000000000002, 0.9600000000000002, 0.9600000000000002, 0.9600000000000002, 0.9600000000000002, 0.9600000000000002, 0.9600000000000002, 0.9600000000000002, 0.9600000000000002, 0.9600000000000002, 0.9600000000000002, 0.9600000000000002, 0.9600000000000002, 0.9600000000000002, 0.9600000000000002, 0.9600000000000002, 0.9600000000000002, 0.9600000000000002, 0.9600000000000002, 0.9600000000000002, 0.9600000000000002, 0.9600000000000002, 0.9600000000000002, 0.9600000000000002, 0.9600000000000002, 0.9600000000000002, 0.9600000000000002, 0.9600000000000002, 0.9600000000000002, 0.9600000000000002, 0.9600000000000002, 0.9600000000000002, 0.9600000000000002, 0.9600000000000002, 0.9600000000000002, 0.9600000000000002, 0.9600000000000002, 0.9600000000000002, 0.9600000000000002, 0.9600000000000002, 0.9600000000000002, 0.9600000000000002, 0.9600000000000002, 0.9600000000000002, 0.9600000000000002, 0.9600000000000002, 0.9600000000000002 ], [ 0.9950000000000001, 0.9950000000000001, 0.9950000000000001, 0.9950000000000001, 0.9950000000000001, 0.9950000000000001, 0.9950000000000001, 0.9950000000000001, 0.9950000000000001, 0.9950000000000001, 0.9950000000000001, 0.9950000000000001, 0.9950000000000001, 0.9950000000000001, 0.9950000000000001, 0.9950000000000001, 0.9950000000000001, 0.9950000000000001, 0.9950000000000001, 0.9950000000000001, 0.9950000000000001, 0.9950000000000001, 0.9950000000000001, 0.9950000000000001, 0.9950000000000001, 0.9950000000000001, 0.9950000000000001, 0.9950000000000001, 0.9950000000000001, 0.9950000000000001, 0.9950000000000001, 0.9950000000000001, 0.9950000000000001, 0.9950000000000001, 0.9950000000000001, 0.9950000000000001, 0.9950000000000001, 0.9950000000000001, 0.9950000000000001, 0.9950000000000001, 0.9950000000000001, 0.9950000000000001, 0.9950000000000001, 0.9950000000000001, 0.9950000000000001, 0.9950000000000001, 0.9950000000000001, 0.9950000000000001, 0.9950000000000001, 0.9950000000000001, 0.9950000000000001, 0.9950000000000001, 0.9950000000000001, 0.9950000000000001, 0.9950000000000001, 0.9950000000000001, 0.9950000000000001, 0.9950000000000001, 0.9950000000000001, 0.9950000000000001, 0.9950000000000001, 0.9950000000000001, 0.9950000000000001, 0.9950000000000001, 0.9950000000000001, 0.9950000000000001, 0.9950000000000001, 0.9950000000000001, 0.9950000000000001, 0.9950000000000001, 0.9950000000000001, 0.9950000000000001, 0.9950000000000001, 0.9950000000000001, 0.9950000000000001, 0.9950000000000001, 0.9950000000000001, 0.9950000000000001, 0.9950000000000001, 0.9950000000000001, 0.9950000000000001, 0.9950000000000001, 0.9950000000000001, 0.9950000000000001, 0.9950000000000001, 0.9950000000000001, 0.9950000000000001, 0.9950000000000001, 0.9950000000000001, 0.9950000000000001, 0.9950000000000001, 0.9950000000000001, 0.9950000000000001, 0.9950000000000001, 0.9950000000000001, 0.9950000000000001, 0.9950000000000001, 0.9950000000000001, 0.9950000000000001, 0.9950000000000001, 0.9950000000000001 ], [ 1.0300000000000002, 1.0300000000000002, 1.0300000000000002, 1.0300000000000002, 1.0300000000000002, 1.0300000000000002, 1.0300000000000002, 1.0300000000000002, 1.0300000000000002, 1.0300000000000002, 1.0300000000000002, 1.0300000000000002, 1.0300000000000002, 1.0300000000000002, 1.0300000000000002, 1.0300000000000002, 1.0300000000000002, 1.0300000000000002, 1.0300000000000002, 1.0300000000000002, 1.0300000000000002, 1.0300000000000002, 1.0300000000000002, 1.0300000000000002, 1.0300000000000002, 1.0300000000000002, 1.0300000000000002, 1.0300000000000002, 1.0300000000000002, 1.0300000000000002, 1.0300000000000002, 1.0300000000000002, 1.0300000000000002, 1.0300000000000002, 1.0300000000000002, 1.0300000000000002, 1.0300000000000002, 1.0300000000000002, 1.0300000000000002, 1.0300000000000002, 1.0300000000000002, 1.0300000000000002, 1.0300000000000002, 1.0300000000000002, 1.0300000000000002, 1.0300000000000002, 1.0300000000000002, 1.0300000000000002, 1.0300000000000002, 1.0300000000000002, 1.0300000000000002, 1.0300000000000002, 1.0300000000000002, 1.0300000000000002, 1.0300000000000002, 1.0300000000000002, 1.0300000000000002, 1.0300000000000002, 1.0300000000000002, 1.0300000000000002, 1.0300000000000002, 1.0300000000000002, 1.0300000000000002, 1.0300000000000002, 1.0300000000000002, 1.0300000000000002, 1.0300000000000002, 1.0300000000000002, 1.0300000000000002, 1.0300000000000002, 1.0300000000000002, 1.0300000000000002, 1.0300000000000002, 1.0300000000000002, 1.0300000000000002, 1.0300000000000002, 1.0300000000000002, 1.0300000000000002, 1.0300000000000002, 1.0300000000000002, 1.0300000000000002, 1.0300000000000002, 1.0300000000000002, 1.0300000000000002, 1.0300000000000002, 1.0300000000000002, 1.0300000000000002, 1.0300000000000002, 1.0300000000000002, 1.0300000000000002, 1.0300000000000002, 1.0300000000000002, 1.0300000000000002, 1.0300000000000002, 1.0300000000000002, 1.0300000000000002, 1.0300000000000002, 1.0300000000000002, 1.0300000000000002, 1.0300000000000002, 1.0300000000000002 ], [ 1.0650000000000004, 1.0650000000000004, 1.0650000000000004, 1.0650000000000004, 1.0650000000000004, 1.0650000000000004, 1.0650000000000004, 1.0650000000000004, 1.0650000000000004, 1.0650000000000004, 1.0650000000000004, 1.0650000000000004, 1.0650000000000004, 1.0650000000000004, 1.0650000000000004, 1.0650000000000004, 1.0650000000000004, 1.0650000000000004, 1.0650000000000004, 1.0650000000000004, 1.0650000000000004, 1.0650000000000004, 1.0650000000000004, 1.0650000000000004, 1.0650000000000004, 1.0650000000000004, 1.0650000000000004, 1.0650000000000004, 1.0650000000000004, 1.0650000000000004, 1.0650000000000004, 1.0650000000000004, 1.0650000000000004, 1.0650000000000004, 1.0650000000000004, 1.0650000000000004, 1.0650000000000004, 1.0650000000000004, 1.0650000000000004, 1.0650000000000004, 1.0650000000000004, 1.0650000000000004, 1.0650000000000004, 1.0650000000000004, 1.0650000000000004, 1.0650000000000004, 1.0650000000000004, 1.0650000000000004, 1.0650000000000004, 1.0650000000000004, 1.0650000000000004, 1.0650000000000004, 1.0650000000000004, 1.0650000000000004, 1.0650000000000004, 1.0650000000000004, 1.0650000000000004, 1.0650000000000004, 1.0650000000000004, 1.0650000000000004, 1.0650000000000004, 1.0650000000000004, 1.0650000000000004, 1.0650000000000004, 1.0650000000000004, 1.0650000000000004, 1.0650000000000004, 1.0650000000000004, 1.0650000000000004, 1.0650000000000004, 1.0650000000000004, 1.0650000000000004, 1.0650000000000004, 1.0650000000000004, 1.0650000000000004, 1.0650000000000004, 1.0650000000000004, 1.0650000000000004, 1.0650000000000004, 1.0650000000000004, 1.0650000000000004, 1.0650000000000004, 1.0650000000000004, 1.0650000000000004, 1.0650000000000004, 1.0650000000000004, 1.0650000000000004, 1.0650000000000004, 1.0650000000000004, 1.0650000000000004, 1.0650000000000004, 1.0650000000000004, 1.0650000000000004, 1.0650000000000004, 1.0650000000000004, 1.0650000000000004, 1.0650000000000004, 1.0650000000000004, 1.0650000000000004, 1.0650000000000004, 1.0650000000000004 ], [ 1.1, 1.1, 1.1, 1.1, 1.1, 1.1, 1.1, 1.1, 1.1, 1.1, 1.1, 1.1, 1.1, 1.1, 1.1, 1.1, 1.1, 1.1, 1.1, 1.1, 1.1, 1.1, 1.1, 1.1, 1.1, 1.1, 1.1, 1.1, 1.1, 1.1, 1.1, 1.1, 1.1, 1.1, 1.1, 1.1, 1.1, 1.1, 1.1, 1.1, 1.1, 1.1, 1.1, 1.1, 1.1, 1.1, 1.1, 1.1, 1.1, 1.1, 1.1, 1.1, 1.1, 1.1, 1.1, 1.1, 1.1, 1.1, 1.1, 1.1, 1.1, 1.1, 1.1, 1.1, 1.1, 1.1, 1.1, 1.1, 1.1, 1.1, 1.1, 1.1, 1.1, 1.1, 1.1, 1.1, 1.1, 1.1, 1.1, 1.1, 1.1, 1.1, 1.1, 1.1, 1.1, 1.1, 1.1, 1.1, 1.1, 1.1, 1.1, 1.1, 1.1, 1.1, 1.1, 1.1, 1.1, 1.1, 1.1, 1.1, 1.1 ], [ 1.1350000000000002, 1.1350000000000002, 1.1350000000000002, 1.1350000000000002, 1.1350000000000002, 1.1350000000000002, 1.1350000000000002, 1.1350000000000002, 1.1350000000000002, 1.1350000000000002, 1.1350000000000002, 1.1350000000000002, 1.1350000000000002, 1.1350000000000002, 1.1350000000000002, 1.1350000000000002, 1.1350000000000002, 1.1350000000000002, 1.1350000000000002, 1.1350000000000002, 1.1350000000000002, 1.1350000000000002, 1.1350000000000002, 1.1350000000000002, 1.1350000000000002, 1.1350000000000002, 1.1350000000000002, 1.1350000000000002, 1.1350000000000002, 1.1350000000000002, 1.1350000000000002, 1.1350000000000002, 1.1350000000000002, 1.1350000000000002, 1.1350000000000002, 1.1350000000000002, 1.1350000000000002, 1.1350000000000002, 1.1350000000000002, 1.1350000000000002, 1.1350000000000002, 1.1350000000000002, 1.1350000000000002, 1.1350000000000002, 1.1350000000000002, 1.1350000000000002, 1.1350000000000002, 1.1350000000000002, 1.1350000000000002, 1.1350000000000002, 1.1350000000000002, 1.1350000000000002, 1.1350000000000002, 1.1350000000000002, 1.1350000000000002, 1.1350000000000002, 1.1350000000000002, 1.1350000000000002, 1.1350000000000002, 1.1350000000000002, 1.1350000000000002, 1.1350000000000002, 1.1350000000000002, 1.1350000000000002, 1.1350000000000002, 1.1350000000000002, 1.1350000000000002, 1.1350000000000002, 1.1350000000000002, 1.1350000000000002, 1.1350000000000002, 1.1350000000000002, 1.1350000000000002, 1.1350000000000002, 1.1350000000000002, 1.1350000000000002, 1.1350000000000002, 1.1350000000000002, 1.1350000000000002, 1.1350000000000002, 1.1350000000000002, 1.1350000000000002, 1.1350000000000002, 1.1350000000000002, 1.1350000000000002, 1.1350000000000002, 1.1350000000000002, 1.1350000000000002, 1.1350000000000002, 1.1350000000000002, 1.1350000000000002, 1.1350000000000002, 1.1350000000000002, 1.1350000000000002, 1.1350000000000002, 1.1350000000000002, 1.1350000000000002, 1.1350000000000002, 1.1350000000000002, 1.1350000000000002, 1.1350000000000002 ], [ 1.1700000000000004, 1.1700000000000004, 1.1700000000000004, 1.1700000000000004, 1.1700000000000004, 1.1700000000000004, 1.1700000000000004, 1.1700000000000004, 1.1700000000000004, 1.1700000000000004, 1.1700000000000004, 1.1700000000000004, 1.1700000000000004, 1.1700000000000004, 1.1700000000000004, 1.1700000000000004, 1.1700000000000004, 1.1700000000000004, 1.1700000000000004, 1.1700000000000004, 1.1700000000000004, 1.1700000000000004, 1.1700000000000004, 1.1700000000000004, 1.1700000000000004, 1.1700000000000004, 1.1700000000000004, 1.1700000000000004, 1.1700000000000004, 1.1700000000000004, 1.1700000000000004, 1.1700000000000004, 1.1700000000000004, 1.1700000000000004, 1.1700000000000004, 1.1700000000000004, 1.1700000000000004, 1.1700000000000004, 1.1700000000000004, 1.1700000000000004, 1.1700000000000004, 1.1700000000000004, 1.1700000000000004, 1.1700000000000004, 1.1700000000000004, 1.1700000000000004, 1.1700000000000004, 1.1700000000000004, 1.1700000000000004, 1.1700000000000004, 1.1700000000000004, 1.1700000000000004, 1.1700000000000004, 1.1700000000000004, 1.1700000000000004, 1.1700000000000004, 1.1700000000000004, 1.1700000000000004, 1.1700000000000004, 1.1700000000000004, 1.1700000000000004, 1.1700000000000004, 1.1700000000000004, 1.1700000000000004, 1.1700000000000004, 1.1700000000000004, 1.1700000000000004, 1.1700000000000004, 1.1700000000000004, 1.1700000000000004, 1.1700000000000004, 1.1700000000000004, 1.1700000000000004, 1.1700000000000004, 1.1700000000000004, 1.1700000000000004, 1.1700000000000004, 1.1700000000000004, 1.1700000000000004, 1.1700000000000004, 1.1700000000000004, 1.1700000000000004, 1.1700000000000004, 1.1700000000000004, 1.1700000000000004, 1.1700000000000004, 1.1700000000000004, 1.1700000000000004, 1.1700000000000004, 1.1700000000000004, 1.1700000000000004, 1.1700000000000004, 1.1700000000000004, 1.1700000000000004, 1.1700000000000004, 1.1700000000000004, 1.1700000000000004, 1.1700000000000004, 1.1700000000000004, 1.1700000000000004, 1.1700000000000004 ], [ 1.205, 1.205, 1.205, 1.205, 1.205, 1.205, 1.205, 1.205, 1.205, 1.205, 1.205, 1.205, 1.205, 1.205, 1.205, 1.205, 1.205, 1.205, 1.205, 1.205, 1.205, 1.205, 1.205, 1.205, 1.205, 1.205, 1.205, 1.205, 1.205, 1.205, 1.205, 1.205, 1.205, 1.205, 1.205, 1.205, 1.205, 1.205, 1.205, 1.205, 1.205, 1.205, 1.205, 1.205, 1.205, 1.205, 1.205, 1.205, 1.205, 1.205, 1.205, 1.205, 1.205, 1.205, 1.205, 1.205, 1.205, 1.205, 1.205, 1.205, 1.205, 1.205, 1.205, 1.205, 1.205, 1.205, 1.205, 1.205, 1.205, 1.205, 1.205, 1.205, 1.205, 1.205, 1.205, 1.205, 1.205, 1.205, 1.205, 1.205, 1.205, 1.205, 1.205, 1.205, 1.205, 1.205, 1.205, 1.205, 1.205, 1.205, 1.205, 1.205, 1.205, 1.205, 1.205, 1.205, 1.205, 1.205, 1.205, 1.205, 1.205 ], [ 1.2400000000000002, 1.2400000000000002, 1.2400000000000002, 1.2400000000000002, 1.2400000000000002, 1.2400000000000002, 1.2400000000000002, 1.2400000000000002, 1.2400000000000002, 1.2400000000000002, 1.2400000000000002, 1.2400000000000002, 1.2400000000000002, 1.2400000000000002, 1.2400000000000002, 1.2400000000000002, 1.2400000000000002, 1.2400000000000002, 1.2400000000000002, 1.2400000000000002, 1.2400000000000002, 1.2400000000000002, 1.2400000000000002, 1.2400000000000002, 1.2400000000000002, 1.2400000000000002, 1.2400000000000002, 1.2400000000000002, 1.2400000000000002, 1.2400000000000002, 1.2400000000000002, 1.2400000000000002, 1.2400000000000002, 1.2400000000000002, 1.2400000000000002, 1.2400000000000002, 1.2400000000000002, 1.2400000000000002, 1.2400000000000002, 1.2400000000000002, 1.2400000000000002, 1.2400000000000002, 1.2400000000000002, 1.2400000000000002, 1.2400000000000002, 1.2400000000000002, 1.2400000000000002, 1.2400000000000002, 1.2400000000000002, 1.2400000000000002, 1.2400000000000002, 1.2400000000000002, 1.2400000000000002, 1.2400000000000002, 1.2400000000000002, 1.2400000000000002, 1.2400000000000002, 1.2400000000000002, 1.2400000000000002, 1.2400000000000002, 1.2400000000000002, 1.2400000000000002, 1.2400000000000002, 1.2400000000000002, 1.2400000000000002, 1.2400000000000002, 1.2400000000000002, 1.2400000000000002, 1.2400000000000002, 1.2400000000000002, 1.2400000000000002, 1.2400000000000002, 1.2400000000000002, 1.2400000000000002, 1.2400000000000002, 1.2400000000000002, 1.2400000000000002, 1.2400000000000002, 1.2400000000000002, 1.2400000000000002, 1.2400000000000002, 1.2400000000000002, 1.2400000000000002, 1.2400000000000002, 1.2400000000000002, 1.2400000000000002, 1.2400000000000002, 1.2400000000000002, 1.2400000000000002, 1.2400000000000002, 1.2400000000000002, 1.2400000000000002, 1.2400000000000002, 1.2400000000000002, 1.2400000000000002, 1.2400000000000002, 1.2400000000000002, 1.2400000000000002, 1.2400000000000002, 1.2400000000000002, 1.2400000000000002 ], [ 1.2750000000000004, 1.2750000000000004, 1.2750000000000004, 1.2750000000000004, 1.2750000000000004, 1.2750000000000004, 1.2750000000000004, 1.2750000000000004, 1.2750000000000004, 1.2750000000000004, 1.2750000000000004, 1.2750000000000004, 1.2750000000000004, 1.2750000000000004, 1.2750000000000004, 1.2750000000000004, 1.2750000000000004, 1.2750000000000004, 1.2750000000000004, 1.2750000000000004, 1.2750000000000004, 1.2750000000000004, 1.2750000000000004, 1.2750000000000004, 1.2750000000000004, 1.2750000000000004, 1.2750000000000004, 1.2750000000000004, 1.2750000000000004, 1.2750000000000004, 1.2750000000000004, 1.2750000000000004, 1.2750000000000004, 1.2750000000000004, 1.2750000000000004, 1.2750000000000004, 1.2750000000000004, 1.2750000000000004, 1.2750000000000004, 1.2750000000000004, 1.2750000000000004, 1.2750000000000004, 1.2750000000000004, 1.2750000000000004, 1.2750000000000004, 1.2750000000000004, 1.2750000000000004, 1.2750000000000004, 1.2750000000000004, 1.2750000000000004, 1.2750000000000004, 1.2750000000000004, 1.2750000000000004, 1.2750000000000004, 1.2750000000000004, 1.2750000000000004, 1.2750000000000004, 1.2750000000000004, 1.2750000000000004, 1.2750000000000004, 1.2750000000000004, 1.2750000000000004, 1.2750000000000004, 1.2750000000000004, 1.2750000000000004, 1.2750000000000004, 1.2750000000000004, 1.2750000000000004, 1.2750000000000004, 1.2750000000000004, 1.2750000000000004, 1.2750000000000004, 1.2750000000000004, 1.2750000000000004, 1.2750000000000004, 1.2750000000000004, 1.2750000000000004, 1.2750000000000004, 1.2750000000000004, 1.2750000000000004, 1.2750000000000004, 1.2750000000000004, 1.2750000000000004, 1.2750000000000004, 1.2750000000000004, 1.2750000000000004, 1.2750000000000004, 1.2750000000000004, 1.2750000000000004, 1.2750000000000004, 1.2750000000000004, 1.2750000000000004, 1.2750000000000004, 1.2750000000000004, 1.2750000000000004, 1.2750000000000004, 1.2750000000000004, 1.2750000000000004, 1.2750000000000004, 1.2750000000000004, 1.2750000000000004 ], [ 1.31, 1.31, 1.31, 1.31, 1.31, 1.31, 1.31, 1.31, 1.31, 1.31, 1.31, 1.31, 1.31, 1.31, 1.31, 1.31, 1.31, 1.31, 1.31, 1.31, 1.31, 1.31, 1.31, 1.31, 1.31, 1.31, 1.31, 1.31, 1.31, 1.31, 1.31, 1.31, 1.31, 1.31, 1.31, 1.31, 1.31, 1.31, 1.31, 1.31, 1.31, 1.31, 1.31, 1.31, 1.31, 1.31, 1.31, 1.31, 1.31, 1.31, 1.31, 1.31, 1.31, 1.31, 1.31, 1.31, 1.31, 1.31, 1.31, 1.31, 1.31, 1.31, 1.31, 1.31, 1.31, 1.31, 1.31, 1.31, 1.31, 1.31, 1.31, 1.31, 1.31, 1.31, 1.31, 1.31, 1.31, 1.31, 1.31, 1.31, 1.31, 1.31, 1.31, 1.31, 1.31, 1.31, 1.31, 1.31, 1.31, 1.31, 1.31, 1.31, 1.31, 1.31, 1.31, 1.31, 1.31, 1.31, 1.31, 1.31, 1.31 ], [ 1.3450000000000002, 1.3450000000000002, 1.3450000000000002, 1.3450000000000002, 1.3450000000000002, 1.3450000000000002, 1.3450000000000002, 1.3450000000000002, 1.3450000000000002, 1.3450000000000002, 1.3450000000000002, 1.3450000000000002, 1.3450000000000002, 1.3450000000000002, 1.3450000000000002, 1.3450000000000002, 1.3450000000000002, 1.3450000000000002, 1.3450000000000002, 1.3450000000000002, 1.3450000000000002, 1.3450000000000002, 1.3450000000000002, 1.3450000000000002, 1.3450000000000002, 1.3450000000000002, 1.3450000000000002, 1.3450000000000002, 1.3450000000000002, 1.3450000000000002, 1.3450000000000002, 1.3450000000000002, 1.3450000000000002, 1.3450000000000002, 1.3450000000000002, 1.3450000000000002, 1.3450000000000002, 1.3450000000000002, 1.3450000000000002, 1.3450000000000002, 1.3450000000000002, 1.3450000000000002, 1.3450000000000002, 1.3450000000000002, 1.3450000000000002, 1.3450000000000002, 1.3450000000000002, 1.3450000000000002, 1.3450000000000002, 1.3450000000000002, 1.3450000000000002, 1.3450000000000002, 1.3450000000000002, 1.3450000000000002, 1.3450000000000002, 1.3450000000000002, 1.3450000000000002, 1.3450000000000002, 1.3450000000000002, 1.3450000000000002, 1.3450000000000002, 1.3450000000000002, 1.3450000000000002, 1.3450000000000002, 1.3450000000000002, 1.3450000000000002, 1.3450000000000002, 1.3450000000000002, 1.3450000000000002, 1.3450000000000002, 1.3450000000000002, 1.3450000000000002, 1.3450000000000002, 1.3450000000000002, 1.3450000000000002, 1.3450000000000002, 1.3450000000000002, 1.3450000000000002, 1.3450000000000002, 1.3450000000000002, 1.3450000000000002, 1.3450000000000002, 1.3450000000000002, 1.3450000000000002, 1.3450000000000002, 1.3450000000000002, 1.3450000000000002, 1.3450000000000002, 1.3450000000000002, 1.3450000000000002, 1.3450000000000002, 1.3450000000000002, 1.3450000000000002, 1.3450000000000002, 1.3450000000000002, 1.3450000000000002, 1.3450000000000002, 1.3450000000000002, 1.3450000000000002, 1.3450000000000002, 1.3450000000000002 ], [ 1.3800000000000003, 1.3800000000000003, 1.3800000000000003, 1.3800000000000003, 1.3800000000000003, 1.3800000000000003, 1.3800000000000003, 1.3800000000000003, 1.3800000000000003, 1.3800000000000003, 1.3800000000000003, 1.3800000000000003, 1.3800000000000003, 1.3800000000000003, 1.3800000000000003, 1.3800000000000003, 1.3800000000000003, 1.3800000000000003, 1.3800000000000003, 1.3800000000000003, 1.3800000000000003, 1.3800000000000003, 1.3800000000000003, 1.3800000000000003, 1.3800000000000003, 1.3800000000000003, 1.3800000000000003, 1.3800000000000003, 1.3800000000000003, 1.3800000000000003, 1.3800000000000003, 1.3800000000000003, 1.3800000000000003, 1.3800000000000003, 1.3800000000000003, 1.3800000000000003, 1.3800000000000003, 1.3800000000000003, 1.3800000000000003, 1.3800000000000003, 1.3800000000000003, 1.3800000000000003, 1.3800000000000003, 1.3800000000000003, 1.3800000000000003, 1.3800000000000003, 1.3800000000000003, 1.3800000000000003, 1.3800000000000003, 1.3800000000000003, 1.3800000000000003, 1.3800000000000003, 1.3800000000000003, 1.3800000000000003, 1.3800000000000003, 1.3800000000000003, 1.3800000000000003, 1.3800000000000003, 1.3800000000000003, 1.3800000000000003, 1.3800000000000003, 1.3800000000000003, 1.3800000000000003, 1.3800000000000003, 1.3800000000000003, 1.3800000000000003, 1.3800000000000003, 1.3800000000000003, 1.3800000000000003, 1.3800000000000003, 1.3800000000000003, 1.3800000000000003, 1.3800000000000003, 1.3800000000000003, 1.3800000000000003, 1.3800000000000003, 1.3800000000000003, 1.3800000000000003, 1.3800000000000003, 1.3800000000000003, 1.3800000000000003, 1.3800000000000003, 1.3800000000000003, 1.3800000000000003, 1.3800000000000003, 1.3800000000000003, 1.3800000000000003, 1.3800000000000003, 1.3800000000000003, 1.3800000000000003, 1.3800000000000003, 1.3800000000000003, 1.3800000000000003, 1.3800000000000003, 1.3800000000000003, 1.3800000000000003, 1.3800000000000003, 1.3800000000000003, 1.3800000000000003, 1.3800000000000003, 1.3800000000000003 ], [ 1.415, 1.415, 1.415, 1.415, 1.415, 1.415, 1.415, 1.415, 1.415, 1.415, 1.415, 1.415, 1.415, 1.415, 1.415, 1.415, 1.415, 1.415, 1.415, 1.415, 1.415, 1.415, 1.415, 1.415, 1.415, 1.415, 1.415, 1.415, 1.415, 1.415, 1.415, 1.415, 1.415, 1.415, 1.415, 1.415, 1.415, 1.415, 1.415, 1.415, 1.415, 1.415, 1.415, 1.415, 1.415, 1.415, 1.415, 1.415, 1.415, 1.415, 1.415, 1.415, 1.415, 1.415, 1.415, 1.415, 1.415, 1.415, 1.415, 1.415, 1.415, 1.415, 1.415, 1.415, 1.415, 1.415, 1.415, 1.415, 1.415, 1.415, 1.415, 1.415, 1.415, 1.415, 1.415, 1.415, 1.415, 1.415, 1.415, 1.415, 1.415, 1.415, 1.415, 1.415, 1.415, 1.415, 1.415, 1.415, 1.415, 1.415, 1.415, 1.415, 1.415, 1.415, 1.415, 1.415, 1.415, 1.415, 1.415, 1.415, 1.415 ], [ 1.4500000000000002, 1.4500000000000002, 1.4500000000000002, 1.4500000000000002, 1.4500000000000002, 1.4500000000000002, 1.4500000000000002, 1.4500000000000002, 1.4500000000000002, 1.4500000000000002, 1.4500000000000002, 1.4500000000000002, 1.4500000000000002, 1.4500000000000002, 1.4500000000000002, 1.4500000000000002, 1.4500000000000002, 1.4500000000000002, 1.4500000000000002, 1.4500000000000002, 1.4500000000000002, 1.4500000000000002, 1.4500000000000002, 1.4500000000000002, 1.4500000000000002, 1.4500000000000002, 1.4500000000000002, 1.4500000000000002, 1.4500000000000002, 1.4500000000000002, 1.4500000000000002, 1.4500000000000002, 1.4500000000000002, 1.4500000000000002, 1.4500000000000002, 1.4500000000000002, 1.4500000000000002, 1.4500000000000002, 1.4500000000000002, 1.4500000000000002, 1.4500000000000002, 1.4500000000000002, 1.4500000000000002, 1.4500000000000002, 1.4500000000000002, 1.4500000000000002, 1.4500000000000002, 1.4500000000000002, 1.4500000000000002, 1.4500000000000002, 1.4500000000000002, 1.4500000000000002, 1.4500000000000002, 1.4500000000000002, 1.4500000000000002, 1.4500000000000002, 1.4500000000000002, 1.4500000000000002, 1.4500000000000002, 1.4500000000000002, 1.4500000000000002, 1.4500000000000002, 1.4500000000000002, 1.4500000000000002, 1.4500000000000002, 1.4500000000000002, 1.4500000000000002, 1.4500000000000002, 1.4500000000000002, 1.4500000000000002, 1.4500000000000002, 1.4500000000000002, 1.4500000000000002, 1.4500000000000002, 1.4500000000000002, 1.4500000000000002, 1.4500000000000002, 1.4500000000000002, 1.4500000000000002, 1.4500000000000002, 1.4500000000000002, 1.4500000000000002, 1.4500000000000002, 1.4500000000000002, 1.4500000000000002, 1.4500000000000002, 1.4500000000000002, 1.4500000000000002, 1.4500000000000002, 1.4500000000000002, 1.4500000000000002, 1.4500000000000002, 1.4500000000000002, 1.4500000000000002, 1.4500000000000002, 1.4500000000000002, 1.4500000000000002, 1.4500000000000002, 1.4500000000000002, 1.4500000000000002, 1.4500000000000002 ], [ 1.4850000000000003, 1.4850000000000003, 1.4850000000000003, 1.4850000000000003, 1.4850000000000003, 1.4850000000000003, 1.4850000000000003, 1.4850000000000003, 1.4850000000000003, 1.4850000000000003, 1.4850000000000003, 1.4850000000000003, 1.4850000000000003, 1.4850000000000003, 1.4850000000000003, 1.4850000000000003, 1.4850000000000003, 1.4850000000000003, 1.4850000000000003, 1.4850000000000003, 1.4850000000000003, 1.4850000000000003, 1.4850000000000003, 1.4850000000000003, 1.4850000000000003, 1.4850000000000003, 1.4850000000000003, 1.4850000000000003, 1.4850000000000003, 1.4850000000000003, 1.4850000000000003, 1.4850000000000003, 1.4850000000000003, 1.4850000000000003, 1.4850000000000003, 1.4850000000000003, 1.4850000000000003, 1.4850000000000003, 1.4850000000000003, 1.4850000000000003, 1.4850000000000003, 1.4850000000000003, 1.4850000000000003, 1.4850000000000003, 1.4850000000000003, 1.4850000000000003, 1.4850000000000003, 1.4850000000000003, 1.4850000000000003, 1.4850000000000003, 1.4850000000000003, 1.4850000000000003, 1.4850000000000003, 1.4850000000000003, 1.4850000000000003, 1.4850000000000003, 1.4850000000000003, 1.4850000000000003, 1.4850000000000003, 1.4850000000000003, 1.4850000000000003, 1.4850000000000003, 1.4850000000000003, 1.4850000000000003, 1.4850000000000003, 1.4850000000000003, 1.4850000000000003, 1.4850000000000003, 1.4850000000000003, 1.4850000000000003, 1.4850000000000003, 1.4850000000000003, 1.4850000000000003, 1.4850000000000003, 1.4850000000000003, 1.4850000000000003, 1.4850000000000003, 1.4850000000000003, 1.4850000000000003, 1.4850000000000003, 1.4850000000000003, 1.4850000000000003, 1.4850000000000003, 1.4850000000000003, 1.4850000000000003, 1.4850000000000003, 1.4850000000000003, 1.4850000000000003, 1.4850000000000003, 1.4850000000000003, 1.4850000000000003, 1.4850000000000003, 1.4850000000000003, 1.4850000000000003, 1.4850000000000003, 1.4850000000000003, 1.4850000000000003, 1.4850000000000003, 1.4850000000000003, 1.4850000000000003, 1.4850000000000003 ], [ 1.5200000000000005, 1.5200000000000005, 1.5200000000000005, 1.5200000000000005, 1.5200000000000005, 1.5200000000000005, 1.5200000000000005, 1.5200000000000005, 1.5200000000000005, 1.5200000000000005, 1.5200000000000005, 1.5200000000000005, 1.5200000000000005, 1.5200000000000005, 1.5200000000000005, 1.5200000000000005, 1.5200000000000005, 1.5200000000000005, 1.5200000000000005, 1.5200000000000005, 1.5200000000000005, 1.5200000000000005, 1.5200000000000005, 1.5200000000000005, 1.5200000000000005, 1.5200000000000005, 1.5200000000000005, 1.5200000000000005, 1.5200000000000005, 1.5200000000000005, 1.5200000000000005, 1.5200000000000005, 1.5200000000000005, 1.5200000000000005, 1.5200000000000005, 1.5200000000000005, 1.5200000000000005, 1.5200000000000005, 1.5200000000000005, 1.5200000000000005, 1.5200000000000005, 1.5200000000000005, 1.5200000000000005, 1.5200000000000005, 1.5200000000000005, 1.5200000000000005, 1.5200000000000005, 1.5200000000000005, 1.5200000000000005, 1.5200000000000005, 1.5200000000000005, 1.5200000000000005, 1.5200000000000005, 1.5200000000000005, 1.5200000000000005, 1.5200000000000005, 1.5200000000000005, 1.5200000000000005, 1.5200000000000005, 1.5200000000000005, 1.5200000000000005, 1.5200000000000005, 1.5200000000000005, 1.5200000000000005, 1.5200000000000005, 1.5200000000000005, 1.5200000000000005, 1.5200000000000005, 1.5200000000000005, 1.5200000000000005, 1.5200000000000005, 1.5200000000000005, 1.5200000000000005, 1.5200000000000005, 1.5200000000000005, 1.5200000000000005, 1.5200000000000005, 1.5200000000000005, 1.5200000000000005, 1.5200000000000005, 1.5200000000000005, 1.5200000000000005, 1.5200000000000005, 1.5200000000000005, 1.5200000000000005, 1.5200000000000005, 1.5200000000000005, 1.5200000000000005, 1.5200000000000005, 1.5200000000000005, 1.5200000000000005, 1.5200000000000005, 1.5200000000000005, 1.5200000000000005, 1.5200000000000005, 1.5200000000000005, 1.5200000000000005, 1.5200000000000005, 1.5200000000000005, 1.5200000000000005, 1.5200000000000005 ], [ 1.5550000000000002, 1.5550000000000002, 1.5550000000000002, 1.5550000000000002, 1.5550000000000002, 1.5550000000000002, 1.5550000000000002, 1.5550000000000002, 1.5550000000000002, 1.5550000000000002, 1.5550000000000002, 1.5550000000000002, 1.5550000000000002, 1.5550000000000002, 1.5550000000000002, 1.5550000000000002, 1.5550000000000002, 1.5550000000000002, 1.5550000000000002, 1.5550000000000002, 1.5550000000000002, 1.5550000000000002, 1.5550000000000002, 1.5550000000000002, 1.5550000000000002, 1.5550000000000002, 1.5550000000000002, 1.5550000000000002, 1.5550000000000002, 1.5550000000000002, 1.5550000000000002, 1.5550000000000002, 1.5550000000000002, 1.5550000000000002, 1.5550000000000002, 1.5550000000000002, 1.5550000000000002, 1.5550000000000002, 1.5550000000000002, 1.5550000000000002, 1.5550000000000002, 1.5550000000000002, 1.5550000000000002, 1.5550000000000002, 1.5550000000000002, 1.5550000000000002, 1.5550000000000002, 1.5550000000000002, 1.5550000000000002, 1.5550000000000002, 1.5550000000000002, 1.5550000000000002, 1.5550000000000002, 1.5550000000000002, 1.5550000000000002, 1.5550000000000002, 1.5550000000000002, 1.5550000000000002, 1.5550000000000002, 1.5550000000000002, 1.5550000000000002, 1.5550000000000002, 1.5550000000000002, 1.5550000000000002, 1.5550000000000002, 1.5550000000000002, 1.5550000000000002, 1.5550000000000002, 1.5550000000000002, 1.5550000000000002, 1.5550000000000002, 1.5550000000000002, 1.5550000000000002, 1.5550000000000002, 1.5550000000000002, 1.5550000000000002, 1.5550000000000002, 1.5550000000000002, 1.5550000000000002, 1.5550000000000002, 1.5550000000000002, 1.5550000000000002, 1.5550000000000002, 1.5550000000000002, 1.5550000000000002, 1.5550000000000002, 1.5550000000000002, 1.5550000000000002, 1.5550000000000002, 1.5550000000000002, 1.5550000000000002, 1.5550000000000002, 1.5550000000000002, 1.5550000000000002, 1.5550000000000002, 1.5550000000000002, 1.5550000000000002, 1.5550000000000002, 1.5550000000000002, 1.5550000000000002, 1.5550000000000002 ], [ 1.5900000000000003, 1.5900000000000003, 1.5900000000000003, 1.5900000000000003, 1.5900000000000003, 1.5900000000000003, 1.5900000000000003, 1.5900000000000003, 1.5900000000000003, 1.5900000000000003, 1.5900000000000003, 1.5900000000000003, 1.5900000000000003, 1.5900000000000003, 1.5900000000000003, 1.5900000000000003, 1.5900000000000003, 1.5900000000000003, 1.5900000000000003, 1.5900000000000003, 1.5900000000000003, 1.5900000000000003, 1.5900000000000003, 1.5900000000000003, 1.5900000000000003, 1.5900000000000003, 1.5900000000000003, 1.5900000000000003, 1.5900000000000003, 1.5900000000000003, 1.5900000000000003, 1.5900000000000003, 1.5900000000000003, 1.5900000000000003, 1.5900000000000003, 1.5900000000000003, 1.5900000000000003, 1.5900000000000003, 1.5900000000000003, 1.5900000000000003, 1.5900000000000003, 1.5900000000000003, 1.5900000000000003, 1.5900000000000003, 1.5900000000000003, 1.5900000000000003, 1.5900000000000003, 1.5900000000000003, 1.5900000000000003, 1.5900000000000003, 1.5900000000000003, 1.5900000000000003, 1.5900000000000003, 1.5900000000000003, 1.5900000000000003, 1.5900000000000003, 1.5900000000000003, 1.5900000000000003, 1.5900000000000003, 1.5900000000000003, 1.5900000000000003, 1.5900000000000003, 1.5900000000000003, 1.5900000000000003, 1.5900000000000003, 1.5900000000000003, 1.5900000000000003, 1.5900000000000003, 1.5900000000000003, 1.5900000000000003, 1.5900000000000003, 1.5900000000000003, 1.5900000000000003, 1.5900000000000003, 1.5900000000000003, 1.5900000000000003, 1.5900000000000003, 1.5900000000000003, 1.5900000000000003, 1.5900000000000003, 1.5900000000000003, 1.5900000000000003, 1.5900000000000003, 1.5900000000000003, 1.5900000000000003, 1.5900000000000003, 1.5900000000000003, 1.5900000000000003, 1.5900000000000003, 1.5900000000000003, 1.5900000000000003, 1.5900000000000003, 1.5900000000000003, 1.5900000000000003, 1.5900000000000003, 1.5900000000000003, 1.5900000000000003, 1.5900000000000003, 1.5900000000000003, 1.5900000000000003, 1.5900000000000003 ], [ 1.6250000000000004, 1.6250000000000004, 1.6250000000000004, 1.6250000000000004, 1.6250000000000004, 1.6250000000000004, 1.6250000000000004, 1.6250000000000004, 1.6250000000000004, 1.6250000000000004, 1.6250000000000004, 1.6250000000000004, 1.6250000000000004, 1.6250000000000004, 1.6250000000000004, 1.6250000000000004, 1.6250000000000004, 1.6250000000000004, 1.6250000000000004, 1.6250000000000004, 1.6250000000000004, 1.6250000000000004, 1.6250000000000004, 1.6250000000000004, 1.6250000000000004, 1.6250000000000004, 1.6250000000000004, 1.6250000000000004, 1.6250000000000004, 1.6250000000000004, 1.6250000000000004, 1.6250000000000004, 1.6250000000000004, 1.6250000000000004, 1.6250000000000004, 1.6250000000000004, 1.6250000000000004, 1.6250000000000004, 1.6250000000000004, 1.6250000000000004, 1.6250000000000004, 1.6250000000000004, 1.6250000000000004, 1.6250000000000004, 1.6250000000000004, 1.6250000000000004, 1.6250000000000004, 1.6250000000000004, 1.6250000000000004, 1.6250000000000004, 1.6250000000000004, 1.6250000000000004, 1.6250000000000004, 1.6250000000000004, 1.6250000000000004, 1.6250000000000004, 1.6250000000000004, 1.6250000000000004, 1.6250000000000004, 1.6250000000000004, 1.6250000000000004, 1.6250000000000004, 1.6250000000000004, 1.6250000000000004, 1.6250000000000004, 1.6250000000000004, 1.6250000000000004, 1.6250000000000004, 1.6250000000000004, 1.6250000000000004, 1.6250000000000004, 1.6250000000000004, 1.6250000000000004, 1.6250000000000004, 1.6250000000000004, 1.6250000000000004, 1.6250000000000004, 1.6250000000000004, 1.6250000000000004, 1.6250000000000004, 1.6250000000000004, 1.6250000000000004, 1.6250000000000004, 1.6250000000000004, 1.6250000000000004, 1.6250000000000004, 1.6250000000000004, 1.6250000000000004, 1.6250000000000004, 1.6250000000000004, 1.6250000000000004, 1.6250000000000004, 1.6250000000000004, 1.6250000000000004, 1.6250000000000004, 1.6250000000000004, 1.6250000000000004, 1.6250000000000004, 1.6250000000000004, 1.6250000000000004, 1.6250000000000004 ], [ 1.6600000000000001, 1.6600000000000001, 1.6600000000000001, 1.6600000000000001, 1.6600000000000001, 1.6600000000000001, 1.6600000000000001, 1.6600000000000001, 1.6600000000000001, 1.6600000000000001, 1.6600000000000001, 1.6600000000000001, 1.6600000000000001, 1.6600000000000001, 1.6600000000000001, 1.6600000000000001, 1.6600000000000001, 1.6600000000000001, 1.6600000000000001, 1.6600000000000001, 1.6600000000000001, 1.6600000000000001, 1.6600000000000001, 1.6600000000000001, 1.6600000000000001, 1.6600000000000001, 1.6600000000000001, 1.6600000000000001, 1.6600000000000001, 1.6600000000000001, 1.6600000000000001, 1.6600000000000001, 1.6600000000000001, 1.6600000000000001, 1.6600000000000001, 1.6600000000000001, 1.6600000000000001, 1.6600000000000001, 1.6600000000000001, 1.6600000000000001, 1.6600000000000001, 1.6600000000000001, 1.6600000000000001, 1.6600000000000001, 1.6600000000000001, 1.6600000000000001, 1.6600000000000001, 1.6600000000000001, 1.6600000000000001, 1.6600000000000001, 1.6600000000000001, 1.6600000000000001, 1.6600000000000001, 1.6600000000000001, 1.6600000000000001, 1.6600000000000001, 1.6600000000000001, 1.6600000000000001, 1.6600000000000001, 1.6600000000000001, 1.6600000000000001, 1.6600000000000001, 1.6600000000000001, 1.6600000000000001, 1.6600000000000001, 1.6600000000000001, 1.6600000000000001, 1.6600000000000001, 1.6600000000000001, 1.6600000000000001, 1.6600000000000001, 1.6600000000000001, 1.6600000000000001, 1.6600000000000001, 1.6600000000000001, 1.6600000000000001, 1.6600000000000001, 1.6600000000000001, 1.6600000000000001, 1.6600000000000001, 1.6600000000000001, 1.6600000000000001, 1.6600000000000001, 1.6600000000000001, 1.6600000000000001, 1.6600000000000001, 1.6600000000000001, 1.6600000000000001, 1.6600000000000001, 1.6600000000000001, 1.6600000000000001, 1.6600000000000001, 1.6600000000000001, 1.6600000000000001, 1.6600000000000001, 1.6600000000000001, 1.6600000000000001, 1.6600000000000001, 1.6600000000000001, 1.6600000000000001, 1.6600000000000001 ], [ 1.6950000000000003, 1.6950000000000003, 1.6950000000000003, 1.6950000000000003, 1.6950000000000003, 1.6950000000000003, 1.6950000000000003, 1.6950000000000003, 1.6950000000000003, 1.6950000000000003, 1.6950000000000003, 1.6950000000000003, 1.6950000000000003, 1.6950000000000003, 1.6950000000000003, 1.6950000000000003, 1.6950000000000003, 1.6950000000000003, 1.6950000000000003, 1.6950000000000003, 1.6950000000000003, 1.6950000000000003, 1.6950000000000003, 1.6950000000000003, 1.6950000000000003, 1.6950000000000003, 1.6950000000000003, 1.6950000000000003, 1.6950000000000003, 1.6950000000000003, 1.6950000000000003, 1.6950000000000003, 1.6950000000000003, 1.6950000000000003, 1.6950000000000003, 1.6950000000000003, 1.6950000000000003, 1.6950000000000003, 1.6950000000000003, 1.6950000000000003, 1.6950000000000003, 1.6950000000000003, 1.6950000000000003, 1.6950000000000003, 1.6950000000000003, 1.6950000000000003, 1.6950000000000003, 1.6950000000000003, 1.6950000000000003, 1.6950000000000003, 1.6950000000000003, 1.6950000000000003, 1.6950000000000003, 1.6950000000000003, 1.6950000000000003, 1.6950000000000003, 1.6950000000000003, 1.6950000000000003, 1.6950000000000003, 1.6950000000000003, 1.6950000000000003, 1.6950000000000003, 1.6950000000000003, 1.6950000000000003, 1.6950000000000003, 1.6950000000000003, 1.6950000000000003, 1.6950000000000003, 1.6950000000000003, 1.6950000000000003, 1.6950000000000003, 1.6950000000000003, 1.6950000000000003, 1.6950000000000003, 1.6950000000000003, 1.6950000000000003, 1.6950000000000003, 1.6950000000000003, 1.6950000000000003, 1.6950000000000003, 1.6950000000000003, 1.6950000000000003, 1.6950000000000003, 1.6950000000000003, 1.6950000000000003, 1.6950000000000003, 1.6950000000000003, 1.6950000000000003, 1.6950000000000003, 1.6950000000000003, 1.6950000000000003, 1.6950000000000003, 1.6950000000000003, 1.6950000000000003, 1.6950000000000003, 1.6950000000000003, 1.6950000000000003, 1.6950000000000003, 1.6950000000000003, 1.6950000000000003, 1.6950000000000003 ], [ 1.7300000000000004, 1.7300000000000004, 1.7300000000000004, 1.7300000000000004, 1.7300000000000004, 1.7300000000000004, 1.7300000000000004, 1.7300000000000004, 1.7300000000000004, 1.7300000000000004, 1.7300000000000004, 1.7300000000000004, 1.7300000000000004, 1.7300000000000004, 1.7300000000000004, 1.7300000000000004, 1.7300000000000004, 1.7300000000000004, 1.7300000000000004, 1.7300000000000004, 1.7300000000000004, 1.7300000000000004, 1.7300000000000004, 1.7300000000000004, 1.7300000000000004, 1.7300000000000004, 1.7300000000000004, 1.7300000000000004, 1.7300000000000004, 1.7300000000000004, 1.7300000000000004, 1.7300000000000004, 1.7300000000000004, 1.7300000000000004, 1.7300000000000004, 1.7300000000000004, 1.7300000000000004, 1.7300000000000004, 1.7300000000000004, 1.7300000000000004, 1.7300000000000004, 1.7300000000000004, 1.7300000000000004, 1.7300000000000004, 1.7300000000000004, 1.7300000000000004, 1.7300000000000004, 1.7300000000000004, 1.7300000000000004, 1.7300000000000004, 1.7300000000000004, 1.7300000000000004, 1.7300000000000004, 1.7300000000000004, 1.7300000000000004, 1.7300000000000004, 1.7300000000000004, 1.7300000000000004, 1.7300000000000004, 1.7300000000000004, 1.7300000000000004, 1.7300000000000004, 1.7300000000000004, 1.7300000000000004, 1.7300000000000004, 1.7300000000000004, 1.7300000000000004, 1.7300000000000004, 1.7300000000000004, 1.7300000000000004, 1.7300000000000004, 1.7300000000000004, 1.7300000000000004, 1.7300000000000004, 1.7300000000000004, 1.7300000000000004, 1.7300000000000004, 1.7300000000000004, 1.7300000000000004, 1.7300000000000004, 1.7300000000000004, 1.7300000000000004, 1.7300000000000004, 1.7300000000000004, 1.7300000000000004, 1.7300000000000004, 1.7300000000000004, 1.7300000000000004, 1.7300000000000004, 1.7300000000000004, 1.7300000000000004, 1.7300000000000004, 1.7300000000000004, 1.7300000000000004, 1.7300000000000004, 1.7300000000000004, 1.7300000000000004, 1.7300000000000004, 1.7300000000000004, 1.7300000000000004, 1.7300000000000004 ], [ 1.7650000000000001, 1.7650000000000001, 1.7650000000000001, 1.7650000000000001, 1.7650000000000001, 1.7650000000000001, 1.7650000000000001, 1.7650000000000001, 1.7650000000000001, 1.7650000000000001, 1.7650000000000001, 1.7650000000000001, 1.7650000000000001, 1.7650000000000001, 1.7650000000000001, 1.7650000000000001, 1.7650000000000001, 1.7650000000000001, 1.7650000000000001, 1.7650000000000001, 1.7650000000000001, 1.7650000000000001, 1.7650000000000001, 1.7650000000000001, 1.7650000000000001, 1.7650000000000001, 1.7650000000000001, 1.7650000000000001, 1.7650000000000001, 1.7650000000000001, 1.7650000000000001, 1.7650000000000001, 1.7650000000000001, 1.7650000000000001, 1.7650000000000001, 1.7650000000000001, 1.7650000000000001, 1.7650000000000001, 1.7650000000000001, 1.7650000000000001, 1.7650000000000001, 1.7650000000000001, 1.7650000000000001, 1.7650000000000001, 1.7650000000000001, 1.7650000000000001, 1.7650000000000001, 1.7650000000000001, 1.7650000000000001, 1.7650000000000001, 1.7650000000000001, 1.7650000000000001, 1.7650000000000001, 1.7650000000000001, 1.7650000000000001, 1.7650000000000001, 1.7650000000000001, 1.7650000000000001, 1.7650000000000001, 1.7650000000000001, 1.7650000000000001, 1.7650000000000001, 1.7650000000000001, 1.7650000000000001, 1.7650000000000001, 1.7650000000000001, 1.7650000000000001, 1.7650000000000001, 1.7650000000000001, 1.7650000000000001, 1.7650000000000001, 1.7650000000000001, 1.7650000000000001, 1.7650000000000001, 1.7650000000000001, 1.7650000000000001, 1.7650000000000001, 1.7650000000000001, 1.7650000000000001, 1.7650000000000001, 1.7650000000000001, 1.7650000000000001, 1.7650000000000001, 1.7650000000000001, 1.7650000000000001, 1.7650000000000001, 1.7650000000000001, 1.7650000000000001, 1.7650000000000001, 1.7650000000000001, 1.7650000000000001, 1.7650000000000001, 1.7650000000000001, 1.7650000000000001, 1.7650000000000001, 1.7650000000000001, 1.7650000000000001, 1.7650000000000001, 1.7650000000000001, 1.7650000000000001, 1.7650000000000001 ], [ 1.8000000000000003, 1.8000000000000003, 1.8000000000000003, 1.8000000000000003, 1.8000000000000003, 1.8000000000000003, 1.8000000000000003, 1.8000000000000003, 1.8000000000000003, 1.8000000000000003, 1.8000000000000003, 1.8000000000000003, 1.8000000000000003, 1.8000000000000003, 1.8000000000000003, 1.8000000000000003, 1.8000000000000003, 1.8000000000000003, 1.8000000000000003, 1.8000000000000003, 1.8000000000000003, 1.8000000000000003, 1.8000000000000003, 1.8000000000000003, 1.8000000000000003, 1.8000000000000003, 1.8000000000000003, 1.8000000000000003, 1.8000000000000003, 1.8000000000000003, 1.8000000000000003, 1.8000000000000003, 1.8000000000000003, 1.8000000000000003, 1.8000000000000003, 1.8000000000000003, 1.8000000000000003, 1.8000000000000003, 1.8000000000000003, 1.8000000000000003, 1.8000000000000003, 1.8000000000000003, 1.8000000000000003, 1.8000000000000003, 1.8000000000000003, 1.8000000000000003, 1.8000000000000003, 1.8000000000000003, 1.8000000000000003, 1.8000000000000003, 1.8000000000000003, 1.8000000000000003, 1.8000000000000003, 1.8000000000000003, 1.8000000000000003, 1.8000000000000003, 1.8000000000000003, 1.8000000000000003, 1.8000000000000003, 1.8000000000000003, 1.8000000000000003, 1.8000000000000003, 1.8000000000000003, 1.8000000000000003, 1.8000000000000003, 1.8000000000000003, 1.8000000000000003, 1.8000000000000003, 1.8000000000000003, 1.8000000000000003, 1.8000000000000003, 1.8000000000000003, 1.8000000000000003, 1.8000000000000003, 1.8000000000000003, 1.8000000000000003, 1.8000000000000003, 1.8000000000000003, 1.8000000000000003, 1.8000000000000003, 1.8000000000000003, 1.8000000000000003, 1.8000000000000003, 1.8000000000000003, 1.8000000000000003, 1.8000000000000003, 1.8000000000000003, 1.8000000000000003, 1.8000000000000003, 1.8000000000000003, 1.8000000000000003, 1.8000000000000003, 1.8000000000000003, 1.8000000000000003, 1.8000000000000003, 1.8000000000000003, 1.8000000000000003, 1.8000000000000003, 1.8000000000000003, 1.8000000000000003, 1.8000000000000003 ], [ 1.8350000000000004, 1.8350000000000004, 1.8350000000000004, 1.8350000000000004, 1.8350000000000004, 1.8350000000000004, 1.8350000000000004, 1.8350000000000004, 1.8350000000000004, 1.8350000000000004, 1.8350000000000004, 1.8350000000000004, 1.8350000000000004, 1.8350000000000004, 1.8350000000000004, 1.8350000000000004, 1.8350000000000004, 1.8350000000000004, 1.8350000000000004, 1.8350000000000004, 1.8350000000000004, 1.8350000000000004, 1.8350000000000004, 1.8350000000000004, 1.8350000000000004, 1.8350000000000004, 1.8350000000000004, 1.8350000000000004, 1.8350000000000004, 1.8350000000000004, 1.8350000000000004, 1.8350000000000004, 1.8350000000000004, 1.8350000000000004, 1.8350000000000004, 1.8350000000000004, 1.8350000000000004, 1.8350000000000004, 1.8350000000000004, 1.8350000000000004, 1.8350000000000004, 1.8350000000000004, 1.8350000000000004, 1.8350000000000004, 1.8350000000000004, 1.8350000000000004, 1.8350000000000004, 1.8350000000000004, 1.8350000000000004, 1.8350000000000004, 1.8350000000000004, 1.8350000000000004, 1.8350000000000004, 1.8350000000000004, 1.8350000000000004, 1.8350000000000004, 1.8350000000000004, 1.8350000000000004, 1.8350000000000004, 1.8350000000000004, 1.8350000000000004, 1.8350000000000004, 1.8350000000000004, 1.8350000000000004, 1.8350000000000004, 1.8350000000000004, 1.8350000000000004, 1.8350000000000004, 1.8350000000000004, 1.8350000000000004, 1.8350000000000004, 1.8350000000000004, 1.8350000000000004, 1.8350000000000004, 1.8350000000000004, 1.8350000000000004, 1.8350000000000004, 1.8350000000000004, 1.8350000000000004, 1.8350000000000004, 1.8350000000000004, 1.8350000000000004, 1.8350000000000004, 1.8350000000000004, 1.8350000000000004, 1.8350000000000004, 1.8350000000000004, 1.8350000000000004, 1.8350000000000004, 1.8350000000000004, 1.8350000000000004, 1.8350000000000004, 1.8350000000000004, 1.8350000000000004, 1.8350000000000004, 1.8350000000000004, 1.8350000000000004, 1.8350000000000004, 1.8350000000000004, 1.8350000000000004, 1.8350000000000004 ], [ 1.87, 1.87, 1.87, 1.87, 1.87, 1.87, 1.87, 1.87, 1.87, 1.87, 1.87, 1.87, 1.87, 1.87, 1.87, 1.87, 1.87, 1.87, 1.87, 1.87, 1.87, 1.87, 1.87, 1.87, 1.87, 1.87, 1.87, 1.87, 1.87, 1.87, 1.87, 1.87, 1.87, 1.87, 1.87, 1.87, 1.87, 1.87, 1.87, 1.87, 1.87, 1.87, 1.87, 1.87, 1.87, 1.87, 1.87, 1.87, 1.87, 1.87, 1.87, 1.87, 1.87, 1.87, 1.87, 1.87, 1.87, 1.87, 1.87, 1.87, 1.87, 1.87, 1.87, 1.87, 1.87, 1.87, 1.87, 1.87, 1.87, 1.87, 1.87, 1.87, 1.87, 1.87, 1.87, 1.87, 1.87, 1.87, 1.87, 1.87, 1.87, 1.87, 1.87, 1.87, 1.87, 1.87, 1.87, 1.87, 1.87, 1.87, 1.87, 1.87, 1.87, 1.87, 1.87, 1.87, 1.87, 1.87, 1.87, 1.87, 1.87 ], [ 1.9050000000000002, 1.9050000000000002, 1.9050000000000002, 1.9050000000000002, 1.9050000000000002, 1.9050000000000002, 1.9050000000000002, 1.9050000000000002, 1.9050000000000002, 1.9050000000000002, 1.9050000000000002, 1.9050000000000002, 1.9050000000000002, 1.9050000000000002, 1.9050000000000002, 1.9050000000000002, 1.9050000000000002, 1.9050000000000002, 1.9050000000000002, 1.9050000000000002, 1.9050000000000002, 1.9050000000000002, 1.9050000000000002, 1.9050000000000002, 1.9050000000000002, 1.9050000000000002, 1.9050000000000002, 1.9050000000000002, 1.9050000000000002, 1.9050000000000002, 1.9050000000000002, 1.9050000000000002, 1.9050000000000002, 1.9050000000000002, 1.9050000000000002, 1.9050000000000002, 1.9050000000000002, 1.9050000000000002, 1.9050000000000002, 1.9050000000000002, 1.9050000000000002, 1.9050000000000002, 1.9050000000000002, 1.9050000000000002, 1.9050000000000002, 1.9050000000000002, 1.9050000000000002, 1.9050000000000002, 1.9050000000000002, 1.9050000000000002, 1.9050000000000002, 1.9050000000000002, 1.9050000000000002, 1.9050000000000002, 1.9050000000000002, 1.9050000000000002, 1.9050000000000002, 1.9050000000000002, 1.9050000000000002, 1.9050000000000002, 1.9050000000000002, 1.9050000000000002, 1.9050000000000002, 1.9050000000000002, 1.9050000000000002, 1.9050000000000002, 1.9050000000000002, 1.9050000000000002, 1.9050000000000002, 1.9050000000000002, 1.9050000000000002, 1.9050000000000002, 1.9050000000000002, 1.9050000000000002, 1.9050000000000002, 1.9050000000000002, 1.9050000000000002, 1.9050000000000002, 1.9050000000000002, 1.9050000000000002, 1.9050000000000002, 1.9050000000000002, 1.9050000000000002, 1.9050000000000002, 1.9050000000000002, 1.9050000000000002, 1.9050000000000002, 1.9050000000000002, 1.9050000000000002, 1.9050000000000002, 1.9050000000000002, 1.9050000000000002, 1.9050000000000002, 1.9050000000000002, 1.9050000000000002, 1.9050000000000002, 1.9050000000000002, 1.9050000000000002, 1.9050000000000002, 1.9050000000000002, 1.9050000000000002 ], [ 1.9400000000000004, 1.9400000000000004, 1.9400000000000004, 1.9400000000000004, 1.9400000000000004, 1.9400000000000004, 1.9400000000000004, 1.9400000000000004, 1.9400000000000004, 1.9400000000000004, 1.9400000000000004, 1.9400000000000004, 1.9400000000000004, 1.9400000000000004, 1.9400000000000004, 1.9400000000000004, 1.9400000000000004, 1.9400000000000004, 1.9400000000000004, 1.9400000000000004, 1.9400000000000004, 1.9400000000000004, 1.9400000000000004, 1.9400000000000004, 1.9400000000000004, 1.9400000000000004, 1.9400000000000004, 1.9400000000000004, 1.9400000000000004, 1.9400000000000004, 1.9400000000000004, 1.9400000000000004, 1.9400000000000004, 1.9400000000000004, 1.9400000000000004, 1.9400000000000004, 1.9400000000000004, 1.9400000000000004, 1.9400000000000004, 1.9400000000000004, 1.9400000000000004, 1.9400000000000004, 1.9400000000000004, 1.9400000000000004, 1.9400000000000004, 1.9400000000000004, 1.9400000000000004, 1.9400000000000004, 1.9400000000000004, 1.9400000000000004, 1.9400000000000004, 1.9400000000000004, 1.9400000000000004, 1.9400000000000004, 1.9400000000000004, 1.9400000000000004, 1.9400000000000004, 1.9400000000000004, 1.9400000000000004, 1.9400000000000004, 1.9400000000000004, 1.9400000000000004, 1.9400000000000004, 1.9400000000000004, 1.9400000000000004, 1.9400000000000004, 1.9400000000000004, 1.9400000000000004, 1.9400000000000004, 1.9400000000000004, 1.9400000000000004, 1.9400000000000004, 1.9400000000000004, 1.9400000000000004, 1.9400000000000004, 1.9400000000000004, 1.9400000000000004, 1.9400000000000004, 1.9400000000000004, 1.9400000000000004, 1.9400000000000004, 1.9400000000000004, 1.9400000000000004, 1.9400000000000004, 1.9400000000000004, 1.9400000000000004, 1.9400000000000004, 1.9400000000000004, 1.9400000000000004, 1.9400000000000004, 1.9400000000000004, 1.9400000000000004, 1.9400000000000004, 1.9400000000000004, 1.9400000000000004, 1.9400000000000004, 1.9400000000000004, 1.9400000000000004, 1.9400000000000004, 1.9400000000000004, 1.9400000000000004 ], [ 1.975, 1.975, 1.975, 1.975, 1.975, 1.975, 1.975, 1.975, 1.975, 1.975, 1.975, 1.975, 1.975, 1.975, 1.975, 1.975, 1.975, 1.975, 1.975, 1.975, 1.975, 1.975, 1.975, 1.975, 1.975, 1.975, 1.975, 1.975, 1.975, 1.975, 1.975, 1.975, 1.975, 1.975, 1.975, 1.975, 1.975, 1.975, 1.975, 1.975, 1.975, 1.975, 1.975, 1.975, 1.975, 1.975, 1.975, 1.975, 1.975, 1.975, 1.975, 1.975, 1.975, 1.975, 1.975, 1.975, 1.975, 1.975, 1.975, 1.975, 1.975, 1.975, 1.975, 1.975, 1.975, 1.975, 1.975, 1.975, 1.975, 1.975, 1.975, 1.975, 1.975, 1.975, 1.975, 1.975, 1.975, 1.975, 1.975, 1.975, 1.975, 1.975, 1.975, 1.975, 1.975, 1.975, 1.975, 1.975, 1.975, 1.975, 1.975, 1.975, 1.975, 1.975, 1.975, 1.975, 1.975, 1.975, 1.975, 1.975, 1.975 ], [ 2.0100000000000002, 2.0100000000000002, 2.0100000000000002, 2.0100000000000002, 2.0100000000000002, 2.0100000000000002, 2.0100000000000002, 2.0100000000000002, 2.0100000000000002, 2.0100000000000002, 2.0100000000000002, 2.0100000000000002, 2.0100000000000002, 2.0100000000000002, 2.0100000000000002, 2.0100000000000002, 2.0100000000000002, 2.0100000000000002, 2.0100000000000002, 2.0100000000000002, 2.0100000000000002, 2.0100000000000002, 2.0100000000000002, 2.0100000000000002, 2.0100000000000002, 2.0100000000000002, 2.0100000000000002, 2.0100000000000002, 2.0100000000000002, 2.0100000000000002, 2.0100000000000002, 2.0100000000000002, 2.0100000000000002, 2.0100000000000002, 2.0100000000000002, 2.0100000000000002, 2.0100000000000002, 2.0100000000000002, 2.0100000000000002, 2.0100000000000002, 2.0100000000000002, 2.0100000000000002, 2.0100000000000002, 2.0100000000000002, 2.0100000000000002, 2.0100000000000002, 2.0100000000000002, 2.0100000000000002, 2.0100000000000002, 2.0100000000000002, 2.0100000000000002, 2.0100000000000002, 2.0100000000000002, 2.0100000000000002, 2.0100000000000002, 2.0100000000000002, 2.0100000000000002, 2.0100000000000002, 2.0100000000000002, 2.0100000000000002, 2.0100000000000002, 2.0100000000000002, 2.0100000000000002, 2.0100000000000002, 2.0100000000000002, 2.0100000000000002, 2.0100000000000002, 2.0100000000000002, 2.0100000000000002, 2.0100000000000002, 2.0100000000000002, 2.0100000000000002, 2.0100000000000002, 2.0100000000000002, 2.0100000000000002, 2.0100000000000002, 2.0100000000000002, 2.0100000000000002, 2.0100000000000002, 2.0100000000000002, 2.0100000000000002, 2.0100000000000002, 2.0100000000000002, 2.0100000000000002, 2.0100000000000002, 2.0100000000000002, 2.0100000000000002, 2.0100000000000002, 2.0100000000000002, 2.0100000000000002, 2.0100000000000002, 2.0100000000000002, 2.0100000000000002, 2.0100000000000002, 2.0100000000000002, 2.0100000000000002, 2.0100000000000002, 2.0100000000000002, 2.0100000000000002, 2.0100000000000002, 2.0100000000000002 ], [ 2.0450000000000004, 2.0450000000000004, 2.0450000000000004, 2.0450000000000004, 2.0450000000000004, 2.0450000000000004, 2.0450000000000004, 2.0450000000000004, 2.0450000000000004, 2.0450000000000004, 2.0450000000000004, 2.0450000000000004, 2.0450000000000004, 2.0450000000000004, 2.0450000000000004, 2.0450000000000004, 2.0450000000000004, 2.0450000000000004, 2.0450000000000004, 2.0450000000000004, 2.0450000000000004, 2.0450000000000004, 2.0450000000000004, 2.0450000000000004, 2.0450000000000004, 2.0450000000000004, 2.0450000000000004, 2.0450000000000004, 2.0450000000000004, 2.0450000000000004, 2.0450000000000004, 2.0450000000000004, 2.0450000000000004, 2.0450000000000004, 2.0450000000000004, 2.0450000000000004, 2.0450000000000004, 2.0450000000000004, 2.0450000000000004, 2.0450000000000004, 2.0450000000000004, 2.0450000000000004, 2.0450000000000004, 2.0450000000000004, 2.0450000000000004, 2.0450000000000004, 2.0450000000000004, 2.0450000000000004, 2.0450000000000004, 2.0450000000000004, 2.0450000000000004, 2.0450000000000004, 2.0450000000000004, 2.0450000000000004, 2.0450000000000004, 2.0450000000000004, 2.0450000000000004, 2.0450000000000004, 2.0450000000000004, 2.0450000000000004, 2.0450000000000004, 2.0450000000000004, 2.0450000000000004, 2.0450000000000004, 2.0450000000000004, 2.0450000000000004, 2.0450000000000004, 2.0450000000000004, 2.0450000000000004, 2.0450000000000004, 2.0450000000000004, 2.0450000000000004, 2.0450000000000004, 2.0450000000000004, 2.0450000000000004, 2.0450000000000004, 2.0450000000000004, 2.0450000000000004, 2.0450000000000004, 2.0450000000000004, 2.0450000000000004, 2.0450000000000004, 2.0450000000000004, 2.0450000000000004, 2.0450000000000004, 2.0450000000000004, 2.0450000000000004, 2.0450000000000004, 2.0450000000000004, 2.0450000000000004, 2.0450000000000004, 2.0450000000000004, 2.0450000000000004, 2.0450000000000004, 2.0450000000000004, 2.0450000000000004, 2.0450000000000004, 2.0450000000000004, 2.0450000000000004, 2.0450000000000004, 2.0450000000000004 ], [ 2.08, 2.08, 2.08, 2.08, 2.08, 2.08, 2.08, 2.08, 2.08, 2.08, 2.08, 2.08, 2.08, 2.08, 2.08, 2.08, 2.08, 2.08, 2.08, 2.08, 2.08, 2.08, 2.08, 2.08, 2.08, 2.08, 2.08, 2.08, 2.08, 2.08, 2.08, 2.08, 2.08, 2.08, 2.08, 2.08, 2.08, 2.08, 2.08, 2.08, 2.08, 2.08, 2.08, 2.08, 2.08, 2.08, 2.08, 2.08, 2.08, 2.08, 2.08, 2.08, 2.08, 2.08, 2.08, 2.08, 2.08, 2.08, 2.08, 2.08, 2.08, 2.08, 2.08, 2.08, 2.08, 2.08, 2.08, 2.08, 2.08, 2.08, 2.08, 2.08, 2.08, 2.08, 2.08, 2.08, 2.08, 2.08, 2.08, 2.08, 2.08, 2.08, 2.08, 2.08, 2.08, 2.08, 2.08, 2.08, 2.08, 2.08, 2.08, 2.08, 2.08, 2.08, 2.08, 2.08, 2.08, 2.08, 2.08, 2.08, 2.08 ], [ 2.115, 2.115, 2.115, 2.115, 2.115, 2.115, 2.115, 2.115, 2.115, 2.115, 2.115, 2.115, 2.115, 2.115, 2.115, 2.115, 2.115, 2.115, 2.115, 2.115, 2.115, 2.115, 2.115, 2.115, 2.115, 2.115, 2.115, 2.115, 2.115, 2.115, 2.115, 2.115, 2.115, 2.115, 2.115, 2.115, 2.115, 2.115, 2.115, 2.115, 2.115, 2.115, 2.115, 2.115, 2.115, 2.115, 2.115, 2.115, 2.115, 2.115, 2.115, 2.115, 2.115, 2.115, 2.115, 2.115, 2.115, 2.115, 2.115, 2.115, 2.115, 2.115, 2.115, 2.115, 2.115, 2.115, 2.115, 2.115, 2.115, 2.115, 2.115, 2.115, 2.115, 2.115, 2.115, 2.115, 2.115, 2.115, 2.115, 2.115, 2.115, 2.115, 2.115, 2.115, 2.115, 2.115, 2.115, 2.115, 2.115, 2.115, 2.115, 2.115, 2.115, 2.115, 2.115, 2.115, 2.115, 2.115, 2.115, 2.115, 2.115 ], [ 2.1500000000000004, 2.1500000000000004, 2.1500000000000004, 2.1500000000000004, 2.1500000000000004, 2.1500000000000004, 2.1500000000000004, 2.1500000000000004, 2.1500000000000004, 2.1500000000000004, 2.1500000000000004, 2.1500000000000004, 2.1500000000000004, 2.1500000000000004, 2.1500000000000004, 2.1500000000000004, 2.1500000000000004, 2.1500000000000004, 2.1500000000000004, 2.1500000000000004, 2.1500000000000004, 2.1500000000000004, 2.1500000000000004, 2.1500000000000004, 2.1500000000000004, 2.1500000000000004, 2.1500000000000004, 2.1500000000000004, 2.1500000000000004, 2.1500000000000004, 2.1500000000000004, 2.1500000000000004, 2.1500000000000004, 2.1500000000000004, 2.1500000000000004, 2.1500000000000004, 2.1500000000000004, 2.1500000000000004, 2.1500000000000004, 2.1500000000000004, 2.1500000000000004, 2.1500000000000004, 2.1500000000000004, 2.1500000000000004, 2.1500000000000004, 2.1500000000000004, 2.1500000000000004, 2.1500000000000004, 2.1500000000000004, 2.1500000000000004, 2.1500000000000004, 2.1500000000000004, 2.1500000000000004, 2.1500000000000004, 2.1500000000000004, 2.1500000000000004, 2.1500000000000004, 2.1500000000000004, 2.1500000000000004, 2.1500000000000004, 2.1500000000000004, 2.1500000000000004, 2.1500000000000004, 2.1500000000000004, 2.1500000000000004, 2.1500000000000004, 2.1500000000000004, 2.1500000000000004, 2.1500000000000004, 2.1500000000000004, 2.1500000000000004, 2.1500000000000004, 2.1500000000000004, 2.1500000000000004, 2.1500000000000004, 2.1500000000000004, 2.1500000000000004, 2.1500000000000004, 2.1500000000000004, 2.1500000000000004, 2.1500000000000004, 2.1500000000000004, 2.1500000000000004, 2.1500000000000004, 2.1500000000000004, 2.1500000000000004, 2.1500000000000004, 2.1500000000000004, 2.1500000000000004, 2.1500000000000004, 2.1500000000000004, 2.1500000000000004, 2.1500000000000004, 2.1500000000000004, 2.1500000000000004, 2.1500000000000004, 2.1500000000000004, 2.1500000000000004, 2.1500000000000004, 2.1500000000000004, 2.1500000000000004 ], [ 2.1850000000000005, 2.1850000000000005, 2.1850000000000005, 2.1850000000000005, 2.1850000000000005, 2.1850000000000005, 2.1850000000000005, 2.1850000000000005, 2.1850000000000005, 2.1850000000000005, 2.1850000000000005, 2.1850000000000005, 2.1850000000000005, 2.1850000000000005, 2.1850000000000005, 2.1850000000000005, 2.1850000000000005, 2.1850000000000005, 2.1850000000000005, 2.1850000000000005, 2.1850000000000005, 2.1850000000000005, 2.1850000000000005, 2.1850000000000005, 2.1850000000000005, 2.1850000000000005, 2.1850000000000005, 2.1850000000000005, 2.1850000000000005, 2.1850000000000005, 2.1850000000000005, 2.1850000000000005, 2.1850000000000005, 2.1850000000000005, 2.1850000000000005, 2.1850000000000005, 2.1850000000000005, 2.1850000000000005, 2.1850000000000005, 2.1850000000000005, 2.1850000000000005, 2.1850000000000005, 2.1850000000000005, 2.1850000000000005, 2.1850000000000005, 2.1850000000000005, 2.1850000000000005, 2.1850000000000005, 2.1850000000000005, 2.1850000000000005, 2.1850000000000005, 2.1850000000000005, 2.1850000000000005, 2.1850000000000005, 2.1850000000000005, 2.1850000000000005, 2.1850000000000005, 2.1850000000000005, 2.1850000000000005, 2.1850000000000005, 2.1850000000000005, 2.1850000000000005, 2.1850000000000005, 2.1850000000000005, 2.1850000000000005, 2.1850000000000005, 2.1850000000000005, 2.1850000000000005, 2.1850000000000005, 2.1850000000000005, 2.1850000000000005, 2.1850000000000005, 2.1850000000000005, 2.1850000000000005, 2.1850000000000005, 2.1850000000000005, 2.1850000000000005, 2.1850000000000005, 2.1850000000000005, 2.1850000000000005, 2.1850000000000005, 2.1850000000000005, 2.1850000000000005, 2.1850000000000005, 2.1850000000000005, 2.1850000000000005, 2.1850000000000005, 2.1850000000000005, 2.1850000000000005, 2.1850000000000005, 2.1850000000000005, 2.1850000000000005, 2.1850000000000005, 2.1850000000000005, 2.1850000000000005, 2.1850000000000005, 2.1850000000000005, 2.1850000000000005, 2.1850000000000005, 2.1850000000000005, 2.1850000000000005 ], [ 2.22, 2.22, 2.22, 2.22, 2.22, 2.22, 2.22, 2.22, 2.22, 2.22, 2.22, 2.22, 2.22, 2.22, 2.22, 2.22, 2.22, 2.22, 2.22, 2.22, 2.22, 2.22, 2.22, 2.22, 2.22, 2.22, 2.22, 2.22, 2.22, 2.22, 2.22, 2.22, 2.22, 2.22, 2.22, 2.22, 2.22, 2.22, 2.22, 2.22, 2.22, 2.22, 2.22, 2.22, 2.22, 2.22, 2.22, 2.22, 2.22, 2.22, 2.22, 2.22, 2.22, 2.22, 2.22, 2.22, 2.22, 2.22, 2.22, 2.22, 2.22, 2.22, 2.22, 2.22, 2.22, 2.22, 2.22, 2.22, 2.22, 2.22, 2.22, 2.22, 2.22, 2.22, 2.22, 2.22, 2.22, 2.22, 2.22, 2.22, 2.22, 2.22, 2.22, 2.22, 2.22, 2.22, 2.22, 2.22, 2.22, 2.22, 2.22, 2.22, 2.22, 2.22, 2.22, 2.22, 2.22, 2.22, 2.22, 2.22, 2.22 ], [ 2.2550000000000003, 2.2550000000000003, 2.2550000000000003, 2.2550000000000003, 2.2550000000000003, 2.2550000000000003, 2.2550000000000003, 2.2550000000000003, 2.2550000000000003, 2.2550000000000003, 2.2550000000000003, 2.2550000000000003, 2.2550000000000003, 2.2550000000000003, 2.2550000000000003, 2.2550000000000003, 2.2550000000000003, 2.2550000000000003, 2.2550000000000003, 2.2550000000000003, 2.2550000000000003, 2.2550000000000003, 2.2550000000000003, 2.2550000000000003, 2.2550000000000003, 2.2550000000000003, 2.2550000000000003, 2.2550000000000003, 2.2550000000000003, 2.2550000000000003, 2.2550000000000003, 2.2550000000000003, 2.2550000000000003, 2.2550000000000003, 2.2550000000000003, 2.2550000000000003, 2.2550000000000003, 2.2550000000000003, 2.2550000000000003, 2.2550000000000003, 2.2550000000000003, 2.2550000000000003, 2.2550000000000003, 2.2550000000000003, 2.2550000000000003, 2.2550000000000003, 2.2550000000000003, 2.2550000000000003, 2.2550000000000003, 2.2550000000000003, 2.2550000000000003, 2.2550000000000003, 2.2550000000000003, 2.2550000000000003, 2.2550000000000003, 2.2550000000000003, 2.2550000000000003, 2.2550000000000003, 2.2550000000000003, 2.2550000000000003, 2.2550000000000003, 2.2550000000000003, 2.2550000000000003, 2.2550000000000003, 2.2550000000000003, 2.2550000000000003, 2.2550000000000003, 2.2550000000000003, 2.2550000000000003, 2.2550000000000003, 2.2550000000000003, 2.2550000000000003, 2.2550000000000003, 2.2550000000000003, 2.2550000000000003, 2.2550000000000003, 2.2550000000000003, 2.2550000000000003, 2.2550000000000003, 2.2550000000000003, 2.2550000000000003, 2.2550000000000003, 2.2550000000000003, 2.2550000000000003, 2.2550000000000003, 2.2550000000000003, 2.2550000000000003, 2.2550000000000003, 2.2550000000000003, 2.2550000000000003, 2.2550000000000003, 2.2550000000000003, 2.2550000000000003, 2.2550000000000003, 2.2550000000000003, 2.2550000000000003, 2.2550000000000003, 2.2550000000000003, 2.2550000000000003, 2.2550000000000003, 2.2550000000000003 ], [ 2.2900000000000005, 2.2900000000000005, 2.2900000000000005, 2.2900000000000005, 2.2900000000000005, 2.2900000000000005, 2.2900000000000005, 2.2900000000000005, 2.2900000000000005, 2.2900000000000005, 2.2900000000000005, 2.2900000000000005, 2.2900000000000005, 2.2900000000000005, 2.2900000000000005, 2.2900000000000005, 2.2900000000000005, 2.2900000000000005, 2.2900000000000005, 2.2900000000000005, 2.2900000000000005, 2.2900000000000005, 2.2900000000000005, 2.2900000000000005, 2.2900000000000005, 2.2900000000000005, 2.2900000000000005, 2.2900000000000005, 2.2900000000000005, 2.2900000000000005, 2.2900000000000005, 2.2900000000000005, 2.2900000000000005, 2.2900000000000005, 2.2900000000000005, 2.2900000000000005, 2.2900000000000005, 2.2900000000000005, 2.2900000000000005, 2.2900000000000005, 2.2900000000000005, 2.2900000000000005, 2.2900000000000005, 2.2900000000000005, 2.2900000000000005, 2.2900000000000005, 2.2900000000000005, 2.2900000000000005, 2.2900000000000005, 2.2900000000000005, 2.2900000000000005, 2.2900000000000005, 2.2900000000000005, 2.2900000000000005, 2.2900000000000005, 2.2900000000000005, 2.2900000000000005, 2.2900000000000005, 2.2900000000000005, 2.2900000000000005, 2.2900000000000005, 2.2900000000000005, 2.2900000000000005, 2.2900000000000005, 2.2900000000000005, 2.2900000000000005, 2.2900000000000005, 2.2900000000000005, 2.2900000000000005, 2.2900000000000005, 2.2900000000000005, 2.2900000000000005, 2.2900000000000005, 2.2900000000000005, 2.2900000000000005, 2.2900000000000005, 2.2900000000000005, 2.2900000000000005, 2.2900000000000005, 2.2900000000000005, 2.2900000000000005, 2.2900000000000005, 2.2900000000000005, 2.2900000000000005, 2.2900000000000005, 2.2900000000000005, 2.2900000000000005, 2.2900000000000005, 2.2900000000000005, 2.2900000000000005, 2.2900000000000005, 2.2900000000000005, 2.2900000000000005, 2.2900000000000005, 2.2900000000000005, 2.2900000000000005, 2.2900000000000005, 2.2900000000000005, 2.2900000000000005, 2.2900000000000005, 2.2900000000000005 ], [ 2.325, 2.325, 2.325, 2.325, 2.325, 2.325, 2.325, 2.325, 2.325, 2.325, 2.325, 2.325, 2.325, 2.325, 2.325, 2.325, 2.325, 2.325, 2.325, 2.325, 2.325, 2.325, 2.325, 2.325, 2.325, 2.325, 2.325, 2.325, 2.325, 2.325, 2.325, 2.325, 2.325, 2.325, 2.325, 2.325, 2.325, 2.325, 2.325, 2.325, 2.325, 2.325, 2.325, 2.325, 2.325, 2.325, 2.325, 2.325, 2.325, 2.325, 2.325, 2.325, 2.325, 2.325, 2.325, 2.325, 2.325, 2.325, 2.325, 2.325, 2.325, 2.325, 2.325, 2.325, 2.325, 2.325, 2.325, 2.325, 2.325, 2.325, 2.325, 2.325, 2.325, 2.325, 2.325, 2.325, 2.325, 2.325, 2.325, 2.325, 2.325, 2.325, 2.325, 2.325, 2.325, 2.325, 2.325, 2.325, 2.325, 2.325, 2.325, 2.325, 2.325, 2.325, 2.325, 2.325, 2.325, 2.325, 2.325, 2.325, 2.325 ], [ 2.3600000000000003, 2.3600000000000003, 2.3600000000000003, 2.3600000000000003, 2.3600000000000003, 2.3600000000000003, 2.3600000000000003, 2.3600000000000003, 2.3600000000000003, 2.3600000000000003, 2.3600000000000003, 2.3600000000000003, 2.3600000000000003, 2.3600000000000003, 2.3600000000000003, 2.3600000000000003, 2.3600000000000003, 2.3600000000000003, 2.3600000000000003, 2.3600000000000003, 2.3600000000000003, 2.3600000000000003, 2.3600000000000003, 2.3600000000000003, 2.3600000000000003, 2.3600000000000003, 2.3600000000000003, 2.3600000000000003, 2.3600000000000003, 2.3600000000000003, 2.3600000000000003, 2.3600000000000003, 2.3600000000000003, 2.3600000000000003, 2.3600000000000003, 2.3600000000000003, 2.3600000000000003, 2.3600000000000003, 2.3600000000000003, 2.3600000000000003, 2.3600000000000003, 2.3600000000000003, 2.3600000000000003, 2.3600000000000003, 2.3600000000000003, 2.3600000000000003, 2.3600000000000003, 2.3600000000000003, 2.3600000000000003, 2.3600000000000003, 2.3600000000000003, 2.3600000000000003, 2.3600000000000003, 2.3600000000000003, 2.3600000000000003, 2.3600000000000003, 2.3600000000000003, 2.3600000000000003, 2.3600000000000003, 2.3600000000000003, 2.3600000000000003, 2.3600000000000003, 2.3600000000000003, 2.3600000000000003, 2.3600000000000003, 2.3600000000000003, 2.3600000000000003, 2.3600000000000003, 2.3600000000000003, 2.3600000000000003, 2.3600000000000003, 2.3600000000000003, 2.3600000000000003, 2.3600000000000003, 2.3600000000000003, 2.3600000000000003, 2.3600000000000003, 2.3600000000000003, 2.3600000000000003, 2.3600000000000003, 2.3600000000000003, 2.3600000000000003, 2.3600000000000003, 2.3600000000000003, 2.3600000000000003, 2.3600000000000003, 2.3600000000000003, 2.3600000000000003, 2.3600000000000003, 2.3600000000000003, 2.3600000000000003, 2.3600000000000003, 2.3600000000000003, 2.3600000000000003, 2.3600000000000003, 2.3600000000000003, 2.3600000000000003, 2.3600000000000003, 2.3600000000000003, 2.3600000000000003, 2.3600000000000003 ], [ 2.3950000000000005, 2.3950000000000005, 2.3950000000000005, 2.3950000000000005, 2.3950000000000005, 2.3950000000000005, 2.3950000000000005, 2.3950000000000005, 2.3950000000000005, 2.3950000000000005, 2.3950000000000005, 2.3950000000000005, 2.3950000000000005, 2.3950000000000005, 2.3950000000000005, 2.3950000000000005, 2.3950000000000005, 2.3950000000000005, 2.3950000000000005, 2.3950000000000005, 2.3950000000000005, 2.3950000000000005, 2.3950000000000005, 2.3950000000000005, 2.3950000000000005, 2.3950000000000005, 2.3950000000000005, 2.3950000000000005, 2.3950000000000005, 2.3950000000000005, 2.3950000000000005, 2.3950000000000005, 2.3950000000000005, 2.3950000000000005, 2.3950000000000005, 2.3950000000000005, 2.3950000000000005, 2.3950000000000005, 2.3950000000000005, 2.3950000000000005, 2.3950000000000005, 2.3950000000000005, 2.3950000000000005, 2.3950000000000005, 2.3950000000000005, 2.3950000000000005, 2.3950000000000005, 2.3950000000000005, 2.3950000000000005, 2.3950000000000005, 2.3950000000000005, 2.3950000000000005, 2.3950000000000005, 2.3950000000000005, 2.3950000000000005, 2.3950000000000005, 2.3950000000000005, 2.3950000000000005, 2.3950000000000005, 2.3950000000000005, 2.3950000000000005, 2.3950000000000005, 2.3950000000000005, 2.3950000000000005, 2.3950000000000005, 2.3950000000000005, 2.3950000000000005, 2.3950000000000005, 2.3950000000000005, 2.3950000000000005, 2.3950000000000005, 2.3950000000000005, 2.3950000000000005, 2.3950000000000005, 2.3950000000000005, 2.3950000000000005, 2.3950000000000005, 2.3950000000000005, 2.3950000000000005, 2.3950000000000005, 2.3950000000000005, 2.3950000000000005, 2.3950000000000005, 2.3950000000000005, 2.3950000000000005, 2.3950000000000005, 2.3950000000000005, 2.3950000000000005, 2.3950000000000005, 2.3950000000000005, 2.3950000000000005, 2.3950000000000005, 2.3950000000000005, 2.3950000000000005, 2.3950000000000005, 2.3950000000000005, 2.3950000000000005, 2.3950000000000005, 2.3950000000000005, 2.3950000000000005, 2.3950000000000005 ], [ 2.43, 2.43, 2.43, 2.43, 2.43, 2.43, 2.43, 2.43, 2.43, 2.43, 2.43, 2.43, 2.43, 2.43, 2.43, 2.43, 2.43, 2.43, 2.43, 2.43, 2.43, 2.43, 2.43, 2.43, 2.43, 2.43, 2.43, 2.43, 2.43, 2.43, 2.43, 2.43, 2.43, 2.43, 2.43, 2.43, 2.43, 2.43, 2.43, 2.43, 2.43, 2.43, 2.43, 2.43, 2.43, 2.43, 2.43, 2.43, 2.43, 2.43, 2.43, 2.43, 2.43, 2.43, 2.43, 2.43, 2.43, 2.43, 2.43, 2.43, 2.43, 2.43, 2.43, 2.43, 2.43, 2.43, 2.43, 2.43, 2.43, 2.43, 2.43, 2.43, 2.43, 2.43, 2.43, 2.43, 2.43, 2.43, 2.43, 2.43, 2.43, 2.43, 2.43, 2.43, 2.43, 2.43, 2.43, 2.43, 2.43, 2.43, 2.43, 2.43, 2.43, 2.43, 2.43, 2.43, 2.43, 2.43, 2.43, 2.43, 2.43 ], [ 2.4650000000000003, 2.4650000000000003, 2.4650000000000003, 2.4650000000000003, 2.4650000000000003, 2.4650000000000003, 2.4650000000000003, 2.4650000000000003, 2.4650000000000003, 2.4650000000000003, 2.4650000000000003, 2.4650000000000003, 2.4650000000000003, 2.4650000000000003, 2.4650000000000003, 2.4650000000000003, 2.4650000000000003, 2.4650000000000003, 2.4650000000000003, 2.4650000000000003, 2.4650000000000003, 2.4650000000000003, 2.4650000000000003, 2.4650000000000003, 2.4650000000000003, 2.4650000000000003, 2.4650000000000003, 2.4650000000000003, 2.4650000000000003, 2.4650000000000003, 2.4650000000000003, 2.4650000000000003, 2.4650000000000003, 2.4650000000000003, 2.4650000000000003, 2.4650000000000003, 2.4650000000000003, 2.4650000000000003, 2.4650000000000003, 2.4650000000000003, 2.4650000000000003, 2.4650000000000003, 2.4650000000000003, 2.4650000000000003, 2.4650000000000003, 2.4650000000000003, 2.4650000000000003, 2.4650000000000003, 2.4650000000000003, 2.4650000000000003, 2.4650000000000003, 2.4650000000000003, 2.4650000000000003, 2.4650000000000003, 2.4650000000000003, 2.4650000000000003, 2.4650000000000003, 2.4650000000000003, 2.4650000000000003, 2.4650000000000003, 2.4650000000000003, 2.4650000000000003, 2.4650000000000003, 2.4650000000000003, 2.4650000000000003, 2.4650000000000003, 2.4650000000000003, 2.4650000000000003, 2.4650000000000003, 2.4650000000000003, 2.4650000000000003, 2.4650000000000003, 2.4650000000000003, 2.4650000000000003, 2.4650000000000003, 2.4650000000000003, 2.4650000000000003, 2.4650000000000003, 2.4650000000000003, 2.4650000000000003, 2.4650000000000003, 2.4650000000000003, 2.4650000000000003, 2.4650000000000003, 2.4650000000000003, 2.4650000000000003, 2.4650000000000003, 2.4650000000000003, 2.4650000000000003, 2.4650000000000003, 2.4650000000000003, 2.4650000000000003, 2.4650000000000003, 2.4650000000000003, 2.4650000000000003, 2.4650000000000003, 2.4650000000000003, 2.4650000000000003, 2.4650000000000003, 2.4650000000000003, 2.4650000000000003 ], [ 2.5, 2.5, 2.5, 2.5, 2.5, 2.5, 2.5, 2.5, 2.5, 2.5, 2.5, 2.5, 2.5, 2.5, 2.5, 2.5, 2.5, 2.5, 2.5, 2.5, 2.5, 2.5, 2.5, 2.5, 2.5, 2.5, 2.5, 2.5, 2.5, 2.5, 2.5, 2.5, 2.5, 2.5, 2.5, 2.5, 2.5, 2.5, 2.5, 2.5, 2.5, 2.5, 2.5, 2.5, 2.5, 2.5, 2.5, 2.5, 2.5, 2.5, 2.5, 2.5, 2.5, 2.5, 2.5, 2.5, 2.5, 2.5, 2.5, 2.5, 2.5, 2.5, 2.5, 2.5, 2.5, 2.5, 2.5, 2.5, 2.5, 2.5, 2.5, 2.5, 2.5, 2.5, 2.5, 2.5, 2.5, 2.5, 2.5, 2.5, 2.5, 2.5, 2.5, 2.5, 2.5, 2.5, 2.5, 2.5, 2.5, 2.5, 2.5, 2.5, 2.5, 2.5, 2.5, 2.5, 2.5, 2.5, 2.5, 2.5, 2.5 ] ], "z": [ [ 12, 11.64405, 11.2962, 10.95645, 10.6248, 10.30125, 9.9858, 9.67845, 9.3792, 9.088049999999999, 8.805, 8.53005, 8.2632, 8.00445, 7.7538, 7.51125, 7.2768, 7.05045, 6.8322, 6.62205, 6.42, 6.226050000000001, 6.0402000000000005, 5.862450000000001, 5.6928, 5.53125, 5.377800000000001, 5.23245, 5.0952, 4.96605, 4.845000000000001, 4.73205, 4.6272, 4.53045, 4.4418, 4.36125, 4.2888, 4.22445, 4.168200000000001, 4.12005, 4.08, 4.04805, 4.0242, 4.00845, 4.0008, 4.001250000000001, 4.0098, 4.0264500000000005, 4.0512, 4.0840499999999995, 4.125, 4.174049999999999, 4.231199999999999, 4.29645, 4.3698, 4.45125, 4.5408, 4.63845, 4.744199999999999, 4.8580499999999995, 4.9799999999999995, 5.11005, 5.2482, 5.39445, 5.5488, 5.71125, 5.881799999999998, 6.0604499999999994, 6.247199999999999, 6.44205, 6.645, 6.85605, 7.075199999999999, 7.3024499999999986, 7.537800000000001, 7.78125, 8.0328, 8.292449999999999, 8.560199999999998, 8.836049999999998, 9.119999999999997, 9.41205, 9.7122, 10.020449999999999, 10.336799999999998, 10.661249999999997, 10.993799999999998, 11.33445, 11.6832, 12.04005, 12.404999999999998, 12.778049999999997, 13.159199999999998, 13.548449999999995, 13.945799999999995, 14.351249999999997, 14.764800000000001, 15.18645, 15.616200000000003, 16.05405, 16.5 ], [ 11.861225, 11.505275, 11.157425, 10.817675, 10.486025, 10.162475, 9.847024999999999, 9.539674999999999, 9.240425, 8.949275, 8.666225, 8.391275, 8.124424999999999, 7.865675, 7.615025, 7.3724750000000006, 7.138025, 6.911675, 6.693425, 6.483275, 6.281225, 6.087275, 5.901425, 5.723675, 5.554024999999999, 5.392475, 5.239025, 5.093675, 4.956424999999999, 4.827275, 4.706225, 4.593275, 4.488425, 4.391675, 4.303025, 4.222475, 4.150025, 4.085675, 4.029425, 3.981275, 3.941225, 3.909275, 3.8854249999999997, 3.869675, 3.862025, 3.862475, 3.8710249999999995, 3.8876749999999998, 3.912425, 3.9452749999999996, 3.9862249999999992, 4.0352749999999995, 4.0924249999999995, 4.157674999999999, 4.231024999999999, 4.312474999999999, 4.402025, 4.499675, 4.6054249999999985, 4.719275, 4.841225, 4.971275, 5.109425, 5.255675, 5.410024999999999, 5.572474999999999, 5.743024999999999, 5.921674999999999, 6.108424999999999, 6.303274999999999, 6.506224999999999, 6.717274999999999, 6.936424999999998, 7.163674999999998, 7.399025, 7.642474999999999, 7.894024999999999, 8.153674999999998, 8.421424999999997, 8.697274999999998, 8.981224999999997, 9.273275, 9.573424999999999, 9.881674999999998, 10.198025, 10.522474999999998, 10.855025, 11.195675000000001, 11.544425, 11.901275000000002, 12.266224999999999, 12.639274999999998, 13.020425, 13.409674999999996, 13.807024999999996, 14.212474999999998, 14.626025000000002, 15.047675000000002, 15.477425000000004, 15.915275000000001, 16.361225 ], [ 11.7249, 11.36895, 11.0211, 10.68135, 10.3497, 10.02615, 9.7107, 9.40335, 9.104099999999999, 8.812949999999999, 8.5299, 8.25495, 7.988099999999999, 7.72935, 7.4787, 7.23615, 7.0017, 6.7753499999999995, 6.5571, 6.34695, 6.1449, 5.95095, 5.7651, 5.58735, 5.4177, 5.25615, 5.1027000000000005, 4.95735, 4.8201, 4.69095, 4.5699000000000005, 4.45695, 4.3521, 4.25535, 4.1667, 4.08615, 4.0137, 3.94935, 3.8931, 3.84495, 3.8049, 3.77295, 3.7491, 3.7333499999999997, 3.7257, 3.72615, 3.7346999999999997, 3.75135, 3.7761, 3.80895, 3.8499, 3.8989499999999997, 3.9560999999999997, 4.02135, 4.0947, 4.17615, 4.265699999999999, 4.363349999999999, 4.469099999999999, 4.5829499999999985, 4.7048999999999985, 4.834949999999999, 4.973099999999999, 5.119349999999999, 5.2737, 5.43615, 5.606699999999998, 5.785349999999999, 5.972099999999999, 6.16695, 6.3698999999999995, 6.58095, 6.800099999999999, 7.027349999999998, 7.262700000000001, 7.50615, 7.7577, 8.017349999999999, 8.285099999999998, 8.560949999999998, 8.844899999999997, 9.13695, 9.4371, 9.745349999999998, 10.061699999999997, 10.386149999999995, 10.718699999999997, 11.059349999999998, 11.408099999999997, 11.764949999999999, 12.129899999999996, 12.502949999999995, 12.884099999999997, 13.273349999999994, 13.670699999999993, 14.076149999999995, 14.4897, 14.911349999999999, 15.3411, 15.778949999999998, 16.224899999999998 ], [ 11.591025, 11.235074999999998, 10.887225, 10.547474999999999, 10.215825, 9.892275000000001, 9.576825, 9.269475, 8.970225, 8.679075000000001, 8.396025, 8.121075000000001, 7.854225, 7.595475, 7.344825, 7.102275000000001, 6.867825, 6.641475, 6.423225, 6.213075, 6.011025, 5.817075, 5.631225000000001, 5.453475, 5.283825, 5.122275, 4.968825000000001, 4.823475, 4.686225, 4.557075, 4.436025000000001, 4.323075, 4.218225, 4.121475, 4.032825, 3.952275, 3.8798250000000003, 3.815475, 3.7592250000000003, 3.711075, 3.671025, 3.639075, 3.615225, 3.599475, 3.591825, 3.5922750000000003, 3.600825, 3.617475, 3.6422250000000003, 3.675075, 3.716025, 3.7650749999999995, 3.8222249999999995, 3.8874749999999993, 3.960825, 4.042275, 4.131825, 4.229475, 4.335224999999999, 4.449075, 4.571025, 4.701075, 4.839225, 4.985475, 5.139825, 5.302274999999999, 5.472824999999999, 5.651474999999999, 5.838224999999999, 6.033074999999999, 6.236024999999999, 6.447074999999999, 6.666224999999998, 6.893474999999998, 7.128825, 7.372274999999999, 7.623824999999999, 7.883474999999998, 8.151224999999997, 8.427074999999999, 8.711024999999996, 9.003074999999999, 9.303224999999998, 9.611474999999999, 9.927824999999999, 10.252274999999997, 10.584824999999999, 10.925474999999999, 11.274224999999998, 11.631075, 11.996024999999996, 12.369074999999995, 12.750224999999997, 13.139474999999994, 13.536824999999993, 13.942274999999995, 14.355825, 14.777474999999999, 15.207225000000001, 15.645074999999999, 16.091025 ], [ 11.4596, 11.10365, 10.7558, 10.416049999999998, 10.084399999999999, 9.760850000000001, 9.4454, 9.13805, 8.838799999999999, 8.54765, 8.2646, 7.98965, 7.722799999999999, 7.46405, 7.2134, 6.97085, 6.7364, 6.51005, 6.2918, 6.08165, 5.8796, 5.68565, 5.4998000000000005, 5.32205, 5.1524, 4.99085, 4.837400000000001, 4.69205, 4.5548, 4.42565, 4.304600000000001, 4.19165, 4.0868, 3.99005, 3.9013999999999998, 3.82085, 3.7484, 3.68405, 3.6277999999999997, 3.57965, 3.5396, 3.50765, 3.4837999999999996, 3.46805, 3.4604, 3.4608499999999998, 3.4693999999999994, 3.4860499999999996, 3.5107999999999997, 3.5436499999999995, 3.5846, 3.6336499999999994, 3.6907999999999994, 3.7560499999999992, 3.8293999999999997, 3.91085, 4.0004, 4.09805, 4.203799999999999, 4.3176499999999995, 4.4395999999999995, 4.56965, 4.7078, 4.85405, 5.0084, 5.170849999999999, 5.341399999999999, 5.520049999999999, 5.7067999999999985, 5.901649999999999, 6.104599999999999, 6.315649999999999, 6.534799999999998, 6.762049999999998, 6.9974, 7.240849999999999, 7.492399999999999, 7.752049999999998, 8.019799999999996, 8.295649999999998, 8.579599999999996, 8.871649999999999, 9.171799999999998, 9.480049999999999, 9.796399999999998, 10.120849999999997, 10.453399999999998, 10.794049999999999, 11.1428, 11.49965, 11.864599999999998, 12.237649999999997, 12.618799999999998, 13.008049999999995, 13.405399999999995, 13.810849999999997, 14.224400000000001, 14.64605, 15.075800000000003, 15.51365, 15.9596 ], [ 11.330625, 10.974675, 10.626825, 10.287075, 9.955425, 9.631875, 9.316425, 9.009075000000001, 8.709825, 8.418675, 8.135625000000001, 7.8606750000000005, 7.593824999999999, 7.335075, 7.0844249999999995, 6.841875, 6.607425, 6.381075, 6.162825, 5.952675, 5.750625, 5.556675, 5.370825, 5.193075, 5.023425, 4.8618749999999995, 4.708425, 4.563075, 4.425825, 4.2966750000000005, 4.175625, 4.062675, 3.9578249999999997, 3.861075, 3.7724249999999997, 3.691875, 3.619425, 3.5550749999999995, 3.498825, 3.450675, 3.410625, 3.378675, 3.354825, 3.339075, 3.331425, 3.331875, 3.3404249999999998, 3.357075, 3.381825, 3.414675, 3.455625, 3.5046749999999993, 3.5618249999999994, 3.627074999999999, 3.7004249999999996, 3.781875, 3.871425, 3.9690749999999997, 4.074824999999999, 4.188675, 4.310625, 4.440675000000001, 4.578825, 4.725075, 4.8794249999999995, 5.041874999999999, 5.212425, 5.391074999999999, 5.577824999999999, 5.772675, 5.975624999999999, 6.186674999999999, 6.405824999999998, 6.633074999999998, 6.868425, 7.1118749999999995, 7.363424999999999, 7.623074999999998, 7.890824999999998, 8.166674999999998, 8.450624999999997, 8.742675, 9.042824999999999, 9.351074999999998, 9.667424999999998, 9.991874999999997, 10.324424999999998, 10.665075, 11.013824999999999, 11.370675, 11.735624999999997, 12.108674999999996, 12.489824999999998, 12.879074999999995, 13.276424999999994, 13.681874999999996, 14.095425, 14.517075, 14.946825000000002, 15.384675, 15.830625 ], [ 11.2041, 10.84815, 10.500300000000001, 10.16055, 9.8289, 9.50535, 9.1899, 8.88255, 8.583300000000001, 8.29215, 8.0091, 7.73415, 7.4673, 7.208550000000001, 6.9579, 6.715350000000001, 6.4809, 6.25455, 6.036300000000001, 5.82615, 5.6241, 5.43015, 5.2443, 5.06655, 4.8969000000000005, 4.73535, 4.581900000000001, 4.43655, 4.299300000000001, 4.1701500000000005, 4.049100000000001, 3.93615, 3.8313000000000006, 3.7345500000000005, 3.6459, 3.5653500000000005, 3.4929000000000006, 3.42855, 3.3723, 3.3241500000000004, 3.2841000000000005, 3.2521500000000003, 3.2283, 3.2125500000000002, 3.2049000000000003, 3.20535, 3.2138999999999998, 3.23055, 3.2553, 3.28815, 3.3291000000000004, 3.3781499999999998, 3.4353, 3.5005499999999996, 3.5739, 3.6553500000000003, 3.7449000000000003, 3.84255, 3.9482999999999997, 4.06215, 4.1841, 4.314150000000001, 4.4523, 4.59855, 4.7529, 4.91535, 5.085899999999999, 5.26455, 5.4513, 5.6461500000000004, 5.8491, 6.06015, 6.279299999999999, 6.506549999999999, 6.741900000000001, 6.98535, 7.2369, 7.496549999999999, 7.764299999999999, 8.040149999999999, 8.324099999999998, 8.616150000000001, 8.9163, 9.224549999999999, 9.540899999999997, 9.865349999999996, 10.197899999999997, 10.538549999999999, 10.887299999999998, 11.24415, 11.609099999999996, 11.982149999999995, 12.363299999999997, 12.752549999999994, 13.149899999999993, 13.555349999999995, 13.9689, 14.39055, 14.820300000000001, 15.258149999999999, 15.704099999999999 ], [ 11.080025000000001, 10.724075, 10.376225, 10.036475, 9.704825, 9.381275, 9.065825, 8.758475, 8.459225, 8.168075, 7.885025000000001, 7.610075, 7.3432249999999994, 7.084475, 6.833825, 6.591275, 6.356825000000001, 6.130475, 5.912225, 5.702075, 5.500025000000001, 5.306075, 5.120225, 4.942475, 4.772824999999999, 4.611275, 4.457825, 4.312475000000001, 4.175225, 4.046075, 3.925025, 3.812075, 3.707225, 3.610475, 3.5218249999999998, 3.441275, 3.368825, 3.304475, 3.248225, 3.200075, 3.160025, 3.128075, 3.1042249999999996, 3.088475, 3.080825, 3.0812749999999998, 3.0898249999999994, 3.1064749999999997, 3.1312249999999997, 3.1640749999999995, 3.205025, 3.2540749999999994, 3.3112249999999994, 3.3764749999999992, 3.4498249999999997, 3.531275, 3.620825, 3.7184749999999998, 3.8242249999999993, 3.9380749999999995, 4.0600249999999996, 4.190075, 4.328225, 4.474475, 4.628825, 4.791275, 4.961824999999999, 5.140474999999999, 5.3272249999999985, 5.522074999999999, 5.725024999999999, 5.936074999999999, 6.155224999999998, 6.382474999999998, 6.617825, 6.861274999999999, 7.112824999999999, 7.372474999999998, 7.640224999999997, 7.9160749999999975, 8.200024999999997, 8.492075, 8.792224999999998, 9.100474999999998, 9.416825, 9.741274999999998, 10.073825, 10.414475000000001, 10.763225, 11.120075000000002, 11.485024999999998, 11.858074999999998, 12.239225, 12.628474999999996, 13.025824999999996, 13.431274999999998, 13.844825000000002, 14.266475000000002, 14.696225000000004, 15.134075000000001, 15.580025000000001 ], [ 10.9584, 10.60245, 10.2546, 9.91485, 9.5832, 9.25965, 8.9442, 8.63685, 8.3376, 8.04645, 7.763400000000001, 7.48845, 7.2216, 6.9628499999999995, 6.712199999999999, 6.46965, 6.2352, 6.00885, 5.7906, 5.58045, 5.3784, 5.18445, 4.9986, 4.82085, 4.651199999999999, 4.48965, 4.3362, 4.19085, 4.053599999999999, 3.92445, 3.8034000000000003, 3.69045, 3.5856000000000003, 3.4888500000000002, 3.4002, 3.31965, 3.2472000000000003, 3.1828499999999997, 3.1266, 3.07845, 3.0384, 3.00645, 2.9826, 2.96685, 2.9591999999999996, 2.9596500000000003, 2.9682, 2.9848500000000002, 3.0096000000000003, 3.04245, 3.0833999999999997, 3.13245, 3.1896, 3.25485, 3.3281999999999994, 3.4096499999999996, 3.4991999999999996, 3.5968499999999994, 3.702599999999999, 3.8164499999999992, 3.9383999999999992, 4.06845, 4.2066, 4.35285, 4.507199999999999, 4.669649999999999, 4.840199999999999, 5.018849999999999, 5.205599999999999, 5.400449999999999, 5.603399999999999, 5.814449999999999, 6.033599999999998, 6.260849999999998, 6.4962, 6.739649999999999, 6.991199999999999, 7.250849999999998, 7.5185999999999975, 7.794449999999998, 8.078399999999997, 8.37045, 8.670599999999999, 8.978849999999998, 9.295199999999998, 9.619649999999996, 9.952199999999998, 10.29285, 10.641599999999999, 10.99845, 11.363399999999997, 11.736449999999996, 12.117599999999998, 12.506849999999995, 12.904199999999994, 13.309649999999996, 13.7232, 14.14485, 14.574600000000002, 15.01245, 15.4584 ], [ 10.839224999999999, 10.483274999999999, 10.135425000000001, 9.795675, 9.464025, 9.140475, 8.825025, 8.517675, 8.218425, 7.927275, 7.6442250000000005, 7.369275, 7.102424999999999, 6.843675, 6.593025, 6.350475, 6.116025, 5.8896749999999995, 5.671425, 5.461275, 5.259225000000001, 5.065275, 4.8794249999999995, 4.701675, 4.532024999999999, 4.370475, 4.217025, 4.071675000000001, 3.934425, 3.805275, 3.684225, 3.571275, 3.466425, 3.369675, 3.2810249999999996, 3.200475, 3.128025, 3.063675, 3.007425, 2.959275, 2.919225, 2.887275, 2.8634249999999994, 2.8476749999999997, 2.840025, 2.8404749999999996, 2.8490249999999993, 2.8656749999999995, 2.8904249999999996, 2.9232749999999994, 2.964225, 3.0132749999999993, 3.070425, 3.135675, 3.2090249999999996, 3.290475, 3.380025, 3.4776749999999996, 3.583424999999999, 3.6972749999999994, 3.8192249999999994, 3.949275, 4.087425, 4.233675, 4.388025, 4.550475, 4.721025, 4.899674999999999, 5.086424999999999, 5.281275, 5.4842249999999995, 5.695275, 5.914424999999999, 6.141674999999998, 6.377025000000001, 6.620475, 6.872025, 7.131674999999999, 7.399424999999998, 7.675274999999998, 7.959224999999997, 8.251275, 8.551424999999998, 8.859675, 9.176025, 9.500474999999998, 9.833025, 10.173675000000003, 10.522425000000002, 10.879275000000003, 11.244224999999998, 11.617274999999998, 11.998425, 12.387674999999996, 12.785024999999996, 13.190474999999998, 13.604025000000002, 14.025675000000001, 14.455425000000004, 14.893275000000001, 15.339225 ], [ 10.7225, 10.36655, 10.018699999999999, 9.67895, 9.3473, 9.02375, 8.7083, 8.40095, 8.1017, 7.810549999999999, 7.5275, 7.252549999999999, 6.9857, 6.72695, 6.4763, 6.233750000000001, 5.9993, 5.77295, 5.5547, 5.34455, 5.1425, 4.94855, 4.7627, 4.58495, 4.415299999999999, 4.25375, 4.1003, 3.95495, 3.8176999999999994, 3.6885499999999998, 3.5675, 3.45455, 3.3497, 3.2529500000000002, 3.1643, 3.0837499999999998, 3.0113, 2.9469499999999997, 2.8907, 2.84255, 2.8024999999999998, 2.7705499999999996, 2.7466999999999997, 2.73095, 2.7232999999999996, 2.72375, 2.7322999999999995, 2.74895, 2.7737, 2.8065499999999997, 2.8474999999999997, 2.8965499999999995, 2.9536999999999995, 3.0189499999999994, 3.092299999999999, 3.173749999999999, 3.2633, 3.36095, 3.4666999999999986, 3.5805499999999997, 3.7024999999999997, 3.8325500000000003, 3.9707, 4.11695, 4.271299999999999, 4.433749999999999, 4.604299999999999, 4.782949999999999, 4.969699999999999, 5.164549999999999, 5.367499999999999, 5.578549999999999, 5.797699999999998, 6.024949999999998, 6.2603, 6.503749999999999, 6.755299999999999, 7.014949999999998, 7.2826999999999975, 7.558549999999998, 7.842499999999997, 8.13455, 8.4347, 8.742949999999997, 9.059299999999997, 9.383749999999996, 9.716299999999997, 10.05695, 10.4057, 10.762550000000001, 11.127499999999998, 11.500549999999997, 11.881699999999999, 12.270949999999996, 12.668299999999995, 13.073749999999997, 13.487300000000001, 13.90895, 14.338700000000003, 14.77655, 15.2225 ], [ 10.608225000000001, 10.252275, 9.904425, 9.564675, 9.233025000000001, 8.909475, 8.594025, 8.286675, 7.987425, 7.696275, 7.413225000000001, 7.138275, 6.871425, 6.612674999999999, 6.362025, 6.119475, 5.885025000000001, 5.658675, 5.440425, 5.230275, 5.028225000000001, 4.834275, 4.648425, 4.470675, 4.301024999999999, 4.139475, 3.986025, 3.8406750000000005, 3.703425, 3.574275, 3.453225, 3.340275, 3.235425, 3.138675, 3.0500249999999998, 2.969475, 2.897025, 2.832675, 2.776425, 2.728275, 2.688225, 2.656275, 2.632425, 2.616675, 2.609025, 2.609475, 2.618025, 2.634675, 2.659425, 2.692275, 2.733225, 2.782275, 2.839425, 2.9046749999999997, 2.9780249999999993, 3.0594749999999995, 3.1490249999999995, 3.2466749999999993, 3.352424999999999, 3.466274999999999, 3.588224999999999, 3.7182749999999998, 3.8564249999999993, 4.002675, 4.157024999999999, 4.319474999999999, 4.490024999999999, 4.668675, 4.855425, 5.050275000000001, 5.2532250000000005, 5.464275000000001, 5.683425, 5.9106749999999995, 6.146025000000002, 6.389475000000001, 6.641025000000001, 6.900675, 7.168424999999999, 7.444274999999999, 7.728224999999998, 8.020275000000002, 8.320425, 8.628675, 8.945025, 9.269474999999998, 9.602025, 9.942675000000001, 10.291425, 10.648275000000002, 11.013224999999998, 11.386274999999998, 11.767425, 12.156674999999996, 12.554024999999996, 12.959474999999998, 13.373025000000002, 13.794675000000002, 14.224425000000004, 14.662275000000001, 15.108225000000001 ], [ 10.4964, 10.14045, 9.7926, 9.45285, 9.121200000000002, 8.79765, 8.482199999999999, 8.17485, 7.8756, 7.5844499999999995, 7.3014, 7.02645, 6.7596, 6.50085, 6.2501999999999995, 6.00765, 5.7732, 5.54685, 5.3286, 5.11845, 4.9164, 4.72245, 4.5366, 4.35885, 4.1892, 4.0276499999999995, 3.8742, 3.7288500000000004, 3.5915999999999997, 3.46245, 3.3414, 3.2284499999999996, 3.1235999999999997, 3.0268500000000005, 2.9382, 2.85765, 2.7852, 2.7208499999999995, 2.6646, 2.6164500000000004, 2.5764, 2.54445, 2.5206, 2.5048500000000002, 2.4971999999999994, 2.49765, 2.5061999999999998, 2.52285, 2.5476, 2.58045, 2.6213999999999995, 2.6704499999999998, 2.7276, 2.7928499999999996, 2.8662, 2.9476500000000003, 3.0372000000000003, 3.13485, 3.2405999999999997, 3.35445, 3.4764, 3.6064500000000006, 3.7446, 3.8908500000000004, 4.0452, 4.20765, 4.3782, 4.556849999999999, 4.743599999999999, 4.93845, 5.141399999999999, 5.352449999999999, 5.571599999999998, 5.798849999999998, 6.0342, 6.2776499999999995, 6.5291999999999994, 6.788849999999998, 7.056599999999998, 7.332449999999998, 7.616399999999997, 7.90845, 8.208599999999999, 8.516849999999998, 8.833199999999998, 9.157649999999999, 9.4902, 9.830850000000002, 10.1796, 10.536450000000002, 10.901399999999999, 11.274449999999998, 11.6556, 12.044849999999997, 12.442199999999996, 12.847649999999998, 13.261200000000002, 13.682850000000002, 14.112600000000004, 14.550450000000001, 14.996400000000001 ], [ 10.387025, 10.031075, 9.683225, 9.343475, 9.011825, 8.688275, 8.372824999999999, 8.065475, 7.766225, 7.4750749999999995, 7.192025, 6.917075, 6.650225, 6.391475, 6.1408249999999995, 5.898275, 5.663825, 5.437474999999999, 5.219225, 5.009075, 4.807025, 4.613075, 4.427225, 4.249475, 4.079825, 3.9182749999999995, 3.764825, 3.6194750000000004, 3.4822249999999997, 3.353075, 3.232025, 3.1190749999999996, 3.0142249999999997, 2.917475, 2.8288249999999997, 2.7482749999999996, 2.6758249999999997, 2.6114749999999995, 2.555225, 2.507075, 2.4670249999999996, 2.4350749999999994, 2.411225, 2.395475, 2.387825, 2.388275, 2.3968249999999998, 2.413475, 2.438225, 2.471075, 2.512025, 2.5610749999999998, 2.618225, 2.6834749999999996, 2.756824999999999, 2.8382749999999994, 2.9278249999999995, 3.0254749999999992, 3.131224999999999, 3.245074999999999, 3.367024999999999, 3.4970749999999997, 3.6352249999999993, 3.7814749999999995, 3.9358249999999995, 4.098274999999999, 4.268825, 4.447474999999999, 4.634224999999999, 4.829075, 5.032024999999999, 5.243074999999999, 5.462224999999998, 5.689474999999998, 5.924825, 6.1682749999999995, 6.4198249999999994, 6.679474999999998, 6.947224999999998, 7.223074999999998, 7.507024999999997, 7.799075, 8.099224999999999, 8.407474999999998, 8.723824999999998, 9.048274999999999, 9.380825, 9.721475000000002, 10.070225, 10.427075000000002, 10.792024999999999, 11.165074999999998, 11.546225, 11.935474999999997, 12.332824999999996, 12.738274999999998, 13.151825000000002, 13.573475000000002, 14.003225000000004, 14.441075000000001, 14.887025000000001 ], [ 10.2801, 9.92415, 9.5763, 9.23655, 8.9049, 8.58135, 8.2659, 7.958550000000001, 7.6593, 7.36815, 7.085100000000001, 6.81015, 6.5433, 6.28455, 6.0339, 5.79135, 5.556900000000001, 5.33055, 5.1123, 4.902150000000001, 4.700100000000001, 4.50615, 4.3203, 4.14255, 3.9728999999999997, 3.81135, 3.6579, 3.5125500000000005, 3.3753, 3.24615, 3.1251, 3.01215, 2.9073, 2.81055, 2.7218999999999998, 2.64135, 2.5689, 2.50455, 2.4483, 2.40015, 2.3601, 2.32815, 2.3043, 2.2885500000000003, 2.2809, 2.28135, 2.2899, 2.30655, 2.3313, 2.36415, 2.4051, 2.45415, 2.5113, 2.5765499999999997, 2.6498999999999993, 2.7313499999999995, 2.8209000000000004, 2.91855, 3.024299999999999, 3.13815, 3.2601, 3.3901500000000007, 3.5283, 3.6745500000000004, 3.8288999999999995, 3.9913499999999993, 4.161899999999999, 4.340549999999999, 4.5272999999999985, 4.722149999999999, 4.925099999999999, 5.136149999999999, 5.355299999999998, 5.582549999999998, 5.8179, 6.061349999999999, 6.312899999999999, 6.572549999999998, 6.840299999999997, 7.1161499999999975, 7.400099999999997, 7.69215, 7.992299999999998, 8.300549999999998, 8.616899999999998, 8.941349999999998, 9.2739, 9.614550000000001, 9.9633, 10.320150000000002, 10.685099999999998, 11.058149999999998, 11.4393, 11.828549999999996, 12.225899999999996, 12.631349999999998, 13.044900000000002, 13.466550000000002, 13.896300000000004, 14.334150000000001, 14.780100000000001 ], [ 10.175625, 9.819675, 9.471824999999999, 9.132075, 8.800425, 8.476875, 8.161425000000001, 7.854075000000001, 7.554825, 7.263675, 6.980625, 6.705675, 6.438825, 6.180075, 5.929425, 5.686875000000001, 5.452425, 5.226075, 5.007825, 4.797675, 4.595625, 4.401675, 4.215825, 4.038075, 3.868425, 3.706875, 3.5534250000000003, 3.408075, 3.270825, 3.141675, 3.0206250000000003, 2.9076750000000002, 2.8028250000000003, 2.7060750000000002, 2.617425, 2.536875, 2.4644250000000003, 2.400075, 2.343825, 2.295675, 2.255625, 2.223675, 2.1998249999999997, 2.184075, 2.176425, 2.176875, 2.1854249999999995, 2.202075, 2.226825, 2.2596749999999997, 2.300625, 2.3496749999999995, 2.4068249999999995, 2.4720750000000002, 2.545425, 2.626875, 2.716425, 2.814075, 2.9198249999999994, 3.0336749999999997, 3.1556249999999997, 3.2856750000000003, 3.423825, 3.570075, 3.724425, 3.886875, 4.057424999999999, 4.2360750000000005, 4.4228250000000005, 4.617675000000001, 4.820625000000001, 5.031675000000001, 5.250825, 5.478075, 5.713425000000002, 5.956875000000001, 6.208425000000001, 6.468075, 6.735824999999999, 7.011674999999999, 7.2956249999999985, 7.587675000000002, 7.887825, 8.196075, 8.512425, 8.836875, 9.169425, 9.510075, 9.858825, 10.215675000000001, 10.580624999999998, 10.953674999999997, 11.334824999999999, 11.724074999999996, 12.121424999999995, 12.526874999999997, 12.940425000000001, 13.362075, 13.791825000000003, 14.229675, 14.675625 ], [ 10.073599999999999, 9.717649999999999, 9.369800000000001, 9.03005, 8.6984, 8.37485, 8.0594, 7.7520500000000006, 7.4528, 7.16165, 6.8786000000000005, 6.60365, 6.3368, 6.07805, 5.8274, 5.58485, 5.3504000000000005, 5.1240499999999995, 4.9058, 4.69565, 4.493600000000001, 4.29965, 4.1137999999999995, 3.9360500000000003, 3.7663999999999995, 3.60485, 3.4514, 3.3060500000000004, 3.1688, 3.03965, 2.9186, 2.80565, 2.7008, 2.60405, 2.5153999999999996, 2.43485, 2.3624, 2.29805, 2.2418, 2.19365, 2.1536, 2.12165, 2.0978, 2.0820499999999997, 2.0744, 2.07485, 2.0833999999999997, 2.10005, 2.1248, 2.15765, 2.1986, 2.2476499999999997, 2.3047999999999997, 2.370049999999999, 2.4433999999999996, 2.52485, 2.6144, 2.7120499999999996, 2.817799999999999, 2.9316499999999994, 3.0535999999999994, 3.18365, 3.3217999999999996, 3.46805, 3.6224, 3.7848499999999996, 3.955399999999999, 4.134049999999999, 4.320799999999999, 4.51565, 4.7185999999999995, 4.92965, 5.148799999999999, 5.376049999999998, 5.611400000000001, 5.85485, 6.1064, 6.366049999999999, 6.633799999999998, 6.909649999999998, 7.193599999999997, 7.485650000000001, 7.785799999999999, 8.09405, 8.4104, 8.734849999999998, 9.0674, 9.40805, 9.756799999999998, 10.11365, 10.478599999999997, 10.851649999999996, 11.232799999999997, 11.622049999999994, 12.019399999999994, 12.424849999999996, 12.8384, 13.26005, 13.689800000000002, 14.12765, 14.573599999999999 ], [ 9.974025000000001, 9.618075, 9.270225, 8.930475, 8.598825, 8.275275, 7.9598249999999995, 7.652475, 7.353225, 7.062074999999999, 6.779025, 6.504074999999999, 6.237225, 5.9784749999999995, 5.727824999999999, 5.485275, 5.250825, 5.024475, 4.8062249999999995, 4.596075, 4.394025, 4.200075, 4.014225, 3.836475, 3.6668249999999993, 3.5052749999999997, 3.351825, 3.206475, 3.069225, 2.940075, 2.819025, 2.706075, 2.601225, 2.5044750000000002, 2.415825, 2.3352749999999998, 2.262825, 2.1984749999999997, 2.142225, 2.094075, 2.0540249999999998, 2.0220749999999996, 1.9982249999999997, 1.9824749999999998, 1.9748249999999998, 1.9752750000000001, 1.9838249999999997, 2.000475, 2.025225, 2.0580749999999997, 2.099025, 2.1480749999999995, 2.2052249999999995, 2.2704749999999994, 2.343824999999999, 2.425274999999999, 2.514824999999999, 2.612474999999999, 2.7182249999999986, 2.832074999999999, 2.954024999999999, 3.0840749999999995, 3.222224999999999, 3.368474999999999, 3.522824999999999, 3.685274999999999, 3.8558249999999985, 4.034475, 4.221225, 4.416075, 4.619025, 4.830075, 5.049224999999999, 5.276474999999999, 5.511825000000001, 5.755275, 6.006825, 6.266474999999999, 6.534224999999998, 6.8100749999999985, 7.094024999999998, 7.386075000000001, 7.686224999999999, 7.994474999999999, 8.310824999999998, 8.635274999999996, 8.967824999999998, 9.308474999999998, 9.657224999999997, 10.014074999999998, 10.379024999999995, 10.752074999999994, 11.133224999999998, 11.522474999999995, 11.919824999999994, 12.325274999999996, 12.738825, 13.160475, 13.590225000000002, 14.028075, 14.474025 ], [ 9.8769, 9.52095, 9.1731, 8.83335, 8.5017, 8.17815, 7.8627, 7.555350000000001, 7.2561, 6.96495, 6.681900000000001, 6.40695, 6.1400999999999994, 5.881349999999999, 5.630699999999999, 5.3881499999999996, 5.1537, 4.92735, 4.709099999999999, 4.49895, 4.2969, 4.10295, 3.9170999999999996, 3.73935, 3.569699999999999, 3.4081499999999996, 3.2546999999999997, 3.10935, 2.9720999999999997, 2.84295, 2.7218999999999998, 2.6089499999999997, 2.5040999999999998, 2.40735, 2.3186999999999998, 2.2381499999999996, 2.1656999999999997, 2.1013499999999996, 2.0450999999999997, 1.99695, 1.9568999999999996, 1.9249499999999995, 1.9010999999999996, 1.8853499999999999, 1.8776999999999995, 1.8781499999999998, 1.8866999999999994, 1.9033499999999997, 1.9280999999999997, 1.9609499999999995, 2.0018999999999996, 2.0509499999999994, 2.1080999999999994, 2.1733499999999992, 2.2466999999999993, 2.3281499999999995, 2.4176999999999995, 2.5153499999999993, 2.621099999999999, 2.734949999999999, 2.856899999999999, 2.9869499999999998, 3.1250999999999993, 3.2713499999999995, 3.4256999999999995, 3.5881499999999993, 3.758699999999999, 3.93735, 4.1241, 4.318950000000001, 4.5219000000000005, 4.732950000000001, 4.9521, 5.1793499999999995, 5.414700000000002, 5.658150000000001, 5.909700000000001, 6.16935, 6.437099999999999, 6.712949999999999, 6.996899999999998, 7.288950000000002, 7.5891, 7.897349999999999, 8.2137, 8.538149999999998, 8.8707, 9.211350000000001, 9.5601, 9.916950000000002, 10.281899999999998, 10.654949999999998, 11.0361, 11.425349999999996, 11.822699999999996, 12.228149999999998, 12.641700000000002, 13.063350000000002, 13.493100000000004, 13.930950000000001, 14.376900000000001 ], [ 9.782225, 9.426275, 9.078425, 8.738675, 8.407025, 8.083475, 7.768025, 7.460675, 7.1614249999999995, 6.8702749999999995, 6.587225, 6.312275, 6.045425, 5.786675000000001, 5.536025, 5.293475000000001, 5.059025, 4.832675, 4.614425000000001, 4.404275, 4.202225, 4.008275, 3.822425, 3.6446750000000003, 3.4750249999999996, 3.313475, 3.160025, 3.0146750000000004, 2.8774249999999997, 2.748275, 2.627225, 2.5142749999999996, 2.409425, 2.312675, 2.224025, 2.143475, 2.071025, 2.006675, 1.950425, 1.9022750000000002, 1.862225, 1.8302749999999999, 1.806425, 1.790675, 1.7830249999999999, 1.7834750000000001, 1.7920249999999998, 1.808675, 1.833425, 1.866275, 1.907225, 1.9562749999999998, 2.013425, 2.0786749999999996, 2.152024999999999, 2.2334749999999994, 2.3230249999999995, 2.4206749999999992, 2.526424999999999, 2.640274999999999, 2.762224999999999, 2.8922749999999997, 3.0304249999999993, 3.1766749999999995, 3.3310249999999995, 3.493474999999999, 3.6640249999999988, 3.842675, 4.029425, 4.2242750000000004, 4.427225, 4.638275, 4.857424999999999, 5.084674999999999, 5.320025000000001, 5.563475, 5.815025, 6.074674999999999, 6.342424999999999, 6.618274999999999, 6.902224999999998, 7.194275000000001, 7.494425, 7.802674999999999, 8.119024999999999, 8.443474999999996, 8.776024999999997, 9.116674999999999, 9.465424999999998, 9.822275, 10.187224999999996, 10.560274999999995, 10.941424999999997, 11.330674999999994, 11.728024999999993, 12.133474999999995, 12.547025, 12.968675, 13.398425000000001, 13.836274999999999, 14.282224999999999 ], [ 9.69, 9.33405, 8.9862, 8.64645, 8.3148, 7.99125, 7.675799999999999, 7.36845, 7.0691999999999995, 6.7780499999999995, 6.495, 6.22005, 5.9532, 5.69445, 5.4437999999999995, 5.20125, 4.9668, 4.74045, 4.5222, 4.31205, 4.11, 3.9160500000000003, 3.7302, 3.5524500000000003, 3.3827999999999996, 3.22125, 3.0678, 2.9224500000000004, 2.7851999999999997, 2.65605, 2.535, 2.4220499999999996, 2.3171999999999997, 2.22045, 2.1318, 2.0512499999999996, 1.9788, 1.9144499999999998, 1.8581999999999999, 1.81005, 1.7699999999999998, 1.7380499999999997, 1.7141999999999997, 1.6984499999999998, 1.6907999999999996, 1.69125, 1.6997999999999995, 1.7164499999999998, 1.7411999999999999, 1.7740499999999997, 1.8149999999999997, 1.8640499999999995, 1.9211999999999996, 1.9864499999999994, 2.059799999999999, 2.1412499999999994, 2.2307999999999995, 2.3284499999999992, 2.434199999999999, 2.548049999999999, 2.669999999999999, 2.8000499999999997, 2.9381999999999993, 3.0844499999999995, 3.2387999999999995, 3.401249999999999, 3.5717999999999988, 3.750449999999999, 3.937199999999999, 4.13205, 4.334999999999999, 4.546049999999999, 4.765199999999998, 4.992449999999998, 5.2278, 5.4712499999999995, 5.722799999999999, 5.982449999999998, 6.250199999999998, 6.526049999999998, 6.809999999999997, 7.10205, 7.402199999999999, 7.710449999999998, 8.026799999999998, 8.351249999999997, 8.683799999999998, 9.02445, 9.373199999999999, 9.73005, 10.094999999999997, 10.468049999999996, 10.849199999999998, 11.238449999999995, 11.635799999999994, 12.041249999999996, 12.4548, 12.87645, 13.306200000000002, 13.74405, 14.19 ], [ 9.600225, 9.244275, 8.896425, 8.556675, 8.225025, 7.901475, 7.586024999999999, 7.278675, 6.979425, 6.688274999999999, 6.405225, 6.130274999999999, 5.863424999999999, 5.604675, 5.354025, 5.111475, 4.877025, 4.650675, 4.432425, 4.222275, 4.020225, 3.826275, 3.6404249999999996, 3.462675, 3.2930249999999996, 3.131475, 2.9780249999999997, 2.832675, 2.6954249999999997, 2.566275, 2.445225, 2.3322749999999997, 2.2274249999999998, 2.130675, 2.0420249999999998, 1.9614749999999999, 1.889025, 1.8246749999999996, 1.768425, 1.720275, 1.6802249999999999, 1.6482749999999997, 1.6244249999999998, 1.6086749999999999, 1.6010249999999997, 1.6014749999999998, 1.6100249999999994, 1.6266749999999996, 1.6514249999999997, 1.6842749999999995, 1.7252249999999996, 1.7742749999999994, 1.8314249999999994, 1.8966749999999992, 1.9700249999999992, 2.0514749999999995, 2.1410249999999995, 2.2386749999999993, 2.344424999999999, 2.458274999999999, 2.580224999999999, 2.7102749999999998, 2.8484249999999993, 2.9946749999999995, 3.1490249999999995, 3.3114749999999993, 3.482024999999999, 3.660675, 3.847425, 4.042275, 4.245225, 4.456275, 4.675424999999999, 4.902674999999999, 5.138025000000001, 5.381475, 5.633025, 5.892674999999999, 6.160424999999998, 6.436274999999998, 6.7202249999999975, 7.012275000000001, 7.312424999999999, 7.6206749999999985, 7.937024999999998, 8.261474999999997, 8.594024999999998, 8.934675, 9.283425, 9.640275, 10.005224999999998, 10.378274999999997, 10.759424999999998, 11.148674999999995, 11.546024999999995, 11.951474999999997, 12.365025000000001, 12.786675, 13.216425000000003, 13.654275, 14.100225 ], [ 9.5129, 9.156949999999998, 8.8091, 8.469349999999999, 8.1377, 7.814150000000001, 7.4986999999999995, 7.19135, 6.8921, 6.600949999999999, 6.3179, 6.042949999999999, 5.7761, 5.51735, 5.2667, 5.024150000000001, 4.7897, 4.56335, 4.3451, 4.13495, 3.9329, 3.73895, 3.5530999999999997, 3.37535, 3.2056999999999998, 3.04415, 2.8907000000000003, 2.74535, 2.6081, 2.47895, 2.3579000000000003, 2.2449500000000002, 2.1401000000000003, 2.04335, 1.9546999999999999, 1.87415, 1.8017, 1.7373499999999997, 1.6811, 1.6329500000000001, 1.5929, 1.5609499999999998, 1.5371, 1.52135, 1.5136999999999998, 1.5141500000000003, 1.5227, 1.5393500000000002, 1.5641000000000003, 1.59695, 1.6379000000000001, 1.68695, 1.7441, 1.8093499999999998, 1.8826999999999998, 1.96415, 2.0537, 2.15135, 2.2570999999999994, 2.3709499999999997, 2.4928999999999997, 2.6229500000000003, 2.7611, 2.90735, 3.0617, 3.22415, 3.3946999999999994, 3.5733499999999987, 3.7600999999999987, 3.9549499999999993, 4.157899999999999, 4.368949999999999, 4.588099999999998, 4.815349999999998, 5.0507, 5.294149999999999, 5.545699999999999, 5.805349999999998, 6.0730999999999975, 6.348949999999998, 6.632899999999997, 6.92495, 7.2250999999999985, 7.533349999999998, 7.849699999999998, 8.174149999999997, 8.506699999999999, 8.847350000000002, 9.196100000000001, 9.552950000000003, 9.9179, 10.290949999999999, 10.6721, 11.061349999999997, 11.458699999999997, 11.864149999999999, 12.277700000000003, 12.699350000000003, 13.129100000000005, 13.566950000000002, 14.012900000000002 ], [ 9.428025, 9.072074999999998, 8.724225, 8.384474999999998, 8.052825, 7.729275, 7.413824999999999, 7.1064750000000005, 6.807225, 6.516075, 6.2330250000000005, 5.958075, 5.691224999999999, 5.432475, 5.181825, 4.939275, 4.704825, 4.4784749999999995, 4.260225, 4.050075, 3.8480250000000003, 3.654075, 3.468225, 3.2904750000000003, 3.1208249999999995, 2.959275, 2.805825, 2.6604750000000004, 2.523225, 2.394075, 2.2730250000000005, 2.160075, 2.055225, 1.9584750000000002, 1.8698249999999998, 1.789275, 1.716825, 1.6524749999999997, 1.596225, 1.548075, 1.508025, 1.4760749999999998, 1.4522249999999999, 1.436475, 1.4288249999999998, 1.4292750000000003, 1.437825, 1.4544750000000002, 1.4792250000000002, 1.512075, 1.553025, 1.602075, 1.659225, 1.7244749999999998, 1.7978249999999998, 1.879275, 1.968825, 2.0664749999999996, 2.172224999999999, 2.2860749999999994, 2.4080249999999994, 2.538075, 2.6762249999999996, 2.822475, 2.976825, 3.1392749999999996, 3.309824999999999, 3.4884749999999984, 3.6752249999999984, 3.870074999999999, 4.073024999999999, 4.284074999999999, 4.503224999999998, 4.7304749999999975, 4.965825, 5.209274999999999, 5.460824999999999, 5.720474999999998, 5.988224999999997, 6.264074999999997, 6.548024999999996, 6.840075, 7.140224999999998, 7.4484749999999975, 7.764824999999997, 8.089274999999997, 8.421824999999998, 8.762475000000002, 9.111225000000001, 9.468075000000002, 9.833025, 10.206074999999998, 10.587225, 10.976474999999997, 11.373824999999997, 11.779274999999998, 12.192825000000003, 12.614475000000002, 13.044225000000004, 13.482075000000002, 13.928025000000002 ], [ 9.3456, 8.98965, 8.6418, 8.30205, 7.9704, 7.646850000000001, 7.3314, 7.024050000000001, 6.7248, 6.43365, 6.150600000000001, 5.87565, 5.6088, 5.3500499999999995, 5.099399999999999, 4.85685, 4.6224, 4.39605, 4.1777999999999995, 3.96765, 3.7656, 3.57165, 3.3857999999999997, 3.20805, 3.0383999999999993, 2.8768499999999997, 2.7234, 2.57805, 2.4408, 2.31165, 2.1906, 2.0776499999999998, 1.9727999999999999, 1.87605, 1.7873999999999999, 1.7068499999999998, 1.6343999999999999, 1.5700499999999997, 1.5138, 1.4656500000000001, 1.4256, 1.3936499999999998, 1.3698, 1.35405, 1.3463999999999998, 1.3468499999999999, 1.3553999999999995, 1.3720499999999998, 1.3967999999999998, 1.4296499999999996, 1.4705999999999997, 1.5196499999999995, 1.5767999999999995, 1.6420499999999993, 1.7153999999999994, 1.7968499999999996, 1.8863999999999996, 1.9840499999999994, 2.089799999999999, 2.203649999999999, 2.3255999999999992, 2.45565, 2.5937999999999994, 2.7400499999999997, 2.8943999999999996, 3.0568499999999994, 3.227399999999999, 3.40605, 3.5928, 3.7876500000000006, 3.9906, 4.201650000000001, 4.4208, 4.64805, 4.883400000000002, 5.126850000000001, 5.378400000000001, 5.63805, 5.905799999999999, 6.181649999999999, 6.4655999999999985, 6.757650000000002, 7.0578, 7.3660499999999995, 7.6823999999999995, 8.006849999999998, 8.3394, 8.680050000000001, 9.0288, 9.385650000000002, 9.750599999999999, 10.123649999999998, 10.5048, 10.894049999999996, 11.291399999999996, 11.696849999999998, 12.110400000000002, 12.532050000000002, 12.961800000000004, 13.399650000000001, 13.845600000000001 ], [ 9.265625, 8.909675, 8.561825, 8.222075, 7.8904250000000005, 7.5668750000000005, 7.251424999999999, 6.944075, 6.644825, 6.353675, 6.070625, 5.795675, 5.528824999999999, 5.270075, 5.019425, 4.776875, 4.542425, 4.316075, 4.097825, 3.8876749999999998, 3.685625, 3.491675, 3.3058249999999996, 3.128075, 2.9584249999999996, 2.796875, 2.643425, 2.498075, 2.3608249999999997, 2.231675, 2.1106249999999998, 1.9976749999999999, 1.892825, 1.7960749999999999, 1.7074249999999997, 1.6268749999999998, 1.554425, 1.4900749999999998, 1.433825, 1.385675, 1.3456249999999998, 1.3136749999999997, 1.2898249999999998, 1.2740749999999998, 1.2664249999999997, 1.266875, 1.2754249999999996, 1.2920749999999999, 1.316825, 1.3496749999999997, 1.3906249999999998, 1.4396749999999996, 1.4968249999999996, 1.5620749999999994, 1.6354249999999995, 1.7168749999999997, 1.8064249999999997, 1.9040749999999995, 2.0098249999999993, 2.1236749999999995, 2.2456249999999995, 2.375675, 2.5138249999999998, 2.660075, 2.814425, 2.9768749999999997, 3.1474249999999993, 3.3260749999999994, 3.5128249999999994, 3.707675, 3.9106249999999996, 4.121675, 4.340824999999999, 4.5680749999999986, 4.803425000000001, 5.046875, 5.298425, 5.558074999999999, 5.825824999999998, 6.101674999999998, 6.385624999999997, 6.677675000000001, 6.977824999999999, 7.2860749999999985, 7.602424999999998, 7.926874999999997, 8.259424999999998, 8.600075, 8.948825, 9.305675, 9.670624999999998, 10.043674999999997, 10.424824999999998, 10.814074999999995, 11.211424999999995, 11.616874999999997, 12.030425000000001, 12.452075, 12.881825000000003, 13.319675, 13.765625 ], [ 9.188099999999999, 8.832149999999999, 8.484300000000001, 8.144549999999999, 7.8129, 7.48935, 7.1739, 6.86655, 6.5672999999999995, 6.2761499999999995, 5.9931, 5.71815, 5.4513, 5.19255, 4.9418999999999995, 4.69935, 4.4649, 4.23855, 4.0203, 3.8101499999999997, 3.6081000000000003, 3.41415, 3.2283, 3.05055, 2.8808999999999996, 2.7193499999999995, 2.5659, 2.4205500000000004, 2.2832999999999997, 2.1541499999999996, 2.0331, 1.9201499999999996, 1.8153, 1.71855, 1.6298999999999997, 1.5493499999999998, 1.4768999999999999, 1.4125499999999995, 1.3562999999999998, 1.30815, 1.2680999999999998, 1.2361499999999996, 1.2122999999999997, 1.1965499999999998, 1.1888999999999996, 1.1893499999999997, 1.1978999999999993, 1.2145499999999996, 1.2392999999999996, 1.2721499999999994, 1.3130999999999995, 1.3621499999999993, 1.4192999999999993, 1.4845499999999991, 1.5578999999999992, 1.6393499999999994, 1.7288999999999994, 1.8265499999999992, 1.9322999999999988, 2.046149999999999, 2.168099999999999, 2.2981499999999997, 2.4362999999999992, 2.5825499999999995, 2.7368999999999994, 2.899349999999999, 3.0698999999999987, 3.24855, 3.4353, 3.6301500000000004, 3.8331, 4.04415, 4.263299999999999, 4.490549999999999, 4.725900000000001, 4.96935, 5.2209, 5.480549999999999, 5.748299999999999, 6.024149999999999, 6.308099999999998, 6.600150000000001, 6.9003, 7.208549999999999, 7.524899999999999, 7.849349999999998, 8.181899999999997, 8.522549999999999, 8.871299999999998, 9.22815, 9.593099999999996, 9.966149999999995, 10.347299999999997, 10.736549999999994, 11.133899999999993, 11.539349999999995, 11.9529, 12.37455, 12.804300000000001, 13.242149999999999, 13.688099999999999 ], [ 9.113025, 8.757075, 8.409225, 8.069475, 7.737825000000001, 7.414275, 7.098825, 6.791475, 6.4922249999999995, 6.2010749999999994, 5.918025, 5.643075, 5.376225, 5.117475000000001, 4.866825, 4.624275, 4.389825, 4.163475, 3.945225, 3.735075, 3.5330250000000003, 3.3390750000000002, 3.153225, 2.9754750000000003, 2.8058249999999996, 2.644275, 2.490825, 2.3454750000000004, 2.2082249999999997, 2.079075, 1.9580250000000001, 1.8450749999999998, 1.7402250000000001, 1.643475, 1.5548249999999997, 1.474275, 1.401825, 1.3374749999999997, 1.281225, 1.2330750000000001, 1.193025, 1.1610749999999999, 1.137225, 1.121475, 1.1138249999999998, 1.1142750000000001, 1.1228249999999997, 1.139475, 1.164225, 1.197075, 1.238025, 1.2870749999999997, 1.3442249999999998, 1.4094749999999996, 1.4828249999999996, 1.5642749999999999, 1.6538249999999999, 1.7514749999999997, 1.8572249999999992, 1.9710749999999995, 2.0930249999999995, 2.223075, 2.3612249999999997, 2.507475, 2.661825, 2.8242749999999996, 2.994824999999999, 3.1734749999999985, 3.3602249999999985, 3.555074999999999, 3.7580249999999986, 3.969074999999999, 4.188224999999997, 4.415474999999997, 4.650824999999999, 4.894274999999999, 5.1458249999999985, 5.405474999999997, 5.673224999999997, 5.949074999999997, 6.233024999999996, 6.525074999999999, 6.825224999999998, 7.133474999999997, 7.449824999999997, 7.774274999999996, 8.106824999999999, 8.447475, 8.796225, 9.153075000000001, 9.518024999999998, 9.891074999999997, 10.272224999999999, 10.661474999999996, 11.058824999999995, 11.464274999999997, 11.877825000000001, 12.299475000000001, 12.729225000000003, 13.167075, 13.613025 ], [ 9.0404, 8.68445, 8.3366, 7.99685, 7.6652000000000005, 7.3416500000000005, 7.026199999999999, 6.71885, 6.4196, 6.12845, 5.8454, 5.57045, 5.303599999999999, 5.04485, 4.7942, 4.55165, 4.3172, 4.09085, 3.8726000000000003, 3.6624499999999998, 3.4604000000000004, 3.26645, 3.0806, 2.90285, 2.7331999999999996, 2.57165, 2.4182, 2.27285, 2.1355999999999997, 2.00645, 1.8854000000000002, 1.7724499999999999, 1.6676, 1.57085, 1.4821999999999997, 1.4016499999999998, 1.3292, 1.2648499999999998, 1.2086, 1.16045, 1.1203999999999998, 1.0884499999999997, 1.0645999999999998, 1.0488499999999998, 1.0411999999999997, 1.04165, 1.0501999999999996, 1.0668499999999999, 1.0916, 1.1244499999999997, 1.1653999999999998, 1.2144499999999996, 1.2715999999999996, 1.3368499999999994, 1.4101999999999995, 1.4916499999999997, 1.5811999999999997, 1.6788499999999995, 1.784599999999999, 1.8984499999999993, 2.0203999999999995, 2.15045, 2.2885999999999997, 2.43485, 2.5892, 2.7516499999999997, 2.9221999999999992, 3.1008500000000003, 3.2876000000000003, 3.482450000000001, 3.6854000000000005, 3.8964500000000006, 4.1156, 4.342849999999999, 4.578200000000002, 4.821650000000001, 5.073200000000001, 5.33285, 5.600599999999999, 5.876449999999999, 6.160399999999998, 6.452450000000002, 6.7526, 7.060849999999999, 7.377199999999999, 7.701649999999998, 8.034199999999998, 8.374849999999999, 8.723599999999998, 9.080449999999999, 9.445399999999996, 9.818449999999995, 10.199599999999997, 10.588849999999994, 10.986199999999993, 11.391649999999995, 11.8052, 12.226849999999999, 12.656600000000001, 13.094449999999998, 13.540399999999998 ], [ 8.970225, 8.614275, 8.266425, 7.9266749999999995, 7.595025000000001, 7.271475, 6.9560249999999995, 6.648675, 6.349425, 6.058275, 5.775225, 5.500275, 5.2334249999999995, 4.9746749999999995, 4.724024999999999, 4.481475, 4.247025, 4.020675, 3.802425, 3.592275, 3.390225, 3.196275, 3.0104249999999997, 2.832675, 2.6630249999999993, 2.5014749999999997, 2.348025, 2.202675, 2.065425, 1.936275, 1.815225, 1.7022749999999998, 1.5974249999999999, 1.500675, 1.4120249999999999, 1.3314749999999997, 1.2590249999999998, 1.1946749999999997, 1.1384249999999998, 1.0902749999999999, 1.0502249999999997, 1.0182749999999996, 0.9944249999999997, 0.978675, 0.9710249999999996, 0.9714749999999999, 0.9800249999999995, 0.9966749999999998, 1.0214249999999998, 1.0542749999999996, 1.0952249999999997, 1.1442749999999995, 1.2014249999999995, 1.2666749999999993, 1.3400249999999998, 1.421475, 1.511025, 1.6086749999999999, 1.7144249999999994, 1.8282749999999997, 1.9502249999999997, 2.0802750000000003, 2.218425, 2.364675, 2.519025, 2.681475, 2.8520249999999994, 3.0306749999999996, 3.2174249999999995, 3.412275, 3.6152249999999997, 3.826275, 4.045424999999999, 4.272674999999999, 4.508025000000001, 4.751475, 5.003025, 5.262674999999999, 5.530424999999998, 5.8062749999999985, 6.090224999999998, 6.382275000000001, 6.682424999999999, 6.990674999999999, 7.3070249999999985, 7.631474999999997, 7.964024999999999, 8.304675, 8.653424999999999, 9.010275, 9.375224999999997, 9.748274999999996, 10.129424999999998, 10.518674999999995, 10.916024999999994, 11.321474999999996, 11.735025, 12.156675, 12.586425000000002, 13.024275, 13.470225 ], [ 8.9025, 8.54655, 8.1987, 7.85895, 7.5273, 7.203749999999999, 6.888299999999999, 6.58095, 6.281700000000001, 5.99055, 5.7075, 5.43255, 5.165699999999999, 4.90695, 4.6563, 4.41375, 4.1793, 3.95295, 3.7347, 3.52455, 3.3225000000000002, 3.12855, 2.9427, 2.7649500000000002, 2.5952999999999995, 2.43375, 2.2803, 2.1349500000000003, 1.9977, 1.8685500000000002, 1.7475000000000003, 1.63455, 1.5297, 1.4329500000000002, 1.3443, 1.26375, 1.1913, 1.12695, 1.0707, 1.02255, 0.9824999999999999, 0.9505499999999998, 0.9266999999999999, 0.9109499999999997, 0.9032999999999998, 0.90375, 0.9122999999999997, 0.9289499999999999, 0.9537, 0.9865499999999998, 1.0274999999999999, 1.0765499999999997, 1.1336999999999997, 1.1989499999999995, 1.2723, 1.3537500000000002, 1.4433000000000002, 1.54095, 1.6466999999999996, 1.7605499999999998, 1.8824999999999998, 2.0125500000000005, 2.1507, 2.2969500000000003, 2.4513000000000003, 2.61375, 2.7842999999999996, 2.962949999999999, 3.149699999999999, 3.3445499999999995, 3.547499999999999, 3.758549999999999, 3.9776999999999982, 4.204949999999998, 4.440300000000001, 4.68375, 4.9353, 5.194949999999999, 5.462699999999998, 5.738549999999998, 6.022499999999997, 6.3145500000000006, 6.614699999999999, 6.922949999999998, 7.239299999999998, 7.563749999999997, 7.8963, 8.236950000000002, 8.585700000000001, 8.942550000000002, 9.3075, 9.680549999999998, 10.0617, 10.450949999999997, 10.848299999999997, 11.253749999999998, 11.667300000000003, 12.088950000000002, 12.518700000000004, 12.956550000000002, 13.402500000000002 ], [ 8.837225, 8.481275, 8.133425, 7.793675, 7.462025000000001, 7.138475, 6.8230249999999995, 6.515675, 6.216424999999999, 5.925274999999999, 5.642225, 5.367274999999999, 5.1004249999999995, 4.841675, 4.591025, 4.3484750000000005, 4.114025, 3.8876749999999998, 3.669425, 3.459275, 3.257225, 3.063275, 2.8774249999999997, 2.699675, 2.5300249999999993, 2.3684749999999997, 2.215025, 2.069675, 1.9324249999999996, 1.8032749999999997, 1.6822249999999999, 1.5692749999999998, 1.4644249999999999, 1.367675, 1.2790249999999996, 1.1984749999999997, 1.1260249999999998, 1.0616749999999997, 1.0054249999999998, 0.9572750000000001, 0.9172249999999997, 0.8852749999999996, 0.8614249999999997, 0.845675, 0.8380249999999996, 0.8384749999999999, 0.8470249999999995, 0.8636749999999997, 0.8884249999999998, 0.9212749999999996, 0.9622249999999997, 1.0112749999999995, 1.0684249999999995, 1.1336749999999993, 1.2070249999999993, 1.2884749999999996, 1.3780249999999996, 1.4756749999999994, 1.581424999999999, 1.6952749999999992, 1.8172249999999992, 1.9472749999999999, 2.0854249999999994, 2.2316749999999996, 2.3860249999999996, 2.5484749999999994, 2.719024999999999, 2.897674999999999, 3.084424999999999, 3.2792749999999997, 3.4822249999999992, 3.6932749999999994, 3.9124249999999985, 4.139674999999999, 4.375025000000001, 4.618475, 4.870025, 5.129674999999999, 5.397424999999998, 5.6732749999999985, 5.9572249999999976, 6.249275000000001, 6.549424999999999, 6.857674999999999, 7.1740249999999985, 7.498474999999997, 7.831024999999997, 8.171674999999999, 8.520424999999998, 8.877275, 9.242224999999996, 9.615274999999995, 9.996424999999997, 10.385674999999994, 10.783024999999993, 11.188474999999995, 11.602025, 12.023674999999999, 12.453425000000001, 12.891274999999998, 13.337224999999998 ], [ 8.7744, 8.41845, 8.070599999999999, 7.73085, 7.3992, 7.0756499999999996, 6.760199999999999, 6.45285, 6.1536, 5.86245, 5.5794, 5.30445, 5.037599999999999, 4.77885, 4.5282, 4.28565, 4.0512, 3.8248499999999996, 3.6066000000000003, 3.3964499999999997, 3.1944000000000004, 3.00045, 2.8146, 2.63685, 2.4671999999999996, 2.30565, 2.1522, 2.00685, 1.8695999999999997, 1.74045, 1.6194000000000002, 1.5064499999999998, 1.4016, 1.30485, 1.2161999999999997, 1.13565, 1.0632000000000001, 0.99885, 0.9425999999999997, 0.89445, 0.8544, 0.8224499999999999, 0.7985999999999995, 0.7828499999999998, 0.7751999999999999, 0.7756500000000002, 0.7841999999999998, 0.8008500000000001, 0.8256000000000001, 0.8584499999999999, 0.8994, 0.9484499999999998, 1.0055999999999998, 1.0708499999999996, 1.1441999999999997, 1.22565, 1.3152, 1.4128499999999997, 1.5185999999999993, 1.6324499999999995, 1.7543999999999995, 1.8844500000000002, 2.0225999999999997, 2.16885, 2.3232, 2.4856499999999997, 2.6561999999999992, 2.8348499999999994, 3.0215999999999994, 3.21645, 3.4193999999999996, 3.6304499999999997, 3.849599999999999, 4.0768499999999985, 4.312200000000001, 4.55565, 4.8072, 5.066849999999999, 5.334599999999998, 5.610449999999998, 5.894399999999997, 6.186450000000001, 6.486599999999999, 6.7948499999999985, 7.111199999999998, 7.435649999999997, 7.768199999999997, 8.108849999999999, 8.457599999999998, 8.814449999999999, 9.179399999999996, 9.552449999999995, 9.933599999999997, 10.322849999999994, 10.720199999999993, 11.125649999999995, 11.5392, 11.960849999999999, 12.390600000000001, 12.828449999999998, 13.274399999999998 ], [ 8.714025, 8.358075, 8.010225, 7.670475, 7.338825, 7.015275000000001, 6.699824999999999, 6.392475, 6.093225, 5.802075, 5.519025, 5.2440750000000005, 4.977225, 4.718475, 4.4678249999999995, 4.225275, 3.990825, 3.764475, 3.546225, 3.336075, 3.1340250000000003, 2.940075, 2.754225, 2.5764750000000003, 2.4068249999999995, 2.245275, 2.091825, 1.9464750000000004, 1.809225, 1.6800750000000002, 1.5590250000000003, 1.446075, 1.3412250000000001, 1.2444750000000002, 1.1558249999999999, 1.075275, 1.002825, 0.938475, 0.882225, 0.8340750000000003, 0.794025, 0.7620749999999998, 0.7382249999999999, 0.7224750000000002, 0.7148249999999998, 0.7152750000000001, 0.7238249999999997, 0.740475, 0.765225, 0.7980749999999999, 0.8390249999999999, 0.8880749999999997, 0.9452249999999998, 1.0104749999999996, 1.0838249999999996, 1.1652749999999998, 1.2548249999999999, 1.3524749999999996, 1.4582249999999992, 1.5720749999999994, 1.6940249999999994, 1.8240750000000001, 1.9622249999999997, 2.108475, 2.262825, 2.4252749999999996, 2.595824999999999, 2.7744749999999994, 2.9612249999999993, 3.156075, 3.3590249999999995, 3.5700749999999997, 3.7892249999999987, 4.016474999999998, 4.251825, 4.4952749999999995, 4.746824999999999, 5.006474999999998, 5.274224999999998, 5.550074999999998, 5.834024999999997, 6.126075, 6.426224999999999, 6.734474999999998, 7.050824999999998, 7.375274999999997, 7.707825, 8.048475000000002, 8.397225, 8.754075000000002, 9.119024999999999, 9.492074999999998, 9.873225, 10.262474999999997, 10.659824999999996, 11.065274999999998, 11.478825000000002, 11.900475000000002, 12.330225000000004, 12.768075000000001, 13.214025000000001 ], [ 8.6561, 8.300149999999999, 7.952299999999999, 7.612549999999999, 7.280900000000001, 6.95735, 6.6419, 6.33455, 6.035299999999999, 5.744149999999999, 5.4611, 5.18615, 4.9193, 4.66055, 4.4098999999999995, 4.16735, 3.9329, 3.7065499999999996, 3.4882999999999997, 3.2781499999999997, 3.0761000000000003, 2.8821499999999998, 2.6963, 2.5185500000000003, 2.3488999999999995, 2.1873499999999995, 2.0339, 1.8885500000000002, 1.7512999999999996, 1.62215, 1.5011, 1.3881499999999998, 1.2832999999999999, 1.18655, 1.0978999999999997, 1.0173499999999998, 0.9448999999999996, 0.8805499999999995, 0.8243, 0.7761499999999999, 0.7360999999999995, 0.7041499999999994, 0.6802999999999999, 0.6645499999999998, 0.6568999999999994, 0.6573500000000001, 0.6658999999999997, 0.68255, 0.7073, 0.7401499999999999, 0.7810999999999999, 0.8301499999999997, 0.8872999999999998, 0.9525499999999996, 1.0258999999999996, 1.1073499999999998, 1.1968999999999999, 1.2945499999999996, 1.4002999999999992, 1.5141499999999994, 1.6360999999999994, 1.76615, 1.9042999999999997, 2.05055, 2.2049, 2.3673499999999996, 2.537899999999999, 2.7165499999999985, 2.9032999999999984, 3.098149999999999, 3.3010999999999986, 3.5121499999999988, 3.731299999999998, 3.9585499999999976, 4.193899999999999, 4.437349999999999, 4.6888999999999985, 4.948549999999997, 5.216299999999997, 5.492149999999997, 5.776099999999996, 6.068149999999999, 6.368299999999998, 6.676549999999997, 6.992899999999997, 7.317349999999996, 7.649899999999999, 7.990550000000001, 8.3393, 8.696150000000001, 9.061099999999998, 9.434149999999997, 9.815299999999999, 10.204549999999996, 10.601899999999995, 11.007349999999997, 11.420900000000001, 11.842550000000001, 12.272300000000003, 12.71015, 13.1561 ], [ 8.600625, 8.244675, 7.896825000000001, 7.557075, 7.225425, 6.901875, 6.586424999999999, 6.279075000000001, 5.979825, 5.688675, 5.405625000000001, 5.130675, 4.863824999999999, 4.605075, 4.354425, 4.111875, 3.8774249999999997, 3.6510749999999996, 3.4328250000000002, 3.2226749999999997, 3.0206250000000003, 2.826675, 2.640825, 2.463075, 2.2934249999999996, 2.131875, 1.978425, 1.8330750000000005, 1.6958249999999997, 1.566675, 1.4456250000000002, 1.3326749999999998, 1.227825, 1.131075, 1.0424249999999997, 0.961875, 0.8894250000000001, 0.825075, 0.7688250000000001, 0.720675, 0.680625, 0.6486749999999999, 0.624825, 0.6090749999999998, 0.6014249999999999, 0.6018750000000002, 0.6104249999999998, 0.627075, 0.6518250000000001, 0.6846749999999999, 0.725625, 0.7746749999999998, 0.8318249999999998, 0.8970749999999996, 0.9704249999999996, 1.051875, 1.141425, 1.2390749999999997, 1.3448249999999993, 1.4586749999999995, 1.5806249999999995, 1.7106750000000002, 1.8488249999999997, 1.995075, 2.149425, 2.3118749999999997, 2.482424999999999, 2.6610749999999994, 2.8478249999999994, 3.042675, 3.2456249999999995, 3.4566749999999997, 3.675824999999999, 3.9030749999999985, 4.138425000000001, 4.381875, 4.633425, 4.893074999999999, 5.160824999999998, 5.436674999999998, 5.720624999999997, 6.012675000000001, 6.312824999999999, 6.6210749999999985, 6.937424999999998, 7.261874999999997, 7.594424999999998, 7.935075, 8.283825, 8.640675000000002, 9.005624999999998, 9.378674999999998, 9.759825, 10.149074999999996, 10.546424999999996, 10.951874999999998, 11.365425000000002, 11.787075000000002, 12.216825000000004, 12.654675000000001, 13.100625 ], [ 8.5476, 8.19165, 7.8438, 7.504049999999999, 7.1724000000000006, 6.84885, 6.533399999999999, 6.22605, 5.9268, 5.63565, 5.3526, 5.07765, 4.8107999999999995, 4.5520499999999995, 4.301399999999999, 4.05885, 3.8244, 3.5980499999999997, 3.3798, 3.16965, 2.9676, 2.77365, 2.5877999999999997, 2.41005, 2.2403999999999993, 2.0788499999999996, 1.9253999999999998, 1.7800500000000001, 1.6427999999999996, 1.5136499999999997, 1.3926, 1.2796499999999997, 1.1747999999999998, 1.07805, 0.9893999999999998, 0.9088499999999997, 0.8363999999999998, 0.7720499999999997, 0.7157999999999998, 0.6676500000000001, 0.6275999999999997, 0.5956499999999996, 0.5717999999999996, 0.5560499999999999, 0.5483999999999996, 0.5488499999999998, 0.5573999999999995, 0.5740499999999997, 0.5987999999999998, 0.6316499999999996, 0.6725999999999996, 0.7216499999999995, 0.7787999999999995, 0.8440499999999993, 0.9173999999999998, 0.99885, 1.0884, 1.1860499999999998, 1.2917999999999994, 1.4056499999999996, 1.5275999999999996, 1.6576500000000003, 1.7957999999999998, 1.94205, 2.0964, 2.25885, 2.4293999999999993, 2.6080499999999995, 2.7947999999999995, 2.98965, 3.1925999999999997, 3.40365, 3.622799999999999, 3.8500499999999986, 4.085400000000001, 4.32885, 4.5804, 4.840049999999999, 5.107799999999998, 5.3836499999999985, 5.6675999999999975, 5.959650000000001, 6.259799999999999, 6.568049999999999, 6.8843999999999985, 7.2088499999999955, 7.541399999999997, 7.882049999999999, 8.230799999999999, 8.58765, 8.952599999999997, 9.325649999999996, 9.706799999999998, 10.096049999999995, 10.493399999999994, 10.898849999999996, 11.3124, 11.73405, 12.163800000000002, 12.60165, 13.0476 ], [ 8.497025, 8.141074999999999, 7.793225, 7.453474999999999, 7.121825000000001, 6.798275, 6.482825, 6.1754750000000005, 5.876225, 5.585075, 5.302025, 5.027075, 4.760225, 4.501475, 4.250825, 4.008275, 3.773825, 3.5474749999999995, 3.329225, 3.1190749999999996, 2.917025, 2.723075, 2.537225, 2.359475, 2.1898249999999995, 2.028275, 1.874825, 1.7294750000000003, 1.5922249999999998, 1.463075, 1.342025, 1.2290749999999997, 1.124225, 1.027475, 0.9388249999999996, 0.8582749999999999, 0.785825, 0.7214749999999999, 0.665225, 0.6170749999999998, 0.5770249999999999, 0.5450749999999998, 0.5212249999999998, 0.5054749999999997, 0.49782499999999974, 0.498275, 0.5068249999999996, 0.5234749999999999, 0.548225, 0.5810749999999998, 0.6220249999999998, 0.6710749999999996, 0.7282249999999997, 0.7934749999999995, 0.8668249999999991, 0.9482749999999993, 1.0378249999999993, 1.1354749999999991, 1.2412249999999987, 1.355074999999999, 1.477024999999999, 1.6070749999999996, 1.7452249999999991, 1.8914749999999994, 2.0458249999999993, 2.208274999999999, 2.3788249999999986, 2.5574749999999997, 2.7442249999999997, 2.9390750000000003, 3.142025, 3.353075, 3.572224999999999, 3.799474999999999, 4.034825000000001, 4.278275000000001, 4.529825000000001, 4.7894749999999995, 5.057224999999999, 5.333074999999999, 5.617024999999998, 5.909075000000001, 6.209225, 6.517474999999999, 6.833824999999999, 7.158274999999998, 7.490824999999999, 7.831475000000001, 8.180225, 8.537075000000002, 8.902024999999998, 9.275074999999998, 9.656225, 10.045474999999996, 10.442824999999996, 10.848274999999997, 11.261825000000002, 11.683475000000001, 12.113225000000003, 12.551075, 12.997025 ], [ 8.4489, 8.09295, 7.7451, 7.40535, 7.0737000000000005, 6.75015, 6.434699999999999, 6.12735, 5.8281, 5.53695, 5.2539, 4.97895, 4.7120999999999995, 4.45335, 4.2027, 3.9601500000000005, 3.7257, 3.4993499999999997, 3.2811000000000003, 3.07095, 2.8689000000000004, 2.67495, 2.4891, 2.31135, 2.1416999999999997, 1.9801499999999999, 1.8267000000000002, 1.6813500000000003, 1.5440999999999998, 1.4149500000000002, 1.2939000000000003, 1.18095, 1.0761, 0.9793500000000002, 0.8906999999999998, 0.8101500000000001, 0.7377000000000002, 0.6733500000000001, 0.6170999999999998, 0.5689500000000001, 0.5289000000000001, 0.49695, 0.47309999999999963, 0.4573499999999999, 0.4497, 0.45015000000000027, 0.4586999999999999, 0.47535000000000016, 0.5001000000000002, 0.53295, 0.5739000000000001, 0.6229499999999999, 0.6800999999999999, 0.7453499999999997, 0.8186999999999998, 0.90015, 0.9897, 1.0873499999999998, 1.1930999999999994, 1.3069499999999996, 1.4288999999999996, 1.5589500000000003, 1.6970999999999998, 1.84335, 1.9977, 2.16015, 2.3306999999999993, 2.5093499999999995, 2.6960999999999995, 2.89095, 3.0938999999999997, 3.30495, 3.524099999999999, 3.7513499999999986, 3.986700000000001, 4.23015, 4.4817, 4.741349999999999, 5.009099999999998, 5.2849499999999985, 5.5688999999999975, 5.860950000000001, 6.161099999999999, 6.469349999999999, 6.7856999999999985, 7.1101499999999955, 7.442699999999997, 7.783349999999999, 8.132099999999998, 8.488949999999999, 8.853899999999996, 9.226949999999995, 9.608099999999997, 9.997349999999994, 10.394699999999993, 10.800149999999995, 11.2137, 11.635349999999999, 12.065100000000001, 12.502949999999998, 12.948899999999998 ], [ 8.403224999999999, 8.047274999999999, 7.699425, 7.359674999999999, 7.028025, 6.704475, 6.389024999999999, 6.081675000000001, 5.782425, 5.491275, 5.2082250000000005, 4.933275, 4.666424999999999, 4.407674999999999, 4.157025, 3.914475, 3.6800249999999997, 3.4536749999999996, 3.235425, 3.0252749999999997, 2.823225, 2.629275, 2.4434249999999995, 2.265675, 2.0960249999999996, 1.9344749999999997, 1.781025, 1.6356750000000002, 1.4984249999999997, 1.369275, 1.2482250000000001, 1.1352749999999996, 1.030425, 0.933675, 0.8450249999999997, 0.764475, 0.6920250000000001, 0.6276749999999995, 0.5714249999999996, 0.5232749999999999, 0.483225, 0.45127499999999987, 0.4274249999999995, 0.4116749999999998, 0.40402499999999986, 0.4044749999999997, 0.4130249999999993, 0.4296749999999996, 0.45442499999999963, 0.48727499999999946, 0.528225, 0.5772749999999993, 0.6344249999999994, 0.6996749999999992, 0.7730249999999996, 0.8544749999999999, 0.9440249999999999, 1.0416749999999997, 1.1474249999999993, 1.2612749999999995, 1.3832249999999995, 1.5132750000000001, 1.6514249999999997, 1.797675, 1.952025, 2.1144749999999997, 2.285024999999999, 2.4636749999999985, 2.6504249999999985, 2.845274999999999, 3.0482249999999986, 3.259274999999999, 3.478424999999998, 3.7056749999999976, 3.941025, 4.184474999999999, 4.436024999999999, 4.695674999999998, 4.963424999999997, 5.2392749999999975, 5.5232249999999965, 5.815275, 6.115424999999998, 6.423674999999998, 6.7400249999999975, 7.064474999999996, 7.3970249999999975, 7.737674999999999, 8.086424999999998, 8.443275, 8.808224999999997, 9.181274999999996, 9.562424999999998, 9.951674999999994, 10.349024999999994, 10.754474999999996, 11.168025, 11.589675, 12.019425000000002, 12.457275, 12.903224999999999 ], [ 8.36, 8.00405, 7.6562, 7.31645, 6.9848, 6.661250000000001, 6.345799999999999, 6.03845, 5.7392, 5.44805, 5.165, 4.8900500000000005, 4.6232, 4.36445, 4.1137999999999995, 3.8712500000000003, 3.6368, 3.41045, 3.1922, 2.98205, 2.7800000000000002, 2.58605, 2.4002, 2.2224500000000003, 2.0527999999999995, 1.8912499999999999, 1.7378, 1.5924500000000004, 1.4551999999999998, 1.32605, 1.205, 1.09205, 0.9872000000000001, 0.89045, 0.8018000000000001, 0.72125, 0.6488, 0.5844499999999995, 0.5282, 0.4800500000000003, 0.43999999999999995, 0.4080499999999998, 0.3841999999999999, 0.36845000000000017, 0.3607999999999998, 0.36125000000000007, 0.3697999999999997, 0.38644999999999996, 0.4112, 0.44404999999999983, 0.4849999999999999, 0.5340499999999997, 0.5911999999999997, 0.6564499999999995, 0.7297999999999996, 0.8112499999999998, 0.9007999999999998, 0.9984499999999996, 1.1041999999999992, 1.2180499999999994, 1.3399999999999994, 1.47005, 1.6081999999999996, 1.7544499999999998, 1.9087999999999998, 2.0712499999999996, 2.241799999999999, 2.4204499999999993, 2.6071999999999993, 2.80205, 3.0049999999999994, 3.2160499999999996, 3.4351999999999987, 3.6624499999999984, 3.8978000000000006, 4.141249999999999, 4.392799999999999, 4.652449999999998, 4.920199999999998, 5.196049999999998, 5.479999999999997, 5.77205, 6.072199999999999, 6.380449999999998, 6.696799999999998, 7.021249999999997, 7.353799999999998, 7.69445, 8.043199999999999, 8.40005, 8.764999999999997, 9.138049999999996, 9.519199999999998, 9.908449999999995, 10.305799999999994, 10.711249999999996, 11.1248, 11.54645, 11.976200000000002, 12.41405, 12.86 ], [ 8.319225, 7.963275, 7.615425, 7.275675, 6.944025000000001, 6.620475, 6.305025, 5.997675, 5.698425, 5.407275, 5.124225, 4.8492750000000004, 4.582425, 4.323675, 4.0730249999999995, 3.8304750000000003, 3.596025, 3.369675, 3.151425, 2.941275, 2.7392250000000002, 2.545275, 2.359425, 2.1816750000000003, 2.0120249999999995, 1.8504749999999999, 1.6970250000000002, 1.5516750000000004, 1.414425, 1.2852750000000002, 1.1642250000000003, 1.051275, 0.9464250000000001, 0.8496750000000004, 0.7610250000000001, 0.6804749999999999, 0.608025, 0.5436749999999999, 0.487425, 0.4392750000000003, 0.39922499999999994, 0.3672749999999998, 0.34342499999999987, 0.32767500000000016, 0.3200249999999998, 0.32047500000000007, 0.3290249999999997, 0.34567499999999995, 0.370425, 0.40327499999999983, 0.44422499999999987, 0.4932749999999997, 0.5504249999999997, 0.6156749999999995, 0.689025, 0.7704750000000002, 0.8600250000000003, 0.957675, 1.0634249999999996, 1.1772749999999998, 1.2992249999999999, 1.4292750000000005, 1.567425, 1.7136750000000003, 1.8680250000000003, 2.030475, 2.2010249999999996, 2.3796749999999998, 2.5664249999999997, 2.7612750000000004, 2.964225, 3.175275, 3.394424999999999, 3.621674999999999, 3.857025000000001, 4.100475, 4.352025, 4.611674999999999, 4.879424999999999, 5.155274999999999, 5.439224999999998, 5.731275000000001, 6.031425, 6.339674999999999, 6.656024999999999, 6.9804749999999975, 7.313024999999999, 7.653675000000001, 8.002424999999999, 8.359275, 8.724224999999997, 9.097274999999996, 9.478424999999998, 9.867674999999995, 10.265024999999994, 10.670474999999996, 11.084025, 11.505675, 11.935425000000002, 12.373275, 12.819225 ], [ 8.280899999999999, 7.92495, 7.5771, 7.23735, 6.9057, 6.5821499999999995, 6.266699999999999, 5.959350000000001, 5.6601, 5.36895, 5.0859000000000005, 4.81095, 4.544099999999999, 4.28535, 4.0347, 3.7921500000000004, 3.5576999999999996, 3.3313499999999996, 3.1131, 2.9029499999999997, 2.7009000000000003, 2.50695, 2.3211, 2.14335, 1.9736999999999996, 1.81215, 1.6587, 1.5133500000000002, 1.3760999999999997, 1.24695, 1.1259000000000001, 1.0129499999999998, 0.9081000000000001, 0.81135, 0.7226999999999997, 0.64215, 0.5697000000000001, 0.50535, 0.4490999999999996, 0.4009499999999999, 0.3609, 0.32894999999999985, 0.3050999999999995, 0.2893499999999998, 0.28169999999999984, 0.2821500000000001, 0.29069999999999974, 0.30735, 0.33210000000000006, 0.3649499999999999, 0.4058999999999999, 0.45494999999999974, 0.5120999999999998, 0.5773499999999991, 0.6506999999999996, 0.7321499999999999, 0.8216999999999999, 0.9193499999999997, 1.0250999999999992, 1.1389499999999995, 1.2608999999999995, 1.3909500000000001, 1.5290999999999997, 1.67535, 1.8296999999999999, 1.9921499999999996, 2.162699999999999, 2.3413499999999994, 2.5280999999999993, 2.72295, 2.9258999999999995, 3.1369499999999997, 3.3560999999999988, 3.5833499999999985, 3.8187000000000006, 4.06215, 4.3137, 4.573349999999999, 4.841099999999998, 5.116949999999998, 5.400899999999997, 5.692950000000001, 5.993099999999999, 6.3013499999999985, 6.617699999999998, 6.942149999999997, 7.274699999999998, 7.61535, 7.964099999999999, 8.32095, 8.685899999999997, 9.058949999999996, 9.440099999999997, 9.829349999999994, 10.226699999999994, 10.632149999999996, 11.0457, 11.46735, 11.897100000000002, 12.33495, 12.780899999999999 ], [ 8.245025, 7.889075, 7.541225000000001, 7.201475, 6.8698250000000005, 6.546275, 6.230824999999999, 5.923475, 5.624225, 5.333074999999999, 5.050025, 4.775074999999999, 4.5082249999999995, 4.249475, 3.9988249999999996, 3.7562750000000005, 3.5218249999999998, 3.2954749999999997, 3.0772250000000003, 2.867075, 2.665025, 2.471075, 2.2852249999999996, 2.107475, 1.9378249999999997, 1.7762749999999998, 1.6228250000000002, 1.4774750000000003, 1.3402249999999998, 1.2110750000000001, 1.0900250000000002, 0.9770750000000001, 0.8722250000000003, 0.7754750000000001, 0.6868249999999998, 0.6062750000000001, 0.5338250000000002, 0.4694750000000001, 0.41322499999999973, 0.36507500000000004, 0.3250250000000001, 0.293075, 0.2692249999999996, 0.2534749999999999, 0.24582499999999996, 0.24627500000000024, 0.25482499999999986, 0.27147500000000013, 0.2962250000000002, 0.329075, 0.37002500000000005, 0.41907499999999986, 0.4762249999999999, 0.5414749999999997, 0.6148249999999993, 0.6962749999999995, 0.7858249999999996, 0.8834749999999993, 0.9892249999999989, 1.1030749999999991, 1.2250249999999991, 1.3550749999999998, 1.4932249999999994, 1.6394749999999996, 1.7938249999999996, 1.9562749999999993, 2.1268249999999997, 2.305474999999999, 2.492224999999999, 2.6870749999999997, 2.890024999999999, 3.1010749999999994, 3.3202249999999984, 3.547474999999998, 3.7828250000000003, 4.026275, 4.277825, 4.537474999999999, 4.805224999999998, 5.0810749999999985, 5.3650249999999975, 5.657075000000001, 5.957224999999999, 6.265474999999999, 6.5818249999999985, 6.906274999999997, 7.2388249999999985, 7.579475, 7.928224999999999, 8.285075, 8.650024999999998, 9.023074999999997, 9.404224999999999, 9.793474999999995, 10.190824999999995, 10.596274999999997, 11.009825000000001, 11.431475, 11.861225000000003, 12.299075, 12.745025 ], [ 8.2116, 7.85565, 7.5078000000000005, 7.16805, 6.8364, 6.51285, 6.1974, 5.8900500000000005, 5.5908, 5.29965, 5.0166, 4.74165, 4.4748, 4.21605, 3.9654, 3.72285, 3.4884, 3.26205, 3.0438, 2.83365, 2.6316000000000006, 2.43765, 2.2518000000000002, 2.07405, 1.9043999999999996, 1.74285, 1.5894000000000001, 1.4440500000000003, 1.3068, 1.17765, 1.0566000000000004, 0.9436499999999999, 0.8388, 0.7420500000000003, 0.6534, 0.5728499999999999, 0.5004, 0.4360499999999998, 0.37980000000000036, 0.3316500000000002, 0.29159999999999986, 0.2596499999999997, 0.23580000000000023, 0.22005000000000008, 0.2123999999999997, 0.21284999999999998, 0.2213999999999996, 0.23804999999999987, 0.2627999999999999, 0.29564999999999975, 0.3365999999999998, 0.3856499999999996, 0.44279999999999964, 0.5080499999999999, 0.5813999999999995, 0.6628499999999997, 0.7523999999999997, 0.8500499999999995, 0.9557999999999991, 1.0696499999999993, 1.1915999999999993, 1.32165, 1.4597999999999995, 1.6060499999999998, 1.7603999999999997, 1.9228499999999995, 2.093399999999999, 2.27205, 2.4588, 2.6536500000000007, 2.8566000000000003, 3.0676500000000004, 3.2867999999999995, 3.5140499999999992, 3.7494000000000014, 3.9928500000000007, 4.244400000000001, 4.504049999999999, 4.771799999999999, 5.047649999999999, 5.331599999999998, 5.623650000000001, 5.9238, 6.232049999999999, 6.548399999999999, 6.872849999999996, 7.205399999999997, 7.546049999999999, 7.894799999999998, 8.25165, 8.616599999999996, 8.989649999999996, 9.370799999999997, 9.760049999999994, 10.157399999999994, 10.562849999999996, 10.9764, 11.39805, 11.827800000000002, 12.265649999999999, 12.711599999999999 ], [ 8.180625, 7.824674999999999, 7.476825, 7.137074999999999, 6.8054250000000005, 6.4818750000000005, 6.166424999999999, 5.859075000000001, 5.559825, 5.268675, 4.985625000000001, 4.710675, 4.4438249999999995, 4.185075, 3.934425, 3.6918750000000005, 3.4574249999999997, 3.2310749999999997, 3.0128250000000003, 2.802675, 2.600625, 2.406675, 2.2208249999999996, 2.043075, 1.8734249999999997, 1.7118749999999998, 1.5584250000000002, 1.4130750000000003, 1.2758249999999998, 1.146675, 1.0256250000000002, 0.9126750000000001, 0.8078250000000002, 0.7110750000000001, 0.6224249999999998, 0.5418750000000001, 0.4694250000000002, 0.4050750000000001, 0.3488249999999997, 0.300675, 0.2606250000000001, 0.22867499999999996, 0.2048249999999996, 0.18907499999999988, 0.18142499999999995, 0.18187500000000023, 0.19042499999999984, 0.20707500000000012, 0.23182500000000017, 0.264675, 0.30562500000000004, 0.35467499999999985, 0.4118249999999999, 0.47707499999999925, 0.5504249999999997, 0.631875, 0.721425, 0.8190749999999998, 0.9248249999999993, 1.0386749999999996, 1.1606249999999996, 1.2906750000000002, 1.4288249999999998, 1.575075, 1.729425, 1.8918749999999998, 2.0624249999999993, 2.2410749999999986, 2.4278249999999986, 2.622674999999999, 2.8256249999999987, 3.036674999999999, 3.255824999999998, 3.4830749999999977, 3.718425, 3.961874999999999, 4.213424999999999, 4.473074999999998, 4.740824999999997, 5.0166749999999976, 5.300624999999997, 5.592675, 5.892824999999998, 6.201074999999998, 6.517424999999998, 6.841874999999998, 7.174424999999999, 7.515075000000001, 7.863825, 8.220675000000002, 8.585624999999999, 8.958674999999998, 9.339825, 9.729074999999996, 10.126424999999996, 10.531874999999998, 10.945425000000002, 11.367075000000002, 11.796825000000004, 12.234675000000001, 12.680625000000001 ], [ 8.152099999999999, 7.796149999999999, 7.4483, 7.108549999999999, 6.7768999999999995, 6.45335, 6.1379, 5.830550000000001, 5.5313, 5.24015, 4.9571000000000005, 4.68215, 4.4153, 4.15655, 3.9059, 3.6633500000000003, 3.4289, 3.2025499999999996, 2.9843, 2.77415, 2.5721000000000003, 2.37815, 2.1923, 2.0145500000000003, 1.8448999999999995, 1.68335, 1.5299, 1.3845500000000004, 1.2472999999999999, 1.11815, 0.9971000000000001, 0.88415, 0.7793000000000001, 0.68255, 0.5938999999999997, 0.51335, 0.44090000000000007, 0.37654999999999994, 0.32030000000000003, 0.2721499999999999, 0.23209999999999997, 0.20014999999999983, 0.1762999999999999, 0.1605500000000002, 0.1528999999999998, 0.1533500000000001, 0.1618999999999997, 0.17855, 0.20330000000000004, 0.23614999999999986, 0.2770999999999999, 0.3261499999999997, 0.38329999999999975, 0.44854999999999956, 0.5218999999999991, 0.6033499999999994, 0.6929000000000003, 0.7905500000000001, 0.8962999999999988, 1.0101499999999999, 1.1320999999999999, 1.2621500000000005, 1.4003, 1.5465500000000003, 1.7008999999999994, 1.8633499999999992, 2.0338999999999996, 2.212549999999999, 2.399299999999999, 2.5941499999999995, 2.797099999999999, 3.008149999999999, 3.2272999999999983, 3.454549999999998, 3.6899, 3.9333499999999995, 4.184899999999999, 4.444549999999998, 4.712299999999997, 4.988149999999997, 5.2720999999999965, 5.56415, 5.864299999999998, 6.1725499999999975, 6.4888999999999974, 6.813349999999996, 7.1458999999999975, 7.486549999999999, 7.835299999999998, 8.19215, 8.557099999999997, 8.930149999999996, 9.311299999999997, 9.700549999999994, 10.097899999999994, 10.503349999999996, 10.9169, 11.33855, 11.768300000000002, 12.20615, 12.652099999999999 ], [ 8.126025, 7.7700749999999985, 7.422224999999999, 7.0824750000000005, 6.750825000000001, 6.427275, 6.111825, 5.804475, 5.505224999999999, 5.214074999999999, 4.931025, 4.6560749999999995, 4.389225, 4.130475000000001, 3.879825, 3.6372750000000003, 3.402825, 3.176475, 2.958225, 2.748075, 2.546025, 2.352075, 2.166225, 1.9884750000000002, 1.8188249999999995, 1.6572749999999998, 1.503825, 1.3584750000000003, 1.2212249999999998, 1.092075, 0.971025, 0.8580749999999999, 0.753225, 0.6564750000000004, 0.567825, 0.4872749999999999, 0.414825, 0.35047499999999987, 0.29422499999999996, 0.24607500000000027, 0.2060249999999999, 0.17407499999999976, 0.15022499999999983, 0.13447499999999968, 0.12682499999999974, 0.12727500000000003, 0.13582499999999964, 0.15247499999999992, 0.17722499999999997, 0.2100749999999998, 0.25102499999999983, 0.30007499999999965, 0.3572250000000001, 0.42247499999999905, 0.4958249999999995, 0.5772749999999998, 0.6668249999999998, 0.7644749999999996, 0.8702249999999991, 0.9840749999999994, 1.1060249999999994, 1.236075, 1.3742249999999996, 1.5204749999999998, 1.6748249999999998, 1.8372749999999995, 2.007824999999999, 2.186475, 2.3732249999999984, 2.5680750000000008, 2.7710249999999985, 2.9820749999999987, 3.2012249999999978, 3.4284749999999993, 3.6638250000000014, 3.9072750000000007, 4.158824999999998, 4.418474999999999, 4.6862249999999985, 4.962074999999997, 5.246024999999998, 5.538074999999999, 5.838224999999998, 6.146474999999999, 6.462824999999999, 6.7872749999999975, 7.119824999999999, 7.460475000000001, 7.809225, 8.166075000000001, 8.531024999999998, 8.904074999999997, 9.285224999999999, 9.674474999999996, 10.071824999999995, 10.477274999999997, 10.890825000000001, 11.312475000000001, 11.742225000000003, 12.180075, 12.626025 ], [ 8.1024, 7.74645, 7.3986, 7.05885, 6.727200000000001, 6.40365, 6.0882, 5.78085, 5.4816, 5.19045, 4.9074, 4.63245, 4.3656, 4.10685, 3.8562, 3.6136500000000003, 3.3792, 3.15285, 2.9346, 2.72445, 2.5224, 2.32845, 2.1426, 1.9648500000000002, 1.7951999999999997, 1.6336499999999998, 1.4802, 1.3348500000000003, 1.1976, 1.0684500000000001, 0.9474, 0.8344499999999999, 0.7296, 0.6328500000000004, 0.5442, 0.4636499999999999, 0.3912, 0.32684999999999986, 0.27059999999999995, 0.22245000000000026, 0.1823999999999999, 0.15044999999999975, 0.12659999999999982, 0.11085000000000012, 0.10319999999999974, 0.10365000000000002, 0.11219999999999963, 0.1288499999999999, 0.15359999999999996, 0.18644999999999978, 0.22739999999999982, 0.27644999999999964, 0.3335999999999997, 0.3988499999999995, 0.47219999999999995, 0.5536500000000002, 0.6432000000000002, 0.74085, 0.8465999999999996, 0.9604499999999998, 1.0823999999999998, 1.2124500000000005, 1.3506, 1.4968500000000002, 1.6512000000000002, 1.81365, 1.9841999999999986, 2.1628499999999997, 2.3495999999999997, 2.5444500000000003, 2.7474, 2.95845, 3.177599999999999, 3.404849999999999, 3.640200000000001, 3.8836500000000003, 4.1352, 4.394849999999999, 4.6625999999999985, 4.938449999999999, 5.222399999999998, 5.514450000000001, 5.8145999999999995, 6.122849999999999, 6.439199999999999, 6.763649999999996, 7.096199999999997, 7.436849999999999, 7.785599999999998, 8.14245, 8.507399999999997, 8.880449999999996, 9.261599999999998, 9.650849999999995, 10.048199999999994, 10.453649999999996, 10.8672, 11.28885, 11.718600000000002, 12.15645, 12.6024 ], [ 8.081225, 7.725275, 7.377425000000001, 7.037675, 6.706025, 6.382475000000001, 6.067024999999999, 5.759675, 5.460425000000001, 5.169275, 4.886225, 4.611275, 4.344424999999999, 4.085675, 3.835025, 3.5924750000000003, 3.358025, 3.131675, 2.913425, 2.703275, 2.5012250000000003, 2.307275, 2.121425, 1.9436750000000003, 1.7740249999999997, 1.6124749999999999, 1.4590250000000002, 1.3136750000000004, 1.176425, 1.0472750000000002, 0.9262250000000005, 0.813275, 0.7084250000000001, 0.611675, 0.5230250000000001, 0.44247499999999995, 0.37002500000000005, 0.3056749999999999, 0.249425, 0.20127500000000031, 0.16122499999999995, 0.1292749999999998, 0.10542499999999988, 0.08967500000000017, 0.08202499999999979, 0.08247500000000008, 0.09102499999999969, 0.10767499999999997, 0.13242500000000001, 0.16527499999999984, 0.20622499999999988, 0.2552749999999997, 0.31242499999999973, 0.37767499999999954, 0.451025, 0.5324750000000003, 0.6220250000000003, 0.7196750000000001, 0.8254249999999996, 0.9392749999999999, 1.0612249999999999, 1.1912750000000005, 1.329425, 1.4756750000000003, 1.6300250000000003, 1.792475, 1.9630249999999996, 2.141674999999999, 2.328424999999999, 2.5232749999999995, 2.726224999999999, 2.937274999999999, 3.1564249999999983, 3.383674999999998, 3.619025, 3.8624749999999994, 4.114025, 4.373674999999999, 4.641424999999998, 4.917274999999998, 5.201224999999997, 5.493275000000001, 5.793424999999999, 6.101674999999998, 6.418024999999998, 6.742474999999997, 7.075024999999998, 7.415675, 7.764424999999999, 8.121275, 8.486224999999997, 8.859274999999997, 9.240424999999998, 9.629674999999995, 10.027024999999995, 10.432474999999997, 10.846025000000001, 11.267675, 11.697425000000003, 12.135275, 12.581225 ], [ 8.0625, 7.70655, 7.358700000000001, 7.01895, 6.6873000000000005, 6.36375, 6.048299999999999, 5.74095, 5.4417, 5.150549999999999, 4.8675, 4.592549999999999, 4.325699999999999, 4.06695, 3.8162999999999996, 3.57375, 3.3393, 3.1129499999999997, 2.8947000000000003, 2.6845499999999998, 2.4825000000000004, 2.28855, 2.1027, 1.9249500000000002, 1.7552999999999996, 1.59375, 1.4403000000000001, 1.2949500000000005, 1.1576999999999997, 1.02855, 0.9075000000000002, 0.7945500000000001, 0.6897000000000002, 0.5929500000000001, 0.5042999999999997, 0.42375000000000007, 0.35130000000000017, 0.28695000000000004, 0.23070000000000013, 0.18255, 0.14250000000000007, 0.11054999999999993, 0.0867, 0.07094999999999985, 0.06329999999999991, 0.0637500000000002, 0.07229999999999981, 0.08895000000000008, 0.11370000000000013, 0.14654999999999996, 0.1875, 0.23654999999999982, 0.29369999999999985, 0.35894999999999966, 0.43229999999999924, 0.5137499999999995, 0.6032999999999995, 0.7009499999999993, 0.8066999999999989, 0.9205499999999991, 1.042499999999999, 1.1725499999999998, 1.3106999999999993, 1.4569499999999995, 1.6112999999999995, 1.7737499999999993, 1.9442999999999997, 2.122949999999999, 2.309699999999999, 2.5045499999999996, 2.707499999999999, 2.9185499999999993, 3.1376999999999984, 3.364949999999998, 3.6003000000000003, 3.8437499999999996, 4.0953, 4.354949999999999, 4.622699999999998, 4.898549999999998, 5.182499999999997, 5.474550000000001, 5.774699999999999, 6.0829499999999985, 6.399299999999998, 6.723749999999997, 7.0562999999999985, 7.39695, 7.745699999999999, 8.10255, 8.467499999999998, 8.840549999999997, 9.221699999999998, 9.610949999999995, 10.008299999999995, 10.413749999999997, 10.827300000000001, 11.24895, 11.678700000000003, 12.11655, 12.5625 ], [ 8.046225, 7.690275, 7.342425, 7.002675, 6.671025, 6.347475, 6.032025, 5.724675, 5.425425000000001, 5.134275, 4.851225, 4.576275, 4.309424999999999, 4.050675, 3.8000249999999998, 3.557475, 3.3230250000000003, 3.096675, 2.878425, 2.668275, 2.4662250000000006, 2.272275, 2.086425, 1.9086750000000001, 1.7390249999999998, 1.577475, 1.4240250000000003, 1.2786750000000005, 1.141425, 1.0122750000000003, 0.8912250000000004, 0.7782749999999998, 0.6734249999999999, 0.5766750000000003, 0.48802499999999993, 0.4074749999999998, 0.3350249999999999, 0.2706749999999998, 0.2144250000000003, 0.16627500000000017, 0.1262249999999998, 0.09427499999999966, 0.07042500000000018, 0.05467500000000003, 0.04702499999999965, 0.04747500000000038, 0.05602499999999999, 0.07267500000000027, 0.09742500000000032, 0.13027500000000014, 0.17122499999999974, 0.220275, 0.27742500000000003, 0.34267499999999984, 0.4160249999999994, 0.49747499999999967, 0.5870249999999997, 0.6846749999999995, 0.790424999999999, 0.9042749999999993, 1.0262249999999993, 1.156275, 1.2944249999999995, 1.4406749999999997, 1.5950249999999997, 1.7574749999999995, 1.9280249999999999, 2.106674999999999, 2.293424999999999, 2.488275, 2.6912249999999993, 2.9022749999999995, 3.1214249999999986, 3.3486749999999983, 3.5840250000000005, 3.8274749999999997, 4.079025, 4.3386749999999985, 4.606424999999998, 4.882274999999998, 5.166224999999997, 5.458275, 5.758424999999999, 6.066674999999998, 6.383024999999998, 6.707474999999997, 7.040024999999998, 7.380675, 7.729424999999999, 8.086275, 8.451224999999997, 8.824274999999997, 9.205424999999998, 9.594674999999995, 9.992024999999995, 10.397474999999996, 10.811025, 11.232675, 11.662425000000002, 12.100275, 12.546225 ], [ 8.032399999999999, 7.676449999999999, 7.3286, 6.988849999999999, 6.6572000000000005, 6.3336500000000004, 6.0182, 5.710850000000001, 5.4116, 5.12045, 4.837400000000001, 4.56245, 4.295599999999999, 4.03685, 3.7862, 3.5436500000000004, 3.3092, 3.0828499999999996, 2.8646000000000003, 2.6544499999999998, 2.4524000000000004, 2.25845, 2.0726, 1.8948500000000001, 1.7251999999999996, 1.56365, 1.4102000000000001, 1.2648500000000005, 1.1275999999999997, 0.9984500000000001, 0.8774000000000002, 0.7644500000000001, 0.6596000000000002, 0.5628500000000001, 0.47419999999999973, 0.39365000000000006, 0.32120000000000015, 0.25685, 0.2006000000000001, 0.15244999999999997, 0.11240000000000006, 0.08044999999999991, 0.056599999999999984, 0.04084999999999983, 0.033199999999999896, 0.03365000000000018, 0.04219999999999979, 0.05885000000000007, 0.08360000000000012, 0.11644999999999994, 0.15739999999999998, 0.20644999999999936, 0.2635999999999994, 0.3288499999999992, 0.40219999999999967, 0.4836499999999999, 0.5731999999999999, 0.6708499999999997, 0.7765999999999993, 0.8904499999999995, 1.0123999999999995, 1.1424500000000002, 1.2805999999999997, 1.42685, 1.5812, 1.7436499999999988, 1.9141999999999992, 2.0928499999999985, 2.2795999999999985, 2.474449999999999, 2.6773999999999987, 2.888449999999999, 3.107599999999998, 3.3348499999999976, 3.5702, 3.813649999999999, 4.065199999999999, 4.324849999999998, 4.592599999999997, 4.8684499999999975, 5.1523999999999965, 5.44445, 5.744599999999998, 6.052849999999998, 6.3691999999999975, 6.693649999999996, 7.026199999999998, 7.3668499999999995, 7.7155999999999985, 8.07245, 8.437399999999997, 8.810449999999996, 9.191599999999998, 9.580849999999995, 9.978199999999994, 10.383649999999996, 10.7972, 11.21885, 11.648600000000002, 12.08645, 12.532399999999999 ], [ 8.021025, 7.665075, 7.3172250000000005, 6.977475, 6.645825, 6.322274999999999, 6.006824999999999, 5.699475, 5.400225000000001, 5.109075, 4.826025, 4.551075, 4.284224999999999, 4.025475, 3.774825, 3.5322750000000003, 3.297825, 3.071475, 2.853225, 2.643075, 2.4410250000000002, 2.247075, 2.061225, 1.8834750000000002, 1.7138249999999997, 1.5522749999999998, 1.3988250000000002, 1.2534750000000003, 1.116225, 0.9870749999999999, 0.8660250000000005, 0.7530749999999999, 0.648225, 0.5514749999999999, 0.46282500000000004, 0.3822749999999999, 0.309825, 0.2454749999999999, 0.18922499999999998, 0.14107500000000028, 0.10102499999999992, 0.06907499999999978, 0.04522499999999985, 0.02947500000000014, 0.02182499999999976, 0.022275000000000045, 0.030824999999999658, 0.047474999999999934, 0.07222499999999998, 0.10507499999999981, 0.14602499999999985, 0.19507499999999967, 0.2522249999999997, 0.3174749999999995, 0.390825, 0.4722750000000002, 0.5618250000000002, 0.659475, 0.7652249999999996, 0.8790749999999998, 1.0010249999999998, 1.1310750000000005, 1.269225, 1.4154750000000003, 1.5698250000000002, 1.732275, 1.9028249999999987, 2.0814749999999997, 2.2682249999999997, 2.4630750000000003, 2.666025, 2.877075, 3.096224999999999, 3.323474999999999, 3.558825000000001, 3.8022750000000003, 4.053825, 4.313474999999999, 4.581224999999998, 4.857074999999998, 5.141024999999997, 5.4330750000000005, 5.733224999999999, 6.041474999999998, 6.3578249999999965, 6.682274999999995, 7.0148249999999965, 7.355474999999998, 7.704224999999997, 8.061074999999999, 8.426024999999996, 8.799074999999995, 9.180224999999997, 9.569474999999994, 9.966824999999993, 10.372274999999995, 10.785824999999999, 11.207474999999999, 11.637225, 12.075074999999998, 12.521024999999998 ], [ 8.0121, 7.656149999999999, 7.3083, 6.96855, 6.636900000000001, 6.313350000000001, 5.9979, 5.690550000000001, 5.3913, 5.100149999999999, 4.8171, 4.5421499999999995, 4.2753, 4.0165500000000005, 3.7659000000000002, 3.5233500000000006, 3.2889, 3.06255, 2.8443000000000005, 2.63415, 2.4321, 2.23815, 2.0523, 1.8745500000000002, 1.7048999999999999, 1.54335, 1.3899000000000004, 1.2445500000000005, 1.1073, 0.9781500000000003, 0.8571000000000004, 0.7441499999999999, 0.6393000000000004, 0.5425500000000003, 0.45389999999999997, 0.3733500000000003, 0.3009000000000004, 0.23654999999999982, 0.1802999999999999, 0.1321500000000002, 0.09210000000000029, 0.06015000000000015, 0.03629999999999978, 0.020550000000000068, 0.012900000000000134, 0.013349999999999973, 0.021899999999999586, 0.03854999999999986, 0.06329999999999991, 0.09614999999999974, 0.13710000000000022, 0.1861499999999996, 0.24329999999999963, 0.30854999999999944, 0.3818999999999999, 0.46335000000000015, 0.5529000000000002, 0.65055, 0.7562999999999995, 0.8701499999999998, 0.9920999999999998, 1.1221500000000004, 1.2603, 1.4065500000000002, 1.5609000000000002, 1.723349999999999, 1.8938999999999995, 2.072549999999999, 2.2592999999999988, 2.4541499999999994, 2.657099999999999, 2.868149999999999, 3.087299999999998, 3.314549999999998, 3.5499, 3.7933499999999993, 4.044899999999999, 4.304549999999998, 4.572299999999998, 4.848149999999998, 5.132099999999997, 5.42415, 5.724299999999999, 6.032549999999998, 6.3488999999999995, 6.673349999999998, 7.0059, 7.3465500000000015, 7.6953000000000005, 8.052150000000001, 8.417099999999998, 8.790149999999997, 9.171299999999999, 9.560549999999996, 9.957899999999995, 10.363349999999997, 10.776900000000001, 11.198550000000001, 11.628300000000003, 12.06615, 12.5121 ], [ 8.005625, 7.649674999999999, 7.301825, 6.962075, 6.630425000000001, 6.306875000000001, 5.991425, 5.684075000000001, 5.384825, 5.093674999999999, 4.810625, 4.5356749999999995, 4.268825, 4.0100750000000005, 3.7594250000000002, 3.5168750000000006, 3.282425, 3.056075, 2.8378250000000005, 2.627675, 2.425625, 2.231675, 2.045825, 1.8680750000000002, 1.6984249999999999, 1.536875, 1.3834250000000003, 1.2380750000000005, 1.100825, 0.9716750000000003, 0.8506250000000004, 0.7376749999999999, 0.6328250000000004, 0.5360750000000003, 0.44742499999999996, 0.3668750000000003, 0.2944250000000004, 0.2300749999999998, 0.1738249999999999, 0.1256750000000002, 0.08562500000000028, 0.05367500000000014, 0.02982499999999977, 0.01407500000000006, 0.006425000000000125, 0.0068749999999999645, 0.015424999999999578, 0.032074999999999854, 0.0568249999999999, 0.08967499999999973, 0.1306250000000002, 0.17967499999999959, 0.23682499999999962, 0.3020749999999994, 0.3754249999999999, 0.45687500000000014, 0.5464250000000002, 0.644075, 0.7498249999999995, 0.8636749999999997, 0.9856249999999998, 1.1156750000000004, 1.253825, 1.4000750000000002, 1.5544250000000002, 1.716874999999999, 1.8874249999999995, 2.0660749999999988, 2.2528249999999987, 2.4476749999999994, 2.650624999999999, 2.861674999999999, 3.080824999999998, 3.308074999999998, 3.543425, 3.7868749999999993, 4.038424999999999, 4.298074999999998, 4.565824999999998, 4.841674999999998, 5.125624999999997, 5.417675, 5.717824999999999, 6.026074999999998, 6.342424999999998, 6.6668749999999966, 6.999424999999998, 7.340075, 7.688824999999999, 8.045675, 8.410624999999996, 8.783674999999995, 9.164824999999997, 9.554074999999994, 9.951424999999993, 10.356874999999995, 10.770425, 11.192074999999999, 11.621825000000001, 12.059674999999999, 12.505624999999998 ], [ 8.0016, 7.64565, 7.2978000000000005, 6.95805, 6.6264, 6.302850000000001, 5.987399999999999, 5.68005, 5.380800000000001, 5.089650000000001, 4.8065999999999995, 4.53165, 4.264799999999999, 4.00605, 3.7554, 3.5128500000000003, 3.2784, 3.05205, 2.8338, 2.62365, 2.4216, 2.22765, 2.0418, 1.8640500000000002, 1.6943999999999997, 1.5328499999999998, 1.3794000000000002, 1.2340500000000003, 1.0968, 0.9676499999999999, 0.8466000000000005, 0.7336499999999999, 0.6288, 0.5320499999999999, 0.4434, 0.3628499999999999, 0.2904, 0.22604999999999986, 0.16979999999999995, 0.12165000000000026, 0.0815999999999999, 0.04964999999999975, 0.025799999999999823, 0.010050000000000114, 0.0023999999999997357, 0.002850000000000019, 0.011399999999999633, 0.02804999999999991, 0.05279999999999996, 0.08564999999999978, 0.12660000000000027, 0.17564999999999964, 0.23279999999999967, 0.2980499999999995, 0.37139999999999995, 0.4528500000000002, 0.5424000000000002, 0.64005, 0.7457999999999996, 0.8596499999999998, 0.9815999999999998, 1.1116500000000005, 1.2498, 1.3960500000000002, 1.5504000000000002, 1.712849999999999, 1.8833999999999995, 2.062049999999999, 2.248799999999999, 2.4436499999999994, 2.646599999999999, 2.857649999999999, 3.076799999999998, 3.304049999999998, 3.5394, 3.7828499999999994, 4.0344, 4.294049999999999, 4.561799999999998, 4.837649999999998, 5.121599999999997, 5.4136500000000005, 5.713799999999999, 6.022049999999998, 6.3384, 6.662849999999999, 6.9954, 7.336050000000002, 7.684800000000001, 8.041650000000002, 8.4066, 8.779649999999998, 9.1608, 9.550049999999997, 9.947399999999996, 10.352849999999998, 10.766400000000003, 11.188050000000002, 11.617800000000004, 12.055650000000002, 12.501600000000002 ], [ 8.000025, 7.644075, 7.296225, 6.956475, 6.624825, 6.301275, 5.985825, 5.678475000000001, 5.379225, 5.088075, 4.8050250000000005, 4.530075, 4.263225, 4.004475, 3.753825, 3.5112750000000004, 3.276825, 3.050475, 2.832225, 2.622075, 2.4200250000000008, 2.2260750000000002, 2.040225, 1.8624750000000003, 1.6928249999999998, 1.531275, 1.3778250000000003, 1.2324750000000004, 1.0952250000000001, 0.966075, 0.8450250000000006, 0.732075, 0.6272250000000001, 0.530475, 0.44182500000000013, 0.361275, 0.2888250000000001, 0.22447499999999998, 0.16822500000000007, 0.12007500000000038, 0.08002500000000001, 0.04807499999999987, 0.02422499999999994, 0.008475000000000232, 0.0008249999999998536, 0.0012750000000001371, 0.00982499999999975, 0.026475000000000026, 0.051225000000000076, 0.0840749999999999, 0.12502499999999994, 0.1740750000000002, 0.23122500000000024, 0.29647500000000004, 0.3698249999999996, 0.45127499999999987, 0.5408249999999999, 0.6384749999999997, 0.7442249999999992, 0.8580749999999995, 0.9800249999999995, 1.1100750000000001, 1.2482249999999997, 1.394475, 1.5488250000000008, 1.7112750000000005, 1.8818249999999992, 2.0604750000000003, 2.2472250000000003, 2.442075000000001, 2.6450250000000004, 2.8560750000000006, 3.0752249999999997, 3.3024749999999994, 3.5378250000000016, 3.781275000000001, 4.032825000000001, 4.292475, 4.560224999999999, 4.836074999999999, 5.120024999999998, 5.4120750000000015, 5.712225, 6.020474999999999, 6.3368249999999975, 6.661274999999996, 6.9938249999999975, 7.334474999999999, 7.683224999999998, 8.040075, 8.405024999999997, 8.778074999999996, 9.159224999999998, 9.548474999999994, 9.945824999999994, 10.351274999999996, 10.764825, 11.186475, 11.616225000000002, 12.054075, 12.500024999999999 ], [ 8.0009, 7.64495, 7.2971, 6.95735, 6.6257, 6.302150000000001, 5.9867, 5.67935, 5.3801000000000005, 5.0889500000000005, 4.8059, 4.530950000000001, 4.2641, 4.00535, 3.7547, 3.5121500000000005, 3.2777000000000003, 3.0513500000000002, 2.8331000000000004, 2.6229500000000003, 2.4209000000000005, 2.2269500000000004, 2.0411, 1.8633500000000005, 1.6936999999999998, 1.5321500000000001, 1.3787000000000003, 1.2333500000000006, 1.0961, 0.9669500000000002, 0.8459000000000003, 0.7329500000000002, 0.6281000000000003, 0.5313500000000007, 0.4427000000000003, 0.3621500000000002, 0.2897000000000003, 0.22535000000000016, 0.16910000000000025, 0.12095000000000056, 0.0809000000000002, 0.04895000000000005, 0.025100000000000122, 0.009350000000000414, 0.0017000000000000348, 0.0021500000000003183, 0.010699999999999932, 0.027350000000000207, 0.05210000000000026, 0.08495000000000008, 0.12589999999999968, 0.17494999999999994, 0.23209999999999997, 0.2973499999999998, 0.37069999999999936, 0.4521499999999996, 0.5417000000000005, 0.6393500000000003, 0.745099999999999, 0.8589500000000001, 0.9809000000000001, 1.1109500000000008, 1.2491000000000003, 1.3953500000000005, 1.5496999999999996, 1.7121499999999994, 1.8826999999999998, 2.061349999999999, 2.248099999999999, 2.4429499999999997, 2.6458999999999993, 2.8569499999999994, 3.0760999999999985, 3.3033499999999982, 3.5387000000000004, 3.7821499999999997, 4.0337, 4.2933499999999984, 4.561099999999998, 4.836949999999998, 5.120899999999997, 5.41295, 5.713099999999999, 6.021349999999998, 6.337699999999998, 6.662149999999997, 6.994699999999998, 7.33535, 7.684099999999999, 8.04095, 8.405899999999997, 8.778949999999996, 9.160099999999998, 9.549349999999995, 9.946699999999995, 10.352149999999996, 10.7657, 11.18735, 11.617100000000002, 12.05495, 12.5009 ], [ 8.004225, 7.648275, 7.300425000000001, 6.960675, 6.629025, 6.305475, 5.990025, 5.682675000000001, 5.383425000000001, 5.092275, 4.8092250000000005, 4.534275, 4.267424999999999, 4.008675, 3.758025, 3.5154750000000003, 3.2810250000000005, 3.054675, 2.836425, 2.626275, 2.4242250000000007, 2.2302750000000007, 2.0444250000000004, 1.8666750000000005, 1.6970249999999998, 1.5354750000000001, 1.3820250000000003, 1.2366750000000006, 1.099425, 0.9702750000000004, 0.8492250000000006, 0.736275, 0.6314250000000001, 0.5346750000000005, 0.4460250000000001, 0.365475, 0.2930250000000001, 0.22867499999999996, 0.1724250000000005, 0.12427500000000036, 0.084225, 0.05227499999999985, 0.028425000000000367, 0.012675000000000214, 0.005024999999999835, 0.005475000000000563, 0.014025000000000176, 0.030675000000000452, 0.0554250000000005, 0.08827500000000033, 0.12922499999999992, 0.17827500000000018, 0.23542500000000022, 0.300675, 0.3740249999999996, 0.45547499999999985, 0.5450249999999999, 0.6426749999999997, 0.7484249999999992, 0.8622749999999995, 0.9842249999999995, 1.1142750000000001, 1.2524249999999997, 1.398675, 1.5530249999999999, 1.7154749999999996, 1.886025, 2.0646749999999994, 2.2514249999999993, 2.446275, 2.6492249999999995, 2.8602749999999997, 3.0794249999999987, 3.3066749999999985, 3.5420250000000006, 3.785475, 4.037025, 4.296674999999999, 4.564424999999998, 4.840274999999998, 5.124224999999997, 5.416275000000001, 5.716424999999999, 6.0246749999999984, 6.341024999999998, 6.665474999999997, 6.998024999999998, 7.338675, 7.687424999999999, 8.044275, 8.409224999999998, 8.782274999999997, 9.163424999999998, 9.552674999999995, 9.950024999999995, 10.355474999999997, 10.769025000000001, 11.190675, 11.620425000000003, 12.058275, 12.504225 ], [ 8.01, 7.65405, 7.3062, 6.96645, 6.6348, 6.31125, 5.9958, 5.6884500000000005, 5.3892, 5.09805, 4.815, 4.54005, 4.273199999999999, 4.01445, 3.7638, 3.52125, 3.2868000000000004, 3.06045, 2.8422, 2.63205, 2.4300000000000006, 2.2360500000000005, 2.0502000000000002, 1.8724500000000004, 1.7027999999999996, 1.54125, 1.3878000000000001, 1.2424500000000005, 1.1052, 0.9760500000000003, 0.8550000000000004, 0.7420499999999999, 0.6372, 0.5404500000000003, 0.4518, 0.37124999999999986, 0.29879999999999995, 0.23444999999999983, 0.17820000000000036, 0.13005000000000022, 0.08999999999999986, 0.05804999999999971, 0.03420000000000023, 0.018450000000000077, 0.010799999999999699, 0.011250000000000426, 0.01980000000000004, 0.036450000000000315, 0.061200000000000365, 0.09405000000000019, 0.1349999999999998, 0.18405000000000005, 0.24120000000000008, 0.3064499999999999, 0.37979999999999947, 0.4612499999999997, 0.5507999999999997, 0.6484499999999995, 0.7541999999999991, 0.8680499999999993, 0.9899999999999993, 1.12005, 1.2581999999999995, 1.4044499999999998, 1.5588000000000006, 1.7212500000000004, 1.891799999999999, 2.07045, 2.2572, 2.4520500000000007, 2.6550000000000002, 2.8660500000000004, 3.0851999999999995, 3.3124499999999992, 3.5478000000000014, 3.7912500000000007, 4.042800000000001, 4.302449999999999, 4.570199999999999, 4.846049999999999, 5.129999999999998, 5.422050000000001, 5.7222, 6.030449999999999, 6.346799999999999, 6.671249999999998, 7.003799999999999, 7.344450000000001, 7.6932, 8.050050000000002, 8.415, 8.788049999999998, 9.1692, 9.558449999999997, 9.955799999999996, 10.361249999999998, 10.774800000000003, 11.196450000000002, 11.626200000000004, 12.064050000000002, 12.510000000000002 ], [ 8.018225000000001, 7.662275, 7.314425000000001, 6.974675, 6.643025000000001, 6.319475, 6.0040249999999995, 5.696675, 5.397425, 5.106275, 4.823225, 4.548275, 4.2814250000000005, 4.0226750000000004, 3.772025, 3.5294750000000006, 3.2950250000000003, 3.068675, 2.8504250000000004, 2.640275, 2.4382250000000005, 2.2442750000000005, 2.058425, 1.8806750000000005, 1.7110249999999998, 1.5494750000000002, 1.3960250000000003, 1.2506750000000006, 1.113425, 0.9842750000000002, 0.8632250000000008, 0.7502750000000002, 0.6454250000000004, 0.5486750000000002, 0.4600249999999999, 0.37947500000000023, 0.3070250000000003, 0.2426750000000002, 0.18642500000000028, 0.13827500000000015, 0.09822500000000023, 0.06627500000000008, 0.04242500000000016, 0.026675000000000004, 0.01902500000000007, 0.019475000000000353, 0.028024999999999967, 0.04467500000000024, 0.06942500000000029, 0.10227499999999967, 0.14322500000000016, 0.19227499999999953, 0.24942499999999956, 0.31467500000000026, 0.38802499999999984, 0.4694750000000001, 0.5590250000000001, 0.6566749999999999, 0.7624249999999995, 0.8762749999999997, 0.9982249999999997, 1.1282750000000004, 1.266425, 1.4126749999999992, 1.5670250000000001, 1.7294749999999999, 1.9000249999999985, 2.0786749999999996, 2.2654249999999996, 2.460275, 2.6632249999999997, 2.874275, 3.093424999999999, 3.3206749999999987, 3.556025000000001, 3.799475, 4.051025, 4.310674999999999, 4.578424999999998, 4.854274999999999, 5.138224999999998, 5.430275000000001, 5.730424999999999, 6.038674999999999, 6.355024999999999, 6.679474999999997, 7.012024999999999, 7.3526750000000005, 7.7014249999999995, 8.058275000000002, 8.423224999999999, 8.796274999999998, 9.177425, 9.566674999999996, 9.964024999999996, 10.369474999999998, 10.783025000000002, 11.204675000000002, 11.634425000000004, 12.072275000000001, 12.518225000000001 ], [ 8.0289, 7.672949999999999, 7.3251, 6.98535, 6.653700000000001, 6.330150000000001, 6.0147, 5.707350000000001, 5.4081, 5.11695, 4.833900000000001, 4.55895, 4.2921, 4.03335, 3.7827, 3.5401500000000006, 3.3057000000000003, 3.07935, 2.8611000000000004, 2.65095, 2.4489000000000005, 2.2549500000000005, 2.0691, 1.8913500000000005, 1.7216999999999998, 1.5601500000000001, 1.4067000000000003, 1.2613500000000006, 1.1241, 0.9949500000000002, 0.8739000000000008, 0.7609500000000002, 0.6561000000000003, 0.5593500000000002, 0.4706999999999999, 0.3901500000000002, 0.3177000000000003, 0.2533500000000002, 0.19710000000000027, 0.14895000000000014, 0.10890000000000022, 0.07695000000000007, 0.05310000000000015, 0.037349999999999994, 0.02970000000000006, 0.030150000000000343, 0.03869999999999996, 0.05535000000000023, 0.08010000000000073, 0.11295000000000055, 0.15390000000000015, 0.2029500000000004, 0.26010000000000044, 0.32534999999999936, 0.39869999999999983, 0.4801500000000001, 0.5697000000000001, 0.6673499999999999, 0.7730999999999995, 0.8869499999999997, 1.0088999999999997, 1.1389500000000004, 1.2771, 1.4233500000000001, 1.5776999999999992, 1.740149999999999, 1.9106999999999994, 2.0893499999999987, 2.2761000000000005, 2.4709499999999993, 2.6739000000000006, 2.884950000000001, 3.1041, 3.331349999999998, 3.5667, 3.8101499999999993, 4.061700000000001, 4.321349999999998, 4.5890999999999975, 4.864949999999999, 5.148899999999997, 5.440950000000002, 5.7411, 6.049349999999998, 6.365699999999998, 6.6901499999999965, 7.022699999999998, 7.36335, 7.712099999999999, 8.068950000000001, 8.433899999999998, 8.806949999999997, 9.188099999999999, 9.577349999999996, 9.974699999999995, 10.380149999999997, 10.793700000000001, 11.21535, 11.645100000000003, 12.08295, 12.5289 ], [ 8.042024999999999, 7.686075, 7.3382249999999996, 6.998475, 6.666825, 6.343275, 6.027825, 5.720475, 5.421225, 5.130075, 4.847025, 4.572075, 4.305225, 4.046475, 3.795825, 3.553275, 3.3188250000000004, 3.092475, 2.874225, 2.664075, 2.4620250000000006, 2.2680750000000005, 2.082225, 1.9044750000000004, 1.7348249999999996, 1.573275, 1.4198250000000001, 1.2744750000000005, 1.137225, 1.0080750000000003, 0.8870250000000004, 0.7740749999999998, 0.669225, 0.5724750000000003, 0.48382499999999995, 0.40327499999999983, 0.3308249999999999, 0.2664749999999998, 0.21022500000000033, 0.1620750000000002, 0.12202499999999983, 0.09007499999999968, 0.0662250000000002, 0.05047500000000005, 0.04282499999999967, 0.043275000000000396, 0.05182500000000001, 0.06847500000000029, 0.09322500000000034, 0.12607500000000016, 0.16702499999999976, 0.21607500000000002, 0.27322500000000005, 0.33847499999999986, 0.41182499999999944, 0.4932749999999997, 0.5828249999999997, 0.6804749999999995, 0.7862249999999991, 0.9000749999999993, 1.0220249999999993, 1.152075, 1.2902249999999995, 1.4364749999999997, 1.5908250000000006, 1.7532750000000004, 1.923824999999999, 2.102475, 2.289225, 2.4840750000000007, 2.687025, 2.8980750000000004, 3.1172249999999995, 3.344474999999999, 3.5798250000000014, 3.8232750000000006, 4.074825000000001, 4.334474999999999, 4.602224999999999, 4.878074999999999, 5.162024999999998, 5.454075000000001, 5.754225, 6.062474999999997, 6.378824999999997, 6.703274999999996, 7.035824999999997, 7.376474999999999, 7.725224999999998, 8.082075, 8.447024999999996, 8.820074999999996, 9.201224999999997, 9.590474999999994, 9.987824999999994, 10.393274999999996, 10.806825, 11.228475, 11.658225000000002, 12.096074999999999, 12.542024999999999 ], [ 8.0576, 7.70165, 7.3538, 7.01405, 6.6824, 6.35885, 6.0434, 5.7360500000000005, 5.4368, 5.14565, 4.8626000000000005, 4.58765, 4.3208, 4.06205, 3.8114, 3.5688500000000003, 3.3344000000000005, 3.10805, 2.8898, 2.67965, 2.4776000000000007, 2.2836500000000006, 2.0978000000000003, 1.9200500000000005, 1.7503999999999997, 1.58885, 1.4354000000000002, 1.2900500000000006, 1.1528, 1.0236500000000004, 0.9026000000000005, 0.78965, 0.6848000000000001, 0.5880500000000004, 0.49940000000000007, 0.41884999999999994, 0.34640000000000004, 0.2820499999999999, 0.22580000000000044, 0.1776500000000003, 0.13759999999999994, 0.1056499999999998, 0.08180000000000032, 0.06605000000000016, 0.058399999999999785, 0.05885000000000051, 0.06740000000000013, 0.0840500000000004, 0.10880000000000045, 0.14165000000000028, 0.18259999999999987, 0.23165000000000013, 0.28880000000000017, 0.35405, 0.42739999999999956, 0.5088499999999998, 0.5983999999999998, 0.6960499999999996, 0.8017999999999992, 0.9156499999999994, 1.0375999999999994, 1.16765, 1.3057999999999996, 1.4520499999999998, 1.6064000000000007, 1.7688500000000005, 1.9393999999999991, 2.11805, 2.3048, 2.499650000000001, 2.7026000000000003, 2.9136500000000005, 3.1327999999999996, 3.3600499999999993, 3.5954000000000015, 3.8388500000000008, 4.090400000000001, 4.3500499999999995, 4.617799999999999, 4.893649999999999, 5.177599999999998, 5.4696500000000015, 5.769799999999998, 6.078049999999999, 6.394399999999999, 6.718849999999998, 7.051399999999999, 7.392050000000001, 7.7408, 8.097650000000002, 8.462599999999998, 8.835649999999998, 9.2168, 9.606049999999996, 10.003399999999996, 10.408849999999997, 10.822400000000002, 11.244050000000001, 11.673800000000004, 12.111650000000001, 12.5576 ], [ 8.075625, 7.7196750000000005, 7.371825000000001, 7.032075000000001, 6.700425000000001, 6.376875, 6.061425, 5.754075, 5.4548250000000005, 5.163675, 4.880625, 4.605675, 4.338825, 4.080075000000001, 3.8294250000000005, 3.586875000000001, 3.352425, 3.126075, 2.9078250000000008, 2.6976750000000003, 2.4956250000000004, 2.3016750000000004, 2.115825, 1.9380750000000007, 1.768425, 1.6068750000000003, 1.4534250000000004, 1.3080750000000008, 1.1708250000000002, 1.0416750000000006, 0.9206250000000007, 0.8076750000000001, 0.7028250000000003, 0.6060750000000006, 0.5174250000000002, 0.43687500000000057, 0.36442500000000067, 0.3000750000000001, 0.24382500000000018, 0.1956750000000005, 0.15562500000000057, 0.12367500000000042, 0.09982500000000005, 0.08407500000000034, 0.07642500000000041, 0.07687500000000025, 0.08542499999999986, 0.10207500000000014, 0.1268250000000002, 0.159675, 0.2006250000000005, 0.24967499999999987, 0.3068249999999999, 0.3720749999999997, 0.4454250000000002, 0.5268750000000004, 0.6164250000000004, 0.7140750000000002, 0.8198249999999998, 0.933675, 1.055625, 1.1856750000000007, 1.3238249999999994, 1.4700749999999996, 1.6244250000000005, 1.7868750000000002, 1.9574249999999989, 2.136075, 2.322825, 2.5176750000000006, 2.720625, 2.9316750000000003, 3.1508249999999993, 3.378074999999999, 3.613425000000001, 3.8568750000000005, 4.108425, 4.368074999999999, 4.635824999999999, 4.911674999999999, 5.195624999999998, 5.487675000000001, 5.787825, 6.096074999999997, 6.412424999999997, 6.736874999999996, 7.069424999999997, 7.410074999999999, 7.758824999999998, 8.115675, 8.480624999999996, 8.853674999999996, 9.234824999999997, 9.624074999999994, 10.021424999999994, 10.426874999999995, 10.840425, 11.262075, 11.691825000000001, 12.129674999999999, 12.575624999999999 ], [ 8.0961, 7.74015, 7.3923, 7.05255, 6.7209, 6.39735, 6.0819, 5.7745500000000005, 5.4753, 5.18415, 4.9011000000000005, 4.62615, 4.359299999999999, 4.10055, 3.8499, 3.6073500000000003, 3.3729000000000005, 3.14655, 2.9283, 2.71815, 2.5161000000000007, 2.3221500000000006, 2.1363000000000003, 1.9585500000000005, 1.7888999999999997, 1.62735, 1.4739000000000002, 1.3285500000000006, 1.1913, 1.0621500000000004, 0.9411000000000005, 0.8281499999999999, 0.7233, 0.6265500000000004, 0.5379, 0.4573499999999999, 0.3849, 0.3205499999999999, 0.2643000000000004, 0.2161500000000003, 0.17609999999999992, 0.14414999999999978, 0.1203000000000003, 0.10455000000000014, 0.09689999999999976, 0.09735000000000049, 0.1059000000000001, 0.12255000000000038, 0.14730000000000043, 0.18015000000000025, 0.22109999999999985, 0.2701500000000001, 0.32730000000000015, 0.39254999999999995, 0.46589999999999954, 0.5473499999999998, 0.6368999999999998, 0.7345499999999996, 0.8402999999999992, 0.9541499999999994, 1.0760999999999994, 1.20615, 1.3442999999999996, 1.4905499999999998, 1.6449000000000007, 1.8073500000000005, 1.977899999999999, 2.15655, 2.3433, 2.538150000000001, 2.7411000000000003, 2.9521500000000005, 3.1712999999999996, 3.3985499999999993, 3.6339000000000015, 3.8773500000000007, 4.128900000000001, 4.3885499999999995, 4.656299999999999, 4.932149999999999, 5.216099999999998, 5.508150000000001, 5.808299999999998, 6.116549999999999, 6.432899999999999, 6.757349999999998, 7.089899999999999, 7.430550000000001, 7.7793, 8.13615, 8.501099999999997, 8.874149999999997, 9.255299999999998, 9.644549999999995, 10.041899999999995, 10.447349999999997, 10.8609, 11.28255, 11.712300000000003, 12.15015, 12.5961 ], [ 8.119025, 7.763075, 7.415225, 7.075475, 6.743825, 6.420275000000001, 6.104825, 5.797475, 5.498225000000001, 5.207075000000001, 4.924025, 4.649075000000001, 4.382225, 4.123475000000001, 3.872825, 3.6302750000000006, 3.3958250000000003, 3.1694750000000003, 2.9512250000000004, 2.7410750000000004, 2.5390250000000005, 2.3450750000000005, 2.159225, 1.9814750000000005, 1.8118249999999998, 1.6502750000000002, 1.4968250000000003, 1.3514750000000006, 1.214225, 1.0850750000000002, 0.9640250000000004, 0.8510750000000002, 0.7462250000000004, 0.6494750000000002, 0.5608250000000004, 0.48027500000000023, 0.4078250000000003, 0.3434750000000002, 0.2872250000000003, 0.2390750000000006, 0.19902500000000023, 0.16707500000000008, 0.14322500000000016, 0.12747500000000045, 0.11982500000000007, 0.12027500000000035, 0.12882499999999997, 0.14547500000000024, 0.1702250000000003, 0.20307500000000012, 0.24402499999999971, 0.293075, 0.350225, 0.4154749999999998, 0.4888249999999994, 0.5702749999999996, 0.6598250000000005, 0.7574750000000003, 0.863224999999999, 0.9770750000000001, 1.0990250000000001, 1.2290750000000008, 1.3672250000000004, 1.5134750000000006, 1.6678249999999997, 1.8302749999999994, 2.000825, 2.179474999999999, 2.366224999999999, 2.5610749999999998, 2.7640249999999993, 2.9750749999999995, 3.1942249999999985, 3.4214749999999983, 3.6568250000000004, 3.9002749999999997, 4.151825, 4.4114749999999985, 4.679224999999998, 4.955074999999998, 5.239024999999997, 5.531075, 5.831224999999999, 6.139475, 6.455825, 6.780274999999999, 7.112825, 7.453475000000002, 7.802225000000001, 8.159075000000001, 8.524024999999998, 8.897074999999997, 9.278224999999999, 9.667474999999996, 10.064824999999995, 10.470274999999997, 10.883825000000002, 11.305475000000001, 11.735225000000003, 12.173075, 12.619025 ], [ 8.144400000000001, 7.78845, 7.440600000000001, 7.10085, 6.7692000000000005, 6.4456500000000005, 6.1302, 5.822850000000001, 5.523600000000001, 5.23245, 4.949400000000001, 4.67445, 4.4075999999999995, 4.14885, 3.8982, 3.6556500000000005, 3.4212000000000002, 3.19485, 2.9766000000000004, 2.7664500000000003, 2.564400000000001, 2.370450000000001, 2.1846000000000005, 2.006850000000001, 1.8372, 1.6756500000000003, 1.5222000000000004, 1.3768500000000008, 1.2396000000000003, 1.1104500000000006, 0.9894000000000007, 0.8764500000000002, 0.7716000000000003, 0.6748500000000006, 0.5862000000000003, 0.5056500000000002, 0.43320000000000025, 0.3688500000000001, 0.31260000000000066, 0.2644500000000005, 0.22440000000000015, 0.19245, 0.16860000000000053, 0.15285000000000037, 0.1452, 0.14565000000000072, 0.15420000000000034, 0.1708500000000006, 0.19560000000000066, 0.22845000000000049, 0.2694000000000001, 0.31845000000000034, 0.3756000000000004, 0.4408500000000002, 0.5141999999999998, 0.59565, 0.6852, 0.7828499999999998, 0.8885999999999994, 1.0024499999999996, 1.1243999999999996, 1.2544500000000003, 1.3926000000000007, 1.538850000000001, 1.6932, 1.8556499999999998, 2.0262000000000002, 2.2048499999999995, 2.3915999999999995, 2.58645, 2.7893999999999997, 3.00045, 3.219599999999999, 3.4468499999999986, 3.682200000000001, 3.92565, 4.1772, 4.436849999999999, 4.704599999999998, 4.9804499999999985, 5.2643999999999975, 5.556450000000001, 5.856599999999999, 6.16485, 6.4812, 6.805649999999999, 7.1382, 7.478850000000002, 7.827600000000001, 8.184450000000002, 8.549399999999999, 8.922449999999998, 9.3036, 9.692849999999996, 10.090199999999996, 10.495649999999998, 10.909200000000002, 11.330850000000002, 11.760600000000004, 12.198450000000001, 12.644400000000001 ], [ 8.172225, 7.816274999999999, 7.468425, 7.128675, 6.7970250000000005, 6.4734750000000005, 6.158025, 5.850675000000001, 5.551425, 5.260275, 4.977225000000001, 4.702275, 4.435425, 4.176675, 3.926025, 3.6834750000000005, 3.4490250000000002, 3.222675, 3.0044250000000003, 2.7942750000000003, 2.5922250000000004, 2.3982750000000004, 2.212425, 2.0346750000000005, 1.8650249999999997, 1.703475, 1.5500250000000002, 1.4046750000000006, 1.2674250000000002, 1.1382750000000001, 1.0172250000000003, 0.9042750000000002, 0.7994250000000003, 0.7026750000000002, 0.6140249999999998, 0.5334750000000001, 0.46102500000000024, 0.3966750000000001, 0.3404250000000002, 0.29227500000000006, 0.25222500000000014, 0.220275, 0.19642500000000007, 0.18067499999999992, 0.17302499999999998, 0.17347499999999982, 0.18202499999999944, 0.1986749999999997, 0.22342500000000065, 0.2562750000000005, 0.2972250000000001, 0.34627500000000033, 0.40342500000000037, 0.4686749999999993, 0.5420249999999998, 0.623475, 0.713025, 0.8106749999999998, 0.9164249999999994, 1.0302749999999996, 1.1522249999999996, 1.2822750000000003, 1.4204249999999998, 1.566675, 1.7210249999999991, 1.883474999999999, 2.0540249999999993, 2.2326749999999986, 2.4194250000000004, 2.6142749999999992, 2.8172250000000005, 3.0282750000000007, 3.247425, 3.4746749999999977, 3.710025, 3.953474999999999, 4.205025000000001, 4.464674999999998, 4.732424999999997, 5.008274999999999, 5.292224999999997, 5.584275000000002, 5.884425, 6.192674999999998, 6.509024999999998, 6.833474999999996, 7.166024999999998, 7.5066749999999995, 7.8554249999999985, 8.212275, 8.577224999999997, 8.950274999999996, 9.331424999999998, 9.720674999999995, 10.118024999999994, 10.523474999999996, 10.937025, 11.358675, 11.788425000000002, 12.226275, 12.672225 ], [ 8.2025, 7.846550000000001, 7.4987, 7.15895, 6.827300000000001, 6.50375, 6.1883, 5.88095, 5.5817, 5.29055, 5.0075, 4.73255, 4.4657, 4.206950000000001, 3.9563, 3.7137500000000006, 3.4793000000000003, 3.2529500000000002, 3.0347000000000004, 2.8245500000000003, 2.6225000000000005, 2.4285500000000004, 2.2427, 2.0649500000000005, 1.8952999999999998, 1.7337500000000001, 1.5803000000000003, 1.4349500000000006, 1.2977000000000003, 1.1685500000000002, 1.0475000000000003, 0.9345500000000002, 0.8297000000000003, 0.7329500000000002, 0.6443000000000003, 0.5637500000000002, 0.4913000000000003, 0.42695000000000016, 0.37070000000000025, 0.32255000000000056, 0.2825000000000002, 0.25055000000000005, 0.22670000000000012, 0.21095000000000041, 0.20330000000000048, 0.20375000000000032, 0.21229999999999993, 0.2289500000000002, 0.25370000000000026, 0.2865500000000001, 0.32750000000000057, 0.37654999999999994, 0.4337, 0.4989499999999998, 0.5723000000000003, 0.6537500000000005, 0.7432999999999996, 0.8409499999999994, 0.9466999999999999, 1.0605499999999992, 1.1824999999999992, 1.3125499999999999, 1.4506999999999994, 1.5969499999999996, 1.7513000000000005, 1.9137500000000003, 2.084299999999999, 2.26295, 2.4497, 2.6445500000000006, 2.8475, 3.0585500000000003, 3.2776999999999994, 3.504949999999999, 3.7403000000000013, 3.9837500000000006, 4.2353000000000005, 4.494949999999999, 4.762699999999999, 5.038549999999999, 5.322499999999998, 5.614550000000001, 5.9147, 6.222949999999997, 6.539299999999997, 6.863749999999996, 7.196299999999997, 7.536949999999999, 7.885699999999998, 8.24255, 8.607499999999996, 8.980549999999996, 9.361699999999997, 9.750949999999994, 10.148299999999994, 10.553749999999996, 10.9673, 11.38895, 11.818700000000002, 12.256549999999999, 12.702499999999999 ], [ 8.235225, 7.879275, 7.5314250000000005, 7.191675, 6.860025, 6.536475000000001, 6.221025, 5.9136750000000005, 5.614425000000001, 5.323275000000001, 5.040225, 4.765275000000001, 4.498425, 4.239675, 3.9890250000000003, 3.7464750000000007, 3.5120250000000004, 3.2856750000000003, 3.0674250000000005, 2.8572750000000005, 2.6552250000000006, 2.4612750000000005, 2.2754250000000003, 2.0976750000000006, 1.9280249999999999, 1.7664750000000002, 1.6130250000000004, 1.4676750000000007, 1.3304250000000004, 1.2012750000000003, 1.0802250000000004, 0.9672750000000003, 0.8624250000000004, 0.7656750000000003, 0.6770250000000004, 0.5964750000000003, 0.5240250000000004, 0.4596750000000003, 0.40342500000000037, 0.3552750000000007, 0.3152250000000003, 0.28327500000000017, 0.25942500000000024, 0.24367500000000053, 0.2360249999999997, 0.23647500000000043, 0.24502500000000005, 0.2616750000000003, 0.2864250000000004, 0.3192750000000002, 0.3602249999999998, 0.40927500000000006, 0.4664250000000001, 0.5316749999999999, 0.6050249999999995, 0.6864749999999997, 0.7760250000000006, 0.8736750000000004, 0.9794249999999991, 1.0932750000000002, 1.2152250000000002, 1.3452750000000009, 1.4834250000000004, 1.6296750000000007, 1.7840249999999997, 1.9464749999999995, 2.117025, 2.2956749999999992, 2.482424999999999, 2.677275, 2.8802249999999994, 3.0912749999999996, 3.3104249999999986, 3.5376749999999983, 3.7730250000000005, 4.016475, 4.268025, 4.527674999999999, 4.795424999999998, 5.071274999999998, 5.355224999999997, 5.6472750000000005, 5.947424999999999, 6.255675, 6.572025, 6.896474999999999, 7.229025, 7.569675000000002, 7.918425000000001, 8.275275000000002, 8.640225, 9.013274999999998, 9.394425, 9.783674999999997, 10.181024999999996, 10.586474999999998, 11.000025000000003, 11.421675000000002, 11.851425000000004, 12.289275000000002, 12.735225000000002 ], [ 8.2704, 7.91445, 7.566600000000001, 7.226850000000001, 6.895200000000001, 6.57165, 6.2562, 5.94885, 5.6496, 5.35845, 5.0754, 4.8004500000000005, 4.5336, 4.274850000000001, 4.0242, 3.781650000000001, 3.5472000000000006, 3.3208500000000005, 3.1026000000000007, 2.8924500000000006, 2.690400000000001, 2.4964500000000007, 2.3106000000000004, 2.132850000000001, 1.9632, 1.8016500000000004, 1.6482000000000006, 1.502850000000001, 1.3656000000000006, 1.2364500000000005, 1.1154000000000006, 1.0024500000000005, 0.8976000000000006, 0.8008500000000005, 0.7122000000000002, 0.6316500000000005, 0.5592000000000006, 0.49485000000000046, 0.43860000000000054, 0.3904500000000004, 0.3504000000000005, 0.31845000000000034, 0.2946000000000004, 0.27885000000000026, 0.27120000000000033, 0.27165000000000106, 0.28020000000000067, 0.29685000000000095, 0.3216000000000001, 0.35444999999999993, 0.3954000000000004, 0.4444499999999998, 0.5015999999999998, 0.5668500000000005, 0.6402000000000001, 0.7216500000000003, 0.8112000000000004, 0.9088500000000002, 1.0145999999999997, 1.12845, 1.250399999999999, 1.3804500000000015, 1.518600000000001, 1.6648499999999995, 1.8192000000000004, 1.9816500000000001, 2.152199999999999, 2.33085, 2.5176, 2.7124500000000005, 2.9154, 3.12645, 3.3455999999999992, 3.572849999999999, 3.808200000000001, 4.05165, 4.3032, 4.562849999999999, 4.830599999999999, 5.106449999999999, 5.390399999999998, 5.682449999999999, 5.9826000000000015, 6.290849999999999, 6.607199999999999, 6.931649999999998, 7.264199999999999, 7.604850000000001, 7.9536, 8.310450000000001, 8.675399999999998, 9.048449999999997, 9.429599999999999, 9.818849999999996, 10.216199999999995, 10.621649999999997, 11.035200000000001, 11.456850000000001, 11.886600000000003, 12.32445, 12.7704 ], [ 8.308025, 7.952075, 7.6042250000000005, 7.264475, 6.932825, 6.609275000000001, 6.293825, 5.986475, 5.687225000000001, 5.396075000000001, 5.113025, 4.838075000000001, 4.571225, 4.312475, 4.061825, 3.8192750000000006, 3.5848250000000004, 3.3584750000000003, 3.1402250000000005, 2.9300750000000004, 2.7280250000000006, 2.5340750000000005, 2.3482250000000002, 2.1704750000000006, 2.000825, 1.8392750000000002, 1.6858250000000004, 1.5404750000000007, 1.4032250000000004, 1.2740750000000003, 1.1530250000000004, 1.0400750000000003, 0.9352250000000004, 0.8384750000000003, 0.7498250000000004, 0.6692750000000003, 0.5968250000000004, 0.5324750000000003, 0.47622500000000034, 0.42807500000000065, 0.3880250000000003, 0.35607500000000014, 0.3322250000000002, 0.3164750000000005, 0.3088249999999997, 0.3092750000000004, 0.317825, 0.3344750000000003, 0.35922500000000035, 0.3920750000000002, 0.43302499999999977, 0.48207500000000003, 0.5392250000000001, 0.6044749999999999, 0.6778249999999995, 0.7592749999999997, 0.8488250000000006, 0.9464750000000004, 1.052224999999999, 1.1660750000000002, 1.2880250000000002, 1.4180750000000009, 1.5562250000000004, 1.7024750000000006, 1.8568249999999997, 2.0192749999999995, 2.189825, 2.368474999999999, 2.555224999999999, 2.750075, 2.9530249999999993, 3.1640749999999995, 3.3832249999999986, 3.6104749999999983, 3.8458250000000005, 4.089275, 4.340825, 4.6004749999999985, 4.868224999999998, 5.144074999999998, 5.428024999999997, 5.720075000000002, 6.020225000000001, 6.328474999999998, 6.644824999999998, 6.969274999999997, 7.301824999999998, 7.642475, 7.991224999999999, 8.348075000000001, 8.713024999999998, 9.086074999999997, 9.467225, 9.856474999999996, 10.253824999999996, 10.659274999999997, 11.072825000000002, 11.494475000000001, 11.924225000000003, 12.362075, 12.808025 ], [ 8.348100000000002, 7.99215, 7.6443, 7.304550000000001, 6.972900000000001, 6.649350000000001, 6.333900000000001, 6.026550000000001, 5.7273000000000005, 5.4361500000000005, 5.153100000000001, 4.878150000000001, 4.611300000000001, 4.352550000000001, 4.1019000000000005, 3.859350000000001, 3.6249000000000007, 3.3985500000000006, 3.180300000000001, 2.9701500000000007, 2.768100000000001, 2.574150000000001, 2.3883000000000005, 2.210550000000001, 2.0409, 1.8793500000000005, 1.7259000000000007, 1.580550000000001, 1.4433000000000007, 1.3141500000000006, 1.1931000000000007, 1.0801500000000006, 0.9753000000000007, 0.8785500000000006, 0.7899000000000003, 0.7093500000000006, 0.6369000000000007, 0.5725500000000006, 0.5163000000000006, 0.4681500000000005, 0.4281000000000006, 0.39615000000000045, 0.37230000000000096, 0.35655000000000037, 0.34890000000000043, 0.34935000000000027, 0.3578999999999999, 0.37455000000000016, 0.3993000000000011, 0.4321500000000009, 0.4731000000000005, 0.5221500000000008, 0.5793000000000008, 0.6445499999999997, 0.7179000000000002, 0.7993500000000004, 0.8889000000000005, 0.9865500000000003, 1.0922999999999998, 1.20615, 1.3281, 1.4581500000000007, 1.5963000000000003, 1.7425500000000005, 1.8969000000000014, 2.059350000000001, 2.2298999999999998, 2.408550000000001, 2.595299999999999, 2.7901500000000015, 2.993099999999999, 3.2041499999999994, 3.4232999999999985, 3.65055, 3.885900000000002, 4.129350000000001, 4.3809, 4.64055, 4.9083, 5.184149999999998, 5.468099999999999, 5.76015, 6.060299999999999, 6.36855, 6.6849, 7.009349999999999, 7.3419, 7.682550000000002, 8.031300000000002, 8.388150000000003, 8.7531, 9.126149999999999, 9.5073, 9.896549999999998, 10.293899999999997, 10.699349999999999, 11.112900000000003, 11.534550000000003, 11.964300000000005, 12.402150000000002, 12.848100000000002 ], [ 8.390625, 8.034675, 7.686825000000002, 7.347075, 7.015425000000001, 6.6918750000000005, 6.376425, 6.069075000000001, 5.769825, 5.478675, 5.195625000000001, 4.920675, 4.653825, 4.395075000000001, 4.144425000000001, 3.9018750000000013, 3.6674250000000006, 3.4410750000000005, 3.2228250000000007, 3.0126750000000007, 2.810625000000001, 2.6166750000000008, 2.4308250000000005, 2.253075000000001, 2.083425, 1.9218750000000004, 1.7684250000000006, 1.623075000000001, 1.4858250000000006, 1.3566750000000005, 1.2356250000000006, 1.1226750000000005, 1.0178250000000006, 0.9210750000000005, 0.8324250000000006, 0.7518750000000005, 0.6794250000000006, 0.6150750000000005, 0.5588250000000006, 0.5106750000000009, 0.4706250000000005, 0.4386749999999999, 0.41482500000000044, 0.39907500000000073, 0.3914250000000008, 0.39187500000000064, 0.40042500000000025, 0.41707500000000053, 0.4418250000000006, 0.4746750000000004, 0.5156250000000009, 0.5646750000000003, 0.6218250000000003, 0.6870750000000001, 0.7604250000000006, 0.8418750000000008, 0.931425, 1.0290749999999997, 1.1348250000000002, 1.2486749999999995, 1.3706249999999995, 1.5006750000000002, 1.6388249999999998, 1.785075, 1.9394250000000008, 2.1018750000000006, 2.2724249999999993, 2.4510750000000003, 2.6378250000000003, 2.832675000000001, 3.0356250000000005, 3.2466750000000006, 3.4658249999999997, 3.6930749999999994, 3.9284250000000016, 4.171875000000001, 4.423425000000001, 4.683075, 4.950824999999999, 5.226674999999999, 5.5106249999999966, 5.802675000000002, 6.102825, 6.411074999999998, 6.7274249999999975, 7.051874999999996, 7.384424999999998, 7.7250749999999995, 8.073825, 8.430675, 8.795624999999998, 9.168674999999997, 9.549824999999998, 9.939074999999995, 10.336424999999995, 10.741874999999997, 11.155425000000001, 11.577075, 12.006825000000003, 12.444675, 12.890625 ], [ 8.4356, 8.079649999999999, 7.7318, 7.39205, 7.0604000000000005, 6.7368500000000004, 6.4214, 6.114050000000001, 5.8148, 5.52365, 5.240600000000001, 4.96565, 4.6988, 4.44005, 4.1894, 3.9468500000000004, 3.7124000000000006, 3.48605, 3.2678000000000003, 3.05765, 2.8556000000000004, 2.6616500000000003, 2.4758, 2.2980500000000004, 2.1283999999999996, 1.96685, 1.8134000000000001, 1.6680500000000005, 1.5308000000000002, 1.40165, 1.2806000000000002, 1.16765, 1.0628000000000002, 0.9660500000000001, 0.8773999999999997, 0.7968500000000001, 0.7244000000000002, 0.66005, 0.6038000000000001, 0.55565, 0.5156000000000001, 0.4836499999999999, 0.45980000000000043, 0.44404999999999983, 0.4363999999999999, 0.43684999999999974, 0.44539999999999935, 0.4620499999999996, 0.48680000000000057, 0.5196500000000004, 0.5606, 0.6096500000000002, 0.6668000000000003, 0.7320499999999992, 0.8053999999999997, 0.8868499999999999, 0.9763999999999999, 1.0740499999999997, 1.1797999999999993, 1.2936499999999995, 1.4155999999999995, 1.5456500000000002, 1.6837999999999997, 1.83005, 1.9844000000000008, 2.1468500000000006, 2.3173999999999992, 2.4960500000000003, 2.6827999999999985, 2.877650000000001, 3.0805999999999987, 3.291649999999999, 3.510799999999998, 3.7380499999999994, 3.9734000000000016, 4.216850000000001, 4.468399999999999, 4.72805, 4.995799999999999, 5.2716499999999975, 5.555599999999998, 5.84765, 6.147799999999998, 6.456049999999999, 6.772399999999999, 7.096849999999998, 7.429399999999999, 7.770050000000001, 8.1188, 8.475650000000002, 8.840599999999998, 9.213649999999998, 9.5948, 9.984049999999996, 10.381399999999996, 10.786849999999998, 11.200400000000002, 11.622050000000002, 12.051800000000004, 12.489650000000001, 12.9356 ], [ 8.483025000000001, 8.127075, 7.779225, 7.439475000000001, 7.107825000000001, 6.784275000000001, 6.468825000000001, 6.161475000000001, 5.8622250000000005, 5.571075, 5.288025000000001, 5.013075000000001, 4.746225000000001, 4.487475000000001, 4.2368250000000005, 3.994275000000001, 3.759825000000001, 3.5334750000000006, 3.3152250000000008, 3.1050750000000007, 2.903025000000001, 2.709075000000001, 2.5232250000000005, 2.345475000000001, 2.175825, 2.0142750000000005, 1.8608250000000006, 1.715475000000001, 1.5782250000000007, 1.4490750000000006, 1.3280250000000007, 1.2150750000000006, 1.1102250000000007, 1.0134750000000006, 0.9248250000000002, 0.8442750000000006, 0.7718250000000006, 0.7074750000000005, 0.6512250000000006, 0.6030750000000005, 0.5630250000000006, 0.5310750000000004, 0.5072250000000009, 0.49147500000000033, 0.4838250000000004, 0.48427500000000023, 0.49282499999999985, 0.5094750000000001, 0.5342250000000011, 0.5670750000000009, 0.6080250000000005, 0.6570750000000007, 0.7142250000000008, 0.7794749999999997, 0.8528250000000002, 0.9342750000000004, 1.0238250000000004, 1.1214750000000002, 1.227224999999999, 1.341075, 1.463025, 1.5930750000000007, 1.7312250000000002, 1.8774750000000004, 2.0318250000000013, 2.194275000000001, 2.3648249999999997, 2.543475000000001, 2.730224999999999, 2.9250750000000014, 3.128024999999999, 3.3390749999999993, 3.5582249999999984, 3.785475, 4.020825000000002, 4.264275000000001, 4.5158249999999995, 4.775475, 5.043225, 5.319074999999998, 5.603024999999999, 5.895075, 6.195224999999999, 6.503475, 6.819825, 7.144274999999999, 7.476825, 7.817475000000002, 8.166225, 8.523075000000002, 8.888024999999999, 9.261074999999998, 9.642225, 10.031474999999997, 10.428824999999996, 10.834274999999998, 11.247825000000002, 11.669475000000002, 12.099225000000004, 12.537075000000002, 12.983025000000001 ], [ 8.532900000000001, 8.17695, 7.8291, 7.489350000000001, 7.157700000000001, 6.834150000000001, 6.518700000000001, 6.211350000000001, 5.912100000000001, 5.620950000000001, 5.337900000000001, 5.062950000000001, 4.796100000000001, 4.537350000000001, 4.286700000000001, 4.044150000000001, 3.809700000000001, 3.583350000000001, 3.365100000000001, 3.154950000000001, 2.952900000000001, 2.758950000000001, 2.5731000000000006, 2.395350000000001, 2.2257000000000002, 2.0641500000000006, 1.9107000000000007, 1.765350000000001, 1.6281000000000008, 1.4989500000000007, 1.3779000000000008, 1.2649500000000007, 1.1601000000000008, 1.0633500000000007, 0.9747000000000003, 0.8941500000000007, 0.8217000000000008, 0.7573500000000006, 0.7011000000000003, 0.6529500000000006, 0.6129000000000007, 0.5809500000000005, 0.557100000000001, 0.5413500000000004, 0.5337000000000005, 0.5341500000000003, 0.5427, 0.5593500000000002, 0.5841000000000012, 0.616950000000001, 0.6579000000000006, 0.7069500000000009, 0.7641000000000009, 0.8293499999999998, 0.9027000000000003, 0.9841500000000005, 1.0737000000000005, 1.1713500000000003, 1.277099999999999, 1.3909500000000001, 1.5129000000000001, 1.6429500000000008, 1.7811000000000003, 1.9273500000000006, 2.0817000000000014, 2.244150000000001, 2.4147, 2.593350000000001, 2.780099999999999, 2.9749500000000015, 3.1778999999999993, 3.3889499999999995, 3.6080999999999985, 3.83535, 4.070700000000002, 4.3141500000000015, 4.5657, 4.82535, 5.0931, 5.368949999999998, 5.652899999999999, 5.94495, 6.245099999999999, 6.55335, 6.8697, 7.194149999999999, 7.5267, 7.867350000000002, 8.2161, 8.572950000000002, 8.937899999999999, 9.310949999999998, 9.6921, 10.081349999999997, 10.478699999999996, 10.884149999999998, 11.297700000000003, 11.719350000000002, 12.149100000000004, 12.586950000000002, 13.032900000000001 ], [ 8.585225000000001, 8.229275000000001, 7.881425000000001, 7.541675000000001, 7.210025000000001, 6.886475, 6.571025, 6.263675000000001, 5.964425, 5.673275, 5.390225, 5.1152750000000005, 4.848425, 4.589675000000001, 4.339025, 4.096475000000001, 3.862025, 3.635675, 3.4174250000000006, 3.2072750000000005, 3.0052250000000007, 2.8112750000000006, 2.6254250000000003, 2.4476750000000007, 2.278025, 2.1164750000000003, 1.9630250000000005, 1.8176750000000008, 1.6804250000000005, 1.5512750000000004, 1.4302250000000005, 1.3172750000000004, 1.2124250000000005, 1.1156750000000004, 1.027025, 0.9464750000000004, 0.8740250000000005, 0.8096750000000004, 0.7534250000000009, 0.7052750000000003, 0.6652250000000004, 0.6332750000000003, 0.6094249999999999, 0.5936750000000002, 0.5860250000000002, 0.586475000000001, 0.5950250000000006, 0.6116750000000009, 0.636425, 0.6692749999999998, 0.7102250000000003, 0.7592749999999997, 0.8164249999999997, 0.8816750000000004, 0.955025, 1.0364750000000003, 1.1260250000000003, 1.2236749999999992, 1.3294249999999996, 1.443274999999999, 1.5652250000000008, 1.6952749999999996, 1.8334249999999992, 1.9796750000000012, 2.1340250000000003, 2.296475, 2.4670250000000005, 2.6456749999999998, 2.8324249999999997, 3.0272750000000004, 3.230225, 3.441275, 3.660424999999999, 3.887674999999999, 4.123025000000001, 4.366475, 4.618025, 4.877674999999999, 5.145424999999999, 5.421274999999999, 5.7052249999999995, 5.997275000000001, 6.297425, 6.605674999999997, 6.922024999999997, 7.246474999999996, 7.579025000000001, 7.9196750000000025, 8.268424999999997, 8.625274999999998, 8.990224999999999, 9.363274999999998, 9.744424999999996, 10.133674999999993, 10.531024999999996, 10.936474999999998, 11.350025000000002, 11.771675000000002, 12.201425, 12.639274999999998, 13.085225000000001 ], [ 8.64, 8.28405, 7.9362, 7.596450000000002, 7.264800000000001, 6.941250000000001, 6.625800000000001, 6.318450000000001, 6.0192000000000005, 5.7280500000000005, 5.445000000000001, 5.170050000000001, 4.903200000000001, 4.644450000000001, 4.393800000000001, 4.151250000000001, 3.916800000000001, 3.690450000000001, 3.472200000000001, 3.262050000000001, 3.060000000000001, 2.866050000000001, 2.6802000000000006, 2.502450000000001, 2.3328, 2.1712500000000006, 2.0178000000000007, 1.872450000000001, 1.7352000000000007, 1.6060500000000006, 1.4850000000000008, 1.3720500000000007, 1.2672000000000008, 1.1704500000000007, 1.0818000000000003, 1.0012500000000006, 0.9288000000000007, 0.8644500000000006, 0.8082000000000003, 0.7600500000000006, 0.7200000000000006, 0.6880500000000005, 0.664200000000001, 0.6484500000000004, 0.6408000000000005, 0.6412500000000003, 0.6497999999999999, 0.6664500000000002, 0.6912000000000011, 0.724050000000001, 0.7650000000000006, 0.8140500000000008, 0.8712000000000009, 0.9364499999999998, 1.0098000000000003, 1.0912500000000005, 1.1808000000000005, 1.2784500000000003, 1.384199999999999, 1.49805, 1.62, 1.7500500000000008, 1.8882000000000003, 2.0344500000000005, 2.1888000000000014, 2.351250000000001, 2.5218, 2.700450000000001, 2.887199999999999, 3.0820500000000015, 3.2849999999999993, 3.4960499999999994, 3.7151999999999985, 3.94245, 4.177800000000002, 4.4212500000000015, 4.6728, 4.93245, 5.2002, 5.476049999999998, 5.759999999999999, 6.05205, 6.352199999999999, 6.66045, 6.9768, 7.301249999999999, 7.6338, 7.974450000000002, 8.3232, 8.680050000000001, 9.044999999999998, 9.418049999999997, 9.799199999999999, 10.188449999999996, 10.585799999999995, 10.991249999999997, 11.404800000000002, 11.826450000000001, 12.256200000000003, 12.69405, 13.14 ], [ 8.697225, 8.341275, 7.993425000000001, 7.653675000000001, 7.322025000000002, 6.998475000000002, 6.683025000000001, 6.375675000000001, 6.076425000000001, 5.785275, 5.502225000000001, 5.2272750000000014, 4.960425000000001, 4.701675000000001, 4.4510250000000005, 4.208475000000001, 3.974025000000001, 3.747675000000001, 3.5294250000000007, 3.319275000000001, 3.1172250000000012, 2.923275000000001, 2.737425000000001, 2.5596750000000013, 2.3900250000000005, 2.228475000000001, 2.075025000000001, 1.9296750000000014, 1.792425000000001, 1.663275000000001, 1.542225000000001, 1.429275000000001, 1.324425000000001, 1.227675000000001, 1.139025000000001, 1.0584750000000014, 0.9860250000000015, 0.9216750000000005, 0.865425000000001, 0.8172750000000013, 0.7772250000000005, 0.7452750000000012, 0.7214250000000009, 0.7056750000000012, 0.6980250000000003, 0.6984750000000011, 0.7070250000000007, 0.723675000000001, 0.748425000000001, 0.7812750000000008, 0.8222250000000004, 0.8712750000000007, 0.9284250000000007, 0.9936750000000005, 1.0670250000000001, 1.1484750000000004, 1.2380250000000013, 1.335675000000001, 1.4414249999999997, 1.5552750000000009, 1.6772250000000009, 1.8072750000000015, 1.945425000000001, 2.0916750000000013, 2.2460250000000004, 2.408475, 2.5790250000000006, 2.757675, 2.944425, 3.1392750000000005, 3.342225, 3.553275, 3.7724249999999993, 3.999674999999999, 4.235025000000001, 4.478475, 4.730025, 4.989674999999999, 5.257424999999999, 5.533275000000001, 5.817224999999998, 6.109275000000003, 6.4094250000000015, 6.717674999999999, 7.034024999999999, 7.358474999999998, 7.691024999999999, 8.031675, 8.380424999999999, 8.737275, 9.102224999999997, 9.475274999999996, 9.856424999999998, 10.245674999999995, 10.643024999999994, 11.048474999999996, 11.462025, 11.883675, 12.313425000000002, 12.751275, 13.197225 ], [ 8.7569, 8.40095, 8.0531, 7.71335, 7.3817, 7.058150000000001, 6.7427, 6.435350000000001, 6.136100000000001, 5.84495, 5.5619000000000005, 5.286950000000001, 5.0201, 4.76135, 4.5107, 4.26815, 4.0337000000000005, 3.8073500000000005, 3.5891, 3.3789500000000006, 3.1769000000000007, 2.9829500000000007, 2.7971000000000004, 2.6193500000000007, 2.4497, 2.2881500000000004, 2.1347000000000005, 1.9893500000000008, 1.8521000000000005, 1.7229500000000004, 1.6019000000000005, 1.4889500000000004, 1.3841000000000006, 1.2873500000000004, 1.1987000000000005, 1.1181500000000009, 1.045700000000001, 0.98135, 0.9251000000000005, 0.8769500000000008, 0.8369, 0.8049500000000007, 0.7811000000000003, 0.7653500000000006, 0.7576999999999998, 0.7581500000000005, 0.7667000000000002, 0.7833500000000004, 0.8081000000000005, 0.8409500000000003, 0.8818999999999999, 0.9309500000000002, 0.9881000000000002, 1.05335, 1.1266999999999996, 1.2081499999999998, 1.2977000000000007, 1.3953500000000005, 1.5010999999999992, 1.6149500000000003, 1.7369000000000003, 1.866950000000001, 2.0051000000000005, 2.1513500000000008, 2.3057, 2.4681499999999996, 2.6387, 2.8173499999999994, 3.0040999999999993, 3.19895, 3.4018999999999995, 3.6129499999999997, 3.8320999999999987, 4.0593499999999985, 4.294700000000001, 4.53815, 4.7897, 5.049349999999999, 5.317099999999998, 5.59295, 5.876899999999997, 6.168950000000002, 6.469100000000001, 6.777349999999998, 7.093699999999998, 7.418149999999997, 7.750699999999998, 8.09135, 8.4401, 8.79695, 9.161899999999997, 9.534949999999997, 9.916099999999998, 10.305349999999995, 10.702699999999995, 11.108149999999997, 11.521700000000001, 11.94335, 12.373100000000003, 12.81095, 13.2569 ], [ 8.819025, 8.463075, 8.115225, 7.775475, 7.443825000000002, 7.120275000000001, 6.804825, 6.4974750000000006, 6.198225000000001, 5.907075, 5.6240250000000005, 5.349075000000001, 5.082225, 4.823475, 4.572825, 4.330275, 4.0958250000000005, 3.8694750000000004, 3.651225, 3.4410750000000005, 3.2390250000000007, 3.0450750000000006, 2.8592250000000003, 2.6814750000000007, 2.511825, 2.3502750000000003, 2.1968250000000005, 2.051475000000001, 1.9142250000000005, 1.7850750000000004, 1.6640250000000005, 1.5510750000000004, 1.446225, 1.3494750000000009, 1.2608250000000005, 1.1802750000000009, 1.107825000000001, 1.043475, 0.9872250000000005, 0.9390750000000008, 0.899025, 0.8670750000000007, 0.8432250000000003, 0.8274750000000006, 0.8198249999999998, 0.8202750000000005, 0.8288250000000001, 0.8454750000000004, 0.8702250000000005, 0.9030750000000003, 0.9440249999999999, 0.9930750000000002, 1.0502250000000002, 1.115475, 1.1888249999999996, 1.2702749999999998, 1.3598250000000007, 1.4574750000000005, 1.5632249999999992, 1.6770750000000003, 1.7990250000000003, 1.929075000000001, 2.0672250000000005, 2.2134750000000007, 2.367825, 2.5302749999999996, 2.700825, 2.8794749999999993, 3.0662249999999993, 3.261075, 3.4640249999999995, 3.6750749999999996, 3.8942249999999987, 4.1214749999999984, 4.356825000000001, 4.600275, 4.851825, 5.111474999999999, 5.379225, 5.655074999999998, 5.939024999999999, 6.231075000000001, 6.531224999999999, 6.839475, 7.155825, 7.480274999999999, 7.812825, 8.153475000000002, 8.502225000000001, 8.859075000000002, 9.224025, 9.597074999999998, 9.978225, 10.367474999999997, 10.764824999999997, 11.170274999999998, 11.583825000000003, 12.005475000000002, 12.435225000000004, 12.873075000000002, 13.319025000000002 ], [ 8.883600000000001, 8.527650000000001, 8.179800000000002, 7.84005, 7.5084, 7.184850000000001, 6.869400000000001, 6.562050000000001, 6.2628, 5.97165, 5.688600000000001, 5.4136500000000005, 5.146800000000001, 4.888050000000001, 4.637400000000001, 4.394850000000002, 4.160400000000001, 3.934050000000001, 3.7158000000000015, 3.505650000000001, 3.303600000000001, 3.109650000000001, 2.923800000000001, 2.746050000000001, 2.5764000000000005, 2.414850000000001, 2.261400000000001, 2.1160500000000013, 1.978800000000001, 1.849650000000001, 1.728600000000001, 1.6156500000000005, 1.5108000000000006, 1.4140500000000014, 1.325400000000001, 1.2448500000000005, 1.1724000000000006, 1.1080500000000004, 1.051800000000001, 1.0036500000000013, 0.9636000000000013, 0.9316500000000003, 0.9078000000000008, 0.8920500000000011, 0.8844000000000012, 0.884850000000001, 0.8934000000000006, 0.9100500000000009, 0.934800000000001, 0.9676500000000008, 1.0086000000000013, 1.0576500000000006, 1.1148000000000007, 1.1800500000000005, 1.253400000000001, 1.3348500000000012, 1.4244000000000003, 1.5220500000000001, 1.6278000000000006, 1.74165, 1.8636, 1.9936500000000006, 2.1318, 2.2780500000000004, 2.4324000000000012, 2.594850000000001, 2.7653999999999996, 2.9440500000000007, 3.1308000000000007, 3.3256500000000013, 3.528600000000001, 3.739650000000001, 3.9588, 4.18605, 4.421400000000002, 4.664850000000001, 4.916400000000001, 5.17605, 5.443799999999998, 5.71965, 6.003599999999997, 6.295650000000002, 6.5958000000000006, 6.904049999999998, 7.220399999999998, 7.544849999999997, 7.877399999999998, 8.21805, 8.566799999999999, 8.92365, 9.288599999999997, 9.661649999999996, 10.042799999999998, 10.432049999999995, 10.829399999999994, 11.234849999999996, 11.6484, 12.07005, 12.499800000000002, 12.93765, 13.3836 ], [ 8.950624999999999, 8.594674999999999, 8.246825000000001, 7.907075, 7.575425000000002, 7.251875000000001, 6.936425, 6.629075, 6.3298250000000005, 6.038675, 5.755625, 5.480675000000001, 5.213825, 4.955075, 4.704425, 4.461875, 4.227425, 4.001074999999999, 3.782825, 3.5726750000000003, 3.3706250000000004, 3.1766750000000004, 2.990825, 2.8130750000000004, 2.6434249999999997, 2.481875, 2.328425, 2.1830750000000005, 2.045825, 1.9166750000000006, 1.7956250000000002, 1.6826749999999997, 1.5778249999999998, 1.4810750000000006, 1.3924250000000002, 1.3118750000000006, 1.2394250000000007, 1.1750749999999996, 1.1188250000000002, 1.0706750000000005, 1.0306249999999997, 0.9986750000000004, 0.974825, 0.9590750000000003, 0.9514249999999995, 0.9518750000000002, 0.9604249999999999, 0.9770750000000001, 1.0018250000000002, 1.034675, 1.0756249999999996, 1.1246749999999999, 1.181825, 1.2470749999999997, 1.3204249999999993, 1.4018749999999995, 1.4914250000000004, 1.5890750000000002, 1.694824999999999, 1.808675, 1.930625, 2.0606750000000007, 2.1988250000000003, 2.3450750000000005, 2.4994249999999996, 2.6618749999999993, 2.8324249999999997, 3.011074999999999, 3.197824999999999, 3.3926749999999997, 3.595624999999999, 3.8066749999999994, 4.025824999999998, 4.253074999999998, 4.488425, 4.731875, 4.9834249999999995, 5.243075, 5.510825, 5.786674999999998, 6.070624999999999, 6.362675, 6.662824999999999, 6.971075, 7.287425, 7.611874999999999, 7.944425, 8.285075000000003, 8.633825000000002, 8.990675000000003, 9.355625, 9.728674999999999, 10.109825, 10.499074999999998, 10.896424999999997, 11.301874999999999, 11.715425000000003, 12.137075000000003, 12.566825000000005, 13.004675000000002, 13.450625000000002 ], [ 9.0201, 8.66415, 8.316300000000002, 7.9765500000000005, 7.644900000000001, 7.32135, 7.0059, 6.698550000000001, 6.399300000000001, 6.10815, 5.825100000000001, 5.5501499999999995, 5.2833, 5.0245500000000005, 4.7739, 4.531350000000001, 4.2969, 4.07055, 3.8523000000000005, 3.64215, 3.440100000000001, 3.246150000000001, 3.0603000000000007, 2.88255, 2.7129000000000003, 2.5513500000000002, 2.397900000000001, 2.2525500000000003, 2.1153000000000004, 1.9861500000000003, 1.8651000000000009, 1.7521500000000003, 1.6473000000000004, 1.5505500000000003, 1.4619, 1.3813500000000003, 1.3089000000000004, 1.2445500000000003, 1.1883000000000008, 1.1401500000000002, 1.1001000000000003, 1.0681500000000002, 1.0442999999999998, 1.02855, 1.0209000000000001, 1.0213500000000009, 1.0299000000000005, 1.0465500000000008, 1.0713, 1.1041499999999997, 1.1451000000000002, 1.1941499999999996, 1.2512999999999996, 1.3165500000000003, 1.3899, 1.4713500000000002, 1.560900000000001, 1.658549999999999, 1.7642999999999995, 1.8781499999999989, 2.0001000000000007, 2.1301499999999995, 2.268299999999999, 2.414550000000001, 2.5689, 2.73135, 2.9019000000000004, 3.0805499999999997, 3.2672999999999996, 3.4621500000000003, 3.6651, 3.87615, 4.095299999999999, 4.322549999999999, 4.557900000000001, 4.80135, 5.0529, 5.312549999999997, 5.580299999999997, 5.856149999999999, 6.1400999999999994, 6.432150000000001, 6.7322999999999995, 7.040549999999997, 7.356899999999997, 7.681349999999996, 8.0139, 8.354550000000003, 8.703299999999999, 9.06015, 9.4251, 9.79815, 10.179299999999998, 10.568549999999995, 10.965899999999998, 11.37135, 11.784900000000004, 12.206550000000004, 12.636300000000002, 13.07415, 13.520100000000003 ], [ 9.092025, 8.736075, 8.388225000000002, 8.048475, 7.716825000000001, 7.393275, 7.077825, 6.770475000000001, 6.471225000000001, 6.180075, 5.897025000000001, 5.622075, 5.355225, 5.096475000000001, 4.8458250000000005, 4.603275000000001, 4.368825, 4.142475, 3.9242250000000007, 3.7140750000000002, 3.5120250000000013, 3.318075000000001, 3.132225000000001, 2.9544750000000004, 2.7848250000000005, 2.6232750000000005, 2.469825000000001, 2.3244750000000005, 2.1872250000000006, 2.0580750000000005, 1.937025000000001, 1.8240750000000006, 1.7192250000000007, 1.6224750000000006, 1.5338250000000002, 1.4532750000000005, 1.3808250000000006, 1.3164750000000005, 1.260225000000001, 1.2120750000000005, 1.1720250000000005, 1.1400750000000004, 1.116225, 1.1004750000000003, 1.0928250000000004, 1.093275000000001, 1.1018250000000007, 1.118475000000001, 1.1432250000000002, 1.176075, 1.2170250000000005, 1.2660749999999998, 1.3232249999999999, 1.3884750000000006, 1.4618250000000002, 1.5432750000000004, 1.6328250000000013, 1.7304749999999993, 1.8362249999999998, 1.9500749999999991, 2.072025000000001, 2.202075, 2.3402249999999993, 2.4864750000000013, 2.6408250000000004, 2.803275, 2.9738250000000006, 3.152475, 3.339225, 3.5340750000000005, 3.737025, 3.9480750000000002, 4.167224999999999, 4.394474999999999, 4.629825000000001, 4.8732750000000005, 5.124825, 5.3844749999999975, 5.652224999999997, 5.928074999999999, 6.212025, 6.504075000000001, 6.804225, 7.112474999999997, 7.428824999999997, 7.753274999999996, 8.085825, 8.426475000000003, 8.775224999999999, 9.132075, 9.497025, 9.870075, 10.251224999999998, 10.640474999999995, 11.037824999999998, 11.443275, 11.856825000000004, 12.278475000000004, 12.708225000000002, 13.146075, 13.592025000000003 ], [ 9.1664, 8.81045, 8.4626, 8.12285, 7.7912, 7.467650000000001, 7.152200000000001, 6.844850000000001, 6.5456, 6.25445, 5.971400000000001, 5.6964500000000005, 5.4296, 5.170850000000001, 4.9201999999999995, 4.67765, 4.4432, 4.216850000000001, 3.9985999999999997, 3.78845, 3.586400000000001, 3.39245, 3.206600000000001, 3.0288500000000003, 2.8592000000000004, 2.6976500000000003, 2.544200000000001, 2.3988500000000004, 2.2616000000000005, 2.1324500000000004, 2.011400000000001, 1.8984500000000004, 1.7936000000000005, 1.6968500000000004, 1.6082, 1.5276500000000004, 1.4552000000000005, 1.3908500000000004, 1.3346, 1.2864500000000003, 1.2464000000000004, 1.2144500000000003, 1.1906000000000008, 1.1748500000000002, 1.1672000000000002, 1.16765, 1.1761999999999997, 1.19285, 1.217600000000001, 1.2504500000000007, 1.2914000000000003, 1.3404500000000006, 1.3976000000000006, 1.4628499999999995, 1.536200000000001, 1.6176499999999994, 1.7072000000000003, 1.80485, 1.9106000000000005, 2.02445, 2.1464, 2.2764500000000005, 2.4146, 2.5608500000000003, 2.7151999999999994, 2.877649999999999, 3.0481999999999996, 3.226849999999999, 3.4136000000000006, 3.6084499999999995, 3.811400000000001, 4.022450000000001, 4.2416, 4.468849999999998, 4.7042, 4.947649999999999, 5.199200000000001, 5.458849999999998, 5.726599999999998, 6.00245, 6.286399999999997, 6.578450000000002, 6.8786000000000005, 7.186849999999998, 7.503199999999998, 7.827649999999997, 8.160199999999998, 8.50085, 8.849599999999999, 9.20645, 9.571399999999997, 9.944449999999996, 10.325599999999998, 10.714849999999995, 11.112199999999994, 11.517649999999996, 11.9312, 12.35285, 12.782600000000002, 13.22045, 13.6664 ], [ 9.243225, 8.887275, 8.539425000000001, 8.199675000000001, 7.868025000000001, 7.544475, 7.229025, 6.9216750000000005, 6.622425000000002, 6.331275000000001, 6.048225000000001, 5.773275, 5.506425, 5.247675000000001, 4.997025000000001, 4.754475000000001, 4.520025, 4.293675, 4.075425000000001, 3.8652750000000005, 3.6632250000000015, 3.4692750000000014, 3.283425000000001, 3.1056750000000006, 2.9360250000000008, 2.7744750000000007, 2.6210250000000013, 2.4756750000000007, 2.338425000000001, 2.2092750000000008, 2.0882250000000013, 1.9752750000000008, 1.870425000000001, 1.7736750000000008, 1.6850250000000004, 1.6044750000000008, 1.5320250000000009, 1.4676750000000007, 1.4114250000000013, 1.3632750000000007, 1.3232250000000008, 1.2912750000000006, 1.2674250000000002, 1.2516750000000005, 1.2440250000000006, 1.2444750000000013, 1.253025000000001, 1.2696750000000012, 1.2944250000000004, 1.3272750000000002, 1.3682250000000007, 1.417275, 1.474425, 1.5396750000000008, 1.6130250000000004, 1.6944750000000006, 1.7840250000000015, 1.8816749999999995, 1.987425, 2.1012749999999993, 2.223225000000001, 2.353275, 2.4914249999999996, 2.6376750000000015, 2.7920250000000006, 2.9544750000000004, 3.125025000000001, 3.303675, 3.490425, 3.6852750000000007, 3.8882250000000003, 4.0992750000000004, 4.3184249999999995, 4.545674999999999, 4.781025000000001, 5.024475000000001, 5.276025000000001, 5.535675000000001, 5.803425000000001, 6.079274999999999, 6.363224999999996, 6.655275000000001, 6.955425, 7.263675000000001, 7.580025000000001, 7.904475, 8.237024999999997, 8.577675, 8.926425000000002, 9.283275000000003, 9.648224999999996, 10.021274999999996, 10.402425000000001, 10.791674999999998, 11.189024999999994, 11.594474999999996, 12.008025, 12.429675, 12.859425000000005, 13.297275000000003, 13.743224999999999 ], [ 9.322500000000002, 8.96655, 8.6187, 8.278950000000002, 7.947300000000002, 7.623750000000001, 7.308300000000001, 7.000950000000001, 6.701700000000001, 6.410550000000001, 6.127500000000001, 5.852550000000001, 5.5857, 5.326950000000001, 5.0763, 4.83375, 4.5993, 4.372950000000001, 4.1547, 3.9445500000000004, 3.7425000000000015, 3.5485500000000005, 3.362700000000001, 3.1849500000000006, 3.0153000000000008, 2.8537500000000007, 2.7003000000000013, 2.5549500000000007, 2.417700000000001, 2.2885500000000008, 2.1675000000000013, 2.0545500000000008, 1.9497000000000009, 1.8529500000000008, 1.7643000000000004, 1.6837500000000007, 1.6113000000000008, 1.5469500000000007, 1.4907000000000004, 1.4425500000000007, 1.4025000000000007, 1.3705500000000006, 1.3467000000000011, 1.3309500000000005, 1.3233000000000006, 1.3237500000000004, 1.3323, 1.3489500000000003, 1.3737000000000013, 1.406550000000001, 1.4475000000000007, 1.496550000000001, 1.553700000000001, 1.61895, 1.6922999999999995, 1.7737500000000015, 1.8633000000000006, 1.9609500000000004, 2.066699999999999, 2.18055, 2.3025, 2.432550000000001, 2.5707000000000004, 2.7169500000000006, 2.8713000000000015, 3.0337500000000013, 3.2043, 3.382950000000001, 3.569699999999999, 3.7645500000000016, 3.9674999999999994, 4.1785499999999995, 4.397699999999999, 4.62495, 4.860300000000002, 5.103750000000002, 5.3553, 5.61495, 5.8827, 6.158549999999998, 6.442499999999999, 6.7345500000000005, 7.034699999999999, 7.34295, 7.6593, 7.983749999999999, 8.3163, 8.656950000000002, 9.005700000000001, 9.362550000000002, 9.7275, 10.100549999999998, 10.4817, 10.870949999999997, 11.268299999999996, 11.673749999999998, 12.087300000000003, 12.508950000000002, 12.938700000000004, 13.376550000000002, 13.822500000000002 ], [ 9.404225, 9.048275, 8.700425000000001, 8.360675, 8.029025, 7.705475000000002, 7.390025000000001, 7.082675000000002, 6.783425000000001, 6.492275000000001, 6.209225000000002, 5.934275000000001, 5.667425000000001, 5.4086750000000015, 5.158025, 4.915475000000001, 4.681025000000001, 4.454675000000002, 4.2364250000000006, 4.026275000000001, 3.824225000000002, 3.630275000000001, 3.4444250000000016, 3.266675000000001, 3.0970250000000012, 2.935475000000001, 2.7820250000000017, 2.636675000000001, 2.4994250000000013, 2.3702750000000012, 2.249225000000002, 2.1362750000000013, 2.0314250000000014, 1.9346750000000013, 1.846025000000001, 1.7654750000000012, 1.6930250000000013, 1.6286750000000012, 1.5724250000000008, 1.5242750000000012, 1.4842250000000012, 1.452275000000001, 1.4284250000000016, 1.412675000000001, 1.405025000000001, 1.405475000000001, 1.4140250000000005, 1.4306750000000008, 1.4554250000000017, 1.4882750000000016, 1.5292250000000012, 1.5782750000000014, 1.6354250000000015, 1.7006750000000004, 1.7740250000000017, 1.8554750000000002, 1.9450250000000011, 2.042675000000001, 2.1484250000000014, 2.2622750000000007, 2.3842250000000007, 2.5142750000000014, 2.652425000000001, 2.798675000000001, 2.9530250000000002, 3.115475, 3.2860250000000004, 3.4646749999999997, 3.6514250000000015, 3.8462750000000003, 4.049225000000002, 4.260275000000002, 4.479425000000001, 4.706674999999999, 4.942025000000001, 5.185475, 5.437025000000002, 5.696674999999999, 5.9644249999999985, 6.2402750000000005, 6.524224999999998, 6.816275000000003, 7.116425000000001, 7.424674999999999, 7.741024999999999, 8.065474999999998, 8.398024999999999, 8.738675, 9.087425, 9.444275000000001, 9.809224999999998, 10.182274999999997, 10.563424999999999, 10.952674999999996, 11.350024999999995, 11.755474999999997, 12.169025000000001, 12.590675000000001, 13.020425000000003, 13.458275, 13.904225 ], [ 9.488399999999999, 9.132449999999999, 8.784600000000001, 8.444849999999999, 8.113199999999999, 7.789650000000001, 7.474200000000001, 7.166850000000001, 6.8676, 6.57645, 6.293400000000001, 6.0184500000000005, 5.7516, 5.492850000000001, 5.2421999999999995, 4.99965, 4.7652, 4.538850000000001, 4.3206, 4.11045, 3.908400000000001, 3.7144500000000003, 3.528600000000001, 3.3508500000000003, 3.1812000000000005, 3.0196500000000004, 2.866200000000001, 2.7208500000000004, 2.5836000000000006, 2.4544500000000005, 2.333400000000001, 2.2204500000000005, 2.1156000000000006, 2.0188500000000005, 1.9302000000000001, 1.8496500000000005, 1.7772000000000006, 1.7128500000000004, 1.6566, 1.6084500000000004, 1.5684000000000005, 1.5364500000000003, 1.5126000000000008, 1.4968500000000002, 1.4892000000000003, 1.4896500000000001, 1.4981999999999998, 1.51485, 1.539600000000001, 1.5724500000000008, 1.6133999999999995, 1.6624500000000006, 1.7196000000000007, 1.7848499999999996, 1.858200000000001, 1.9396499999999994, 2.0292000000000003, 2.12685, 2.2326000000000006, 2.34645, 2.4684, 2.5984500000000006, 2.7366, 2.8828500000000004, 3.0371999999999995, 3.199649999999999, 3.3701999999999996, 3.548849999999999, 3.7356000000000007, 3.9304499999999996, 4.133400000000001, 4.344450000000001, 4.5636, 4.790849999999998, 5.0262, 5.2696499999999995, 5.521200000000001, 5.780849999999998, 6.048599999999998, 6.32445, 6.608399999999997, 6.900450000000002, 7.2006000000000006, 7.508849999999998, 7.825199999999998, 8.149649999999998, 8.482199999999999, 8.822849999999999, 9.171599999999998, 9.52845, 9.893399999999996, 10.266449999999995, 10.647599999999997, 11.036849999999994, 11.434199999999993, 11.839649999999995, 12.2532, 12.67485, 13.104600000000001, 13.542449999999999, 13.988399999999999 ], [ 9.575025, 9.219075, 8.871225000000003, 8.531475, 8.199825, 7.876275000000001, 7.560825, 7.253475000000003, 6.954225000000002, 6.663075, 6.380025000000002, 6.105075, 5.838225, 5.579475000000001, 5.328825000000001, 5.086275000000001, 4.851825000000001, 4.625475000000001, 4.407225000000001, 4.197075000000001, 3.995025000000002, 3.8010750000000018, 3.6152250000000015, 3.437475000000001, 3.267825000000001, 3.106275000000001, 2.9528250000000016, 2.807475000000001, 2.670225000000001, 2.541075000000001, 2.4200250000000016, 2.307075000000001, 2.202225000000001, 2.105475000000001, 2.0168250000000008, 1.936275000000001, 1.8638250000000012, 1.799475000000001, 1.7432250000000016, 1.695075000000001, 1.655025000000001, 1.623075000000001, 1.5992250000000006, 1.5834750000000009, 1.575825000000001, 1.5762750000000016, 1.5848250000000013, 1.6014750000000015, 1.6262250000000007, 1.6590750000000005, 1.700025000000001, 1.7490750000000004, 1.8062250000000004, 1.871475000000001, 1.9448250000000007, 2.026275000000001, 2.115825, 2.2134750000000016, 2.3192250000000003, 2.4330750000000014, 2.5550249999999997, 2.685075000000002, 2.8232250000000016, 2.969475, 3.123825000000001, 3.2862750000000007, 3.4568249999999994, 3.6354750000000005, 3.8222250000000004, 4.017075000000001, 4.220025000000001, 4.431075000000001, 4.650225, 4.877475, 5.112825000000002, 5.356275000000001, 5.607825000000003, 5.867475, 6.135224999999999, 6.411075000000001, 6.6950249999999984, 6.987075, 7.287225000000002, 7.5954749999999995, 7.911824999999999, 8.236275, 8.568825, 8.909475, 9.258225, 9.615075000000001, 9.980024999999998, 10.353074999999997, 10.734224999999999, 11.123474999999996, 11.520824999999995, 11.926274999999997, 12.339825000000001, 12.761475, 13.191225000000003, 13.629075, 14.075025 ], [ 9.664100000000001, 9.308150000000001, 8.9603, 8.620550000000001, 8.288900000000002, 7.965350000000002, 7.6499000000000015, 7.342550000000002, 7.043300000000001, 6.752150000000001, 6.469100000000002, 6.194150000000001, 5.927300000000001, 5.6685500000000015, 5.4179, 5.175350000000001, 4.940900000000001, 4.714550000000002, 4.496300000000001, 4.286150000000001, 4.084100000000002, 3.890150000000001, 3.7043000000000017, 3.526550000000001, 3.3569000000000013, 3.1953500000000012, 3.041900000000002, 2.8965500000000013, 2.7593000000000014, 2.6301500000000013, 2.509100000000002, 2.3961500000000013, 2.2913000000000014, 2.1945500000000013, 2.105900000000001, 2.0253500000000013, 1.9529000000000014, 1.8885500000000013, 1.832300000000001, 1.7841500000000012, 1.7441000000000013, 1.7121500000000012, 1.6883000000000017, 1.672550000000001, 1.6649000000000012, 1.665350000000001, 1.6739000000000006, 1.6905500000000009, 1.7153000000000018, 1.7481500000000016, 1.7891000000000021, 1.8381500000000015, 1.8953000000000015, 1.9605500000000005, 2.0339, 2.115350000000002, 2.204900000000001, 2.302550000000001, 2.4082999999999997, 2.522150000000001, 2.644100000000001, 2.7741500000000014, 2.912300000000001, 3.058550000000001, 3.212900000000002, 3.375350000000002, 3.5459000000000005, 3.7245500000000016, 3.9112999999999998, 4.106150000000002, 4.3091, 4.52015, 4.739299999999999, 4.966550000000001, 5.201900000000003, 5.445350000000002, 5.6969, 5.956550000000001, 6.2243, 6.500149999999999, 6.7841, 7.076150000000001, 7.3763, 7.684550000000001, 8.000900000000001, 8.32535, 8.657900000000001, 8.998550000000002, 9.3473, 9.704150000000002, 10.069099999999999, 10.442149999999998, 10.8233, 11.212549999999997, 11.609899999999996, 12.015349999999998, 12.428900000000002, 12.850550000000002, 13.280300000000004, 13.718150000000001, 14.164100000000001 ], [ 9.755625, 9.399675, 9.051825000000001, 8.712075, 8.380425, 8.056875, 7.741425, 7.434075000000002, 7.134825000000001, 6.843674999999999, 6.560625, 6.2856749999999995, 6.018825, 5.7600750000000005, 5.509425, 5.266875000000001, 5.032425, 4.806075, 4.5878250000000005, 4.377675, 4.175625000000001, 3.981675000000001, 3.7958250000000007, 3.618075, 3.4484250000000003, 3.286875, 3.133425000000001, 2.9880750000000003, 2.8508250000000004, 2.7216750000000003, 2.600625000000001, 2.4876750000000003, 2.3828250000000004, 2.2860750000000003, 2.197425, 2.1168750000000003, 2.0444250000000004, 1.9800750000000003, 1.9238250000000008, 1.8756750000000002, 1.8356250000000003, 1.8036750000000001, 1.7798249999999998, 1.764075, 1.7564250000000001, 1.7568750000000009, 1.7654250000000005, 1.7820750000000007, 1.806825, 1.8396749999999997, 1.8806250000000002, 1.9296749999999996, 1.9868249999999996, 2.0520750000000003, 2.125425, 2.206875, 2.2964249999999993, 2.394075000000001, 2.4998249999999995, 2.6136750000000006, 2.735624999999999, 2.8656750000000013, 3.003825000000001, 3.1500749999999993, 3.304425, 3.466875, 3.6374249999999986, 3.8160749999999997, 4.002825, 4.197675, 4.400625, 4.611675, 4.830824999999999, 5.058074999999999, 5.293425000000001, 5.536875, 5.788425000000002, 6.048074999999999, 6.3158249999999985, 6.591675, 6.875624999999998, 7.167674999999999, 7.467825000000001, 7.776074999999999, 8.092424999999999, 8.416874999999997, 8.749424999999999, 9.090075, 9.438825, 9.795675000000001, 10.160624999999998, 10.533674999999997, 10.914824999999999, 11.304074999999996, 11.701424999999995, 12.106874999999997, 12.520425000000001, 12.942075, 13.371825000000003, 13.809675, 14.255625 ], [ 9.8496, 9.49365, 9.145800000000001, 8.80605, 8.474400000000001, 8.15085, 7.8354, 7.528050000000002, 7.2288000000000014, 6.93765, 6.6546, 6.37965, 6.1128, 5.854050000000001, 5.603400000000001, 5.360850000000001, 5.1264, 4.90005, 4.681800000000001, 4.47165, 4.269600000000001, 4.075650000000001, 3.889800000000001, 3.7120500000000005, 3.5424000000000007, 3.3808500000000006, 3.227400000000001, 3.0820500000000006, 2.9448000000000008, 2.8156500000000007, 2.694600000000001, 2.5816500000000007, 2.476800000000001, 2.3800500000000007, 2.2914000000000003, 2.2108500000000006, 2.1384000000000007, 2.0740500000000006, 2.017800000000001, 1.9696500000000006, 1.9296000000000006, 1.8976500000000005, 1.8738000000000001, 1.8580500000000004, 1.8504000000000005, 1.8508500000000012, 1.8594000000000008, 1.876050000000001, 1.9008000000000003, 1.93365, 1.9746000000000006, 2.02365, 2.0808, 2.1460500000000007, 2.2194000000000003, 2.3008500000000005, 2.3903999999999996, 2.488050000000001, 2.5938, 2.707650000000001, 2.8295999999999992, 2.9596500000000017, 3.097800000000001, 3.2440499999999997, 3.3984000000000005, 3.5608500000000003, 3.731399999999999, 3.91005, 4.0968, 4.291650000000001, 4.4946, 4.70565, 4.924799999999999, 5.152049999999999, 5.387400000000001, 5.630850000000001, 5.882400000000002, 6.142049999999999, 6.409799999999999, 6.685650000000001, 6.969599999999998, 7.2616499999999995, 7.561800000000002, 7.870049999999999, 8.186399999999999, 8.510849999999998, 8.843399999999999, 9.184050000000001, 9.5328, 9.889650000000001, 10.254599999999998, 10.627649999999997, 11.008799999999999, 11.398049999999996, 11.795399999999995, 12.200849999999997, 12.614400000000002, 13.036050000000001, 13.465800000000003, 13.90365, 14.3496 ], [ 9.946025, 9.590075, 9.242225000000001, 8.902475, 8.570825000000001, 8.247275000000002, 7.931825000000002, 7.624475000000002, 7.325225000000001, 7.034075000000001, 6.751025000000002, 6.476075000000002, 6.209225000000001, 5.950475000000002, 5.699825000000001, 5.457275000000001, 5.222825000000001, 4.996475000000002, 4.778225000000001, 4.568075000000001, 4.366025000000002, 4.172075000000001, 3.986225000000002, 3.8084750000000014, 3.6388250000000015, 3.4772750000000014, 3.323825000000002, 3.1784750000000015, 3.0412250000000016, 2.9120750000000015, 2.791025000000002, 2.6780750000000015, 2.5732250000000016, 2.4764750000000015, 2.387825000000001, 2.3072750000000015, 2.2348250000000016, 2.1704750000000015, 2.114225000000001, 2.0660750000000014, 2.0260250000000015, 1.9940750000000014, 1.970225000000002, 1.9544750000000013, 1.9468250000000014, 1.9472750000000012, 1.9558250000000008, 1.972475000000001, 1.997225000000002, 2.030075000000002, 2.0710250000000006, 2.1200750000000017, 2.1772250000000017, 2.2424750000000007, 2.315825000000002, 2.3972750000000005, 2.4868250000000014, 2.584475000000001, 2.6902250000000016, 2.804075000000001, 2.926025000000001, 3.0560750000000017, 3.194225000000001, 3.3404750000000014, 3.4948250000000005, 3.6572750000000003, 3.8278250000000007, 4.006475, 4.193225000000002, 4.388075000000001, 4.591025000000002, 4.802075000000002, 5.021225000000001, 5.248474999999999, 5.483825000000001, 5.727275000000001, 5.978825000000002, 6.238474999999999, 6.506224999999999, 6.782075000000001, 7.066024999999998, 7.358075000000003, 7.658225000000002, 7.966474999999999, 8.282824999999999, 8.607274999999998, 8.939824999999999, 9.280475000000001, 9.629225, 9.986075000000001, 10.351024999999998, 10.724074999999997, 11.105224999999999, 11.494474999999996, 11.891824999999995, 12.297274999999997, 12.710825000000002, 13.132475000000001, 13.562225000000003, 14.000075, 14.446025 ], [ 10.044900000000002, 9.688949999999998, 9.3411, 9.001350000000002, 8.669700000000002, 8.346150000000002, 8.0307, 7.723350000000001, 7.4241, 7.13295, 6.849900000000001, 6.57495, 6.308099999999999, 6.0493500000000004, 5.798699999999999, 5.55615, 5.3217, 5.095350000000001, 4.8770999999999995, 4.66695, 4.464900000000001, 4.27095, 4.085100000000001, 3.90735, 3.7377000000000002, 3.57615, 3.4227000000000007, 3.27735, 3.1401000000000003, 3.0109500000000002, 2.889900000000001, 2.7769500000000003, 2.6721000000000004, 2.5753500000000003, 2.4867, 2.4061500000000002, 2.3337000000000003, 2.26935, 2.2131, 2.16495, 2.1249000000000002, 2.09295, 2.0691000000000006, 2.05335, 2.0457, 2.04615, 2.0546999999999995, 2.07135, 2.0961000000000007, 2.1289500000000006, 2.169900000000001, 2.2189500000000004, 2.2761000000000005, 2.3413499999999994, 2.414699999999999, 2.496150000000001, 2.5857, 2.68335, 2.7890999999999986, 2.9029499999999997, 3.0248999999999997, 3.1549500000000004, 3.2931, 3.43935, 3.593700000000001, 3.7561500000000008, 3.9266999999999994, 4.1053500000000005, 4.292099999999999, 4.486950000000001, 4.689899999999999, 4.900949999999999, 5.120099999999998, 5.34735, 5.582700000000002, 5.826150000000001, 6.077699999999999, 6.33735, 6.605099999999999, 6.880949999999998, 7.1648999999999985, 7.45695, 7.7570999999999986, 8.065349999999999, 8.381699999999999, 8.706149999999997, 9.038699999999999, 9.379350000000002, 9.728100000000001, 10.084950000000003, 10.4499, 10.822949999999999, 11.2041, 11.593349999999997, 11.990699999999997, 12.396149999999999, 12.809700000000003, 13.231350000000003, 13.661100000000005, 14.098950000000002, 14.544900000000002 ], [ 10.146225000000001, 9.790275000000001, 9.442425, 9.102675000000001, 8.771025000000002, 8.447475, 8.132025000000002, 7.824675000000002, 7.525425000000001, 7.234275000000001, 6.951225000000002, 6.676275000000001, 6.4094250000000015, 6.150675000000002, 5.900025, 5.657475000000001, 5.423025000000001, 5.196675000000002, 4.9784250000000005, 4.768275000000001, 4.566225000000002, 4.372275000000001, 4.186425000000002, 4.008675000000001, 3.8390250000000012, 3.677475000000001, 3.5240250000000017, 3.378675000000001, 3.2414250000000013, 3.1122750000000012, 2.991225000000002, 2.8782750000000012, 2.7734250000000014, 2.6766750000000012, 2.588025000000001, 2.5074750000000012, 2.4350250000000013, 2.370675000000001, 2.314425000000001, 2.266275000000001, 2.2262250000000012, 2.194275000000001, 2.1704250000000016, 2.154675000000001, 2.147025000000001, 2.147475000000001, 2.1560250000000005, 2.172675000000001, 2.1974250000000017, 2.2302750000000016, 2.2712250000000003, 2.3202750000000014, 2.3774250000000015, 2.4426750000000004, 2.5160250000000017, 2.597475, 2.687025000000001, 2.784675000000001, 2.8904250000000014, 3.0042750000000007, 3.1262250000000007, 3.2562750000000014, 3.394425000000001, 3.540675000000001, 3.6950250000000002, 3.857475, 4.028025, 4.206675, 4.3934250000000015, 4.588275, 4.791225000000002, 5.002275000000002, 5.221425000000001, 5.448674999999999, 5.684025000000001, 5.927475, 6.179025000000002, 6.438674999999999, 6.7064249999999985, 6.9822750000000005, 7.266224999999998, 7.558275000000003, 7.858425000000001, 8.166674999999998, 8.483024999999998, 8.807474999999997, 9.140024999999998, 9.480675000000002, 9.829425, 10.186275000000002, 10.551224999999999, 10.924274999999998, 11.305425, 11.694674999999997, 12.092024999999996, 12.497474999999998, 12.911025000000002, 13.332675000000002, 13.762425000000004, 14.200275000000001, 14.646225000000001 ], [ 10.25, 9.89405, 9.5462, 9.20645, 8.8748, 8.55125, 8.2358, 7.9284500000000016, 7.629200000000001, 7.338049999999999, 7.055, 6.780049999999999, 6.513199999999999, 6.25445, 6.0038, 5.76125, 5.5268, 5.30045, 5.0822, 4.87205, 4.670000000000001, 4.476050000000001, 4.2902000000000005, 4.11245, 3.9428, 3.78125, 3.6278000000000006, 3.48245, 3.3452, 3.21605, 3.0950000000000006, 2.98205, 2.8772, 2.78045, 2.6917999999999997, 2.61125, 2.5388, 2.47445, 2.4182000000000006, 2.37005, 2.33, 2.29805, 2.2741999999999996, 2.25845, 2.2508, 2.2512500000000006, 2.2598000000000003, 2.2764500000000005, 2.3011999999999997, 2.3340499999999995, 2.375, 2.4240499999999994, 2.4811999999999994, 2.54645, 2.6197999999999997, 2.70125, 2.790799999999999, 2.8884500000000006, 2.9941999999999993, 3.1080500000000004, 3.2299999999999986, 3.360050000000001, 3.4982000000000006, 3.644449999999999, 3.7988, 3.9612499999999997, 4.131799999999998, 4.3104499999999994, 4.497199999999999, 4.69205, 4.895, 5.10605, 5.325199999999997, 5.5524499999999986, 5.787800000000001, 6.03125, 6.282800000000002, 6.542449999999999, 6.810199999999998, 7.08605, 7.369999999999997, 7.662049999999999, 7.962200000000001, 8.270449999999999, 8.586799999999998, 8.911249999999997, 9.243799999999998, 9.58445, 9.9332, 10.29005, 10.654999999999998, 11.028049999999997, 11.409199999999998, 11.798449999999995, 12.195799999999995, 12.601249999999997, 13.014800000000001, 13.43645, 13.866200000000003, 14.30405, 14.75 ] ] }, { "line": { "color": "orange", "width": 5 }, "mode": "lines", "type": "scatter3d", "x": [ 0, 0.03, 0.06, 0.09, 0.12, 0.15000000000000002, 0.18, 0.21000000000000002, 0.24, 0.27, 0.30000000000000004, 0.33, 0.36, 0.39, 0.42000000000000004, 0.44999999999999996, 0.48, 0.51, 0.54, 0.5700000000000001, 0.6000000000000001, 0.63, 0.66, 0.6900000000000001, 0.72, 0.75, 0.78, 0.81, 0.8400000000000001, 0.8699999999999999, 0.8999999999999999, 0.9299999999999999, 0.96, 0.99, 1.02, 1.05, 1.08, 1.1099999999999999, 1.1400000000000001, 1.17, 1.2000000000000002, 1.23, 1.26, 1.29, 1.32, 1.35, 1.3800000000000001, 1.4100000000000001, 1.44, 1.47, 1.5, 1.53, 1.56, 1.59, 1.62, 1.6500000000000001, 1.6800000000000002, 1.7100000000000002, 1.7399999999999998, 1.77, 1.7999999999999998, 1.83, 1.8599999999999999, 1.8900000000000001, 1.92, 1.9500000000000002, 1.98, 2.0100000000000002, 2.04, 2.0700000000000003, 2.1, 2.13, 2.16, 2.19, 2.2199999999999998, 2.25, 2.2800000000000002, 2.31, 2.34, 2.37, 2.4000000000000004, 2.43, 2.46, 2.49, 2.52, 2.55, 2.58, 2.61, 2.64, 2.67, 2.7, 2.73, 2.7600000000000002, 2.79, 2.8200000000000003, 2.85, 2.88, 2.91, 2.94, 2.9699999999999998, 3 ], "y": [ 0, 0, 0, 0, 0, 0, 0, 0, 0, 0, 0, 0, 0, 0, 0, 0, 0, 0, 0, 0, 0, 0, 0, 0, 0, 0, 0, 0, 0, 0, 0, 0, 0, 0, 0, 0, 0, 0, 0, 0, 0, 0, 0, 0, 0, 0, 0, 0, 0, 0, 0, 0, 0, 0, 0, 0, 0, 0, 0, 0, 0, 0, 0, 0, 0, 0, 0, 0, 0, 0, 0, 0, 0, 0, 0, 0, 0, 0, 0, 0, 0, 0, 0, 0, 0, 0, 0, 0, 0, 0, 0, 0, 0, 0, 0, 0, 0, 0, 0, 0, 0 ], "z": [ 0, 0, 0, 0, 0, 0, 0, 0, 0, 0, 0, 0, 0, 0, 0, 0, 0, 0, 0, 0, 0, 0, 0, 0, 0, 0, 0, 0, 0, 0, 0, 0, 0, 0, 0, 0, 0, 0, 0, 0, 0, 0, 0, 0, 0, 0, 0, 0, 0, 0, 0, 0, 0, 0, 0, 0, 0, 0, 0, 0, 0, 0, 0, 0, 0, 0, 0, 0, 0, 0, 0, 0, 0, 0, 0, 0, 0, 0, 0, 0, 0, 0, 0, 0, 0, 0, 0, 0, 0, 0, 0, 0, 0, 0, 0, 0, 0, 0, 0, 0, 0 ] }, { "line": { "color": "orange", "width": 5 }, "mode": "lines", "type": "scatter3d", "x": [ 3, 3, 3, 3, 3, 3, 3, 3, 3, 3, 3, 3, 3, 3, 3, 3, 3, 3, 3, 3, 3, 3, 3, 3, 3, 3, 3, 3, 3, 3, 3, 3, 3, 3, 3, 3, 3, 3, 3, 3, 3, 3, 3, 3, 3, 3, 3, 3, 3, 3, 3, 3, 3, 3, 3, 3, 3, 3, 3, 3, 3, 3, 3, 3, 3, 3, 3, 3, 3, 3, 3, 3, 3, 3, 3, 3, 3, 3, 3, 3, 3, 3, 3, 3, 3, 3, 3, 3, 3, 3, 3, 3, 3, 3, 3, 3, 3, 3, 3, 3, 3 ], "y": [ 0, 0.02, 0.04, 0.06, 0.08, 0.1, 0.12, 0.14, 0.16, 0.18, 0.2, 0.22, 0.24, 0.26, 0.28, 0.3, 0.32, 0.34, 0.36, 0.38, 0.4, 0.42, 0.44, 0.46, 0.48, 0.5, 0.52, 0.54, 0.56, 0.58, 0.6, 0.62, 0.64, 0.66, 0.68, 0.7000000000000001, 0.72, 0.74, 0.76, 0.78, 0.8, 0.8200000000000001, 0.84, 0.86, 0.88, 0.9, 0.92, 0.9400000000000001, 0.96, 0.98, 1, 1.02, 1.04, 1.06, 1.08, 1.1, 1.12, 1.1400000000000001, 1.16, 1.18, 1.2, 1.22, 1.24, 1.26, 1.28, 1.3, 1.32, 1.34, 1.36, 1.3800000000000001, 1.4000000000000001, 1.42, 1.44, 1.46, 1.48, 1.5, 1.52, 1.54, 1.56, 1.58, 1.6, 1.62, 1.6400000000000001, 1.6600000000000001, 1.68, 1.7, 1.72, 1.74, 1.76, 1.78, 1.8, 1.82, 1.84, 1.86, 1.8800000000000001, 1.9000000000000001, 1.92, 1.94, 1.96, 1.98, 2 ], "z": [ 0, 0, 0, 0, 0, 0, 0, 0, 0, 0, 0, 0, 0, 0, 0, 0, 0, 0, 0, 0, 0, 0, 0, 0, 0, 0, 0, 0, 0, 0, 0, 0, 0, 0, 0, 0, 0, 0, 0, 0, 0, 0, 0, 0, 0, 0, 0, 0, 0, 0, 0, 0, 0, 0, 0, 0, 0, 0, 0, 0, 0, 0, 0, 0, 0, 0, 0, 0, 0, 0, 0, 0, 0, 0, 0, 0, 0, 0, 0, 0, 0, 0, 0, 0, 0, 0, 0, 0, 0, 0, 0, 0, 0, 0, 0, 0, 0, 0, 0, 0, 0 ] }, { "line": { "color": "orange", "width": 5 }, "mode": "lines", "type": "scatter3d", "x": [ 0, 0.03, 0.06, 0.09, 0.12, 0.15000000000000002, 0.18, 0.21000000000000002, 0.24, 0.27, 0.30000000000000004, 0.33, 0.36, 0.39, 0.42000000000000004, 0.44999999999999996, 0.48, 0.51, 0.54, 0.5700000000000001, 0.6000000000000001, 0.63, 0.66, 0.6900000000000001, 0.72, 0.75, 0.78, 0.81, 0.8400000000000001, 0.8699999999999999, 0.8999999999999999, 0.9299999999999999, 0.96, 0.99, 1.02, 1.05, 1.08, 1.1099999999999999, 1.1400000000000001, 1.17, 1.2000000000000002, 1.23, 1.26, 1.29, 1.32, 1.35, 1.3800000000000001, 1.4100000000000001, 1.44, 1.47, 1.5, 1.53, 1.56, 1.59, 1.62, 1.6500000000000001, 1.6800000000000002, 1.7100000000000002, 1.7399999999999998, 1.77, 1.7999999999999998, 1.83, 1.8599999999999999, 1.8900000000000001, 1.92, 1.9500000000000002, 1.98, 2.0100000000000002, 2.04, 2.0700000000000003, 2.1, 2.13, 2.16, 2.19, 2.2199999999999998, 2.25, 2.2800000000000002, 2.31, 2.34, 2.37, 2.4000000000000004, 2.43, 2.46, 2.49, 2.52, 2.55, 2.58, 2.61, 2.64, 2.67, 2.7, 2.73, 2.7600000000000002, 2.79, 2.8200000000000003, 2.85, 2.88, 2.91, 2.94, 2.9699999999999998, 3 ], "y": [ 2, 2, 2, 2, 2, 2, 2, 2, 2, 2, 2, 2, 2, 2, 2, 2, 2, 2, 2, 2, 2, 2, 2, 2, 2, 2, 2, 2, 2, 2, 2, 2, 2, 2, 2, 2, 2, 2, 2, 2, 2, 2, 2, 2, 2, 2, 2, 2, 2, 2, 2, 2, 2, 2, 2, 2, 2, 2, 2, 2, 2, 2, 2, 2, 2, 2, 2, 2, 2, 2, 2, 2, 2, 2, 2, 2, 2, 2, 2, 2, 2, 2, 2, 2, 2, 2, 2, 2, 2, 2, 2, 2, 2, 2, 2, 2, 2, 2, 2, 2, 2 ], "z": [ 0, 0, 0, 0, 0, 0, 0, 0, 0, 0, 0, 0, 0, 0, 0, 0, 0, 0, 0, 0, 0, 0, 0, 0, 0, 0, 0, 0, 0, 0, 0, 0, 0, 0, 0, 0, 0, 0, 0, 0, 0, 0, 0, 0, 0, 0, 0, 0, 0, 0, 0, 0, 0, 0, 0, 0, 0, 0, 0, 0, 0, 0, 0, 0, 0, 0, 0, 0, 0, 0, 0, 0, 0, 0, 0, 0, 0, 0, 0, 0, 0, 0, 0, 0, 0, 0, 0, 0, 0, 0, 0, 0, 0, 0, 0, 0, 0, 0, 0, 0, 0 ] }, { "line": { "color": "orange", "width": 5 }, "mode": "lines", "type": "scatter3d", "x": [ 0, 0, 0, 0, 0, 0, 0, 0, 0, 0, 0, 0, 0, 0, 0, 0, 0, 0, 0, 0, 0, 0, 0, 0, 0, 0, 0, 0, 0, 0, 0, 0, 0, 0, 0, 0, 0, 0, 0, 0, 0, 0, 0, 0, 0, 0, 0, 0, 0, 0, 0, 0, 0, 0, 0, 0, 0, 0, 0, 0, 0, 0, 0, 0, 0, 0, 0, 0, 0, 0, 0, 0, 0, 0, 0, 0, 0, 0, 0, 0, 0, 0, 0, 0, 0, 0, 0, 0, 0, 0, 0, 0, 0, 0, 0, 0, 0, 0, 0, 0, 0 ], "y": [ 0, 0.02, 0.04, 0.06, 0.08, 0.1, 0.12, 0.14, 0.16, 0.18, 0.2, 0.22, 0.24, 0.26, 0.28, 0.3, 0.32, 0.34, 0.36, 0.38, 0.4, 0.42, 0.44, 0.46, 0.48, 0.5, 0.52, 0.54, 0.56, 0.58, 0.6, 0.62, 0.64, 0.66, 0.68, 0.7000000000000001, 0.72, 0.74, 0.76, 0.78, 0.8, 0.8200000000000001, 0.84, 0.86, 0.88, 0.9, 0.92, 0.9400000000000001, 0.96, 0.98, 1, 1.02, 1.04, 1.06, 1.08, 1.1, 1.12, 1.1400000000000001, 1.16, 1.18, 1.2, 1.22, 1.24, 1.26, 1.28, 1.3, 1.32, 1.34, 1.36, 1.3800000000000001, 1.4000000000000001, 1.42, 1.44, 1.46, 1.48, 1.5, 1.52, 1.54, 1.56, 1.58, 1.6, 1.62, 1.6400000000000001, 1.6600000000000001, 1.68, 1.7, 1.72, 1.74, 1.76, 1.78, 1.8, 1.82, 1.84, 1.86, 1.8800000000000001, 1.9000000000000001, 1.92, 1.94, 1.96, 1.98, 2 ], "z": [ 0, 0, 0, 0, 0, 0, 0, 0, 0, 0, 0, 0, 0, 0, 0, 0, 0, 0, 0, 0, 0, 0, 0, 0, 0, 0, 0, 0, 0, 0, 0, 0, 0, 0, 0, 0, 0, 0, 0, 0, 0, 0, 0, 0, 0, 0, 0, 0, 0, 0, 0, 0, 0, 0, 0, 0, 0, 0, 0, 0, 0, 0, 0, 0, 0, 0, 0, 0, 0, 0, 0, 0, 0, 0, 0, 0, 0, 0, 0, 0, 0, 0, 0, 0, 0, 0, 0, 0, 0, 0, 0, 0, 0, 0, 0, 0, 0, 0, 0, 0, 0 ] }, { "line": { "color": "red", "width": 5 }, "mode": "lines", "type": "scatter3d", "x": [ 0, 0.03, 0.06, 0.09, 0.12, 0.15000000000000002, 0.18, 0.21000000000000002, 0.24, 0.27, 0.30000000000000004, 0.33, 0.36, 0.39, 0.42000000000000004, 0.44999999999999996, 0.48, 0.51, 0.54, 0.5700000000000001, 0.6000000000000001, 0.63, 0.66, 0.6900000000000001, 0.72, 0.75, 0.78, 0.81, 0.8400000000000001, 0.8699999999999999, 0.8999999999999999, 0.9299999999999999, 0.96, 0.99, 1.02, 1.05, 1.08, 1.1099999999999999, 1.1400000000000001, 1.17, 1.2000000000000002, 1.23, 1.26, 1.29, 1.32, 1.35, 1.3800000000000001, 1.4100000000000001, 1.44, 1.47, 1.5, 1.53, 1.56, 1.59, 1.62, 1.6500000000000001, 1.6800000000000002, 1.7100000000000002, 1.7399999999999998, 1.77, 1.7999999999999998, 1.83, 1.8599999999999999, 1.8900000000000001, 1.92, 1.9500000000000002, 1.98, 2.0100000000000002, 2.04, 2.0700000000000003, 2.1, 2.13, 2.16, 2.19, 2.2199999999999998, 2.25, 2.2800000000000002, 2.31, 2.34, 2.37, 2.4000000000000004, 2.43, 2.46, 2.49, 2.52, 2.55, 2.58, 2.61, 2.64, 2.67, 2.7, 2.73, 2.7600000000000002, 2.79, 2.8200000000000003, 2.85, 2.88, 2.91, 2.94, 2.9699999999999998, 3 ], "y": [ 0, 0, 0, 0, 0, 0, 0, 0, 0, 0, 0, 0, 0, 0, 0, 0, 0, 0, 0, 0, 0, 0, 0, 0, 0, 0, 0, 0, 0, 0, 0, 0, 0, 0, 0, 0, 0, 0, 0, 0, 0, 0, 0, 0, 0, 0, 0, 0, 0, 0, 0, 0, 0, 0, 0, 0, 0, 0, 0, 0, 0, 0, 0, 0, 0, 0, 0, 0, 0, 0, 0, 0, 0, 0, 0, 0, 0, 0, 0, 0, 0, 0, 0, 0, 0, 0, 0, 0, 0, 0, 0, 0, 0, 0, 0, 0, 0, 0, 0, 0, 0 ], "z": [ 3, 2.8818, 2.7672, 2.6562, 2.5488, 2.445, 2.3448, 2.2481999999999998, 2.1552, 2.0658, 1.98, 1.8978, 1.8192, 1.7442, 1.6727999999999998, 1.605, 1.5408, 1.4802, 1.4232, 1.3698, 1.3199999999999998, 1.2738, 1.2311999999999999, 1.1922, 1.1568, 1.125, 1.0968, 1.0722, 1.0512, 1.0338, 1.02, 1.0098, 1.0032, 1.0002, 1.0008, 1.005, 1.0128, 1.0242, 1.0392000000000001, 1.0577999999999999, 1.08, 1.1058, 1.1352000000000002, 1.1682000000000001, 1.2048, 1.245, 1.2888000000000002, 1.3362000000000003, 1.3872, 1.4417999999999997, 1.5, 1.5617999999999999, 1.6272000000000002, 1.6962000000000002, 1.7688000000000006, 1.8450000000000006, 1.9248000000000003, 2.0082000000000004, 2.0951999999999993, 2.1858000000000004, 2.2799999999999994, 2.3778000000000006, 2.4791999999999996, 2.5842, 2.6928, 2.8050000000000006, 2.9208, 3.0402000000000005, 3.1632, 3.2898000000000014, 3.42, 3.553799999999999, 3.6912000000000003, 3.8322000000000003, 3.976799999999999, 4.125, 4.2768000000000015, 4.4322, 4.591199999999999, 4.7538, 4.920000000000002, 5.0898, 5.263199999999999, 5.440200000000001, 5.620800000000001, 5.805, 5.992800000000001, 6.184199999999999, 6.379200000000001, 6.5778, 6.780000000000001, 6.985799999999999, 7.195200000000002, 7.408200000000001, 7.624800000000002, 7.845000000000001, 8.0688, 8.296200000000002, 8.527199999999999, 8.761799999999997, 9 ] }, { "line": { "color": "red", "width": 5 }, "mode": "lines", "type": "scatter3d", "x": [ 0, 0.03, 0.06, 0.09, 0.12, 0.15000000000000002, 0.18, 0.21000000000000002, 0.24, 0.27, 0.30000000000000004, 0.33, 0.36, 0.39, 0.42000000000000004, 0.44999999999999996, 0.48, 0.51, 0.54, 0.5700000000000001, 0.6000000000000001, 0.63, 0.66, 0.6900000000000001, 0.72, 0.75, 0.78, 0.81, 0.8400000000000001, 0.8699999999999999, 0.8999999999999999, 0.9299999999999999, 0.96, 0.99, 1.02, 1.05, 1.08, 1.1099999999999999, 1.1400000000000001, 1.17, 1.2000000000000002, 1.23, 1.26, 1.29, 1.32, 1.35, 1.3800000000000001, 1.4100000000000001, 1.44, 1.47, 1.5, 1.53, 1.56, 1.59, 1.62, 1.6500000000000001, 1.6800000000000002, 1.7100000000000002, 1.7399999999999998, 1.77, 1.7999999999999998, 1.83, 1.8599999999999999, 1.8900000000000001, 1.92, 1.9500000000000002, 1.98, 2.0100000000000002, 2.04, 2.0700000000000003, 2.1, 2.13, 2.16, 2.19, 2.2199999999999998, 2.25, 2.2800000000000002, 2.31, 2.34, 2.37, 2.4000000000000004, 2.43, 2.46, 2.49, 2.52, 2.55, 2.58, 2.61, 2.64, 2.67, 2.7, 2.73, 2.7600000000000002, 2.79, 2.8200000000000003, 2.85, 2.88, 2.91, 2.94, 2.9699999999999998, 3 ], "y": [ 2, 2, 2, 2, 2, 2, 2, 2, 2, 2, 2, 2, 2, 2, 2, 2, 2, 2, 2, 2, 2, 2, 2, 2, 2, 2, 2, 2, 2, 2, 2, 2, 2, 2, 2, 2, 2, 2, 2, 2, 2, 2, 2, 2, 2, 2, 2, 2, 2, 2, 2, 2, 2, 2, 2, 2, 2, 2, 2, 2, 2, 2, 2, 2, 2, 2, 2, 2, 2, 2, 2, 2, 2, 2, 2, 2, 2, 2, 2, 2, 2, 2, 2, 2, 2, 2, 2, 2, 2, 2, 2, 2, 2, 2, 2, 2, 2, 2, 2, 2, 2 ], "z": [ 3, 2.8818, 2.7672, 2.6562, 2.5488, 2.445, 2.3448, 2.2481999999999998, 2.1552, 2.0658, 1.98, 1.8978, 1.8192, 1.7442, 1.6727999999999998, 1.605, 1.5408, 1.4802, 1.4232, 1.3698, 1.3199999999999998, 1.2738, 1.2311999999999999, 1.1922, 1.1568, 1.125, 1.0968, 1.0722, 1.0512, 1.0338, 1.02, 1.0098, 1.0032, 1.0002, 1.0008, 1.005, 1.0128, 1.0242, 1.0392000000000001, 1.0577999999999999, 1.08, 1.1058, 1.1352000000000002, 1.1682000000000001, 1.2048, 1.245, 1.2888000000000002, 1.3362000000000003, 1.3872, 1.4417999999999997, 1.5, 1.5617999999999999, 1.6272000000000002, 1.6962000000000002, 1.7688000000000006, 1.8450000000000006, 1.9248000000000003, 2.0082000000000004, 2.0951999999999993, 2.1858000000000004, 2.2799999999999994, 2.3778000000000006, 2.4791999999999996, 2.5842, 2.6928, 2.8050000000000006, 2.9208, 3.0402000000000005, 3.1632, 3.2898000000000014, 3.42, 3.553799999999999, 3.6912000000000003, 3.8322000000000003, 3.976799999999999, 4.125, 4.2768000000000015, 4.4322, 4.591199999999999, 4.7538, 4.920000000000002, 5.0898, 5.263199999999999, 5.440200000000001, 5.620800000000001, 5.805, 5.992800000000001, 6.184199999999999, 6.379200000000001, 6.5778, 6.780000000000001, 6.985799999999999, 7.195200000000002, 7.408200000000001, 7.624800000000002, 7.845000000000001, 8.0688, 8.296200000000002, 8.527199999999999, 8.761799999999997, 9 ] }, { "line": { "color": "red", "width": 5 }, "mode": "lines", "type": "scatter3d", "x": [ 3, 3, 3, 3, 3, 3, 3, 3, 3, 3, 3, 3, 3, 3, 3, 3, 3, 3, 3, 3, 3, 3, 3, 3, 3, 3, 3, 3, 3, 3, 3, 3, 3, 3, 3, 3, 3, 3, 3, 3, 3, 3, 3, 3, 3, 3, 3, 3, 3, 3, 3, 3, 3, 3, 3, 3, 3, 3, 3, 3, 3, 3, 3, 3, 3, 3, 3, 3, 3, 3, 3, 3, 3, 3, 3, 3, 3, 3, 3, 3, 3, 3, 3, 3, 3, 3, 3, 3, 3, 3, 3, 3, 3, 3, 3, 3, 3, 3, 3, 3, 3 ], "y": [ 0, 0.02, 0.04, 0.06, 0.08, 0.1, 0.12, 0.14, 0.16, 0.18, 0.2, 0.22, 0.24, 0.26, 0.28, 0.3, 0.32, 0.34, 0.36, 0.38, 0.4, 0.42, 0.44, 0.46, 0.48, 0.5, 0.52, 0.54, 0.56, 0.58, 0.6, 0.62, 0.64, 0.66, 0.68, 0.7000000000000001, 0.72, 0.74, 0.76, 0.78, 0.8, 0.8200000000000001, 0.84, 0.86, 0.88, 0.9, 0.92, 0.9400000000000001, 0.96, 0.98, 1, 1.02, 1.04, 1.06, 1.08, 1.1, 1.12, 1.1400000000000001, 1.16, 1.18, 1.2, 1.22, 1.24, 1.26, 1.28, 1.3, 1.32, 1.34, 1.36, 1.3800000000000001, 1.4000000000000001, 1.42, 1.44, 1.46, 1.48, 1.5, 1.52, 1.54, 1.56, 1.58, 1.6, 1.62, 1.6400000000000001, 1.6600000000000001, 1.68, 1.7, 1.72, 1.74, 1.76, 1.78, 1.8, 1.82, 1.84, 1.86, 1.8800000000000001, 1.9000000000000001, 1.92, 1.94, 1.96, 1.98, 2 ], "z": [ 9, 8.9604, 8.9216, 8.883599999999998, 8.8464, 8.810000000000002, 8.774399999999998, 8.7396, 8.7056, 8.6724, 8.639999999999999, 8.6084, 8.5776, 8.5476, 8.518399999999998, 8.49, 8.462399999999999, 8.4356, 8.409600000000001, 8.384400000000001, 8.36, 8.336400000000001, 8.313600000000001, 8.2916, 8.270399999999999, 8.25, 8.2304, 8.211599999999999, 8.1936, 8.176400000000001, 8.16, 8.1444, 8.1296, 8.1156, 8.1024, 8.089999999999998, 8.0784, 8.067599999999999, 8.0576, 8.048399999999999, 8.040000000000001, 8.032399999999999, 8.0256, 8.0196, 8.0144, 8.009999999999998, 8.0064, 8.003600000000002, 8.001600000000002, 8.000399999999999, 8, 8.000399999999999, 8.001600000000002, 8.003599999999999, 8.0064, 8.010000000000002, 8.0144, 8.0196, 8.0256, 8.032399999999999, 8.040000000000001, 8.048399999999999, 8.0576, 8.067600000000002, 8.0784, 8.090000000000002, 8.1024, 8.1156, 8.129599999999998, 8.1444, 8.16, 8.176400000000001, 8.1936, 8.211599999999999, 8.2304, 8.25, 8.270400000000002, 8.2916, 8.313599999999997, 8.336400000000001, 8.360000000000003, 8.384400000000001, 8.409599999999998, 8.4356, 8.462399999999999, 8.49, 8.518400000000002, 8.5476, 8.5776, 8.608399999999998, 8.640000000000002, 8.6724, 8.7056, 8.739600000000001, 8.774400000000002, 8.809999999999999, 8.8464, 8.883600000000001, 8.9216, 8.9604, 9 ] }, { "line": { "color": "red", "width": 5 }, "mode": "lines", "type": "scatter3d", "x": [ 0, 0, 0, 0, 0, 0, 0, 0, 0, 0, 0, 0, 0, 0, 0, 0, 0, 0, 0, 0, 0, 0, 0, 0, 0, 0, 0, 0, 0, 0, 0, 0, 0, 0, 0, 0, 0, 0, 0, 0, 0, 0, 0, 0, 0, 0, 0, 0, 0, 0, 0, 0, 0, 0, 0, 0, 0, 0, 0, 0, 0, 0, 0, 0, 0, 0, 0, 0, 0, 0, 0, 0, 0, 0, 0, 0, 0, 0, 0, 0, 0, 0, 0, 0, 0, 0, 0, 0, 0, 0, 0, 0, 0, 0, 0, 0, 0, 0, 0, 0, 0 ], "y": [ 0, 0.02, 0.04, 0.06, 0.08, 0.1, 0.12, 0.14, 0.16, 0.18, 0.2, 0.22, 0.24, 0.26, 0.28, 0.3, 0.32, 0.34, 0.36, 0.38, 0.4, 0.42, 0.44, 0.46, 0.48, 0.5, 0.52, 0.54, 0.56, 0.58, 0.6, 0.62, 0.64, 0.66, 0.68, 0.7000000000000001, 0.72, 0.74, 0.76, 0.78, 0.8, 0.8200000000000001, 0.84, 0.86, 0.88, 0.9, 0.92, 0.9400000000000001, 0.96, 0.98, 1, 1.02, 1.04, 1.06, 1.08, 1.1, 1.12, 1.1400000000000001, 1.16, 1.18, 1.2, 1.22, 1.24, 1.26, 1.28, 1.3, 1.32, 1.34, 1.36, 1.3800000000000001, 1.4000000000000001, 1.42, 1.44, 1.46, 1.48, 1.5, 1.52, 1.54, 1.56, 1.58, 1.6, 1.62, 1.6400000000000001, 1.6600000000000001, 1.68, 1.7, 1.72, 1.74, 1.76, 1.78, 1.8, 1.82, 1.84, 1.86, 1.8800000000000001, 1.9000000000000001, 1.92, 1.94, 1.96, 1.98, 2 ], "z": [ 3, 2.9604, 2.9216, 2.8836, 2.8464, 2.81, 2.7744, 2.7396, 2.7056, 2.6724, 2.64, 2.6084, 2.5776, 2.5476, 2.5183999999999997, 2.49, 2.4624, 2.4356, 2.4096, 2.3844, 2.36, 2.3364000000000003, 2.3136, 2.2916, 2.2704, 2.25, 2.2304, 2.2116, 2.1936, 2.1764, 2.16, 2.1444, 2.1296, 2.1155999999999997, 2.1024000000000003, 2.09, 2.0784000000000002, 2.0676, 2.0576, 2.0484, 2.04, 2.0324, 2.0256, 2.0196, 2.0144, 2.01, 2.0064, 2.0036, 2.0016, 2.0004, 2, 2.0004, 2.0016, 2.0036, 2.0064, 2.01, 2.0144, 2.0196, 2.0256, 2.0324, 2.04, 2.0484, 2.0576, 2.0676, 2.0784000000000002, 2.09, 2.1024000000000003, 2.1156, 2.1296, 2.1444, 2.16, 2.1764, 2.1936, 2.2116, 2.2304, 2.25, 2.2704, 2.2916, 2.3136, 2.3364000000000003, 2.3600000000000003, 2.3844000000000003, 2.4096, 2.4356, 2.4623999999999997, 2.4899999999999998, 2.5183999999999997, 2.5476, 2.5776, 2.6084, 2.64, 2.6724, 2.7056, 2.7396000000000003, 2.7744000000000004, 2.81, 2.8464, 2.8836, 2.9215999999999998, 2.9604, 3 ] } ], "layout": {} }, "text/html": [ "
" ] }, "metadata": {}, "output_type": "display_data" } ], "source": [ "import plotly.graph_objs as go\n", "\n", "x=np.linspace(-1,3.5,101)\n", "y=np.linspace(-1,2.5,101)\n", "\n", "x,y=np.meshgrid(x,y)\n", "f= 2*x**2+y**2-4*x-2*y+3\n", "\n", "t=np.linspace(0,1,101)\n", "L1x=3*t\n", "L1z=2*L1x**2-4*L1x+3\n", "L3z=2*L1x**2-4*L1x+3\n", "\n", "L2y=2*t\n", "L2z=2*3**2+L2y**2-4*3-2*L2y+3\n", "L4z=2*0**2+L2y**2-4*0-2*L2y+3\n", "#zt=xt**2-xt*yt+yt**2-xt+yt-6\n", "surface = go.Surface(x=x, y=y, z=f,opacity=0.8)\n", "L1 = go.Scatter3d(x=L1x, y=0*L1x, z=0*L1x,\n", " mode = \"lines\",\n", " line = dict(color='orange',width = 5)\n", " ) \n", "L3 = go.Scatter3d(x=L1x, y=0*L1x+2, z=0*L1x,\n", " mode = \"lines\",\n", " line = dict(color='orange',width = 5)\n", " )\n", "L2 = go.Scatter3d(x=0*L2y+3, y=L2y, z=0*L2y,\n", " mode = \"lines\",\n", " line = dict(color='orange',width = 5)\n", " )\n", "L4 = go.Scatter3d(x=0*L2y, y=L2y, z=0*L2y,\n", " mode = \"lines\",\n", " line = dict(color='orange',width = 5)\n", " )\n", "L1f = go.Scatter3d(x=L1x, y=0*L1x, z=L1z,\n", " mode = \"lines\",\n", " line = dict(color='red',width = 5)\n", " ) \n", "L3f = go.Scatter3d(x=L1x, y=0*L1x+2, z=L3z,\n", " mode = \"lines\",\n", " line = dict(color='red',width = 5)\n", " ) \n", "L2f = go.Scatter3d(x=0*L2y+3, y=L2y, z=L2z,\n", " mode = \"lines\",\n", " line = dict(color='red',width = 5)\n", " )\n", "L4f = go.Scatter3d(x=0*L2y, y=L2y, z=L4z,\n", " mode = \"lines\",\n", " line = dict(color='red',width = 5)\n", " )\n", "\n", "data = [surface,L1,L2,L3,L4,L1f,L3f,L2f,L4f]\n", "\n", "fig = go.Figure(data=data)\n", "iplot(fig)" ] }, { "cell_type": "code", "execution_count": 4, "metadata": {}, "outputs": [ { "data": { "text/html": [ "" ], "text/plain": [ "" ] }, "metadata": {}, "output_type": "display_data" }, { "data": { "application/vnd.plotly.v1+json": { "data": [ { "opacity": 0.9, "type": "surface", "x": [ [ 0, 0.03, 0.06, 0.09, 0.12, 0.15, 0.18, 0.21, 0.24, 0.27, 0.3, 0.32999999999999996, 0.36, 0.39, 0.42, 0.44999999999999996, 0.48, 0.51, 0.54, 0.57, 0.6, 0.63, 0.6599999999999999, 0.69, 0.72, 0.75, 0.78, 0.8099999999999999, 0.84, 0.87, 0.8999999999999999, 0.9299999999999999, 0.96, 0.99, 1.02, 1.05, 1.08, 1.1099999999999999, 1.14, 1.17, 1.2, 1.23, 1.26, 1.29, 1.3199999999999998, 1.3499999999999999, 1.38, 1.41, 1.44, 1.47, 1.5, 1.53, 1.56, 1.5899999999999999, 1.6199999999999999, 1.65, 1.68, 1.71, 1.74, 1.77, 1.7999999999999998, 1.8299999999999998, 1.8599999999999999, 1.89, 1.92, 1.95, 1.98, 2.01, 2.04, 2.07, 2.1, 2.13, 2.16, 2.19, 2.2199999999999998, 2.25, 2.28, 2.31, 2.34, 2.37, 2.4, 2.4299999999999997, 2.46, 2.4899999999999998, 2.52, 2.55, 2.58, 2.61, 2.6399999999999997, 2.67, 2.6999999999999997, 2.73, 2.76, 2.79, 2.82, 2.85, 2.88, 2.9099999999999997, 2.94, 2.9699999999999998, 3 ], [ 0, 0.029940801852848146, 0.05988160370569629, 0.08982240555854444, 0.11976320741139258, 0.14970400926424074, 0.17964481111708888, 0.20958561296993702, 0.23952641482278517, 0.26946721667563334, 0.2994080185284815, 0.32934882038132957, 0.35928962223417776, 0.3892304240870259, 0.41917122593987405, 0.44911202779272213, 0.47905282964557033, 0.5089936314984185, 0.5389344333512667, 0.5688752352041148, 0.598816037056963, 0.628756838909811, 0.6586976407626591, 0.6886384426155073, 0.7185792444683555, 0.7485200463212036, 0.7784608481740518, 0.8084016500268999, 0.8383424518797481, 0.8682832537325963, 0.8982240555854443, 0.9281648574382925, 0.9581056592911407, 0.9880464611439889, 1.017987262996837, 1.0479280648496851, 1.0778688667025333, 1.1078096685553813, 1.1377504704082295, 1.1676912722610777, 1.197632074113926, 1.227572875966774, 1.257513677819622, 1.2874544796724703, 1.3173952815253183, 1.3473360833781665, 1.3772768852310147, 1.4072176870838629, 1.437158488936711, 1.4670992907895593, 1.4970400926424072, 1.5269808944952554, 1.5569216963481036, 1.5868624982009516, 1.6168033000537998, 1.646744101906648, 1.6766849037594962, 1.7066257056123444, 1.7365665074651926, 1.7665073093180408, 1.7964481111708885, 1.8263889130237367, 1.856329714876585, 1.8862705167294331, 1.9162113185822813, 1.9461521204351295, 1.9760929222879777, 2.0060337241408255, 2.035974525993674, 2.065915327846522, 2.0958561296993703, 2.1257969315522183, 2.1557377334050667, 2.1856785352579147, 2.2156193371107626, 2.245560138963611, 2.275500940816459, 2.3054417426693075, 2.3353825445221554, 2.365323346375004, 2.395264148227852, 2.4252049500807, 2.455145751933548, 2.4850865537863958, 2.515027355639244, 2.544968157492092, 2.5749089593449406, 2.6048497611977885, 2.6347905630506365, 2.664731364903485, 2.694672166756333, 2.7246129686091813, 2.7545537704620293, 2.7844945723148777, 2.8144353741677257, 2.844376176020574, 2.874316977873422, 2.90425777972627, 2.9341985815791185, 2.9641393834319665, 2.9940801852848145 ], [ 0, 0.029763441039434336, 0.05952688207886867, 0.08929032311830301, 0.11905376415773734, 0.14881720519717168, 0.17858064623660602, 0.20834408727604034, 0.2381075283154747, 0.26787096935490906, 0.29763441039434335, 0.32739785143377764, 0.35716129247321204, 0.3869247335126464, 0.4166881745520807, 0.446451615591515, 0.4762150566309494, 0.5059784976703837, 0.5357419387098181, 0.5655053797492523, 0.5952688207886867, 0.6250322618281211, 0.6547957028675553, 0.6845591439069897, 0.7143225849464241, 0.7440860259858584, 0.7738494670252928, 0.8036129080647271, 0.8333763491041614, 0.8631397901435958, 0.89290323118303, 0.9226666722224643, 0.9524301132618987, 0.982193554301333, 1.0119569953407674, 1.0417204363802017, 1.0714838774196362, 1.1012473184590703, 1.1310107594985046, 1.1607742005379391, 1.1905376415773734, 1.2203010826168077, 1.2500645236562422, 1.2798279646956765, 1.3095914057351106, 1.339354846774545, 1.3691182878139794, 1.3988817288534137, 1.4286451698928482, 1.4584086109322825, 1.4881720519717168, 1.5179354930111513, 1.5476989340505856, 1.5774623750900196, 1.6072258161294541, 1.6369892571688884, 1.6667526982083227, 1.6965161392477572, 1.7262795802871915, 1.7560430213266258, 1.78580646236606, 1.8155699034054944, 1.8453333444449287, 1.875096785484363, 1.9048602265237975, 1.9346236675632318, 1.964387108602666, 1.9941505496421004, 2.023913990681535, 2.053677431720969, 2.0834408727604035, 2.1132043137998378, 2.1429677548392725, 2.1727311958787063, 2.2024946369181406, 2.2322580779575754, 2.262021518997009, 2.291784960036444, 2.3215484010758782, 2.3513118421153125, 2.381075283154747, 2.410838724194181, 2.4406021652336154, 2.4703656062730497, 2.5001290473124844, 2.5298924883519183, 2.559655929391353, 2.5894193704307873, 2.619182811470221, 2.648946252509656, 2.67870969354909, 2.7084731345885245, 2.7382365756279587, 2.7680000166673935, 2.7977634577068273, 2.827526898746262, 2.8572903397856964, 2.88705378082513, 2.916817221864565, 2.9465806629039992, 2.9763441039434335 ], [ 0, 0.02946861752186066, 0.05893723504372132, 0.08840585256558198, 0.11787447008744265, 0.1473430876093033, 0.17681170513116395, 0.20628032265302462, 0.2357489401748853, 0.265217557696746, 0.2946861752186066, 0.3241547927404672, 0.3536234102623279, 0.3830920277841886, 0.41256064530604925, 0.4420292628279099, 0.4714978803497706, 0.5009664978716313, 0.530435115393492, 0.5599037329153526, 0.5893723504372133, 0.618840967959074, 0.6483095854809344, 0.6777782030027951, 0.7072468205246558, 0.7367154380465165, 0.7661840555683772, 0.7956526730902378, 0.8251212906120985, 0.8545899081339592, 0.8840585256558198, 0.9135271431776805, 0.9429957606995412, 0.9724643782214019, 1.0019329957432626, 1.0314016132651231, 1.060870230786984, 1.0903388483088443, 1.1198074658307051, 1.1492760833525657, 1.1787447008744265, 1.208213318396287, 1.237681935918148, 1.2671505534400085, 1.2966191709618688, 1.3260877884837297, 1.3555564060055902, 1.385025023527451, 1.4144936410493116, 1.4439622585711724, 1.473430876093033, 1.5028994936148938, 1.5323681111367544, 1.561836728658615, 1.5913053461804756, 1.6207739637023364, 1.650242581224197, 1.6797111987460578, 1.7091798162679184, 1.738648433789779, 1.7681170513116395, 1.7975856688335001, 1.827054286355361, 1.8565229038772215, 1.8859915213990823, 1.915460138920943, 1.9449287564428037, 1.974397373964664, 2.003865991486525, 2.0333346090083855, 2.0628032265302463, 2.092271844052107, 2.121740461573968, 2.1512090790958283, 2.1806776966176886, 2.2101463141395494, 2.2396149316614102, 2.269083549183271, 2.2985521667051314, 2.328020784226992, 2.357489401748853, 2.3869580192707134, 2.416426636792574, 2.4458952543144346, 2.475363871836296, 2.504832489358156, 2.534301106880017, 2.5637697244018773, 2.5932383419237377, 2.622706959445599, 2.6521755769674593, 2.68164419448932, 2.7111128120111805, 2.7405814295330417, 2.770050047054902, 2.799518664576763, 2.8289872820986233, 2.858455899620484, 2.887924517142345, 2.9173931346642052, 2.946861752186066 ], [ 0, 0.02905749483385893, 0.05811498966771786, 0.08717248450157679, 0.11622997933543572, 0.14528747416929466, 0.17434496900315358, 0.20340246383701252, 0.23245995867087144, 0.26151745350473043, 0.2905749483385893, 0.3196324431724482, 0.34868993800630715, 0.37774743284016615, 0.40680492767402504, 0.43586242250788393, 0.46491991734174287, 0.49397741217560187, 0.5230349070094609, 0.5520924018433196, 0.5811498966771786, 0.6102073915110375, 0.6392648863448964, 0.6683223811787554, 0.6973798760126143, 0.7264373708464733, 0.7554948656803323, 0.7845523605141911, 0.8136098553480501, 0.8426673501819091, 0.8717248450157679, 0.9007823398496269, 0.9298398346834857, 0.9588973295173447, 0.9879548243512037, 1.0170123191850626, 1.0460698140189217, 1.0751273088527804, 1.1041848036866393, 1.1332422985204982, 1.1622997933543573, 1.1913572881882162, 1.220414783022075, 1.2494722778559342, 1.2785297726897928, 1.3075872675236517, 1.3366447623575108, 1.3657022571913697, 1.3947597520252286, 1.4238172468590877, 1.4528747416929466, 1.4819322365268055, 1.5109897313606646, 1.5400472261945233, 1.5691047210283822, 1.5981622158622413, 1.6272197106961002, 1.656277205529959, 1.6853347003638182, 1.714392195197677, 1.7434496900315357, 1.7725071848653948, 1.8015646796992537, 1.8306221745331126, 1.8596796693669715, 1.8887371642008306, 1.9177946590346895, 1.9468521538685482, 1.9759096487024075, 2.004967143536266, 2.0340246383701253, 2.063082133203984, 2.0921396280378435, 2.121197122871702, 2.150254617705561, 2.17931211253942, 2.2083696073732786, 2.2374271022071377, 2.2664845970409964, 2.295542091874856, 2.3245995867087146, 2.3536570815425732, 2.3827145763764324, 2.411772071210291, 2.44082956604415, 2.4698870608780092, 2.4989445557118684, 2.528002050545727, 2.5570595453795857, 2.586117040213445, 2.6151745350473035, 2.644232029881163, 2.6732895247150217, 2.702347019548881, 2.7314045143827395, 2.7604620092165986, 2.7895195040504572, 2.8185769988843163, 2.8476344937181755, 2.876691988552034, 2.9057494833858932 ], [ 0, 0.028531695488854605, 0.05706339097770921, 0.08559508646656382, 0.11412678195541842, 0.142658477444273, 0.17119017293312763, 0.19972186842198222, 0.22825356391083684, 0.2567852593996915, 0.285316954888546, 0.3138486503774006, 0.34238034586625526, 0.3709120413551099, 0.39944373684396445, 0.42797543233281904, 0.4565071278216737, 0.48503882331052833, 0.513570518799383, 0.5421022142882375, 0.570633909777092, 0.5991656052659468, 0.6276973007548012, 0.6562289962436559, 0.6847606917325105, 0.7132923872213651, 0.7418240827102198, 0.7703557781990743, 0.7988874736879289, 0.8274191691767836, 0.8559508646656381, 0.8844825601544927, 0.9130142556433474, 0.941545951132202, 0.9700776466210567, 0.9986093421099113, 1.027141037598766, 1.0556727330876203, 1.084204428576475, 1.1127361240653295, 1.141267819554184, 1.169799515043039, 1.1983312105318935, 1.226862906020748, 1.2553946015096025, 1.283926296998457, 1.3124579924873119, 1.3409896879761665, 1.369521383465021, 1.3980530789538756, 1.4265847744427302, 1.4551164699315848, 1.4836481654204396, 1.512179860909294, 1.5407115563981486, 1.5692432518870032, 1.5977749473758578, 1.6263066428647126, 1.6548383383535672, 1.6833700338424218, 1.7119017293312762, 1.7404334248201307, 1.7689651203089853, 1.7974968157978402, 1.8260285112866947, 1.8545602067755493, 1.883091902264404, 1.9116235977532583, 1.9401552932421133, 1.9686869887309677, 1.9972186842198225, 2.025750379708677, 2.054282075197532, 2.082813770686386, 2.1113454661752407, 2.1398771616640953, 2.16840885715295, 2.196940552641805, 2.225472248130659, 2.254003943619514, 2.282535639108368, 2.311067334597223, 2.339599030086078, 2.368130725574932, 2.396662421063787, 2.425194116552641, 2.453725812041496, 2.482257507530351, 2.510789203019205, 2.53932089850806, 2.567852593996914, 2.596384289485769, 2.6249159849746238, 2.6534476804634783, 2.681979375952333, 2.7105110714411875, 2.739042766930042, 2.7675744624188967, 2.7961061579077513, 2.824637853396606, 2.8531695488854605 ], [ 0, 0.027893294576647538, 0.055786589153295076, 0.08367988372994262, 0.11157317830659015, 0.13946647288323769, 0.16735976745988523, 0.19525306203653278, 0.2231463566131803, 0.2510396511898279, 0.27893294576647537, 0.3068262403431229, 0.33471953491977047, 0.36261282949641804, 0.39050612407306556, 0.4183994186497131, 0.4462927132263606, 0.4741860078030082, 0.5020793023796558, 0.5299725969563033, 0.5578658915329507, 0.5857591861095983, 0.6136524806862458, 0.6415457752628934, 0.6694390698395409, 0.6973323644161885, 0.7252256589928361, 0.7531189535694836, 0.7810122481461311, 0.8089055427227787, 0.8367988372994262, 0.8646921318760737, 0.8925854264527212, 0.9204787210293688, 0.9483720156060164, 0.9762653101826639, 1.0041586047593116, 1.0320518993359589, 1.0599451939126066, 1.087838488489254, 1.1157317830659015, 1.1436250776425492, 1.1715183722191966, 1.1994116667958443, 1.2273049613724916, 1.2551982559491393, 1.2830915505257867, 1.3109848451024344, 1.3388781396790819, 1.3667714342557296, 1.394664728832377, 1.4225580234090245, 1.4504513179856722, 1.4783446125623194, 1.506237907138967, 1.5341312017156146, 1.5620244962922623, 1.5899177908689097, 1.6178110854455574, 1.6457043800222049, 1.6735976745988523, 1.7014909691754998, 1.7293842637521475, 1.757277558328795, 1.7851708529054424, 1.81306414748209, 1.8409574420587376, 1.868850736635385, 1.8967440312120327, 1.9246373257886802, 1.9525306203653279, 1.9804239149419753, 2.0083172095186232, 2.0362105040952705, 2.0641037986719177, 2.0919970932485654, 2.119890387825213, 2.147783682401861, 2.175676976978508, 2.2035702715551557, 2.231463566131803, 2.2593568607084507, 2.2872501552850983, 2.3151434498617456, 2.3430367444383933, 2.370930039015041, 2.3988233335916886, 2.426716628168336, 2.454609922744983, 2.482503217321631, 2.5103965118982785, 2.538289806474926, 2.5661831010515734, 2.594076395628221, 2.621969690204869, 2.6498629847815165, 2.6777562793581637, 2.705649573934811, 2.733542868511459, 2.7614361630881064, 2.789329457664754 ], [ 0, 0.027144811573980583, 0.054289623147961166, 0.08143443472194174, 0.10857924629592233, 0.1357240578699029, 0.16286886944388348, 0.1900136810178641, 0.21715849259184467, 0.24430330416582527, 0.2714481157398058, 0.2985929273137864, 0.32573773888776697, 0.3528825504617476, 0.3800273620357282, 0.4071721736097087, 0.43431698518368933, 0.46146179675766996, 0.48860660833165054, 0.515751419905631, 0.5428962314796116, 0.5700410430535923, 0.5971858546275728, 0.6243306662015534, 0.6514754777755339, 0.6786202893495146, 0.7057651009234952, 0.7329099124974757, 0.7600547240714564, 0.7871995356454369, 0.8143443472194174, 0.841489158793398, 0.8686339703673787, 0.8957787819413593, 0.9229235935153399, 0.9500684050893204, 0.9772132166633011, 1.0043580282372815, 1.031502839811262, 1.0586476513852427, 1.0857924629592233, 1.112937274533204, 1.1400820861071845, 1.167226897681165, 1.1943717092551456, 1.221516520829126, 1.2486613324031068, 1.2758061439770874, 1.3029509555510679, 1.3300957671250486, 1.3572405786990291, 1.3843853902730099, 1.4115302018469904, 1.438675013420971, 1.4658198249949514, 1.492964636568932, 1.5201094481429127, 1.5472542597168932, 1.5743990712908738, 1.6015438828648545, 1.6286886944388348, 1.6558335060128155, 1.682978317586796, 1.7101231291607768, 1.7372679407347573, 1.7644127523087378, 1.7915575638827186, 1.8187023754566989, 1.8458471870306798, 1.8729919986046601, 1.900136810178641, 1.9272816217526214, 1.9544264333266022, 1.9815712449005827, 2.008716056474563, 2.0358608680485437, 2.063005679622524, 2.090150491196505, 2.1172953027704855, 2.1444401143444662, 2.1715849259184465, 2.198729737492427, 2.225874549066408, 2.2530193606403883, 2.280164172214369, 2.3073089837883494, 2.33445379536233, 2.361598606936311, 2.388743418510291, 2.415888230084272, 2.443033041658252, 2.470177853232233, 2.4973226648062137, 2.5244674763801944, 2.5516122879541747, 2.5787570995281555, 2.6059019111021358, 2.6330467226761165, 2.6601915342500972, 2.6873363458240775, 2.7144811573980583 ], [ 0, 0.026289200401315906, 0.05257840080263181, 0.07886760120394772, 0.10515680160526363, 0.13144600200657952, 0.15773520240789543, 0.18402440280921134, 0.21031360321052725, 0.2366028036118432, 0.26289200401315904, 0.2891812044144749, 0.31547040481579086, 0.3417596052171068, 0.3680488056184227, 0.39433800601973856, 0.4206272064210545, 0.44691640682237044, 0.4732056072236864, 0.4994948076250022, 0.5257840080263181, 0.5520732084276341, 0.5783624088289498, 0.6046516092302658, 0.6309408096315817, 0.6572300100328977, 0.6835192104342136, 0.7098084108355295, 0.7360976112368454, 0.7623868116381614, 0.7886760120394771, 0.8149652124407931, 0.841254412842109, 0.867543613243425, 0.8938328136447409, 0.9201220140460568, 0.9464112144473728, 0.9727004148486885, 0.9989896152500044, 1.0252788156513204, 1.0515680160526362, 1.0778572164539522, 1.1041464168552682, 1.1304356172565841, 1.1567248176578997, 1.1830140180592157, 1.2093032184605317, 1.2355924188618477, 1.2618816192631634, 1.2881708196644794, 1.3144600200657954, 1.3407492204671112, 1.3670384208684272, 1.393327621269743, 1.419616821671059, 1.4459060220723747, 1.4721952224736907, 1.4984844228750067, 1.5247736232763227, 1.5510628236776385, 1.5773520240789543, 1.6036412244802702, 1.6299304248815862, 1.656219625282902, 1.682508825684218, 1.708798026085534, 1.73508722648685, 1.7613764268881655, 1.7876656272894818, 1.8139548276907975, 1.8402440280921135, 1.8665332284934293, 1.8928224288947455, 1.9191116292960613, 1.945400829697377, 1.971690030098693, 1.9979792305000088, 2.024268430901325, 2.050557631302641, 2.076846831703957, 2.1031360321052723, 2.129425232506588, 2.1557144329079043, 2.18200363330922, 2.2082928337105363, 2.234582034111852, 2.2608712345131683, 2.287160434914484, 2.3134496353157994, 2.3397388357171156, 2.3660280361184314, 2.3923172365197476, 2.4186064369210634, 2.4448956373223796, 2.4711848377236953, 2.497474038125011, 2.523763238526327, 2.5500524389276427, 2.576341639328959, 2.6026308397302746, 2.628920040131591 ], [ 0, 0.02532983776506045, 0.0506596755301209, 0.07598951329518136, 0.1013193510602418, 0.12664918882530227, 0.15197902659036272, 0.17730886435542315, 0.2026387021204836, 0.22796853988554408, 0.25329837765060453, 0.27862821541566496, 0.30395805318072544, 0.32928789094578587, 0.3546177287108463, 0.3799475664759068, 0.4052774042409672, 0.4306072420060277, 0.45593707977108816, 0.48126691753614853, 0.5065967553012091, 0.5319265930662695, 0.5572564308313299, 0.5825862685963904, 0.6079161063614509, 0.6332459441265113, 0.6585757818915717, 0.6839056196566322, 0.7092354574216926, 0.7345652951867531, 0.7598951329518135, 0.7852249707168739, 0.8105548084819344, 0.8358846462469949, 0.8612144840120554, 0.8865443217771158, 0.9118741595421763, 0.9372039973072366, 0.9625338350722971, 0.9878636728373575, 1.0131935106024181, 1.0385233483674785, 1.063853186132539, 1.0891830238975995, 1.1145128616626598, 1.1398426994277202, 1.1651725371927808, 1.1905023749578412, 1.2158322127229018, 1.2411620504879621, 1.2664918882530225, 1.291821726018083, 1.3171515637831435, 1.3424814015482038, 1.3678112393132644, 1.3931410770783248, 1.4184709148433852, 1.4438007526084458, 1.4691305903735061, 1.4944604281385667, 1.519790265903627, 1.5451201036686875, 1.5704499414337478, 1.5957797791988084, 1.6211096169638688, 1.6464394547289294, 1.6717692924939898, 1.6970991302590501, 1.7224289680241107, 1.747758805789171, 1.7730886435542317, 1.798418481319292, 1.8237483190843526, 1.849078156849413, 1.8744079946144732, 1.899737832379534, 1.9250676701445941, 1.950397507909655, 1.975727345674715, 2.0010571834397757, 2.0263870212048363, 2.0517168589698964, 2.077046696734957, 2.102376534500017, 2.127706372265078, 2.1530362100301383, 2.178366047795199, 2.203695885560259, 2.2290257233253197, 2.2543555610903803, 2.2796853988554404, 2.305015236620501, 2.3303450743855616, 2.355674912150622, 2.3810047499156823, 2.406334587680743, 2.4316644254458035, 2.4569942632108637, 2.4823241009759243, 2.5076539387409844, 2.532983776506045 ], [ 0, 0.024270509831248423, 0.04854101966249685, 0.07281152949374527, 0.0970820393249937, 0.1213525491562421, 0.14562305898749053, 0.16989356881873896, 0.1941640786499874, 0.21843458848123581, 0.2427050983124842, 0.2669756081437326, 0.29124611797498107, 0.3155166278062295, 0.3397871376374779, 0.3640576474687263, 0.3883281572999748, 0.41259866713122323, 0.43686917696247163, 0.46113968679372, 0.4854101966249684, 0.5096807064562169, 0.5339512162874652, 0.5582217261187137, 0.5824922359499621, 0.6067627457812106, 0.631033255612459, 0.6553037654437074, 0.6795742752749558, 0.7038447851062043, 0.7281152949374526, 0.7523858047687011, 0.7766563145999495, 0.800926824431198, 0.8251973342624465, 0.8494678440936949, 0.8737383539249433, 0.8980088637561916, 0.92227937358744, 0.9465498834186885, 0.9708203932499369, 0.9950909030811853, 1.0193614129124338, 1.0436319227436823, 1.0679024325749304, 1.092172942406179, 1.1164434522374274, 1.140713962068676, 1.1649844718999243, 1.1892549817311728, 1.2135254915624212, 1.2377960013936695, 1.262066511224918, 1.2863370210561664, 1.3106075308874148, 1.3348780407186631, 1.3591485505499117, 1.38341906038116, 1.4076895702124086, 1.431960080043657, 1.4562305898749053, 1.4805010997061536, 1.5047716095374022, 1.5290421193686505, 1.553312629199899, 1.5775831390311474, 1.601853648862396, 1.6261241586936441, 1.650394668524893, 1.674665178356141, 1.6989356881873898, 1.723206198018638, 1.7474767078498865, 1.7717472176811349, 1.7960177275123832, 1.8202882373436318, 1.84455874717488, 1.8688292570061287, 1.893099766837377, 1.9173702766686256, 1.9416407864998737, 1.965911296331122, 1.9901818061623706, 2.014452315993619, 2.0387228258248675, 2.062993335656116, 2.0872638454873647, 2.1115343553186126, 2.135804865149861, 2.1600753749811097, 2.184345884812358, 2.2086163946436064, 2.2328869044748547, 2.2571574143061035, 2.281427924137352, 2.3056984339686, 2.3299689437998485, 2.354239453631097, 2.3785099634623457, 2.4027804732935936, 2.4270509831248424 ], [ 0, 0.023115397283273675, 0.04623079456654735, 0.06934619184982102, 0.0924615891330947, 0.11557698641636836, 0.13869238369964204, 0.1618077809829157, 0.1849231782661894, 0.2080385755494631, 0.23115397283273673, 0.25426937011601036, 0.2773847673992841, 0.3005001646825578, 0.3236155619658314, 0.3467309592491051, 0.3698463565323788, 0.39296175381565246, 0.4160771510989262, 0.4391925483821998, 0.46230794566547345, 0.48542334294874717, 0.5085387402320207, 0.5316541375152944, 0.5547695347985682, 0.5778849320818419, 0.6010003293651156, 0.6241157266483892, 0.6472311239316628, 0.6703465212149365, 0.6934619184982102, 0.7165773157814839, 0.7396927130647576, 0.7628081103480312, 0.7859235076313049, 0.8090389049145786, 0.8321543021978524, 0.8552696994811259, 0.8783850967643996, 0.9015004940476733, 0.9246158913309469, 0.9477312886142206, 0.9708466858974943, 0.9939620831807681, 1.0170774804640415, 1.0401928777473153, 1.063308275030589, 1.0864236723138627, 1.1095390695971363, 1.13265446688041, 1.1557698641636838, 1.1788852614469574, 1.2020006587302312, 1.2251160560135046, 1.2482314532967784, 1.271346850580052, 1.2944622478633256, 1.3175776451465995, 1.340693042429873, 1.3638084397131467, 1.3869238369964203, 1.410039234279694, 1.4331546315629677, 1.4562700288462413, 1.4793854261295152, 1.5025008234127888, 1.5256162206960624, 1.548731617979336, 1.5718470152626098, 1.5949624125458834, 1.6180778098291573, 1.6411932071124309, 1.6643086043957047, 1.687424001678978, 1.7105393989622517, 1.7336547962455255, 1.7567701935287992, 1.779885590812073, 1.8030009880953466, 1.8261163853786204, 1.8492317826618938, 1.8723471799451674, 1.8954625772284412, 1.9185779745117149, 1.9416933717949887, 1.964808769078262, 1.9879241663615361, 2.0110395636448097, 2.034154960928083, 2.057270358211357, 2.0803857554946306, 2.103501152777904, 2.126616550061178, 2.149731947344452, 2.1728473446277254, 2.195962741910999, 2.2190781391942727, 2.2421935364775463, 2.26530893376082, 2.2884243310440935, 2.3115397283273675 ], [ 0, 0.021869058822642347, 0.043738117645284694, 0.06560717646792703, 0.08747623529056939, 0.10934529411321173, 0.13121435293585407, 0.1530834117584964, 0.17495247058113877, 0.19682152940378114, 0.21869058822642345, 0.2405596470490658, 0.26242870587170813, 0.28429776469435053, 0.3061668235169928, 0.32803588233963515, 0.34990494116227755, 0.3717739999849199, 0.3936430588075623, 0.41551211763020457, 0.4373811764528469, 0.4592502352754893, 0.4811192940981316, 0.5029883529207739, 0.5248574117434163, 0.5467264705660586, 0.5685955293887011, 0.5904645882113433, 0.6123336470339856, 0.6342027058566281, 0.6560717646792703, 0.6779408235019126, 0.6998098823245551, 0.7216789411471974, 0.7435479999698398, 0.7654170587924821, 0.7872861176151246, 0.8091551764377667, 0.8310242352604091, 0.8528932940830515, 0.8747623529056938, 0.8966314117283362, 0.9185004705509786, 0.940369529373621, 0.9622385881962632, 0.9841076470189055, 1.0059767058415479, 1.0278457646641903, 1.0497148234868325, 1.071583882309475, 1.0934529411321172, 1.1153219999547597, 1.1371910587774021, 1.1590601176000443, 1.1809291764226866, 1.202798235245329, 1.2246672940679713, 1.2465363528906137, 1.2684054117132562, 1.2902744705358984, 1.3121435293585406, 1.334012588181183, 1.3558816470038253, 1.3777507058264677, 1.3996197646491102, 1.4214888234717524, 1.4433578822943949, 1.465226941117037, 1.4870959999396796, 1.5089650587623218, 1.5308341175849642, 1.5527031764076065, 1.5745722352302491, 1.5964412940528914, 1.6183103528755334, 1.640179411698176, 1.6620484705208183, 1.6839175293434607, 1.705786588166103, 1.7276556469887454, 1.7495247058113876, 1.7713937646340299, 1.7932628234566723, 1.8151318822793145, 1.8370009411019572, 1.8588699999245992, 1.880739058747242, 1.9026081175698841, 1.9244771763925264, 1.9463462352151688, 1.968215294037811, 1.9900843528604535, 2.0119534116830957, 2.033822470505738, 2.0556915293283806, 2.077560588151023, 2.099429646973665, 2.1212987057963075, 2.14316776461895, 2.165036823441592, 2.1869058822642344 ], [ 0, 0.02053641317786066, 0.04107282635572132, 0.06160923953358198, 0.08214565271144264, 0.1026820658893033, 0.12321847906716396, 0.14375489224502463, 0.16429130542288528, 0.18482771860074595, 0.2053641317786066, 0.22590054495646725, 0.24643695813432792, 0.2669733713121886, 0.28750978449004927, 0.3080461976679099, 0.32858261084577056, 0.34911902402363126, 0.3696554372014919, 0.39019185037935256, 0.4107282635572132, 0.4312646767350739, 0.4518010899129345, 0.4723375030907952, 0.49287391626865584, 0.5134103294465165, 0.5339467426243772, 0.5544831558022378, 0.5750195689800985, 0.5955559821579592, 0.6160923953358198, 0.6366288085136804, 0.6571652216915411, 0.6777016348694018, 0.6982380480472625, 0.7187744612251232, 0.7393108744029838, 0.7598472875808444, 0.7803837007587051, 0.8009201139365658, 0.8214565271144264, 0.8419929402922871, 0.8625293534701478, 0.8830657666480085, 0.903602179825869, 0.9241385930037297, 0.9446750061815904, 0.9652114193594511, 0.9857478325373117, 1.0062842457151724, 1.026820658893033, 1.0473570720708938, 1.0678934852487545, 1.088429898426615, 1.1089663116044757, 1.1295027247823364, 1.150039137960197, 1.1705755511380578, 1.1911119643159185, 1.211648377493779, 1.2321847906716397, 1.2527212038495004, 1.2732576170273608, 1.2937940302052215, 1.3143304433830822, 1.334866856560943, 1.3554032697388037, 1.3759396829166641, 1.396476096094525, 1.4170125092723855, 1.4375489224502465, 1.458085335628107, 1.4786217488059676, 1.4991581619838283, 1.5196945751616888, 1.5402309883395495, 1.5607674015174102, 1.581303814695271, 1.6018402278731316, 1.6223766410509923, 1.6429130542288528, 1.6634494674067135, 1.6839858805845742, 1.7045222937624347, 1.7250587069402956, 1.745595120118156, 1.766131533296017, 1.7866679464738775, 1.807204359651738, 1.827740772829599, 1.8482771860074594, 1.8688135991853203, 1.8893500123631808, 1.9098864255410415, 1.9304228387189022, 1.950959251896763, 1.9714956650746234, 1.992032078252484, 2.0125684914303448, 2.0331049046082055, 2.053641317786066 ], [ 0, 0.01912271969246069, 0.03824543938492138, 0.05736815907738207, 0.07649087876984276, 0.09561359846230344, 0.11473631815476414, 0.13385903784722483, 0.15298175753968551, 0.17210447723214622, 0.19122719692460688, 0.21034991661706756, 0.22947263630952827, 0.24859535600198895, 0.26771807569444966, 0.2868407953869103, 0.30596351507937103, 0.32508623477183174, 0.34420895446429245, 0.36333167415675305, 0.38245439384921376, 0.40157711354167447, 0.4206998332341351, 0.43982255292659583, 0.45894527261905654, 0.4780679923115172, 0.4971907120039779, 0.5163134316964386, 0.5354361513888993, 0.5545588710813599, 0.5736815907738206, 0.5928043104662813, 0.6119270301587421, 0.6310497498512028, 0.6501724695436635, 0.6692951892361242, 0.6884179089285849, 0.7075406286210454, 0.7266633483135061, 0.7457860680059668, 0.7649087876984275, 0.7840315073908882, 0.8031542270833489, 0.8222769467758096, 0.8413996664682702, 0.860522386160731, 0.8796451058531917, 0.8987678255456524, 0.9178905452381131, 0.9370132649305738, 0.9561359846230344, 0.9752587043154951, 0.9943814240079558, 1.0135041437004164, 1.0326268633928772, 1.0517495830853378, 1.0708723027777987, 1.0899950224702593, 1.1091177421627199, 1.1282404618551807, 1.1473631815476413, 1.1664859012401019, 1.1856086209325627, 1.2047313406250233, 1.2238540603174841, 1.2429767800099447, 1.2620994997024055, 1.2812222193948661, 1.300344939087327, 1.3194676587797873, 1.3385903784722484, 1.3577130981647088, 1.3768358178571698, 1.3959585375496302, 1.4150812572420908, 1.4342039769345516, 1.4533266966270122, 1.472449416319473, 1.4915721360119336, 1.5106948557043944, 1.529817575396855, 1.5489402950893156, 1.5680630147817765, 1.587185734474237, 1.6063084541666979, 1.6254311738591585, 1.6445538935516193, 1.66367661324408, 1.6827993329365405, 1.7019220526290013, 1.721044772321462, 1.7401674920139227, 1.7592902117063833, 1.7784129313988442, 1.7975356510913048, 1.8166583707837656, 1.8357810904762262, 1.8549038101686866, 1.8740265298611476, 1.893149249553608, 1.9122719692460688 ], [ 0, 0.017633557568774192, 0.035267115137548384, 0.05290067270632258, 0.07053423027509677, 0.08816778784387097, 0.10580134541264516, 0.12343490298141935, 0.14106846055019354, 0.15870201811896775, 0.17633557568774194, 0.1939691332565161, 0.21160269082529032, 0.22923624839406453, 0.2468698059628387, 0.2645033635316129, 0.2821369211003871, 0.2997704786691613, 0.3174040362379355, 0.33503759380670967, 0.3526711513754839, 0.3703047089442581, 0.3879382665130322, 0.4055718240818064, 0.42320538165058064, 0.44083893921935485, 0.45847249678812907, 0.47610605435690323, 0.4937396119256774, 0.5113731694944517, 0.5290067270632258, 0.546640284632, 0.5642738422007741, 0.5819073997695484, 0.5995409573383226, 0.6171745149070968, 0.634808072475871, 0.6524416300446451, 0.6700751876134193, 0.6877087451821935, 0.7053423027509678, 0.7229758603197419, 0.7406094178885162, 0.7582429754572904, 0.7758765330260644, 0.7935100905948387, 0.8111436481636128, 0.8287772057323871, 0.8464107633011613, 0.8640443208699355, 0.8816778784387097, 0.8993114360074839, 0.9169449935762581, 0.9345785511450322, 0.9522121087138065, 0.9698456662825806, 0.9874792238513548, 1.005112781420129, 1.0227463389889033, 1.0403798965576774, 1.0580134541264516, 1.0756470116952257, 1.093280569264, 1.1109141268327742, 1.1285476844015483, 1.1461812419703226, 1.1638147995390968, 1.1814483571078709, 1.1990819146766452, 1.2167154722454192, 1.2343490298141937, 1.2519825873829677, 1.269616144951742, 1.287249702520516, 1.3048832600892901, 1.3225168176580646, 1.3401503752268387, 1.357783932795613, 1.375417490364387, 1.3930510479331615, 1.4106846055019355, 1.4283181630707096, 1.4459517206394839, 1.463585278208258, 1.4812188357770324, 1.4988523933458064, 1.5164859509145807, 1.5341195084833548, 1.5517530660521288, 1.5693866236209033, 1.5870201811896774, 1.6046537387584516, 1.6222872963272257, 1.6399208538960002, 1.6575544114647742, 1.6751879690335485, 1.6928215266023225, 1.7104550841710966, 1.728088641739871, 1.7457221993086451, 1.7633557568774194 ], [ 0, 0.0160748038493699, 0.0321496076987398, 0.0482244115481097, 0.0642992153974796, 0.08037401924684949, 0.0964488230962194, 0.11252362694558929, 0.1285984307949592, 0.1446732346443291, 0.16074803849369898, 0.1768228423430689, 0.1928976461924388, 0.2089724500418087, 0.22504725389117858, 0.24112205774054848, 0.2571968615899184, 0.2732716654392883, 0.2893464692886582, 0.3054212731380281, 0.32149607698739796, 0.3375708808367679, 0.3536456846861378, 0.36972048853550765, 0.3857952923848776, 0.4018700962342475, 0.4179449000836174, 0.4340197039329873, 0.45009450778235716, 0.4661693116317271, 0.48224411548109697, 0.49831891933046685, 0.5143937231798368, 0.5304685270292067, 0.5465433308785766, 0.5626181347279465, 0.5786929385773164, 0.5947677424266862, 0.6108425462760562, 0.626917350125426, 0.6429921539747959, 0.6590669578241659, 0.6751417616735358, 0.6912165655229057, 0.7072913693722755, 0.7233661732216454, 0.7394409770710153, 0.7555157809203853, 0.7715905847697552, 0.787665388619125, 0.803740192468495, 0.8198149963178649, 0.8358898001672348, 0.8519646040166046, 0.8680394078659746, 0.8841142117153444, 0.9001890155647143, 0.9162638194140843, 0.9323386232634542, 0.9484134271128241, 0.9644882309621939, 0.9805630348115638, 0.9966378386609337, 1.0127126425103037, 1.0287874463596736, 1.0448622502090434, 1.0609370540584133, 1.0770118579077832, 1.0930866617571533, 1.109161465606523, 1.125236269455893, 1.1413110733052627, 1.1573858771546328, 1.1734606810040027, 1.1895354848533723, 1.2056102887027424, 1.2216850925521123, 1.2377598964014822, 1.253834700250852, 1.2699095041002222, 1.2859843079495918, 1.3020591117989617, 1.3181339156483318, 1.3342087194977015, 1.3502835233470716, 1.3663583271964415, 1.3824331310458113, 1.3985079348951812, 1.414582738744551, 1.430657542593921, 1.4467323464432909, 1.462807150292661, 1.4788819541420306, 1.4949567579914007, 1.5110315618407706, 1.5271063656901405, 1.5431811695395103, 1.55925597338888, 1.57533077723825, 1.59140558108762, 1.60748038493699 ], [ 0, 0.014452610223051455, 0.02890522044610291, 0.04335783066915437, 0.05781044089220582, 0.07226305111525728, 0.08671566133830874, 0.10116827156136018, 0.11562088178441164, 0.13007349200746313, 0.14452610223051457, 0.158978712453566, 0.17343132267661748, 0.18788393289966893, 0.20233654312272037, 0.2167891533457718, 0.23124176356882328, 0.24569437379187475, 0.26014698401492625, 0.27459959423797764, 0.28905220446102914, 0.3035048146840806, 0.317957424907132, 0.33241003513018347, 0.34686264535323497, 0.3613152555762864, 0.37576786579933785, 0.3902204760223893, 0.40467308624544074, 0.41912569646849224, 0.4335783066915436, 0.4480309169145951, 0.46248352713764657, 0.47693613736069806, 0.4913887475837495, 0.505841357806801, 0.5202939680298525, 0.5347465782529038, 0.5491991884759553, 0.5636517986990067, 0.5781044089220583, 0.5925570191451097, 0.6070096293681612, 0.6214622395912126, 0.635914849814264, 0.6503674600373155, 0.6648200702603669, 0.6792726804834184, 0.6937252907064699, 0.7081779009295214, 0.7226305111525728, 0.7370831213756243, 0.7515357315986757, 0.7659883418217271, 0.7804409520447786, 0.79489356226783, 0.8093461724908815, 0.823798782713933, 0.8382513929369845, 0.8527040031600359, 0.8671566133830872, 0.8816092236061388, 0.8960618338291902, 0.9105144440522417, 0.9249670542752931, 0.9394196644983447, 0.9538722747213961, 0.9683248849444475, 0.982777495167499, 0.9972301053905505, 1.011682715613602, 1.0261353258366535, 1.040587936059705, 1.0550405462827563, 1.0694931565058077, 1.0839457667288592, 1.0983983769519106, 1.1128509871749621, 1.1273035973980134, 1.141756207621065, 1.1562088178441166, 1.1706614280671679, 1.1851140382902194, 1.1995666485132708, 1.2140192587363223, 1.2284718689593737, 1.2429244791824252, 1.2573770894054765, 1.271829699628528, 1.2862823098515797, 1.300734920074631, 1.3151875302976825, 1.3296401405207339, 1.3440927507437854, 1.3585453609668368, 1.3729979711898883, 1.3874505814129399, 1.4019031916359912, 1.4163558018590428, 1.430808412082094, 1.4452610223051456 ], [ 0, 0.012773378746952179, 0.025546757493904358, 0.03832013624085654, 0.051093514987808716, 0.0638668937347609, 0.07664027248171308, 0.08941365122866525, 0.10218702997561743, 0.11496040872256963, 0.1277337874695218, 0.14050716621647397, 0.15328054496342616, 0.16605392371037833, 0.1788273024573305, 0.19160068120428267, 0.20437405995123487, 0.21714743869818706, 0.22992081744513926, 0.2426941961920914, 0.2554675749390436, 0.26824095368599576, 0.28101433243294793, 0.2937877111799001, 0.3065610899268523, 0.3193344686738045, 0.33210784742075666, 0.34488122616770883, 0.357654604914661, 0.3704279836616132, 0.38320136240856534, 0.39597474115551756, 0.40874811990246973, 0.42152149864942196, 0.4342948773963741, 0.4470682561433263, 0.4598416348902785, 0.4726150136372306, 0.4853883923841828, 0.49816177113113497, 0.5109351498780872, 0.5237085286250394, 0.5364819073719915, 0.5492552861189437, 0.5620286648658959, 0.574802043612848, 0.5875754223598002, 0.6003488011067524, 0.6131221798537047, 0.6258955586006568, 0.638668937347609, 0.6514423160945612, 0.6642156948415133, 0.6769890735884655, 0.6897624523354177, 0.7025358310823698, 0.715309209829322, 0.7280825885762743, 0.7408559673232264, 0.7536293460701786, 0.7664027248171307, 0.779176103564083, 0.7919494823110351, 0.8047228610579873, 0.8174962398049395, 0.8302696185518916, 0.8430429972988439, 0.855816376045796, 0.8685897547927482, 0.8813631335397003, 0.8941365122866526, 0.9069098910336048, 0.919683269780557, 0.9324566485275091, 0.9452300272744611, 0.9580034060214135, 0.9707767847683656, 0.9835501635153179, 0.9963235422622699, 1.0090969210092222, 1.0218702997561744, 1.0346436785031266, 1.0474170572500787, 1.0601904359970309, 1.072963814743983, 1.0857371934909352, 1.0985105722378874, 1.1112839509848396, 1.1240573297317917, 1.136830708478744, 1.149604087225696, 1.1623774659726485, 1.1751508447196004, 1.1879242234665528, 1.2006976022135047, 1.2134709809604571, 1.2262443597074093, 1.2390177384543613, 1.2517911172013136, 1.2645644959482656, 1.277337874695218 ], [ 0, 0.011043736580540335, 0.02208747316108067, 0.03313120974162101, 0.04417494632216134, 0.055218682902701675, 0.06626241948324202, 0.07730615606378236, 0.08834989264432268, 0.09939362922486303, 0.11043736580540335, 0.12148110238594367, 0.13252483896648404, 0.14356857554702437, 0.1546123121275647, 0.16565604870810502, 0.17669978528864536, 0.18774352186918572, 0.19878725844972606, 0.20983099503026637, 0.2208747316108067, 0.23191846819134707, 0.24296220477188735, 0.2540059413524277, 0.2650496779329681, 0.2760934145135084, 0.28713715109404875, 0.29818088767458906, 0.3092246242551294, 0.32026836083566973, 0.33131209741621004, 0.3423558339967504, 0.3533995705772907, 0.3644433071578311, 0.37548704373837144, 0.38653078031891175, 0.3975745168994521, 0.40861825347999237, 0.41966199006053273, 0.4307057266410731, 0.4417494632216134, 0.45279319980215377, 0.46383693638269413, 0.47488067296323444, 0.4859244095437747, 0.49696814612431506, 0.5080118827048554, 0.5190556192853958, 0.5300993558659362, 0.5411430924464764, 0.5521868290270168, 0.5632305656075571, 0.5742743021880975, 0.5853180387686377, 0.5963617753491781, 0.6074055119297185, 0.6184492485102588, 0.6294929850907991, 0.6405367216713395, 0.6515804582518798, 0.6626241948324201, 0.6736679314129604, 0.6847116679935008, 0.6957554045740412, 0.7067991411545814, 0.7178428777351218, 0.7288866143156622, 0.7399303508962024, 0.7509740874767429, 0.7620178240572831, 0.7730615606378235, 0.7841052972183639, 0.7951490337989042, 0.8061927703794445, 0.8172365069599847, 0.8282802435405252, 0.8393239801210655, 0.8503677167016059, 0.8614114532821462, 0.8724551898626866, 0.8834989264432268, 0.8945426630237671, 0.9055863996043075, 0.9166301361848478, 0.9276738727653883, 0.9387176093459285, 0.9497613459264689, 0.9608050825070091, 0.9718488190875494, 0.9828925556680899, 0.9939362922486301, 1.0049800288291706, 1.0160237654097108, 1.0270675019902513, 1.0381112385707916, 1.049154975151332, 1.0601987117318723, 1.0712424483124126, 1.0822861848929528, 1.093329921473493, 1.1043736580540335 ], [ 0, 0.009270509831248424, 0.018541019662496848, 0.02781152949374527, 0.037082039324993696, 0.046352549156242116, 0.05562305898749054, 0.06489356881873896, 0.07416407864998739, 0.08343458848123582, 0.09270509831248423, 0.10197560814373265, 0.11124611797498107, 0.12051662780622951, 0.12978713763747793, 0.13905764746872634, 0.14832815729997478, 0.1575986671312232, 0.16686917696247164, 0.17613968679372002, 0.18541019662496847, 0.1946807064562169, 0.2039512162874653, 0.21322172611871373, 0.22249223594996215, 0.2317627457812106, 0.24103325561245903, 0.2503037654437074, 0.25957427527495586, 0.2688447851062043, 0.2781152949374527, 0.2873858047687011, 0.29665631459994957, 0.30592682443119795, 0.3151973342624464, 0.32446784409369483, 0.3337383539249433, 0.3430088637561916, 0.35227937358744005, 0.3615498834186885, 0.37082039324993693, 0.38009090308118537, 0.3893614129124338, 0.3986319227436822, 0.4079024325749306, 0.417172942406179, 0.42644345223742747, 0.4357139620686759, 0.4449844718999243, 0.45425498173117274, 0.4635254915624212, 0.4727960013936696, 0.48206651122491806, 0.4913370210561664, 0.5006075308874148, 0.5098780407186633, 0.5191485505499117, 0.5284190603811602, 0.5376895702124086, 0.546960080043657, 0.5562305898749054, 0.5655010997061538, 0.5747716095374023, 0.5840421193686507, 0.5933126291998991, 0.6025831390311475, 0.6118536488623959, 0.6211241586936443, 0.6303946685248928, 0.6396651783561412, 0.6489356881873897, 0.658206198018638, 0.6674767078498866, 0.6767472176811349, 0.6860177275123832, 0.6952882373436318, 0.7045587471748801, 0.7138292570061286, 0.723099766837377, 0.7323702766686255, 0.7416407864998739, 0.7509112963311222, 0.7601818061623707, 0.7694523159936191, 0.7787228258248676, 0.787993335656116, 0.7972638454873644, 0.8065343553186128, 0.8158048651498612, 0.8250753749811097, 0.834345884812358, 0.8436163946436065, 0.8528869044748549, 0.8621574143061034, 0.8714279241373518, 0.8806984339686003, 0.8899689437998486, 0.899239453631097, 0.9085099634623455, 0.9177804732935938, 0.9270509831248424 ], [ 0, 0.007460696614945642, 0.014921393229891285, 0.022382089844836927, 0.02984278645978257, 0.03730348307472821, 0.044764179689673854, 0.05222487630461949, 0.05968557291956514, 0.06714626953451078, 0.07460696614945642, 0.08206766276440206, 0.08952835937934771, 0.09698905599429335, 0.10444975260923899, 0.11191044922418462, 0.11937114583913028, 0.12683184245407592, 0.13429253906902155, 0.1417532356839672, 0.14921393229891283, 0.1566746289138585, 0.1641353255288041, 0.17159602214374975, 0.17905671875869542, 0.18651741537364105, 0.1939781119885867, 0.20143880860353233, 0.20889950521847797, 0.2163602018334236, 0.22382089844836925, 0.2312815950633149, 0.23874229167826055, 0.2462029882932062, 0.25366368490815183, 0.26112438152309747, 0.2685850781380431, 0.27604577475298875, 0.2835064713679344, 0.29096716798288, 0.29842786459782566, 0.3058885612127713, 0.313349257827717, 0.32080995444266264, 0.3282706510576082, 0.33573134767255386, 0.3431920442874995, 0.35065274090244514, 0.35811343751739083, 0.36557413413233647, 0.3730348307472821, 0.38049552736222775, 0.3879562239771734, 0.395416920592119, 0.40287761720706466, 0.4103383138220103, 0.41779901043695594, 0.4252597070519016, 0.4327204036668472, 0.4401811002817929, 0.4476417968967385, 0.45510249351168414, 0.4625631901266298, 0.4700238867415754, 0.4774845833565211, 0.48494527997146675, 0.4924059765864124, 0.49986667320135797, 0.5073273698163037, 0.5147880664312493, 0.5222487630461949, 0.5297094596611406, 0.5371701562760862, 0.5446308528910319, 0.5520915495059775, 0.5595522461209231, 0.5670129427358688, 0.5744736393508144, 0.58193433596576, 0.5893950325807058, 0.5968557291956513, 0.604316425810597, 0.6117771224255426, 0.6192378190404882, 0.626698515655434, 0.6341592122703795, 0.6416199088853253, 0.6490806055002708, 0.6565413021152164, 0.6640019987301622, 0.6714626953451077, 0.6789233919600535, 0.686384088574999, 0.6938447851899447, 0.7013054818048903, 0.708766178419836, 0.7162268750347817, 0.7236875716497272, 0.7311482682646729, 0.7386089648796185, 0.7460696614945642 ], [ 0, 0.0056214394375717356, 0.011242878875143471, 0.016864318312715208, 0.022485757750286942, 0.028107197187858677, 0.033728636625430415, 0.03935007606300215, 0.044971515500573885, 0.050592954938145626, 0.056214394375717354, 0.06183583381328908, 0.06745727325086083, 0.07307871268843256, 0.0787001521260043, 0.08432159156357602, 0.08994303100114777, 0.0955644704387195, 0.10118590987629125, 0.10680734931386296, 0.11242878875143471, 0.11805022818900644, 0.12367166762657816, 0.1292931070641499, 0.13491454650172166, 0.1405359859392934, 0.14615742537686513, 0.15177886481443684, 0.1574003042520086, 0.16302174368958033, 0.16864318312715204, 0.1742646225647238, 0.17988606200229554, 0.18550750143986727, 0.191128940877439, 0.19675038031501074, 0.2023718197525825, 0.20799325919015418, 0.21361469862772592, 0.21923613806529768, 0.22485757750286942, 0.23047901694044115, 0.23610045637801289, 0.24172189581558465, 0.24734333525315633, 0.2529647746907281, 0.2585862141282998, 0.26420765356587156, 0.2698290930034433, 0.27545053244101503, 0.2810719718785868, 0.2866934113161585, 0.29231485075373026, 0.29793629019130197, 0.3035577296288737, 0.30917916906644544, 0.3148006085040172, 0.3204220479415889, 0.32604348737916067, 0.3316649268167324, 0.3372863662543041, 0.34290780569187584, 0.3485292451294476, 0.3541506845670193, 0.3597721240045911, 0.3653935634421628, 0.37101500287973455, 0.37663644231730625, 0.382257881754878, 0.3878793211924497, 0.3935007606300215, 0.3991222000675932, 0.404743639505165, 0.41036507894273666, 0.41598651838030837, 0.4216079578178802, 0.42722939725545184, 0.43285083669302366, 0.43847227613059536, 0.4440937155681671, 0.44971515500573883, 0.45533659444331054, 0.4609580338808823, 0.466579473318454, 0.47220091275602577, 0.4778223521935975, 0.4834437916311693, 0.48906523106874095, 0.49468667050631265, 0.5003081099438844, 0.5059295493814562, 0.5115509888190279, 0.5171724282565996, 0.5227938676941715, 0.5284153071317431, 0.5340367465693149, 0.5396581860068866, 0.5452796254444583, 0.5509010648820301, 0.5565225043196018, 0.5621439437571736 ], [ 0, 0.0037599970069291276, 0.007519994013858255, 0.011279991020787384, 0.01503998802771651, 0.01879998503464564, 0.022559982041574767, 0.026319979048503892, 0.03007997605543302, 0.03383997306236215, 0.03759997006929128, 0.0413599670762204, 0.045119964083149534, 0.04887996109007866, 0.052639958097007784, 0.05639995510393691, 0.06015995211086604, 0.06391994911779517, 0.0676799461247243, 0.07143994313165342, 0.07519994013858255, 0.07895993714551168, 0.0827199341524408, 0.08647993115936993, 0.09023992816629907, 0.0939999251732282, 0.09775992218015732, 0.10151991918708644, 0.10527991619401557, 0.10903991320094471, 0.11279991020787382, 0.11655990721480296, 0.12031990422173208, 0.12407990122866122, 0.12783989823559033, 0.13159989524251947, 0.1353598922494486, 0.13911988925637772, 0.14287988626330683, 0.14663988327023597, 0.1503998802771651, 0.15415987728409425, 0.15791987429102336, 0.1616798712979525, 0.1654398683048816, 0.16919986531181072, 0.17295986231873986, 0.176719859325669, 0.18047985633259814, 0.18423985333952725, 0.1879998503464564, 0.19175984735338553, 0.19551984436031464, 0.19927984136724375, 0.2030398383741729, 0.20679983538110203, 0.21055983238803114, 0.21431982939496028, 0.21807982640188942, 0.22183982340881853, 0.22559982041574764, 0.22935981742267678, 0.23311981442960591, 0.23687981143653503, 0.24063980844346416, 0.2443998054503933, 0.24815980245732244, 0.2519197994642515, 0.25567979647118066, 0.2594397934781098, 0.26319979048503894, 0.2669597874919681, 0.2707197844988972, 0.2744797815058263, 0.27823977851275544, 0.2819997755196846, 0.28575977252661366, 0.28951976953354286, 0.29327976654047194, 0.2970397635474011, 0.3007997605543302, 0.3045597575612593, 0.3083197545681885, 0.3120797515751176, 0.3158397485820467, 0.31959974558897586, 0.323359742595905, 0.3271197396028341, 0.3308797366097632, 0.33463973361669236, 0.33839973062362144, 0.34215972763055064, 0.3459197246374797, 0.34967972164440886, 0.353439718651338, 0.35719971565826714, 0.3609597126651963, 0.36471970967212536, 0.3684797066790545, 0.37223970368598364, 0.3759997006929128 ], [ 0, 0.0018837155858793986, 0.003767431171758797, 0.005651146757638196, 0.007534862343517594, 0.009418577929396993, 0.011302293515276392, 0.01318600910115579, 0.015069724687035189, 0.01695344027291459, 0.018837155858793985, 0.020720871444673383, 0.022604587030552785, 0.024488302616432183, 0.02637201820231158, 0.02825573378819098, 0.030139449374070378, 0.03202316495994978, 0.03390688054582918, 0.035790596131708576, 0.03767431171758797, 0.03955802730346737, 0.04144174288934677, 0.04332545847522617, 0.04520917406110557, 0.047092889646984964, 0.048976605232864366, 0.05086032081874376, 0.05274403640462316, 0.054627751990502564, 0.05651146757638196, 0.058395183162261353, 0.060278898748140755, 0.06216261433402016, 0.06404632991989956, 0.06593004550577895, 0.06781376109165836, 0.06969747667753774, 0.07158119226341715, 0.07346490784929655, 0.07534862343517594, 0.07723233902105535, 0.07911605460693474, 0.08099977019281415, 0.08288348577869353, 0.08476720136457293, 0.08665091695045234, 0.08853463253633173, 0.09041834812221114, 0.09230206370809053, 0.09418577929396993, 0.09606949487984934, 0.09795321046572873, 0.09983692605160813, 0.10172064163748752, 0.10360435722336693, 0.10548807280924632, 0.10737178839512572, 0.10925550398100513, 0.11113921956688452, 0.11302293515276392, 0.11490665073864331, 0.11679036632452271, 0.11867408191040212, 0.12055779749628151, 0.12244151308216092, 0.12432522866804031, 0.1262089442539197, 0.12809265983979912, 0.1299763754256785, 0.1318600910115579, 0.13374380659743731, 0.13562752218331672, 0.1375112377691961, 0.13939495335507548, 0.1412786689409549, 0.1431623845268343, 0.1450461001127137, 0.1469298156985931, 0.1488135312844725, 0.15069724687035188, 0.1525809624562313, 0.1544646780421107, 0.15634839362799008, 0.1582321092138695, 0.16011582479974887, 0.1619995403856283, 0.16388325597150769, 0.16576697155738707, 0.16765068714326647, 0.16953440272914586, 0.1714181183150253, 0.17330183390090467, 0.17518554948678408, 0.17706926507266346, 0.17895298065854287, 0.18083669624442228, 0.18272041183030166, 0.18460412741618107, 0.18648784300206045, 0.18837155858793986 ], [ 0, -4.8243679490299094e-18, -9.648735898059819e-18, -1.4473103847089727e-17, -1.9297471796119638e-17, -2.4121839745149548e-17, -2.8946207694179455e-17, -3.377057564320937e-17, -3.8594943592239275e-17, -4.341931154126919e-17, -4.8243679490299096e-17, -5.3068047439329e-17, -5.789241538835891e-17, -6.271678333738883e-17, -6.754115128641874e-17, -7.236551923544864e-17, -7.718988718447855e-17, -8.201425513350847e-17, -8.683862308253838e-17, -9.166299103156828e-17, -9.648735898059819e-17, -1.0131172692962811e-16, -1.06136094878658e-16, -1.1096046282768791e-16, -1.1578483077671782e-16, -1.2060919872574775e-16, -1.2543356667477766e-16, -1.3025793462380757e-16, -1.3508230257283747e-16, -1.3990667052186738e-16, -1.4473103847089729e-16, -1.495554064199272e-16, -1.543797743689571e-16, -1.5920414231798703e-16, -1.6402851026701694e-16, -1.6885287821604685e-16, -1.7367724616507675e-16, -1.7850161411410664e-16, -1.8332598206313657e-16, -1.8815035001216648e-16, -1.9297471796119638e-16, -1.977990859102263e-16, -2.0262345385925622e-16, -2.0744782180828613e-16, -2.12272189757316e-16, -2.1709655770634592e-16, -2.2192092565537582e-16, -2.2674529360440576e-16, -2.3156966155343564e-16, -2.3639402950246557e-16, -2.412183974514955e-16, -2.460427654005254e-16, -2.508671333495553e-16, -2.556915012985852e-16, -2.6051586924761513e-16, -2.65340237196645e-16, -2.7016460514567494e-16, -2.749889730947049e-16, -2.7981334104373476e-16, -2.846377089927647e-16, -2.8946207694179457e-16, -2.9428644489082446e-16, -2.991108128398544e-16, -3.039351807888843e-16, -3.087595487379142e-16, -3.1358391668694413e-16, -3.1840828463597407e-16, -3.232326525850039e-16, -3.280570205340339e-16, -3.3288138848306376e-16, -3.377057564320937e-16, -3.425301243811236e-16, -3.473544923301535e-16, -3.521788602791834e-16, -3.5700322822821327e-16, -3.6182759617724325e-16, -3.6665196412627314e-16, -3.7147633207530307e-16, -3.7630070002433295e-16, -3.811250679733629e-16, -3.8594943592239276e-16, -3.9077380387142265e-16, -3.955981718204526e-16, -4.0042253976948246e-16, -4.0524690771851244e-16, -4.100712756675423e-16, -4.1489564361657226e-16, -4.1972001156560214e-16, -4.24544379514632e-16, -4.2936874746366195e-16, -4.3419311541269184e-16, -4.3901748336172177e-16, -4.4384185131075165e-16, -4.486662192597816e-16, -4.534905872088115e-16, -4.583149551578414e-16, -4.631393231068713e-16, -4.679636910559012e-16, -4.727880590049311e-16, -4.776124269539611e-16, -4.82436794902991e-16 ], [ 0, -0.001883715585879402, -0.003767431171758804, -0.005651146757638206, -0.007534862343517608, -0.00941857792939701, -0.011302293515276412, -0.013186009101155813, -0.015069724687035217, -0.016953440272914618, -0.01883715585879402, -0.02072087144467342, -0.022604587030552823, -0.024488302616432228, -0.026372018202311626, -0.028255733788191028, -0.030139449374070433, -0.032023164959949835, -0.033906880545829236, -0.03579059613170864, -0.03767431171758804, -0.03955802730346744, -0.04144174288934684, -0.043325458475226244, -0.045209174061105646, -0.04709288964698505, -0.048976605232864456, -0.05086032081874385, -0.05274403640462325, -0.05462775199050266, -0.056511467576382056, -0.05839518316226146, -0.060278898748140866, -0.06216261433402027, -0.06404632991989967, -0.06593004550577908, -0.06781376109165847, -0.06969747667753787, -0.07158119226341728, -0.07346490784929667, -0.07534862343517608, -0.07723233902105549, -0.07911605460693488, -0.08099977019281429, -0.08288348577869369, -0.08476720136457308, -0.08665091695045249, -0.0885346325363319, -0.09041834812221129, -0.0923020637080907, -0.0941857792939701, -0.0960694948798495, -0.09795321046572891, -0.0998369260516083, -0.1017206416374877, -0.10360435722336711, -0.1054880728092465, -0.10737178839512591, -0.10925550398100532, -0.11113921956688472, -0.11302293515276411, -0.11490665073864352, -0.11679036632452292, -0.11867408191040232, -0.12055779749628173, -0.12244151308216113, -0.12432522866804054, -0.12620894425391993, -0.12809265983979934, -0.12997637542567872, -0.13186009101155816, -0.13374380659743754, -0.13562752218331695, -0.13751123776919635, -0.13939495335507573, -0.14127866894095514, -0.14316238452683455, -0.14504610011271396, -0.14692981569859334, -0.14881353128447278, -0.15069724687035216, -0.15258096245623154, -0.15446467804211098, -0.15634839362799036, -0.15823210921386976, -0.16011582479974917, -0.16199954038562858, -0.16388325597150796, -0.16576697155738737, -0.16765068714326678, -0.16953440272914616, -0.1714181183150256, -0.17330183390090498, -0.1751855494867844, -0.1770692650726638, -0.1789529806585432, -0.18083669624442258, -0.182720411830302, -0.1846041274161814, -0.18648784300206078, -0.1883715585879402 ], [ 0, -0.003759997006929131, -0.007519994013858262, -0.011279991020787392, -0.015039988027716524, -0.018799985034645656, -0.022559982041574785, -0.026319979048503916, -0.03007997605543305, -0.03383997306236218, -0.03759997006929131, -0.04135996707622044, -0.04511996408314957, -0.04887996109007871, -0.05263995809700783, -0.05639995510393696, -0.0601599521108661, -0.06391994911779524, -0.06767994612472436, -0.07143994313165349, -0.07519994013858262, -0.07895993714551175, -0.08271993415244087, -0.08647993115937001, -0.09023992816629914, -0.09399992517322828, -0.09775992218015742, -0.10151991918708653, -0.10527991619401567, -0.1090399132009448, -0.11279991020787392, -0.11655990721480305, -0.1203199042217322, -0.12407990122866132, -0.12783989823559047, -0.13159989524251958, -0.13535989224944872, -0.13911988925637783, -0.14287988626330697, -0.1466398832702361, -0.15039988027716525, -0.15415987728409436, -0.1579198742910235, -0.16167987129795264, -0.16543986830488175, -0.1691998653118109, -0.17295986231874003, -0.17671985932566914, -0.18047985633259828, -0.18423985333952742, -0.18799985034645655, -0.1917598473533857, -0.19551984436031483, -0.19927984136724394, -0.20303983837417305, -0.2067998353811022, -0.21055983238803133, -0.21431982939496047, -0.2180798264018896, -0.22183982340881875, -0.22559982041574783, -0.22935981742267697, -0.2331198144296061, -0.23687981143653525, -0.2406398084434644, -0.24439980545039353, -0.24815980245732264, -0.25191979946425175, -0.25567979647118094, -0.25943979347811, -0.26319979048503916, -0.2669597874919683, -0.27071978449889744, -0.2744797815058266, -0.27823977851275566, -0.2819997755196848, -0.28575977252661394, -0.2895197695335431, -0.2932797665404722, -0.29703976354740136, -0.3007997605543305, -0.3045597575612596, -0.3083197545681887, -0.31207975157511786, -0.315839748582047, -0.31959974558897614, -0.3233597425959053, -0.3271197396028344, -0.3308797366097635, -0.33463973361669264, -0.3383997306236218, -0.3421597276305509, -0.34591972463748005, -0.3496797216444092, -0.3534397186513383, -0.35719971565826747, -0.36095971266519655, -0.3647197096721257, -0.36847970667905483, -0.37223970368598397, -0.3759997006929131 ], [ 0, -0.005621439437571744, -0.011242878875143488, -0.016864318312715232, -0.022485757750286977, -0.028107197187858722, -0.033728636625430464, -0.03935007606300221, -0.044971515500573954, -0.0505929549381457, -0.056214394375717444, -0.061835833813289186, -0.06745727325086093, -0.07307871268843269, -0.07870015212600442, -0.08432159156357616, -0.08994303100114791, -0.09556447043871966, -0.1011859098762914, -0.10680734931386314, -0.11242878875143489, -0.11805022818900664, -0.12367166762657837, -0.12929310706415012, -0.13491454650172185, -0.14053598593929362, -0.14615742537686538, -0.1517788648144371, -0.15740030425200885, -0.16302174368958058, -0.16864318312715232, -0.17426462256472408, -0.17988606200229582, -0.18550750143986758, -0.1911289408774393, -0.19675038031501108, -0.2023718197525828, -0.20799325919015452, -0.21361469862772628, -0.21923613806529804, -0.22485757750286978, -0.23047901694044154, -0.23610045637801327, -0.24172189581558504, -0.24734333525315674, -0.2529647746907285, -0.25858621412830024, -0.264207653565872, -0.2698290930034437, -0.27545053244101547, -0.28107197187858723, -0.286693411316159, -0.29231485075373076, -0.29793629019130247, -0.3035577296288742, -0.30917916906644594, -0.3148006085040177, -0.32042204794158946, -0.32604348737916117, -0.33166492681673293, -0.33728636625430464, -0.3429078056918764, -0.34852924512944816, -0.3541506845670199, -0.35977212400459163, -0.3653935634421634, -0.37101500287973516, -0.37663644231730686, -0.3822578817548786, -0.38787932119245033, -0.39350076063002215, -0.39912220006759386, -0.4047436395051656, -0.4103650789427373, -0.41598651838030903, -0.42160795781788085, -0.42722939725545256, -0.4328508366930244, -0.4384722761305961, -0.44409371556816785, -0.44971515500573955, -0.45533659444331126, -0.4609580338808831, -0.4665794733184548, -0.47220091275602655, -0.47782235219359825, -0.4834437916311701, -0.4890652310687418, -0.4946866705063135, -0.5003081099438853, -0.505929549381457, -0.5115509888190287, -0.5171724282566005, -0.5227938676941722, -0.528415307131744, -0.5340367465693158, -0.5396581860068874, -0.5452796254444592, -0.5509010648820309, -0.5565225043196027, -0.5621439437571745 ], [ 0, -0.007460696614945645, -0.01492139322989129, -0.022382089844836937, -0.02984278645978258, -0.03730348307472823, -0.044764179689673875, -0.052224876304619514, -0.05968557291956516, -0.06714626953451082, -0.07460696614945646, -0.0820676627644021, -0.08952835937934775, -0.09698905599429339, -0.10444975260923903, -0.11191044922418467, -0.11937114583913032, -0.12683184245407597, -0.13429253906902164, -0.14175323568396725, -0.14921393229891292, -0.15667462891385855, -0.1641353255288042, -0.17159602214374983, -0.1790567187586955, -0.18651741537364114, -0.19397811198858678, -0.20143880860353242, -0.20889950521847805, -0.21636020183342372, -0.22382089844836933, -0.231281595063315, -0.23874229167826064, -0.2462029882932063, -0.25366368490815194, -0.2611243815230976, -0.2685850781380433, -0.27604577475298886, -0.2835064713679345, -0.29096716798288014, -0.29842786459782583, -0.30588856121277147, -0.3133492578277171, -0.32080995444266275, -0.3282706510576084, -0.335731347672554, -0.34319204428749966, -0.3506527409024453, -0.358113437517391, -0.36557413413233664, -0.3730348307472823, -0.3804955273622279, -0.38795622397717355, -0.3954169205921192, -0.40287761720706483, -0.41033831382201047, -0.4177990104369561, -0.4252597070519018, -0.43272040366684744, -0.4401811002817931, -0.44764179689673866, -0.45510249351168436, -0.46256319012663, -0.47002388674157564, -0.4774845833565213, -0.48494527997146697, -0.4924059765864126, -0.4998666732013582, -0.5073273698163039, -0.5147880664312495, -0.5222487630461952, -0.5297094596611408, -0.5371701562760866, -0.5446308528910321, -0.5520915495059777, -0.5595522461209235, -0.567012942735869, -0.5744736393508147, -0.5819343359657603, -0.589395032580706, -0.5968557291956517, -0.6043164258105972, -0.6117771224255429, -0.6192378190404885, -0.6266985156554342, -0.6341592122703799, -0.6416199088853255, -0.6490806055002711, -0.6565413021152168, -0.6640019987301624, -0.671462695345108, -0.6789233919600537, -0.6863840885749993, -0.6938447851899451, -0.7013054818048906, -0.7087661784198364, -0.716226875034782, -0.7236875716497275, -0.7311482682646733, -0.7386089648796188, -0.7460696614945646 ], [ 0, -0.009270509831248426, -0.01854101966249685, -0.02781152949374528, -0.0370820393249937, -0.04635254915624213, -0.05562305898749056, -0.06489356881873899, -0.0741640786499874, -0.08343458848123585, -0.09270509831248426, -0.10197560814373269, -0.11124611797498112, -0.12051662780622956, -0.12978713763747798, -0.1390576474687264, -0.1483281572999748, -0.15759866713122325, -0.1668691769624717, -0.1761396867937201, -0.18541019662496852, -0.19468070645621696, -0.20395121628746538, -0.2132217261187138, -0.22249223594996223, -0.23176274578121067, -0.2410332556124591, -0.2503037654437075, -0.25957427527495597, -0.26884478510620435, -0.2781152949374528, -0.28738580476870124, -0.2966563145999496, -0.30592682443119806, -0.3151973342624465, -0.32446784409369495, -0.3337383539249434, -0.3430088637561918, -0.3522793735874402, -0.3615498834186886, -0.37082039324993704, -0.3800909030811855, -0.3893614129124339, -0.39863192274368237, -0.40790243257493075, -0.4171729424061792, -0.4264434522374276, -0.435713962068676, -0.44498447189992446, -0.4542549817311729, -0.46352549156242134, -0.4727960013936698, -0.4820665112249182, -0.49133702105616656, -0.500607530887415, -0.5098780407186635, -0.5191485505499119, -0.5284190603811603, -0.5376895702124087, -0.5469600800436571, -0.5562305898749056, -0.565501099706154, -0.5747716095374025, -0.5840421193686509, -0.5933126291998992, -0.6025831390311477, -0.6118536488623961, -0.6211241586936446, -0.630394668524893, -0.6396651783561415, -0.6489356881873899, -0.6582061980186382, -0.6674767078498868, -0.6767472176811351, -0.6860177275123835, -0.695288237343632, -0.7045587471748804, -0.7138292570061289, -0.7230997668373772, -0.7323702766686258, -0.7416407864998741, -0.7509112963311225, -0.760181806162371, -0.7694523159936194, -0.7787228258248678, -0.7879933356561162, -0.7972638454873647, -0.8065343553186131, -0.8158048651498615, -0.8250753749811099, -0.8343458848123584, -0.8436163946436068, -0.8528869044748552, -0.8621574143061037, -0.871427924137352, -0.8806984339686006, -0.8899689437998489, -0.8992394536310974, -0.9085099634623458, -0.9177804732935941, -0.9270509831248427 ], [ 0, -0.011043736580540338, -0.022087473161080676, -0.033131209741621016, -0.04417494632216135, -0.055218682902701696, -0.06626241948324203, -0.07730615606378237, -0.0883498926443227, -0.09939362922486306, -0.11043736580540339, -0.12148110238594372, -0.13252483896648407, -0.1435685755470244, -0.15461231212756474, -0.16565604870810507, -0.1766997852886454, -0.18774352186918578, -0.1987872584497261, -0.20983099503026642, -0.22087473161080678, -0.23191846819134712, -0.24296220477188743, -0.25400594135242777, -0.26504967793296813, -0.2760934145135085, -0.2871371510940488, -0.2981808876745891, -0.3092246242551295, -0.32026836083566984, -0.33131209741621015, -0.3423558339967505, -0.3533995705772908, -0.3644433071578312, -0.37548704373837155, -0.3865307803189119, -0.3975745168994522, -0.40861825347999253, -0.41966199006053284, -0.4307057266410732, -0.44174946322161357, -0.4527931998021539, -0.46383693638269424, -0.4748806729632346, -0.48592440954377486, -0.4969681461243152, -0.5080118827048555, -0.5190556192853959, -0.5300993558659363, -0.5411430924464766, -0.552186829027017, -0.5632305656075574, -0.5742743021880976, -0.585318038768638, -0.5963617753491782, -0.6074055119297186, -0.618449248510259, -0.6294929850907993, -0.6405367216713397, -0.65158045825188, -0.6626241948324203, -0.6736679314129607, -0.684711667993501, -0.6957554045740414, -0.7067991411545816, -0.717842877735122, -0.7288866143156624, -0.7399303508962026, -0.7509740874767431, -0.7620178240572834, -0.7730615606378238, -0.7841052972183641, -0.7951490337989044, -0.8061927703794447, -0.8172365069599851, -0.8282802435405254, -0.8393239801210657, -0.8503677167016062, -0.8614114532821464, -0.8724551898626869, -0.8834989264432271, -0.8945426630237674, -0.9055863996043078, -0.9166301361848481, -0.9276738727653885, -0.9387176093459287, -0.9497613459264692, -0.9608050825070095, -0.9718488190875497, -0.9828925556680902, -0.9939362922486304, -1.0049800288291708, -1.016023765409711, -1.0270675019902515, -1.0381112385707918, -1.0491549751513323, -1.0601987117318725, -1.0712424483124128, -1.0822861848929533, -1.0933299214734935, -1.104373658054034 ], [ 0, -0.012773378746952183, -0.025546757493904365, -0.03832013624085655, -0.05109351498780873, -0.06386689373476091, -0.0766402724817131, -0.08941365122866528, -0.10218702997561746, -0.11496040872256966, -0.12773378746952183, -0.140507166216474, -0.1532805449634262, -0.1660539237103784, -0.17882730245733056, -0.19160068120428272, -0.20437405995123492, -0.21714743869818712, -0.2299208174451393, -0.24269419619209145, -0.25546757493904365, -0.2682409536859958, -0.281014332432948, -0.2937877111799002, -0.3065610899268524, -0.31933446867380455, -0.3321078474207568, -0.34488122616770894, -0.3576546049146611, -0.37042798366161334, -0.38320136240856545, -0.3959747411555177, -0.40874811990246984, -0.42152149864942207, -0.43429487739637423, -0.4470682561433264, -0.4598416348902786, -0.47261501363723074, -0.4853883923841829, -0.49816177113113513, -0.5109351498780873, -0.5237085286250395, -0.5364819073719916, -0.5492552861189439, -0.562028664865896, -0.5748020436128481, -0.5875754223598004, -0.6003488011067526, -0.6131221798537048, -0.6258955586006569, -0.6386689373476091, -0.6514423160945614, -0.6642156948415135, -0.6769890735884656, -0.6897624523354179, -0.70253583108237, -0.7153092098293222, -0.7280825885762744, -0.7408559673232267, -0.7536293460701788, -0.7664027248171309, -0.7791761035640831, -0.7919494823110353, -0.8047228610579875, -0.8174962398049397, -0.8302696185518919, -0.8430429972988441, -0.8558163760457962, -0.8685897547927485, -0.8813631335397005, -0.8941365122866528, -0.906909891033605, -0.9196832697805573, -0.9324566485275093, -0.9452300272744615, -0.9580034060214138, -0.9707767847683658, -0.9835501635153181, -0.9963235422622703, -1.0090969210092224, -1.0218702997561746, -1.0346436785031268, -1.047417057250079, -1.060190435997031, -1.0729638147439833, -1.0857371934909354, -1.0985105722378878, -1.1112839509848398, -1.124057329731792, -1.1368307084787443, -1.1496040872256963, -1.1623774659726487, -1.1751508447196009, -1.187924223466553, -1.2006976022135052, -1.2134709809604574, -1.2262443597074095, -1.2390177384543617, -1.2517911172013139, -1.264564495948266, -1.2773378746952182 ], [ 0, -0.01445261022305146, -0.02890522044610292, -0.043357830669154385, -0.05781044089220584, -0.0722630511152573, -0.08671566133830877, -0.10116827156136023, -0.11562088178441168, -0.13007349200746315, -0.1445261022305146, -0.15897871245356607, -0.17343132267661754, -0.187883932899669, -0.20233654312272045, -0.2167891533457719, -0.23124176356882337, -0.24569437379187484, -0.2601469840149263, -0.27459959423797775, -0.2890522044610292, -0.3035048146840807, -0.31795742490713214, -0.3324100351301836, -0.3468626453532351, -0.3613152555762865, -0.375767865799338, -0.3902204760223894, -0.4046730862454409, -0.4191256964684924, -0.4335783066915438, -0.4480309169145953, -0.46248352713764673, -0.47693613736069823, -0.4913887475837497, -0.5058413578068012, -0.5202939680298526, -0.5347465782529041, -0.5491991884759555, -0.563651798699007, -0.5781044089220584, -0.5925570191451099, -0.6070096293681614, -0.6214622395912128, -0.6359148498142643, -0.6503674600373157, -0.6648200702603672, -0.6792726804834186, -0.6937252907064702, -0.7081779009295216, -0.722630511152573, -0.7370831213756246, -0.751535731598676, -0.7659883418217274, -0.7804409520447788, -0.7948935622678304, -0.8093461724908818, -0.8237987827139333, -0.8382513929369848, -0.8527040031600363, -0.8671566133830876, -0.881609223606139, -0.8960618338291906, -0.910514444052242, -0.9249670542752935, -0.939419664498345, -0.9538722747213965, -0.9683248849444478, -0.9827774951674993, -0.9972301053905508, -1.0116827156136023, -1.0261353258366537, -1.0405879360597052, -1.0550405462827566, -1.0694931565058081, -1.0839457667288597, -1.098398376951911, -1.1128509871749626, -1.127303597398014, -1.1417562076210654, -1.1562088178441168, -1.1706614280671683, -1.1851140382902199, -1.1995666485132712, -1.2140192587363228, -1.228471868959374, -1.2429244791824257, -1.257377089405477, -1.2718296996285285, -1.28628230985158, -1.3007349200746314, -1.315187530297683, -1.3296401405207343, -1.3440927507437859, -1.3585453609668372, -1.372997971189889, -1.3874505814129403, -1.4019031916359916, -1.4163558018590432, -1.4308084120820945, -1.445261022305146 ], [ 0, -0.016074803849369903, -0.032149607698739806, -0.048224411548109705, -0.06429921539747961, -0.08037401924684952, -0.09644882309621941, -0.11252362694558932, -0.12859843079495922, -0.14467323464432913, -0.16074803849369904, -0.17682284234306891, -0.19289764619243882, -0.20897245004180875, -0.22504725389117863, -0.2411220577405485, -0.25719686158991845, -0.2732716654392884, -0.28934646928865826, -0.30542127313802814, -0.32149607698739807, -0.33757088083676795, -0.35364568468613783, -0.36972048853550776, -0.38579529238487764, -0.4018700962342476, -0.4179449000836175, -0.43401970393298733, -0.45009450778235727, -0.4661693116317272, -0.482244115481097, -0.49831891933046696, -0.5143937231798369, -0.5304685270292068, -0.5465433308785768, -0.5626181347279466, -0.5786929385773165, -0.5947677424266864, -0.6108425462760563, -0.6269173501254262, -0.6429921539747961, -0.659066957824166, -0.6751417616735359, -0.6912165655229059, -0.7072913693722757, -0.7233661732216455, -0.7394409770710155, -0.7555157809203854, -0.7715905847697553, -0.7876653886191253, -0.8037401924684952, -0.819814996317865, -0.835889800167235, -0.8519646040166048, -0.8680394078659747, -0.8841142117153447, -0.9001890155647145, -0.9162638194140844, -0.9323386232634544, -0.9484134271128243, -0.964488230962194, -0.980563034811564, -0.9966378386609339, -1.012712642510304, -1.0287874463596738, -1.0448622502090437, -1.0609370540584135, -1.0770118579077834, -1.0930866617571535, -1.1091614656065232, -1.1252362694558933, -1.1413110733052632, -1.157385877154633, -1.173460681004003, -1.1895354848533728, -1.2056102887027427, -1.2216850925521126, -1.2377598964014827, -1.2538347002508523, -1.2699095041002224, -1.2859843079495923, -1.302059111798962, -1.318133915648332, -1.334208719497702, -1.3502835233470718, -1.3663583271964417, -1.3824331310458118, -1.3985079348951814, -1.4145827387445513, -1.4306575425939214, -1.446732346443291, -1.4628071502926612, -1.478881954142031, -1.494956757991401, -1.5110315618407708, -1.527106365690141, -1.5431811695395106, -1.5592559733888804, -1.5753307772382505, -1.5914055810876202, -1.6074803849369903 ], [ 0, -0.01763355756877419, -0.03526711513754838, -0.05290067270632257, -0.07053423027509675, -0.08816778784387096, -0.10580134541264515, -0.12343490298141933, -0.1410684605501935, -0.15870201811896772, -0.1763355756877419, -0.19396913325651607, -0.2116026908252903, -0.22923624839406448, -0.24686980596283867, -0.26450336353161286, -0.282136921100387, -0.29977047866916123, -0.31740403623793545, -0.3350375938067096, -0.3526711513754838, -0.370304708944258, -0.38793826651303215, -0.40557182408180636, -0.4232053816505806, -0.4408389392193548, -0.45847249678812896, -0.4761060543569031, -0.49373961192567734, -0.5113731694944516, -0.5290067270632257, -0.5466402846319999, -0.564273842200774, -0.5819073997695483, -0.5995409573383225, -0.6171745149070967, -0.6348080724758709, -0.652441630044645, -0.6700751876134192, -0.6877087451821934, -0.7053423027509677, -0.7229758603197418, -0.740609417888516, -0.7582429754572902, -0.7758765330260643, -0.7935100905948385, -0.8111436481636127, -0.8287772057323869, -0.8464107633011612, -0.8640443208699353, -0.8816778784387096, -0.8993114360074838, -0.9169449935762579, -0.9345785511450321, -0.9522121087138062, -0.9698456662825804, -0.9874792238513547, -1.0051127814201288, -1.022746338988903, -1.0403798965576774, -1.0580134541264514, -1.0756470116952255, -1.0932805692639997, -1.110914126832774, -1.128547684401548, -1.1461812419703223, -1.1638147995390966, -1.1814483571078707, -1.199081914676645, -1.216715472245419, -1.2343490298141935, -1.2519825873829675, -1.2696161449517418, -1.2872497025205158, -1.30488326008929, -1.3225168176580644, -1.3401503752268384, -1.3577839327956127, -1.3754174903643868, -1.393051047933161, -1.4106846055019353, -1.4283181630707094, -1.4459517206394836, -1.4635852782082577, -1.481218835777032, -1.4988523933458062, -1.5164859509145805, -1.5341195084833545, -1.5517530660521286, -1.5693866236209029, -1.587020181189677, -1.6046537387584514, -1.6222872963272255, -1.6399208538959997, -1.6575544114647738, -1.6751879690335483, -1.6928215266023223, -1.7104550841710964, -1.7280886417398706, -1.7457221993086447, -1.7633557568774192 ], [ 0, -0.01912271969246069, -0.03824543938492138, -0.05736815907738207, -0.07649087876984276, -0.09561359846230344, -0.11473631815476414, -0.13385903784722483, -0.15298175753968551, -0.17210447723214622, -0.19122719692460688, -0.21034991661706756, -0.22947263630952827, -0.24859535600198895, -0.26771807569444966, -0.2868407953869103, -0.30596351507937103, -0.32508623477183174, -0.34420895446429245, -0.36333167415675305, -0.38245439384921376, -0.40157711354167447, -0.4206998332341351, -0.43982255292659583, -0.45894527261905654, -0.4780679923115172, -0.4971907120039779, -0.5163134316964386, -0.5354361513888993, -0.5545588710813599, -0.5736815907738206, -0.5928043104662813, -0.6119270301587421, -0.6310497498512028, -0.6501724695436635, -0.6692951892361242, -0.6884179089285849, -0.7075406286210454, -0.7266633483135061, -0.7457860680059668, -0.7649087876984275, -0.7840315073908882, -0.8031542270833489, -0.8222769467758096, -0.8413996664682702, -0.860522386160731, -0.8796451058531917, -0.8987678255456524, -0.9178905452381131, -0.9370132649305738, -0.9561359846230344, -0.9752587043154951, -0.9943814240079558, -1.0135041437004164, -1.0326268633928772, -1.0517495830853378, -1.0708723027777987, -1.0899950224702593, -1.1091177421627199, -1.1282404618551807, -1.1473631815476413, -1.1664859012401019, -1.1856086209325627, -1.2047313406250233, -1.2238540603174841, -1.2429767800099447, -1.2620994997024055, -1.2812222193948661, -1.300344939087327, -1.3194676587797873, -1.3385903784722484, -1.3577130981647088, -1.3768358178571698, -1.3959585375496302, -1.4150812572420908, -1.4342039769345516, -1.4533266966270122, -1.472449416319473, -1.4915721360119336, -1.5106948557043944, -1.529817575396855, -1.5489402950893156, -1.5680630147817765, -1.587185734474237, -1.6063084541666979, -1.6254311738591585, -1.6445538935516193, -1.66367661324408, -1.6827993329365405, -1.7019220526290013, -1.721044772321462, -1.7401674920139227, -1.7592902117063833, -1.7784129313988442, -1.7975356510913048, -1.8166583707837656, -1.8357810904762262, -1.8549038101686866, -1.8740265298611476, -1.893149249553608, -1.9122719692460688 ], [ 0, -0.02053641317786066, -0.04107282635572132, -0.06160923953358198, -0.08214565271144264, -0.1026820658893033, -0.12321847906716396, -0.14375489224502463, -0.16429130542288528, -0.18482771860074595, -0.2053641317786066, -0.22590054495646725, -0.24643695813432792, -0.2669733713121886, -0.28750978449004927, -0.3080461976679099, -0.32858261084577056, -0.34911902402363126, -0.3696554372014919, -0.39019185037935256, -0.4107282635572132, -0.4312646767350739, -0.4518010899129345, -0.4723375030907952, -0.49287391626865584, -0.5134103294465165, -0.5339467426243772, -0.5544831558022378, -0.5750195689800985, -0.5955559821579592, -0.6160923953358198, -0.6366288085136804, -0.6571652216915411, -0.6777016348694018, -0.6982380480472625, -0.7187744612251232, -0.7393108744029838, -0.7598472875808444, -0.7803837007587051, -0.8009201139365658, -0.8214565271144264, -0.8419929402922871, -0.8625293534701478, -0.8830657666480085, -0.903602179825869, -0.9241385930037297, -0.9446750061815904, -0.9652114193594511, -0.9857478325373117, -1.0062842457151724, -1.026820658893033, -1.0473570720708938, -1.0678934852487545, -1.088429898426615, -1.1089663116044757, -1.1295027247823364, -1.150039137960197, -1.1705755511380578, -1.1911119643159185, -1.211648377493779, -1.2321847906716397, -1.2527212038495004, -1.2732576170273608, -1.2937940302052215, -1.3143304433830822, -1.334866856560943, -1.3554032697388037, -1.3759396829166641, -1.396476096094525, -1.4170125092723855, -1.4375489224502465, -1.458085335628107, -1.4786217488059676, -1.4991581619838283, -1.5196945751616888, -1.5402309883395495, -1.5607674015174102, -1.581303814695271, -1.6018402278731316, -1.6223766410509923, -1.6429130542288528, -1.6634494674067135, -1.6839858805845742, -1.7045222937624347, -1.7250587069402956, -1.745595120118156, -1.766131533296017, -1.7866679464738775, -1.807204359651738, -1.827740772829599, -1.8482771860074594, -1.8688135991853203, -1.8893500123631808, -1.9098864255410415, -1.9304228387189022, -1.950959251896763, -1.9714956650746234, -1.992032078252484, -2.0125684914303448, -2.0331049046082055, -2.053641317786066 ], [ 0, -0.02186905882264235, -0.0437381176452847, -0.06560717646792705, -0.0874762352905694, -0.10934529411321174, -0.1312143529358541, -0.15308341175849643, -0.1749524705811388, -0.19682152940378117, -0.21869058822642348, -0.24055964704906582, -0.2624287058717082, -0.2842977646943506, -0.30616682351699287, -0.3280358823396352, -0.3499049411622776, -0.37177399998491995, -0.39364305880756234, -0.4155121176302046, -0.43738117645284696, -0.45925023527548936, -0.48111929409813164, -0.502988352920774, -0.5248574117434164, -0.5467264705660587, -0.5685955293887012, -0.5904645882113434, -0.6123336470339857, -0.6342027058566282, -0.6560717646792704, -0.6779408235019128, -0.6998098823245552, -0.7216789411471976, -0.7435479999698399, -0.7654170587924822, -0.7872861176151247, -0.8091551764377669, -0.8310242352604092, -0.8528932940830516, -0.8747623529056939, -0.8966314117283364, -0.9185004705509787, -0.9403695293736211, -0.9622385881962633, -0.9841076470189056, -1.005976705841548, -1.0278457646641903, -1.0497148234868328, -1.0715838823094752, -1.0934529411321174, -1.11532199995476, -1.1371910587774023, -1.1590601176000443, -1.1809291764226868, -1.2027982352453293, -1.2246672940679715, -1.246536352890614, -1.2684054117132564, -1.2902744705358986, -1.3121435293585408, -1.3340125881811833, -1.3558816470038255, -1.377750705826468, -1.3996197646491104, -1.4214888234717526, -1.443357882294395, -1.4652269411170373, -1.4870959999396798, -1.508965058762322, -1.5308341175849645, -1.5527031764076067, -1.5745722352302494, -1.5964412940528916, -1.6183103528755338, -1.6401794116981763, -1.6620484705208185, -1.683917529343461, -1.7057865881661032, -1.7276556469887456, -1.7495247058113879, -1.77139376463403, -1.7932628234566728, -1.8151318822793148, -1.8370009411019574, -1.8588699999245997, -1.8807390587472421, -1.9026081175698843, -1.9244771763925266, -1.946346235215169, -1.9682152940378113, -1.990084352860454, -2.011953411683096, -2.0338224705057386, -2.0556915293283806, -2.0775605881510235, -2.0994296469736655, -2.1212987057963075, -2.1431677646189504, -2.1650368234415924, -2.186905882264235 ], [ 0, -0.023115397283273685, -0.04623079456654737, -0.06934619184982105, -0.09246158913309474, -0.11557698641636842, -0.1386923836996421, -0.1618077809829158, -0.18492317826618948, -0.20803857554946317, -0.23115397283273684, -0.2542693701160105, -0.2773847673992842, -0.3005001646825579, -0.3236155619658316, -0.34673095924910524, -0.36984635653237896, -0.3929617538156526, -0.41607715109892635, -0.43919254838219995, -0.4623079456654737, -0.4854233429487474, -0.508538740232021, -0.5316541375152947, -0.5547695347985684, -0.5778849320818421, -0.6010003293651158, -0.6241157266483894, -0.6472311239316632, -0.6703465212149369, -0.6934619184982105, -0.7165773157814842, -0.7396927130647579, -0.7628081103480315, -0.7859235076313053, -0.809038904914579, -0.8321543021978527, -0.8552696994811262, -0.8783850967643999, -0.9015004940476736, -0.9246158913309473, -0.9477312886142211, -0.9708466858974948, -0.9939620831807684, -1.017077480464042, -1.0401928777473157, -1.0633082750305893, -1.0864236723138632, -1.1095390695971368, -1.1326544668804106, -1.1557698641636842, -1.1788852614469578, -1.2020006587302317, -1.2251160560135053, -1.2482314532967789, -1.2713468505800525, -1.2944622478633263, -1.3175776451466, -1.3406930424298737, -1.3638084397131474, -1.386923836996421, -1.4100392342796946, -1.4331546315629684, -1.456270028846242, -1.4793854261295158, -1.5025008234127895, -1.525616220696063, -1.5487316179793367, -1.5718470152626105, -1.5949624125458841, -1.618077809829158, -1.6411932071124316, -1.6643086043957054, -1.687424001678979, -1.7105393989622524, -1.7336547962455264, -1.7567701935287998, -1.7798855908120736, -1.8030009880953473, -1.826116385378621, -1.8492317826618947, -1.8723471799451683, -1.8954625772284421, -1.9185779745117155, -1.9416933717949896, -1.964808769078263, -1.9879241663615368, -2.0110395636448106, -2.034154960928084, -2.057270358211358, -2.0803857554946315, -2.103501152777905, -2.1266165500611787, -2.1497319473444527, -2.1728473446277263, -2.195962741911, -2.2190781391942735, -2.242193536477547, -2.265308933760821, -2.2884243310440944, -2.3115397283273684 ], [ 0, -0.02427050983124842, -0.04854101966249684, -0.07281152949374525, -0.09708203932499368, -0.12135254915624209, -0.1456230589874905, -0.16989356881873893, -0.19416407864998736, -0.2184345884812358, -0.24270509831248419, -0.2669756081437326, -0.291246117974981, -0.31551662780622947, -0.33978713763747787, -0.36405764746872626, -0.3883281572999747, -0.4125986671312232, -0.4368691769624716, -0.46113968679371997, -0.48541019662496837, -0.5096807064562169, -0.5339512162874652, -0.5582217261187136, -0.582492235949962, -0.6067627457812105, -0.6310332556124589, -0.6553037654437073, -0.6795742752749557, -0.7038447851062042, -0.7281152949374525, -0.752385804768701, -0.7766563145999494, -0.8009268244311979, -0.8251973342624463, -0.8494678440936947, -0.8737383539249431, -0.8980088637561915, -0.9222793735874399, -0.9465498834186883, -0.9708203932499367, -0.9950909030811852, -1.0193614129124338, -1.043631922743682, -1.0679024325749304, -1.0921729424061788, -1.1164434522374271, -1.1407139620686757, -1.164984471899924, -1.1892549817311726, -1.213525491562421, -1.2377960013936695, -1.2620665112249179, -1.2863370210561662, -1.3106075308874146, -1.3348780407186631, -1.3591485505499115, -1.38341906038116, -1.4076895702124084, -1.4319600800436567, -1.456230589874905, -1.4805010997061534, -1.504771609537402, -1.5290421193686503, -1.5533126291998989, -1.5775831390311472, -1.6018536488623958, -1.626124158693644, -1.6503946685248927, -1.6746651783561408, -1.6989356881873894, -1.7232061980186377, -1.7474767078498863, -1.7717472176811346, -1.796017727512383, -1.8202882373436315, -1.8445587471748799, -1.8688292570061285, -1.8930997668373766, -1.9173702766686254, -1.9416407864998735, -1.9659112963311218, -1.9901818061623704, -2.0144523159936187, -2.0387228258248675, -2.0629933356561154, -2.087263845487364, -2.1115343553186126, -2.135804865149861, -2.1600753749811092, -2.1843458848123576, -2.2086163946436064, -2.2328869044748543, -2.257157414306103, -2.2814279241373514, -2.3056984339686, -2.329968943799848, -2.3542394536310964, -2.3785099634623452, -2.4027804732935936, -2.427050983124842 ], [ 0, -0.02532983776506045, -0.0506596755301209, -0.07598951329518136, -0.1013193510602418, -0.12664918882530227, -0.15197902659036272, -0.17730886435542315, -0.2026387021204836, -0.22796853988554408, -0.25329837765060453, -0.27862821541566496, -0.30395805318072544, -0.32928789094578587, -0.3546177287108463, -0.3799475664759068, -0.4052774042409672, -0.4306072420060277, -0.45593707977108816, -0.48126691753614853, -0.5065967553012091, -0.5319265930662695, -0.5572564308313299, -0.5825862685963904, -0.6079161063614509, -0.6332459441265113, -0.6585757818915717, -0.6839056196566322, -0.7092354574216926, -0.7345652951867531, -0.7598951329518135, -0.7852249707168739, -0.8105548084819344, -0.8358846462469949, -0.8612144840120554, -0.8865443217771158, -0.9118741595421763, -0.9372039973072366, -0.9625338350722971, -0.9878636728373575, -1.0131935106024181, -1.0385233483674785, -1.063853186132539, -1.0891830238975995, -1.1145128616626598, -1.1398426994277202, -1.1651725371927808, -1.1905023749578412, -1.2158322127229018, -1.2411620504879621, -1.2664918882530225, -1.291821726018083, -1.3171515637831435, -1.3424814015482038, -1.3678112393132644, -1.3931410770783248, -1.4184709148433852, -1.4438007526084458, -1.4691305903735061, -1.4944604281385667, -1.519790265903627, -1.5451201036686875, -1.5704499414337478, -1.5957797791988084, -1.6211096169638688, -1.6464394547289294, -1.6717692924939898, -1.6970991302590501, -1.7224289680241107, -1.747758805789171, -1.7730886435542317, -1.798418481319292, -1.8237483190843526, -1.849078156849413, -1.8744079946144732, -1.899737832379534, -1.9250676701445941, -1.950397507909655, -1.975727345674715, -2.0010571834397757, -2.0263870212048363, -2.0517168589698964, -2.077046696734957, -2.102376534500017, -2.127706372265078, -2.1530362100301383, -2.178366047795199, -2.203695885560259, -2.2290257233253197, -2.2543555610903803, -2.2796853988554404, -2.305015236620501, -2.3303450743855616, -2.355674912150622, -2.3810047499156823, -2.406334587680743, -2.4316644254458035, -2.4569942632108637, -2.4823241009759243, -2.5076539387409844, -2.532983776506045 ], [ 0, -0.02628920040131591, -0.05257840080263182, -0.07886760120394773, -0.10515680160526364, -0.13144600200657955, -0.15773520240789546, -0.18402440280921137, -0.21031360321052728, -0.23660280361184322, -0.2628920040131591, -0.289181204414475, -0.3154704048157909, -0.34175960521710685, -0.36804880561842274, -0.3943380060197386, -0.42062720642105456, -0.4469164068223705, -0.47320560722368643, -0.49949480762500226, -0.5257840080263182, -0.5520732084276341, -0.57836240882895, -0.604651609230266, -0.6309408096315818, -0.6572300100328978, -0.6835192104342137, -0.7098084108355296, -0.7360976112368455, -0.7623868116381615, -0.7886760120394772, -0.8149652124407932, -0.8412544128421091, -0.8675436132434251, -0.893832813644741, -0.9201220140460569, -0.9464112144473729, -0.9727004148486886, -0.9989896152500045, -1.0252788156513204, -1.0515680160526364, -1.0778572164539524, -1.1041464168552682, -1.1304356172565841, -1.1567248176579, -1.183014018059216, -1.209303218460532, -1.2355924188618477, -1.2618816192631637, -1.2881708196644797, -1.3144600200657957, -1.3407492204671114, -1.3670384208684274, -1.3933276212697432, -1.4196168216710592, -1.445906022072375, -1.472195222473691, -1.498484422875007, -1.524773623276323, -1.5510628236776387, -1.5773520240789545, -1.6036412244802705, -1.6299304248815865, -1.6562196252829022, -1.6825088256842182, -1.7087980260855342, -1.7350872264868502, -1.7613764268881658, -1.787665627289482, -1.8139548276907977, -1.8402440280921137, -1.8665332284934295, -1.8928224288947457, -1.9191116292960615, -1.9454008296973773, -1.9716900300986933, -1.997979230500009, -2.024268430901325, -2.050557631302641, -2.076846831703957, -2.103136032105273, -2.1294252325065886, -2.1557144329079048, -2.1820036333092205, -2.2082928337105363, -2.234582034111852, -2.2608712345131683, -2.287160434914484, -2.3134496353158, -2.339738835717116, -2.366028036118432, -2.392317236519748, -2.418606436921064, -2.4448956373223796, -2.4711848377236953, -2.4974740381250116, -2.5237632385263273, -2.550052438927643, -2.5763416393289593, -2.602630839730275, -2.6289200401315913 ], [ 0, -0.027144811573980587, -0.05428962314796117, -0.08143443472194176, -0.10857924629592235, -0.13572405786990294, -0.1628688694438835, -0.19001368101786412, -0.2171584925918447, -0.2443033041658253, -0.2714481157398059, -0.29859292731378645, -0.325737738887767, -0.35288255046174766, -0.38002736203572823, -0.40717217360970875, -0.4343169851836894, -0.46146179675767, -0.4886066083316506, -0.5157514199056311, -0.5428962314796117, -0.5700410430535924, -0.5971858546275729, -0.6243306662015534, -0.651475477775534, -0.6786202893495147, -0.7057651009234953, -0.7329099124974758, -0.7600547240714565, -0.787199535645437, -0.8143443472194175, -0.8414891587933981, -0.8686339703673788, -0.8957787819413594, -0.92292359351534, -0.9500684050893206, -0.9772132166633012, -1.0043580282372817, -1.0315028398112622, -1.0586476513852427, -1.0857924629592235, -1.112937274533204, -1.1400820861071848, -1.1672268976811653, -1.1943717092551458, -1.2215165208291263, -1.2486613324031068, -1.2758061439770876, -1.302950955551068, -1.3300957671250488, -1.3572405786990294, -1.3843853902730099, -1.4115302018469906, -1.438675013420971, -1.4658198249949517, -1.4929646365689322, -1.520109448142913, -1.5472542597168935, -1.574399071290874, -1.6015438828648547, -1.628688694438835, -1.6558335060128158, -1.6829783175867963, -1.710123129160777, -1.7372679407347575, -1.764412752308738, -1.7915575638827188, -1.818702375456699, -1.84584718703068, -1.8729919986046604, -1.9001368101786411, -1.9272816217526216, -1.9544264333266024, -1.981571244900583, -2.0087160564745634, -2.035860868048544, -2.0630056796225245, -2.090150491196505, -2.1172953027704855, -2.1444401143444667, -2.171584925918447, -2.1987297374924273, -2.225874549066408, -2.2530193606403883, -2.2801641722143695, -2.30730898378835, -2.3344537953623306, -2.361598606936311, -2.3887434185102916, -2.4158882300842723, -2.4430330416582526, -2.4701778532322334, -2.4973226648062137, -2.524467476380195, -2.551612287954175, -2.578757099528156, -2.605901911102136, -2.6330467226761165, -2.6601915342500977, -2.687336345824078, -2.7144811573980587 ], [ 0, -0.02789329457664754, -0.05578658915329508, -0.08367988372994263, -0.11157317830659016, -0.1394664728832377, -0.16735976745988526, -0.1952530620365328, -0.22314635661318033, -0.2510396511898279, -0.2789329457664754, -0.30682624034312295, -0.3347195349197705, -0.3626128294964181, -0.3905061240730656, -0.41839941864971314, -0.44629271322636066, -0.47418600780300824, -0.5020793023796558, -0.5299725969563033, -0.5578658915329509, -0.5857591861095984, -0.6136524806862459, -0.6415457752628935, -0.669439069839541, -0.6973323644161886, -0.7252256589928362, -0.7531189535694837, -0.7810122481461312, -0.8089055427227788, -0.8367988372994263, -0.8646921318760737, -0.8925854264527213, -0.9204787210293689, -0.9483720156060165, -0.976265310182664, -1.0041586047593116, -1.032051899335959, -1.0599451939126066, -1.0878384884892542, -1.1157317830659017, -1.1436250776425492, -1.1715183722191969, -1.1994116667958443, -1.2273049613724918, -1.2551982559491393, -1.283091550525787, -1.3109848451024344, -1.338878139679082, -1.3667714342557296, -1.3946647288323772, -1.4225580234090247, -1.4504513179856724, -1.4783446125623196, -1.5062379071389673, -1.5341312017156148, -1.5620244962922625, -1.58991779086891, -1.6178110854455576, -1.645704380022205, -1.6735976745988526, -1.7014909691755, -1.7293842637521475, -1.7572775583287952, -1.7851708529054426, -1.8130641474820903, -1.8409574420587378, -1.8688507366353853, -1.896744031212033, -1.9246373257886804, -1.952530620365328, -1.9804239149419756, -2.0083172095186232, -2.0362105040952705, -2.064103798671918, -2.091997093248566, -2.119890387825213, -2.147783682401861, -2.1756769769785085, -2.203570271555156, -2.2314635661318034, -2.2593568607084507, -2.2872501552850983, -2.315143449861746, -2.3430367444383937, -2.370930039015041, -2.3988233335916886, -2.4267166281683363, -2.4546099227449836, -2.4825032173216313, -2.5103965118982785, -2.5382898064749266, -2.566183101051574, -2.5940763956282216, -2.621969690204869, -2.649862984781517, -2.677756279358164, -2.7056495739348114, -2.733542868511459, -2.761436163088107, -2.7893294576647545 ], [ 0, -0.02853169548885461, -0.05706339097770922, -0.08559508646656383, -0.11412678195541844, -0.14265847744427304, -0.17119017293312766, -0.19972186842198225, -0.22825356391083687, -0.2567852593996915, -0.2853169548885461, -0.3138486503774007, -0.3423803458662553, -0.3709120413551099, -0.3994437368439645, -0.4279754323328191, -0.45650712782167374, -0.4850388233105284, -0.513570518799383, -0.5421022142882376, -0.5706339097770922, -0.5991656052659468, -0.6276973007548013, -0.6562289962436559, -0.6847606917325106, -0.7132923872213652, -0.7418240827102198, -0.7703557781990744, -0.798887473687929, -0.8274191691767837, -0.8559508646656382, -0.8844825601544928, -0.9130142556433475, -0.9415459511322021, -0.9700776466210568, -0.9986093421099114, -1.027141037598766, -1.0556727330876203, -1.0842044285764751, -1.1127361240653297, -1.1412678195541843, -1.169799515043039, -1.1983312105318935, -1.2268629060207483, -1.2553946015096027, -1.2839262969984573, -1.3124579924873119, -1.3409896879761665, -1.3695213834650213, -1.3980530789538759, -1.4265847744427305, -1.455116469931585, -1.4836481654204396, -1.5121798609092942, -1.5407115563981488, -1.5692432518870034, -1.597774947375858, -1.6263066428647126, -1.6548383383535674, -1.683370033842422, -1.7119017293312764, -1.740433424820131, -1.7689651203089856, -1.7974968157978404, -1.826028511286695, -1.8545602067755496, -1.8830919022644041, -1.9116235977532585, -1.9401552932421136, -1.968686988730968, -1.9972186842198227, -2.0257503797086773, -2.054282075197532, -2.0828137706863865, -2.1113454661752407, -2.1398771616640957, -2.1684088571529503, -2.196940552641805, -2.2254722481306595, -2.254003943619514, -2.2825356391083687, -2.3110673345972232, -2.339599030086078, -2.3681307255749324, -2.396662421063787, -2.4251941165526416, -2.4537258120414966, -2.482257507530351, -2.5107892030192054, -2.53932089850806, -2.5678525939969146, -2.5963842894857696, -2.6249159849746238, -2.653447680463479, -2.681979375952333, -2.710511071441188, -2.7390427669300426, -2.7675744624188967, -2.7961061579077517, -2.824637853396606, -2.853169548885461 ], [ 0, -0.02905749483385893, -0.05811498966771786, -0.08717248450157679, -0.11622997933543572, -0.14528747416929466, -0.17434496900315358, -0.20340246383701252, -0.23245995867087144, -0.26151745350473043, -0.2905749483385893, -0.3196324431724482, -0.34868993800630715, -0.37774743284016615, -0.40680492767402504, -0.43586242250788393, -0.46491991734174287, -0.49397741217560187, -0.5230349070094609, -0.5520924018433196, -0.5811498966771786, -0.6102073915110375, -0.6392648863448964, -0.6683223811787554, -0.6973798760126143, -0.7264373708464733, -0.7554948656803323, -0.7845523605141911, -0.8136098553480501, -0.8426673501819091, -0.8717248450157679, -0.9007823398496269, -0.9298398346834857, -0.9588973295173447, -0.9879548243512037, -1.0170123191850626, -1.0460698140189217, -1.0751273088527804, -1.1041848036866393, -1.1332422985204982, -1.1622997933543573, -1.1913572881882162, -1.220414783022075, -1.2494722778559342, -1.2785297726897928, -1.3075872675236517, -1.3366447623575108, -1.3657022571913697, -1.3947597520252286, -1.4238172468590877, -1.4528747416929466, -1.4819322365268055, -1.5109897313606646, -1.5400472261945233, -1.5691047210283822, -1.5981622158622413, -1.6272197106961002, -1.656277205529959, -1.6853347003638182, -1.714392195197677, -1.7434496900315357, -1.7725071848653948, -1.8015646796992537, -1.8306221745331126, -1.8596796693669715, -1.8887371642008306, -1.9177946590346895, -1.9468521538685482, -1.9759096487024075, -2.004967143536266, -2.0340246383701253, -2.063082133203984, -2.0921396280378435, -2.121197122871702, -2.150254617705561, -2.17931211253942, -2.2083696073732786, -2.2374271022071377, -2.2664845970409964, -2.295542091874856, -2.3245995867087146, -2.3536570815425732, -2.3827145763764324, -2.411772071210291, -2.44082956604415, -2.4698870608780092, -2.4989445557118684, -2.528002050545727, -2.5570595453795857, -2.586117040213445, -2.6151745350473035, -2.644232029881163, -2.6732895247150217, -2.702347019548881, -2.7314045143827395, -2.7604620092165986, -2.7895195040504572, -2.8185769988843163, -2.8476344937181755, -2.876691988552034, -2.9057494833858932 ], [ 0, -0.02946861752186066, -0.05893723504372132, -0.08840585256558198, -0.11787447008744265, -0.1473430876093033, -0.17681170513116395, -0.20628032265302462, -0.2357489401748853, -0.265217557696746, -0.2946861752186066, -0.3241547927404672, -0.3536234102623279, -0.3830920277841886, -0.41256064530604925, -0.4420292628279099, -0.4714978803497706, -0.5009664978716313, -0.530435115393492, -0.5599037329153526, -0.5893723504372133, -0.618840967959074, -0.6483095854809344, -0.6777782030027951, -0.7072468205246558, -0.7367154380465165, -0.7661840555683772, -0.7956526730902378, -0.8251212906120985, -0.8545899081339592, -0.8840585256558198, -0.9135271431776805, -0.9429957606995412, -0.9724643782214019, -1.0019329957432626, -1.0314016132651231, -1.060870230786984, -1.0903388483088443, -1.1198074658307051, -1.1492760833525657, -1.1787447008744265, -1.208213318396287, -1.237681935918148, -1.2671505534400085, -1.2966191709618688, -1.3260877884837297, -1.3555564060055902, -1.385025023527451, -1.4144936410493116, -1.4439622585711724, -1.473430876093033, -1.5028994936148938, -1.5323681111367544, -1.561836728658615, -1.5913053461804756, -1.6207739637023364, -1.650242581224197, -1.6797111987460578, -1.7091798162679184, -1.738648433789779, -1.7681170513116395, -1.7975856688335001, -1.827054286355361, -1.8565229038772215, -1.8859915213990823, -1.915460138920943, -1.9449287564428037, -1.974397373964664, -2.003865991486525, -2.0333346090083855, -2.0628032265302463, -2.092271844052107, -2.121740461573968, -2.1512090790958283, -2.1806776966176886, -2.2101463141395494, -2.2396149316614102, -2.269083549183271, -2.2985521667051314, -2.328020784226992, -2.357489401748853, -2.3869580192707134, -2.416426636792574, -2.4458952543144346, -2.475363871836296, -2.504832489358156, -2.534301106880017, -2.5637697244018773, -2.5932383419237377, -2.622706959445599, -2.6521755769674593, -2.68164419448932, -2.7111128120111805, -2.7405814295330417, -2.770050047054902, -2.799518664576763, -2.8289872820986233, -2.858455899620484, -2.887924517142345, -2.9173931346642052, -2.946861752186066 ], [ 0, -0.029763441039434336, -0.05952688207886867, -0.08929032311830301, -0.11905376415773734, -0.14881720519717168, -0.17858064623660602, -0.20834408727604034, -0.2381075283154747, -0.26787096935490906, -0.29763441039434335, -0.32739785143377764, -0.35716129247321204, -0.3869247335126464, -0.4166881745520807, -0.446451615591515, -0.4762150566309494, -0.5059784976703837, -0.5357419387098181, -0.5655053797492523, -0.5952688207886867, -0.6250322618281211, -0.6547957028675553, -0.6845591439069897, -0.7143225849464241, -0.7440860259858584, -0.7738494670252928, -0.8036129080647271, -0.8333763491041614, -0.8631397901435958, -0.89290323118303, -0.9226666722224643, -0.9524301132618987, -0.982193554301333, -1.0119569953407674, -1.0417204363802017, -1.0714838774196362, -1.1012473184590703, -1.1310107594985046, -1.1607742005379391, -1.1905376415773734, -1.2203010826168077, -1.2500645236562422, -1.2798279646956765, -1.3095914057351106, -1.339354846774545, -1.3691182878139794, -1.3988817288534137, -1.4286451698928482, -1.4584086109322825, -1.4881720519717168, -1.5179354930111513, -1.5476989340505856, -1.5774623750900196, -1.6072258161294541, -1.6369892571688884, -1.6667526982083227, -1.6965161392477572, -1.7262795802871915, -1.7560430213266258, -1.78580646236606, -1.8155699034054944, -1.8453333444449287, -1.875096785484363, -1.9048602265237975, -1.9346236675632318, -1.964387108602666, -1.9941505496421004, -2.023913990681535, -2.053677431720969, -2.0834408727604035, -2.1132043137998378, -2.1429677548392725, -2.1727311958787063, -2.2024946369181406, -2.2322580779575754, -2.262021518997009, -2.291784960036444, -2.3215484010758782, -2.3513118421153125, -2.381075283154747, -2.410838724194181, -2.4406021652336154, -2.4703656062730497, -2.5001290473124844, -2.5298924883519183, -2.559655929391353, -2.5894193704307873, -2.619182811470221, -2.648946252509656, -2.67870969354909, -2.7084731345885245, -2.7382365756279587, -2.7680000166673935, -2.7977634577068273, -2.827526898746262, -2.8572903397856964, -2.88705378082513, -2.916817221864565, -2.9465806629039992, -2.9763441039434335 ], [ 0, -0.029940801852848146, -0.05988160370569629, -0.08982240555854444, -0.11976320741139258, -0.14970400926424074, -0.17964481111708888, -0.20958561296993702, -0.23952641482278517, -0.26946721667563334, -0.2994080185284815, -0.32934882038132957, -0.35928962223417776, -0.3892304240870259, -0.41917122593987405, -0.44911202779272213, -0.47905282964557033, -0.5089936314984185, -0.5389344333512667, -0.5688752352041148, -0.598816037056963, -0.628756838909811, -0.6586976407626591, -0.6886384426155073, -0.7185792444683555, -0.7485200463212036, -0.7784608481740518, -0.8084016500268999, -0.8383424518797481, -0.8682832537325963, -0.8982240555854443, -0.9281648574382925, -0.9581056592911407, -0.9880464611439889, -1.017987262996837, -1.0479280648496851, -1.0778688667025333, -1.1078096685553813, -1.1377504704082295, -1.1676912722610777, -1.197632074113926, -1.227572875966774, -1.257513677819622, -1.2874544796724703, -1.3173952815253183, -1.3473360833781665, -1.3772768852310147, -1.4072176870838629, -1.437158488936711, -1.4670992907895593, -1.4970400926424072, -1.5269808944952554, -1.5569216963481036, -1.5868624982009516, -1.6168033000537998, -1.646744101906648, -1.6766849037594962, -1.7066257056123444, -1.7365665074651926, -1.7665073093180408, -1.7964481111708885, -1.8263889130237367, -1.856329714876585, -1.8862705167294331, -1.9162113185822813, -1.9461521204351295, -1.9760929222879777, -2.0060337241408255, -2.035974525993674, -2.065915327846522, -2.0958561296993703, -2.1257969315522183, -2.1557377334050667, -2.1856785352579147, -2.2156193371107626, -2.245560138963611, -2.275500940816459, -2.3054417426693075, -2.3353825445221554, -2.365323346375004, -2.395264148227852, -2.4252049500807, -2.455145751933548, -2.4850865537863958, -2.515027355639244, -2.544968157492092, -2.5749089593449406, -2.6048497611977885, -2.6347905630506365, -2.664731364903485, -2.694672166756333, -2.7246129686091813, -2.7545537704620293, -2.7844945723148777, -2.8144353741677257, -2.844376176020574, -2.874316977873422, -2.90425777972627, -2.9341985815791185, -2.9641393834319665, -2.9940801852848145 ], [ 0, -0.03, -0.06, -0.09, -0.12, -0.15, -0.18, -0.21, -0.24, -0.27, -0.3, -0.32999999999999996, -0.36, -0.39, -0.42, -0.44999999999999996, -0.48, -0.51, -0.54, -0.57, -0.6, -0.63, -0.6599999999999999, -0.69, -0.72, -0.75, -0.78, -0.8099999999999999, -0.84, -0.87, -0.8999999999999999, -0.9299999999999999, -0.96, -0.99, -1.02, -1.05, -1.08, -1.1099999999999999, -1.14, -1.17, -1.2, -1.23, -1.26, -1.29, -1.3199999999999998, -1.3499999999999999, -1.38, -1.41, -1.44, -1.47, -1.5, -1.53, -1.56, -1.5899999999999999, -1.6199999999999999, -1.65, -1.68, -1.71, -1.74, -1.77, -1.7999999999999998, -1.8299999999999998, -1.8599999999999999, -1.89, -1.92, -1.95, -1.98, -2.01, -2.04, -2.07, -2.1, -2.13, -2.16, -2.19, -2.2199999999999998, -2.25, -2.28, -2.31, -2.34, -2.37, -2.4, -2.4299999999999997, -2.46, -2.4899999999999998, -2.52, -2.55, -2.58, -2.61, -2.6399999999999997, -2.67, -2.6999999999999997, -2.73, -2.76, -2.79, -2.82, -2.85, -2.88, -2.9099999999999997, -2.94, -2.9699999999999998, -3 ], [ 0, -0.029940801852848146, -0.05988160370569629, -0.08982240555854444, -0.11976320741139258, -0.14970400926424074, -0.17964481111708888, -0.20958561296993702, -0.23952641482278517, -0.26946721667563334, -0.2994080185284815, -0.32934882038132957, -0.35928962223417776, -0.3892304240870259, -0.41917122593987405, -0.44911202779272213, -0.47905282964557033, -0.5089936314984185, -0.5389344333512667, -0.5688752352041148, -0.598816037056963, -0.628756838909811, -0.6586976407626591, -0.6886384426155073, -0.7185792444683555, -0.7485200463212036, -0.7784608481740518, -0.8084016500268999, -0.8383424518797481, -0.8682832537325963, -0.8982240555854443, -0.9281648574382925, -0.9581056592911407, -0.9880464611439889, -1.017987262996837, -1.0479280648496851, -1.0778688667025333, -1.1078096685553813, -1.1377504704082295, -1.1676912722610777, -1.197632074113926, -1.227572875966774, -1.257513677819622, -1.2874544796724703, -1.3173952815253183, -1.3473360833781665, -1.3772768852310147, -1.4072176870838629, -1.437158488936711, -1.4670992907895593, -1.4970400926424072, -1.5269808944952554, -1.5569216963481036, -1.5868624982009516, -1.6168033000537998, -1.646744101906648, -1.6766849037594962, -1.7066257056123444, -1.7365665074651926, -1.7665073093180408, -1.7964481111708885, -1.8263889130237367, -1.856329714876585, -1.8862705167294331, -1.9162113185822813, -1.9461521204351295, -1.9760929222879777, -2.0060337241408255, -2.035974525993674, -2.065915327846522, -2.0958561296993703, -2.1257969315522183, -2.1557377334050667, -2.1856785352579147, -2.2156193371107626, -2.245560138963611, -2.275500940816459, -2.3054417426693075, -2.3353825445221554, -2.365323346375004, -2.395264148227852, -2.4252049500807, -2.455145751933548, -2.4850865537863958, -2.515027355639244, -2.544968157492092, -2.5749089593449406, -2.6048497611977885, -2.6347905630506365, -2.664731364903485, -2.694672166756333, -2.7246129686091813, -2.7545537704620293, -2.7844945723148777, -2.8144353741677257, -2.844376176020574, -2.874316977873422, -2.90425777972627, -2.9341985815791185, -2.9641393834319665, -2.9940801852848145 ], [ 0, -0.029763441039434336, -0.05952688207886867, -0.08929032311830301, -0.11905376415773734, -0.14881720519717168, -0.17858064623660602, -0.20834408727604034, -0.2381075283154747, -0.26787096935490906, -0.29763441039434335, -0.32739785143377764, -0.35716129247321204, -0.3869247335126464, -0.4166881745520807, -0.446451615591515, -0.4762150566309494, -0.5059784976703837, -0.5357419387098181, -0.5655053797492523, -0.5952688207886867, -0.6250322618281211, -0.6547957028675553, -0.6845591439069897, -0.7143225849464241, -0.7440860259858584, -0.7738494670252928, -0.8036129080647271, -0.8333763491041614, -0.8631397901435958, -0.89290323118303, -0.9226666722224643, -0.9524301132618987, -0.982193554301333, -1.0119569953407674, -1.0417204363802017, -1.0714838774196362, -1.1012473184590703, -1.1310107594985046, -1.1607742005379391, -1.1905376415773734, -1.2203010826168077, -1.2500645236562422, -1.2798279646956765, -1.3095914057351106, -1.339354846774545, -1.3691182878139794, -1.3988817288534137, -1.4286451698928482, -1.4584086109322825, -1.4881720519717168, -1.5179354930111513, -1.5476989340505856, -1.5774623750900196, -1.6072258161294541, -1.6369892571688884, -1.6667526982083227, -1.6965161392477572, -1.7262795802871915, -1.7560430213266258, -1.78580646236606, -1.8155699034054944, -1.8453333444449287, -1.875096785484363, -1.9048602265237975, -1.9346236675632318, -1.964387108602666, -1.9941505496421004, -2.023913990681535, -2.053677431720969, -2.0834408727604035, -2.1132043137998378, -2.1429677548392725, -2.1727311958787063, -2.2024946369181406, -2.2322580779575754, -2.262021518997009, -2.291784960036444, -2.3215484010758782, -2.3513118421153125, -2.381075283154747, -2.410838724194181, -2.4406021652336154, -2.4703656062730497, -2.5001290473124844, -2.5298924883519183, -2.559655929391353, -2.5894193704307873, -2.619182811470221, -2.648946252509656, -2.67870969354909, -2.7084731345885245, -2.7382365756279587, -2.7680000166673935, -2.7977634577068273, -2.827526898746262, -2.8572903397856964, -2.88705378082513, -2.916817221864565, -2.9465806629039992, -2.9763441039434335 ], [ 0, -0.029468617521860658, -0.058937235043721316, -0.08840585256558198, -0.11787447008744263, -0.14734308760930329, -0.17681170513116395, -0.2062803226530246, -0.23574894017488526, -0.26521755769674593, -0.29468617521860657, -0.3241547927404672, -0.3536234102623279, -0.38309202778418855, -0.4125606453060492, -0.44202926282790983, -0.4714978803497705, -0.5009664978716312, -0.5304351153934919, -0.5599037329153524, -0.5893723504372131, -0.6188409679590738, -0.6483095854809344, -0.6777782030027951, -0.7072468205246558, -0.7367154380465164, -0.7661840555683771, -0.7956526730902377, -0.8251212906120984, -0.8545899081339591, -0.8840585256558197, -0.9135271431776804, -0.942995760699541, -0.9724643782214017, -1.0019329957432623, -1.0314016132651231, -1.0608702307869837, -1.0903388483088443, -1.119807465830705, -1.1492760833525657, -1.1787447008744263, -1.2082133183962869, -1.2376819359181477, -1.2671505534400083, -1.2966191709618688, -1.3260877884837294, -1.3555564060055902, -1.3850250235274508, -1.4144936410493116, -1.4439622585711722, -1.4734308760930328, -1.5028994936148936, -1.5323681111367542, -1.5618367286586148, -1.5913053461804754, -1.6207739637023362, -1.6502425812241968, -1.6797111987460576, -1.7091798162679182, -1.738648433789779, -1.7681170513116393, -1.7975856688335, -1.8270542863553607, -1.8565229038772213, -1.885991521399082, -1.9154601389209427, -1.9449287564428035, -1.9743973739646639, -2.0038659914865247, -2.0333346090083855, -2.0628032265302463, -2.0922718440521066, -2.1217404615739675, -2.151209079095828, -2.1806776966176886, -2.2101463141395494, -2.23961493166141, -2.2690835491832706, -2.2985521667051314, -2.328020784226992, -2.3574894017488526, -2.386958019270713, -2.4164266367925737, -2.4458952543144346, -2.4753638718362954, -2.5048324893581557, -2.5343011068800165, -2.5637697244018773, -2.5932383419237377, -2.6227069594455985, -2.652175576967459, -2.6816441944893197, -2.7111128120111805, -2.7405814295330413, -2.7700500470549017, -2.7995186645767625, -2.8289872820986233, -2.8584558996204836, -2.8879245171423444, -2.917393134664205, -2.9468617521860656 ], [ 0, -0.02905749483385893, -0.05811498966771786, -0.08717248450157679, -0.11622997933543572, -0.14528747416929466, -0.17434496900315358, -0.20340246383701252, -0.23245995867087144, -0.26151745350473043, -0.2905749483385893, -0.3196324431724482, -0.34868993800630715, -0.37774743284016615, -0.40680492767402504, -0.43586242250788393, -0.46491991734174287, -0.49397741217560187, -0.5230349070094609, -0.5520924018433196, -0.5811498966771786, -0.6102073915110375, -0.6392648863448964, -0.6683223811787554, -0.6973798760126143, -0.7264373708464733, -0.7554948656803323, -0.7845523605141911, -0.8136098553480501, -0.8426673501819091, -0.8717248450157679, -0.9007823398496269, -0.9298398346834857, -0.9588973295173447, -0.9879548243512037, -1.0170123191850626, -1.0460698140189217, -1.0751273088527804, -1.1041848036866393, -1.1332422985204982, -1.1622997933543573, -1.1913572881882162, -1.220414783022075, -1.2494722778559342, -1.2785297726897928, -1.3075872675236517, -1.3366447623575108, -1.3657022571913697, -1.3947597520252286, -1.4238172468590877, -1.4528747416929466, -1.4819322365268055, -1.5109897313606646, -1.5400472261945233, -1.5691047210283822, -1.5981622158622413, -1.6272197106961002, -1.656277205529959, -1.6853347003638182, -1.714392195197677, -1.7434496900315357, -1.7725071848653948, -1.8015646796992537, -1.8306221745331126, -1.8596796693669715, -1.8887371642008306, -1.9177946590346895, -1.9468521538685482, -1.9759096487024075, -2.004967143536266, -2.0340246383701253, -2.063082133203984, -2.0921396280378435, -2.121197122871702, -2.150254617705561, -2.17931211253942, -2.2083696073732786, -2.2374271022071377, -2.2664845970409964, -2.295542091874856, -2.3245995867087146, -2.3536570815425732, -2.3827145763764324, -2.411772071210291, -2.44082956604415, -2.4698870608780092, -2.4989445557118684, -2.528002050545727, -2.5570595453795857, -2.586117040213445, -2.6151745350473035, -2.644232029881163, -2.6732895247150217, -2.702347019548881, -2.7314045143827395, -2.7604620092165986, -2.7895195040504572, -2.8185769988843163, -2.8476344937181755, -2.876691988552034, -2.9057494833858932 ], [ 0, -0.028531695488854602, -0.057063390977709204, -0.0855950864665638, -0.11412678195541841, -0.142658477444273, -0.1711901729331276, -0.19972186842198222, -0.22825356391083682, -0.25678525939969143, -0.285316954888546, -0.3138486503774006, -0.3423803458662552, -0.37091204135510986, -0.39944373684396445, -0.427975432332819, -0.45650712782167363, -0.4850388233105283, -0.5135705187993829, -0.5421022142882373, -0.570633909777092, -0.5991656052659466, -0.6276973007548012, -0.6562289962436558, -0.6847606917325104, -0.713292387221365, -0.7418240827102197, -0.7703557781990742, -0.7988874736879289, -0.8274191691767835, -0.855950864665638, -0.8844825601544927, -0.9130142556433473, -0.9415459511322019, -0.9700776466210566, -0.9986093421099111, -1.0271410375987657, -1.05567273308762, -1.0842044285764747, -1.1127361240653295, -1.141267819554184, -1.1697995150430387, -1.1983312105318933, -1.2268629060207479, -1.2553946015096025, -1.283926296998457, -1.3124579924873117, -1.3409896879761662, -1.3695213834650208, -1.3980530789538754, -1.42658477444273, -1.4551164699315848, -1.4836481654204394, -1.5121798609092938, -1.5407115563981484, -1.569243251887003, -1.5977749473758578, -1.6263066428647124, -1.654838338353567, -1.6833700338424216, -1.711901729331276, -1.7404334248201305, -1.7689651203089853, -1.79749681579784, -1.8260285112866945, -1.8545602067755491, -1.8830919022644037, -1.911623597753258, -1.940155293242113, -1.9686869887309675, -1.9972186842198223, -2.025750379708677, -2.0542820751975315, -2.082813770686386, -2.11134546617524, -2.1398771616640953, -2.1684088571529494, -2.1969405526418044, -2.225472248130659, -2.2540039436195136, -2.282535639108368, -2.3110673345972224, -2.3395990300860774, -2.368130725574932, -2.3966624210637866, -2.425194116552641, -2.4537258120414958, -2.4822575075303503, -2.510789203019205, -2.5393208985080595, -2.567852593996914, -2.5963842894857687, -2.6249159849746233, -2.653447680463478, -2.6819793759523325, -2.7105110714411875, -2.7390427669300417, -2.7675744624188963, -2.796106157907751, -2.8246378533966054, -2.85316954888546 ], [ 0, -0.027893294576647538, -0.055786589153295076, -0.0836798837299426, -0.11157317830659015, -0.13946647288323769, -0.1673597674598852, -0.19525306203653275, -0.2231463566131803, -0.25103965118982785, -0.27893294576647537, -0.3068262403431229, -0.3347195349197704, -0.362612829496418, -0.3905061240730655, -0.41839941864971303, -0.4462927132263606, -0.4741860078030081, -0.5020793023796557, -0.5299725969563032, -0.5578658915329507, -0.5857591861095983, -0.6136524806862458, -0.6415457752628934, -0.6694390698395408, -0.6973323644161884, -0.725225658992836, -0.7531189535694834, -0.781012248146131, -0.8089055427227786, -0.8367988372994261, -0.8646921318760736, -0.8925854264527212, -0.9204787210293687, -0.9483720156060162, -0.9762653101826638, -1.0041586047593114, -1.0320518993359589, -1.0599451939126063, -1.0878384884892538, -1.1157317830659015, -1.143625077642549, -1.1715183722191966, -1.199411666795844, -1.2273049613724916, -1.255198255949139, -1.2830915505257867, -1.3109848451024342, -1.3388781396790816, -1.3667714342557293, -1.3946647288323768, -1.4225580234090245, -1.450451317985672, -1.4783446125623194, -1.5062379071389669, -1.5341312017156146, -1.562024496292262, -1.5899177908689095, -1.6178110854455572, -1.6457043800222046, -1.6735976745988521, -1.7014909691754996, -1.7293842637521473, -1.7572775583287947, -1.7851708529054424, -1.8130641474820899, -1.8409574420587373, -1.8688507366353848, -1.8967440312120325, -1.92463732578868, -1.9525306203653277, -1.9804239149419751, -2.008317209518623, -2.03621050409527, -2.0641037986719177, -2.0919970932485654, -2.1198903878252127, -2.1477836824018604, -2.1756769769785076, -2.2035702715551557, -2.231463566131803, -2.25935686070845, -2.287250155285098, -2.3151434498617456, -2.3430367444383933, -2.3709300390150405, -2.398823333591688, -2.4267166281683354, -2.454609922744983, -2.482503217321631, -2.510396511898278, -2.5382898064749257, -2.5661831010515734, -2.594076395628221, -2.6219696902048684, -2.649862984781516, -2.6777562793581633, -2.705649573934811, -2.7335428685114587, -2.761436163088106, -2.7893294576647536 ], [ 0, -0.027144811573980583, -0.054289623147961166, -0.08143443472194174, -0.10857924629592233, -0.1357240578699029, -0.16286886944388348, -0.1900136810178641, -0.21715849259184467, -0.24430330416582527, -0.2714481157398058, -0.2985929273137864, -0.32573773888776697, -0.3528825504617476, -0.3800273620357282, -0.4071721736097087, -0.43431698518368933, -0.46146179675766996, -0.48860660833165054, -0.515751419905631, -0.5428962314796116, -0.5700410430535923, -0.5971858546275728, -0.6243306662015534, -0.6514754777755339, -0.6786202893495146, -0.7057651009234952, -0.7329099124974757, -0.7600547240714564, -0.7871995356454369, -0.8143443472194174, -0.841489158793398, -0.8686339703673787, -0.8957787819413593, -0.9229235935153399, -0.9500684050893204, -0.9772132166633011, -1.0043580282372815, -1.031502839811262, -1.0586476513852427, -1.0857924629592233, -1.112937274533204, -1.1400820861071845, -1.167226897681165, -1.1943717092551456, -1.221516520829126, -1.2486613324031068, -1.2758061439770874, -1.3029509555510679, -1.3300957671250486, -1.3572405786990291, -1.3843853902730099, -1.4115302018469904, -1.438675013420971, -1.4658198249949514, -1.492964636568932, -1.5201094481429127, -1.5472542597168932, -1.5743990712908738, -1.6015438828648545, -1.6286886944388348, -1.6558335060128155, -1.682978317586796, -1.7101231291607768, -1.7372679407347573, -1.7644127523087378, -1.7915575638827186, -1.8187023754566989, -1.8458471870306798, -1.8729919986046601, -1.900136810178641, -1.9272816217526214, -1.9544264333266022, -1.9815712449005827, -2.008716056474563, -2.0358608680485437, -2.063005679622524, -2.090150491196505, -2.1172953027704855, -2.1444401143444662, -2.1715849259184465, -2.198729737492427, -2.225874549066408, -2.2530193606403883, -2.280164172214369, -2.3073089837883494, -2.33445379536233, -2.361598606936311, -2.388743418510291, -2.415888230084272, -2.443033041658252, -2.470177853232233, -2.4973226648062137, -2.5244674763801944, -2.5516122879541747, -2.5787570995281555, -2.6059019111021358, -2.6330467226761165, -2.6601915342500972, -2.6873363458240775, -2.7144811573980583 ], [ 0, -0.026289200401315903, -0.052578400802631806, -0.07886760120394772, -0.10515680160526361, -0.13144600200657952, -0.15773520240789543, -0.1840244028092113, -0.21031360321052722, -0.23660280361184316, -0.26289200401315904, -0.2891812044144749, -0.31547040481579086, -0.34175960521710674, -0.3680488056184226, -0.3943380060197385, -0.42062720642105444, -0.4469164068223704, -0.4732056072236863, -0.49949480762500215, -0.5257840080263181, -0.552073208427634, -0.5783624088289498, -0.6046516092302657, -0.6309408096315817, -0.6572300100328976, -0.6835192104342135, -0.7098084108355294, -0.7360976112368453, -0.7623868116381612, -0.788676012039477, -0.814965212440793, -0.8412544128421089, -0.8675436132434249, -0.8938328136447408, -0.9201220140460566, -0.9464112144473726, -0.9727004148486883, -0.9989896152500043, -1.0252788156513202, -1.0515680160526362, -1.0778572164539522, -1.104146416855268, -1.130435617256584, -1.1567248176578997, -1.1830140180592155, -1.2093032184605315, -1.2355924188618475, -1.2618816192631634, -1.2881708196644792, -1.3144600200657952, -1.3407492204671112, -1.367038420868427, -1.3933276212697427, -1.4196168216710587, -1.4459060220723747, -1.4721952224736905, -1.4984844228750065, -1.5247736232763225, -1.5510628236776383, -1.577352024078954, -1.60364122448027, -1.629930424881586, -1.6562196252829018, -1.6825088256842178, -1.7087980260855338, -1.7350872264868498, -1.7613764268881653, -1.7876656272894815, -1.8139548276907973, -1.8402440280921133, -1.866533228493429, -1.8928224288947453, -1.919111629296061, -1.9454008296973766, -1.9716900300986928, -1.9979792305000086, -2.0242684309013246, -2.0505576313026403, -2.0768468317039566, -2.1031360321052723, -2.129425232506588, -2.1557144329079043, -2.1820036333092196, -2.208292833710536, -2.2345820341118516, -2.260871234513168, -2.2871604349144836, -2.3134496353157994, -2.3397388357171156, -2.366028036118431, -2.392317236519747, -2.418606436921063, -2.444895637322379, -2.471184837723695, -2.497474038125011, -2.523763238526327, -2.5500524389276427, -2.5763416393289584, -2.602630839730274, -2.6289200401315904 ], [ 0, -0.025329837765060446, -0.05065967553012089, -0.07598951329518135, -0.10131935106024179, -0.12664918882530224, -0.1519790265903627, -0.17730886435542315, -0.20263870212048357, -0.22796853988554405, -0.2532983776506045, -0.2786282154156649, -0.3039580531807254, -0.32928789094578587, -0.3546177287108463, -0.3799475664759067, -0.40527740424096714, -0.4306072420060276, -0.4559370797710881, -0.4812669175361485, -0.506596755301209, -0.5319265930662694, -0.5572564308313298, -0.5825862685963903, -0.6079161063614508, -0.6332459441265113, -0.6585757818915717, -0.6839056196566321, -0.7092354574216926, -0.7345652951867531, -0.7598951329518134, -0.7852249707168739, -0.8105548084819343, -0.8358846462469948, -0.8612144840120552, -0.8865443217771157, -0.9118741595421762, -0.9372039973072365, -0.962533835072297, -0.9878636728373574, -1.013193510602418, -1.0385233483674783, -1.0638531861325389, -1.0891830238975992, -1.1145128616626596, -1.13984269942772, -1.1651725371927806, -1.190502374957841, -1.2158322127229015, -1.241162050487962, -1.2664918882530225, -1.2918217260180829, -1.3171515637831435, -1.3424814015482036, -1.3678112393132642, -1.3931410770783246, -1.4184709148433852, -1.4438007526084455, -1.4691305903735061, -1.4944604281385665, -1.5197902659036269, -1.5451201036686872, -1.5704499414337478, -1.5957797791988082, -1.6211096169638686, -1.6464394547289292, -1.6717692924939895, -1.69709913025905, -1.7224289680241105, -1.7477588057891709, -1.7730886435542315, -1.7984184813192918, -1.8237483190843524, -1.8490781568494128, -1.874407994614473, -1.8997378323795338, -1.925067670144594, -1.9503975079096547, -1.9757273456747149, -2.0010571834397757, -2.026387021204836, -2.051716858969896, -2.0770466967349566, -2.102376534500017, -2.1277063722650777, -2.153036210030138, -2.1783660477951985, -2.203695885560259, -2.2290257233253192, -2.25435556109038, -2.27968539885544, -2.305015236620501, -2.330345074385561, -2.3556749121506217, -2.381004749915682, -2.406334587680743, -2.431664425445803, -2.4569942632108632, -2.482324100975924, -2.5076539387409844, -2.532983776506045 ], [ 0, -0.024270509831248416, -0.04854101966249683, -0.07281152949374525, -0.09708203932499367, -0.12135254915624208, -0.1456230589874905, -0.1698935688187389, -0.19416407864998733, -0.21843458848123576, -0.24270509831248416, -0.26697560814373256, -0.291246117974981, -0.3155166278062294, -0.3397871376374778, -0.3640576474687262, -0.38832815729997466, -0.4125986671312231, -0.4368691769624715, -0.46113968679371986, -0.4854101966249683, -0.5096807064562168, -0.5339512162874651, -0.5582217261187136, -0.582492235949962, -0.6067627457812104, -0.6310332556124588, -0.6553037654437072, -0.6795742752749556, -0.7038447851062041, -0.7281152949374524, -0.7523858047687009, -0.7766563145999493, -0.8009268244311978, -0.8251973342624462, -0.8494678440936946, -0.873738353924943, -0.8980088637561914, -0.9222793735874397, -0.9465498834186882, -0.9708203932499366, -0.9950909030811851, -1.0193614129124335, -1.0436319227436819, -1.0679024325749302, -1.0921729424061786, -1.1164434522374271, -1.1407139620686755, -1.164984471899924, -1.1892549817311724, -1.2135254915624207, -1.2377960013936693, -1.2620665112249176, -1.286337021056166, -1.3106075308874143, -1.334878040718663, -1.3591485505499112, -1.3834190603811598, -1.4076895702124081, -1.4319600800436567, -1.4562305898749048, -1.4805010997061534, -1.5047716095374017, -1.52904211936865, -1.5533126291998987, -1.577583139031147, -1.6018536488623956, -1.6261241586936437, -1.6503946685248925, -1.6746651783561406, -1.6989356881873892, -1.7232061980186375, -1.747476707849886, -1.7717472176811344, -1.7960177275123828, -1.8202882373436313, -1.8445587471748794, -1.8688292570061282, -1.8930997668373764, -1.917370276668625, -1.9416407864998733, -1.9659112963311216, -1.9901818061623702, -2.0144523159936183, -2.038722825824867, -2.0629933356561154, -2.0872638454873638, -2.111534355318612, -2.1358048651498605, -2.1600753749811092, -2.184345884812357, -2.208616394643606, -2.2328869044748543, -2.2571574143061026, -2.281427924137351, -2.3056984339685997, -2.329968943799848, -2.354239453631096, -2.378509963462345, -2.402780473293593, -2.4270509831248415 ], [ 0, -0.02311539728327367, -0.04623079456654734, -0.069346191849821, -0.09246158913309468, -0.11557698641636835, -0.138692383699642, -0.16180778098291568, -0.18492317826618937, -0.20803857554946306, -0.2311539728327367, -0.25426937011601036, -0.277384767399284, -0.30050016468255775, -0.32361556196583136, -0.346730959249105, -0.36984635653237874, -0.3929617538156524, -0.4160771510989261, -0.43919254838219973, -0.4623079456654734, -0.4854233429487471, -0.5085387402320207, -0.5316541375152944, -0.554769534798568, -0.5778849320818418, -0.6010003293651155, -0.6241157266483891, -0.6472311239316627, -0.6703465212149364, -0.69346191849821, -0.7165773157814838, -0.7396927130647575, -0.7628081103480311, -0.7859235076313048, -0.8090389049145785, -0.8321543021978522, -0.8552696994811257, -0.8783850967643995, -0.9015004940476731, -0.9246158913309468, -0.9477312886142205, -0.9708466858974942, -0.9939620831807678, -1.0170774804640415, -1.040192877747315, -1.063308275030589, -1.0864236723138625, -1.109539069597136, -1.13265446688041, -1.1557698641636835, -1.1788852614469572, -1.202000658730231, -1.2251160560135044, -1.2482314532967782, -1.2713468505800518, -1.2944622478633254, -1.3175776451465993, -1.3406930424298729, -1.3638084397131467, -1.38692383699642, -1.410039234279694, -1.4331546315629675, -1.4562700288462411, -1.479385426129515, -1.5025008234127886, -1.5256162206960622, -1.5487316179793358, -1.5718470152626096, -1.5949624125458832, -1.618077809829157, -1.6411932071124307, -1.6643086043957045, -1.6874240016789779, -1.7105393989622515, -1.7336547962455253, -1.756770193528799, -1.7798855908120728, -1.8030009880953461, -1.82611638537862, -1.8492317826618936, -1.8723471799451672, -1.895462577228441, -1.9185779745117144, -1.9416933717949885, -1.9648087690782619, -1.9879241663615357, -2.0110395636448093, -2.034154960928083, -2.0572703582113565, -2.08038575549463, -2.103501152777904, -2.126616550061178, -2.1497319473444514, -2.172847344627725, -2.1959627419109986, -2.219078139194272, -2.242193536477546, -2.26530893376082, -2.288424331044093, -2.311539728327367 ], [ 0, -0.021869058822642347, -0.043738117645284694, -0.06560717646792703, -0.08747623529056939, -0.10934529411321173, -0.13121435293585407, -0.1530834117584964, -0.17495247058113877, -0.19682152940378114, -0.21869058822642345, -0.2405596470490658, -0.26242870587170813, -0.28429776469435053, -0.3061668235169928, -0.32803588233963515, -0.34990494116227755, -0.3717739999849199, -0.3936430588075623, -0.41551211763020457, -0.4373811764528469, -0.4592502352754893, -0.4811192940981316, -0.5029883529207739, -0.5248574117434163, -0.5467264705660586, -0.5685955293887011, -0.5904645882113433, -0.6123336470339856, -0.6342027058566281, -0.6560717646792703, -0.6779408235019126, -0.6998098823245551, -0.7216789411471974, -0.7435479999698398, -0.7654170587924821, -0.7872861176151246, -0.8091551764377667, -0.8310242352604091, -0.8528932940830515, -0.8747623529056938, -0.8966314117283362, -0.9185004705509786, -0.940369529373621, -0.9622385881962632, -0.9841076470189055, -1.0059767058415479, -1.0278457646641903, -1.0497148234868325, -1.071583882309475, -1.0934529411321172, -1.1153219999547597, -1.1371910587774021, -1.1590601176000443, -1.1809291764226866, -1.202798235245329, -1.2246672940679713, -1.2465363528906137, -1.2684054117132562, -1.2902744705358984, -1.3121435293585406, -1.334012588181183, -1.3558816470038253, -1.3777507058264677, -1.3996197646491102, -1.4214888234717524, -1.4433578822943949, -1.465226941117037, -1.4870959999396796, -1.5089650587623218, -1.5308341175849642, -1.5527031764076065, -1.5745722352302491, -1.5964412940528914, -1.6183103528755334, -1.640179411698176, -1.6620484705208183, -1.6839175293434607, -1.705786588166103, -1.7276556469887454, -1.7495247058113876, -1.7713937646340299, -1.7932628234566723, -1.8151318822793145, -1.8370009411019572, -1.8588699999245992, -1.880739058747242, -1.9026081175698841, -1.9244771763925264, -1.9463462352151688, -1.968215294037811, -1.9900843528604535, -2.0119534116830957, -2.033822470505738, -2.0556915293283806, -2.077560588151023, -2.099429646973665, -2.1212987057963075, -2.14316776461895, -2.165036823441592, -2.1869058822642344 ], [ 0, -0.020536413177860657, -0.04107282635572131, -0.06160923953358197, -0.08214565271144263, -0.10268206588930329, -0.12321847906716395, -0.1437548922450246, -0.16429130542288525, -0.18482771860074593, -0.20536413177860657, -0.22590054495646722, -0.2464369581343279, -0.26697337131218857, -0.2875097844900492, -0.30804619766790986, -0.3285826108457705, -0.3491190240236312, -0.36965543720149185, -0.3901918503793525, -0.41072826355721315, -0.43126467673507385, -0.45180108991293444, -0.4723375030907951, -0.4928739162686558, -0.5134103294465164, -0.5339467426243771, -0.5544831558022377, -0.5750195689800984, -0.5955559821579591, -0.6160923953358197, -0.6366288085136804, -0.657165221691541, -0.6777016348694017, -0.6982380480472624, -0.7187744612251231, -0.7393108744029837, -0.7598472875808443, -0.780383700758705, -0.8009201139365656, -0.8214565271144263, -0.841992940292287, -0.8625293534701477, -0.8830657666480083, -0.9036021798258689, -0.9241385930037296, -0.9446750061815902, -0.9652114193594509, -0.9857478325373116, -1.0062842457151722, -1.0268206588930329, -1.0473570720708936, -1.0678934852487543, -1.0884298984266148, -1.1089663116044755, -1.1295027247823362, -1.1500391379601969, -1.1705755511380576, -1.1911119643159183, -1.211648377493779, -1.2321847906716394, -1.2527212038495001, -1.2732576170273608, -1.2937940302052213, -1.314330443383082, -1.3348668565609427, -1.3554032697388034, -1.375939682916664, -1.3964760960945248, -1.4170125092723853, -1.4375489224502462, -1.4580853356281067, -1.4786217488059674, -1.4991581619838281, -1.5196945751616886, -1.5402309883395493, -1.56076740151741, -1.5813038146952707, -1.6018402278731312, -1.622376641050992, -1.6429130542288526, -1.663449467406713, -1.683985880584574, -1.7045222937624345, -1.7250587069402954, -1.7455951201181559, -1.7661315332960166, -1.7866679464738773, -1.8072043596517378, -1.8277407728295985, -1.8482771860074592, -1.8688135991853199, -1.8893500123631803, -1.9098864255410413, -1.9304228387189017, -1.9509592518967627, -1.9714956650746231, -1.9920320782524836, -2.0125684914303443, -2.033104904608205, -2.0536413177860657 ], [ 0, -0.01912271969246069, -0.03824543938492138, -0.05736815907738207, -0.07649087876984276, -0.09561359846230344, -0.11473631815476414, -0.13385903784722483, -0.15298175753968551, -0.17210447723214622, -0.19122719692460688, -0.21034991661706756, -0.22947263630952827, -0.24859535600198895, -0.26771807569444966, -0.2868407953869103, -0.30596351507937103, -0.32508623477183174, -0.34420895446429245, -0.36333167415675305, -0.38245439384921376, -0.40157711354167447, -0.4206998332341351, -0.43982255292659583, -0.45894527261905654, -0.4780679923115172, -0.4971907120039779, -0.5163134316964386, -0.5354361513888993, -0.5545588710813599, -0.5736815907738206, -0.5928043104662813, -0.6119270301587421, -0.6310497498512028, -0.6501724695436635, -0.6692951892361242, -0.6884179089285849, -0.7075406286210454, -0.7266633483135061, -0.7457860680059668, -0.7649087876984275, -0.7840315073908882, -0.8031542270833489, -0.8222769467758096, -0.8413996664682702, -0.860522386160731, -0.8796451058531917, -0.8987678255456524, -0.9178905452381131, -0.9370132649305738, -0.9561359846230344, -0.9752587043154951, -0.9943814240079558, -1.0135041437004164, -1.0326268633928772, -1.0517495830853378, -1.0708723027777987, -1.0899950224702593, -1.1091177421627199, -1.1282404618551807, -1.1473631815476413, -1.1664859012401019, -1.1856086209325627, -1.2047313406250233, -1.2238540603174841, -1.2429767800099447, -1.2620994997024055, -1.2812222193948661, -1.300344939087327, -1.3194676587797873, -1.3385903784722484, -1.3577130981647088, -1.3768358178571698, -1.3959585375496302, -1.4150812572420908, -1.4342039769345516, -1.4533266966270122, -1.472449416319473, -1.4915721360119336, -1.5106948557043944, -1.529817575396855, -1.5489402950893156, -1.5680630147817765, -1.587185734474237, -1.6063084541666979, -1.6254311738591585, -1.6445538935516193, -1.66367661324408, -1.6827993329365405, -1.7019220526290013, -1.721044772321462, -1.7401674920139227, -1.7592902117063833, -1.7784129313988442, -1.7975356510913048, -1.8166583707837656, -1.8357810904762262, -1.8549038101686866, -1.8740265298611476, -1.893149249553608, -1.9122719692460688 ], [ 0, -0.017633557568774196, -0.03526711513754839, -0.05290067270632259, -0.07053423027509678, -0.08816778784387098, -0.10580134541264519, -0.12343490298141938, -0.14106846055019356, -0.15870201811896778, -0.17633557568774197, -0.19396913325651616, -0.21160269082529037, -0.22923624839406456, -0.24686980596283875, -0.2645033635316129, -0.28213692110038713, -0.29977047866916134, -0.31740403623793556, -0.3350375938067097, -0.35267115137548394, -0.37030470894425815, -0.3879382665130323, -0.40557182408180653, -0.42320538165058075, -0.4408389392193549, -0.4584724967881291, -0.4761060543569033, -0.4937396119256775, -0.5113731694944518, -0.5290067270632258, -0.5466402846320001, -0.5642738422007743, -0.5819073997695485, -0.5995409573383227, -0.617174514907097, -0.6348080724758711, -0.6524416300446453, -0.6700751876134194, -0.6877087451821936, -0.7053423027509679, -0.722975860319742, -0.7406094178885163, -0.7582429754572905, -0.7758765330260646, -0.7935100905948388, -0.8111436481636131, -0.8287772057323872, -0.8464107633011615, -0.8640443208699357, -0.8816778784387098, -0.8993114360074841, -0.9169449935762582, -0.9345785511450324, -0.9522121087138066, -0.9698456662825808, -0.987479223851355, -1.0051127814201293, -1.0227463389889035, -1.0403798965576776, -1.0580134541264516, -1.075647011695226, -1.0932805692640002, -1.1109141268327745, -1.1285476844015485, -1.1461812419703228, -1.163814799539097, -1.181448357107871, -1.1990819146766454, -1.2167154722454194, -1.234349029814194, -1.251982587382968, -1.2696161449517422, -1.2872497025205163, -1.3048832600892906, -1.3225168176580648, -1.3401503752268389, -1.3577839327956132, -1.3754174903643872, -1.3930510479331617, -1.4106846055019358, -1.4283181630707098, -1.445951720639484, -1.4635852782082583, -1.4812188357770326, -1.4988523933458067, -1.516485950914581, -1.5341195084833552, -1.5517530660521293, -1.5693866236209035, -1.5870201811896776, -1.6046537387584519, -1.6222872963272261, -1.6399208538960004, -1.6575544114647744, -1.6751879690335487, -1.692821526602323, -1.710455084171097, -1.7280886417398713, -1.7457221993086454, -1.7633557568774196 ], [ 0, -0.01607480384936989, -0.03214960769873978, -0.04822441154810967, -0.06429921539747956, -0.08037401924684945, -0.09644882309621934, -0.11252362694558922, -0.1285984307949591, -0.14467323464432902, -0.1607480384936989, -0.17682284234306878, -0.19289764619243868, -0.20897245004180856, -0.22504725389117844, -0.24112205774054832, -0.2571968615899182, -0.2732716654392881, -0.28934646928865804, -0.30542127313802786, -0.3214960769873978, -0.3375708808367677, -0.35364568468613755, -0.36972048853550743, -0.38579529238487736, -0.40187009623424724, -0.4179449000836171, -0.434019703932987, -0.4500945077823569, -0.4661693116317268, -0.48224411548109664, -0.49831891933046657, -0.5143937231798364, -0.5304685270292063, -0.5465433308785762, -0.5626181347279462, -0.5786929385773161, -0.5947677424266858, -0.6108425462760557, -0.6269173501254257, -0.6429921539747956, -0.6590669578241655, -0.6751417616735353, -0.6912165655229052, -0.7072913693722751, -0.723366173221645, -0.7394409770710149, -0.7555157809203847, -0.7715905847697547, -0.7876653886191246, -0.8037401924684945, -0.8198149963178644, -0.8358898001672342, -0.8519646040166041, -0.868039407865974, -0.8841142117153439, -0.9001890155647138, -0.9162638194140837, -0.9323386232634536, -0.9484134271128235, -0.9644882309621933, -0.9805630348115632, -0.9966378386609331, -1.012712642510303, -1.028787446359673, -1.0448622502090428, -1.0609370540584127, -1.0770118579077825, -1.0930866617571524, -1.1091614656065223, -1.1252362694558924, -1.141311073305262, -1.1573858771546321, -1.173460681004002, -1.1895354848533717, -1.2056102887027418, -1.2216850925521114, -1.2377598964014815, -1.2538347002508514, -1.2699095041002213, -1.2859843079495912, -1.3020591117989608, -1.318133915648331, -1.3342087194977008, -1.3502835233470707, -1.3663583271964406, -1.3824331310458104, -1.3985079348951803, -1.4145827387445502, -1.43065754259392, -1.44673234644329, -1.46280715029266, -1.4788819541420297, -1.4949567579913998, -1.5110315618407695, -1.5271063656901396, -1.5431811695395095, -1.5592559733888791, -1.5753307772382492, -1.5914055810876189, -1.607480384936989 ], [ 0, -0.014452610223051459, -0.028905220446102917, -0.04335783066915438, -0.057810440892205835, -0.0722630511152573, -0.08671566133830876, -0.10116827156136021, -0.11562088178441167, -0.13007349200746315, -0.1445261022305146, -0.15897871245356604, -0.1734313226766175, -0.18788393289966898, -0.20233654312272042, -0.21678915334577187, -0.23124176356882334, -0.2456943737918748, -0.2601469840149263, -0.2745995942379777, -0.2890522044610292, -0.30350481468408064, -0.3179574249071321, -0.3324100351301835, -0.346862645353235, -0.3613152555762865, -0.37576786579933796, -0.3902204760223894, -0.40467308624544085, -0.41912569646849235, -0.43357830669154374, -0.44803091691459523, -0.4624835271376467, -0.4769361373606982, -0.4913887475837496, -0.5058413578068011, -0.5202939680298526, -0.534746578252904, -0.5491991884759554, -0.563651798699007, -0.5781044089220584, -0.5925570191451098, -0.6070096293681613, -0.6214622395912128, -0.6359148498142642, -0.6503674600373156, -0.664820070260367, -0.6792726804834186, -0.69372529070647, -0.7081779009295215, -0.722630511152573, -0.7370831213756245, -0.7515357315986759, -0.7659883418217273, -0.7804409520447788, -0.7948935622678303, -0.8093461724908817, -0.8237987827139331, -0.8382513929369847, -0.8527040031600361, -0.8671566133830875, -0.8816092236061389, -0.8960618338291905, -0.9105144440522419, -0.9249670542752934, -0.9394196644983449, -0.9538722747213964, -0.9683248849444477, -0.9827774951674992, -0.9972301053905507, -1.0116827156136021, -1.0261353258366537, -1.0405879360597052, -1.0550405462827566, -1.069493156505808, -1.0839457667288595, -1.0983983769519108, -1.1128509871749623, -1.127303597398014, -1.1417562076210654, -1.1562088178441168, -1.170661428067168, -1.1851140382902197, -1.199566648513271, -1.2140192587363225, -1.2284718689593739, -1.2429244791824257, -1.257377089405477, -1.2718296996285283, -1.2862823098515799, -1.3007349200746312, -1.3151875302976828, -1.329640140520734, -1.3440927507437859, -1.3585453609668372, -1.3729979711898888, -1.38745058141294, -1.4019031916359914, -1.416355801859043, -1.4308084120820943, -1.445261022305146 ], [ 0, -0.012773378746952163, -0.025546757493904327, -0.03832013624085649, -0.051093514987808654, -0.06386689373476082, -0.07664027248171298, -0.08941365122866515, -0.10218702997561731, -0.11496040872256949, -0.12773378746952163, -0.1405071662164738, -0.15328054496342597, -0.16605392371037814, -0.1788273024573303, -0.19160068120428245, -0.20437405995123462, -0.2171474386981868, -0.22992081744513898, -0.24269419619209112, -0.25546757493904326, -0.2682409536859955, -0.2810143324329476, -0.29378771117989977, -0.30656108992685194, -0.3193344686738041, -0.3321078474207563, -0.34488122616770844, -0.3576546049146606, -0.3704279836616128, -0.3832013624085649, -0.39597474115551706, -0.40874811990246923, -0.42152149864942146, -0.4342948773963736, -0.4470682561433258, -0.45984163489027796, -0.47261501363723, -0.48538839238418224, -0.4981617711311344, -0.5109351498780865, -0.5237085286250387, -0.536481907371991, -0.5492552861189431, -0.5620286648658952, -0.5748020436128474, -0.5875754223597995, -0.6003488011067517, -0.6131221798537039, -0.625895558600656, -0.6386689373476082, -0.6514423160945604, -0.6642156948415125, -0.6769890735884647, -0.6897624523354169, -0.7025358310823691, -0.7153092098293212, -0.7280825885762734, -0.7408559673232256, -0.7536293460701777, -0.7664027248171298, -0.779176103564082, -0.7919494823110341, -0.8047228610579863, -0.8174962398049385, -0.8302696185518907, -0.8430429972988429, -0.855816376045795, -0.8685897547927472, -0.8813631335396993, -0.8941365122866516, -0.9069098910336036, -0.9196832697805559, -0.932456648527508, -0.94523002727446, -0.9580034060214123, -0.9707767847683645, -0.9835501635153168, -0.9963235422622688, -1.009096921009221, -1.021870299756173, -1.0346436785031252, -1.0474170572500774, -1.0601904359970296, -1.072963814743982, -1.085737193490934, -1.0985105722378863, -1.1112839509848382, -1.1240573297317904, -1.1368307084787426, -1.1496040872256947, -1.162377465972647, -1.175150844719599, -1.1879242234665512, -1.2006976022135034, -1.2134709809604558, -1.2262443597074077, -1.23901773845436, -1.251791117201312, -1.2645644959482643, -1.2773378746952164 ], [ 0, -0.011043736580540333, -0.022087473161080666, -0.033131209741620996, -0.04417494632216133, -0.05521868290270166, -0.06626241948324199, -0.07730615606378233, -0.08834989264432266, -0.099393629224863, -0.11043736580540332, -0.12148110238594365, -0.13252483896648398, -0.14356857554702432, -0.15461231212756466, -0.16565604870810496, -0.17669978528864533, -0.18774352186918566, -0.198787258449726, -0.2098309950302663, -0.22087473161080665, -0.23191846819134698, -0.2429622047718873, -0.25400594135242766, -0.26504967793296796, -0.27609341451350833, -0.28713715109404864, -0.29818088767458895, -0.3092246242551293, -0.3202683608356696, -0.3313120974162099, -0.3423558339967503, -0.35339957057729066, -0.36444330715783096, -0.37548704373837133, -0.38653078031891164, -0.397574516899452, -0.40861825347999225, -0.4196619900605326, -0.43070572664107293, -0.4417494632216133, -0.45279319980215366, -0.46383693638269397, -0.47488067296323433, -0.4859244095437746, -0.4969681461243149, -0.5080118827048553, -0.5190556192853956, -0.5300993558659359, -0.5411430924464763, -0.5521868290270167, -0.563230565607557, -0.5742743021880973, -0.5853180387686375, -0.5963617753491779, -0.6074055119297183, -0.6184492485102586, -0.629492985090799, -0.6405367216713392, -0.6515804582518796, -0.6626241948324199, -0.6736679314129602, -0.6847116679935006, -0.695755404574041, -0.7067991411545813, -0.7178428777351216, -0.7288866143156619, -0.7399303508962022, -0.7509740874767427, -0.7620178240572829, -0.7730615606378233, -0.7841052972183635, -0.795149033798904, -0.8061927703794443, -0.8172365069599845, -0.828280243540525, -0.8393239801210652, -0.8503677167016056, -0.8614114532821459, -0.8724551898626863, -0.8834989264432266, -0.8945426630237668, -0.9055863996043073, -0.9166301361848476, -0.9276738727653879, -0.9387176093459282, -0.9497613459264687, -0.9608050825070089, -0.9718488190875492, -0.9828925556680895, -0.9939362922486298, -1.0049800288291704, -1.0160237654097106, -1.0270675019902509, -1.0381112385707911, -1.0491549751513316, -1.0601987117318719, -1.071242448312412, -1.0822861848929526, -1.0933299214734928, -1.1043736580540333 ], [ 0, -0.009270509831248426, -0.01854101966249685, -0.02781152949374528, -0.0370820393249937, -0.04635254915624213, -0.05562305898749056, -0.06489356881873899, -0.0741640786499874, -0.08343458848123585, -0.09270509831248426, -0.10197560814373269, -0.11124611797498112, -0.12051662780622956, -0.12978713763747798, -0.1390576474687264, -0.1483281572999748, -0.15759866713122325, -0.1668691769624717, -0.1761396867937201, -0.18541019662496852, -0.19468070645621696, -0.20395121628746538, -0.2132217261187138, -0.22249223594996223, -0.23176274578121067, -0.2410332556124591, -0.2503037654437075, -0.25957427527495597, -0.26884478510620435, -0.2781152949374528, -0.28738580476870124, -0.2966563145999496, -0.30592682443119806, -0.3151973342624465, -0.32446784409369495, -0.3337383539249434, -0.3430088637561918, -0.3522793735874402, -0.3615498834186886, -0.37082039324993704, -0.3800909030811855, -0.3893614129124339, -0.39863192274368237, -0.40790243257493075, -0.4171729424061792, -0.4264434522374276, -0.435713962068676, -0.44498447189992446, -0.4542549817311729, -0.46352549156242134, -0.4727960013936698, -0.4820665112249182, -0.49133702105616656, -0.500607530887415, -0.5098780407186635, -0.5191485505499119, -0.5284190603811603, -0.5376895702124087, -0.5469600800436571, -0.5562305898749056, -0.565501099706154, -0.5747716095374025, -0.5840421193686509, -0.5933126291998992, -0.6025831390311477, -0.6118536488623961, -0.6211241586936446, -0.630394668524893, -0.6396651783561415, -0.6489356881873899, -0.6582061980186382, -0.6674767078498868, -0.6767472176811351, -0.6860177275123835, -0.695288237343632, -0.7045587471748804, -0.7138292570061289, -0.7230997668373772, -0.7323702766686258, -0.7416407864998741, -0.7509112963311225, -0.760181806162371, -0.7694523159936194, -0.7787228258248678, -0.7879933356561162, -0.7972638454873647, -0.8065343553186131, -0.8158048651498615, -0.8250753749811099, -0.8343458848123584, -0.8436163946436068, -0.8528869044748552, -0.8621574143061037, -0.871427924137352, -0.8806984339686006, -0.8899689437998489, -0.8992394536310974, -0.9085099634623458, -0.9177804732935941, -0.9270509831248427 ], [ 0, -0.007460696614945633, -0.014921393229891266, -0.0223820898448369, -0.02984278645978253, -0.037303483074728166, -0.0447641796896738, -0.05222487630461943, -0.05968557291956506, -0.06714626953451071, -0.07460696614945633, -0.08206766276440196, -0.0895283593793476, -0.09698905599429324, -0.10444975260923886, -0.11191044922418449, -0.11937114583913012, -0.12683184245407578, -0.13429253906902142, -0.14175323568396703, -0.14921393229891267, -0.1566746289138583, -0.16413532552880392, -0.17159602214374955, -0.1790567187586952, -0.18651741537364083, -0.19397811198858647, -0.20143880860353208, -0.20889950521847772, -0.21636020183342336, -0.22382089844836897, -0.2312815950633146, -0.23874229167826025, -0.2462029882932059, -0.25366368490815155, -0.2611243815230972, -0.26858507813804283, -0.2760457747529884, -0.28350647136793405, -0.2909671679828797, -0.29842786459782533, -0.30588856121277097, -0.3133492578277166, -0.32080995444266225, -0.32827065105760783, -0.33573134767255347, -0.3431920442874991, -0.35065274090244475, -0.3581134375173904, -0.365574134132336, -0.37303483074728166, -0.3804955273622273, -0.38795622397717294, -0.3954169205921185, -0.40287761720706416, -0.4103383138220098, -0.41779901043695544, -0.4252597070519011, -0.4327204036668467, -0.44018110028179236, -0.44764179689673794, -0.4551024935116836, -0.4625631901266292, -0.47002388674157486, -0.4774845833565205, -0.48494527997146614, -0.4924059765864118, -0.49986667320135736, -0.5073273698163031, -0.5147880664312486, -0.5222487630461944, -0.5297094596611399, -0.5371701562760857, -0.5446308528910312, -0.5520915495059768, -0.5595522461209225, -0.5670129427358681, -0.5744736393508137, -0.5819343359657594, -0.589395032580705, -0.5968557291956507, -0.6043164258105962, -0.6117771224255419, -0.6192378190404875, -0.6266985156554332, -0.6341592122703787, -0.6416199088853245, -0.64908060550027, -0.6565413021152157, -0.6640019987301613, -0.6714626953451069, -0.6789233919600526, -0.6863840885749982, -0.6938447851899439, -0.7013054818048895, -0.7087661784198351, -0.7162268750347808, -0.7236875716497263, -0.731148268264672, -0.7386089648796176, -0.7460696614945633 ], [ 0, -0.005621439437571739, -0.011242878875143478, -0.016864318312715215, -0.022485757750286956, -0.028107197187858694, -0.03372863662543043, -0.03935007606300217, -0.04497151550057391, -0.050592954938145654, -0.05621439437571739, -0.06183583381328912, -0.06745727325086086, -0.0730787126884326, -0.07870015212600434, -0.08432159156357608, -0.08994303100114782, -0.09556447043871956, -0.10118590987629131, -0.10680734931386303, -0.11242878875143478, -0.11805022818900651, -0.12367166762657825, -0.12929310706414998, -0.13491454650172172, -0.14053598593929348, -0.1461574253768652, -0.15177886481443695, -0.15740030425200868, -0.16302174368958042, -0.16864318312715215, -0.1742646225647239, -0.17988606200229565, -0.18550750143986738, -0.19112894087743912, -0.19675038031501088, -0.20237181975258262, -0.20799325919015432, -0.21361469862772606, -0.2192361380652978, -0.22485757750286955, -0.2304790169404413, -0.23610045637801302, -0.2417218958155848, -0.2473433352531565, -0.2529647746907282, -0.25858621412829996, -0.2642076535658717, -0.26982909300344343, -0.2754505324410152, -0.28107197187858696, -0.28669341131615866, -0.2923148507537304, -0.29793629019130213, -0.3035577296288739, -0.3091791690664456, -0.31480060850401737, -0.3204220479415891, -0.32604348737916083, -0.3316649268167326, -0.3372863662543043, -0.34290780569187607, -0.3485292451294478, -0.35415068456701954, -0.3597721240045913, -0.365393563442163, -0.37101500287973477, -0.3766364423173065, -0.38225788175487824, -0.38787932119244994, -0.39350076063002176, -0.3991222000675934, -0.40474363950516523, -0.41036507894273694, -0.41598651838030865, -0.4216079578178804, -0.4272293972554521, -0.4328508366930239, -0.4384722761305956, -0.4440937155681674, -0.4497151550057391, -0.4553365944433108, -0.4609580338808826, -0.4665794733184543, -0.47220091275602605, -0.47782235219359775, -0.4834437916311696, -0.4890652310687413, -0.494686670506313, -0.5003081099438847, -0.5059295493814564, -0.5115509888190283, -0.5171724282565999, -0.5227938676941717, -0.5284153071317435, -0.5340367465693152, -0.5396581860068869, -0.5452796254444586, -0.5509010648820304, -0.5565225043196022, -0.5621439437571739 ], [ 0, -0.003759997006929112, -0.007519994013858224, -0.011279991020787335, -0.015039988027716448, -0.01879998503464556, -0.02255998204157467, -0.02631997904850378, -0.030079976055432896, -0.03383997306236201, -0.03759997006929112, -0.04135996707622023, -0.04511996408314934, -0.04887996109007846, -0.05263995809700756, -0.05639995510393667, -0.06015995211086579, -0.0639199491177949, -0.06767994612472401, -0.07143994313165312, -0.07519994013858224, -0.07895993714551135, -0.08271993415244046, -0.08647993115936957, -0.09023992816629868, -0.0939999251732278, -0.09775992218015692, -0.10151991918708601, -0.10527991619401512, -0.10903991320094425, -0.11279991020787335, -0.11655990721480246, -0.12031990422173158, -0.1240799012286607, -0.1278398982355898, -0.13159989524251892, -0.13535989224944803, -0.13911988925637714, -0.14287988626330625, -0.14663988327023536, -0.15039988027716447, -0.15415987728409358, -0.1579198742910227, -0.1616798712979518, -0.16543986830488092, -0.16919986531181003, -0.17295986231873914, -0.17671985932566825, -0.18047985633259736, -0.18423985333952647, -0.1879998503464556, -0.19175984735338472, -0.19551984436031383, -0.19927984136724292, -0.20303983837417203, -0.20679983538110114, -0.21055983238803025, -0.2143198293949594, -0.2180798264018885, -0.2218398234088176, -0.2255998204157467, -0.2293598174226758, -0.23311981442960492, -0.23687981143653403, -0.24063980844346317, -0.24439980545039228, -0.2481598024573214, -0.25191979946425047, -0.2556797964711796, -0.2594397934781087, -0.26319979048503783, -0.2669597874919669, -0.27071978449889605, -0.27447978150582514, -0.2782397785127543, -0.2819997755196834, -0.2857597725266125, -0.28951976953354164, -0.2932797665404707, -0.29703976354739986, -0.30079976055432894, -0.304559757561258, -0.30831975456818717, -0.31207975157511625, -0.3158397485820454, -0.31959974558897447, -0.3233597425959036, -0.32711973960283275, -0.33087973660976183, -0.33463973361669097, -0.33839973062362005, -0.3421597276305492, -0.3459197246374783, -0.3496797216444074, -0.3534397186513365, -0.35719971565826564, -0.3609597126651947, -0.3647197096721238, -0.36847970667905294, -0.372239703685982, -0.3759997006929112 ], [ 0, -0.0018837155858793958, -0.0037674311717587916, -0.0056511467576381876, -0.007534862343517583, -0.009418577929396979, -0.011302293515276375, -0.01318600910115577, -0.015069724687035166, -0.016953440272914563, -0.018837155858793957, -0.020720871444673352, -0.02260458703055275, -0.024488302616432145, -0.02637201820231154, -0.028255733788190934, -0.030139449374070332, -0.03202316495994973, -0.033906880545829125, -0.03579059613170852, -0.037674311717587915, -0.03955802730346731, -0.041441742889346704, -0.0433254584752261, -0.0452091740611055, -0.047092889646984895, -0.04897660523286429, -0.050860320818743684, -0.05274403640462308, -0.05462775199050248, -0.05651146757638187, -0.05839518316226126, -0.060278898748140665, -0.06216261433402006, -0.06404632991989946, -0.06593004550577886, -0.06781376109165825, -0.06969747667753763, -0.07158119226341704, -0.07346490784929643, -0.07534862343517583, -0.07723233902105522, -0.07911605460693462, -0.08099977019281403, -0.08288348577869341, -0.0847672013645728, -0.0866509169504522, -0.08853463253633159, -0.090418348122211, -0.0923020637080904, -0.09418577929396979, -0.09606949487984918, -0.09795321046572858, -0.09983692605160797, -0.10172064163748737, -0.10360435722336676, -0.10548807280924616, -0.10737178839512555, -0.10925550398100496, -0.11113921956688436, -0.11302293515276374, -0.11490665073864313, -0.11679036632452253, -0.11867408191040194, -0.12055779749628133, -0.12244151308216072, -0.12432522866804012, -0.12620894425391951, -0.12809265983979892, -0.1299763754256783, -0.1318600910115577, -0.1337438065974371, -0.1356275221833165, -0.13751123776919588, -0.13939495335507526, -0.1412786689409547, -0.14316238452683408, -0.1450461001127135, -0.14692981569859287, -0.14881353128447228, -0.15069724687035166, -0.15258096245623104, -0.15446467804211045, -0.15634839362798983, -0.15823210921386924, -0.16011582479974862, -0.16199954038562805, -0.16388325597150744, -0.16576697155738682, -0.16765068714326622, -0.1695344027291456, -0.17141811831502501, -0.1733018339009044, -0.1751855494867838, -0.17706926507266318, -0.17895298065854262, -0.180836696244422, -0.18272041183030138, -0.1846041274161808, -0.18648784300206017, -0.18837155858793958 ], [ 0, -5.510910596163089e-18, -1.1021821192326178e-17, -1.6532731788489266e-17, -2.2043642384652355e-17, -2.7554552980815445e-17, -3.306546357697853e-17, -3.857637417314162e-17, -4.408728476930471e-17, -4.9598195365467806e-17, -5.510910596163089e-17, -6.062001655779397e-17, -6.613092715395706e-17, -7.164183775012016e-17, -7.715274834628324e-17, -8.266365894244633e-17, -8.817456953860942e-17, -9.368548013477251e-17, -9.919639073093561e-17, -1.0470730132709869e-16, -1.1021821192326178e-16, -1.1572912251942488e-16, -1.2124003311558795e-16, -1.2675094371175104e-16, -1.3226185430791412e-16, -1.3777276490407724e-16, -1.4328367550024033e-16, -1.487945860964034e-16, -1.5430549669256648e-16, -1.5981640728872957e-16, -1.6532731788489266e-16, -1.7083822848105575e-16, -1.7634913907721884e-16, -1.8186004967338193e-16, -1.8737096026954502e-16, -1.9288187086570813e-16, -1.9839278146187122e-16, -2.0390369205803426e-16, -2.0941460265419738e-16, -2.1492551325036047e-16, -2.2043642384652356e-16, -2.2594733444268667e-16, -2.3145824503884976e-16, -2.3696915563501285e-16, -2.424800662311759e-16, -2.47990976827339e-16, -2.5350188742350207e-16, -2.5901279801966516e-16, -2.6452370861582825e-16, -2.7003461921199134e-16, -2.755455298081545e-16, -2.8105644040431757e-16, -2.8656735100048066e-16, -2.920782615966437e-16, -2.975891721928068e-16, -3.031000827889699e-16, -3.0861099338513297e-16, -3.1412190398129606e-16, -3.1963281457745914e-16, -3.2514372517362223e-16, -3.306546357697853e-16, -3.361655463659484e-16, -3.416764569621115e-16, -3.471873675582746e-16, -3.526982781544377e-16, -3.5820918875060077e-16, -3.6372009934676386e-16, -3.6923100994292695e-16, -3.7474192053909004e-16, -3.8025283113525313e-16, -3.8576374173141627e-16, -3.912746523275793e-16, -3.9678556292374245e-16, -4.022964735199055e-16, -4.0780738411606853e-16, -4.1331829471223167e-16, -4.1882920530839476e-16, -4.2434011590455785e-16, -4.2985102650072094e-16, -4.3536193709688407e-16, -4.408728476930471e-16, -4.463837582892102e-16, -4.518946688853733e-16, -4.574055794815363e-16, -4.629164900776995e-16, -4.684274006738625e-16, -4.739383112700257e-16, -4.794492218661887e-16, -4.849601324623518e-16, -4.904710430585149e-16, -4.95981953654678e-16, -5.0149286425084115e-16, -5.070037748470041e-16, -5.125146854431673e-16, -5.180255960393303e-16, -5.235365066354935e-16, -5.290474172316565e-16, -5.345583278278196e-16, -5.400692384239827e-16, -5.455801490201458e-16, -5.51091059616309e-16 ], [ 0, 0.0018837155858794116, 0.003767431171758823, 0.005651146757638234, 0.007534862343517646, 0.009418577929397058, 0.011302293515276469, 0.01318600910115588, 0.015069724687035293, 0.016953440272914705, 0.018837155858794117, 0.020720871444673526, 0.022604587030552938, 0.024488302616432353, 0.02637201820231176, 0.02825573378819117, 0.030139449374070586, 0.03202316495995, 0.03390688054582941, 0.03579059613170882, 0.037674311717588234, 0.03955802730346764, 0.04144174288934705, 0.04332545847522647, 0.045209174061105875, 0.04709288964698529, 0.048976605232864706, 0.05086032081874411, 0.05274403640462352, 0.05462775199050294, 0.05651146757638234, 0.058395183162261756, 0.06027889874814117, 0.06216261433402058, 0.0640463299199, 0.06593004550577941, 0.06781376109165882, 0.06969747667753823, 0.07158119226341764, 0.07346490784929705, 0.07534862343517647, 0.07723233902105588, 0.07911605460693528, 0.08099977019281471, 0.0828834857786941, 0.08476720136457351, 0.08665091695045293, 0.08853463253633234, 0.09041834812221175, 0.09230206370809117, 0.09418577929397058, 0.09606949487984999, 0.09795321046572941, 0.09983692605160881, 0.10172064163748822, 0.10360435722336764, 0.10548807280924705, 0.10737178839512646, 0.10925550398100588, 0.11113921956688529, 0.11302293515276468, 0.1149066507386441, 0.11679036632452351, 0.11867408191040292, 0.12055779749628234, 0.12244151308216175, 0.12432522866804116, 0.12620894425392057, 0.1280926598398, 0.12997637542567939, 0.13186009101155882, 0.13374380659743823, 0.13562752218331764, 0.13751123776919705, 0.13939495335507646, 0.14127866894095586, 0.14316238452683527, 0.1450461001127147, 0.1469298156985941, 0.14881353128447353, 0.15069724687035294, 0.15258096245623232, 0.15446467804211175, 0.15634839362799116, 0.15823210921387057, 0.16011582479974998, 0.16199954038562941, 0.1638832559715088, 0.1657669715573882, 0.16765068714326764, 0.16953440272914702, 0.17141811831502646, 0.17330183390090587, 0.17518554948678527, 0.17706926507266468, 0.17895298065854412, 0.1808366962444235, 0.1827204118303029, 0.18460412741618235, 0.18648784300206173, 0.18837155858794116 ], [ 0, 0.0037599970069291267, 0.007519994013858253, 0.01127999102078738, 0.015039988027716507, 0.018799985034645635, 0.02255998204157476, 0.02631997904850389, 0.030079976055433014, 0.033839973062362146, 0.03759997006929127, 0.04135996707622039, 0.04511996408314952, 0.04887996109007865, 0.05263995809700778, 0.056399955103936895, 0.06015995211086603, 0.06391994911779515, 0.06767994612472429, 0.0714399431316534, 0.07519994013858254, 0.07895993714551167, 0.08271993415244078, 0.08647993115936992, 0.09023992816629904, 0.09399992517322817, 0.0977599221801573, 0.10151991918708642, 0.10527991619401555, 0.10903991320094468, 0.11279991020787379, 0.11655990721480293, 0.12031990422173205, 0.1240799012286612, 0.1278398982355903, 0.13159989524251944, 0.13535989224944858, 0.1391198892563777, 0.1428798862633068, 0.14663988327023594, 0.15039988027716508, 0.1541598772840942, 0.15791987429102333, 0.16167987129795247, 0.16543986830488155, 0.1691998653118107, 0.17295986231873983, 0.17671985932566894, 0.18047985633259808, 0.18423985333952722, 0.18799985034645633, 0.19175984735338547, 0.1955198443603146, 0.19927984136724372, 0.20303983837417283, 0.20679983538110197, 0.2105598323880311, 0.21431982939496022, 0.21807982640188936, 0.2218398234088185, 0.22559982041574758, 0.22935981742267672, 0.23311981442960586, 0.23687981143653497, 0.2406398084434641, 0.24439980545039325, 0.2481598024573224, 0.25191979946425147, 0.2556797964711806, 0.25943979347810975, 0.2631997904850389, 0.26695978749196797, 0.27071978449889716, 0.27447978150582625, 0.2782397785127554, 0.2819997755196845, 0.2857597725266136, 0.2895197695335428, 0.2932797665404719, 0.297039763547401, 0.30079976055433016, 0.30455975756125925, 0.3083197545681884, 0.3120797515751175, 0.31583974858204666, 0.31959974558897575, 0.32335974259590494, 0.327119739602834, 0.3308797366097631, 0.3346397336166923, 0.3383997306236214, 0.3421597276305505, 0.34591972463747966, 0.3496797216444088, 0.3534397186513379, 0.3571997156582671, 0.36095971266519616, 0.36471970967212525, 0.36847970667905444, 0.3722397036859835, 0.37599970069291266 ], [ 0, 0.005621439437571754, 0.011242878875143508, 0.01686431831271526, 0.022485757750287015, 0.028107197187858767, 0.03372863662543052, 0.039350076063002275, 0.04497151550057403, 0.050592954938145786, 0.056214394375717534, 0.06183583381328928, 0.06745727325086104, 0.0730787126884328, 0.07870015212600455, 0.0843215915635763, 0.08994303100114806, 0.09556447043871982, 0.10118590987629157, 0.10680734931386332, 0.11242878875143507, 0.11805022818900683, 0.12367166762657857, 0.12929310706415034, 0.13491454650172208, 0.14053598593929384, 0.1461574253768656, 0.15177886481443734, 0.1574003042520091, 0.16302174368958086, 0.1686431831271526, 0.17426462256472436, 0.17988606200229612, 0.18550750143986788, 0.19112894087743965, 0.19675038031501138, 0.20237181975258314, 0.20799325919015488, 0.21361469862772664, 0.21923613806529837, 0.22485757750287014, 0.2304790169404419, 0.23610045637801366, 0.24172189581558542, 0.24734333525315713, 0.2529647746907289, 0.2585862141283007, 0.2642076535658724, 0.26982909300344415, 0.2754505324410159, 0.2810719718785877, 0.28669341131615944, 0.2923148507537312, 0.2979362901913029, 0.3035577296288747, 0.30917916906644644, 0.3148006085040182, 0.32042204794158996, 0.3260434873791617, 0.3316649268167335, 0.3372863662543052, 0.34290780569187695, 0.3485292451294487, 0.3541506845670205, 0.35977212400459224, 0.365393563442164, 0.37101500287973577, 0.3766364423173075, 0.3822578817548793, 0.387879321192451, 0.39350076063002276, 0.3991222000675945, 0.4047436395051663, 0.41036507894273805, 0.41598651838030976, 0.4216079578178815, 0.4272293972554533, 0.43285083669302504, 0.43847227613059675, 0.44409371556816857, 0.4497151550057403, 0.455336594443312, 0.4609580338808838, 0.4665794733184555, 0.4722009127560273, 0.47782235219359903, 0.48344379163117085, 0.48906523106874256, 0.49468667050631426, 0.5003081099438861, 0.5059295493814578, 0.5115509888190296, 0.5171724282566014, 0.5227938676941731, 0.5284153071317448, 0.5340367465693167, 0.5396581860068883, 0.5452796254444601, 0.5509010648820318, 0.5565225043196036, 0.5621439437571754 ], [ 0, 0.007460696614945648, 0.014921393229891297, 0.022382089844836944, 0.029842786459782594, 0.03730348307472824, 0.04476417968967389, 0.05222487630461954, 0.05968557291956519, 0.06714626953451085, 0.07460696614945649, 0.08206766276440212, 0.08952835937934778, 0.09698905599429344, 0.10444975260923908, 0.11191044922418472, 0.11937114583913037, 0.12683184245407603, 0.1342925390690217, 0.1417532356839673, 0.14921393229891297, 0.15667462891385864, 0.16413532552880425, 0.17159602214374992, 0.17905671875869555, 0.18651741537364122, 0.1939781119885869, 0.2014388086035325, 0.20889950521847817, 0.2163602018334238, 0.22382089844836944, 0.2312815950633151, 0.23874229167826075, 0.24620298829320642, 0.25366368490815205, 0.2611243815230977, 0.2685850781380434, 0.27604577475298897, 0.2835064713679346, 0.2909671679828803, 0.29842786459782594, 0.3058885612127716, 0.3133492578277173, 0.3208099544426629, 0.3282706510576085, 0.3357313476725542, 0.34319204428749983, 0.35065274090244547, 0.3581134375173911, 0.3655741341323368, 0.37303483074728244, 0.3804955273622281, 0.3879562239771738, 0.39541692059211936, 0.402877617207065, 0.4103383138220107, 0.41779901043695633, 0.42525970705190197, 0.4327204036668476, 0.4401811002817933, 0.4476417968967389, 0.4551024935116845, 0.4625631901266302, 0.47002388674157586, 0.4774845833565215, 0.4849452799714672, 0.49240597658641283, 0.4998666732013584, 0.5073273698163041, 0.5147880664312497, 0.5222487630461954, 0.529709459661141, 0.5371701562760868, 0.5446308528910323, 0.5520915495059779, 0.5595522461209237, 0.5670129427358692, 0.574473639350815, 0.5819343359657606, 0.5893950325807062, 0.5968557291956519, 0.6043164258105975, 0.6117771224255432, 0.6192378190404888, 0.6266985156554346, 0.6341592122703801, 0.6416199088853258, 0.6490806055002715, 0.656541302115217, 0.6640019987301627, 0.6714626953451084, 0.678923391960054, 0.6863840885749997, 0.6938447851899453, 0.7013054818048909, 0.7087661784198367, 0.7162268750347822, 0.7236875716497279, 0.7311482682646736, 0.7386089648796191, 0.7460696614945649 ], [ 0, 0.009270509831248417, 0.018541019662496834, 0.02781152949374525, 0.03708203932499367, 0.04635254915624208, 0.0556230589874905, 0.06489356881873892, 0.07416407864998734, 0.08343458848123576, 0.09270509831248416, 0.10197560814373258, 0.111246117974981, 0.12051662780622942, 0.12978713763747785, 0.13905764746872623, 0.14832815729997467, 0.15759866713122309, 0.16686917696247153, 0.1761396867937199, 0.18541019662496833, 0.19468070645621677, 0.20395121628746515, 0.21322172611871357, 0.222492235949962, 0.23176274578121042, 0.24103325561245884, 0.25030376544370725, 0.2595742752749557, 0.2688447851062041, 0.27811529493745246, 0.2873858047687009, 0.29665631459994934, 0.30592682443119773, 0.31519733426244617, 0.3244678440936946, 0.33373835392494305, 0.3430088637561914, 0.3522793735874398, 0.3615498834186882, 0.37082039324993665, 0.3800909030811851, 0.38936141291243354, 0.3986319227436819, 0.4079024325749303, 0.4171729424061787, 0.42644345223742713, 0.4357139620686756, 0.444984471899924, 0.4542549817311724, 0.46352549156242084, 0.4727960013936693, 0.48206651122491767, 0.49133702105616606, 0.5006075308874145, 0.5098780407186629, 0.5191485505499114, 0.5284190603811597, 0.5376895702124082, 0.5469600800436566, 0.5562305898749049, 0.5655010997061534, 0.5747716095374018, 0.5840421193686502, 0.5933126291998987, 0.6025831390311471, 0.6118536488623955, 0.6211241586936439, 0.6303946685248923, 0.6396651783561407, 0.6489356881873892, 0.6582061980186376, 0.6674767078498861, 0.6767472176811344, 0.6860177275123828, 0.6952882373436313, 0.7045587471748797, 0.7138292570061281, 0.7230997668373764, 0.732370276668625, 0.7416407864998733, 0.7509112963311216, 0.7601818061623702, 0.7694523159936185, 0.7787228258248671, 0.7879933356561154, 0.7972638454873638, 0.8065343553186123, 0.8158048651498606, 0.825075374981109, 0.8343458848123574, 0.8436163946436059, 0.8528869044748543, 0.8621574143061028, 0.8714279241373512, 0.8806984339685996, 0.889968943799848, 0.8992394536310964, 0.9085099634623448, 0.9177804732935932, 0.9270509831248417 ], [ 0, 0.011043736580540349, 0.022087473161080697, 0.033131209741621044, 0.044174946322161394, 0.055218682902701745, 0.06626241948324209, 0.07730615606378244, 0.08834989264432279, 0.09939362922486315, 0.11043736580540349, 0.12148110238594383, 0.13252483896648418, 0.14356857554702454, 0.15461231212756488, 0.1656560487081052, 0.17669978528864558, 0.18774352186918594, 0.1987872584497263, 0.20983099503026661, 0.22087473161080698, 0.23191846819134734, 0.24296220477188765, 0.254005941352428, 0.26504967793296835, 0.2760934145135087, 0.2871371510940491, 0.2981808876745894, 0.30922462425512975, 0.3202683608356701, 0.3313120974162104, 0.3423558339967508, 0.35339957057729116, 0.3644433071578315, 0.3754870437383719, 0.38653078031891225, 0.3975745168994526, 0.40861825347999287, 0.41966199006053323, 0.4307057266410736, 0.44174946322161396, 0.4527931998021543, 0.4638369363826947, 0.47488067296323505, 0.4859244095437753, 0.49696814612431567, 0.508011882704856, 0.5190556192853963, 0.5300993558659367, 0.5411430924464771, 0.5521868290270174, 0.5632305656075578, 0.5742743021880982, 0.5853180387686384, 0.5963617753491788, 0.6074055119297191, 0.6184492485102595, 0.6294929850907999, 0.6405367216713402, 0.6515804582518806, 0.6626241948324209, 0.6736679314129612, 0.6847116679935016, 0.695755404574042, 0.7067991411545823, 0.7178428777351227, 0.728886614315663, 0.7399303508962033, 0.7509740874767438, 0.762017824057284, 0.7730615606378245, 0.7841052972183647, 0.7951490337989052, 0.8061927703794455, 0.8172365069599857, 0.8282802435405262, 0.8393239801210665, 0.8503677167016069, 0.8614114532821472, 0.8724551898626877, 0.8834989264432279, 0.8945426630237682, 0.9055863996043086, 0.9166301361848489, 0.9276738727653894, 0.9387176093459296, 0.9497613459264701, 0.9608050825070104, 0.9718488190875506, 0.9828925556680911, 0.9939362922486313, 1.0049800288291717, 1.016023765409712, 1.0270675019902524, 1.0381112385707927, 1.0491549751513332, 1.0601987117318734, 1.0712424483124137, 1.0822861848929541, 1.0933299214734944, 1.1043736580540349 ], [ 0, 0.012773378746952179, 0.025546757493904358, 0.03832013624085654, 0.051093514987808716, 0.0638668937347609, 0.07664027248171308, 0.08941365122866525, 0.10218702997561743, 0.11496040872256963, 0.1277337874695218, 0.14050716621647397, 0.15328054496342616, 0.16605392371037833, 0.1788273024573305, 0.19160068120428267, 0.20437405995123487, 0.21714743869818706, 0.22992081744513926, 0.2426941961920914, 0.2554675749390436, 0.26824095368599576, 0.28101433243294793, 0.2937877111799001, 0.3065610899268523, 0.3193344686738045, 0.33210784742075666, 0.34488122616770883, 0.357654604914661, 0.3704279836616132, 0.38320136240856534, 0.39597474115551756, 0.40874811990246973, 0.42152149864942196, 0.4342948773963741, 0.4470682561433263, 0.4598416348902785, 0.4726150136372306, 0.4853883923841828, 0.49816177113113497, 0.5109351498780872, 0.5237085286250394, 0.5364819073719915, 0.5492552861189437, 0.5620286648658959, 0.574802043612848, 0.5875754223598002, 0.6003488011067524, 0.6131221798537047, 0.6258955586006568, 0.638668937347609, 0.6514423160945612, 0.6642156948415133, 0.6769890735884655, 0.6897624523354177, 0.7025358310823698, 0.715309209829322, 0.7280825885762743, 0.7408559673232264, 0.7536293460701786, 0.7664027248171307, 0.779176103564083, 0.7919494823110351, 0.8047228610579873, 0.8174962398049395, 0.8302696185518916, 0.8430429972988439, 0.855816376045796, 0.8685897547927482, 0.8813631335397003, 0.8941365122866526, 0.9069098910336048, 0.919683269780557, 0.9324566485275091, 0.9452300272744611, 0.9580034060214135, 0.9707767847683656, 0.9835501635153179, 0.9963235422622699, 1.0090969210092222, 1.0218702997561744, 1.0346436785031266, 1.0474170572500787, 1.0601904359970309, 1.072963814743983, 1.0857371934909352, 1.0985105722378874, 1.1112839509848396, 1.1240573297317917, 1.136830708478744, 1.149604087225696, 1.1623774659726485, 1.1751508447196004, 1.1879242234665528, 1.2006976022135047, 1.2134709809604571, 1.2262443597074093, 1.2390177384543613, 1.2517911172013136, 1.2645644959482656, 1.277337874695218 ], [ 0, 0.01445261022305147, 0.02890522044610294, 0.04335783066915441, 0.05781044089220588, 0.07226305111525735, 0.08671566133830882, 0.1011682715613603, 0.11562088178441177, 0.13007349200746324, 0.1445261022305147, 0.15897871245356618, 0.17343132267661765, 0.18788393289966912, 0.2023365431227206, 0.21678915334577206, 0.23124176356882353, 0.245694373791875, 0.2601469840149265, 0.2745995942379779, 0.2890522044610294, 0.3035048146840809, 0.31795742490713236, 0.3324100351301838, 0.3468626453532353, 0.3613152555762868, 0.37576786579933824, 0.3902204760223897, 0.4046730862454412, 0.4191256964684927, 0.4335783066915441, 0.44803091691459557, 0.46248352713764707, 0.47693613736069856, 0.49138874758375, 0.5058413578068015, 0.520293968029853, 0.5347465782529044, 0.5491991884759558, 0.5636517986990074, 0.5781044089220588, 0.5925570191451103, 0.6070096293681618, 0.6214622395912133, 0.6359148498142647, 0.6503674600373162, 0.6648200702603676, 0.6792726804834192, 0.6937252907064706, 0.708177900929522, 0.7226305111525736, 0.737083121375625, 0.7515357315986765, 0.7659883418217279, 0.7804409520447794, 0.7948935622678309, 0.8093461724908824, 0.8237987827139338, 0.8382513929369854, 0.8527040031600368, 0.8671566133830882, 0.8816092236061397, 0.8960618338291911, 0.9105144440522427, 0.9249670542752941, 0.9394196644983456, 0.9538722747213971, 0.9683248849444485, 0.9827774951675, 0.9972301053905515, 1.011682715613603, 1.0261353258366543, 1.040587936059706, 1.0550405462827575, 1.0694931565058088, 1.0839457667288603, 1.0983983769519117, 1.1128509871749632, 1.1273035973980148, 1.1417562076210663, 1.1562088178441177, 1.170661428067169, 1.1851140382902206, 1.199566648513272, 1.2140192587363237, 1.228471868959375, 1.2429244791824265, 1.2573770894054779, 1.2718296996285294, 1.286282309851581, 1.3007349200746323, 1.3151875302976839, 1.3296401405207352, 1.3440927507437868, 1.3585453609668383, 1.3729979711898899, 1.3874505814129412, 1.4019031916359925, 1.416355801859044, 1.4308084120820956, 1.4452610223051472 ], [ 0, 0.0160748038493699, 0.0321496076987398, 0.0482244115481097, 0.0642992153974796, 0.08037401924684949, 0.0964488230962194, 0.11252362694558929, 0.1285984307949592, 0.1446732346443291, 0.16074803849369898, 0.1768228423430689, 0.1928976461924388, 0.2089724500418087, 0.22504725389117858, 0.24112205774054848, 0.2571968615899184, 0.2732716654392883, 0.2893464692886582, 0.3054212731380281, 0.32149607698739796, 0.3375708808367679, 0.3536456846861378, 0.36972048853550765, 0.3857952923848776, 0.4018700962342475, 0.4179449000836174, 0.4340197039329873, 0.45009450778235716, 0.4661693116317271, 0.48224411548109697, 0.49831891933046685, 0.5143937231798368, 0.5304685270292067, 0.5465433308785766, 0.5626181347279465, 0.5786929385773164, 0.5947677424266862, 0.6108425462760562, 0.626917350125426, 0.6429921539747959, 0.6590669578241659, 0.6751417616735358, 0.6912165655229057, 0.7072913693722755, 0.7233661732216454, 0.7394409770710153, 0.7555157809203853, 0.7715905847697552, 0.787665388619125, 0.803740192468495, 0.8198149963178649, 0.8358898001672348, 0.8519646040166046, 0.8680394078659746, 0.8841142117153444, 0.9001890155647143, 0.9162638194140843, 0.9323386232634542, 0.9484134271128241, 0.9644882309621939, 0.9805630348115638, 0.9966378386609337, 1.0127126425103037, 1.0287874463596736, 1.0448622502090434, 1.0609370540584133, 1.0770118579077832, 1.0930866617571533, 1.109161465606523, 1.125236269455893, 1.1413110733052627, 1.1573858771546328, 1.1734606810040027, 1.1895354848533723, 1.2056102887027424, 1.2216850925521123, 1.2377598964014822, 1.253834700250852, 1.2699095041002222, 1.2859843079495918, 1.3020591117989617, 1.3181339156483318, 1.3342087194977015, 1.3502835233470716, 1.3663583271964415, 1.3824331310458113, 1.3985079348951812, 1.414582738744551, 1.430657542593921, 1.4467323464432909, 1.462807150292661, 1.4788819541420306, 1.4949567579914007, 1.5110315618407706, 1.5271063656901405, 1.5431811695395103, 1.55925597338888, 1.57533077723825, 1.59140558108762, 1.60748038493699 ], [ 0, 0.01763355756877421, 0.03526711513754842, 0.05290067270632263, 0.07053423027509684, 0.08816778784387105, 0.10580134541264526, 0.12343490298141947, 0.14106846055019368, 0.15870201811896792, 0.1763355756877421, 0.1939691332565163, 0.2116026908252905, 0.22923624839406476, 0.24686980596283895, 0.26450336353161313, 0.28213692110038735, 0.29977047866916157, 0.31740403623793584, 0.33503759380671, 0.3526711513754842, 0.37030470894425843, 0.3879382665130326, 0.4055718240818068, 0.423205381650581, 0.44083893921935524, 0.4584724967881295, 0.4761060543569037, 0.4937396119256779, 0.5113731694944521, 0.5290067270632263, 0.5466402846320005, 0.5642738422007747, 0.581907399769549, 0.5995409573383231, 0.6171745149070974, 0.6348080724758717, 0.6524416300446457, 0.67007518761342, 0.6877087451821942, 0.7053423027509684, 0.7229758603197426, 0.7406094178885169, 0.7582429754572911, 0.7758765330260652, 0.7935100905948395, 0.8111436481636136, 0.8287772057323879, 0.846410763301162, 0.8640443208699363, 0.8816778784387105, 0.8993114360074848, 0.916944993576259, 0.9345785511450331, 0.9522121087138073, 0.9698456662825815, 0.9874792238513558, 1.00511278142013, 1.0227463389889042, 1.0403798965576785, 1.0580134541264525, 1.0756470116952268, 1.093280569264001, 1.1109141268327751, 1.1285476844015494, 1.1461812419703237, 1.163814799539098, 1.181448357107872, 1.1990819146766463, 1.2167154722454205, 1.2343490298141948, 1.2519825873829689, 1.2696161449517434, 1.2872497025205174, 1.3048832600892915, 1.3225168176580657, 1.34015037522684, 1.3577839327956143, 1.3754174903643883, 1.3930510479331628, 1.4106846055019369, 1.428318163070711, 1.4459517206394852, 1.4635852782082595, 1.4812188357770337, 1.4988523933458078, 1.5164859509145823, 1.5341195084833563, 1.5517530660521304, 1.5693866236209046, 1.587020181189679, 1.6046537387584532, 1.6222872963272272, 1.6399208538960017, 1.6575544114647758, 1.67518796903355, 1.692821526602324, 1.7104550841710984, 1.7280886417398726, 1.7457221993086467, 1.763355756877421 ], [ 0, 0.0191227196924607, 0.0382454393849214, 0.057368159077382096, 0.0764908787698428, 0.0956135984623035, 0.11473631815476419, 0.1338590378472249, 0.1529817575396856, 0.1721044772321463, 0.191227196924607, 0.21034991661706767, 0.22947263630952838, 0.2485953560019891, 0.2677180756944498, 0.28684079538691043, 0.3059635150793712, 0.3250862347718319, 0.3442089544642926, 0.36333167415675327, 0.382454393849214, 0.4015771135416747, 0.42069983323413535, 0.43982255292659606, 0.45894527261905677, 0.4780679923115175, 0.4971907120039782, 0.5163134316964388, 0.5354361513888996, 0.5545588710813603, 0.5736815907738209, 0.5928043104662817, 0.6119270301587424, 0.6310497498512031, 0.6501724695436638, 0.6692951892361245, 0.6884179089285852, 0.7075406286210458, 0.7266633483135065, 0.7457860680059673, 0.764908787698428, 0.7840315073908887, 0.8031542270833494, 0.8222769467758101, 0.8413996664682707, 0.8605223861607314, 0.8796451058531921, 0.8987678255456528, 0.9178905452381135, 0.9370132649305742, 0.956135984623035, 0.9752587043154957, 0.9943814240079564, 1.0135041437004169, 1.0326268633928777, 1.0517495830853383, 1.070872302777799, 1.08999502247026, 1.1091177421627205, 1.1282404618551813, 1.1473631815476417, 1.1664859012401025, 1.1856086209325634, 1.204731340625024, 1.2238540603174848, 1.2429767800099454, 1.2620994997024062, 1.2812222193948668, 1.3003449390873276, 1.3194676587797882, 1.338590378472249, 1.3577130981647096, 1.3768358178571705, 1.395958537549631, 1.4150812572420917, 1.4342039769345525, 1.453326696627013, 1.472449416319474, 1.4915721360119345, 1.5106948557043953, 1.529817575396856, 1.5489402950893165, 1.5680630147817773, 1.587185734474238, 1.6063084541666988, 1.6254311738591594, 1.6445538935516202, 1.6636766132440808, 1.6827993329365414, 1.7019220526290022, 1.7210447723214628, 1.7401674920139236, 1.7592902117063842, 1.778412931398845, 1.7975356510913056, 1.8166583707837665, 1.835781090476227, 1.8549038101686877, 1.8740265298611485, 1.893149249553609, 1.91227196924607 ], [ 0, 0.020536413177860657, 0.04107282635572131, 0.06160923953358197, 0.08214565271144263, 0.10268206588930329, 0.12321847906716395, 0.1437548922450246, 0.16429130542288525, 0.18482771860074593, 0.20536413177860657, 0.22590054495646722, 0.2464369581343279, 0.26697337131218857, 0.2875097844900492, 0.30804619766790986, 0.3285826108457705, 0.3491190240236312, 0.36965543720149185, 0.3901918503793525, 0.41072826355721315, 0.43126467673507385, 0.45180108991293444, 0.4723375030907951, 0.4928739162686558, 0.5134103294465164, 0.5339467426243771, 0.5544831558022377, 0.5750195689800984, 0.5955559821579591, 0.6160923953358197, 0.6366288085136804, 0.657165221691541, 0.6777016348694017, 0.6982380480472624, 0.7187744612251231, 0.7393108744029837, 0.7598472875808443, 0.780383700758705, 0.8009201139365656, 0.8214565271144263, 0.841992940292287, 0.8625293534701477, 0.8830657666480083, 0.9036021798258689, 0.9241385930037296, 0.9446750061815902, 0.9652114193594509, 0.9857478325373116, 1.0062842457151722, 1.0268206588930329, 1.0473570720708936, 1.0678934852487543, 1.0884298984266148, 1.1089663116044755, 1.1295027247823362, 1.1500391379601969, 1.1705755511380576, 1.1911119643159183, 1.211648377493779, 1.2321847906716394, 1.2527212038495001, 1.2732576170273608, 1.2937940302052213, 1.314330443383082, 1.3348668565609427, 1.3554032697388034, 1.375939682916664, 1.3964760960945248, 1.4170125092723853, 1.4375489224502462, 1.4580853356281067, 1.4786217488059674, 1.4991581619838281, 1.5196945751616886, 1.5402309883395493, 1.56076740151741, 1.5813038146952707, 1.6018402278731312, 1.622376641050992, 1.6429130542288526, 1.663449467406713, 1.683985880584574, 1.7045222937624345, 1.7250587069402954, 1.7455951201181559, 1.7661315332960166, 1.7866679464738773, 1.8072043596517378, 1.8277407728295985, 1.8482771860074592, 1.8688135991853199, 1.8893500123631803, 1.9098864255410413, 1.9304228387189017, 1.9509592518967627, 1.9714956650746231, 1.9920320782524836, 2.0125684914303443, 2.033104904608205, 2.0536413177860657 ], [ 0, 0.021869058822642357, 0.043738117645284715, 0.06560717646792706, 0.08747623529056943, 0.10934529411321178, 0.13121435293585412, 0.1530834117584965, 0.17495247058113886, 0.19682152940378123, 0.21869058822642357, 0.2405596470490659, 0.26242870587170825, 0.28429776469435064, 0.306166823516993, 0.3280358823396353, 0.3499049411622777, 0.37177399998492006, 0.39364305880756245, 0.41551211763020474, 0.43738117645284713, 0.45925023527548947, 0.4811192940981318, 0.5029883529207742, 0.5248574117434165, 0.5467264705660589, 0.5685955293887013, 0.5904645882113436, 0.612333647033986, 0.6342027058566283, 0.6560717646792706, 0.677940823501913, 0.6998098823245554, 0.7216789411471978, 0.7435479999698401, 0.7654170587924826, 0.7872861176151249, 0.8091551764377671, 0.8310242352604095, 0.8528932940830518, 0.8747623529056943, 0.8966314117283366, 0.9185004705509789, 0.9403695293736214, 0.9622385881962636, 0.984107647018906, 1.0059767058415483, 1.0278457646641908, 1.049714823486833, 1.0715838823094754, 1.0934529411321179, 1.1153219999547601, 1.1371910587774026, 1.1590601176000448, 1.1809291764226872, 1.2027982352453295, 1.224667294067972, 1.2465363528906144, 1.2684054117132566, 1.290274470535899, 1.3121435293585413, 1.3340125881811837, 1.355881647003826, 1.3777507058264684, 1.3996197646491109, 1.421488823471753, 1.4433578822943955, 1.4652269411170378, 1.4870959999396802, 1.5089650587623225, 1.5308341175849651, 1.5527031764076071, 1.5745722352302498, 1.596441294052892, 1.6183103528755343, 1.6401794116981767, 1.662048470520819, 1.6839175293434614, 1.7057865881661036, 1.7276556469887463, 1.7495247058113885, 1.7713937646340308, 1.7932628234566732, 1.8151318822793154, 1.8370009411019579, 1.8588699999246001, 1.8807390587472428, 1.902608117569885, 1.9244771763925272, 1.9463462352151697, 1.968215294037812, 1.9900843528604544, 2.0119534116830966, 2.033822470505739, 2.0556915293283815, 2.077560588151024, 2.099429646973666, 2.1212987057963084, 2.143167764618951, 2.1650368234415933, 2.1869058822642358 ], [ 0, 0.02311539728327368, 0.04623079456654736, 0.06934619184982103, 0.09246158913309473, 0.1155769864163684, 0.13869238369964207, 0.16180778098291576, 0.18492317826618945, 0.20803857554946315, 0.2311539728327368, 0.2542693701160105, 0.27738476739928414, 0.30050016468255786, 0.3236155619658315, 0.3467309592491052, 0.3698463565323789, 0.39296175381565257, 0.4160771510989263, 0.4391925483821999, 0.4623079456654736, 0.4854233429487473, 0.508538740232021, 0.5316541375152947, 0.5547695347985683, 0.577884932081842, 0.6010003293651157, 0.6241157266483893, 0.647231123931663, 0.6703465212149368, 0.6934619184982104, 0.7165773157814841, 0.7396927130647578, 0.7628081103480314, 0.7859235076313051, 0.8090389049145789, 0.8321543021978526, 0.8552696994811261, 0.8783850967643998, 0.9015004940476735, 0.9246158913309472, 0.947731288614221, 0.9708466858974946, 0.9939620831807683, 1.017077480464042, 1.0401928777473155, 1.0633082750305893, 1.086423672313863, 1.1095390695971366, 1.1326544668804104, 1.155769864163684, 1.1788852614469578, 1.2020006587302314, 1.225116056013505, 1.2482314532967786, 1.2713468505800525, 1.294462247863326, 1.3175776451465997, 1.3406930424298735, 1.3638084397131471, 1.3869238369964207, 1.4100392342796944, 1.4331546315629682, 1.4562700288462418, 1.4793854261295156, 1.5025008234127892, 1.5256162206960628, 1.5487316179793365, 1.5718470152626103, 1.594962412545884, 1.6180778098291577, 1.6411932071124313, 1.6643086043957052, 1.6874240016789788, 1.7105393989622522, 1.733654796245526, 1.7567701935287996, 1.7798855908120734, 1.803000988095347, 1.8261163853786209, 1.8492317826618945, 1.8723471799451679, 1.895462577228442, 1.9185779745117153, 1.9416933717949891, 1.9648087690782627, 1.9879241663615366, 2.01103956364481, 2.034154960928084, 2.0572703582113574, 2.080385755494631, 2.103501152777905, 2.1266165500611787, 2.1497319473444523, 2.172847344627726, 2.195962741911, 2.219078139194273, 2.2421935364775467, 2.2653089337608208, 2.2884243310440944, 2.311539728327368 ], [ 0, 0.024270509831248437, 0.048541019662496875, 0.07281152949374531, 0.09708203932499375, 0.12135254915624218, 0.14562305898749062, 0.16989356881873904, 0.1941640786499875, 0.21843458848123595, 0.24270509831248435, 0.2669756081437328, 0.29124611797498123, 0.3155166278062297, 0.3397871376374781, 0.36405764746872654, 0.388328157299975, 0.41259866713122345, 0.4368691769624719, 0.46113968679372025, 0.4854101966249687, 0.5096807064562172, 0.5339512162874656, 0.558221726118714, 0.5824922359499625, 0.6067627457812109, 0.6310332556124594, 0.6553037654437077, 0.6795742752749562, 0.7038447851062046, 0.7281152949374531, 0.7523858047687015, 0.77665631459995, 0.8009268244311984, 0.8251973342624469, 0.8494678440936954, 0.8737383539249438, 0.898008863756192, 0.9222793735874405, 0.946549883418689, 0.9708203932499374, 0.9950909030811859, 1.0193614129124344, 1.0436319227436828, 1.0679024325749311, 1.0921729424061795, 1.116443452237428, 1.1407139620686764, 1.164984471899925, 1.1892549817311733, 1.2135254915624218, 1.2377960013936704, 1.2620665112249188, 1.286337021056167, 1.3106075308874154, 1.334878040718664, 1.3591485505499123, 1.383419060381161, 1.4076895702124093, 1.4319600800436578, 1.4562305898749062, 1.4805010997061545, 1.504771609537403, 1.5290421193686514, 1.5533126291999, 1.5775831390311483, 1.601853648862397, 1.626124158693645, 1.6503946685248938, 1.674665178356142, 1.6989356881873907, 1.7232061980186388, 1.7474767078498876, 1.7717472176811357, 1.796017727512384, 1.8202882373436329, 1.844558747174881, 1.8688292570061298, 1.893099766837378, 1.9173702766686267, 1.9416407864998748, 1.9659112963311232, 1.9901818061623717, 2.01445231599362, 2.038722825824869, 2.062993335656117, 2.0872638454873655, 2.111534355318614, 2.1358048651498622, 2.160075374981111, 2.184345884812359, 2.2086163946436077, 2.232886904474856, 2.257157414306105, 2.2814279241373527, 2.3056984339686015, 2.32996894379985, 2.354239453631098, 2.3785099634623466, 2.402780473293595, 2.4270509831248437 ], [ 0, 0.025329837765060457, 0.050659675530120914, 0.07598951329518137, 0.10131935106024183, 0.1266491888253023, 0.15197902659036275, 0.1773088643554232, 0.20263870212048365, 0.22796853988554414, 0.2532983776506046, 0.278628215415665, 0.3039580531807255, 0.329287890945786, 0.3546177287108464, 0.37994756647590683, 0.4052774042409673, 0.4306072420060278, 0.45593707977108827, 0.4812669175361487, 0.5065967553012092, 0.5319265930662697, 0.55725643083133, 0.5825862685963905, 0.607916106361451, 0.6332459441265115, 0.658575781891572, 0.6839056196566323, 0.7092354574216928, 0.7345652951867533, 0.7598951329518137, 0.7852249707168741, 0.8105548084819346, 0.8358846462469951, 0.8612144840120556, 0.8865443217771161, 0.9118741595421765, 0.9372039973072369, 0.9625338350722974, 0.9878636728373579, 1.0131935106024184, 1.0385233483674787, 1.0638531861325393, 1.0891830238975997, 1.11451286166266, 1.1398426994277204, 1.165172537192781, 1.1905023749578414, 1.215832212722902, 1.2411620504879624, 1.266491888253023, 1.2918217260180835, 1.317151563783144, 1.3424814015482043, 1.3678112393132646, 1.3931410770783252, 1.4184709148433856, 1.4438007526084462, 1.4691305903735066, 1.4944604281385672, 1.5197902659036273, 1.545120103668688, 1.5704499414337483, 1.5957797791988089, 1.6211096169638692, 1.6464394547289298, 1.6717692924939902, 1.6970991302590506, 1.7224289680241112, 1.7477588057891715, 1.7730886435542321, 1.7984184813192925, 1.823748319084353, 1.8490781568494135, 1.8744079946144738, 1.8997378323795344, 1.9250676701445948, 1.9503975079096554, 1.9757273456747158, 2.0010571834397766, 2.0263870212048367, 2.051716858969897, 2.0770466967349575, 2.102376534500018, 2.1277063722650786, 2.153036210030139, 2.1783660477951994, 2.20369588556026, 2.22902572332532, 2.2543555610903807, 2.279685398855441, 2.305015236620502, 2.330345074385562, 2.3556749121506226, 2.381004749915683, 2.406334587680744, 2.431664425445804, 2.456994263210864, 2.4823241009759247, 2.5076539387409853, 2.532983776506046 ], [ 0, 0.026289200401315906, 0.05257840080263181, 0.07886760120394772, 0.10515680160526363, 0.13144600200657952, 0.15773520240789543, 0.18402440280921134, 0.21031360321052725, 0.2366028036118432, 0.26289200401315904, 0.2891812044144749, 0.31547040481579086, 0.3417596052171068, 0.3680488056184227, 0.39433800601973856, 0.4206272064210545, 0.44691640682237044, 0.4732056072236864, 0.4994948076250022, 0.5257840080263181, 0.5520732084276341, 0.5783624088289498, 0.6046516092302658, 0.6309408096315817, 0.6572300100328977, 0.6835192104342136, 0.7098084108355295, 0.7360976112368454, 0.7623868116381614, 0.7886760120394771, 0.8149652124407931, 0.841254412842109, 0.867543613243425, 0.8938328136447409, 0.9201220140460568, 0.9464112144473728, 0.9727004148486885, 0.9989896152500044, 1.0252788156513204, 1.0515680160526362, 1.0778572164539522, 1.1041464168552682, 1.1304356172565841, 1.1567248176578997, 1.1830140180592157, 1.2093032184605317, 1.2355924188618477, 1.2618816192631634, 1.2881708196644794, 1.3144600200657954, 1.3407492204671112, 1.3670384208684272, 1.393327621269743, 1.419616821671059, 1.4459060220723747, 1.4721952224736907, 1.4984844228750067, 1.5247736232763227, 1.5510628236776385, 1.5773520240789543, 1.6036412244802702, 1.6299304248815862, 1.656219625282902, 1.682508825684218, 1.708798026085534, 1.73508722648685, 1.7613764268881655, 1.7876656272894818, 1.8139548276907975, 1.8402440280921135, 1.8665332284934293, 1.8928224288947455, 1.9191116292960613, 1.945400829697377, 1.971690030098693, 1.9979792305000088, 2.024268430901325, 2.050557631302641, 2.076846831703957, 2.1031360321052723, 2.129425232506588, 2.1557144329079043, 2.18200363330922, 2.2082928337105363, 2.234582034111852, 2.2608712345131683, 2.287160434914484, 2.3134496353157994, 2.3397388357171156, 2.3660280361184314, 2.3923172365197476, 2.4186064369210634, 2.4448956373223796, 2.4711848377236953, 2.497474038125011, 2.523763238526327, 2.5500524389276427, 2.576341639328959, 2.6026308397302746, 2.628920040131591 ], [ 0, 0.027144811573980594, 0.05428962314796119, 0.08143443472194178, 0.10857924629592237, 0.13572405786990296, 0.16286886944388357, 0.19001368101786414, 0.21715849259184475, 0.24430330416582535, 0.27144811573980593, 0.2985929273137865, 0.32573773888776714, 0.3528825504617477, 0.3800273620357283, 0.40717217360970887, 0.4343169851836895, 0.46146179675767013, 0.4886066083316507, 0.5157514199056312, 0.5428962314796119, 0.5700410430535925, 0.597185854627573, 0.6243306662015536, 0.6514754777755343, 0.6786202893495148, 0.7057651009234954, 0.732909912497476, 0.7600547240714566, 0.7871995356454372, 0.8143443472194177, 0.8414891587933984, 0.868633970367379, 0.8957787819413596, 0.9229235935153403, 0.9500684050893208, 0.9772132166633014, 1.004358028237282, 1.0315028398112625, 1.0586476513852432, 1.0857924629592237, 1.1129372745332042, 1.140082086107185, 1.1672268976811655, 1.194371709255146, 1.2215165208291265, 1.2486613324031073, 1.2758061439770878, 1.3029509555510685, 1.330095767125049, 1.3572405786990296, 1.3843853902730103, 1.4115302018469909, 1.4386750134209714, 1.465819824994952, 1.4929646365689326, 1.5201094481429132, 1.547254259716894, 1.5743990712908744, 1.6015438828648552, 1.6286886944388355, 1.6558335060128162, 1.6829783175867967, 1.7101231291607772, 1.737267940734758, 1.7644127523087385, 1.7915575638827193, 1.8187023754566995, 1.8458471870306805, 1.8729919986046608, 1.9001368101786416, 1.927281621752622, 1.9544264333266028, 1.9815712449005833, 2.008716056474564, 2.0358608680485446, 2.063005679622525, 2.0901504911965056, 2.1172953027704864, 2.144440114344467, 2.1715849259184474, 2.1987297374924277, 2.2258745490664085, 2.253019360640389, 2.28016417221437, 2.3073089837883503, 2.334453795362331, 2.3615986069363117, 2.388743418510292, 2.415888230084273, 2.443033041658253, 2.470177853232234, 2.4973226648062146, 2.5244674763801953, 2.5516122879541756, 2.5787570995281563, 2.605901911102137, 2.6330467226761174, 2.660191534250098, 2.6873363458240784, 2.714481157398059 ], [ 0, 0.02789329457664754, 0.05578658915329508, 0.08367988372994263, 0.11157317830659016, 0.1394664728832377, 0.16735976745988526, 0.1952530620365328, 0.22314635661318033, 0.2510396511898279, 0.2789329457664754, 0.30682624034312295, 0.3347195349197705, 0.3626128294964181, 0.3905061240730656, 0.41839941864971314, 0.44629271322636066, 0.47418600780300824, 0.5020793023796558, 0.5299725969563033, 0.5578658915329509, 0.5857591861095984, 0.6136524806862459, 0.6415457752628935, 0.669439069839541, 0.6973323644161886, 0.7252256589928362, 0.7531189535694837, 0.7810122481461312, 0.8089055427227788, 0.8367988372994263, 0.8646921318760737, 0.8925854264527213, 0.9204787210293689, 0.9483720156060165, 0.976265310182664, 1.0041586047593116, 1.032051899335959, 1.0599451939126066, 1.0878384884892542, 1.1157317830659017, 1.1436250776425492, 1.1715183722191969, 1.1994116667958443, 1.2273049613724918, 1.2551982559491393, 1.283091550525787, 1.3109848451024344, 1.338878139679082, 1.3667714342557296, 1.3946647288323772, 1.4225580234090247, 1.4504513179856724, 1.4783446125623196, 1.5062379071389673, 1.5341312017156148, 1.5620244962922625, 1.58991779086891, 1.6178110854455576, 1.645704380022205, 1.6735976745988526, 1.7014909691755, 1.7293842637521475, 1.7572775583287952, 1.7851708529054426, 1.8130641474820903, 1.8409574420587378, 1.8688507366353853, 1.896744031212033, 1.9246373257886804, 1.952530620365328, 1.9804239149419756, 2.0083172095186232, 2.0362105040952705, 2.064103798671918, 2.091997093248566, 2.119890387825213, 2.147783682401861, 2.1756769769785085, 2.203570271555156, 2.2314635661318034, 2.2593568607084507, 2.2872501552850983, 2.315143449861746, 2.3430367444383937, 2.370930039015041, 2.3988233335916886, 2.4267166281683363, 2.4546099227449836, 2.4825032173216313, 2.5103965118982785, 2.5382898064749266, 2.566183101051574, 2.5940763956282216, 2.621969690204869, 2.649862984781517, 2.677756279358164, 2.7056495739348114, 2.733542868511459, 2.761436163088107, 2.7893294576647545 ], [ 0, 0.028531695488854612, 0.057063390977709225, 0.08559508646656383, 0.11412678195541845, 0.14265847744427307, 0.17119017293312766, 0.19972186842198228, 0.2282535639108369, 0.25678525939969155, 0.28531695488854614, 0.3138486503774007, 0.3423803458662553, 0.37091204135510997, 0.39944373684396456, 0.42797543233281915, 0.4565071278216738, 0.48503882331052844, 0.5135705187993831, 0.5421022142882376, 0.5706339097770923, 0.5991656052659469, 0.6276973007548013, 0.656228996243656, 0.6847606917325106, 0.7132923872213653, 0.7418240827102199, 0.7703557781990745, 0.7988874736879291, 0.8274191691767837, 0.8559508646656383, 0.8844825601544929, 0.9130142556433476, 0.9415459511322022, 0.9700776466210569, 0.9986093421099115, 1.0271410375987662, 1.0556727330876206, 1.0842044285764751, 1.1127361240653297, 1.1412678195541845, 1.1697995150430391, 1.1983312105318937, 1.2268629060207483, 1.2553946015096027, 1.2839262969984575, 1.312457992487312, 1.3409896879761667, 1.3695213834650213, 1.398053078953876, 1.4265847744427307, 1.4551164699315853, 1.4836481654204399, 1.5121798609092942, 1.540711556398149, 1.5692432518870036, 1.5977749473758582, 1.6263066428647128, 1.6548383383535674, 1.6833700338424222, 1.7119017293312766, 1.7404334248201312, 1.7689651203089858, 1.7974968157978406, 1.8260285112866952, 1.8545602067755498, 1.8830919022644044, 1.9116235977532587, 1.9401552932421138, 1.9686869887309681, 1.997218684219823, 2.0257503797086773, 2.0542820751975324, 2.0828137706863865, 2.111345466175241, 2.139877161664096, 2.1684088571529503, 2.1969405526418053, 2.2254722481306595, 2.2540039436195145, 2.282535639108369, 2.3110673345972232, 2.3395990300860783, 2.3681307255749324, 2.3966624210637875, 2.425194116552642, 2.4537258120414966, 2.4822575075303512, 2.5107892030192054, 2.5393208985080604, 2.567852593996915, 2.5963842894857696, 2.624915984974624, 2.653447680463479, 2.6819793759523334, 2.7105110714411884, 2.7390427669300426, 2.767574462418897, 2.796106157907752, 2.8246378533966063, 2.8531695488854614 ], [ 0, 0.029057494833858933, 0.058114989667717866, 0.0871724845015768, 0.11622997933543573, 0.14528747416929466, 0.1743449690031536, 0.20340246383701255, 0.23245995867087146, 0.26151745350473043, 0.2905749483385893, 0.31963244317244827, 0.3486899380063072, 0.37774743284016615, 0.4068049276740251, 0.435862422507884, 0.4649199173417429, 0.4939774121756019, 0.5230349070094609, 0.5520924018433198, 0.5811498966771786, 0.6102073915110376, 0.6392648863448965, 0.6683223811787554, 0.6973798760126144, 0.7264373708464734, 0.7554948656803323, 0.7845523605141912, 0.8136098553480502, 0.8426673501819091, 0.871724845015768, 0.900782339849627, 0.9298398346834859, 0.9588973295173449, 0.9879548243512039, 1.0170123191850629, 1.0460698140189217, 1.0751273088527804, 1.1041848036866395, 1.1332422985204984, 1.1622997933543573, 1.1913572881882164, 1.2204147830220753, 1.2494722778559342, 1.278529772689793, 1.307587267523652, 1.3366447623575108, 1.36570225719137, 1.3947597520252288, 1.4238172468590877, 1.4528747416929468, 1.4819322365268057, 1.5109897313606646, 1.5400472261945235, 1.5691047210283824, 1.5981622158622413, 1.6272197106961004, 1.6562772055299593, 1.6853347003638182, 1.7143921951976773, 1.743449690031536, 1.7725071848653948, 1.801564679699254, 1.8306221745331128, 1.8596796693669717, 1.8887371642008308, 1.9177946590346897, 1.9468521538685484, 1.9759096487024077, 2.0049671435362666, 2.0340246383701257, 2.0630821332039844, 2.0921396280378435, 2.121197122871702, 2.150254617705561, 2.1793121125394204, 2.208369607373279, 2.237427102207138, 2.266484597040997, 2.295542091874856, 2.3245995867087146, 2.3536570815425737, 2.382714576376433, 2.4117720712102915, 2.4408295660441506, 2.4698870608780092, 2.4989445557118684, 2.5280020505457275, 2.557059545379586, 2.5861170402134452, 2.615174535047304, 2.644232029881163, 2.6732895247150217, 2.7023470195488812, 2.73140451438274, 2.760462009216599, 2.7895195040504577, 2.8185769988843163, 2.8476344937181755, 2.8766919885520346, 2.9057494833858937 ], [ 0, 0.02946861752186066, 0.05893723504372132, 0.08840585256558198, 0.11787447008744265, 0.1473430876093033, 0.17681170513116395, 0.20628032265302462, 0.2357489401748853, 0.265217557696746, 0.2946861752186066, 0.3241547927404672, 0.3536234102623279, 0.3830920277841886, 0.41256064530604925, 0.4420292628279099, 0.4714978803497706, 0.5009664978716313, 0.530435115393492, 0.5599037329153526, 0.5893723504372133, 0.618840967959074, 0.6483095854809344, 0.6777782030027951, 0.7072468205246558, 0.7367154380465165, 0.7661840555683772, 0.7956526730902378, 0.8251212906120985, 0.8545899081339592, 0.8840585256558198, 0.9135271431776805, 0.9429957606995412, 0.9724643782214019, 1.0019329957432626, 1.0314016132651231, 1.060870230786984, 1.0903388483088443, 1.1198074658307051, 1.1492760833525657, 1.1787447008744265, 1.208213318396287, 1.237681935918148, 1.2671505534400085, 1.2966191709618688, 1.3260877884837297, 1.3555564060055902, 1.385025023527451, 1.4144936410493116, 1.4439622585711724, 1.473430876093033, 1.5028994936148938, 1.5323681111367544, 1.561836728658615, 1.5913053461804756, 1.6207739637023364, 1.650242581224197, 1.6797111987460578, 1.7091798162679184, 1.738648433789779, 1.7681170513116395, 1.7975856688335001, 1.827054286355361, 1.8565229038772215, 1.8859915213990823, 1.915460138920943, 1.9449287564428037, 1.974397373964664, 2.003865991486525, 2.0333346090083855, 2.0628032265302463, 2.092271844052107, 2.121740461573968, 2.1512090790958283, 2.1806776966176886, 2.2101463141395494, 2.2396149316614102, 2.269083549183271, 2.2985521667051314, 2.328020784226992, 2.357489401748853, 2.3869580192707134, 2.416426636792574, 2.4458952543144346, 2.475363871836296, 2.504832489358156, 2.534301106880017, 2.5637697244018773, 2.5932383419237377, 2.622706959445599, 2.6521755769674593, 2.68164419448932, 2.7111128120111805, 2.7405814295330417, 2.770050047054902, 2.799518664576763, 2.8289872820986233, 2.858455899620484, 2.887924517142345, 2.9173931346642052, 2.946861752186066 ], [ 0, 0.029763441039434336, 0.05952688207886867, 0.08929032311830301, 0.11905376415773734, 0.14881720519717168, 0.17858064623660602, 0.20834408727604034, 0.2381075283154747, 0.26787096935490906, 0.29763441039434335, 0.32739785143377764, 0.35716129247321204, 0.3869247335126464, 0.4166881745520807, 0.446451615591515, 0.4762150566309494, 0.5059784976703837, 0.5357419387098181, 0.5655053797492523, 0.5952688207886867, 0.6250322618281211, 0.6547957028675553, 0.6845591439069897, 0.7143225849464241, 0.7440860259858584, 0.7738494670252928, 0.8036129080647271, 0.8333763491041614, 0.8631397901435958, 0.89290323118303, 0.9226666722224643, 0.9524301132618987, 0.982193554301333, 1.0119569953407674, 1.0417204363802017, 1.0714838774196362, 1.1012473184590703, 1.1310107594985046, 1.1607742005379391, 1.1905376415773734, 1.2203010826168077, 1.2500645236562422, 1.2798279646956765, 1.3095914057351106, 1.339354846774545, 1.3691182878139794, 1.3988817288534137, 1.4286451698928482, 1.4584086109322825, 1.4881720519717168, 1.5179354930111513, 1.5476989340505856, 1.5774623750900196, 1.6072258161294541, 1.6369892571688884, 1.6667526982083227, 1.6965161392477572, 1.7262795802871915, 1.7560430213266258, 1.78580646236606, 1.8155699034054944, 1.8453333444449287, 1.875096785484363, 1.9048602265237975, 1.9346236675632318, 1.964387108602666, 1.9941505496421004, 2.023913990681535, 2.053677431720969, 2.0834408727604035, 2.1132043137998378, 2.1429677548392725, 2.1727311958787063, 2.2024946369181406, 2.2322580779575754, 2.262021518997009, 2.291784960036444, 2.3215484010758782, 2.3513118421153125, 2.381075283154747, 2.410838724194181, 2.4406021652336154, 2.4703656062730497, 2.5001290473124844, 2.5298924883519183, 2.559655929391353, 2.5894193704307873, 2.619182811470221, 2.648946252509656, 2.67870969354909, 2.7084731345885245, 2.7382365756279587, 2.7680000166673935, 2.7977634577068273, 2.827526898746262, 2.8572903397856964, 2.88705378082513, 2.916817221864565, 2.9465806629039992, 2.9763441039434335 ], [ 0, 0.029940801852848146, 0.05988160370569629, 0.08982240555854444, 0.11976320741139258, 0.14970400926424074, 0.17964481111708888, 0.20958561296993702, 0.23952641482278517, 0.26946721667563334, 0.2994080185284815, 0.32934882038132957, 0.35928962223417776, 0.3892304240870259, 0.41917122593987405, 0.44911202779272213, 0.47905282964557033, 0.5089936314984185, 0.5389344333512667, 0.5688752352041148, 0.598816037056963, 0.628756838909811, 0.6586976407626591, 0.6886384426155073, 0.7185792444683555, 0.7485200463212036, 0.7784608481740518, 0.8084016500268999, 0.8383424518797481, 0.8682832537325963, 0.8982240555854443, 0.9281648574382925, 0.9581056592911407, 0.9880464611439889, 1.017987262996837, 1.0479280648496851, 1.0778688667025333, 1.1078096685553813, 1.1377504704082295, 1.1676912722610777, 1.197632074113926, 1.227572875966774, 1.257513677819622, 1.2874544796724703, 1.3173952815253183, 1.3473360833781665, 1.3772768852310147, 1.4072176870838629, 1.437158488936711, 1.4670992907895593, 1.4970400926424072, 1.5269808944952554, 1.5569216963481036, 1.5868624982009516, 1.6168033000537998, 1.646744101906648, 1.6766849037594962, 1.7066257056123444, 1.7365665074651926, 1.7665073093180408, 1.7964481111708885, 1.8263889130237367, 1.856329714876585, 1.8862705167294331, 1.9162113185822813, 1.9461521204351295, 1.9760929222879777, 2.0060337241408255, 2.035974525993674, 2.065915327846522, 2.0958561296993703, 2.1257969315522183, 2.1557377334050667, 2.1856785352579147, 2.2156193371107626, 2.245560138963611, 2.275500940816459, 2.3054417426693075, 2.3353825445221554, 2.365323346375004, 2.395264148227852, 2.4252049500807, 2.455145751933548, 2.4850865537863958, 2.515027355639244, 2.544968157492092, 2.5749089593449406, 2.6048497611977885, 2.6347905630506365, 2.664731364903485, 2.694672166756333, 2.7246129686091813, 2.7545537704620293, 2.7844945723148777, 2.8144353741677257, 2.844376176020574, 2.874316977873422, 2.90425777972627, 2.9341985815791185, 2.9641393834319665, 2.9940801852848145 ], [ 0, 0.03, 0.06, 0.09, 0.12, 0.15, 0.18, 0.21, 0.24, 0.27, 0.3, 0.32999999999999996, 0.36, 0.39, 0.42, 0.44999999999999996, 0.48, 0.51, 0.54, 0.57, 0.6, 0.63, 0.6599999999999999, 0.69, 0.72, 0.75, 0.78, 0.8099999999999999, 0.84, 0.87, 0.8999999999999999, 0.9299999999999999, 0.96, 0.99, 1.02, 1.05, 1.08, 1.1099999999999999, 1.14, 1.17, 1.2, 1.23, 1.26, 1.29, 1.3199999999999998, 1.3499999999999999, 1.38, 1.41, 1.44, 1.47, 1.5, 1.53, 1.56, 1.5899999999999999, 1.6199999999999999, 1.65, 1.68, 1.71, 1.74, 1.77, 1.7999999999999998, 1.8299999999999998, 1.8599999999999999, 1.89, 1.92, 1.95, 1.98, 2.01, 2.04, 2.07, 2.1, 2.13, 2.16, 2.19, 2.2199999999999998, 2.25, 2.28, 2.31, 2.34, 2.37, 2.4, 2.4299999999999997, 2.46, 2.4899999999999998, 2.52, 2.55, 2.58, 2.61, 2.6399999999999997, 2.67, 2.6999999999999997, 2.73, 2.76, 2.79, 2.82, 2.85, 2.88, 2.9099999999999997, 2.94, 2.9699999999999998, 3 ] ], "y": [ [ 0, 0, 0, 0, 0, 0, 0, 0, 0, 0, 0, 0, 0, 0, 0, 0, 0, 0, 0, 0, 0, 0, 0, 0, 0, 0, 0, 0, 0, 0, 0, 0, 0, 0, 0, 0, 0, 0, 0, 0, 0, 0, 0, 0, 0, 0, 0, 0, 0, 0, 0, 0, 0, 0, 0, 0, 0, 0, 0, 0, 0, 0, 0, 0, 0, 0, 0, 0, 0, 0, 0, 0, 0, 0, 0, 0, 0, 0, 0, 0, 0, 0, 0, 0, 0, 0, 0, 0, 0, 0, 0, 0, 0, 0, 0, 0, 0, 0, 0, 0, 0 ], [ 0, 0.0018837155858794012, 0.0037674311717588024, 0.005651146757638203, 0.007534862343517605, 0.009418577929397006, 0.011302293515276406, 0.013186009101155808, 0.01506972468703521, 0.01695344027291461, 0.018837155858794013, 0.02072087144467341, 0.022604587030552813, 0.024488302616432218, 0.026372018202311616, 0.028255733788191014, 0.03013944937407042, 0.03202316495994982, 0.03390688054582922, 0.03579059613170862, 0.037674311717588026, 0.03955802730346743, 0.04144174288934682, 0.043325458475226224, 0.045209174061105625, 0.047092889646985034, 0.048976605232864436, 0.05086032081874383, 0.05274403640462323, 0.054627751990502633, 0.05651146757638203, 0.05839518316226144, 0.06027889874814084, 0.06216261433402024, 0.06404632991989964, 0.06593004550577905, 0.06781376109165844, 0.06969747667753784, 0.07158119226341723, 0.07346490784929664, 0.07534862343517605, 0.07723233902105545, 0.07911605460693485, 0.08099977019281425, 0.08288348577869364, 0.08476720136457305, 0.08665091695045245, 0.08853463253633186, 0.09041834812221125, 0.09230206370809066, 0.09418577929397007, 0.09606949487984946, 0.09795321046572887, 0.09983692605160825, 0.10172064163748766, 0.10360435722336706, 0.10548807280924646, 0.10737178839512587, 0.10925550398100527, 0.11113921956688468, 0.11302293515276406, 0.11490665073864346, 0.11679036632452287, 0.11867408191040227, 0.12055779749628168, 0.12244151308216107, 0.12432522866804048, 0.12620894425391987, 0.12809265983979928, 0.12997637542567866, 0.1318600910115581, 0.13374380659743748, 0.1356275221833169, 0.1375112377691963, 0.13939495335507568, 0.1412786689409551, 0.14316238452683447, 0.1450461001127139, 0.14692981569859329, 0.1488135312844727, 0.1506972468703521, 0.15258096245623148, 0.1544646780421109, 0.15634839362799027, 0.1582321092138697, 0.1601158247997491, 0.1619995403856285, 0.1638832559715079, 0.1657669715573873, 0.1676506871432667, 0.1695344027291461, 0.17141811831502551, 0.1733018339009049, 0.1751855494867843, 0.1770692650726637, 0.17895298065854312, 0.1808366962444225, 0.1827204118303019, 0.18460412741618132, 0.1864878430020607, 0.18837155858794014 ], [ 0, 0.0037599970069291276, 0.007519994013858255, 0.011279991020787384, 0.01503998802771651, 0.01879998503464564, 0.022559982041574767, 0.026319979048503892, 0.03007997605543302, 0.03383997306236215, 0.03759997006929128, 0.0413599670762204, 0.045119964083149534, 0.04887996109007866, 0.052639958097007784, 0.05639995510393691, 0.06015995211086604, 0.06391994911779517, 0.0676799461247243, 0.07143994313165342, 0.07519994013858255, 0.07895993714551168, 0.0827199341524408, 0.08647993115936993, 0.09023992816629907, 0.0939999251732282, 0.09775992218015732, 0.10151991918708644, 0.10527991619401557, 0.10903991320094471, 0.11279991020787382, 0.11655990721480296, 0.12031990422173208, 0.12407990122866122, 0.12783989823559033, 0.13159989524251947, 0.1353598922494486, 0.13911988925637772, 0.14287988626330683, 0.14663988327023597, 0.1503998802771651, 0.15415987728409425, 0.15791987429102336, 0.1616798712979525, 0.1654398683048816, 0.16919986531181072, 0.17295986231873986, 0.176719859325669, 0.18047985633259814, 0.18423985333952725, 0.1879998503464564, 0.19175984735338553, 0.19551984436031464, 0.19927984136724375, 0.2030398383741729, 0.20679983538110203, 0.21055983238803114, 0.21431982939496028, 0.21807982640188942, 0.22183982340881853, 0.22559982041574764, 0.22935981742267678, 0.23311981442960591, 0.23687981143653503, 0.24063980844346416, 0.2443998054503933, 0.24815980245732244, 0.2519197994642515, 0.25567979647118066, 0.2594397934781098, 0.26319979048503894, 0.2669597874919681, 0.2707197844988972, 0.2744797815058263, 0.27823977851275544, 0.2819997755196846, 0.28575977252661366, 0.28951976953354286, 0.29327976654047194, 0.2970397635474011, 0.3007997605543302, 0.3045597575612593, 0.3083197545681885, 0.3120797515751176, 0.3158397485820467, 0.31959974558897586, 0.323359742595905, 0.3271197396028341, 0.3308797366097632, 0.33463973361669236, 0.33839973062362144, 0.34215972763055064, 0.3459197246374797, 0.34967972164440886, 0.353439718651338, 0.35719971565826714, 0.3609597126651963, 0.36471970967212536, 0.3684797066790545, 0.37223970368598364, 0.3759997006929128 ], [ 0, 0.005621439437571739, 0.011242878875143478, 0.016864318312715215, 0.022485757750286956, 0.028107197187858694, 0.03372863662543043, 0.03935007606300217, 0.04497151550057391, 0.050592954938145654, 0.05621439437571739, 0.06183583381328912, 0.06745727325086086, 0.0730787126884326, 0.07870015212600434, 0.08432159156357608, 0.08994303100114782, 0.09556447043871956, 0.10118590987629131, 0.10680734931386303, 0.11242878875143478, 0.11805022818900651, 0.12367166762657825, 0.12929310706414998, 0.13491454650172172, 0.14053598593929348, 0.1461574253768652, 0.15177886481443695, 0.15740030425200868, 0.16302174368958042, 0.16864318312715215, 0.1742646225647239, 0.17988606200229565, 0.18550750143986738, 0.19112894087743912, 0.19675038031501088, 0.20237181975258262, 0.20799325919015432, 0.21361469862772606, 0.2192361380652978, 0.22485757750286955, 0.2304790169404413, 0.23610045637801302, 0.2417218958155848, 0.2473433352531565, 0.2529647746907282, 0.25858621412829996, 0.2642076535658717, 0.26982909300344343, 0.2754505324410152, 0.28107197187858696, 0.28669341131615866, 0.2923148507537304, 0.29793629019130213, 0.3035577296288739, 0.3091791690664456, 0.31480060850401737, 0.3204220479415891, 0.32604348737916083, 0.3316649268167326, 0.3372863662543043, 0.34290780569187607, 0.3485292451294478, 0.35415068456701954, 0.3597721240045913, 0.365393563442163, 0.37101500287973477, 0.3766364423173065, 0.38225788175487824, 0.38787932119244994, 0.39350076063002176, 0.3991222000675934, 0.40474363950516523, 0.41036507894273694, 0.41598651838030865, 0.4216079578178804, 0.4272293972554521, 0.4328508366930239, 0.4384722761305956, 0.4440937155681674, 0.4497151550057391, 0.4553365944433108, 0.4609580338808826, 0.4665794733184543, 0.47220091275602605, 0.47782235219359775, 0.4834437916311696, 0.4890652310687413, 0.494686670506313, 0.5003081099438847, 0.5059295493814564, 0.5115509888190283, 0.5171724282565999, 0.5227938676941717, 0.5284153071317435, 0.5340367465693152, 0.5396581860068869, 0.5452796254444586, 0.5509010648820304, 0.5565225043196022, 0.5621439437571739 ], [ 0, 0.007460696614945643, 0.014921393229891286, 0.02238208984483693, 0.029842786459782573, 0.037303483074728215, 0.04476417968967386, 0.05222487630461951, 0.059685572919565145, 0.0671462695345108, 0.07460696614945643, 0.08206766276440207, 0.08952835937934772, 0.09698905599429337, 0.10444975260923901, 0.11191044922418465, 0.11937114583913029, 0.12683184245407594, 0.1342925390690216, 0.14175323568396722, 0.14921393229891286, 0.15667462891385853, 0.16413532552880414, 0.1715960221437498, 0.17905671875869544, 0.18651741537364108, 0.19397811198858675, 0.20143880860353236, 0.20889950521847803, 0.21636020183342367, 0.2238208984483693, 0.23128159506331494, 0.23874229167826058, 0.24620298829320625, 0.2536636849081519, 0.2611243815230975, 0.2685850781380432, 0.2760457747529888, 0.28350647136793444, 0.2909671679828801, 0.2984278645978257, 0.3058885612127714, 0.31334925782771705, 0.3208099544426627, 0.3282706510576083, 0.3357313476725539, 0.3431920442874996, 0.35065274090244525, 0.3581134375173909, 0.3655741341323365, 0.37303483074728216, 0.38049552736222786, 0.3879562239771735, 0.3954169205921191, 0.4028776172070647, 0.4103383138220104, 0.41779901043695605, 0.4252597070519017, 0.43272040366684733, 0.44018110028179297, 0.4476417968967386, 0.45510249351168425, 0.4625631901266299, 0.4700238867415755, 0.47748458335652116, 0.48494527997146686, 0.4924059765864125, 0.4998666732013581, 0.5073273698163038, 0.5147880664312494, 0.522248763046195, 0.5297094596611407, 0.5371701562760864, 0.544630852891032, 0.5520915495059776, 0.5595522461209232, 0.5670129427358689, 0.5744736393508146, 0.5819343359657602, 0.5893950325807059, 0.5968557291956514, 0.6043164258105971, 0.6117771224255428, 0.6192378190404884, 0.6266985156554341, 0.6341592122703796, 0.6416199088853254, 0.649080605500271, 0.6565413021152166, 0.6640019987301623, 0.6714626953451078, 0.6789233919600536, 0.6863840885749992, 0.6938447851899449, 0.7013054818048905, 0.7087661784198361, 0.7162268750347818, 0.7236875716497274, 0.731148268264673, 0.7386089648796187, 0.7460696614945643 ], [ 0, 0.009270509831248422, 0.018541019662496844, 0.027811529493745265, 0.03708203932499369, 0.04635254915624211, 0.05562305898749053, 0.06489356881873895, 0.07416407864998738, 0.0834345884812358, 0.09270509831248422, 0.10197560814373263, 0.11124611797498106, 0.12051662780622949, 0.1297871376374779, 0.13905764746872631, 0.14832815729997476, 0.15759866713122317, 0.1668691769624716, 0.17613968679372, 0.18541019662496844, 0.19468070645621685, 0.20395121628746526, 0.21322172611871368, 0.22249223594996212, 0.23176274578121053, 0.24103325561245897, 0.25030376544370736, 0.2595742752749558, 0.26884478510620424, 0.27811529493745263, 0.28738580476870107, 0.2966563145999495, 0.3059268244311979, 0.31519733426244634, 0.3244678440936948, 0.3337383539249432, 0.34300886375619155, 0.35227937358744, 0.36154988341868843, 0.3708203932499369, 0.3800909030811853, 0.3893614129124337, 0.39863192274368214, 0.40790243257493053, 0.41717294240617897, 0.42644345223742736, 0.4357139620686758, 0.44498447189992424, 0.4542549817311727, 0.46352549156242107, 0.4727960013936695, 0.48206651122491795, 0.49133702105616633, 0.5006075308874147, 0.5098780407186632, 0.5191485505499116, 0.52841906038116, 0.5376895702124085, 0.5469600800436569, 0.5562305898749053, 0.5655010997061537, 0.5747716095374021, 0.5840421193686506, 0.593312629199899, 0.6025831390311475, 0.6118536488623958, 0.6211241586936442, 0.6303946685248927, 0.639665178356141, 0.6489356881873896, 0.6582061980186379, 0.6674767078498864, 0.6767472176811348, 0.6860177275123831, 0.6952882373436317, 0.70455874717488, 0.7138292570061285, 0.7230997668373769, 0.7323702766686253, 0.7416407864998737, 0.7509112963311221, 0.7601818061623706, 0.769452315993619, 0.7787228258248674, 0.7879933356561158, 0.7972638454873643, 0.8065343553186126, 0.8158048651498611, 0.8250753749811095, 0.8343458848123579, 0.8436163946436064, 0.8528869044748547, 0.8621574143061033, 0.8714279241373516, 0.8806984339686001, 0.8899689437998485, 0.8992394536310968, 0.9085099634623454, 0.9177804732935937, 0.9270509831248421 ], [ 0, 0.011043736580540338, 0.022087473161080676, 0.033131209741621016, 0.04417494632216135, 0.055218682902701696, 0.06626241948324203, 0.07730615606378237, 0.0883498926443227, 0.09939362922486306, 0.11043736580540339, 0.12148110238594372, 0.13252483896648407, 0.1435685755470244, 0.15461231212756474, 0.16565604870810507, 0.1766997852886454, 0.18774352186918578, 0.1987872584497261, 0.20983099503026642, 0.22087473161080678, 0.23191846819134712, 0.24296220477188743, 0.25400594135242777, 0.26504967793296813, 0.2760934145135085, 0.2871371510940488, 0.2981808876745891, 0.3092246242551295, 0.32026836083566984, 0.33131209741621015, 0.3423558339967505, 0.3533995705772908, 0.3644433071578312, 0.37548704373837155, 0.3865307803189119, 0.3975745168994522, 0.40861825347999253, 0.41966199006053284, 0.4307057266410732, 0.44174946322161357, 0.4527931998021539, 0.46383693638269424, 0.4748806729632346, 0.48592440954377486, 0.4969681461243152, 0.5080118827048555, 0.5190556192853959, 0.5300993558659363, 0.5411430924464766, 0.552186829027017, 0.5632305656075574, 0.5742743021880976, 0.585318038768638, 0.5963617753491782, 0.6074055119297186, 0.618449248510259, 0.6294929850907993, 0.6405367216713397, 0.65158045825188, 0.6626241948324203, 0.6736679314129607, 0.684711667993501, 0.6957554045740414, 0.7067991411545816, 0.717842877735122, 0.7288866143156624, 0.7399303508962026, 0.7509740874767431, 0.7620178240572834, 0.7730615606378238, 0.7841052972183641, 0.7951490337989044, 0.8061927703794447, 0.8172365069599851, 0.8282802435405254, 0.8393239801210657, 0.8503677167016062, 0.8614114532821464, 0.8724551898626869, 0.8834989264432271, 0.8945426630237674, 0.9055863996043078, 0.9166301361848481, 0.9276738727653885, 0.9387176093459287, 0.9497613459264692, 0.9608050825070095, 0.9718488190875497, 0.9828925556680902, 0.9939362922486304, 1.0049800288291708, 1.016023765409711, 1.0270675019902515, 1.0381112385707918, 1.0491549751513323, 1.0601987117318725, 1.0712424483124128, 1.0822861848929533, 1.0933299214734935, 1.104373658054034 ], [ 0, 0.01277337874695218, 0.02554675749390436, 0.03832013624085654, 0.05109351498780872, 0.0638668937347609, 0.07664027248171308, 0.08941365122866526, 0.10218702997561745, 0.11496040872256964, 0.1277337874695218, 0.14050716621647397, 0.15328054496342616, 0.16605392371037836, 0.17882730245733053, 0.1916006812042827, 0.2043740599512349, 0.2171474386981871, 0.22992081744513929, 0.24269419619209143, 0.2554675749390436, 0.2682409536859958, 0.28101433243294793, 0.29378771117990016, 0.3065610899268523, 0.31933446867380455, 0.3321078474207567, 0.3448812261677089, 0.35765460491466106, 0.3704279836616133, 0.3832013624085654, 0.3959747411555176, 0.4087481199024698, 0.421521498649422, 0.4342948773963742, 0.44706825614332635, 0.45984163489027857, 0.4726150136372307, 0.48538839238418285, 0.498161771131135, 0.5109351498780872, 0.5237085286250395, 0.5364819073719916, 0.5492552861189438, 0.5620286648658959, 0.5748020436128481, 0.5875754223598003, 0.6003488011067525, 0.6131221798537047, 0.6258955586006569, 0.6386689373476091, 0.6514423160945613, 0.6642156948415134, 0.6769890735884656, 0.6897624523354178, 0.7025358310823699, 0.7153092098293221, 0.7280825885762743, 0.7408559673232266, 0.7536293460701787, 0.7664027248171308, 0.779176103564083, 0.7919494823110352, 0.8047228610579874, 0.8174962398049396, 0.8302696185518917, 0.843042997298844, 0.8558163760457961, 0.8685897547927484, 0.8813631335397004, 0.8941365122866527, 0.9069098910336049, 0.9196832697805571, 0.9324566485275092, 0.9452300272744614, 0.9580034060214137, 0.9707767847683657, 0.983550163515318, 0.99632354226227, 1.0090969210092224, 1.0218702997561744, 1.0346436785031266, 1.047417057250079, 1.0601904359970309, 1.0729638147439833, 1.0857371934909354, 1.0985105722378876, 1.1112839509848398, 1.1240573297317917, 1.1368307084787441, 1.1496040872256963, 1.1623774659726485, 1.1751508447196006, 1.1879242234665528, 1.200697602213505, 1.2134709809604574, 1.2262443597074093, 1.2390177384543615, 1.2517911172013139, 1.2645644959482658, 1.2773378746952182 ], [ 0, 0.014452610223051459, 0.028905220446102917, 0.04335783066915438, 0.057810440892205835, 0.0722630511152573, 0.08671566133830876, 0.10116827156136021, 0.11562088178441167, 0.13007349200746315, 0.1445261022305146, 0.15897871245356604, 0.1734313226766175, 0.18788393289966898, 0.20233654312272042, 0.21678915334577187, 0.23124176356882334, 0.2456943737918748, 0.2601469840149263, 0.2745995942379777, 0.2890522044610292, 0.30350481468408064, 0.3179574249071321, 0.3324100351301835, 0.346862645353235, 0.3613152555762865, 0.37576786579933796, 0.3902204760223894, 0.40467308624544085, 0.41912569646849235, 0.43357830669154374, 0.44803091691459523, 0.4624835271376467, 0.4769361373606982, 0.4913887475837496, 0.5058413578068011, 0.5202939680298526, 0.534746578252904, 0.5491991884759554, 0.563651798699007, 0.5781044089220584, 0.5925570191451098, 0.6070096293681613, 0.6214622395912128, 0.6359148498142642, 0.6503674600373156, 0.664820070260367, 0.6792726804834186, 0.69372529070647, 0.7081779009295215, 0.722630511152573, 0.7370831213756245, 0.7515357315986759, 0.7659883418217273, 0.7804409520447788, 0.7948935622678303, 0.8093461724908817, 0.8237987827139331, 0.8382513929369847, 0.8527040031600361, 0.8671566133830875, 0.8816092236061389, 0.8960618338291905, 0.9105144440522419, 0.9249670542752934, 0.9394196644983449, 0.9538722747213964, 0.9683248849444477, 0.9827774951674992, 0.9972301053905507, 1.0116827156136021, 1.0261353258366537, 1.0405879360597052, 1.0550405462827566, 1.069493156505808, 1.0839457667288595, 1.0983983769519108, 1.1128509871749623, 1.127303597398014, 1.1417562076210654, 1.1562088178441168, 1.170661428067168, 1.1851140382902197, 1.199566648513271, 1.2140192587363225, 1.2284718689593739, 1.2429244791824257, 1.257377089405477, 1.2718296996285283, 1.2862823098515799, 1.3007349200746312, 1.3151875302976828, 1.329640140520734, 1.3440927507437859, 1.3585453609668372, 1.3729979711898888, 1.38745058141294, 1.4019031916359914, 1.416355801859043, 1.4308084120820943, 1.445261022305146 ], [ 0, 0.0160748038493699, 0.0321496076987398, 0.0482244115481097, 0.0642992153974796, 0.08037401924684949, 0.0964488230962194, 0.11252362694558929, 0.1285984307949592, 0.1446732346443291, 0.16074803849369898, 0.1768228423430689, 0.1928976461924388, 0.2089724500418087, 0.22504725389117858, 0.24112205774054848, 0.2571968615899184, 0.2732716654392883, 0.2893464692886582, 0.3054212731380281, 0.32149607698739796, 0.3375708808367679, 0.3536456846861378, 0.36972048853550765, 0.3857952923848776, 0.4018700962342475, 0.4179449000836174, 0.4340197039329873, 0.45009450778235716, 0.4661693116317271, 0.48224411548109697, 0.49831891933046685, 0.5143937231798368, 0.5304685270292067, 0.5465433308785766, 0.5626181347279465, 0.5786929385773164, 0.5947677424266862, 0.6108425462760562, 0.626917350125426, 0.6429921539747959, 0.6590669578241659, 0.6751417616735358, 0.6912165655229057, 0.7072913693722755, 0.7233661732216454, 0.7394409770710153, 0.7555157809203853, 0.7715905847697552, 0.787665388619125, 0.803740192468495, 0.8198149963178649, 0.8358898001672348, 0.8519646040166046, 0.8680394078659746, 0.8841142117153444, 0.9001890155647143, 0.9162638194140843, 0.9323386232634542, 0.9484134271128241, 0.9644882309621939, 0.9805630348115638, 0.9966378386609337, 1.0127126425103037, 1.0287874463596736, 1.0448622502090434, 1.0609370540584133, 1.0770118579077832, 1.0930866617571533, 1.109161465606523, 1.125236269455893, 1.1413110733052627, 1.1573858771546328, 1.1734606810040027, 1.1895354848533723, 1.2056102887027424, 1.2216850925521123, 1.2377598964014822, 1.253834700250852, 1.2699095041002222, 1.2859843079495918, 1.3020591117989617, 1.3181339156483318, 1.3342087194977015, 1.3502835233470716, 1.3663583271964415, 1.3824331310458113, 1.3985079348951812, 1.414582738744551, 1.430657542593921, 1.4467323464432909, 1.462807150292661, 1.4788819541420306, 1.4949567579914007, 1.5110315618407706, 1.5271063656901405, 1.5431811695395103, 1.55925597338888, 1.57533077723825, 1.59140558108762, 1.60748038493699 ], [ 0, 0.017633557568774192, 0.035267115137548384, 0.05290067270632258, 0.07053423027509677, 0.08816778784387097, 0.10580134541264516, 0.12343490298141935, 0.14106846055019354, 0.15870201811896775, 0.17633557568774194, 0.1939691332565161, 0.21160269082529032, 0.22923624839406453, 0.2468698059628387, 0.2645033635316129, 0.2821369211003871, 0.2997704786691613, 0.3174040362379355, 0.33503759380670967, 0.3526711513754839, 0.3703047089442581, 0.3879382665130322, 0.4055718240818064, 0.42320538165058064, 0.44083893921935485, 0.45847249678812907, 0.47610605435690323, 0.4937396119256774, 0.5113731694944517, 0.5290067270632258, 0.546640284632, 0.5642738422007741, 0.5819073997695484, 0.5995409573383226, 0.6171745149070968, 0.634808072475871, 0.6524416300446451, 0.6700751876134193, 0.6877087451821935, 0.7053423027509678, 0.7229758603197419, 0.7406094178885162, 0.7582429754572904, 0.7758765330260644, 0.7935100905948387, 0.8111436481636128, 0.8287772057323871, 0.8464107633011613, 0.8640443208699355, 0.8816778784387097, 0.8993114360074839, 0.9169449935762581, 0.9345785511450322, 0.9522121087138065, 0.9698456662825806, 0.9874792238513548, 1.005112781420129, 1.0227463389889033, 1.0403798965576774, 1.0580134541264516, 1.0756470116952257, 1.093280569264, 1.1109141268327742, 1.1285476844015483, 1.1461812419703226, 1.1638147995390968, 1.1814483571078709, 1.1990819146766452, 1.2167154722454192, 1.2343490298141937, 1.2519825873829677, 1.269616144951742, 1.287249702520516, 1.3048832600892901, 1.3225168176580646, 1.3401503752268387, 1.357783932795613, 1.375417490364387, 1.3930510479331615, 1.4106846055019355, 1.4283181630707096, 1.4459517206394839, 1.463585278208258, 1.4812188357770324, 1.4988523933458064, 1.5164859509145807, 1.5341195084833548, 1.5517530660521288, 1.5693866236209033, 1.5870201811896774, 1.6046537387584516, 1.6222872963272257, 1.6399208538960002, 1.6575544114647742, 1.6751879690335485, 1.6928215266023225, 1.7104550841710966, 1.728088641739871, 1.7457221993086451, 1.7633557568774194 ], [ 0, 0.019122719692460693, 0.038245439384921386, 0.057368159077382075, 0.07649087876984277, 0.09561359846230345, 0.11473631815476415, 0.13385903784722483, 0.15298175753968554, 0.17210447723214625, 0.1912271969246069, 0.2103499166170676, 0.2294726363095283, 0.248595356001989, 0.26771807569444966, 0.2868407953869104, 0.3059635150793711, 0.3250862347718318, 0.3442089544642925, 0.3633316741567531, 0.3824543938492138, 0.4015771135416745, 0.4206998332341352, 0.4398225529265959, 0.4589452726190566, 0.4780679923115173, 0.497190712003978, 0.5163134316964386, 0.5354361513888993, 0.55455887108136, 0.5736815907738207, 0.5928043104662815, 0.6119270301587422, 0.6310497498512029, 0.6501724695436636, 0.6692951892361243, 0.688417908928585, 0.7075406286210455, 0.7266633483135062, 0.7457860680059669, 0.7649087876984276, 0.7840315073908883, 0.803154227083349, 0.8222769467758098, 0.8413996664682704, 0.8605223861607311, 0.8796451058531918, 0.8987678255456525, 0.9178905452381132, 0.9370132649305739, 0.9561359846230346, 0.9752587043154953, 0.994381424007956, 1.0135041437004166, 1.0326268633928772, 1.051749583085338, 1.0708723027777987, 1.0899950224702595, 1.10911774216272, 1.128240461855181, 1.1473631815476415, 1.166485901240102, 1.185608620932563, 1.2047313406250235, 1.2238540603174843, 1.242976780009945, 1.2620994997024058, 1.2812222193948664, 1.3003449390873272, 1.3194676587797878, 1.3385903784722486, 1.3577130981647092, 1.37683581785717, 1.3959585375496306, 1.415081257242091, 1.434203976934552, 1.4533266966270124, 1.4724494163194732, 1.4915721360119338, 1.5106948557043947, 1.5298175753968553, 1.5489402950893159, 1.5680630147817767, 1.5871857344742373, 1.606308454166698, 1.6254311738591587, 1.6445538935516195, 1.6636766132440801, 1.6827993329365407, 1.7019220526290015, 1.7210447723214621, 1.740167492013923, 1.7592902117063836, 1.7784129313988444, 1.797535651091305, 1.8166583707837658, 1.8357810904762264, 1.854903810168687, 1.8740265298611478, 1.8931492495536084, 1.9122719692460692 ], [ 0, 0.02053641317786066, 0.04107282635572132, 0.06160923953358198, 0.08214565271144264, 0.1026820658893033, 0.12321847906716396, 0.14375489224502463, 0.16429130542288528, 0.18482771860074595, 0.2053641317786066, 0.22590054495646725, 0.24643695813432792, 0.2669733713121886, 0.28750978449004927, 0.3080461976679099, 0.32858261084577056, 0.34911902402363126, 0.3696554372014919, 0.39019185037935256, 0.4107282635572132, 0.4312646767350739, 0.4518010899129345, 0.4723375030907952, 0.49287391626865584, 0.5134103294465165, 0.5339467426243772, 0.5544831558022378, 0.5750195689800985, 0.5955559821579592, 0.6160923953358198, 0.6366288085136804, 0.6571652216915411, 0.6777016348694018, 0.6982380480472625, 0.7187744612251232, 0.7393108744029838, 0.7598472875808444, 0.7803837007587051, 0.8009201139365658, 0.8214565271144264, 0.8419929402922871, 0.8625293534701478, 0.8830657666480085, 0.903602179825869, 0.9241385930037297, 0.9446750061815904, 0.9652114193594511, 0.9857478325373117, 1.0062842457151724, 1.026820658893033, 1.0473570720708938, 1.0678934852487545, 1.088429898426615, 1.1089663116044757, 1.1295027247823364, 1.150039137960197, 1.1705755511380578, 1.1911119643159185, 1.211648377493779, 1.2321847906716397, 1.2527212038495004, 1.2732576170273608, 1.2937940302052215, 1.3143304433830822, 1.334866856560943, 1.3554032697388037, 1.3759396829166641, 1.396476096094525, 1.4170125092723855, 1.4375489224502465, 1.458085335628107, 1.4786217488059676, 1.4991581619838283, 1.5196945751616888, 1.5402309883395495, 1.5607674015174102, 1.581303814695271, 1.6018402278731316, 1.6223766410509923, 1.6429130542288528, 1.6634494674067135, 1.6839858805845742, 1.7045222937624347, 1.7250587069402956, 1.745595120118156, 1.766131533296017, 1.7866679464738775, 1.807204359651738, 1.827740772829599, 1.8482771860074594, 1.8688135991853203, 1.8893500123631808, 1.9098864255410415, 1.9304228387189022, 1.950959251896763, 1.9714956650746234, 1.992032078252484, 2.0125684914303448, 2.0331049046082055, 2.053641317786066 ], [ 0, 0.021869058822642347, 0.043738117645284694, 0.06560717646792703, 0.08747623529056939, 0.10934529411321173, 0.13121435293585407, 0.1530834117584964, 0.17495247058113877, 0.19682152940378114, 0.21869058822642345, 0.2405596470490658, 0.26242870587170813, 0.28429776469435053, 0.3061668235169928, 0.32803588233963515, 0.34990494116227755, 0.3717739999849199, 0.3936430588075623, 0.41551211763020457, 0.4373811764528469, 0.4592502352754893, 0.4811192940981316, 0.5029883529207739, 0.5248574117434163, 0.5467264705660586, 0.5685955293887011, 0.5904645882113433, 0.6123336470339856, 0.6342027058566281, 0.6560717646792703, 0.6779408235019126, 0.6998098823245551, 0.7216789411471974, 0.7435479999698398, 0.7654170587924821, 0.7872861176151246, 0.8091551764377667, 0.8310242352604091, 0.8528932940830515, 0.8747623529056938, 0.8966314117283362, 0.9185004705509786, 0.940369529373621, 0.9622385881962632, 0.9841076470189055, 1.0059767058415479, 1.0278457646641903, 1.0497148234868325, 1.071583882309475, 1.0934529411321172, 1.1153219999547597, 1.1371910587774021, 1.1590601176000443, 1.1809291764226866, 1.202798235245329, 1.2246672940679713, 1.2465363528906137, 1.2684054117132562, 1.2902744705358984, 1.3121435293585406, 1.334012588181183, 1.3558816470038253, 1.3777507058264677, 1.3996197646491102, 1.4214888234717524, 1.4433578822943949, 1.465226941117037, 1.4870959999396796, 1.5089650587623218, 1.5308341175849642, 1.5527031764076065, 1.5745722352302491, 1.5964412940528914, 1.6183103528755334, 1.640179411698176, 1.6620484705208183, 1.6839175293434607, 1.705786588166103, 1.7276556469887454, 1.7495247058113876, 1.7713937646340299, 1.7932628234566723, 1.8151318822793145, 1.8370009411019572, 1.8588699999245992, 1.880739058747242, 1.9026081175698841, 1.9244771763925264, 1.9463462352151688, 1.968215294037811, 1.9900843528604535, 2.0119534116830957, 2.033822470505738, 2.0556915293283806, 2.077560588151023, 2.099429646973665, 2.1212987057963075, 2.14316776461895, 2.165036823441592, 2.1869058822642344 ], [ 0, 0.023115397283273678, 0.046230794566547356, 0.06934619184982103, 0.09246158913309471, 0.11557698641636838, 0.13869238369964207, 0.16180778098291573, 0.18492317826618943, 0.20803857554946312, 0.23115397283273675, 0.2542693701160104, 0.27738476739928414, 0.3005001646825578, 0.32361556196583147, 0.34673095924910513, 0.36984635653237885, 0.3929617538156525, 0.41607715109892623, 0.43919254838219984, 0.4623079456654735, 0.4854233429487472, 0.5085387402320208, 0.5316541375152946, 0.5547695347985683, 0.577884932081842, 0.6010003293651156, 0.6241157266483892, 0.6472311239316629, 0.6703465212149367, 0.6934619184982103, 0.716577315781484, 0.7396927130647577, 0.7628081103480313, 0.785923507631305, 0.8090389049145787, 0.8321543021978525, 0.855269699481126, 0.8783850967643997, 0.9015004940476734, 0.924615891330947, 0.9477312886142207, 0.9708466858974945, 0.9939620831807682, 1.0170774804640417, 1.0401928777473153, 1.0633082750305891, 1.0864236723138627, 1.1095390695971366, 1.1326544668804102, 1.155769864163684, 1.1788852614469576, 1.2020006587302312, 1.2251160560135048, 1.2482314532967784, 1.2713468505800523, 1.2944622478633259, 1.3175776451465997, 1.3406930424298733, 1.363808439713147, 1.3869238369964205, 1.4100392342796941, 1.433154631562968, 1.4562700288462416, 1.4793854261295154, 1.502500823412789, 1.5256162206960626, 1.5487316179793362, 1.57184701526261, 1.5949624125458837, 1.6180778098291575, 1.641193207112431, 1.664308604395705, 1.6874240016789783, 1.710539398962252, 1.7336547962455258, 1.7567701935287994, 1.7798855908120732, 1.8030009880953468, 1.8261163853786206, 1.849231782661894, 1.8723471799451676, 1.8954625772284415, 1.918577974511715, 1.941693371794989, 1.9648087690782625, 1.9879241663615363, 2.0110395636448097, 2.0341549609280833, 2.0572703582113574, 2.0803857554946306, 2.1035011527779046, 2.1266165500611782, 2.149731947344452, 2.1728473446277254, 2.1959627419109995, 2.219078139194273, 2.2421935364775467, 2.2653089337608203, 2.288424331044094, 2.311539728327368 ], [ 0, 0.024270509831248423, 0.04854101966249685, 0.07281152949374527, 0.0970820393249937, 0.1213525491562421, 0.14562305898749053, 0.16989356881873896, 0.1941640786499874, 0.21843458848123581, 0.2427050983124842, 0.2669756081437326, 0.29124611797498107, 0.3155166278062295, 0.3397871376374779, 0.3640576474687263, 0.3883281572999748, 0.41259866713122323, 0.43686917696247163, 0.46113968679372, 0.4854101966249684, 0.5096807064562169, 0.5339512162874652, 0.5582217261187137, 0.5824922359499621, 0.6067627457812106, 0.631033255612459, 0.6553037654437074, 0.6795742752749558, 0.7038447851062043, 0.7281152949374526, 0.7523858047687011, 0.7766563145999495, 0.800926824431198, 0.8251973342624465, 0.8494678440936949, 0.8737383539249433, 0.8980088637561916, 0.92227937358744, 0.9465498834186885, 0.9708203932499369, 0.9950909030811853, 1.0193614129124338, 1.0436319227436823, 1.0679024325749304, 1.092172942406179, 1.1164434522374274, 1.140713962068676, 1.1649844718999243, 1.1892549817311728, 1.2135254915624212, 1.2377960013936695, 1.262066511224918, 1.2863370210561664, 1.3106075308874148, 1.3348780407186631, 1.3591485505499117, 1.38341906038116, 1.4076895702124086, 1.431960080043657, 1.4562305898749053, 1.4805010997061536, 1.5047716095374022, 1.5290421193686505, 1.553312629199899, 1.5775831390311474, 1.601853648862396, 1.6261241586936441, 1.650394668524893, 1.674665178356141, 1.6989356881873898, 1.723206198018638, 1.7474767078498865, 1.7717472176811349, 1.7960177275123832, 1.8202882373436318, 1.84455874717488, 1.8688292570061287, 1.893099766837377, 1.9173702766686256, 1.9416407864998737, 1.965911296331122, 1.9901818061623706, 2.014452315993619, 2.0387228258248675, 2.062993335656116, 2.0872638454873647, 2.1115343553186126, 2.135804865149861, 2.1600753749811097, 2.184345884812358, 2.2086163946436064, 2.2328869044748547, 2.2571574143061035, 2.281427924137352, 2.3056984339686, 2.3299689437998485, 2.354239453631097, 2.3785099634623457, 2.4027804732935936, 2.4270509831248424 ], [ 0, 0.02532983776506045, 0.0506596755301209, 0.07598951329518136, 0.1013193510602418, 0.12664918882530227, 0.15197902659036272, 0.17730886435542315, 0.2026387021204836, 0.22796853988554408, 0.25329837765060453, 0.27862821541566496, 0.30395805318072544, 0.32928789094578587, 0.3546177287108463, 0.3799475664759068, 0.4052774042409672, 0.4306072420060277, 0.45593707977108816, 0.48126691753614853, 0.5065967553012091, 0.5319265930662695, 0.5572564308313299, 0.5825862685963904, 0.6079161063614509, 0.6332459441265113, 0.6585757818915717, 0.6839056196566322, 0.7092354574216926, 0.7345652951867531, 0.7598951329518135, 0.7852249707168739, 0.8105548084819344, 0.8358846462469949, 0.8612144840120554, 0.8865443217771158, 0.9118741595421763, 0.9372039973072366, 0.9625338350722971, 0.9878636728373575, 1.0131935106024181, 1.0385233483674785, 1.063853186132539, 1.0891830238975995, 1.1145128616626598, 1.1398426994277202, 1.1651725371927808, 1.1905023749578412, 1.2158322127229018, 1.2411620504879621, 1.2664918882530225, 1.291821726018083, 1.3171515637831435, 1.3424814015482038, 1.3678112393132644, 1.3931410770783248, 1.4184709148433852, 1.4438007526084458, 1.4691305903735061, 1.4944604281385667, 1.519790265903627, 1.5451201036686875, 1.5704499414337478, 1.5957797791988084, 1.6211096169638688, 1.6464394547289294, 1.6717692924939898, 1.6970991302590501, 1.7224289680241107, 1.747758805789171, 1.7730886435542317, 1.798418481319292, 1.8237483190843526, 1.849078156849413, 1.8744079946144732, 1.899737832379534, 1.9250676701445941, 1.950397507909655, 1.975727345674715, 2.0010571834397757, 2.0263870212048363, 2.0517168589698964, 2.077046696734957, 2.102376534500017, 2.127706372265078, 2.1530362100301383, 2.178366047795199, 2.203695885560259, 2.2290257233253197, 2.2543555610903803, 2.2796853988554404, 2.305015236620501, 2.3303450743855616, 2.355674912150622, 2.3810047499156823, 2.406334587680743, 2.4316644254458035, 2.4569942632108637, 2.4823241009759243, 2.5076539387409844, 2.532983776506045 ], [ 0, 0.02628920040131591, 0.05257840080263182, 0.07886760120394773, 0.10515680160526364, 0.13144600200657955, 0.15773520240789546, 0.18402440280921137, 0.21031360321052728, 0.23660280361184322, 0.2628920040131591, 0.289181204414475, 0.3154704048157909, 0.34175960521710685, 0.36804880561842274, 0.3943380060197386, 0.42062720642105456, 0.4469164068223705, 0.47320560722368643, 0.49949480762500226, 0.5257840080263182, 0.5520732084276341, 0.57836240882895, 0.604651609230266, 0.6309408096315818, 0.6572300100328978, 0.6835192104342137, 0.7098084108355296, 0.7360976112368455, 0.7623868116381615, 0.7886760120394772, 0.8149652124407932, 0.8412544128421091, 0.8675436132434251, 0.893832813644741, 0.9201220140460569, 0.9464112144473729, 0.9727004148486886, 0.9989896152500045, 1.0252788156513204, 1.0515680160526364, 1.0778572164539524, 1.1041464168552682, 1.1304356172565841, 1.1567248176579, 1.183014018059216, 1.209303218460532, 1.2355924188618477, 1.2618816192631637, 1.2881708196644797, 1.3144600200657957, 1.3407492204671114, 1.3670384208684274, 1.3933276212697432, 1.4196168216710592, 1.445906022072375, 1.472195222473691, 1.498484422875007, 1.524773623276323, 1.5510628236776387, 1.5773520240789545, 1.6036412244802705, 1.6299304248815865, 1.6562196252829022, 1.6825088256842182, 1.7087980260855342, 1.7350872264868502, 1.7613764268881658, 1.787665627289482, 1.8139548276907977, 1.8402440280921137, 1.8665332284934295, 1.8928224288947457, 1.9191116292960615, 1.9454008296973773, 1.9716900300986933, 1.997979230500009, 2.024268430901325, 2.050557631302641, 2.076846831703957, 2.103136032105273, 2.1294252325065886, 2.1557144329079048, 2.1820036333092205, 2.2082928337105363, 2.234582034111852, 2.2608712345131683, 2.287160434914484, 2.3134496353158, 2.339738835717116, 2.366028036118432, 2.392317236519748, 2.418606436921064, 2.4448956373223796, 2.4711848377236953, 2.4974740381250116, 2.5237632385263273, 2.550052438927643, 2.5763416393289593, 2.602630839730275, 2.6289200401315913 ], [ 0, 0.027144811573980587, 0.05428962314796117, 0.08143443472194176, 0.10857924629592235, 0.13572405786990294, 0.1628688694438835, 0.19001368101786412, 0.2171584925918447, 0.2443033041658253, 0.2714481157398059, 0.29859292731378645, 0.325737738887767, 0.35288255046174766, 0.38002736203572823, 0.40717217360970875, 0.4343169851836894, 0.46146179675767, 0.4886066083316506, 0.5157514199056311, 0.5428962314796117, 0.5700410430535924, 0.5971858546275729, 0.6243306662015534, 0.651475477775534, 0.6786202893495147, 0.7057651009234953, 0.7329099124974758, 0.7600547240714565, 0.787199535645437, 0.8143443472194175, 0.8414891587933981, 0.8686339703673788, 0.8957787819413594, 0.92292359351534, 0.9500684050893206, 0.9772132166633012, 1.0043580282372817, 1.0315028398112622, 1.0586476513852427, 1.0857924629592235, 1.112937274533204, 1.1400820861071848, 1.1672268976811653, 1.1943717092551458, 1.2215165208291263, 1.2486613324031068, 1.2758061439770876, 1.302950955551068, 1.3300957671250488, 1.3572405786990294, 1.3843853902730099, 1.4115302018469906, 1.438675013420971, 1.4658198249949517, 1.4929646365689322, 1.520109448142913, 1.5472542597168935, 1.574399071290874, 1.6015438828648547, 1.628688694438835, 1.6558335060128158, 1.6829783175867963, 1.710123129160777, 1.7372679407347575, 1.764412752308738, 1.7915575638827188, 1.818702375456699, 1.84584718703068, 1.8729919986046604, 1.9001368101786411, 1.9272816217526216, 1.9544264333266024, 1.981571244900583, 2.0087160564745634, 2.035860868048544, 2.0630056796225245, 2.090150491196505, 2.1172953027704855, 2.1444401143444667, 2.171584925918447, 2.1987297374924273, 2.225874549066408, 2.2530193606403883, 2.2801641722143695, 2.30730898378835, 2.3344537953623306, 2.361598606936311, 2.3887434185102916, 2.4158882300842723, 2.4430330416582526, 2.4701778532322334, 2.4973226648062137, 2.524467476380195, 2.551612287954175, 2.578757099528156, 2.605901911102136, 2.6330467226761165, 2.6601915342500977, 2.687336345824078, 2.7144811573980587 ], [ 0, 0.02789329457664754, 0.05578658915329508, 0.08367988372994263, 0.11157317830659016, 0.1394664728832377, 0.16735976745988526, 0.1952530620365328, 0.22314635661318033, 0.2510396511898279, 0.2789329457664754, 0.30682624034312295, 0.3347195349197705, 0.3626128294964181, 0.3905061240730656, 0.41839941864971314, 0.44629271322636066, 0.47418600780300824, 0.5020793023796558, 0.5299725969563033, 0.5578658915329509, 0.5857591861095984, 0.6136524806862459, 0.6415457752628935, 0.669439069839541, 0.6973323644161886, 0.7252256589928362, 0.7531189535694837, 0.7810122481461312, 0.8089055427227788, 0.8367988372994263, 0.8646921318760737, 0.8925854264527213, 0.9204787210293689, 0.9483720156060165, 0.976265310182664, 1.0041586047593116, 1.032051899335959, 1.0599451939126066, 1.0878384884892542, 1.1157317830659017, 1.1436250776425492, 1.1715183722191969, 1.1994116667958443, 1.2273049613724918, 1.2551982559491393, 1.283091550525787, 1.3109848451024344, 1.338878139679082, 1.3667714342557296, 1.3946647288323772, 1.4225580234090247, 1.4504513179856724, 1.4783446125623196, 1.5062379071389673, 1.5341312017156148, 1.5620244962922625, 1.58991779086891, 1.6178110854455576, 1.645704380022205, 1.6735976745988526, 1.7014909691755, 1.7293842637521475, 1.7572775583287952, 1.7851708529054426, 1.8130641474820903, 1.8409574420587378, 1.8688507366353853, 1.896744031212033, 1.9246373257886804, 1.952530620365328, 1.9804239149419756, 2.0083172095186232, 2.0362105040952705, 2.064103798671918, 2.091997093248566, 2.119890387825213, 2.147783682401861, 2.1756769769785085, 2.203570271555156, 2.2314635661318034, 2.2593568607084507, 2.2872501552850983, 2.315143449861746, 2.3430367444383937, 2.370930039015041, 2.3988233335916886, 2.4267166281683363, 2.4546099227449836, 2.4825032173216313, 2.5103965118982785, 2.5382898064749266, 2.566183101051574, 2.5940763956282216, 2.621969690204869, 2.649862984781517, 2.677756279358164, 2.7056495739348114, 2.733542868511459, 2.761436163088107, 2.7893294576647545 ], [ 0, 0.028531695488854605, 0.05706339097770921, 0.08559508646656382, 0.11412678195541842, 0.142658477444273, 0.17119017293312763, 0.19972186842198222, 0.22825356391083684, 0.2567852593996915, 0.285316954888546, 0.3138486503774006, 0.34238034586625526, 0.3709120413551099, 0.39944373684396445, 0.42797543233281904, 0.4565071278216737, 0.48503882331052833, 0.513570518799383, 0.5421022142882375, 0.570633909777092, 0.5991656052659468, 0.6276973007548012, 0.6562289962436559, 0.6847606917325105, 0.7132923872213651, 0.7418240827102198, 0.7703557781990743, 0.7988874736879289, 0.8274191691767836, 0.8559508646656381, 0.8844825601544927, 0.9130142556433474, 0.941545951132202, 0.9700776466210567, 0.9986093421099113, 1.027141037598766, 1.0556727330876203, 1.084204428576475, 1.1127361240653295, 1.141267819554184, 1.169799515043039, 1.1983312105318935, 1.226862906020748, 1.2553946015096025, 1.283926296998457, 1.3124579924873119, 1.3409896879761665, 1.369521383465021, 1.3980530789538756, 1.4265847744427302, 1.4551164699315848, 1.4836481654204396, 1.512179860909294, 1.5407115563981486, 1.5692432518870032, 1.5977749473758578, 1.6263066428647126, 1.6548383383535672, 1.6833700338424218, 1.7119017293312762, 1.7404334248201307, 1.7689651203089853, 1.7974968157978402, 1.8260285112866947, 1.8545602067755493, 1.883091902264404, 1.9116235977532583, 1.9401552932421133, 1.9686869887309677, 1.9972186842198225, 2.025750379708677, 2.054282075197532, 2.082813770686386, 2.1113454661752407, 2.1398771616640953, 2.16840885715295, 2.196940552641805, 2.225472248130659, 2.254003943619514, 2.282535639108368, 2.311067334597223, 2.339599030086078, 2.368130725574932, 2.396662421063787, 2.425194116552641, 2.453725812041496, 2.482257507530351, 2.510789203019205, 2.53932089850806, 2.567852593996914, 2.596384289485769, 2.6249159849746238, 2.6534476804634783, 2.681979375952333, 2.7105110714411875, 2.739042766930042, 2.7675744624188967, 2.7961061579077513, 2.824637853396606, 2.8531695488854605 ], [ 0, 0.02905749483385893, 0.05811498966771786, 0.08717248450157679, 0.11622997933543572, 0.14528747416929466, 0.17434496900315358, 0.20340246383701252, 0.23245995867087144, 0.26151745350473043, 0.2905749483385893, 0.3196324431724482, 0.34868993800630715, 0.37774743284016615, 0.40680492767402504, 0.43586242250788393, 0.46491991734174287, 0.49397741217560187, 0.5230349070094609, 0.5520924018433196, 0.5811498966771786, 0.6102073915110375, 0.6392648863448964, 0.6683223811787554, 0.6973798760126143, 0.7264373708464733, 0.7554948656803323, 0.7845523605141911, 0.8136098553480501, 0.8426673501819091, 0.8717248450157679, 0.9007823398496269, 0.9298398346834857, 0.9588973295173447, 0.9879548243512037, 1.0170123191850626, 1.0460698140189217, 1.0751273088527804, 1.1041848036866393, 1.1332422985204982, 1.1622997933543573, 1.1913572881882162, 1.220414783022075, 1.2494722778559342, 1.2785297726897928, 1.3075872675236517, 1.3366447623575108, 1.3657022571913697, 1.3947597520252286, 1.4238172468590877, 1.4528747416929466, 1.4819322365268055, 1.5109897313606646, 1.5400472261945233, 1.5691047210283822, 1.5981622158622413, 1.6272197106961002, 1.656277205529959, 1.6853347003638182, 1.714392195197677, 1.7434496900315357, 1.7725071848653948, 1.8015646796992537, 1.8306221745331126, 1.8596796693669715, 1.8887371642008306, 1.9177946590346895, 1.9468521538685482, 1.9759096487024075, 2.004967143536266, 2.0340246383701253, 2.063082133203984, 2.0921396280378435, 2.121197122871702, 2.150254617705561, 2.17931211253942, 2.2083696073732786, 2.2374271022071377, 2.2664845970409964, 2.295542091874856, 2.3245995867087146, 2.3536570815425732, 2.3827145763764324, 2.411772071210291, 2.44082956604415, 2.4698870608780092, 2.4989445557118684, 2.528002050545727, 2.5570595453795857, 2.586117040213445, 2.6151745350473035, 2.644232029881163, 2.6732895247150217, 2.702347019548881, 2.7314045143827395, 2.7604620092165986, 2.7895195040504572, 2.8185769988843163, 2.8476344937181755, 2.876691988552034, 2.9057494833858932 ], [ 0, 0.02946861752186066, 0.05893723504372132, 0.08840585256558198, 0.11787447008744265, 0.1473430876093033, 0.17681170513116395, 0.20628032265302462, 0.2357489401748853, 0.265217557696746, 0.2946861752186066, 0.3241547927404672, 0.3536234102623279, 0.3830920277841886, 0.41256064530604925, 0.4420292628279099, 0.4714978803497706, 0.5009664978716313, 0.530435115393492, 0.5599037329153526, 0.5893723504372133, 0.618840967959074, 0.6483095854809344, 0.6777782030027951, 0.7072468205246558, 0.7367154380465165, 0.7661840555683772, 0.7956526730902378, 0.8251212906120985, 0.8545899081339592, 0.8840585256558198, 0.9135271431776805, 0.9429957606995412, 0.9724643782214019, 1.0019329957432626, 1.0314016132651231, 1.060870230786984, 1.0903388483088443, 1.1198074658307051, 1.1492760833525657, 1.1787447008744265, 1.208213318396287, 1.237681935918148, 1.2671505534400085, 1.2966191709618688, 1.3260877884837297, 1.3555564060055902, 1.385025023527451, 1.4144936410493116, 1.4439622585711724, 1.473430876093033, 1.5028994936148938, 1.5323681111367544, 1.561836728658615, 1.5913053461804756, 1.6207739637023364, 1.650242581224197, 1.6797111987460578, 1.7091798162679184, 1.738648433789779, 1.7681170513116395, 1.7975856688335001, 1.827054286355361, 1.8565229038772215, 1.8859915213990823, 1.915460138920943, 1.9449287564428037, 1.974397373964664, 2.003865991486525, 2.0333346090083855, 2.0628032265302463, 2.092271844052107, 2.121740461573968, 2.1512090790958283, 2.1806776966176886, 2.2101463141395494, 2.2396149316614102, 2.269083549183271, 2.2985521667051314, 2.328020784226992, 2.357489401748853, 2.3869580192707134, 2.416426636792574, 2.4458952543144346, 2.475363871836296, 2.504832489358156, 2.534301106880017, 2.5637697244018773, 2.5932383419237377, 2.622706959445599, 2.6521755769674593, 2.68164419448932, 2.7111128120111805, 2.7405814295330417, 2.770050047054902, 2.799518664576763, 2.8289872820986233, 2.858455899620484, 2.887924517142345, 2.9173931346642052, 2.946861752186066 ], [ 0, 0.029763441039434336, 0.05952688207886867, 0.08929032311830301, 0.11905376415773734, 0.14881720519717168, 0.17858064623660602, 0.20834408727604034, 0.2381075283154747, 0.26787096935490906, 0.29763441039434335, 0.32739785143377764, 0.35716129247321204, 0.3869247335126464, 0.4166881745520807, 0.446451615591515, 0.4762150566309494, 0.5059784976703837, 0.5357419387098181, 0.5655053797492523, 0.5952688207886867, 0.6250322618281211, 0.6547957028675553, 0.6845591439069897, 0.7143225849464241, 0.7440860259858584, 0.7738494670252928, 0.8036129080647271, 0.8333763491041614, 0.8631397901435958, 0.89290323118303, 0.9226666722224643, 0.9524301132618987, 0.982193554301333, 1.0119569953407674, 1.0417204363802017, 1.0714838774196362, 1.1012473184590703, 1.1310107594985046, 1.1607742005379391, 1.1905376415773734, 1.2203010826168077, 1.2500645236562422, 1.2798279646956765, 1.3095914057351106, 1.339354846774545, 1.3691182878139794, 1.3988817288534137, 1.4286451698928482, 1.4584086109322825, 1.4881720519717168, 1.5179354930111513, 1.5476989340505856, 1.5774623750900196, 1.6072258161294541, 1.6369892571688884, 1.6667526982083227, 1.6965161392477572, 1.7262795802871915, 1.7560430213266258, 1.78580646236606, 1.8155699034054944, 1.8453333444449287, 1.875096785484363, 1.9048602265237975, 1.9346236675632318, 1.964387108602666, 1.9941505496421004, 2.023913990681535, 2.053677431720969, 2.0834408727604035, 2.1132043137998378, 2.1429677548392725, 2.1727311958787063, 2.2024946369181406, 2.2322580779575754, 2.262021518997009, 2.291784960036444, 2.3215484010758782, 2.3513118421153125, 2.381075283154747, 2.410838724194181, 2.4406021652336154, 2.4703656062730497, 2.5001290473124844, 2.5298924883519183, 2.559655929391353, 2.5894193704307873, 2.619182811470221, 2.648946252509656, 2.67870969354909, 2.7084731345885245, 2.7382365756279587, 2.7680000166673935, 2.7977634577068273, 2.827526898746262, 2.8572903397856964, 2.88705378082513, 2.916817221864565, 2.9465806629039992, 2.9763441039434335 ], [ 0, 0.029940801852848146, 0.05988160370569629, 0.08982240555854444, 0.11976320741139258, 0.14970400926424074, 0.17964481111708888, 0.20958561296993702, 0.23952641482278517, 0.26946721667563334, 0.2994080185284815, 0.32934882038132957, 0.35928962223417776, 0.3892304240870259, 0.41917122593987405, 0.44911202779272213, 0.47905282964557033, 0.5089936314984185, 0.5389344333512667, 0.5688752352041148, 0.598816037056963, 0.628756838909811, 0.6586976407626591, 0.6886384426155073, 0.7185792444683555, 0.7485200463212036, 0.7784608481740518, 0.8084016500268999, 0.8383424518797481, 0.8682832537325963, 0.8982240555854443, 0.9281648574382925, 0.9581056592911407, 0.9880464611439889, 1.017987262996837, 1.0479280648496851, 1.0778688667025333, 1.1078096685553813, 1.1377504704082295, 1.1676912722610777, 1.197632074113926, 1.227572875966774, 1.257513677819622, 1.2874544796724703, 1.3173952815253183, 1.3473360833781665, 1.3772768852310147, 1.4072176870838629, 1.437158488936711, 1.4670992907895593, 1.4970400926424072, 1.5269808944952554, 1.5569216963481036, 1.5868624982009516, 1.6168033000537998, 1.646744101906648, 1.6766849037594962, 1.7066257056123444, 1.7365665074651926, 1.7665073093180408, 1.7964481111708885, 1.8263889130237367, 1.856329714876585, 1.8862705167294331, 1.9162113185822813, 1.9461521204351295, 1.9760929222879777, 2.0060337241408255, 2.035974525993674, 2.065915327846522, 2.0958561296993703, 2.1257969315522183, 2.1557377334050667, 2.1856785352579147, 2.2156193371107626, 2.245560138963611, 2.275500940816459, 2.3054417426693075, 2.3353825445221554, 2.365323346375004, 2.395264148227852, 2.4252049500807, 2.455145751933548, 2.4850865537863958, 2.515027355639244, 2.544968157492092, 2.5749089593449406, 2.6048497611977885, 2.6347905630506365, 2.664731364903485, 2.694672166756333, 2.7246129686091813, 2.7545537704620293, 2.7844945723148777, 2.8144353741677257, 2.844376176020574, 2.874316977873422, 2.90425777972627, 2.9341985815791185, 2.9641393834319665, 2.9940801852848145 ], [ 0, 0.03, 0.06, 0.09, 0.12, 0.15, 0.18, 0.21, 0.24, 0.27, 0.3, 0.32999999999999996, 0.36, 0.39, 0.42, 0.44999999999999996, 0.48, 0.51, 0.54, 0.57, 0.6, 0.63, 0.6599999999999999, 0.69, 0.72, 0.75, 0.78, 0.8099999999999999, 0.84, 0.87, 0.8999999999999999, 0.9299999999999999, 0.96, 0.99, 1.02, 1.05, 1.08, 1.1099999999999999, 1.14, 1.17, 1.2, 1.23, 1.26, 1.29, 1.3199999999999998, 1.3499999999999999, 1.38, 1.41, 1.44, 1.47, 1.5, 1.53, 1.56, 1.5899999999999999, 1.6199999999999999, 1.65, 1.68, 1.71, 1.74, 1.77, 1.7999999999999998, 1.8299999999999998, 1.8599999999999999, 1.89, 1.92, 1.95, 1.98, 2.01, 2.04, 2.07, 2.1, 2.13, 2.16, 2.19, 2.2199999999999998, 2.25, 2.28, 2.31, 2.34, 2.37, 2.4, 2.4299999999999997, 2.46, 2.4899999999999998, 2.52, 2.55, 2.58, 2.61, 2.6399999999999997, 2.67, 2.6999999999999997, 2.73, 2.76, 2.79, 2.82, 2.85, 2.88, 2.9099999999999997, 2.94, 2.9699999999999998, 3 ], [ 0, 0.029940801852848146, 0.05988160370569629, 0.08982240555854444, 0.11976320741139258, 0.14970400926424074, 0.17964481111708888, 0.20958561296993702, 0.23952641482278517, 0.26946721667563334, 0.2994080185284815, 0.32934882038132957, 0.35928962223417776, 0.3892304240870259, 0.41917122593987405, 0.44911202779272213, 0.47905282964557033, 0.5089936314984185, 0.5389344333512667, 0.5688752352041148, 0.598816037056963, 0.628756838909811, 0.6586976407626591, 0.6886384426155073, 0.7185792444683555, 0.7485200463212036, 0.7784608481740518, 0.8084016500268999, 0.8383424518797481, 0.8682832537325963, 0.8982240555854443, 0.9281648574382925, 0.9581056592911407, 0.9880464611439889, 1.017987262996837, 1.0479280648496851, 1.0778688667025333, 1.1078096685553813, 1.1377504704082295, 1.1676912722610777, 1.197632074113926, 1.227572875966774, 1.257513677819622, 1.2874544796724703, 1.3173952815253183, 1.3473360833781665, 1.3772768852310147, 1.4072176870838629, 1.437158488936711, 1.4670992907895593, 1.4970400926424072, 1.5269808944952554, 1.5569216963481036, 1.5868624982009516, 1.6168033000537998, 1.646744101906648, 1.6766849037594962, 1.7066257056123444, 1.7365665074651926, 1.7665073093180408, 1.7964481111708885, 1.8263889130237367, 1.856329714876585, 1.8862705167294331, 1.9162113185822813, 1.9461521204351295, 1.9760929222879777, 2.0060337241408255, 2.035974525993674, 2.065915327846522, 2.0958561296993703, 2.1257969315522183, 2.1557377334050667, 2.1856785352579147, 2.2156193371107626, 2.245560138963611, 2.275500940816459, 2.3054417426693075, 2.3353825445221554, 2.365323346375004, 2.395264148227852, 2.4252049500807, 2.455145751933548, 2.4850865537863958, 2.515027355639244, 2.544968157492092, 2.5749089593449406, 2.6048497611977885, 2.6347905630506365, 2.664731364903485, 2.694672166756333, 2.7246129686091813, 2.7545537704620293, 2.7844945723148777, 2.8144353741677257, 2.844376176020574, 2.874316977873422, 2.90425777972627, 2.9341985815791185, 2.9641393834319665, 2.9940801852848145 ], [ 0, 0.029763441039434332, 0.059526882078868665, 0.089290323118303, 0.11905376415773733, 0.14881720519717165, 0.178580646236606, 0.2083440872760403, 0.23810752831547466, 0.267870969354909, 0.2976344103943433, 0.32739785143377764, 0.357161292473212, 0.38692473351264633, 0.4166881745520806, 0.44645161559151497, 0.4762150566309493, 0.5059784976703837, 0.535741938709818, 0.5655053797492523, 0.5952688207886866, 0.625032261828121, 0.6547957028675553, 0.6845591439069896, 0.714322584946424, 0.7440860259858584, 0.7738494670252927, 0.803612908064727, 0.8333763491041613, 0.8631397901435957, 0.8929032311830299, 0.9226666722224642, 0.9524301132618986, 0.9821935543013329, 1.0119569953407674, 1.0417204363802017, 1.071483877419636, 1.10124731845907, 1.1310107594985046, 1.160774200537939, 1.1905376415773732, 1.2203010826168077, 1.250064523656242, 1.2798279646956763, 1.3095914057351106, 1.3393548467745449, 1.3691182878139792, 1.3988817288534137, 1.428645169892848, 1.4584086109322822, 1.4881720519717168, 1.517935493011151, 1.5476989340505853, 1.5774623750900194, 1.607225816129454, 1.6369892571688882, 1.6667526982083225, 1.696516139247757, 1.7262795802871913, 1.7560430213266256, 1.7858064623660599, 1.8155699034054942, 1.8453333444449285, 1.875096785484363, 1.9048602265237973, 1.9346236675632316, 1.9643871086026659, 1.9941505496421001, 2.023913990681535, 2.0536774317209687, 2.0834408727604035, 2.1132043137998378, 2.142967754839272, 2.1727311958787063, 2.20249463691814, 2.232258077957575, 2.262021518997009, 2.2917849600364435, 2.321548401075878, 2.3513118421153125, 2.3810752831547464, 2.4108387241941807, 2.4406021652336154, 2.4703656062730492, 2.500129047312484, 2.5298924883519183, 2.5596559293913526, 2.589419370430787, 2.619182811470221, 2.6489462525096554, 2.6787096935490897, 2.7084731345885245, 2.7382365756279583, 2.768000016667393, 2.7977634577068273, 2.8275268987462616, 2.857290339785696, 2.88705378082513, 2.9168172218645645, 2.946580662903999, 2.9763441039434335 ], [ 0, 0.029468617521860658, 0.058937235043721316, 0.08840585256558198, 0.11787447008744263, 0.14734308760930329, 0.17681170513116395, 0.2062803226530246, 0.23574894017488526, 0.26521755769674593, 0.29468617521860657, 0.3241547927404672, 0.3536234102623279, 0.38309202778418855, 0.4125606453060492, 0.44202926282790983, 0.4714978803497705, 0.5009664978716312, 0.5304351153934919, 0.5599037329153524, 0.5893723504372131, 0.6188409679590738, 0.6483095854809344, 0.6777782030027951, 0.7072468205246558, 0.7367154380465164, 0.7661840555683771, 0.7956526730902377, 0.8251212906120984, 0.8545899081339591, 0.8840585256558197, 0.9135271431776804, 0.942995760699541, 0.9724643782214017, 1.0019329957432623, 1.0314016132651231, 1.0608702307869837, 1.0903388483088443, 1.119807465830705, 1.1492760833525657, 1.1787447008744263, 1.2082133183962869, 1.2376819359181477, 1.2671505534400083, 1.2966191709618688, 1.3260877884837294, 1.3555564060055902, 1.3850250235274508, 1.4144936410493116, 1.4439622585711722, 1.4734308760930328, 1.5028994936148936, 1.5323681111367542, 1.5618367286586148, 1.5913053461804754, 1.6207739637023362, 1.6502425812241968, 1.6797111987460576, 1.7091798162679182, 1.738648433789779, 1.7681170513116393, 1.7975856688335, 1.8270542863553607, 1.8565229038772213, 1.885991521399082, 1.9154601389209427, 1.9449287564428035, 1.9743973739646639, 2.0038659914865247, 2.0333346090083855, 2.0628032265302463, 2.0922718440521066, 2.1217404615739675, 2.151209079095828, 2.1806776966176886, 2.2101463141395494, 2.23961493166141, 2.2690835491832706, 2.2985521667051314, 2.328020784226992, 2.3574894017488526, 2.386958019270713, 2.4164266367925737, 2.4458952543144346, 2.4753638718362954, 2.5048324893581557, 2.5343011068800165, 2.5637697244018773, 2.5932383419237377, 2.6227069594455985, 2.652175576967459, 2.6816441944893197, 2.7111128120111805, 2.7405814295330413, 2.7700500470549017, 2.7995186645767625, 2.8289872820986233, 2.8584558996204836, 2.8879245171423444, 2.917393134664205, 2.9468617521860656 ], [ 0, 0.02905749483385893, 0.05811498966771786, 0.08717248450157679, 0.11622997933543572, 0.14528747416929466, 0.17434496900315358, 0.20340246383701252, 0.23245995867087144, 0.26151745350473043, 0.2905749483385893, 0.3196324431724482, 0.34868993800630715, 0.37774743284016615, 0.40680492767402504, 0.43586242250788393, 0.46491991734174287, 0.49397741217560187, 0.5230349070094609, 0.5520924018433196, 0.5811498966771786, 0.6102073915110375, 0.6392648863448964, 0.6683223811787554, 0.6973798760126143, 0.7264373708464733, 0.7554948656803323, 0.7845523605141911, 0.8136098553480501, 0.8426673501819091, 0.8717248450157679, 0.9007823398496269, 0.9298398346834857, 0.9588973295173447, 0.9879548243512037, 1.0170123191850626, 1.0460698140189217, 1.0751273088527804, 1.1041848036866393, 1.1332422985204982, 1.1622997933543573, 1.1913572881882162, 1.220414783022075, 1.2494722778559342, 1.2785297726897928, 1.3075872675236517, 1.3366447623575108, 1.3657022571913697, 1.3947597520252286, 1.4238172468590877, 1.4528747416929466, 1.4819322365268055, 1.5109897313606646, 1.5400472261945233, 1.5691047210283822, 1.5981622158622413, 1.6272197106961002, 1.656277205529959, 1.6853347003638182, 1.714392195197677, 1.7434496900315357, 1.7725071848653948, 1.8015646796992537, 1.8306221745331126, 1.8596796693669715, 1.8887371642008306, 1.9177946590346895, 1.9468521538685482, 1.9759096487024075, 2.004967143536266, 2.0340246383701253, 2.063082133203984, 2.0921396280378435, 2.121197122871702, 2.150254617705561, 2.17931211253942, 2.2083696073732786, 2.2374271022071377, 2.2664845970409964, 2.295542091874856, 2.3245995867087146, 2.3536570815425732, 2.3827145763764324, 2.411772071210291, 2.44082956604415, 2.4698870608780092, 2.4989445557118684, 2.528002050545727, 2.5570595453795857, 2.586117040213445, 2.6151745350473035, 2.644232029881163, 2.6732895247150217, 2.702347019548881, 2.7314045143827395, 2.7604620092165986, 2.7895195040504572, 2.8185769988843163, 2.8476344937181755, 2.876691988552034, 2.9057494833858932 ], [ 0, 0.028531695488854605, 0.05706339097770921, 0.08559508646656382, 0.11412678195541842, 0.142658477444273, 0.17119017293312763, 0.19972186842198222, 0.22825356391083684, 0.2567852593996915, 0.285316954888546, 0.3138486503774006, 0.34238034586625526, 0.3709120413551099, 0.39944373684396445, 0.42797543233281904, 0.4565071278216737, 0.48503882331052833, 0.513570518799383, 0.5421022142882375, 0.570633909777092, 0.5991656052659468, 0.6276973007548012, 0.6562289962436559, 0.6847606917325105, 0.7132923872213651, 0.7418240827102198, 0.7703557781990743, 0.7988874736879289, 0.8274191691767836, 0.8559508646656381, 0.8844825601544927, 0.9130142556433474, 0.941545951132202, 0.9700776466210567, 0.9986093421099113, 1.027141037598766, 1.0556727330876203, 1.084204428576475, 1.1127361240653295, 1.141267819554184, 1.169799515043039, 1.1983312105318935, 1.226862906020748, 1.2553946015096025, 1.283926296998457, 1.3124579924873119, 1.3409896879761665, 1.369521383465021, 1.3980530789538756, 1.4265847744427302, 1.4551164699315848, 1.4836481654204396, 1.512179860909294, 1.5407115563981486, 1.5692432518870032, 1.5977749473758578, 1.6263066428647126, 1.6548383383535672, 1.6833700338424218, 1.7119017293312762, 1.7404334248201307, 1.7689651203089853, 1.7974968157978402, 1.8260285112866947, 1.8545602067755493, 1.883091902264404, 1.9116235977532583, 1.9401552932421133, 1.9686869887309677, 1.9972186842198225, 2.025750379708677, 2.054282075197532, 2.082813770686386, 2.1113454661752407, 2.1398771616640953, 2.16840885715295, 2.196940552641805, 2.225472248130659, 2.254003943619514, 2.282535639108368, 2.311067334597223, 2.339599030086078, 2.368130725574932, 2.396662421063787, 2.425194116552641, 2.453725812041496, 2.482257507530351, 2.510789203019205, 2.53932089850806, 2.567852593996914, 2.596384289485769, 2.6249159849746238, 2.6534476804634783, 2.681979375952333, 2.7105110714411875, 2.739042766930042, 2.7675744624188967, 2.7961061579077513, 2.824637853396606, 2.8531695488854605 ], [ 0, 0.027893294576647538, 0.055786589153295076, 0.08367988372994262, 0.11157317830659015, 0.13946647288323769, 0.16735976745988523, 0.19525306203653278, 0.2231463566131803, 0.2510396511898279, 0.27893294576647537, 0.3068262403431229, 0.33471953491977047, 0.36261282949641804, 0.39050612407306556, 0.4183994186497131, 0.4462927132263606, 0.4741860078030082, 0.5020793023796558, 0.5299725969563033, 0.5578658915329507, 0.5857591861095983, 0.6136524806862458, 0.6415457752628934, 0.6694390698395409, 0.6973323644161885, 0.7252256589928361, 0.7531189535694836, 0.7810122481461311, 0.8089055427227787, 0.8367988372994262, 0.8646921318760737, 0.8925854264527212, 0.9204787210293688, 0.9483720156060164, 0.9762653101826639, 1.0041586047593116, 1.0320518993359589, 1.0599451939126066, 1.087838488489254, 1.1157317830659015, 1.1436250776425492, 1.1715183722191966, 1.1994116667958443, 1.2273049613724916, 1.2551982559491393, 1.2830915505257867, 1.3109848451024344, 1.3388781396790819, 1.3667714342557296, 1.394664728832377, 1.4225580234090245, 1.4504513179856722, 1.4783446125623194, 1.506237907138967, 1.5341312017156146, 1.5620244962922623, 1.5899177908689097, 1.6178110854455574, 1.6457043800222049, 1.6735976745988523, 1.7014909691754998, 1.7293842637521475, 1.757277558328795, 1.7851708529054424, 1.81306414748209, 1.8409574420587376, 1.868850736635385, 1.8967440312120327, 1.9246373257886802, 1.9525306203653279, 1.9804239149419753, 2.0083172095186232, 2.0362105040952705, 2.0641037986719177, 2.0919970932485654, 2.119890387825213, 2.147783682401861, 2.175676976978508, 2.2035702715551557, 2.231463566131803, 2.2593568607084507, 2.2872501552850983, 2.3151434498617456, 2.3430367444383933, 2.370930039015041, 2.3988233335916886, 2.426716628168336, 2.454609922744983, 2.482503217321631, 2.5103965118982785, 2.538289806474926, 2.5661831010515734, 2.594076395628221, 2.621969690204869, 2.6498629847815165, 2.6777562793581637, 2.705649573934811, 2.733542868511459, 2.7614361630881064, 2.789329457664754 ], [ 0, 0.027144811573980583, 0.054289623147961166, 0.08143443472194174, 0.10857924629592233, 0.1357240578699029, 0.16286886944388348, 0.1900136810178641, 0.21715849259184467, 0.24430330416582527, 0.2714481157398058, 0.2985929273137864, 0.32573773888776697, 0.3528825504617476, 0.3800273620357282, 0.4071721736097087, 0.43431698518368933, 0.46146179675766996, 0.48860660833165054, 0.515751419905631, 0.5428962314796116, 0.5700410430535923, 0.5971858546275728, 0.6243306662015534, 0.6514754777755339, 0.6786202893495146, 0.7057651009234952, 0.7329099124974757, 0.7600547240714564, 0.7871995356454369, 0.8143443472194174, 0.841489158793398, 0.8686339703673787, 0.8957787819413593, 0.9229235935153399, 0.9500684050893204, 0.9772132166633011, 1.0043580282372815, 1.031502839811262, 1.0586476513852427, 1.0857924629592233, 1.112937274533204, 1.1400820861071845, 1.167226897681165, 1.1943717092551456, 1.221516520829126, 1.2486613324031068, 1.2758061439770874, 1.3029509555510679, 1.3300957671250486, 1.3572405786990291, 1.3843853902730099, 1.4115302018469904, 1.438675013420971, 1.4658198249949514, 1.492964636568932, 1.5201094481429127, 1.5472542597168932, 1.5743990712908738, 1.6015438828648545, 1.6286886944388348, 1.6558335060128155, 1.682978317586796, 1.7101231291607768, 1.7372679407347573, 1.7644127523087378, 1.7915575638827186, 1.8187023754566989, 1.8458471870306798, 1.8729919986046601, 1.900136810178641, 1.9272816217526214, 1.9544264333266022, 1.9815712449005827, 2.008716056474563, 2.0358608680485437, 2.063005679622524, 2.090150491196505, 2.1172953027704855, 2.1444401143444662, 2.1715849259184465, 2.198729737492427, 2.225874549066408, 2.2530193606403883, 2.280164172214369, 2.3073089837883494, 2.33445379536233, 2.361598606936311, 2.388743418510291, 2.415888230084272, 2.443033041658252, 2.470177853232233, 2.4973226648062137, 2.5244674763801944, 2.5516122879541747, 2.5787570995281555, 2.6059019111021358, 2.6330467226761165, 2.6601915342500972, 2.6873363458240775, 2.7144811573980583 ], [ 0, 0.026289200401315903, 0.052578400802631806, 0.07886760120394772, 0.10515680160526361, 0.13144600200657952, 0.15773520240789543, 0.1840244028092113, 0.21031360321052722, 0.23660280361184316, 0.26289200401315904, 0.2891812044144749, 0.31547040481579086, 0.34175960521710674, 0.3680488056184226, 0.3943380060197385, 0.42062720642105444, 0.4469164068223704, 0.4732056072236863, 0.49949480762500215, 0.5257840080263181, 0.552073208427634, 0.5783624088289498, 0.6046516092302657, 0.6309408096315817, 0.6572300100328976, 0.6835192104342135, 0.7098084108355294, 0.7360976112368453, 0.7623868116381612, 0.788676012039477, 0.814965212440793, 0.8412544128421089, 0.8675436132434249, 0.8938328136447408, 0.9201220140460566, 0.9464112144473726, 0.9727004148486883, 0.9989896152500043, 1.0252788156513202, 1.0515680160526362, 1.0778572164539522, 1.104146416855268, 1.130435617256584, 1.1567248176578997, 1.1830140180592155, 1.2093032184605315, 1.2355924188618475, 1.2618816192631634, 1.2881708196644792, 1.3144600200657952, 1.3407492204671112, 1.367038420868427, 1.3933276212697427, 1.4196168216710587, 1.4459060220723747, 1.4721952224736905, 1.4984844228750065, 1.5247736232763225, 1.5510628236776383, 1.577352024078954, 1.60364122448027, 1.629930424881586, 1.6562196252829018, 1.6825088256842178, 1.7087980260855338, 1.7350872264868498, 1.7613764268881653, 1.7876656272894815, 1.8139548276907973, 1.8402440280921133, 1.866533228493429, 1.8928224288947453, 1.919111629296061, 1.9454008296973766, 1.9716900300986928, 1.9979792305000086, 2.0242684309013246, 2.0505576313026403, 2.0768468317039566, 2.1031360321052723, 2.129425232506588, 2.1557144329079043, 2.1820036333092196, 2.208292833710536, 2.2345820341118516, 2.260871234513168, 2.2871604349144836, 2.3134496353157994, 2.3397388357171156, 2.366028036118431, 2.392317236519747, 2.418606436921063, 2.444895637322379, 2.471184837723695, 2.497474038125011, 2.523763238526327, 2.5500524389276427, 2.5763416393289584, 2.602630839730274, 2.6289200401315904 ], [ 0, 0.025329837765060446, 0.05065967553012089, 0.07598951329518135, 0.10131935106024179, 0.12664918882530224, 0.1519790265903627, 0.17730886435542315, 0.20263870212048357, 0.22796853988554405, 0.2532983776506045, 0.2786282154156649, 0.3039580531807254, 0.32928789094578587, 0.3546177287108463, 0.3799475664759067, 0.40527740424096714, 0.4306072420060276, 0.4559370797710881, 0.4812669175361485, 0.506596755301209, 0.5319265930662694, 0.5572564308313298, 0.5825862685963903, 0.6079161063614508, 0.6332459441265113, 0.6585757818915717, 0.6839056196566321, 0.7092354574216926, 0.7345652951867531, 0.7598951329518134, 0.7852249707168739, 0.8105548084819343, 0.8358846462469948, 0.8612144840120552, 0.8865443217771157, 0.9118741595421762, 0.9372039973072365, 0.962533835072297, 0.9878636728373574, 1.013193510602418, 1.0385233483674783, 1.0638531861325389, 1.0891830238975992, 1.1145128616626596, 1.13984269942772, 1.1651725371927806, 1.190502374957841, 1.2158322127229015, 1.241162050487962, 1.2664918882530225, 1.2918217260180829, 1.3171515637831435, 1.3424814015482036, 1.3678112393132642, 1.3931410770783246, 1.4184709148433852, 1.4438007526084455, 1.4691305903735061, 1.4944604281385665, 1.5197902659036269, 1.5451201036686872, 1.5704499414337478, 1.5957797791988082, 1.6211096169638686, 1.6464394547289292, 1.6717692924939895, 1.69709913025905, 1.7224289680241105, 1.7477588057891709, 1.7730886435542315, 1.7984184813192918, 1.8237483190843524, 1.8490781568494128, 1.874407994614473, 1.8997378323795338, 1.925067670144594, 1.9503975079096547, 1.9757273456747149, 2.0010571834397757, 2.026387021204836, 2.051716858969896, 2.0770466967349566, 2.102376534500017, 2.1277063722650777, 2.153036210030138, 2.1783660477951985, 2.203695885560259, 2.2290257233253192, 2.25435556109038, 2.27968539885544, 2.305015236620501, 2.330345074385561, 2.3556749121506217, 2.381004749915682, 2.406334587680743, 2.431664425445803, 2.4569942632108632, 2.482324100975924, 2.5076539387409844, 2.532983776506045 ], [ 0, 0.024270509831248423, 0.04854101966249685, 0.07281152949374527, 0.0970820393249937, 0.1213525491562421, 0.14562305898749053, 0.16989356881873896, 0.1941640786499874, 0.21843458848123581, 0.2427050983124842, 0.2669756081437326, 0.29124611797498107, 0.3155166278062295, 0.3397871376374779, 0.3640576474687263, 0.3883281572999748, 0.41259866713122323, 0.43686917696247163, 0.46113968679372, 0.4854101966249684, 0.5096807064562169, 0.5339512162874652, 0.5582217261187137, 0.5824922359499621, 0.6067627457812106, 0.631033255612459, 0.6553037654437074, 0.6795742752749558, 0.7038447851062043, 0.7281152949374526, 0.7523858047687011, 0.7766563145999495, 0.800926824431198, 0.8251973342624465, 0.8494678440936949, 0.8737383539249433, 0.8980088637561916, 0.92227937358744, 0.9465498834186885, 0.9708203932499369, 0.9950909030811853, 1.0193614129124338, 1.0436319227436823, 1.0679024325749304, 1.092172942406179, 1.1164434522374274, 1.140713962068676, 1.1649844718999243, 1.1892549817311728, 1.2135254915624212, 1.2377960013936695, 1.262066511224918, 1.2863370210561664, 1.3106075308874148, 1.3348780407186631, 1.3591485505499117, 1.38341906038116, 1.4076895702124086, 1.431960080043657, 1.4562305898749053, 1.4805010997061536, 1.5047716095374022, 1.5290421193686505, 1.553312629199899, 1.5775831390311474, 1.601853648862396, 1.6261241586936441, 1.650394668524893, 1.674665178356141, 1.6989356881873898, 1.723206198018638, 1.7474767078498865, 1.7717472176811349, 1.7960177275123832, 1.8202882373436318, 1.84455874717488, 1.8688292570061287, 1.893099766837377, 1.9173702766686256, 1.9416407864998737, 1.965911296331122, 1.9901818061623706, 2.014452315993619, 2.0387228258248675, 2.062993335656116, 2.0872638454873647, 2.1115343553186126, 2.135804865149861, 2.1600753749811097, 2.184345884812358, 2.2086163946436064, 2.2328869044748547, 2.2571574143061035, 2.281427924137352, 2.3056984339686, 2.3299689437998485, 2.354239453631097, 2.3785099634623457, 2.4027804732935936, 2.4270509831248424 ], [ 0, 0.023115397283273678, 0.046230794566547356, 0.06934619184982103, 0.09246158913309471, 0.11557698641636838, 0.13869238369964207, 0.16180778098291573, 0.18492317826618943, 0.20803857554946312, 0.23115397283273675, 0.2542693701160104, 0.27738476739928414, 0.3005001646825578, 0.32361556196583147, 0.34673095924910513, 0.36984635653237885, 0.3929617538156525, 0.41607715109892623, 0.43919254838219984, 0.4623079456654735, 0.4854233429487472, 0.5085387402320208, 0.5316541375152946, 0.5547695347985683, 0.577884932081842, 0.6010003293651156, 0.6241157266483892, 0.6472311239316629, 0.6703465212149367, 0.6934619184982103, 0.716577315781484, 0.7396927130647577, 0.7628081103480313, 0.785923507631305, 0.8090389049145787, 0.8321543021978525, 0.855269699481126, 0.8783850967643997, 0.9015004940476734, 0.924615891330947, 0.9477312886142207, 0.9708466858974945, 0.9939620831807682, 1.0170774804640417, 1.0401928777473153, 1.0633082750305891, 1.0864236723138627, 1.1095390695971366, 1.1326544668804102, 1.155769864163684, 1.1788852614469576, 1.2020006587302312, 1.2251160560135048, 1.2482314532967784, 1.2713468505800523, 1.2944622478633259, 1.3175776451465997, 1.3406930424298733, 1.363808439713147, 1.3869238369964205, 1.4100392342796941, 1.433154631562968, 1.4562700288462416, 1.4793854261295154, 1.502500823412789, 1.5256162206960626, 1.5487316179793362, 1.57184701526261, 1.5949624125458837, 1.6180778098291575, 1.641193207112431, 1.664308604395705, 1.6874240016789783, 1.710539398962252, 1.7336547962455258, 1.7567701935287994, 1.7798855908120732, 1.8030009880953468, 1.8261163853786206, 1.849231782661894, 1.8723471799451676, 1.8954625772284415, 1.918577974511715, 1.941693371794989, 1.9648087690782625, 1.9879241663615363, 2.0110395636448097, 2.0341549609280833, 2.0572703582113574, 2.0803857554946306, 2.1035011527779046, 2.1266165500611782, 2.149731947344452, 2.1728473446277254, 2.1959627419109995, 2.219078139194273, 2.2421935364775467, 2.2653089337608203, 2.288424331044094, 2.311539728327368 ], [ 0, 0.021869058822642347, 0.043738117645284694, 0.06560717646792703, 0.08747623529056939, 0.10934529411321173, 0.13121435293585407, 0.1530834117584964, 0.17495247058113877, 0.19682152940378114, 0.21869058822642345, 0.2405596470490658, 0.26242870587170813, 0.28429776469435053, 0.3061668235169928, 0.32803588233963515, 0.34990494116227755, 0.3717739999849199, 0.3936430588075623, 0.41551211763020457, 0.4373811764528469, 0.4592502352754893, 0.4811192940981316, 0.5029883529207739, 0.5248574117434163, 0.5467264705660586, 0.5685955293887011, 0.5904645882113433, 0.6123336470339856, 0.6342027058566281, 0.6560717646792703, 0.6779408235019126, 0.6998098823245551, 0.7216789411471974, 0.7435479999698398, 0.7654170587924821, 0.7872861176151246, 0.8091551764377667, 0.8310242352604091, 0.8528932940830515, 0.8747623529056938, 0.8966314117283362, 0.9185004705509786, 0.940369529373621, 0.9622385881962632, 0.9841076470189055, 1.0059767058415479, 1.0278457646641903, 1.0497148234868325, 1.071583882309475, 1.0934529411321172, 1.1153219999547597, 1.1371910587774021, 1.1590601176000443, 1.1809291764226866, 1.202798235245329, 1.2246672940679713, 1.2465363528906137, 1.2684054117132562, 1.2902744705358984, 1.3121435293585406, 1.334012588181183, 1.3558816470038253, 1.3777507058264677, 1.3996197646491102, 1.4214888234717524, 1.4433578822943949, 1.465226941117037, 1.4870959999396796, 1.5089650587623218, 1.5308341175849642, 1.5527031764076065, 1.5745722352302491, 1.5964412940528914, 1.6183103528755334, 1.640179411698176, 1.6620484705208183, 1.6839175293434607, 1.705786588166103, 1.7276556469887454, 1.7495247058113876, 1.7713937646340299, 1.7932628234566723, 1.8151318822793145, 1.8370009411019572, 1.8588699999245992, 1.880739058747242, 1.9026081175698841, 1.9244771763925264, 1.9463462352151688, 1.968215294037811, 1.9900843528604535, 2.0119534116830957, 2.033822470505738, 2.0556915293283806, 2.077560588151023, 2.099429646973665, 2.1212987057963075, 2.14316776461895, 2.165036823441592, 2.1869058822642344 ], [ 0, 0.020536413177860653, 0.041072826355721306, 0.061609239533581966, 0.08214565271144261, 0.10268206588930327, 0.12321847906716393, 0.14375489224502458, 0.16429130542288523, 0.1848277186007459, 0.20536413177860655, 0.2259005449564672, 0.24643695813432787, 0.2669733713121885, 0.28750978449004916, 0.3080461976679098, 0.32858261084577045, 0.34911902402363115, 0.3696554372014918, 0.3901918503793524, 0.4107282635572131, 0.43126467673507374, 0.4518010899129344, 0.47233750309079503, 0.49287391626865573, 0.5134103294465164, 0.533946742624377, 0.5544831558022376, 0.5750195689800983, 0.595555982157959, 0.6160923953358196, 0.6366288085136803, 0.6571652216915409, 0.6777016348694016, 0.6982380480472623, 0.718774461225123, 0.7393108744029836, 0.7598472875808442, 0.7803837007587048, 0.8009201139365655, 0.8214565271144262, 0.8419929402922869, 0.8625293534701475, 0.8830657666480082, 0.9036021798258688, 0.9241385930037294, 0.9446750061815901, 0.9652114193594508, 0.9857478325373115, 1.0062842457151722, 1.0268206588930329, 1.0473570720708933, 1.067893485248754, 1.0884298984266145, 1.1089663116044752, 1.129502724782336, 1.1500391379601966, 1.1705755511380573, 1.191111964315918, 1.2116483774937787, 1.2321847906716392, 1.2527212038495, 1.2732576170273606, 1.293794030205221, 1.3143304433830818, 1.3348668565609425, 1.3554032697388032, 1.3759396829166637, 1.3964760960945246, 1.417012509272385, 1.437548922450246, 1.4580853356281065, 1.4786217488059672, 1.499158161983828, 1.5196945751616884, 1.540230988339549, 1.5607674015174096, 1.5813038146952705, 1.601840227873131, 1.6223766410509919, 1.6429130542288524, 1.6634494674067128, 1.6839858805845738, 1.7045222937624342, 1.725058706940295, 1.7455951201181557, 1.7661315332960164, 1.7866679464738768, 1.8072043596517375, 1.8277407728295982, 1.8482771860074587, 1.8688135991853196, 1.8893500123631801, 1.909886425541041, 1.9304228387189015, 1.9509592518967622, 1.971495665074623, 1.9920320782524834, 2.0125684914303443, 2.0331049046082046, 2.0536413177860657 ], [ 0, 0.019122719692460686, 0.03824543938492137, 0.057368159077382054, 0.07649087876984274, 0.09561359846230343, 0.11473631815476411, 0.1338590378472248, 0.1529817575396855, 0.17210447723214617, 0.19122719692460685, 0.2103499166170675, 0.22947263630952822, 0.24859535600198893, 0.2677180756944496, 0.28684079538691026, 0.305963515079371, 0.3250862347718317, 0.34420895446429234, 0.363331674156753, 0.3824543938492137, 0.4015771135416744, 0.420699833234135, 0.4398225529265957, 0.45894527261905643, 0.47806799231151714, 0.49719071200397785, 0.5163134316964385, 0.5354361513888992, 0.5545588710813599, 0.5736815907738205, 0.5928043104662812, 0.611927030158742, 0.6310497498512027, 0.6501724695436634, 0.6692951892361241, 0.6884179089285847, 0.7075406286210453, 0.726663348313506, 0.7457860680059667, 0.7649087876984274, 0.7840315073908881, 0.8031542270833488, 0.8222769467758095, 0.84139966646827, 0.8605223861607307, 0.8796451058531914, 0.8987678255456522, 0.9178905452381129, 0.9370132649305736, 0.9561359846230343, 0.975258704315495, 0.9943814240079557, 1.0135041437004162, 1.032626863392877, 1.0517495830853376, 1.0708723027777984, 1.089995022470259, 1.1091177421627199, 1.1282404618551805, 1.147363181547641, 1.1664859012401017, 1.1856086209325625, 1.204731340625023, 1.223854060317484, 1.2429767800099445, 1.2620994997024053, 1.281222219394866, 1.3003449390873267, 1.3194676587797871, 1.3385903784722482, 1.3577130981647085, 1.3768358178571694, 1.39595853754963, 1.4150812572420906, 1.4342039769345514, 1.453326696627012, 1.4724494163194728, 1.4915721360119334, 1.5106948557043942, 1.5298175753968548, 1.5489402950893154, 1.5680630147817762, 1.5871857344742368, 1.6063084541666977, 1.6254311738591583, 1.644553893551619, 1.6636766132440797, 1.68279933293654, 1.701922052629001, 1.7210447723214615, 1.7401674920139223, 1.759290211706383, 1.7784129313988437, 1.7975356510913043, 1.8166583707837651, 1.8357810904762257, 1.8549038101686863, 1.8740265298611472, 1.8931492495536077, 1.9122719692460686 ], [ 0, 0.017633557568774196, 0.03526711513754839, 0.05290067270632259, 0.07053423027509678, 0.08816778784387098, 0.10580134541264519, 0.12343490298141938, 0.14106846055019356, 0.15870201811896778, 0.17633557568774197, 0.19396913325651616, 0.21160269082529037, 0.22923624839406456, 0.24686980596283875, 0.2645033635316129, 0.28213692110038713, 0.29977047866916134, 0.31740403623793556, 0.3350375938067097, 0.35267115137548394, 0.37030470894425815, 0.3879382665130323, 0.40557182408180653, 0.42320538165058075, 0.4408389392193549, 0.4584724967881291, 0.4761060543569033, 0.4937396119256775, 0.5113731694944518, 0.5290067270632258, 0.5466402846320001, 0.5642738422007743, 0.5819073997695485, 0.5995409573383227, 0.617174514907097, 0.6348080724758711, 0.6524416300446453, 0.6700751876134194, 0.6877087451821936, 0.7053423027509679, 0.722975860319742, 0.7406094178885163, 0.7582429754572905, 0.7758765330260646, 0.7935100905948388, 0.8111436481636131, 0.8287772057323872, 0.8464107633011615, 0.8640443208699357, 0.8816778784387098, 0.8993114360074841, 0.9169449935762582, 0.9345785511450324, 0.9522121087138066, 0.9698456662825808, 0.987479223851355, 1.0051127814201293, 1.0227463389889035, 1.0403798965576776, 1.0580134541264516, 1.075647011695226, 1.0932805692640002, 1.1109141268327745, 1.1285476844015485, 1.1461812419703228, 1.163814799539097, 1.181448357107871, 1.1990819146766454, 1.2167154722454194, 1.234349029814194, 1.251982587382968, 1.2696161449517422, 1.2872497025205163, 1.3048832600892906, 1.3225168176580648, 1.3401503752268389, 1.3577839327956132, 1.3754174903643872, 1.3930510479331617, 1.4106846055019358, 1.4283181630707098, 1.445951720639484, 1.4635852782082583, 1.4812188357770326, 1.4988523933458067, 1.516485950914581, 1.5341195084833552, 1.5517530660521293, 1.5693866236209035, 1.5870201811896776, 1.6046537387584519, 1.6222872963272261, 1.6399208538960004, 1.6575544114647744, 1.6751879690335487, 1.692821526602323, 1.710455084171097, 1.7280886417398713, 1.7457221993086454, 1.7633557568774196 ], [ 0, 0.0160748038493699, 0.0321496076987398, 0.0482244115481097, 0.0642992153974796, 0.08037401924684949, 0.0964488230962194, 0.11252362694558929, 0.1285984307949592, 0.1446732346443291, 0.16074803849369898, 0.1768228423430689, 0.1928976461924388, 0.2089724500418087, 0.22504725389117858, 0.24112205774054848, 0.2571968615899184, 0.2732716654392883, 0.2893464692886582, 0.3054212731380281, 0.32149607698739796, 0.3375708808367679, 0.3536456846861378, 0.36972048853550765, 0.3857952923848776, 0.4018700962342475, 0.4179449000836174, 0.4340197039329873, 0.45009450778235716, 0.4661693116317271, 0.48224411548109697, 0.49831891933046685, 0.5143937231798368, 0.5304685270292067, 0.5465433308785766, 0.5626181347279465, 0.5786929385773164, 0.5947677424266862, 0.6108425462760562, 0.626917350125426, 0.6429921539747959, 0.6590669578241659, 0.6751417616735358, 0.6912165655229057, 0.7072913693722755, 0.7233661732216454, 0.7394409770710153, 0.7555157809203853, 0.7715905847697552, 0.787665388619125, 0.803740192468495, 0.8198149963178649, 0.8358898001672348, 0.8519646040166046, 0.8680394078659746, 0.8841142117153444, 0.9001890155647143, 0.9162638194140843, 0.9323386232634542, 0.9484134271128241, 0.9644882309621939, 0.9805630348115638, 0.9966378386609337, 1.0127126425103037, 1.0287874463596736, 1.0448622502090434, 1.0609370540584133, 1.0770118579077832, 1.0930866617571533, 1.109161465606523, 1.125236269455893, 1.1413110733052627, 1.1573858771546328, 1.1734606810040027, 1.1895354848533723, 1.2056102887027424, 1.2216850925521123, 1.2377598964014822, 1.253834700250852, 1.2699095041002222, 1.2859843079495918, 1.3020591117989617, 1.3181339156483318, 1.3342087194977015, 1.3502835233470716, 1.3663583271964415, 1.3824331310458113, 1.3985079348951812, 1.414582738744551, 1.430657542593921, 1.4467323464432909, 1.462807150292661, 1.4788819541420306, 1.4949567579914007, 1.5110315618407706, 1.5271063656901405, 1.5431811695395103, 1.55925597338888, 1.57533077723825, 1.59140558108762, 1.60748038493699 ], [ 0, 0.014452610223051455, 0.02890522044610291, 0.04335783066915437, 0.05781044089220582, 0.07226305111525728, 0.08671566133830874, 0.10116827156136018, 0.11562088178441164, 0.13007349200746313, 0.14452610223051457, 0.158978712453566, 0.17343132267661748, 0.18788393289966893, 0.20233654312272037, 0.2167891533457718, 0.23124176356882328, 0.24569437379187475, 0.26014698401492625, 0.27459959423797764, 0.28905220446102914, 0.3035048146840806, 0.317957424907132, 0.33241003513018347, 0.34686264535323497, 0.3613152555762864, 0.37576786579933785, 0.3902204760223893, 0.40467308624544074, 0.41912569646849224, 0.4335783066915436, 0.4480309169145951, 0.46248352713764657, 0.47693613736069806, 0.4913887475837495, 0.505841357806801, 0.5202939680298525, 0.5347465782529038, 0.5491991884759553, 0.5636517986990067, 0.5781044089220583, 0.5925570191451097, 0.6070096293681612, 0.6214622395912126, 0.635914849814264, 0.6503674600373155, 0.6648200702603669, 0.6792726804834184, 0.6937252907064699, 0.7081779009295214, 0.7226305111525728, 0.7370831213756243, 0.7515357315986757, 0.7659883418217271, 0.7804409520447786, 0.79489356226783, 0.8093461724908815, 0.823798782713933, 0.8382513929369845, 0.8527040031600359, 0.8671566133830872, 0.8816092236061388, 0.8960618338291902, 0.9105144440522417, 0.9249670542752931, 0.9394196644983447, 0.9538722747213961, 0.9683248849444475, 0.982777495167499, 0.9972301053905505, 1.011682715613602, 1.0261353258366535, 1.040587936059705, 1.0550405462827563, 1.0694931565058077, 1.0839457667288592, 1.0983983769519106, 1.1128509871749621, 1.1273035973980134, 1.141756207621065, 1.1562088178441166, 1.1706614280671679, 1.1851140382902194, 1.1995666485132708, 1.2140192587363223, 1.2284718689593737, 1.2429244791824252, 1.2573770894054765, 1.271829699628528, 1.2862823098515797, 1.300734920074631, 1.3151875302976825, 1.3296401405207339, 1.3440927507437854, 1.3585453609668368, 1.3729979711898883, 1.3874505814129399, 1.4019031916359912, 1.4163558018590428, 1.430808412082094, 1.4452610223051456 ], [ 0, 0.012773378746952174, 0.025546757493904348, 0.03832013624085652, 0.051093514987808696, 0.06386689373476087, 0.07664027248171304, 0.08941365122866522, 0.10218702997561739, 0.11496040872256959, 0.12773378746952174, 0.1405071662164739, 0.15328054496342608, 0.16605392371037828, 0.17882730245733044, 0.1916006812042826, 0.20437405995123478, 0.21714743869818698, 0.22992081744513918, 0.2426941961920913, 0.2554675749390435, 0.26824095368599565, 0.2810143324329478, 0.2937877111799, 0.30656108992685216, 0.3193344686738044, 0.33210784742075655, 0.3448812261677087, 0.3576546049146609, 0.37042798366161306, 0.3832013624085652, 0.3959747411555174, 0.40874811990246956, 0.4215214986494218, 0.43429487739637396, 0.4470682561433261, 0.45984163489027835, 0.4726150136372304, 0.4853883923841826, 0.4981617711311348, 0.510935149878087, 0.5237085286250391, 0.5364819073719913, 0.5492552861189436, 0.5620286648658956, 0.5748020436128478, 0.5875754223598, 0.6003488011067522, 0.6131221798537043, 0.6258955586006566, 0.6386689373476088, 0.6514423160945609, 0.6642156948415131, 0.6769890735884652, 0.6897624523354174, 0.7025358310823696, 0.7153092098293218, 0.728082588576274, 0.7408559673232261, 0.7536293460701783, 0.7664027248171305, 0.7791761035640826, 0.7919494823110348, 0.804722861057987, 0.8174962398049391, 0.8302696185518913, 0.8430429972988436, 0.8558163760457956, 0.8685897547927479, 0.8813631335397, 0.8941365122866523, 0.9069098910336043, 0.9196832697805567, 0.9324566485275088, 0.9452300272744608, 0.9580034060214131, 0.9707767847683652, 0.9835501635153174, 0.9963235422622696, 1.0090969210092218, 1.021870299756174, 1.034643678503126, 1.0474170572500783, 1.0601904359970304, 1.0729638147439826, 1.0857371934909348, 1.0985105722378872, 1.1112839509848391, 1.1240573297317913, 1.1368307084787435, 1.1496040872256956, 1.1623774659726478, 1.1751508447196, 1.1879242234665524, 1.2006976022135043, 1.2134709809604567, 1.2262443597074086, 1.2390177384543608, 1.2517911172013132, 1.2645644959482651, 1.2773378746952175 ], [ 0, 0.011043736580540331, 0.022087473161080663, 0.03313120974162099, 0.044174946322161325, 0.055218682902701655, 0.06626241948324198, 0.07730615606378231, 0.08834989264432265, 0.09939362922486299, 0.11043736580540331, 0.12148110238594363, 0.13252483896648395, 0.14356857554702432, 0.15461231212756463, 0.16565604870810494, 0.1766997852886453, 0.18774352186918564, 0.19878725844972597, 0.20983099503026628, 0.22087473161080662, 0.23191846819134695, 0.24296220477188726, 0.2540059413524276, 0.2650496779329679, 0.2760934145135083, 0.28713715109404864, 0.2981808876745889, 0.30922462425512925, 0.3202683608356696, 0.33131209741620987, 0.34235583399675024, 0.3533995705772906, 0.3644433071578309, 0.3754870437383713, 0.3865307803189116, 0.39757451689945195, 0.4086182534799922, 0.41966199006053256, 0.4307057266410729, 0.44174946322161324, 0.45279319980215355, 0.4638369363826939, 0.4748806729632342, 0.4859244095437745, 0.49696814612431484, 0.5080118827048552, 0.5190556192853956, 0.5300993558659358, 0.5411430924464762, 0.5521868290270165, 0.5632305656075569, 0.5742743021880973, 0.5853180387686375, 0.5963617753491778, 0.6074055119297181, 0.6184492485102585, 0.6294929850907989, 0.6405367216713392, 0.6515804582518795, 0.6626241948324197, 0.6736679314129601, 0.6847116679935005, 0.6957554045740408, 0.7067991411545812, 0.7178428777351215, 0.7288866143156618, 0.7399303508962021, 0.7509740874767425, 0.7620178240572828, 0.7730615606378232, 0.7841052972183634, 0.7951490337989039, 0.8061927703794441, 0.8172365069599844, 0.8282802435405248, 0.8393239801210651, 0.8503677167016055, 0.8614114532821457, 0.8724551898626862, 0.8834989264432265, 0.8945426630237667, 0.9055863996043071, 0.9166301361848473, 0.9276738727653878, 0.9387176093459281, 0.9497613459264684, 0.9608050825070087, 0.971848819087549, 0.9828925556680894, 0.9939362922486297, 1.0049800288291701, 1.0160237654097104, 1.0270675019902509, 1.0381112385707911, 1.0491549751513314, 1.0601987117318716, 1.0712424483124119, 1.0822861848929524, 1.0933299214734926, 1.104373658054033 ], [ 0, 0.009270509831248414, 0.018541019662496827, 0.02781152949374524, 0.037082039324993654, 0.04635254915624207, 0.05562305898749048, 0.0648935688187389, 0.07416407864998731, 0.08343458848123572, 0.09270509831248414, 0.10197560814373254, 0.11124611797498096, 0.12051662780622938, 0.1297871376374778, 0.1390576474687262, 0.14832815729997462, 0.15759866713122303, 0.16686917696247144, 0.17613968679371983, 0.18541019662496827, 0.19468070645621668, 0.20395121628746507, 0.21322172611871348, 0.22249223594996193, 0.23176274578121034, 0.24103325561245875, 0.25030376544370714, 0.2595742752749556, 0.26884478510620397, 0.2781152949374524, 0.2873858047687008, 0.29665631459994923, 0.3059268244311976, 0.31519733426244606, 0.3244678440936945, 0.3337383539249429, 0.3430088637561913, 0.35227937358743966, 0.3615498834186881, 0.37082039324993654, 0.3800909030811849, 0.38936141291243337, 0.3986319227436818, 0.40790243257493014, 0.4171729424061786, 0.42644345223742697, 0.4357139620686754, 0.44498447189992385, 0.45425498173117224, 0.4635254915624207, 0.4727960013936691, 0.4820665112249175, 0.4913370210561659, 0.5006075308874143, 0.5098780407186627, 0.5191485505499112, 0.5284190603811596, 0.5376895702124079, 0.5469600800436564, 0.5562305898749048, 0.5655010997061531, 0.5747716095374016, 0.58404211936865, 0.5933126291998985, 0.6025831390311469, 0.6118536488623952, 0.6211241586936437, 0.6303946685248921, 0.6396651783561405, 0.648935688187389, 0.6582061980186373, 0.6674767078498858, 0.6767472176811342, 0.6860177275123825, 0.695288237343631, 0.7045587471748793, 0.7138292570061279, 0.7230997668373762, 0.7323702766686248, 0.7416407864998731, 0.7509112963311214, 0.7601818061623699, 0.7694523159936183, 0.7787228258248667, 0.7879933356561151, 0.7972638454873636, 0.806534355318612, 0.8158048651498603, 0.8250753749811088, 0.8343458848123572, 0.8436163946436056, 0.8528869044748539, 0.8621574143061025, 0.8714279241373508, 0.8806984339685993, 0.8899689437998477, 0.899239453631096, 0.9085099634623445, 0.9177804732935929, 0.9270509831248414 ], [ 0, 0.007460696614945643, 0.014921393229891286, 0.02238208984483693, 0.029842786459782573, 0.037303483074728215, 0.04476417968967386, 0.05222487630461951, 0.059685572919565145, 0.0671462695345108, 0.07460696614945643, 0.08206766276440207, 0.08952835937934772, 0.09698905599429337, 0.10444975260923901, 0.11191044922418465, 0.11937114583913029, 0.12683184245407594, 0.1342925390690216, 0.14175323568396722, 0.14921393229891286, 0.15667462891385853, 0.16413532552880414, 0.1715960221437498, 0.17905671875869544, 0.18651741537364108, 0.19397811198858675, 0.20143880860353236, 0.20889950521847803, 0.21636020183342367, 0.2238208984483693, 0.23128159506331494, 0.23874229167826058, 0.24620298829320625, 0.2536636849081519, 0.2611243815230975, 0.2685850781380432, 0.2760457747529888, 0.28350647136793444, 0.2909671679828801, 0.2984278645978257, 0.3058885612127714, 0.31334925782771705, 0.3208099544426627, 0.3282706510576083, 0.3357313476725539, 0.3431920442874996, 0.35065274090244525, 0.3581134375173909, 0.3655741341323365, 0.37303483074728216, 0.38049552736222786, 0.3879562239771735, 0.3954169205921191, 0.4028776172070647, 0.4103383138220104, 0.41779901043695605, 0.4252597070519017, 0.43272040366684733, 0.44018110028179297, 0.4476417968967386, 0.45510249351168425, 0.4625631901266299, 0.4700238867415755, 0.47748458335652116, 0.48494527997146686, 0.4924059765864125, 0.4998666732013581, 0.5073273698163038, 0.5147880664312494, 0.522248763046195, 0.5297094596611407, 0.5371701562760864, 0.544630852891032, 0.5520915495059776, 0.5595522461209232, 0.5670129427358689, 0.5744736393508146, 0.5819343359657602, 0.5893950325807059, 0.5968557291956514, 0.6043164258105971, 0.6117771224255428, 0.6192378190404884, 0.6266985156554341, 0.6341592122703796, 0.6416199088853254, 0.649080605500271, 0.6565413021152166, 0.6640019987301623, 0.6714626953451078, 0.6789233919600536, 0.6863840885749992, 0.6938447851899449, 0.7013054818048905, 0.7087661784198361, 0.7162268750347818, 0.7236875716497274, 0.731148268264673, 0.7386089648796187, 0.7460696614945643 ], [ 0, 0.005621439437571736, 0.011242878875143473, 0.016864318312715208, 0.022485757750286946, 0.02810719718785868, 0.033728636625430415, 0.03935007606300215, 0.04497151550057389, 0.05059295493814563, 0.05621439437571736, 0.061835833813289096, 0.06745727325086083, 0.07307871268843258, 0.0787001521260043, 0.08432159156357603, 0.08994303100114778, 0.09556447043871952, 0.10118590987629127, 0.10680734931386299, 0.11242878875143472, 0.11805022818900647, 0.12367166762657819, 0.12929310706414993, 0.13491454650172166, 0.1405359859392934, 0.14615742537686516, 0.15177886481443686, 0.1574003042520086, 0.16302174368958036, 0.16864318312715207, 0.1742646225647238, 0.17988606200229557, 0.1855075014398673, 0.19112894087743904, 0.19675038031501077, 0.20237181975258253, 0.2079932591901542, 0.21361469862772597, 0.2192361380652977, 0.22485757750286944, 0.23047901694044118, 0.23610045637801294, 0.24172189581558468, 0.24734333525315638, 0.2529647746907281, 0.25858621412829985, 0.2642076535658716, 0.2698290930034433, 0.2754505324410151, 0.2810719718785868, 0.28669341131615855, 0.2923148507537303, 0.297936290191302, 0.30355772962887373, 0.3091791690664455, 0.3148006085040172, 0.32042204794158896, 0.3260434873791607, 0.33166492681673243, 0.33728636625430414, 0.3429078056918759, 0.3485292451294476, 0.35415068456701937, 0.35977212400459113, 0.36539356344216284, 0.3710150028797346, 0.3766364423173063, 0.38225788175487807, 0.3878793211924498, 0.39350076063002154, 0.39912220006759325, 0.40474363950516506, 0.41036507894273677, 0.4159865183803084, 0.42160795781788024, 0.42722939725545195, 0.4328508366930237, 0.4384722761305954, 0.4440937155681672, 0.4497151550057389, 0.4553365944433106, 0.46095803388088236, 0.46657947331845406, 0.4722009127560259, 0.47782235219359753, 0.48344379163116935, 0.48906523106874106, 0.49468667050631276, 0.5003081099438845, 0.5059295493814562, 0.511550988819028, 0.5171724282565997, 0.5227938676941715, 0.5284153071317432, 0.534036746569315, 0.5396581860068866, 0.5452796254444584, 0.5509010648820302, 0.5565225043196018, 0.5621439437571736 ], [ 0, 0.003759997006929123, 0.007519994013858246, 0.011279991020787368, 0.015039988027716491, 0.018799985034645614, 0.022559982041574736, 0.026319979048503857, 0.030079976055432982, 0.033839973062362104, 0.03759997006929123, 0.04135996707622035, 0.04511996408314947, 0.0488799610900786, 0.052639958097007715, 0.05639995510393683, 0.060159952110865965, 0.06391994911779508, 0.06767994612472421, 0.07143994313165333, 0.07519994013858246, 0.07895993714551158, 0.0827199341524407, 0.08647993115936982, 0.09023992816629894, 0.09399992517322807, 0.0977599221801572, 0.1015199191870863, 0.10527991619401543, 0.10903991320094455, 0.11279991020787367, 0.11655990721480279, 0.12031990422173193, 0.12407990122866105, 0.12783989823559017, 0.1315998952425193, 0.13535989224944842, 0.13911988925637753, 0.14287988626330667, 0.14663988327023578, 0.15039988027716492, 0.15415987728409403, 0.15791987429102317, 0.16167987129795228, 0.1654398683048814, 0.1691998653118105, 0.17295986231873964, 0.17671985932566875, 0.1804798563325979, 0.184239853339527, 0.18799985034645614, 0.19175984735338528, 0.1955198443603144, 0.1992798413672435, 0.2030398383741726, 0.20679983538110175, 0.21055983238803086, 0.21431982939496, 0.2180798264018891, 0.22183982340881825, 0.22559982041574733, 0.22935981742267647, 0.23311981442960558, 0.23687981143653472, 0.24063980844346386, 0.24439980545039297, 0.2481598024573221, 0.2519197994642512, 0.25567979647118033, 0.25943979347810947, 0.2631997904850386, 0.2669597874919677, 0.27071978449889683, 0.27447978150582597, 0.27823977851275505, 0.2819997755196842, 0.28575977252661333, 0.28951976953354247, 0.29327976654047155, 0.2970397635474007, 0.30079976055432983, 0.3045597575612589, 0.30831975456818805, 0.31207975157511714, 0.31583974858204633, 0.3195997455889754, 0.32335974259590455, 0.32711973960283364, 0.3308797366097628, 0.3346397336166919, 0.338399730623621, 0.3421597276305502, 0.3459197246374793, 0.3496797216444084, 0.3534397186513375, 0.3571997156582667, 0.3609597126651958, 0.36471970967212486, 0.368479706679054, 0.37223970368598314, 0.3759997006929123 ], [ 0, 0.001883715585879394, 0.003767431171758788, 0.005651146757638182, 0.007534862343517576, 0.00941857792939697, 0.011302293515276365, 0.013186009101155758, 0.015069724687035152, 0.01695344027291455, 0.01883715585879394, 0.020720871444673335, 0.02260458703055273, 0.024488302616432124, 0.026372018202311515, 0.02825573378819091, 0.030139449374070305, 0.0320231649599497, 0.0339068805458291, 0.035790596131708485, 0.03767431171758788, 0.039558027303467275, 0.04144174288934667, 0.043325458475226064, 0.04520917406110546, 0.04709288964698485, 0.04897660523286425, 0.050860320818743636, 0.05274403640462303, 0.05462775199050243, 0.05651146757638182, 0.058395183162261215, 0.06027889874814061, 0.062162614334020004, 0.0640463299198994, 0.0659300455057788, 0.0678137610916582, 0.06969747667753758, 0.07158119226341697, 0.07346490784929637, 0.07534862343517576, 0.07723233902105515, 0.07911605460693455, 0.08099977019281394, 0.08288348577869334, 0.08476720136457273, 0.08665091695045213, 0.08853463253633152, 0.09041834812221092, 0.09230206370809031, 0.0941857792939697, 0.0960694948798491, 0.0979532104657285, 0.09983692605160788, 0.10172064163748727, 0.10360435722336667, 0.10548807280924606, 0.10737178839512547, 0.10925550398100486, 0.11113921956688426, 0.11302293515276364, 0.11490665073864303, 0.11679036632452243, 0.11867408191040182, 0.12055779749628122, 0.12244151308216061, 0.12432522866804001, 0.1262089442539194, 0.1280926598397988, 0.1299763754256782, 0.1318600910115576, 0.13374380659743698, 0.1356275221833164, 0.13751123776919577, 0.13939495335507515, 0.14127866894095456, 0.14316238452683394, 0.14504610011271335, 0.14692981569859273, 0.14881353128447214, 0.15069724687035152, 0.1525809624562309, 0.1544646780421103, 0.1563483936279897, 0.1582321092138691, 0.16011582479974848, 0.1619995403856279, 0.16388325597150727, 0.16576697155738668, 0.16765068714326609, 0.16953440272914547, 0.17141811831502488, 0.17330183390090426, 0.17518554948678366, 0.17706926507266305, 0.17895298065854245, 0.18083669624442184, 0.18272041183030122, 0.18460412741618062, 0.18648784300206, 0.1883715585879394 ], [ 0, -9.648735898059819e-18, -1.9297471796119638e-17, -2.8946207694179455e-17, -3.8594943592239275e-17, -4.8243679490299096e-17, -5.789241538835891e-17, -6.754115128641874e-17, -7.718988718447855e-17, -8.683862308253838e-17, -9.648735898059819e-17, -1.06136094878658e-16, -1.1578483077671782e-16, -1.2543356667477766e-16, -1.3508230257283747e-16, -1.4473103847089729e-16, -1.543797743689571e-16, -1.6402851026701694e-16, -1.7367724616507675e-16, -1.8332598206313657e-16, -1.9297471796119638e-16, -2.0262345385925622e-16, -2.12272189757316e-16, -2.2192092565537582e-16, -2.3156966155343564e-16, -2.412183974514955e-16, -2.508671333495553e-16, -2.6051586924761513e-16, -2.7016460514567494e-16, -2.7981334104373476e-16, -2.8946207694179457e-16, -2.991108128398544e-16, -3.087595487379142e-16, -3.1840828463597407e-16, -3.280570205340339e-16, -3.377057564320937e-16, -3.473544923301535e-16, -3.5700322822821327e-16, -3.6665196412627314e-16, -3.7630070002433295e-16, -3.8594943592239276e-16, -3.955981718204526e-16, -4.0524690771851244e-16, -4.1489564361657226e-16, -4.24544379514632e-16, -4.3419311541269184e-16, -4.4384185131075165e-16, -4.534905872088115e-16, -4.631393231068713e-16, -4.727880590049311e-16, -4.82436794902991e-16, -4.920855308010508e-16, -5.017342666991106e-16, -5.113830025971704e-16, -5.210317384952303e-16, -5.3068047439329e-16, -5.403292102913499e-16, -5.499779461894098e-16, -5.596266820874695e-16, -5.692754179855294e-16, -5.789241538835891e-16, -5.885728897816489e-16, -5.982216256797088e-16, -6.078703615777686e-16, -6.175190974758284e-16, -6.271678333738883e-16, -6.368165692719481e-16, -6.464653051700078e-16, -6.561140410680678e-16, -6.657627769661275e-16, -6.754115128641874e-16, -6.850602487622472e-16, -6.94708984660307e-16, -7.043577205583668e-16, -7.140064564564265e-16, -7.236551923544865e-16, -7.333039282525463e-16, -7.429526641506061e-16, -7.526014000486659e-16, -7.622501359467258e-16, -7.718988718447855e-16, -7.815476077428453e-16, -7.911963436409052e-16, -8.008450795389649e-16, -8.104938154370249e-16, -8.201425513350846e-16, -8.297912872331445e-16, -8.394400231312043e-16, -8.49088759029264e-16, -8.587374949273239e-16, -8.683862308253837e-16, -8.780349667234435e-16, -8.876837026215033e-16, -8.973324385195632e-16, -9.06981174417623e-16, -9.166299103156829e-16, -9.262786462137426e-16, -9.359273821118024e-16, -9.455761180098623e-16, -9.552248539079221e-16, -9.64873589805982e-16 ], [ 0, -0.0018837155858794, -0.0037674311717588, -0.0056511467576382, -0.0075348623435176, -0.009418577929397, -0.0113022935152764, -0.0131860091011558, -0.0150697246870352, -0.0169534402729146, -0.018837155858794, -0.020720871444673397, -0.0226045870305528, -0.0244883026164322, -0.0263720182023116, -0.028255733788190997, -0.0301394493740704, -0.0320231649599498, -0.0339068805458292, -0.035790596131708596, -0.037674311717588, -0.0395580273034674, -0.041441742889346794, -0.043325458475226196, -0.0452091740611056, -0.047092889646985, -0.0489766052328644, -0.050860320818743796, -0.0527440364046232, -0.0546277519905026, -0.056511467576381993, -0.058395183162261395, -0.0602788987481408, -0.0621626143340202, -0.0640463299198996, -0.06593004550577901, -0.0678137610916584, -0.06969747667753778, -0.07158119226341719, -0.07346490784929659, -0.075348623435176, -0.07723233902105539, -0.0791160546069348, -0.08099977019281421, -0.08288348577869359, -0.08476720136457298, -0.08665091695045239, -0.08853463253633179, -0.0904183481222112, -0.09230206370809059, -0.09418577929397, -0.09606949487984941, -0.0979532104657288, -0.09983692605160818, -0.10172064163748759, -0.10360435722336699, -0.1054880728092464, -0.10737178839512579, -0.1092555039810052, -0.1111392195668846, -0.11302293515276399, -0.11490665073864338, -0.11679036632452279, -0.11867408191040218, -0.1205577974962816, -0.12244151308216099, -0.1243252286680404, -0.1262089442539198, -0.1280926598397992, -0.12997637542567858, -0.13186009101155802, -0.1337438065974374, -0.1356275221833168, -0.1375112377691962, -0.13939495335507557, -0.141278668940955, -0.14316238452683439, -0.1450461001127138, -0.14692981569859317, -0.1488135312844726, -0.150697246870352, -0.15258096245623137, -0.15446467804211078, -0.1563483936279902, -0.1582321092138696, -0.16011582479974898, -0.16199954038562842, -0.1638832559715078, -0.16576697155738718, -0.16765068714326659, -0.16953440272914597, -0.1714181183150254, -0.17330183390090478, -0.1751855494867842, -0.17706926507266357, -0.178952980658543, -0.1808366962444224, -0.18272041183030177, -0.18460412741618118, -0.1864878430020606, -0.18837155858794 ], [ 0, -0.0037599970069291293, -0.007519994013858259, -0.011279991020787387, -0.015039988027716517, -0.018799985034645646, -0.022559982041574774, -0.026319979048503906, -0.030079976055433034, -0.033839973062362166, -0.03759997006929129, -0.041359967076220416, -0.04511996408314955, -0.04887996109007869, -0.05263995809700781, -0.05639995510393694, -0.06015995211086607, -0.06391994911779521, -0.06767994612472433, -0.07143994313165346, -0.07519994013858258, -0.07895993714551172, -0.08271993415244083, -0.08647993115936997, -0.0902399281662991, -0.09399992517322824, -0.09775992218015737, -0.10151991918708649, -0.10527991619401562, -0.10903991320094475, -0.11279991020787387, -0.116559907214803, -0.12031990422173214, -0.12407990122866126, -0.12783989823559042, -0.13159989524251953, -0.13535989224944867, -0.13911988925637778, -0.14287988626330692, -0.14663988327023603, -0.15039988027716517, -0.1541598772840943, -0.15791987429102344, -0.16167987129795258, -0.16543986830488167, -0.1691998653118108, -0.17295986231873994, -0.17671985932566908, -0.1804798563325982, -0.18423985333952733, -0.18799985034645647, -0.1917598473533856, -0.19551984436031475, -0.19927984136724383, -0.20303983837417297, -0.2067998353811021, -0.21055983238803125, -0.21431982939496036, -0.2180798264018895, -0.22183982340881864, -0.22559982041574775, -0.2293598174226769, -0.233119814429606, -0.23687981143653514, -0.24063980844346428, -0.24439980545039341, -0.24815980245732253, -0.25191979946425164, -0.25567979647118083, -0.2594397934781099, -0.26319979048503905, -0.2669597874919682, -0.27071978449889733, -0.2744797815058264, -0.27823977851275555, -0.2819997755196847, -0.28575977252661383, -0.28951976953354297, -0.29327976654047205, -0.29703976354740125, -0.30079976055433033, -0.30455975756125947, -0.3083197545681886, -0.3120797515751177, -0.3158397485820469, -0.31959974558897597, -0.32335974259590516, -0.32711973960283425, -0.33087973660976333, -0.3346397336166925, -0.3383997306236216, -0.34215972763055075, -0.3459197246374799, -0.349679721644409, -0.35343971865133816, -0.3571997156582673, -0.3609597126651964, -0.3647197096721255, -0.36847970667905466, -0.3722397036859838, -0.37599970069291294 ], [ 0, -0.0056214394375717425, -0.011242878875143485, -0.01686431831271523, -0.02248575775028697, -0.028107197187858715, -0.03372863662543046, -0.0393500760630022, -0.04497151550057394, -0.05059295493814569, -0.05621439437571743, -0.061835833813289165, -0.06745727325086091, -0.07307871268843266, -0.0787001521260044, -0.08432159156357613, -0.08994303100114788, -0.09556447043871963, -0.10118590987629138, -0.10680734931386311, -0.11242878875143486, -0.11805022818900661, -0.12367166762657833, -0.1292931070641501, -0.13491454650172183, -0.14053598593929356, -0.14615742537686532, -0.15177886481443706, -0.1574003042520088, -0.16302174368958056, -0.16864318312715226, -0.17426462256472403, -0.17988606200229576, -0.18550750143986752, -0.19112894087743926, -0.19675038031501102, -0.20237181975258275, -0.20799325919015446, -0.21361469862772622, -0.21923613806529796, -0.22485757750286972, -0.23047901694044146, -0.23610045637801322, -0.24172189581558495, -0.24734333525315666, -0.2529647746907284, -0.2585862141283002, -0.2642076535658719, -0.26982909300344365, -0.2754505324410154, -0.2810719718785871, -0.2866934113161589, -0.29231485075373065, -0.29793629019130236, -0.3035577296288741, -0.3091791690664458, -0.3148006085040176, -0.32042204794158935, -0.3260434873791611, -0.3316649268167328, -0.3372863662543045, -0.3429078056918763, -0.34852924512944805, -0.3541506845670198, -0.3597721240045915, -0.3653935634421633, -0.37101500287973505, -0.37663644231730675, -0.3822578817548785, -0.3878793211924502, -0.39350076063002204, -0.39912220006759375, -0.4047436395051655, -0.4103650789427372, -0.4159865183803089, -0.42160795781788074, -0.42722939725545245, -0.4328508366930242, -0.4384722761305959, -0.44409371556816774, -0.44971515500573944, -0.45533659444331115, -0.4609580338808829, -0.4665794733184546, -0.47220091275602644, -0.47782235219359814, -0.4834437916311699, -0.4890652310687416, -0.4946866705063133, -0.5003081099438851, -0.5059295493814568, -0.5115509888190286, -0.5171724282566004, -0.5227938676941721, -0.5284153071317438, -0.5340367465693157, -0.5396581860068873, -0.5452796254444591, -0.5509010648820308, -0.5565225043196025, -0.5621439437571742 ], [ 0, -0.00746069661494565, -0.0149213932298913, -0.02238208984483695, -0.0298427864597826, -0.03730348307472825, -0.0447641796896739, -0.05222487630461955, -0.0596855729195652, -0.06714626953451086, -0.0746069661494565, -0.08206766276440215, -0.0895283593793478, -0.09698905599429346, -0.1044497526092391, -0.11191044922418475, -0.1193711458391304, -0.12683184245407605, -0.13429253906902172, -0.14175323568396736, -0.149213932298913, -0.15667462891385867, -0.1641353255288043, -0.17159602214374994, -0.1790567187586956, -0.18651741537364125, -0.19397811198858692, -0.20143880860353255, -0.2088995052184782, -0.21636020183342386, -0.2238208984483695, -0.23128159506331514, -0.2387422916782608, -0.24620298829320647, -0.2536636849081521, -0.2611243815230978, -0.26858507813804344, -0.276045774752989, -0.2835064713679347, -0.29096716798288036, -0.298427864597826, -0.3058885612127717, -0.31334925782771733, -0.32080995444266297, -0.3282706510576086, -0.33573134767255425, -0.3431920442874999, -0.3506527409024456, -0.3581134375173912, -0.36557413413233686, -0.3730348307472825, -0.3804955273622282, -0.38795622397717383, -0.3954169205921194, -0.4028776172070651, -0.41033831382201075, -0.4177990104369564, -0.4252597070519021, -0.4327204036668477, -0.44018110028179336, -0.447641796896739, -0.45510249351168464, -0.4625631901266303, -0.47002388674157597, -0.4774845833565216, -0.48494527997146725, -0.49240597658641294, -0.4998666732013585, -0.5073273698163042, -0.5147880664312499, -0.5222487630461956, -0.5297094596611411, -0.5371701562760869, -0.5446308528910325, -0.552091549505978, -0.5595522461209238, -0.5670129427358694, -0.5744736393508151, -0.5819343359657607, -0.5893950325807065, -0.596855729195652, -0.6043164258105976, -0.6117771224255434, -0.6192378190404889, -0.6266985156554347, -0.6341592122703803, -0.6416199088853259, -0.6490806055002716, -0.6565413021152172, -0.6640019987301629, -0.6714626953451085, -0.6789233919600542, -0.6863840885749998, -0.6938447851899455, -0.7013054818048912, -0.7087661784198368, -0.7162268750347824, -0.7236875716497281, -0.7311482682646737, -0.7386089648796194, -0.746069661494565 ], [ 0, -0.009270509831248431, -0.018541019662496862, -0.027811529493745296, -0.037082039324993724, -0.04635254915624216, -0.05562305898749059, -0.06489356881873902, -0.07416407864998745, -0.08343458848123589, -0.09270509831248432, -0.10197560814373274, -0.11124611797498118, -0.12051662780622961, -0.12978713763747804, -0.13905764746872645, -0.1483281572999749, -0.15759866713122334, -0.16686917696247178, -0.1761396867937202, -0.18541019662496863, -0.19468070645621707, -0.2039512162874655, -0.21322172611871393, -0.22249223594996237, -0.2317627457812108, -0.24103325561245922, -0.25030376544370764, -0.2595742752749561, -0.2688447851062045, -0.2781152949374529, -0.28738580476870135, -0.2966563145999498, -0.30592682443119823, -0.31519733426244667, -0.3244678440936951, -0.33373835392494355, -0.34300886375619194, -0.3522793735874404, -0.3615498834186888, -0.37082039324993726, -0.3800909030811857, -0.38936141291243415, -0.3986319227436826, -0.407902432574931, -0.4171729424061794, -0.42644345223742786, -0.4357139620686763, -0.44498447189992474, -0.4542549817311732, -0.4635254915624216, -0.47279600139367, -0.48206651122491845, -0.49133702105616683, -0.5006075308874153, -0.5098780407186637, -0.5191485505499122, -0.5284190603811606, -0.537689570212409, -0.5469600800436575, -0.5562305898749058, -0.5655010997061543, -0.5747716095374027, -0.5840421193686511, -0.5933126291998996, -0.602583139031148, -0.6118536488623965, -0.6211241586936449, -0.6303946685248933, -0.6396651783561418, -0.6489356881873902, -0.6582061980186387, -0.6674767078498871, -0.6767472176811355, -0.6860177275123839, -0.6952882373436324, -0.7045587471748808, -0.7138292570061293, -0.7230997668373776, -0.7323702766686262, -0.7416407864998745, -0.7509112963311229, -0.7601818061623714, -0.7694523159936197, -0.7787228258248683, -0.7879933356561166, -0.7972638454873652, -0.8065343553186135, -0.815804865149862, -0.8250753749811104, -0.8343458848123588, -0.8436163946436073, -0.8528869044748557, -0.8621574143061042, -0.8714279241373526, -0.880698433968601, -0.8899689437998495, -0.8992394536310978, -0.9085099634623464, -0.9177804732935947, -0.9270509831248432 ], [ 0, -0.01104373658054035, -0.0220874731610807, -0.03313120974162105, -0.0441749463221614, -0.05521868290270175, -0.0662624194832421, -0.07730615606378245, -0.0883498926443228, -0.09939362922486317, -0.1104373658054035, -0.12148110238594384, -0.1325248389664842, -0.14356857554702457, -0.1546123121275649, -0.16565604870810524, -0.1766997852886456, -0.18774352186918597, -0.19878725844972633, -0.20983099503026664, -0.220874731610807, -0.23191846819134737, -0.24296220477188768, -0.25400594135242804, -0.2650496779329684, -0.2760934145135088, -0.28713715109404914, -0.29818088767458945, -0.3092246242551298, -0.3202683608356702, -0.3313120974162105, -0.34235583399675085, -0.3533995705772912, -0.3644433071578316, -0.37548704373837194, -0.3865307803189123, -0.39757451689945267, -0.4086182534799929, -0.4196619900605333, -0.43070572664107365, -0.441749463221614, -0.4527931998021544, -0.46383693638269474, -0.4748806729632351, -0.48592440954377536, -0.4969681461243157, -0.5080118827048561, -0.5190556192853965, -0.5300993558659368, -0.5411430924464772, -0.5521868290270175, -0.5632305656075579, -0.5742743021880983, -0.5853180387686385, -0.5963617753491789, -0.6074055119297193, -0.6184492485102596, -0.6294929850908, -0.6405367216713403, -0.6515804582518807, -0.662624194832421, -0.6736679314129613, -0.6847116679935017, -0.6957554045740421, -0.7067991411545824, -0.7178428777351228, -0.7288866143156632, -0.7399303508962034, -0.7509740874767439, -0.7620178240572841, -0.7730615606378246, -0.7841052972183649, -0.7951490337989053, -0.8061927703794456, -0.8172365069599858, -0.8282802435405263, -0.8393239801210666, -0.850367716701607, -0.8614114532821473, -0.8724551898626878, -0.883498926443228, -0.8945426630237683, -0.9055863996043088, -0.916630136184849, -0.9276738727653895, -0.9387176093459297, -0.9497613459264702, -0.9608050825070105, -0.9718488190875507, -0.9828925556680912, -0.9939362922486314, -1.004980028829172, -1.0160237654097122, -1.0270675019902527, -1.038111238570793, -1.0491549751513334, -1.0601987117318736, -1.0712424483124139, -1.0822861848929544, -1.0933299214734946, -1.104373658054035 ], [ 0, -0.01277337874695218, -0.02554675749390436, -0.03832013624085654, -0.05109351498780872, -0.0638668937347609, -0.07664027248171308, -0.08941365122866526, -0.10218702997561745, -0.11496040872256964, -0.1277337874695218, -0.14050716621647397, -0.15328054496342616, -0.16605392371037836, -0.17882730245733053, -0.1916006812042827, -0.2043740599512349, -0.2171474386981871, -0.22992081744513929, -0.24269419619209143, -0.2554675749390436, -0.2682409536859958, -0.28101433243294793, -0.29378771117990016, -0.3065610899268523, -0.31933446867380455, -0.3321078474207567, -0.3448812261677089, -0.35765460491466106, -0.3704279836616133, -0.3832013624085654, -0.3959747411555176, -0.4087481199024698, -0.421521498649422, -0.4342948773963742, -0.44706825614332635, -0.45984163489027857, -0.4726150136372307, -0.48538839238418285, -0.498161771131135, -0.5109351498780872, -0.5237085286250395, -0.5364819073719916, -0.5492552861189438, -0.5620286648658959, -0.5748020436128481, -0.5875754223598003, -0.6003488011067525, -0.6131221798537047, -0.6258955586006569, -0.6386689373476091, -0.6514423160945613, -0.6642156948415134, -0.6769890735884656, -0.6897624523354178, -0.7025358310823699, -0.7153092098293221, -0.7280825885762743, -0.7408559673232266, -0.7536293460701787, -0.7664027248171308, -0.779176103564083, -0.7919494823110352, -0.8047228610579874, -0.8174962398049396, -0.8302696185518917, -0.843042997298844, -0.8558163760457961, -0.8685897547927484, -0.8813631335397004, -0.8941365122866527, -0.9069098910336049, -0.9196832697805571, -0.9324566485275092, -0.9452300272744614, -0.9580034060214137, -0.9707767847683657, -0.983550163515318, -0.99632354226227, -1.0090969210092224, -1.0218702997561744, -1.0346436785031266, -1.047417057250079, -1.0601904359970309, -1.0729638147439833, -1.0857371934909354, -1.0985105722378876, -1.1112839509848398, -1.1240573297317917, -1.1368307084787441, -1.1496040872256963, -1.1623774659726485, -1.1751508447196006, -1.1879242234665528, -1.200697602213505, -1.2134709809604574, -1.2262443597074093, -1.2390177384543615, -1.2517911172013139, -1.2645644959482658, -1.2773378746952182 ], [ 0, -0.01445261022305146, -0.02890522044610292, -0.043357830669154385, -0.05781044089220584, -0.0722630511152573, -0.08671566133830877, -0.10116827156136023, -0.11562088178441168, -0.13007349200746315, -0.1445261022305146, -0.15897871245356607, -0.17343132267661754, -0.187883932899669, -0.20233654312272045, -0.2167891533457719, -0.23124176356882337, -0.24569437379187484, -0.2601469840149263, -0.27459959423797775, -0.2890522044610292, -0.3035048146840807, -0.31795742490713214, -0.3324100351301836, -0.3468626453532351, -0.3613152555762865, -0.375767865799338, -0.3902204760223894, -0.4046730862454409, -0.4191256964684924, -0.4335783066915438, -0.4480309169145953, -0.46248352713764673, -0.47693613736069823, -0.4913887475837497, -0.5058413578068012, -0.5202939680298526, -0.5347465782529041, -0.5491991884759555, -0.563651798699007, -0.5781044089220584, -0.5925570191451099, -0.6070096293681614, -0.6214622395912128, -0.6359148498142643, -0.6503674600373157, -0.6648200702603672, -0.6792726804834186, -0.6937252907064702, -0.7081779009295216, -0.722630511152573, -0.7370831213756246, -0.751535731598676, -0.7659883418217274, -0.7804409520447788, -0.7948935622678304, -0.8093461724908818, -0.8237987827139333, -0.8382513929369848, -0.8527040031600363, -0.8671566133830876, -0.881609223606139, -0.8960618338291906, -0.910514444052242, -0.9249670542752935, -0.939419664498345, -0.9538722747213965, -0.9683248849444478, -0.9827774951674993, -0.9972301053905508, -1.0116827156136023, -1.0261353258366537, -1.0405879360597052, -1.0550405462827566, -1.0694931565058081, -1.0839457667288597, -1.098398376951911, -1.1128509871749626, -1.127303597398014, -1.1417562076210654, -1.1562088178441168, -1.1706614280671683, -1.1851140382902199, -1.1995666485132712, -1.2140192587363228, -1.228471868959374, -1.2429244791824257, -1.257377089405477, -1.2718296996285285, -1.28628230985158, -1.3007349200746314, -1.315187530297683, -1.3296401405207343, -1.3440927507437859, -1.3585453609668372, -1.372997971189889, -1.3874505814129403, -1.4019031916359916, -1.4163558018590432, -1.4308084120820945, -1.445261022305146 ], [ 0, -0.016074803849369903, -0.032149607698739806, -0.048224411548109705, -0.06429921539747961, -0.08037401924684952, -0.09644882309621941, -0.11252362694558932, -0.12859843079495922, -0.14467323464432913, -0.16074803849369904, -0.17682284234306891, -0.19289764619243882, -0.20897245004180875, -0.22504725389117863, -0.2411220577405485, -0.25719686158991845, -0.2732716654392884, -0.28934646928865826, -0.30542127313802814, -0.32149607698739807, -0.33757088083676795, -0.35364568468613783, -0.36972048853550776, -0.38579529238487764, -0.4018700962342476, -0.4179449000836175, -0.43401970393298733, -0.45009450778235727, -0.4661693116317272, -0.482244115481097, -0.49831891933046696, -0.5143937231798369, -0.5304685270292068, -0.5465433308785768, -0.5626181347279466, -0.5786929385773165, -0.5947677424266864, -0.6108425462760563, -0.6269173501254262, -0.6429921539747961, -0.659066957824166, -0.6751417616735359, -0.6912165655229059, -0.7072913693722757, -0.7233661732216455, -0.7394409770710155, -0.7555157809203854, -0.7715905847697553, -0.7876653886191253, -0.8037401924684952, -0.819814996317865, -0.835889800167235, -0.8519646040166048, -0.8680394078659747, -0.8841142117153447, -0.9001890155647145, -0.9162638194140844, -0.9323386232634544, -0.9484134271128243, -0.964488230962194, -0.980563034811564, -0.9966378386609339, -1.012712642510304, -1.0287874463596738, -1.0448622502090437, -1.0609370540584135, -1.0770118579077834, -1.0930866617571535, -1.1091614656065232, -1.1252362694558933, -1.1413110733052632, -1.157385877154633, -1.173460681004003, -1.1895354848533728, -1.2056102887027427, -1.2216850925521126, -1.2377598964014827, -1.2538347002508523, -1.2699095041002224, -1.2859843079495923, -1.302059111798962, -1.318133915648332, -1.334208719497702, -1.3502835233470718, -1.3663583271964417, -1.3824331310458118, -1.3985079348951814, -1.4145827387445513, -1.4306575425939214, -1.446732346443291, -1.4628071502926612, -1.478881954142031, -1.494956757991401, -1.5110315618407708, -1.527106365690141, -1.5431811695395106, -1.5592559733888804, -1.5753307772382505, -1.5914055810876202, -1.6074803849369903 ], [ 0, -0.0176335575687742, -0.0352671151375484, -0.0529006727063226, -0.0705342302750968, -0.088167787843871, -0.1058013454126452, -0.1234349029814194, -0.1410684605501936, -0.1587020181189678, -0.176335575687742, -0.19396913325651619, -0.2116026908252904, -0.22923624839406462, -0.2468698059628388, -0.26450336353161297, -0.2821369211003872, -0.2997704786691614, -0.3174040362379356, -0.3350375938067098, -0.352671151375484, -0.3703047089442582, -0.38793826651303237, -0.4055718240818066, -0.4232053816505808, -0.440838939219355, -0.45847249678812924, -0.4761060543569034, -0.4937396119256776, -0.5113731694944518, -0.5290067270632259, -0.5466402846320002, -0.5642738422007744, -0.5819073997695486, -0.5995409573383228, -0.6171745149070971, -0.6348080724758712, -0.6524416300446454, -0.6700751876134196, -0.6877087451821938, -0.705342302750968, -0.7229758603197423, -0.7406094178885164, -0.7582429754572907, -0.7758765330260647, -0.7935100905948389, -0.8111436481636132, -0.8287772057323873, -0.8464107633011616, -0.8640443208699358, -0.88167787843871, -0.8993114360074843, -0.9169449935762585, -0.9345785511450325, -0.9522121087138068, -0.969845666282581, -0.9874792238513552, -1.0051127814201295, -1.0227463389889035, -1.0403798965576778, -1.0580134541264519, -1.0756470116952261, -1.0932805692640004, -1.1109141268327747, -1.1285476844015487, -1.146181241970323, -1.1638147995390973, -1.1814483571078713, -1.1990819146766456, -1.2167154722454196, -1.2343490298141941, -1.2519825873829682, -1.2696161449517425, -1.2872497025205165, -1.3048832600892908, -1.322516817658065, -1.340150375226839, -1.3577839327956136, -1.3754174903643877, -1.393051047933162, -1.410684605501936, -1.42831816307071, -1.4459517206394845, -1.4635852782082586, -1.4812188357770328, -1.4988523933458069, -1.5164859509145814, -1.5341195084833554, -1.5517530660521295, -1.5693866236209038, -1.5870201811896778, -1.6046537387584523, -1.6222872963272263, -1.6399208538960006, -1.6575544114647747, -1.6751879690335492, -1.6928215266023232, -1.7104550841710973, -1.7280886417398715, -1.7457221993086458, -1.76335575687742 ], [ 0, -0.0191227196924607, -0.0382454393849214, -0.057368159077382096, -0.0764908787698428, -0.0956135984623035, -0.11473631815476419, -0.1338590378472249, -0.1529817575396856, -0.1721044772321463, -0.191227196924607, -0.21034991661706767, -0.22947263630952838, -0.2485953560019891, -0.2677180756944498, -0.28684079538691043, -0.3059635150793712, -0.3250862347718319, -0.3442089544642926, -0.36333167415675327, -0.382454393849214, -0.4015771135416747, -0.42069983323413535, -0.43982255292659606, -0.45894527261905677, -0.4780679923115175, -0.4971907120039782, -0.5163134316964388, -0.5354361513888996, -0.5545588710813603, -0.5736815907738209, -0.5928043104662817, -0.6119270301587424, -0.6310497498512031, -0.6501724695436638, -0.6692951892361245, -0.6884179089285852, -0.7075406286210458, -0.7266633483135065, -0.7457860680059673, -0.764908787698428, -0.7840315073908887, -0.8031542270833494, -0.8222769467758101, -0.8413996664682707, -0.8605223861607314, -0.8796451058531921, -0.8987678255456528, -0.9178905452381135, -0.9370132649305742, -0.956135984623035, -0.9752587043154957, -0.9943814240079564, -1.0135041437004169, -1.0326268633928777, -1.0517495830853383, -1.070872302777799, -1.08999502247026, -1.1091177421627205, -1.1282404618551813, -1.1473631815476417, -1.1664859012401025, -1.1856086209325634, -1.204731340625024, -1.2238540603174848, -1.2429767800099454, -1.2620994997024062, -1.2812222193948668, -1.3003449390873276, -1.3194676587797882, -1.338590378472249, -1.3577130981647096, -1.3768358178571705, -1.395958537549631, -1.4150812572420917, -1.4342039769345525, -1.453326696627013, -1.472449416319474, -1.4915721360119345, -1.5106948557043953, -1.529817575396856, -1.5489402950893165, -1.5680630147817773, -1.587185734474238, -1.6063084541666988, -1.6254311738591594, -1.6445538935516202, -1.6636766132440808, -1.6827993329365414, -1.7019220526290022, -1.7210447723214628, -1.7401674920139236, -1.7592902117063842, -1.778412931398845, -1.7975356510913056, -1.8166583707837665, -1.835781090476227, -1.8549038101686877, -1.8740265298611485, -1.893149249553609, -1.91227196924607 ], [ 0, -0.02053641317786066, -0.04107282635572132, -0.06160923953358198, -0.08214565271144264, -0.1026820658893033, -0.12321847906716396, -0.14375489224502463, -0.16429130542288528, -0.18482771860074595, -0.2053641317786066, -0.22590054495646725, -0.24643695813432792, -0.2669733713121886, -0.28750978449004927, -0.3080461976679099, -0.32858261084577056, -0.34911902402363126, -0.3696554372014919, -0.39019185037935256, -0.4107282635572132, -0.4312646767350739, -0.4518010899129345, -0.4723375030907952, -0.49287391626865584, -0.5134103294465165, -0.5339467426243772, -0.5544831558022378, -0.5750195689800985, -0.5955559821579592, -0.6160923953358198, -0.6366288085136804, -0.6571652216915411, -0.6777016348694018, -0.6982380480472625, -0.7187744612251232, -0.7393108744029838, -0.7598472875808444, -0.7803837007587051, -0.8009201139365658, -0.8214565271144264, -0.8419929402922871, -0.8625293534701478, -0.8830657666480085, -0.903602179825869, -0.9241385930037297, -0.9446750061815904, -0.9652114193594511, -0.9857478325373117, -1.0062842457151724, -1.026820658893033, -1.0473570720708938, -1.0678934852487545, -1.088429898426615, -1.1089663116044757, -1.1295027247823364, -1.150039137960197, -1.1705755511380578, -1.1911119643159185, -1.211648377493779, -1.2321847906716397, -1.2527212038495004, -1.2732576170273608, -1.2937940302052215, -1.3143304433830822, -1.334866856560943, -1.3554032697388037, -1.3759396829166641, -1.396476096094525, -1.4170125092723855, -1.4375489224502465, -1.458085335628107, -1.4786217488059676, -1.4991581619838283, -1.5196945751616888, -1.5402309883395495, -1.5607674015174102, -1.581303814695271, -1.6018402278731316, -1.6223766410509923, -1.6429130542288528, -1.6634494674067135, -1.6839858805845742, -1.7045222937624347, -1.7250587069402956, -1.745595120118156, -1.766131533296017, -1.7866679464738775, -1.807204359651738, -1.827740772829599, -1.8482771860074594, -1.8688135991853203, -1.8893500123631808, -1.9098864255410415, -1.9304228387189022, -1.950959251896763, -1.9714956650746234, -1.992032078252484, -2.0125684914303448, -2.0331049046082055, -2.053641317786066 ], [ 0, -0.021869058822642347, -0.043738117645284694, -0.06560717646792703, -0.08747623529056939, -0.10934529411321173, -0.13121435293585407, -0.1530834117584964, -0.17495247058113877, -0.19682152940378114, -0.21869058822642345, -0.2405596470490658, -0.26242870587170813, -0.28429776469435053, -0.3061668235169928, -0.32803588233963515, -0.34990494116227755, -0.3717739999849199, -0.3936430588075623, -0.41551211763020457, -0.4373811764528469, -0.4592502352754893, -0.4811192940981316, -0.5029883529207739, -0.5248574117434163, -0.5467264705660586, -0.5685955293887011, -0.5904645882113433, -0.6123336470339856, -0.6342027058566281, -0.6560717646792703, -0.6779408235019126, -0.6998098823245551, -0.7216789411471974, -0.7435479999698398, -0.7654170587924821, -0.7872861176151246, -0.8091551764377667, -0.8310242352604091, -0.8528932940830515, -0.8747623529056938, -0.8966314117283362, -0.9185004705509786, -0.940369529373621, -0.9622385881962632, -0.9841076470189055, -1.0059767058415479, -1.0278457646641903, -1.0497148234868325, -1.071583882309475, -1.0934529411321172, -1.1153219999547597, -1.1371910587774021, -1.1590601176000443, -1.1809291764226866, -1.202798235245329, -1.2246672940679713, -1.2465363528906137, -1.2684054117132562, -1.2902744705358984, -1.3121435293585406, -1.334012588181183, -1.3558816470038253, -1.3777507058264677, -1.3996197646491102, -1.4214888234717524, -1.4433578822943949, -1.465226941117037, -1.4870959999396796, -1.5089650587623218, -1.5308341175849642, -1.5527031764076065, -1.5745722352302491, -1.5964412940528914, -1.6183103528755334, -1.640179411698176, -1.6620484705208183, -1.6839175293434607, -1.705786588166103, -1.7276556469887454, -1.7495247058113876, -1.7713937646340299, -1.7932628234566723, -1.8151318822793145, -1.8370009411019572, -1.8588699999245992, -1.880739058747242, -1.9026081175698841, -1.9244771763925264, -1.9463462352151688, -1.968215294037811, -1.9900843528604535, -2.0119534116830957, -2.033822470505738, -2.0556915293283806, -2.077560588151023, -2.099429646973665, -2.1212987057963075, -2.14316776461895, -2.165036823441592, -2.1869058822642344 ], [ 0, -0.02311539728327368, -0.04623079456654736, -0.06934619184982103, -0.09246158913309473, -0.1155769864163684, -0.13869238369964207, -0.16180778098291576, -0.18492317826618945, -0.20803857554946315, -0.2311539728327368, -0.2542693701160105, -0.27738476739928414, -0.30050016468255786, -0.3236155619658315, -0.3467309592491052, -0.3698463565323789, -0.39296175381565257, -0.4160771510989263, -0.4391925483821999, -0.4623079456654736, -0.4854233429487473, -0.508538740232021, -0.5316541375152947, -0.5547695347985683, -0.577884932081842, -0.6010003293651157, -0.6241157266483893, -0.647231123931663, -0.6703465212149368, -0.6934619184982104, -0.7165773157814841, -0.7396927130647578, -0.7628081103480314, -0.7859235076313051, -0.8090389049145789, -0.8321543021978526, -0.8552696994811261, -0.8783850967643998, -0.9015004940476735, -0.9246158913309472, -0.947731288614221, -0.9708466858974946, -0.9939620831807683, -1.017077480464042, -1.0401928777473155, -1.0633082750305893, -1.086423672313863, -1.1095390695971366, -1.1326544668804104, -1.155769864163684, -1.1788852614469578, -1.2020006587302314, -1.225116056013505, -1.2482314532967786, -1.2713468505800525, -1.294462247863326, -1.3175776451465997, -1.3406930424298735, -1.3638084397131471, -1.3869238369964207, -1.4100392342796944, -1.4331546315629682, -1.4562700288462418, -1.4793854261295156, -1.5025008234127892, -1.5256162206960628, -1.5487316179793365, -1.5718470152626103, -1.594962412545884, -1.6180778098291577, -1.6411932071124313, -1.6643086043957052, -1.6874240016789788, -1.7105393989622522, -1.733654796245526, -1.7567701935287996, -1.7798855908120734, -1.803000988095347, -1.8261163853786209, -1.8492317826618945, -1.8723471799451679, -1.895462577228442, -1.9185779745117153, -1.9416933717949891, -1.9648087690782627, -1.9879241663615366, -2.01103956364481, -2.034154960928084, -2.0572703582113574, -2.080385755494631, -2.103501152777905, -2.1266165500611787, -2.1497319473444523, -2.172847344627726, -2.195962741911, -2.219078139194273, -2.2421935364775467, -2.2653089337608208, -2.2884243310440944, -2.311539728327368 ], [ 0, -0.02427050983124842, -0.04854101966249684, -0.07281152949374525, -0.09708203932499368, -0.12135254915624209, -0.1456230589874905, -0.16989356881873893, -0.19416407864998736, -0.2184345884812358, -0.24270509831248419, -0.2669756081437326, -0.291246117974981, -0.31551662780622947, -0.33978713763747787, -0.36405764746872626, -0.3883281572999747, -0.4125986671312232, -0.4368691769624716, -0.46113968679371997, -0.48541019662496837, -0.5096807064562169, -0.5339512162874652, -0.5582217261187136, -0.582492235949962, -0.6067627457812105, -0.6310332556124589, -0.6553037654437073, -0.6795742752749557, -0.7038447851062042, -0.7281152949374525, -0.752385804768701, -0.7766563145999494, -0.8009268244311979, -0.8251973342624463, -0.8494678440936947, -0.8737383539249431, -0.8980088637561915, -0.9222793735874399, -0.9465498834186883, -0.9708203932499367, -0.9950909030811852, -1.0193614129124338, -1.043631922743682, -1.0679024325749304, -1.0921729424061788, -1.1164434522374271, -1.1407139620686757, -1.164984471899924, -1.1892549817311726, -1.213525491562421, -1.2377960013936695, -1.2620665112249179, -1.2863370210561662, -1.3106075308874146, -1.3348780407186631, -1.3591485505499115, -1.38341906038116, -1.4076895702124084, -1.4319600800436567, -1.456230589874905, -1.4805010997061534, -1.504771609537402, -1.5290421193686503, -1.5533126291998989, -1.5775831390311472, -1.6018536488623958, -1.626124158693644, -1.6503946685248927, -1.6746651783561408, -1.6989356881873894, -1.7232061980186377, -1.7474767078498863, -1.7717472176811346, -1.796017727512383, -1.8202882373436315, -1.8445587471748799, -1.8688292570061285, -1.8930997668373766, -1.9173702766686254, -1.9416407864998735, -1.9659112963311218, -1.9901818061623704, -2.0144523159936187, -2.0387228258248675, -2.0629933356561154, -2.087263845487364, -2.1115343553186126, -2.135804865149861, -2.1600753749811092, -2.1843458848123576, -2.2086163946436064, -2.2328869044748543, -2.257157414306103, -2.2814279241373514, -2.3056984339686, -2.329968943799848, -2.3542394536310964, -2.3785099634623452, -2.4027804732935936, -2.427050983124842 ], [ 0, -0.025329837765060457, -0.050659675530120914, -0.07598951329518137, -0.10131935106024183, -0.1266491888253023, -0.15197902659036275, -0.1773088643554232, -0.20263870212048365, -0.22796853988554414, -0.2532983776506046, -0.278628215415665, -0.3039580531807255, -0.329287890945786, -0.3546177287108464, -0.37994756647590683, -0.4052774042409673, -0.4306072420060278, -0.45593707977108827, -0.4812669175361487, -0.5065967553012092, -0.5319265930662697, -0.55725643083133, -0.5825862685963905, -0.607916106361451, -0.6332459441265115, -0.658575781891572, -0.6839056196566323, -0.7092354574216928, -0.7345652951867533, -0.7598951329518137, -0.7852249707168741, -0.8105548084819346, -0.8358846462469951, -0.8612144840120556, -0.8865443217771161, -0.9118741595421765, -0.9372039973072369, -0.9625338350722974, -0.9878636728373579, -1.0131935106024184, -1.0385233483674787, -1.0638531861325393, -1.0891830238975997, -1.11451286166266, -1.1398426994277204, -1.165172537192781, -1.1905023749578414, -1.215832212722902, -1.2411620504879624, -1.266491888253023, -1.2918217260180835, -1.317151563783144, -1.3424814015482043, -1.3678112393132646, -1.3931410770783252, -1.4184709148433856, -1.4438007526084462, -1.4691305903735066, -1.4944604281385672, -1.5197902659036273, -1.545120103668688, -1.5704499414337483, -1.5957797791988089, -1.6211096169638692, -1.6464394547289298, -1.6717692924939902, -1.6970991302590506, -1.7224289680241112, -1.7477588057891715, -1.7730886435542321, -1.7984184813192925, -1.823748319084353, -1.8490781568494135, -1.8744079946144738, -1.8997378323795344, -1.9250676701445948, -1.9503975079096554, -1.9757273456747158, -2.0010571834397766, -2.0263870212048367, -2.051716858969897, -2.0770466967349575, -2.102376534500018, -2.1277063722650786, -2.153036210030139, -2.1783660477951994, -2.20369588556026, -2.22902572332532, -2.2543555610903807, -2.279685398855441, -2.305015236620502, -2.330345074385562, -2.3556749121506226, -2.381004749915683, -2.406334587680744, -2.431664425445804, -2.456994263210864, -2.4823241009759247, -2.5076539387409853, -2.532983776506046 ], [ 0, -0.026289200401315906, -0.05257840080263181, -0.07886760120394772, -0.10515680160526363, -0.13144600200657952, -0.15773520240789543, -0.18402440280921134, -0.21031360321052725, -0.2366028036118432, -0.26289200401315904, -0.2891812044144749, -0.31547040481579086, -0.3417596052171068, -0.3680488056184227, -0.39433800601973856, -0.4206272064210545, -0.44691640682237044, -0.4732056072236864, -0.4994948076250022, -0.5257840080263181, -0.5520732084276341, -0.5783624088289498, -0.6046516092302658, -0.6309408096315817, -0.6572300100328977, -0.6835192104342136, -0.7098084108355295, -0.7360976112368454, -0.7623868116381614, -0.7886760120394771, -0.8149652124407931, -0.841254412842109, -0.867543613243425, -0.8938328136447409, -0.9201220140460568, -0.9464112144473728, -0.9727004148486885, -0.9989896152500044, -1.0252788156513204, -1.0515680160526362, -1.0778572164539522, -1.1041464168552682, -1.1304356172565841, -1.1567248176578997, -1.1830140180592157, -1.2093032184605317, -1.2355924188618477, -1.2618816192631634, -1.2881708196644794, -1.3144600200657954, -1.3407492204671112, -1.3670384208684272, -1.393327621269743, -1.419616821671059, -1.4459060220723747, -1.4721952224736907, -1.4984844228750067, -1.5247736232763227, -1.5510628236776385, -1.5773520240789543, -1.6036412244802702, -1.6299304248815862, -1.656219625282902, -1.682508825684218, -1.708798026085534, -1.73508722648685, -1.7613764268881655, -1.7876656272894818, -1.8139548276907975, -1.8402440280921135, -1.8665332284934293, -1.8928224288947455, -1.9191116292960613, -1.945400829697377, -1.971690030098693, -1.9979792305000088, -2.024268430901325, -2.050557631302641, -2.076846831703957, -2.1031360321052723, -2.129425232506588, -2.1557144329079043, -2.18200363330922, -2.2082928337105363, -2.234582034111852, -2.2608712345131683, -2.287160434914484, -2.3134496353157994, -2.3397388357171156, -2.3660280361184314, -2.3923172365197476, -2.4186064369210634, -2.4448956373223796, -2.4711848377236953, -2.497474038125011, -2.523763238526327, -2.5500524389276427, -2.576341639328959, -2.6026308397302746, -2.628920040131591 ], [ 0, -0.027144811573980594, -0.05428962314796119, -0.08143443472194178, -0.10857924629592237, -0.13572405786990296, -0.16286886944388357, -0.19001368101786414, -0.21715849259184475, -0.24430330416582535, -0.27144811573980593, -0.2985929273137865, -0.32573773888776714, -0.3528825504617477, -0.3800273620357283, -0.40717217360970887, -0.4343169851836895, -0.46146179675767013, -0.4886066083316507, -0.5157514199056312, -0.5428962314796119, -0.5700410430535925, -0.597185854627573, -0.6243306662015536, -0.6514754777755343, -0.6786202893495148, -0.7057651009234954, -0.732909912497476, -0.7600547240714566, -0.7871995356454372, -0.8143443472194177, -0.8414891587933984, -0.868633970367379, -0.8957787819413596, -0.9229235935153403, -0.9500684050893208, -0.9772132166633014, -1.004358028237282, -1.0315028398112625, -1.0586476513852432, -1.0857924629592237, -1.1129372745332042, -1.140082086107185, -1.1672268976811655, -1.194371709255146, -1.2215165208291265, -1.2486613324031073, -1.2758061439770878, -1.3029509555510685, -1.330095767125049, -1.3572405786990296, -1.3843853902730103, -1.4115302018469909, -1.4386750134209714, -1.465819824994952, -1.4929646365689326, -1.5201094481429132, -1.547254259716894, -1.5743990712908744, -1.6015438828648552, -1.6286886944388355, -1.6558335060128162, -1.6829783175867967, -1.7101231291607772, -1.737267940734758, -1.7644127523087385, -1.7915575638827193, -1.8187023754566995, -1.8458471870306805, -1.8729919986046608, -1.9001368101786416, -1.927281621752622, -1.9544264333266028, -1.9815712449005833, -2.008716056474564, -2.0358608680485446, -2.063005679622525, -2.0901504911965056, -2.1172953027704864, -2.144440114344467, -2.1715849259184474, -2.1987297374924277, -2.2258745490664085, -2.253019360640389, -2.28016417221437, -2.3073089837883503, -2.334453795362331, -2.3615986069363117, -2.388743418510292, -2.415888230084273, -2.443033041658253, -2.470177853232234, -2.4973226648062146, -2.5244674763801953, -2.5516122879541756, -2.5787570995281563, -2.605901911102137, -2.6330467226761174, -2.660191534250098, -2.6873363458240784, -2.714481157398059 ], [ 0, -0.02789329457664754, -0.05578658915329508, -0.08367988372994263, -0.11157317830659016, -0.1394664728832377, -0.16735976745988526, -0.1952530620365328, -0.22314635661318033, -0.2510396511898279, -0.2789329457664754, -0.30682624034312295, -0.3347195349197705, -0.3626128294964181, -0.3905061240730656, -0.41839941864971314, -0.44629271322636066, -0.47418600780300824, -0.5020793023796558, -0.5299725969563033, -0.5578658915329509, -0.5857591861095984, -0.6136524806862459, -0.6415457752628935, -0.669439069839541, -0.6973323644161886, -0.7252256589928362, -0.7531189535694837, -0.7810122481461312, -0.8089055427227788, -0.8367988372994263, -0.8646921318760737, -0.8925854264527213, -0.9204787210293689, -0.9483720156060165, -0.976265310182664, -1.0041586047593116, -1.032051899335959, -1.0599451939126066, -1.0878384884892542, -1.1157317830659017, -1.1436250776425492, -1.1715183722191969, -1.1994116667958443, -1.2273049613724918, -1.2551982559491393, -1.283091550525787, -1.3109848451024344, -1.338878139679082, -1.3667714342557296, -1.3946647288323772, -1.4225580234090247, -1.4504513179856724, -1.4783446125623196, -1.5062379071389673, -1.5341312017156148, -1.5620244962922625, -1.58991779086891, -1.6178110854455576, -1.645704380022205, -1.6735976745988526, -1.7014909691755, -1.7293842637521475, -1.7572775583287952, -1.7851708529054426, -1.8130641474820903, -1.8409574420587378, -1.8688507366353853, -1.896744031212033, -1.9246373257886804, -1.952530620365328, -1.9804239149419756, -2.0083172095186232, -2.0362105040952705, -2.064103798671918, -2.091997093248566, -2.119890387825213, -2.147783682401861, -2.1756769769785085, -2.203570271555156, -2.2314635661318034, -2.2593568607084507, -2.2872501552850983, -2.315143449861746, -2.3430367444383937, -2.370930039015041, -2.3988233335916886, -2.4267166281683363, -2.4546099227449836, -2.4825032173216313, -2.5103965118982785, -2.5382898064749266, -2.566183101051574, -2.5940763956282216, -2.621969690204869, -2.649862984781517, -2.677756279358164, -2.7056495739348114, -2.733542868511459, -2.761436163088107, -2.7893294576647545 ], [ 0, -0.028531695488854605, -0.05706339097770921, -0.08559508646656382, -0.11412678195541842, -0.142658477444273, -0.17119017293312763, -0.19972186842198222, -0.22825356391083684, -0.2567852593996915, -0.285316954888546, -0.3138486503774006, -0.34238034586625526, -0.3709120413551099, -0.39944373684396445, -0.42797543233281904, -0.4565071278216737, -0.48503882331052833, -0.513570518799383, -0.5421022142882375, -0.570633909777092, -0.5991656052659468, -0.6276973007548012, -0.6562289962436559, -0.6847606917325105, -0.7132923872213651, -0.7418240827102198, -0.7703557781990743, -0.7988874736879289, -0.8274191691767836, -0.8559508646656381, -0.8844825601544927, -0.9130142556433474, -0.941545951132202, -0.9700776466210567, -0.9986093421099113, -1.027141037598766, -1.0556727330876203, -1.084204428576475, -1.1127361240653295, -1.141267819554184, -1.169799515043039, -1.1983312105318935, -1.226862906020748, -1.2553946015096025, -1.283926296998457, -1.3124579924873119, -1.3409896879761665, -1.369521383465021, -1.3980530789538756, -1.4265847744427302, -1.4551164699315848, -1.4836481654204396, -1.512179860909294, -1.5407115563981486, -1.5692432518870032, -1.5977749473758578, -1.6263066428647126, -1.6548383383535672, -1.6833700338424218, -1.7119017293312762, -1.7404334248201307, -1.7689651203089853, -1.7974968157978402, -1.8260285112866947, -1.8545602067755493, -1.883091902264404, -1.9116235977532583, -1.9401552932421133, -1.9686869887309677, -1.9972186842198225, -2.025750379708677, -2.054282075197532, -2.082813770686386, -2.1113454661752407, -2.1398771616640953, -2.16840885715295, -2.196940552641805, -2.225472248130659, -2.254003943619514, -2.282535639108368, -2.311067334597223, -2.339599030086078, -2.368130725574932, -2.396662421063787, -2.425194116552641, -2.453725812041496, -2.482257507530351, -2.510789203019205, -2.53932089850806, -2.567852593996914, -2.596384289485769, -2.6249159849746238, -2.6534476804634783, -2.681979375952333, -2.7105110714411875, -2.739042766930042, -2.7675744624188967, -2.7961061579077513, -2.824637853396606, -2.8531695488854605 ], [ 0, -0.029057494833858933, -0.058114989667717866, -0.0871724845015768, -0.11622997933543573, -0.14528747416929466, -0.1743449690031536, -0.20340246383701255, -0.23245995867087146, -0.26151745350473043, -0.2905749483385893, -0.31963244317244827, -0.3486899380063072, -0.37774743284016615, -0.4068049276740251, -0.435862422507884, -0.4649199173417429, -0.4939774121756019, -0.5230349070094609, -0.5520924018433198, -0.5811498966771786, -0.6102073915110376, -0.6392648863448965, -0.6683223811787554, -0.6973798760126144, -0.7264373708464734, -0.7554948656803323, -0.7845523605141912, -0.8136098553480502, -0.8426673501819091, -0.871724845015768, -0.900782339849627, -0.9298398346834859, -0.9588973295173449, -0.9879548243512039, -1.0170123191850629, -1.0460698140189217, -1.0751273088527804, -1.1041848036866395, -1.1332422985204984, -1.1622997933543573, -1.1913572881882164, -1.2204147830220753, -1.2494722778559342, -1.278529772689793, -1.307587267523652, -1.3366447623575108, -1.36570225719137, -1.3947597520252288, -1.4238172468590877, -1.4528747416929468, -1.4819322365268057, -1.5109897313606646, -1.5400472261945235, -1.5691047210283824, -1.5981622158622413, -1.6272197106961004, -1.6562772055299593, -1.6853347003638182, -1.7143921951976773, -1.743449690031536, -1.7725071848653948, -1.801564679699254, -1.8306221745331128, -1.8596796693669717, -1.8887371642008308, -1.9177946590346897, -1.9468521538685484, -1.9759096487024077, -2.0049671435362666, -2.0340246383701257, -2.0630821332039844, -2.0921396280378435, -2.121197122871702, -2.150254617705561, -2.1793121125394204, -2.208369607373279, -2.237427102207138, -2.266484597040997, -2.295542091874856, -2.3245995867087146, -2.3536570815425737, -2.382714576376433, -2.4117720712102915, -2.4408295660441506, -2.4698870608780092, -2.4989445557118684, -2.5280020505457275, -2.557059545379586, -2.5861170402134452, -2.615174535047304, -2.644232029881163, -2.6732895247150217, -2.7023470195488812, -2.73140451438274, -2.760462009216599, -2.7895195040504577, -2.8185769988843163, -2.8476344937181755, -2.8766919885520346, -2.9057494833858937 ], [ 0, -0.02946861752186066, -0.05893723504372132, -0.08840585256558198, -0.11787447008744265, -0.1473430876093033, -0.17681170513116395, -0.20628032265302462, -0.2357489401748853, -0.265217557696746, -0.2946861752186066, -0.3241547927404672, -0.3536234102623279, -0.3830920277841886, -0.41256064530604925, -0.4420292628279099, -0.4714978803497706, -0.5009664978716313, -0.530435115393492, -0.5599037329153526, -0.5893723504372133, -0.618840967959074, -0.6483095854809344, -0.6777782030027951, -0.7072468205246558, -0.7367154380465165, -0.7661840555683772, -0.7956526730902378, -0.8251212906120985, -0.8545899081339592, -0.8840585256558198, -0.9135271431776805, -0.9429957606995412, -0.9724643782214019, -1.0019329957432626, -1.0314016132651231, -1.060870230786984, -1.0903388483088443, -1.1198074658307051, -1.1492760833525657, -1.1787447008744265, -1.208213318396287, -1.237681935918148, -1.2671505534400085, -1.2966191709618688, -1.3260877884837297, -1.3555564060055902, -1.385025023527451, -1.4144936410493116, -1.4439622585711724, -1.473430876093033, -1.5028994936148938, -1.5323681111367544, -1.561836728658615, -1.5913053461804756, -1.6207739637023364, -1.650242581224197, -1.6797111987460578, -1.7091798162679184, -1.738648433789779, -1.7681170513116395, -1.7975856688335001, -1.827054286355361, -1.8565229038772215, -1.8859915213990823, -1.915460138920943, -1.9449287564428037, -1.974397373964664, -2.003865991486525, -2.0333346090083855, -2.0628032265302463, -2.092271844052107, -2.121740461573968, -2.1512090790958283, -2.1806776966176886, -2.2101463141395494, -2.2396149316614102, -2.269083549183271, -2.2985521667051314, -2.328020784226992, -2.357489401748853, -2.3869580192707134, -2.416426636792574, -2.4458952543144346, -2.475363871836296, -2.504832489358156, -2.534301106880017, -2.5637697244018773, -2.5932383419237377, -2.622706959445599, -2.6521755769674593, -2.68164419448932, -2.7111128120111805, -2.7405814295330417, -2.770050047054902, -2.799518664576763, -2.8289872820986233, -2.858455899620484, -2.887924517142345, -2.9173931346642052, -2.946861752186066 ], [ 0, -0.029763441039434336, -0.05952688207886867, -0.08929032311830301, -0.11905376415773734, -0.14881720519717168, -0.17858064623660602, -0.20834408727604034, -0.2381075283154747, -0.26787096935490906, -0.29763441039434335, -0.32739785143377764, -0.35716129247321204, -0.3869247335126464, -0.4166881745520807, -0.446451615591515, -0.4762150566309494, -0.5059784976703837, -0.5357419387098181, -0.5655053797492523, -0.5952688207886867, -0.6250322618281211, -0.6547957028675553, -0.6845591439069897, -0.7143225849464241, -0.7440860259858584, -0.7738494670252928, -0.8036129080647271, -0.8333763491041614, -0.8631397901435958, -0.89290323118303, -0.9226666722224643, -0.9524301132618987, -0.982193554301333, -1.0119569953407674, -1.0417204363802017, -1.0714838774196362, -1.1012473184590703, -1.1310107594985046, -1.1607742005379391, -1.1905376415773734, -1.2203010826168077, -1.2500645236562422, -1.2798279646956765, -1.3095914057351106, -1.339354846774545, -1.3691182878139794, -1.3988817288534137, -1.4286451698928482, -1.4584086109322825, -1.4881720519717168, -1.5179354930111513, -1.5476989340505856, -1.5774623750900196, -1.6072258161294541, -1.6369892571688884, -1.6667526982083227, -1.6965161392477572, -1.7262795802871915, -1.7560430213266258, -1.78580646236606, -1.8155699034054944, -1.8453333444449287, -1.875096785484363, -1.9048602265237975, -1.9346236675632318, -1.964387108602666, -1.9941505496421004, -2.023913990681535, -2.053677431720969, -2.0834408727604035, -2.1132043137998378, -2.1429677548392725, -2.1727311958787063, -2.2024946369181406, -2.2322580779575754, -2.262021518997009, -2.291784960036444, -2.3215484010758782, -2.3513118421153125, -2.381075283154747, -2.410838724194181, -2.4406021652336154, -2.4703656062730497, -2.5001290473124844, -2.5298924883519183, -2.559655929391353, -2.5894193704307873, -2.619182811470221, -2.648946252509656, -2.67870969354909, -2.7084731345885245, -2.7382365756279587, -2.7680000166673935, -2.7977634577068273, -2.827526898746262, -2.8572903397856964, -2.88705378082513, -2.916817221864565, -2.9465806629039992, -2.9763441039434335 ], [ 0, -0.029940801852848146, -0.05988160370569629, -0.08982240555854444, -0.11976320741139258, -0.14970400926424074, -0.17964481111708888, -0.20958561296993702, -0.23952641482278517, -0.26946721667563334, -0.2994080185284815, -0.32934882038132957, -0.35928962223417776, -0.3892304240870259, -0.41917122593987405, -0.44911202779272213, -0.47905282964557033, -0.5089936314984185, -0.5389344333512667, -0.5688752352041148, -0.598816037056963, -0.628756838909811, -0.6586976407626591, -0.6886384426155073, -0.7185792444683555, -0.7485200463212036, -0.7784608481740518, -0.8084016500268999, -0.8383424518797481, -0.8682832537325963, -0.8982240555854443, -0.9281648574382925, -0.9581056592911407, -0.9880464611439889, -1.017987262996837, -1.0479280648496851, -1.0778688667025333, -1.1078096685553813, -1.1377504704082295, -1.1676912722610777, -1.197632074113926, -1.227572875966774, -1.257513677819622, -1.2874544796724703, -1.3173952815253183, -1.3473360833781665, -1.3772768852310147, -1.4072176870838629, -1.437158488936711, -1.4670992907895593, -1.4970400926424072, -1.5269808944952554, -1.5569216963481036, -1.5868624982009516, -1.6168033000537998, -1.646744101906648, -1.6766849037594962, -1.7066257056123444, -1.7365665074651926, -1.7665073093180408, -1.7964481111708885, -1.8263889130237367, -1.856329714876585, -1.8862705167294331, -1.9162113185822813, -1.9461521204351295, -1.9760929222879777, -2.0060337241408255, -2.035974525993674, -2.065915327846522, -2.0958561296993703, -2.1257969315522183, -2.1557377334050667, -2.1856785352579147, -2.2156193371107626, -2.245560138963611, -2.275500940816459, -2.3054417426693075, -2.3353825445221554, -2.365323346375004, -2.395264148227852, -2.4252049500807, -2.455145751933548, -2.4850865537863958, -2.515027355639244, -2.544968157492092, -2.5749089593449406, -2.6048497611977885, -2.6347905630506365, -2.664731364903485, -2.694672166756333, -2.7246129686091813, -2.7545537704620293, -2.7844945723148777, -2.8144353741677257, -2.844376176020574, -2.874316977873422, -2.90425777972627, -2.9341985815791185, -2.9641393834319665, -2.9940801852848145 ], [ 0, -0.03, -0.06, -0.09, -0.12, -0.15, -0.18, -0.21, -0.24, -0.27, -0.3, -0.32999999999999996, -0.36, -0.39, -0.42, -0.44999999999999996, -0.48, -0.51, -0.54, -0.57, -0.6, -0.63, -0.6599999999999999, -0.69, -0.72, -0.75, -0.78, -0.8099999999999999, -0.84, -0.87, -0.8999999999999999, -0.9299999999999999, -0.96, -0.99, -1.02, -1.05, -1.08, -1.1099999999999999, -1.14, -1.17, -1.2, -1.23, -1.26, -1.29, -1.3199999999999998, -1.3499999999999999, -1.38, -1.41, -1.44, -1.47, -1.5, -1.53, -1.56, -1.5899999999999999, -1.6199999999999999, -1.65, -1.68, -1.71, -1.74, -1.77, -1.7999999999999998, -1.8299999999999998, -1.8599999999999999, -1.89, -1.92, -1.95, -1.98, -2.01, -2.04, -2.07, -2.1, -2.13, -2.16, -2.19, -2.2199999999999998, -2.25, -2.28, -2.31, -2.34, -2.37, -2.4, -2.4299999999999997, -2.46, -2.4899999999999998, -2.52, -2.55, -2.58, -2.61, -2.6399999999999997, -2.67, -2.6999999999999997, -2.73, -2.76, -2.79, -2.82, -2.85, -2.88, -2.9099999999999997, -2.94, -2.9699999999999998, -3 ], [ 0, -0.029940801852848146, -0.05988160370569629, -0.08982240555854444, -0.11976320741139258, -0.14970400926424074, -0.17964481111708888, -0.20958561296993702, -0.23952641482278517, -0.26946721667563334, -0.2994080185284815, -0.32934882038132957, -0.35928962223417776, -0.3892304240870259, -0.41917122593987405, -0.44911202779272213, -0.47905282964557033, -0.5089936314984185, -0.5389344333512667, -0.5688752352041148, -0.598816037056963, -0.628756838909811, -0.6586976407626591, -0.6886384426155073, -0.7185792444683555, -0.7485200463212036, -0.7784608481740518, -0.8084016500268999, -0.8383424518797481, -0.8682832537325963, -0.8982240555854443, -0.9281648574382925, -0.9581056592911407, -0.9880464611439889, -1.017987262996837, -1.0479280648496851, -1.0778688667025333, -1.1078096685553813, -1.1377504704082295, -1.1676912722610777, -1.197632074113926, -1.227572875966774, -1.257513677819622, -1.2874544796724703, -1.3173952815253183, -1.3473360833781665, -1.3772768852310147, -1.4072176870838629, -1.437158488936711, -1.4670992907895593, -1.4970400926424072, -1.5269808944952554, -1.5569216963481036, -1.5868624982009516, -1.6168033000537998, -1.646744101906648, -1.6766849037594962, -1.7066257056123444, -1.7365665074651926, -1.7665073093180408, -1.7964481111708885, -1.8263889130237367, -1.856329714876585, -1.8862705167294331, -1.9162113185822813, -1.9461521204351295, -1.9760929222879777, -2.0060337241408255, -2.035974525993674, -2.065915327846522, -2.0958561296993703, -2.1257969315522183, -2.1557377334050667, -2.1856785352579147, -2.2156193371107626, -2.245560138963611, -2.275500940816459, -2.3054417426693075, -2.3353825445221554, -2.365323346375004, -2.395264148227852, -2.4252049500807, -2.455145751933548, -2.4850865537863958, -2.515027355639244, -2.544968157492092, -2.5749089593449406, -2.6048497611977885, -2.6347905630506365, -2.664731364903485, -2.694672166756333, -2.7246129686091813, -2.7545537704620293, -2.7844945723148777, -2.8144353741677257, -2.844376176020574, -2.874316977873422, -2.90425777972627, -2.9341985815791185, -2.9641393834319665, -2.9940801852848145 ], [ 0, -0.029763441039434336, -0.05952688207886867, -0.08929032311830301, -0.11905376415773734, -0.14881720519717168, -0.17858064623660602, -0.20834408727604034, -0.2381075283154747, -0.26787096935490906, -0.29763441039434335, -0.32739785143377764, -0.35716129247321204, -0.3869247335126464, -0.4166881745520807, -0.446451615591515, -0.4762150566309494, -0.5059784976703837, -0.5357419387098181, -0.5655053797492523, -0.5952688207886867, -0.6250322618281211, -0.6547957028675553, -0.6845591439069897, -0.7143225849464241, -0.7440860259858584, -0.7738494670252928, -0.8036129080647271, -0.8333763491041614, -0.8631397901435958, -0.89290323118303, -0.9226666722224643, -0.9524301132618987, -0.982193554301333, -1.0119569953407674, -1.0417204363802017, -1.0714838774196362, -1.1012473184590703, -1.1310107594985046, -1.1607742005379391, -1.1905376415773734, -1.2203010826168077, -1.2500645236562422, -1.2798279646956765, -1.3095914057351106, -1.339354846774545, -1.3691182878139794, -1.3988817288534137, -1.4286451698928482, -1.4584086109322825, -1.4881720519717168, -1.5179354930111513, -1.5476989340505856, -1.5774623750900196, -1.6072258161294541, -1.6369892571688884, -1.6667526982083227, -1.6965161392477572, -1.7262795802871915, -1.7560430213266258, -1.78580646236606, -1.8155699034054944, -1.8453333444449287, -1.875096785484363, -1.9048602265237975, -1.9346236675632318, -1.964387108602666, -1.9941505496421004, -2.023913990681535, -2.053677431720969, -2.0834408727604035, -2.1132043137998378, -2.1429677548392725, -2.1727311958787063, -2.2024946369181406, -2.2322580779575754, -2.262021518997009, -2.291784960036444, -2.3215484010758782, -2.3513118421153125, -2.381075283154747, -2.410838724194181, -2.4406021652336154, -2.4703656062730497, -2.5001290473124844, -2.5298924883519183, -2.559655929391353, -2.5894193704307873, -2.619182811470221, -2.648946252509656, -2.67870969354909, -2.7084731345885245, -2.7382365756279587, -2.7680000166673935, -2.7977634577068273, -2.827526898746262, -2.8572903397856964, -2.88705378082513, -2.916817221864565, -2.9465806629039992, -2.9763441039434335 ], [ 0, -0.029468617521860658, -0.058937235043721316, -0.08840585256558198, -0.11787447008744263, -0.14734308760930329, -0.17681170513116395, -0.2062803226530246, -0.23574894017488526, -0.26521755769674593, -0.29468617521860657, -0.3241547927404672, -0.3536234102623279, -0.38309202778418855, -0.4125606453060492, -0.44202926282790983, -0.4714978803497705, -0.5009664978716312, -0.5304351153934919, -0.5599037329153524, -0.5893723504372131, -0.6188409679590738, -0.6483095854809344, -0.6777782030027951, -0.7072468205246558, -0.7367154380465164, -0.7661840555683771, -0.7956526730902377, -0.8251212906120984, -0.8545899081339591, -0.8840585256558197, -0.9135271431776804, -0.942995760699541, -0.9724643782214017, -1.0019329957432623, -1.0314016132651231, -1.0608702307869837, -1.0903388483088443, -1.119807465830705, -1.1492760833525657, -1.1787447008744263, -1.2082133183962869, -1.2376819359181477, -1.2671505534400083, -1.2966191709618688, -1.3260877884837294, -1.3555564060055902, -1.3850250235274508, -1.4144936410493116, -1.4439622585711722, -1.4734308760930328, -1.5028994936148936, -1.5323681111367542, -1.5618367286586148, -1.5913053461804754, -1.6207739637023362, -1.6502425812241968, -1.6797111987460576, -1.7091798162679182, -1.738648433789779, -1.7681170513116393, -1.7975856688335, -1.8270542863553607, -1.8565229038772213, -1.885991521399082, -1.9154601389209427, -1.9449287564428035, -1.9743973739646639, -2.0038659914865247, -2.0333346090083855, -2.0628032265302463, -2.0922718440521066, -2.1217404615739675, -2.151209079095828, -2.1806776966176886, -2.2101463141395494, -2.23961493166141, -2.2690835491832706, -2.2985521667051314, -2.328020784226992, -2.3574894017488526, -2.386958019270713, -2.4164266367925737, -2.4458952543144346, -2.4753638718362954, -2.5048324893581557, -2.5343011068800165, -2.5637697244018773, -2.5932383419237377, -2.6227069594455985, -2.652175576967459, -2.6816441944893197, -2.7111128120111805, -2.7405814295330413, -2.7700500470549017, -2.7995186645767625, -2.8289872820986233, -2.8584558996204836, -2.8879245171423444, -2.917393134664205, -2.9468617521860656 ], [ 0, -0.02905749483385893, -0.05811498966771786, -0.08717248450157679, -0.11622997933543572, -0.14528747416929466, -0.17434496900315358, -0.20340246383701252, -0.23245995867087144, -0.26151745350473043, -0.2905749483385893, -0.3196324431724482, -0.34868993800630715, -0.37774743284016615, -0.40680492767402504, -0.43586242250788393, -0.46491991734174287, -0.49397741217560187, -0.5230349070094609, -0.5520924018433196, -0.5811498966771786, -0.6102073915110375, -0.6392648863448964, -0.6683223811787554, -0.6973798760126143, -0.7264373708464733, -0.7554948656803323, -0.7845523605141911, -0.8136098553480501, -0.8426673501819091, -0.8717248450157679, -0.9007823398496269, -0.9298398346834857, -0.9588973295173447, -0.9879548243512037, -1.0170123191850626, -1.0460698140189217, -1.0751273088527804, -1.1041848036866393, -1.1332422985204982, -1.1622997933543573, -1.1913572881882162, -1.220414783022075, -1.2494722778559342, -1.2785297726897928, -1.3075872675236517, -1.3366447623575108, -1.3657022571913697, -1.3947597520252286, -1.4238172468590877, -1.4528747416929466, -1.4819322365268055, -1.5109897313606646, -1.5400472261945233, -1.5691047210283822, -1.5981622158622413, -1.6272197106961002, -1.656277205529959, -1.6853347003638182, -1.714392195197677, -1.7434496900315357, -1.7725071848653948, -1.8015646796992537, -1.8306221745331126, -1.8596796693669715, -1.8887371642008306, -1.9177946590346895, -1.9468521538685482, -1.9759096487024075, -2.004967143536266, -2.0340246383701253, -2.063082133203984, -2.0921396280378435, -2.121197122871702, -2.150254617705561, -2.17931211253942, -2.2083696073732786, -2.2374271022071377, -2.2664845970409964, -2.295542091874856, -2.3245995867087146, -2.3536570815425732, -2.3827145763764324, -2.411772071210291, -2.44082956604415, -2.4698870608780092, -2.4989445557118684, -2.528002050545727, -2.5570595453795857, -2.586117040213445, -2.6151745350473035, -2.644232029881163, -2.6732895247150217, -2.702347019548881, -2.7314045143827395, -2.7604620092165986, -2.7895195040504572, -2.8185769988843163, -2.8476344937181755, -2.876691988552034, -2.9057494833858932 ], [ 0, -0.02853169548885461, -0.05706339097770922, -0.08559508646656383, -0.11412678195541844, -0.14265847744427304, -0.17119017293312766, -0.19972186842198225, -0.22825356391083687, -0.2567852593996915, -0.2853169548885461, -0.3138486503774007, -0.3423803458662553, -0.3709120413551099, -0.3994437368439645, -0.4279754323328191, -0.45650712782167374, -0.4850388233105284, -0.513570518799383, -0.5421022142882376, -0.5706339097770922, -0.5991656052659468, -0.6276973007548013, -0.6562289962436559, -0.6847606917325106, -0.7132923872213652, -0.7418240827102198, -0.7703557781990744, -0.798887473687929, -0.8274191691767837, -0.8559508646656382, -0.8844825601544928, -0.9130142556433475, -0.9415459511322021, -0.9700776466210568, -0.9986093421099114, -1.027141037598766, -1.0556727330876203, -1.0842044285764751, -1.1127361240653297, -1.1412678195541843, -1.169799515043039, -1.1983312105318935, -1.2268629060207483, -1.2553946015096027, -1.2839262969984573, -1.3124579924873119, -1.3409896879761665, -1.3695213834650213, -1.3980530789538759, -1.4265847744427305, -1.455116469931585, -1.4836481654204396, -1.5121798609092942, -1.5407115563981488, -1.5692432518870034, -1.597774947375858, -1.6263066428647126, -1.6548383383535674, -1.683370033842422, -1.7119017293312764, -1.740433424820131, -1.7689651203089856, -1.7974968157978404, -1.826028511286695, -1.8545602067755496, -1.8830919022644041, -1.9116235977532585, -1.9401552932421136, -1.968686988730968, -1.9972186842198227, -2.0257503797086773, -2.054282075197532, -2.0828137706863865, -2.1113454661752407, -2.1398771616640957, -2.1684088571529503, -2.196940552641805, -2.2254722481306595, -2.254003943619514, -2.2825356391083687, -2.3110673345972232, -2.339599030086078, -2.3681307255749324, -2.396662421063787, -2.4251941165526416, -2.4537258120414966, -2.482257507530351, -2.5107892030192054, -2.53932089850806, -2.5678525939969146, -2.5963842894857696, -2.6249159849746238, -2.653447680463479, -2.681979375952333, -2.710511071441188, -2.7390427669300426, -2.7675744624188967, -2.7961061579077517, -2.824637853396606, -2.853169548885461 ], [ 0, -0.027893294576647538, -0.055786589153295076, -0.0836798837299426, -0.11157317830659015, -0.13946647288323769, -0.1673597674598852, -0.19525306203653275, -0.2231463566131803, -0.25103965118982785, -0.27893294576647537, -0.3068262403431229, -0.3347195349197704, -0.362612829496418, -0.3905061240730655, -0.41839941864971303, -0.4462927132263606, -0.4741860078030081, -0.5020793023796557, -0.5299725969563032, -0.5578658915329507, -0.5857591861095983, -0.6136524806862458, -0.6415457752628934, -0.6694390698395408, -0.6973323644161884, -0.725225658992836, -0.7531189535694834, -0.781012248146131, -0.8089055427227786, -0.8367988372994261, -0.8646921318760736, -0.8925854264527212, -0.9204787210293687, -0.9483720156060162, -0.9762653101826638, -1.0041586047593114, -1.0320518993359589, -1.0599451939126063, -1.0878384884892538, -1.1157317830659015, -1.143625077642549, -1.1715183722191966, -1.199411666795844, -1.2273049613724916, -1.255198255949139, -1.2830915505257867, -1.3109848451024342, -1.3388781396790816, -1.3667714342557293, -1.3946647288323768, -1.4225580234090245, -1.450451317985672, -1.4783446125623194, -1.5062379071389669, -1.5341312017156146, -1.562024496292262, -1.5899177908689095, -1.6178110854455572, -1.6457043800222046, -1.6735976745988521, -1.7014909691754996, -1.7293842637521473, -1.7572775583287947, -1.7851708529054424, -1.8130641474820899, -1.8409574420587373, -1.8688507366353848, -1.8967440312120325, -1.92463732578868, -1.9525306203653277, -1.9804239149419751, -2.008317209518623, -2.03621050409527, -2.0641037986719177, -2.0919970932485654, -2.1198903878252127, -2.1477836824018604, -2.1756769769785076, -2.2035702715551557, -2.231463566131803, -2.25935686070845, -2.287250155285098, -2.3151434498617456, -2.3430367444383933, -2.3709300390150405, -2.398823333591688, -2.4267166281683354, -2.454609922744983, -2.482503217321631, -2.510396511898278, -2.5382898064749257, -2.5661831010515734, -2.594076395628221, -2.6219696902048684, -2.649862984781516, -2.6777562793581633, -2.705649573934811, -2.7335428685114587, -2.761436163088106, -2.7893294576647536 ], [ 0, -0.027144811573980587, -0.05428962314796117, -0.08143443472194176, -0.10857924629592235, -0.13572405786990294, -0.1628688694438835, -0.19001368101786412, -0.2171584925918447, -0.2443033041658253, -0.2714481157398059, -0.29859292731378645, -0.325737738887767, -0.35288255046174766, -0.38002736203572823, -0.40717217360970875, -0.4343169851836894, -0.46146179675767, -0.4886066083316506, -0.5157514199056311, -0.5428962314796117, -0.5700410430535924, -0.5971858546275729, -0.6243306662015534, -0.651475477775534, -0.6786202893495147, -0.7057651009234953, -0.7329099124974758, -0.7600547240714565, -0.787199535645437, -0.8143443472194175, -0.8414891587933981, -0.8686339703673788, -0.8957787819413594, -0.92292359351534, -0.9500684050893206, -0.9772132166633012, -1.0043580282372817, -1.0315028398112622, -1.0586476513852427, -1.0857924629592235, -1.112937274533204, -1.1400820861071848, -1.1672268976811653, -1.1943717092551458, -1.2215165208291263, -1.2486613324031068, -1.2758061439770876, -1.302950955551068, -1.3300957671250488, -1.3572405786990294, -1.3843853902730099, -1.4115302018469906, -1.438675013420971, -1.4658198249949517, -1.4929646365689322, -1.520109448142913, -1.5472542597168935, -1.574399071290874, -1.6015438828648547, -1.628688694438835, -1.6558335060128158, -1.6829783175867963, -1.710123129160777, -1.7372679407347575, -1.764412752308738, -1.7915575638827188, -1.818702375456699, -1.84584718703068, -1.8729919986046604, -1.9001368101786411, -1.9272816217526216, -1.9544264333266024, -1.981571244900583, -2.0087160564745634, -2.035860868048544, -2.0630056796225245, -2.090150491196505, -2.1172953027704855, -2.1444401143444667, -2.171584925918447, -2.1987297374924273, -2.225874549066408, -2.2530193606403883, -2.2801641722143695, -2.30730898378835, -2.3344537953623306, -2.361598606936311, -2.3887434185102916, -2.4158882300842723, -2.4430330416582526, -2.4701778532322334, -2.4973226648062137, -2.524467476380195, -2.551612287954175, -2.578757099528156, -2.605901911102136, -2.6330467226761165, -2.6601915342500977, -2.687336345824078, -2.7144811573980587 ], [ 0, -0.0262892004013159, -0.0525784008026318, -0.0788676012039477, -0.1051568016052636, -0.1314460020065795, -0.1577352024078954, -0.1840244028092113, -0.2103136032105272, -0.23660280361184313, -0.262892004013159, -0.28918120441447487, -0.3154704048157908, -0.34175960521710674, -0.3680488056184226, -0.39433800601973845, -0.4206272064210544, -0.4469164068223703, -0.47320560722368626, -0.4994948076250021, -0.525784008026318, -0.552073208427634, -0.5783624088289497, -0.6046516092302656, -0.6309408096315816, -0.6572300100328975, -0.6835192104342135, -0.7098084108355293, -0.7360976112368453, -0.7623868116381611, -0.7886760120394769, -0.8149652124407929, -0.8412544128421088, -0.8675436132434248, -0.8938328136447407, -0.9201220140460565, -0.9464112144473725, -0.9727004148486882, -0.9989896152500042, -1.0252788156513202, -1.051568016052636, -1.077857216453952, -1.104146416855268, -1.1304356172565837, -1.1567248176578995, -1.1830140180592155, -1.2093032184605312, -1.2355924188618472, -1.2618816192631632, -1.2881708196644792, -1.314460020065795, -1.340749220467111, -1.367038420868427, -1.3933276212697425, -1.4196168216710585, -1.4459060220723745, -1.4721952224736905, -1.4984844228750063, -1.5247736232763223, -1.5510628236776383, -1.5773520240789538, -1.6036412244802698, -1.6299304248815858, -1.6562196252829016, -1.6825088256842176, -1.7087980260855336, -1.7350872264868495, -1.761376426888165, -1.7876656272894813, -1.813954827690797, -1.840244028092113, -1.8665332284934288, -1.892822428894745, -1.9191116292960606, -1.9454008296973764, -1.9716900300986926, -1.9979792305000084, -2.0242684309013246, -2.0505576313026403, -2.076846831703956, -2.103136032105272, -2.1294252325065877, -2.155714432907904, -2.1820036333092196, -2.208292833710536, -2.2345820341118516, -2.2608712345131674, -2.287160434914483, -2.313449635315799, -2.339738835717115, -2.366028036118431, -2.392317236519747, -2.4186064369210625, -2.4448956373223787, -2.4711848377236945, -2.4974740381250107, -2.5237632385263264, -2.550052438927642, -2.5763416393289584, -2.6026308397302738, -2.62892004013159 ], [ 0, -0.02532983776506045, -0.0506596755301209, -0.07598951329518136, -0.1013193510602418, -0.12664918882530227, -0.15197902659036272, -0.17730886435542315, -0.2026387021204836, -0.22796853988554408, -0.25329837765060453, -0.27862821541566496, -0.30395805318072544, -0.32928789094578587, -0.3546177287108463, -0.3799475664759068, -0.4052774042409672, -0.4306072420060277, -0.45593707977108816, -0.48126691753614853, -0.5065967553012091, -0.5319265930662695, -0.5572564308313299, -0.5825862685963904, -0.6079161063614509, -0.6332459441265113, -0.6585757818915717, -0.6839056196566322, -0.7092354574216926, -0.7345652951867531, -0.7598951329518135, -0.7852249707168739, -0.8105548084819344, -0.8358846462469949, -0.8612144840120554, -0.8865443217771158, -0.9118741595421763, -0.9372039973072366, -0.9625338350722971, -0.9878636728373575, -1.0131935106024181, -1.0385233483674785, -1.063853186132539, -1.0891830238975995, -1.1145128616626598, -1.1398426994277202, -1.1651725371927808, -1.1905023749578412, -1.2158322127229018, -1.2411620504879621, -1.2664918882530225, -1.291821726018083, -1.3171515637831435, -1.3424814015482038, -1.3678112393132644, -1.3931410770783248, -1.4184709148433852, -1.4438007526084458, -1.4691305903735061, -1.4944604281385667, -1.519790265903627, -1.5451201036686875, -1.5704499414337478, -1.5957797791988084, -1.6211096169638688, -1.6464394547289294, -1.6717692924939898, -1.6970991302590501, -1.7224289680241107, -1.747758805789171, -1.7730886435542317, -1.798418481319292, -1.8237483190843526, -1.849078156849413, -1.8744079946144732, -1.899737832379534, -1.9250676701445941, -1.950397507909655, -1.975727345674715, -2.0010571834397757, -2.0263870212048363, -2.0517168589698964, -2.077046696734957, -2.102376534500017, -2.127706372265078, -2.1530362100301383, -2.178366047795199, -2.203695885560259, -2.2290257233253197, -2.2543555610903803, -2.2796853988554404, -2.305015236620501, -2.3303450743855616, -2.355674912150622, -2.3810047499156823, -2.406334587680743, -2.4316644254458035, -2.4569942632108637, -2.4823241009759243, -2.5076539387409844, -2.532983776506045 ], [ 0, -0.02427050983124841, -0.04854101966249682, -0.07281152949374523, -0.09708203932499364, -0.12135254915624205, -0.14562305898749045, -0.16989356881873888, -0.19416407864998728, -0.2184345884812357, -0.2427050983124841, -0.2669756081437325, -0.2912461179749809, -0.31551662780622936, -0.33978713763747775, -0.3640576474687261, -0.38832815729997455, -0.412598667131223, -0.4368691769624714, -0.46113968679371975, -0.4854101966249682, -0.5096807064562167, -0.533951216287465, -0.5582217261187133, -0.5824922359499618, -0.6067627457812103, -0.6310332556124587, -0.655303765443707, -0.6795742752749555, -0.7038447851062039, -0.7281152949374522, -0.7523858047687007, -0.7766563145999491, -0.8009268244311976, -0.825197334262446, -0.8494678440936944, -0.8737383539249428, -0.898008863756191, -0.9222793735874395, -0.946549883418688, -0.9708203932499364, -0.9950909030811848, -1.0193614129124333, -1.0436319227436817, -1.06790243257493, -1.0921729424061783, -1.1164434522374267, -1.1407139620686753, -1.1649844718999236, -1.1892549817311722, -1.2135254915624205, -1.2377960013936689, -1.2620665112249174, -1.2863370210561655, -1.310607530887414, -1.3348780407186625, -1.359148550549911, -1.3834190603811594, -1.4076895702124077, -1.4319600800436563, -1.4562305898749044, -1.480501099706153, -1.5047716095374013, -1.5290421193686499, -1.5533126291998982, -1.5775831390311466, -1.6018536488623951, -1.6261241586936432, -1.650394668524892, -1.6746651783561401, -1.6989356881873887, -1.723206198018637, -1.7474767078498856, -1.771747217681134, -1.796017727512382, -1.8202882373436307, -1.844558747174879, -1.8688292570061276, -1.893099766837376, -1.9173702766686245, -1.9416407864998728, -1.965911296331121, -1.9901818061623695, -2.014452315993618, -2.0387228258248666, -2.0629933356561145, -2.0872638454873633, -2.1115343553186117, -2.13580486514986, -2.1600753749811084, -2.1843458848123567, -2.2086163946436055, -2.2328869044748534, -2.257157414306102, -2.2814279241373505, -2.305698433968599, -2.329968943799847, -2.3542394536310955, -2.3785099634623443, -2.4027804732935922, -2.427050983124841 ], [ 0, -0.02311539728327367, -0.04623079456654734, -0.069346191849821, -0.09246158913309468, -0.11557698641636835, -0.138692383699642, -0.16180778098291568, -0.18492317826618937, -0.20803857554946306, -0.2311539728327367, -0.25426937011601036, -0.277384767399284, -0.30050016468255775, -0.32361556196583136, -0.346730959249105, -0.36984635653237874, -0.3929617538156524, -0.4160771510989261, -0.43919254838219973, -0.4623079456654734, -0.4854233429487471, -0.5085387402320207, -0.5316541375152944, -0.554769534798568, -0.5778849320818418, -0.6010003293651155, -0.6241157266483891, -0.6472311239316627, -0.6703465212149364, -0.69346191849821, -0.7165773157814838, -0.7396927130647575, -0.7628081103480311, -0.7859235076313048, -0.8090389049145785, -0.8321543021978522, -0.8552696994811257, -0.8783850967643995, -0.9015004940476731, -0.9246158913309468, -0.9477312886142205, -0.9708466858974942, -0.9939620831807678, -1.0170774804640415, -1.040192877747315, -1.063308275030589, -1.0864236723138625, -1.109539069597136, -1.13265446688041, -1.1557698641636835, -1.1788852614469572, -1.202000658730231, -1.2251160560135044, -1.2482314532967782, -1.2713468505800518, -1.2944622478633254, -1.3175776451465993, -1.3406930424298729, -1.3638084397131467, -1.38692383699642, -1.410039234279694, -1.4331546315629675, -1.4562700288462411, -1.479385426129515, -1.5025008234127886, -1.5256162206960622, -1.5487316179793358, -1.5718470152626096, -1.5949624125458832, -1.618077809829157, -1.6411932071124307, -1.6643086043957045, -1.6874240016789779, -1.7105393989622515, -1.7336547962455253, -1.756770193528799, -1.7798855908120728, -1.8030009880953461, -1.82611638537862, -1.8492317826618936, -1.8723471799451672, -1.895462577228441, -1.9185779745117144, -1.9416933717949885, -1.9648087690782619, -1.9879241663615357, -2.0110395636448093, -2.034154960928083, -2.0572703582113565, -2.08038575549463, -2.103501152777904, -2.126616550061178, -2.1497319473444514, -2.172847344627725, -2.1959627419109986, -2.219078139194272, -2.242193536477546, -2.26530893376082, -2.288424331044093, -2.311539728327367 ], [ 0, -0.021869058822642347, -0.043738117645284694, -0.06560717646792703, -0.08747623529056939, -0.10934529411321173, -0.13121435293585407, -0.1530834117584964, -0.17495247058113877, -0.19682152940378114, -0.21869058822642345, -0.2405596470490658, -0.26242870587170813, -0.28429776469435053, -0.3061668235169928, -0.32803588233963515, -0.34990494116227755, -0.3717739999849199, -0.3936430588075623, -0.41551211763020457, -0.4373811764528469, -0.4592502352754893, -0.4811192940981316, -0.5029883529207739, -0.5248574117434163, -0.5467264705660586, -0.5685955293887011, -0.5904645882113433, -0.6123336470339856, -0.6342027058566281, -0.6560717646792703, -0.6779408235019126, -0.6998098823245551, -0.7216789411471974, -0.7435479999698398, -0.7654170587924821, -0.7872861176151246, -0.8091551764377667, -0.8310242352604091, -0.8528932940830515, -0.8747623529056938, -0.8966314117283362, -0.9185004705509786, -0.940369529373621, -0.9622385881962632, -0.9841076470189055, -1.0059767058415479, -1.0278457646641903, -1.0497148234868325, -1.071583882309475, -1.0934529411321172, -1.1153219999547597, -1.1371910587774021, -1.1590601176000443, -1.1809291764226866, -1.202798235245329, -1.2246672940679713, -1.2465363528906137, -1.2684054117132562, -1.2902744705358984, -1.3121435293585406, -1.334012588181183, -1.3558816470038253, -1.3777507058264677, -1.3996197646491102, -1.4214888234717524, -1.4433578822943949, -1.465226941117037, -1.4870959999396796, -1.5089650587623218, -1.5308341175849642, -1.5527031764076065, -1.5745722352302491, -1.5964412940528914, -1.6183103528755334, -1.640179411698176, -1.6620484705208183, -1.6839175293434607, -1.705786588166103, -1.7276556469887454, -1.7495247058113876, -1.7713937646340299, -1.7932628234566723, -1.8151318822793145, -1.8370009411019572, -1.8588699999245992, -1.880739058747242, -1.9026081175698841, -1.9244771763925264, -1.9463462352151688, -1.968215294037811, -1.9900843528604535, -2.0119534116830957, -2.033822470505738, -2.0556915293283806, -2.077560588151023, -2.099429646973665, -2.1212987057963075, -2.14316776461895, -2.165036823441592, -2.1869058822642344 ], [ 0, -0.020536413177860646, -0.04107282635572129, -0.061609239533581946, -0.08214565271144258, -0.10268206588930325, -0.12321847906716389, -0.14375489224502452, -0.16429130542288517, -0.18482771860074584, -0.2053641317786065, -0.2259005449564671, -0.24643695813432778, -0.26697337131218846, -0.28750978449004905, -0.3080461976679097, -0.32858261084577034, -0.34911902402363104, -0.3696554372014917, -0.3901918503793523, -0.410728263557213, -0.4312646767350736, -0.4518010899129342, -0.47233750309079486, -0.49287391626865557, -0.5134103294465162, -0.5339467426243769, -0.5544831558022375, -0.5750195689800981, -0.5955559821579588, -0.6160923953358194, -0.6366288085136801, -0.6571652216915407, -0.6777016348694014, -0.6982380480472621, -0.7187744612251227, -0.7393108744029834, -0.7598472875808439, -0.7803837007587046, -0.8009201139365653, -0.821456527114426, -0.8419929402922866, -0.8625293534701473, -0.883065766648008, -0.9036021798258684, -0.9241385930037291, -0.9446750061815897, -0.9652114193594504, -0.9857478325373111, -1.0062842457151717, -1.0268206588930324, -1.0473570720708931, -1.0678934852487538, -1.0884298984266143, -1.108966311604475, -1.1295027247823357, -1.1500391379601962, -1.170575551138057, -1.1911119643159176, -1.2116483774937783, -1.2321847906716388, -1.2527212038494995, -1.2732576170273602, -1.2937940302052209, -1.3143304433830814, -1.334866856560942, -1.3554032697388028, -1.3759396829166632, -1.3964760960945242, -1.4170125092723846, -1.4375489224502453, -1.458085335628106, -1.4786217488059668, -1.4991581619838272, -1.5196945751616877, -1.5402309883395486, -1.5607674015174091, -1.58130381469527, -1.6018402278731305, -1.6223766410509912, -1.642913054228852, -1.6634494674067124, -1.683985880584573, -1.7045222937624336, -1.7250587069402945, -1.745595120118155, -1.766131533296016, -1.7866679464738764, -1.8072043596517369, -1.8277407728295976, -1.8482771860074583, -1.868813599185319, -1.8893500123631795, -1.9098864255410404, -1.9304228387189009, -1.9509592518967616, -1.9714956650746223, -1.9920320782524827, -2.0125684914303434, -2.033104904608204, -2.053641317786065 ], [ 0, -0.01912271969246069, -0.03824543938492138, -0.05736815907738207, -0.07649087876984276, -0.09561359846230344, -0.11473631815476414, -0.13385903784722483, -0.15298175753968551, -0.17210447723214622, -0.19122719692460688, -0.21034991661706756, -0.22947263630952827, -0.24859535600198895, -0.26771807569444966, -0.2868407953869103, -0.30596351507937103, -0.32508623477183174, -0.34420895446429245, -0.36333167415675305, -0.38245439384921376, -0.40157711354167447, -0.4206998332341351, -0.43982255292659583, -0.45894527261905654, -0.4780679923115172, -0.4971907120039779, -0.5163134316964386, -0.5354361513888993, -0.5545588710813599, -0.5736815907738206, -0.5928043104662813, -0.6119270301587421, -0.6310497498512028, -0.6501724695436635, -0.6692951892361242, -0.6884179089285849, -0.7075406286210454, -0.7266633483135061, -0.7457860680059668, -0.7649087876984275, -0.7840315073908882, -0.8031542270833489, -0.8222769467758096, -0.8413996664682702, -0.860522386160731, -0.8796451058531917, -0.8987678255456524, -0.9178905452381131, -0.9370132649305738, -0.9561359846230344, -0.9752587043154951, -0.9943814240079558, -1.0135041437004164, -1.0326268633928772, -1.0517495830853378, -1.0708723027777987, -1.0899950224702593, -1.1091177421627199, -1.1282404618551807, -1.1473631815476413, -1.1664859012401019, -1.1856086209325627, -1.2047313406250233, -1.2238540603174841, -1.2429767800099447, -1.2620994997024055, -1.2812222193948661, -1.300344939087327, -1.3194676587797873, -1.3385903784722484, -1.3577130981647088, -1.3768358178571698, -1.3959585375496302, -1.4150812572420908, -1.4342039769345516, -1.4533266966270122, -1.472449416319473, -1.4915721360119336, -1.5106948557043944, -1.529817575396855, -1.5489402950893156, -1.5680630147817765, -1.587185734474237, -1.6063084541666979, -1.6254311738591585, -1.6445538935516193, -1.66367661324408, -1.6827993329365405, -1.7019220526290013, -1.721044772321462, -1.7401674920139227, -1.7592902117063833, -1.7784129313988442, -1.7975356510913048, -1.8166583707837656, -1.8357810904762262, -1.8549038101686866, -1.8740265298611476, -1.893149249553608, -1.9122719692460688 ], [ 0, -0.017633557568774178, -0.035267115137548356, -0.05290067270632253, -0.07053423027509671, -0.08816778784387089, -0.10580134541264506, -0.12343490298141924, -0.14106846055019343, -0.1587020181189676, -0.17633557568774177, -0.19396913325651594, -0.21160269082529012, -0.2292362483940643, -0.24686980596283847, -0.26450336353161263, -0.28213692110038685, -0.299770478669161, -0.3174040362379352, -0.33503759380670933, -0.35267115137548355, -0.3703047089442577, -0.38793826651303187, -0.40557182408180603, -0.42320538165058025, -0.44083893921935446, -0.4584724967881286, -0.47610605435690273, -0.49373961192567695, -0.5113731694944511, -0.5290067270632253, -0.5466402846319994, -0.5642738422007737, -0.5819073997695479, -0.599540957338322, -0.6171745149070962, -0.6348080724758705, -0.6524416300446445, -0.6700751876134187, -0.6877087451821928, -0.7053423027509671, -0.7229758603197413, -0.7406094178885154, -0.7582429754572897, -0.7758765330260637, -0.7935100905948379, -0.8111436481636121, -0.8287772057323863, -0.8464107633011605, -0.8640443208699347, -0.8816778784387089, -0.8993114360074831, -0.9169449935762573, -0.9345785511450313, -0.9522121087138055, -0.9698456662825797, -0.9874792238513539, -1.0051127814201282, -1.0227463389889022, -1.0403798965576765, -1.0580134541264505, -1.0756470116952248, -1.0932805692639989, -1.1109141268327731, -1.1285476844015474, -1.1461812419703215, -1.1638147995390957, -1.1814483571078698, -1.199081914676644, -1.216715472245418, -1.2343490298141924, -1.2519825873829666, -1.269616144951741, -1.287249702520515, -1.304883260089289, -1.3225168176580633, -1.3401503752268373, -1.3577839327956116, -1.3754174903643857, -1.3930510479331601, -1.4106846055019342, -1.4283181630707082, -1.4459517206394825, -1.4635852782082566, -1.4812188357770308, -1.498852393345805, -1.5164859509145794, -1.5341195084833534, -1.5517530660521275, -1.5693866236209018, -1.5870201811896758, -1.60465373875845, -1.6222872963272241, -1.6399208538959986, -1.6575544114647727, -1.675187969033547, -1.692821526602321, -1.710455084171095, -1.7280886417398693, -1.7457221993086434, -1.7633557568774179 ], [ 0, -0.01607480384936989, -0.03214960769873978, -0.04822441154810967, -0.06429921539747956, -0.08037401924684945, -0.09644882309621934, -0.11252362694558922, -0.1285984307949591, -0.14467323464432902, -0.1607480384936989, -0.17682284234306878, -0.19289764619243868, -0.20897245004180856, -0.22504725389117844, -0.24112205774054832, -0.2571968615899182, -0.2732716654392881, -0.28934646928865804, -0.30542127313802786, -0.3214960769873978, -0.3375708808367677, -0.35364568468613755, -0.36972048853550743, -0.38579529238487736, -0.40187009623424724, -0.4179449000836171, -0.434019703932987, -0.4500945077823569, -0.4661693116317268, -0.48224411548109664, -0.49831891933046657, -0.5143937231798364, -0.5304685270292063, -0.5465433308785762, -0.5626181347279462, -0.5786929385773161, -0.5947677424266858, -0.6108425462760557, -0.6269173501254257, -0.6429921539747956, -0.6590669578241655, -0.6751417616735353, -0.6912165655229052, -0.7072913693722751, -0.723366173221645, -0.7394409770710149, -0.7555157809203847, -0.7715905847697547, -0.7876653886191246, -0.8037401924684945, -0.8198149963178644, -0.8358898001672342, -0.8519646040166041, -0.868039407865974, -0.8841142117153439, -0.9001890155647138, -0.9162638194140837, -0.9323386232634536, -0.9484134271128235, -0.9644882309621933, -0.9805630348115632, -0.9966378386609331, -1.012712642510303, -1.028787446359673, -1.0448622502090428, -1.0609370540584127, -1.0770118579077825, -1.0930866617571524, -1.1091614656065223, -1.1252362694558924, -1.141311073305262, -1.1573858771546321, -1.173460681004002, -1.1895354848533717, -1.2056102887027418, -1.2216850925521114, -1.2377598964014815, -1.2538347002508514, -1.2699095041002213, -1.2859843079495912, -1.3020591117989608, -1.318133915648331, -1.3342087194977008, -1.3502835233470707, -1.3663583271964406, -1.3824331310458104, -1.3985079348951803, -1.4145827387445502, -1.43065754259392, -1.44673234644329, -1.46280715029266, -1.4788819541420297, -1.4949567579913998, -1.5110315618407695, -1.5271063656901396, -1.5431811695395095, -1.5592559733888791, -1.5753307772382492, -1.5914055810876189, -1.607480384936989 ], [ 0, -0.014452610223051459, -0.028905220446102917, -0.04335783066915438, -0.057810440892205835, -0.0722630511152573, -0.08671566133830876, -0.10116827156136021, -0.11562088178441167, -0.13007349200746315, -0.1445261022305146, -0.15897871245356604, -0.1734313226766175, -0.18788393289966898, -0.20233654312272042, -0.21678915334577187, -0.23124176356882334, -0.2456943737918748, -0.2601469840149263, -0.2745995942379777, -0.2890522044610292, -0.30350481468408064, -0.3179574249071321, -0.3324100351301835, -0.346862645353235, -0.3613152555762865, -0.37576786579933796, -0.3902204760223894, -0.40467308624544085, -0.41912569646849235, -0.43357830669154374, -0.44803091691459523, -0.4624835271376467, -0.4769361373606982, -0.4913887475837496, -0.5058413578068011, -0.5202939680298526, -0.534746578252904, -0.5491991884759554, -0.563651798699007, -0.5781044089220584, -0.5925570191451098, -0.6070096293681613, -0.6214622395912128, -0.6359148498142642, -0.6503674600373156, -0.664820070260367, -0.6792726804834186, -0.69372529070647, -0.7081779009295215, -0.722630511152573, -0.7370831213756245, -0.7515357315986759, -0.7659883418217273, -0.7804409520447788, -0.7948935622678303, -0.8093461724908817, -0.8237987827139331, -0.8382513929369847, -0.8527040031600361, -0.8671566133830875, -0.8816092236061389, -0.8960618338291905, -0.9105144440522419, -0.9249670542752934, -0.9394196644983449, -0.9538722747213964, -0.9683248849444477, -0.9827774951674992, -0.9972301053905507, -1.0116827156136021, -1.0261353258366537, -1.0405879360597052, -1.0550405462827566, -1.069493156505808, -1.0839457667288595, -1.0983983769519108, -1.1128509871749623, -1.127303597398014, -1.1417562076210654, -1.1562088178441168, -1.170661428067168, -1.1851140382902197, -1.199566648513271, -1.2140192587363225, -1.2284718689593739, -1.2429244791824257, -1.257377089405477, -1.2718296996285283, -1.2862823098515799, -1.3007349200746312, -1.3151875302976828, -1.329640140520734, -1.3440927507437859, -1.3585453609668372, -1.3729979711898888, -1.38745058141294, -1.4019031916359914, -1.416355801859043, -1.4308084120820943, -1.445261022305146 ], [ 0, -0.012773378746952165, -0.02554675749390433, -0.0383201362408565, -0.05109351498780866, -0.06386689373476083, -0.076640272481713, -0.08941365122866517, -0.10218702997561732, -0.1149604087225695, -0.12773378746952166, -0.14050716621647383, -0.153280544963426, -0.16605392371037817, -0.17882730245733033, -0.19160068120428247, -0.20437405995123464, -0.21714743869818684, -0.229920817445139, -0.24269419619209115, -0.2554675749390433, -0.2682409536859955, -0.28101433243294766, -0.2937877111798998, -0.306561089926852, -0.31933446867380416, -0.33210784742075633, -0.34488122616770844, -0.35765460491466067, -0.37042798366161284, -0.38320136240856495, -0.3959747411555171, -0.4087481199024693, -0.4215214986494215, -0.4342948773963737, -0.44706825614332585, -0.459841634890278, -0.47261501363723013, -0.4853883923841823, -0.49816177113113447, -0.5109351498780866, -0.5237085286250388, -0.536481907371991, -0.5492552861189431, -0.5620286648658953, -0.5748020436128475, -0.5875754223597996, -0.6003488011067518, -0.613122179853704, -0.6258955586006562, -0.6386689373476083, -0.6514423160945605, -0.6642156948415127, -0.6769890735884647, -0.6897624523354169, -0.7025358310823692, -0.7153092098293213, -0.7280825885762735, -0.7408559673232257, -0.7536293460701778, -0.7664027248171299, -0.7791761035640821, -0.7919494823110342, -0.8047228610579864, -0.8174962398049386, -0.8302696185518909, -0.843042997298843, -0.8558163760457951, -0.8685897547927474, -0.8813631335396994, -0.8941365122866517, -0.9069098910336038, -0.919683269780556, -0.9324566485275081, -0.9452300272744603, -0.9580034060214124, -0.9707767847683646, -0.9835501635153169, -0.9963235422622689, -1.009096921009221, -1.0218702997561733, -1.0346436785031254, -1.0474170572500776, -1.0601904359970298, -1.072963814743982, -1.0857371934909341, -1.0985105722378863, -1.1112839509848385, -1.1240573297317906, -1.1368307084787428, -1.149604087225695, -1.1623774659726471, -1.1751508447195993, -1.1879242234665515, -1.2006976022135036, -1.2134709809604558, -1.226244359707408, -1.23901773845436, -1.2517911172013123, -1.2645644959482645, -1.2773378746952166 ], [ 0, -0.011043736580540333, -0.022087473161080666, -0.033131209741621, -0.04417494632216133, -0.05521868290270167, -0.066262419483242, -0.07730615606378234, -0.08834989264432266, -0.09939362922486301, -0.11043736580540334, -0.12148110238594366, -0.132524838966484, -0.14356857554702435, -0.15461231212756468, -0.165656048708105, -0.17669978528864533, -0.1877435218691857, -0.19878725844972603, -0.20983099503026634, -0.22087473161080667, -0.231918468191347, -0.24296220477188732, -0.25400594135242766, -0.265049677932968, -0.27609341451350833, -0.2871371510940487, -0.298180887674589, -0.30922462425512937, -0.3202683608356697, -0.33131209741621, -0.34235583399675035, -0.35339957057729066, -0.364443307157831, -0.3754870437383714, -0.3865307803189117, -0.39757451689945206, -0.4086182534799923, -0.4196619900605327, -0.430705726641073, -0.44174946322161335, -0.4527931998021537, -0.463836936382694, -0.4748806729632344, -0.48592440954377464, -0.496968146124315, -0.5080118827048553, -0.5190556192853957, -0.530099355865936, -0.5411430924464764, -0.5521868290270167, -0.563230565607557, -0.5742743021880974, -0.5853180387686376, -0.596361775349178, -0.6074055119297184, -0.6184492485102587, -0.629492985090799, -0.6405367216713393, -0.6515804582518797, -0.66262419483242, -0.6736679314129603, -0.6847116679935007, -0.6957554045740411, -0.7067991411545813, -0.7178428777351217, -0.728886614315662, -0.7399303508962023, -0.7509740874767428, -0.762017824057283, -0.7730615606378234, -0.7841052972183636, -0.7951490337989041, -0.8061927703794444, -0.8172365069599846, -0.8282802435405251, -0.8393239801210653, -0.8503677167016057, -0.861411453282146, -0.8724551898626864, -0.8834989264432267, -0.894542663023767, -0.9055863996043074, -0.9166301361848477, -0.927673872765388, -0.9387176093459283, -0.9497613459264688, -0.960805082507009, -0.9718488190875493, -0.9828925556680898, -0.99393629224863, -1.0049800288291704, -1.0160237654097106, -1.027067501990251, -1.0381112385707914, -1.0491549751513318, -1.060198711731872, -1.0712424483124123, -1.0822861848929528, -1.093329921473493, -1.1043736580540333 ], [ 0, -0.009270509831248403, -0.018541019662496806, -0.02781152949374521, -0.03708203932499361, -0.04635254915624202, -0.05562305898749042, -0.06489356881873883, -0.07416407864998723, -0.08343458848123564, -0.09270509831248404, -0.10197560814373242, -0.11124611797498084, -0.12051662780622925, -0.12978713763747765, -0.13905764746872604, -0.14832815729997445, -0.15759866713122286, -0.16686917696247128, -0.17613968679371966, -0.18541019662496808, -0.19468070645621646, -0.20395121628746485, -0.21322172611871326, -0.22249223594996168, -0.2317627457812101, -0.2410332556124585, -0.25030376544370686, -0.2595742752749553, -0.2688447851062037, -0.2781152949374521, -0.2873858047687005, -0.2966563145999489, -0.30592682443119734, -0.3151973342624457, -0.3244678440936941, -0.33373835392494255, -0.3430088637561909, -0.3522793735874393, -0.3615498834186877, -0.37082039324993615, -0.38009090308118454, -0.3893614129124329, -0.39863192274368137, -0.4079024325749297, -0.41717294240617814, -0.4264434522374265, -0.43571396206867496, -0.44498447189992335, -0.4542549817311718, -0.4635254915624202, -0.47279600139366856, -0.482066511224917, -0.49133702105616534, -0.5006075308874137, -0.5098780407186622, -0.5191485505499106, -0.528419060381159, -0.5376895702124074, -0.5469600800436558, -0.5562305898749041, -0.5655010997061526, -0.574771609537401, -0.5840421193686494, -0.5933126291998978, -0.6025831390311462, -0.6118536488623947, -0.621124158693643, -0.6303946685248915, -0.6396651783561398, -0.6489356881873882, -0.6582061980186367, -0.6674767078498851, -0.6767472176811334, -0.6860177275123818, -0.6952882373436302, -0.7045587471748787, -0.7138292570061271, -0.7230997668373754, -0.7323702766686239, -0.7416407864998723, -0.7509112963311206, -0.7601818061623691, -0.7694523159936174, -0.7787228258248658, -0.7879933356561143, -0.7972638454873627, -0.8065343553186111, -0.8158048651498594, -0.825075374981108, -0.8343458848123563, -0.8436163946436047, -0.852886904474853, -0.8621574143061015, -0.8714279241373499, -0.8806984339685984, -0.8899689437998467, -0.899239453631095, -0.9085099634623436, -0.9177804732935919, -0.9270509831248404 ], [ 0, -0.0074606966149456345, -0.014921393229891269, -0.022382089844836903, -0.029842786459782538, -0.03730348307472817, -0.044764179689673805, -0.052224876304619444, -0.059685572919565076, -0.06714626953451072, -0.07460696614945635, -0.08206766276440197, -0.08952835937934761, -0.09698905599429325, -0.10444975260923889, -0.11191044922418451, -0.11937114583913015, -0.1268318424540758, -0.13429253906902144, -0.14175323568396705, -0.1492139322989127, -0.15667462891385833, -0.16413532552880394, -0.17159602214374958, -0.17905671875869522, -0.18651741537364086, -0.1939781119885865, -0.2014388086035321, -0.20889950521847778, -0.21636020183342342, -0.22382089844836903, -0.23128159506331467, -0.2387422916782603, -0.24620298829320594, -0.2536636849081516, -0.26112438152309725, -0.2685850781380429, -0.27604577475298847, -0.2835064713679341, -0.29096716798287975, -0.2984278645978254, -0.305888561212771, -0.31334925782771667, -0.3208099544426623, -0.3282706510576079, -0.3357313476725535, -0.34319204428749917, -0.3506527409024448, -0.35811343751739044, -0.3655741341323361, -0.3730348307472817, -0.38049552736222736, -0.387956223977173, -0.3954169205921186, -0.4028776172070642, -0.41033831382200986, -0.41779901043695555, -0.4252597070519012, -0.43272040366684683, -0.44018110028179247, -0.44764179689673805, -0.4551024935116837, -0.46256319012662933, -0.47002388674157497, -0.4774845833565206, -0.48494527997146625, -0.4924059765864119, -0.49986667320135747, -0.5073273698163032, -0.5147880664312487, -0.5222487630461945, -0.52970945966114, -0.5371701562760858, -0.5446308528910313, -0.5520915495059769, -0.5595522461209226, -0.5670129427358682, -0.5744736393508139, -0.5819343359657595, -0.5893950325807051, -0.5968557291956508, -0.6043164258105963, -0.611777122425542, -0.6192378190404876, -0.6266985156554333, -0.6341592122703789, -0.6416199088853246, -0.6490806055002701, -0.6565413021152158, -0.6640019987301615, -0.671462695345107, -0.6789233919600528, -0.6863840885749983, -0.6938447851899441, -0.7013054818048896, -0.7087661784198354, -0.7162268750347809, -0.7236875716497265, -0.7311482682646722, -0.7386089648796178, -0.7460696614945634 ], [ 0, -0.00562143943757174, -0.01124287887514348, -0.01686431831271522, -0.02248575775028696, -0.0281071971878587, -0.03372863662543044, -0.039350076063002185, -0.04497151550057392, -0.05059295493814567, -0.0562143943757174, -0.06183583381328914, -0.06745727325086089, -0.07307871268843263, -0.07870015212600437, -0.0843215915635761, -0.08994303100114784, -0.09556447043871959, -0.10118590987629134, -0.10680734931386306, -0.1124287887514348, -0.11805022818900655, -0.12367166762657827, -0.12929310706415, -0.13491454650172177, -0.1405359859392935, -0.14615742537686527, -0.15177886481443698, -0.15740030425200874, -0.16302174368958047, -0.1686431831271522, -0.17426462256472394, -0.17988606200229568, -0.18550750143986744, -0.19112894087743917, -0.19675038031501094, -0.20237181975258267, -0.20799325919015438, -0.2136146986277261, -0.21923613806529788, -0.2248575775028696, -0.23047901694044134, -0.2361004563780131, -0.24172189581558484, -0.24734333525315655, -0.2529647746907283, -0.2585862141283, -0.2642076535658718, -0.26982909300344354, -0.2754505324410153, -0.281071971878587, -0.2866934113161588, -0.29231485075373054, -0.29793629019130224, -0.30355772962887395, -0.3091791690664457, -0.3148006085040175, -0.3204220479415892, -0.32604348737916095, -0.3316649268167327, -0.3372863662543044, -0.3429078056918761, -0.3485292451294479, -0.35415068456701965, -0.35977212400459135, -0.3653935634421631, -0.3710150028797349, -0.3766364423173066, -0.38225788175487835, -0.38787932119245006, -0.3935007606300219, -0.3991222000675936, -0.40474363950516534, -0.41036507894273705, -0.41598651838030876, -0.4216079578178805, -0.4272293972554522, -0.43285083669302404, -0.43847227613059575, -0.4440937155681675, -0.4497151550057392, -0.4553365944433109, -0.4609580338808827, -0.4665794733184544, -0.4722009127560262, -0.4778223521935979, -0.4834437916311697, -0.4890652310687414, -0.4946866705063131, -0.5003081099438849, -0.5059295493814566, -0.5115509888190284, -0.5171724282566, -0.5227938676941719, -0.5284153071317436, -0.5340367465693153, -0.5396581860068871, -0.5452796254444587, -0.5509010648820306, -0.5565225043196023, -0.562143943757174 ], [ 0, -0.0037599970069291133, -0.0075199940138582265, -0.01127999102078734, -0.015039988027716453, -0.018799985034645566, -0.02255998204157468, -0.026319979048503795, -0.030079976055432906, -0.03383997306236203, -0.03759997006929113, -0.04135996707622024, -0.04511996408314936, -0.04887996109007848, -0.05263995809700759, -0.0563999551039367, -0.06015995211086581, -0.06391994911779493, -0.06767994612472406, -0.07143994313165315, -0.07519994013858226, -0.07895993714551139, -0.08271993415244049, -0.08647993115936961, -0.09023992816629872, -0.09399992517322783, -0.09775992218015696, -0.10151991918708606, -0.10527991619401518, -0.10903991320094429, -0.1127999102078734, -0.11655990721480251, -0.12031990422173162, -0.12407990122866075, -0.12783989823558986, -0.13159989524251897, -0.1353598922494481, -0.1391198892563772, -0.1428798862633063, -0.14663988327023542, -0.15039988027716453, -0.15415987728409367, -0.15791987429102278, -0.1616798712979519, -0.16543986830488097, -0.16919986531181008, -0.17295986231873922, -0.17671985932566833, -0.18047985633259744, -0.18423985333952655, -0.18799985034645567, -0.1917598473533848, -0.19551984436031392, -0.199279841367243, -0.2030398383741721, -0.20679983538110125, -0.21055983238803036, -0.21431982939495947, -0.21807982640188858, -0.2218398234088177, -0.2255998204157468, -0.22935981742267592, -0.23311981442960503, -0.23687981143653414, -0.24063980844346325, -0.2443998054503924, -0.2481598024573215, -0.2519197994642506, -0.2556797964711797, -0.2594397934781088, -0.26319979048503794, -0.266959787491967, -0.2707197844988962, -0.2744797815058253, -0.2782397785127544, -0.2819997755196835, -0.2857597725266126, -0.28951976953354175, -0.29327976654047083, -0.29703976354739997, -0.30079976055432905, -0.3045597575612582, -0.30831975456818733, -0.3120797515751164, -0.31583974858204555, -0.31959974558897464, -0.3233597425959038, -0.32711973960283286, -0.33087973660976194, -0.3346397336166911, -0.33839973062362017, -0.34215972763054936, -0.34591972463747844, -0.3496797216444076, -0.35343971865133667, -0.3571997156582658, -0.3609597126651949, -0.36471970967212397, -0.3684797066790531, -0.3722397036859822, -0.37599970069291133 ], [ 0, -0.0018837155858793977, -0.0037674311717587955, -0.005651146757638194, -0.007534862343517591, -0.009418577929396989, -0.011302293515276387, -0.013186009101155785, -0.015069724687035182, -0.016953440272914583, -0.018837155858793978, -0.020720871444673373, -0.022604587030552774, -0.024488302616432173, -0.02637201820231157, -0.028255733788190966, -0.030139449374070364, -0.032023164959949765, -0.03390688054582917, -0.035790596131708555, -0.037674311717587956, -0.03955802730346736, -0.041441742889346746, -0.04332545847522615, -0.04520917406110555, -0.04709288964698495, -0.048976605232864345, -0.05086032081874374, -0.05274403640462314, -0.054627751990502536, -0.05651146757638193, -0.05839518316226133, -0.06027889874814073, -0.06216261433402013, -0.06404632991989953, -0.06593004550577893, -0.06781376109165833, -0.06969747667753771, -0.07158119226341711, -0.07346490784929652, -0.07534862343517591, -0.07723233902105531, -0.07911605460693472, -0.08099977019281411, -0.08288348577869349, -0.0847672013645729, -0.0866509169504523, -0.08853463253633169, -0.0904183481222111, -0.09230206370809049, -0.0941857792939699, -0.0960694948798493, -0.09795321046572869, -0.09983692605160809, -0.10172064163748748, -0.10360435722336687, -0.10548807280924628, -0.10737178839512568, -0.10925550398100507, -0.11113921956688448, -0.11302293515276386, -0.11490665073864326, -0.11679036632452267, -0.11867408191040206, -0.12055779749628145, -0.12244151308216086, -0.12432522866804026, -0.12620894425391965, -0.12809265983979906, -0.12997637542567844, -0.13186009101155785, -0.13374380659743723, -0.13562752218331667, -0.13751123776919605, -0.13939495335507543, -0.14127866894095484, -0.14316238452683422, -0.14504610011271363, -0.14692981569859304, -0.14881353128447244, -0.15069724687035183, -0.1525809624562312, -0.15446467804211061, -0.15634839362799002, -0.15823210921386943, -0.1601158247997488, -0.16199954038562822, -0.1638832559715076, -0.16576697155738698, -0.16765068714326642, -0.1695344027291458, -0.1714181183150252, -0.1733018339009046, -0.175185549486784, -0.17706926507266338, -0.17895298065854282, -0.1808366962444222, -0.18272041183030158, -0.18460412741618099, -0.18648784300206037, -0.1883715585879398 ], [ 0, -7.347880794884118e-18, -1.4695761589768237e-17, -2.2043642384652358e-17, -2.9391523179536474e-17, -3.6739403974420595e-17, -4.4087284769304716e-17, -5.143516556418883e-17, -5.878304635907295e-17, -6.613092715395707e-17, -7.347880794884119e-17, -8.08266887437253e-17, -8.817456953860943e-17, -9.552245033349355e-17, -1.0287033112837766e-16, -1.1021821192326178e-16, -1.175660927181459e-16, -1.2491397351303002e-16, -1.3226185430791415e-16, -1.3960973510279825e-16, -1.4695761589768238e-16, -1.543054966925665e-16, -1.616533774874506e-16, -1.6900125828233474e-16, -1.7634913907721887e-16, -1.8369701987210297e-16, -1.910449006669871e-16, -1.983927814618712e-16, -2.0574066225675533e-16, -2.1308854305163945e-16, -2.2043642384652356e-16, -2.277843046414077e-16, -2.351321854362918e-16, -2.4248006623117594e-16, -2.4982794702606004e-16, -2.571758278209442e-16, -2.645237086158283e-16, -2.718715894107124e-16, -2.792194702055965e-16, -2.8656735100048066e-16, -2.9391523179536476e-16, -3.0126311259024886e-16, -3.08610993385133e-16, -3.159588741800171e-16, -3.233067549749012e-16, -3.306546357697853e-16, -3.380025165646695e-16, -3.453503973595536e-16, -3.5269827815443773e-16, -3.6004615894932183e-16, -3.6739403974420594e-16, -3.747419205390901e-16, -3.820898013339742e-16, -3.894376821288583e-16, -3.967855629237424e-16, -4.0413344371862655e-16, -4.1148132451351065e-16, -4.188292053083948e-16, -4.261770861032789e-16, -4.3352496689816306e-16, -4.408728476930471e-16, -4.482207284879312e-16, -4.555686092828154e-16, -4.629164900776995e-16, -4.702643708725836e-16, -4.776122516674677e-16, -4.849601324623519e-16, -4.923080132572359e-16, -4.996558940521201e-16, -5.070037748470041e-16, -5.143516556418884e-16, -5.216995364367724e-16, -5.290474172316566e-16, -5.363952980265407e-16, -5.437431788214248e-16, -5.51091059616309e-16, -5.58438940411193e-16, -5.657868212060772e-16, -5.731347020009613e-16, -5.804825827958455e-16, -5.878304635907295e-16, -5.951783443856136e-16, -6.025262251804977e-16, -6.098741059753819e-16, -6.17221986770266e-16, -6.245698675651501e-16, -6.319177483600342e-16, -6.392656291549184e-16, -6.466135099498024e-16, -6.539613907446866e-16, -6.613092715395706e-16, -6.686571523344548e-16, -6.76005033129339e-16, -6.833529139242231e-16, -6.907007947191072e-16, -6.980486755139913e-16, -7.053965563088755e-16, -7.127444371037595e-16, -7.200923178986437e-16, -7.274401986935277e-16, -7.347880794884119e-16 ] ], "z": [ [ 0, 0.03, 0.06, 0.09, 0.12, 0.15, 0.18, 0.21, 0.24, 0.27, 0.3, 0.32999999999999996, 0.36, 0.39, 0.42, 0.44999999999999996, 0.48, 0.51, 0.54, 0.57, 0.6, 0.63, 0.6599999999999999, 0.69, 0.72, 0.75, 0.78, 0.8099999999999999, 0.84, 0.87, 0.8999999999999999, 0.9299999999999999, 0.96, 0.99, 1.02, 1.05, 1.08, 1.1099999999999999, 1.14, 1.17, 1.2, 1.23, 1.26, 1.29, 1.3199999999999998, 1.3499999999999999, 1.38, 1.41, 1.44, 1.47, 1.5, 1.53, 1.56, 1.5899999999999999, 1.6199999999999999, 1.65, 1.68, 1.71, 1.74, 1.77, 1.7999999999999998, 1.8299999999999998, 1.8599999999999999, 1.89, 1.92, 1.95, 1.98, 2.01, 2.04, 2.07, 2.1, 2.13, 2.16, 2.19, 2.2199999999999998, 2.25, 2.28, 2.31, 2.34, 2.37, 2.4, 2.4299999999999997, 2.46, 2.4899999999999998, 2.52, 2.55, 2.58, 2.61, 2.6399999999999997, 2.67, 2.6999999999999997, 2.73, 2.76, 2.79, 2.82, 2.85, 2.88, 2.9099999999999997, 2.94, 2.9699999999999998, 3 ], [ 0, 0.03, 0.06, 0.09, 0.12, 0.15, 0.18, 0.21, 0.24, 0.27, 0.3, 0.32999999999999996, 0.36, 0.39, 0.42, 0.4499999999999999, 0.48, 0.5099999999999999, 0.54, 0.57, 0.6, 0.63, 0.6599999999999999, 0.69, 0.72, 0.7499999999999999, 0.78, 0.8099999999999999, 0.84, 0.8700000000000001, 0.8999999999999998, 0.9299999999999998, 0.96, 0.99, 1.0199999999999998, 1.05, 1.08, 1.1099999999999999, 1.14, 1.17, 1.2, 1.23, 1.26, 1.29, 1.3199999999999998, 1.3499999999999999, 1.38, 1.41, 1.44, 1.4700000000000002, 1.4999999999999998, 1.5299999999999998, 1.56, 1.5899999999999999, 1.6199999999999999, 1.65, 1.68, 1.71, 1.7400000000000002, 1.77, 1.7999999999999996, 1.8299999999999998, 1.8599999999999997, 1.89, 1.92, 1.95, 1.98, 2.0099999999999993, 2.0399999999999996, 2.07, 2.1, 2.13, 2.16, 2.19, 2.2199999999999998, 2.25, 2.28, 2.31, 2.34, 2.37, 2.4, 2.4299999999999997, 2.46, 2.4899999999999993, 2.52, 2.5499999999999994, 2.58, 2.61, 2.6399999999999997, 2.67, 2.6999999999999997, 2.73, 2.76, 2.79, 2.82, 2.8500000000000005, 2.88, 2.91, 2.9400000000000004, 2.97, 2.9999999999999996 ], [ 0, 0.030000000000000002, 0.060000000000000005, 0.09000000000000001, 0.12000000000000001, 0.15, 0.18000000000000002, 0.21, 0.24000000000000002, 0.27, 0.3, 0.32999999999999996, 0.36000000000000004, 0.39, 0.42, 0.45, 0.48000000000000004, 0.51, 0.54, 0.57, 0.6, 0.6300000000000001, 0.6599999999999999, 0.69, 0.7200000000000001, 0.75, 0.78, 0.81, 0.84, 0.87, 0.9, 0.93, 0.9600000000000001, 0.99, 1.02, 1.05, 1.08, 1.1099999999999999, 1.14, 1.17, 1.2, 1.23, 1.2600000000000002, 1.29, 1.3199999999999998, 1.35, 1.38, 1.41, 1.4400000000000002, 1.4700000000000002, 1.5, 1.5300000000000002, 1.56, 1.5899999999999999, 1.62, 1.65, 1.68, 1.7100000000000002, 1.74, 1.77, 1.8, 1.83, 1.86, 1.89, 1.9200000000000002, 1.95, 1.98, 2.01, 2.04, 2.0700000000000003, 2.1, 2.13, 2.16, 2.19, 2.2199999999999998, 2.25, 2.28, 2.31, 2.34, 2.37, 2.4, 2.4299999999999997, 2.46, 2.4899999999999998, 2.5200000000000005, 2.55, 2.58, 2.6100000000000003, 2.6399999999999997, 2.67, 2.7, 2.73, 2.76, 2.7900000000000005, 2.82, 2.85, 2.8800000000000003, 2.9099999999999997, 2.9400000000000004, 2.97, 3 ], [ 0, 0.03, 0.06, 0.09, 0.12, 0.15000000000000002, 0.18, 0.21, 0.24, 0.2700000000000001, 0.30000000000000004, 0.32999999999999996, 0.36, 0.39, 0.42, 0.44999999999999996, 0.48, 0.51, 0.5400000000000001, 0.5700000000000001, 0.6000000000000001, 0.6300000000000001, 0.6599999999999999, 0.69, 0.72, 0.75, 0.78, 0.8099999999999999, 0.84, 0.87, 0.8999999999999999, 0.93, 0.96, 0.99, 1.02, 1.05, 1.0800000000000003, 1.1099999999999999, 1.1400000000000001, 1.17, 1.2000000000000002, 1.23, 1.2600000000000002, 1.29, 1.3199999999999998, 1.3499999999999999, 1.38, 1.41, 1.44, 1.47, 1.5, 1.5300000000000002, 1.56, 1.5899999999999999, 1.6199999999999999, 1.6500000000000001, 1.68, 1.7100000000000002, 1.74, 1.77, 1.7999999999999998, 1.8299999999999998, 1.86, 1.89, 1.92, 1.95, 1.98, 2.01, 2.04, 2.0700000000000003, 2.1, 2.1300000000000003, 2.1600000000000006, 2.19, 2.2199999999999998, 2.25, 2.2800000000000002, 2.31, 2.34, 2.37, 2.4000000000000004, 2.4299999999999997, 2.46, 2.4899999999999998, 2.5200000000000005, 2.5500000000000003, 2.58, 2.61, 2.6399999999999997, 2.6700000000000004, 2.6999999999999997, 2.73, 2.76, 2.79, 2.82, 2.85, 2.88, 2.91, 2.94, 2.97, 3 ], [ 0, 0.029999999999999995, 0.05999999999999999, 0.08999999999999998, 0.11999999999999998, 0.15, 0.17999999999999997, 0.21, 0.23999999999999996, 0.27, 0.3, 0.32999999999999996, 0.35999999999999993, 0.39, 0.42, 0.4499999999999999, 0.4799999999999999, 0.51, 0.54, 0.5699999999999998, 0.6, 0.6299999999999999, 0.6599999999999999, 0.69, 0.7199999999999999, 0.75, 0.78, 0.8099999999999998, 0.84, 0.87, 0.8999999999999998, 0.9299999999999999, 0.9599999999999999, 0.9899999999999999, 1.02, 1.05, 1.08, 1.1099999999999999, 1.1399999999999997, 1.1699999999999997, 1.2, 1.23, 1.2599999999999998, 1.29, 1.3199999999999998, 1.3499999999999999, 1.38, 1.41, 1.4399999999999997, 1.47, 1.5, 1.5299999999999998, 1.56, 1.5899999999999999, 1.6199999999999997, 1.6500000000000001, 1.68, 1.7099999999999997, 1.74, 1.77, 1.7999999999999996, 1.8299999999999998, 1.8599999999999999, 1.89, 1.9199999999999997, 1.95, 1.9799999999999998, 2.0099999999999993, 2.04, 2.07, 2.1, 2.1299999999999994, 2.16, 2.19, 2.2199999999999998, 2.25, 2.2799999999999994, 2.3099999999999996, 2.3399999999999994, 2.37, 2.4, 2.4299999999999997, 2.46, 2.4899999999999993, 2.5199999999999996, 2.55, 2.58, 2.61, 2.6399999999999997, 2.67, 2.6999999999999997, 2.73, 2.76, 2.79, 2.82, 2.8499999999999996, 2.8799999999999994, 2.9099999999999997, 2.94, 2.9699999999999998, 3 ], [ 0, 0.03, 0.06, 0.09, 0.12, 0.14999999999999997, 0.18, 0.20999999999999996, 0.24, 0.27, 0.29999999999999993, 0.3299999999999999, 0.36, 0.39, 0.41999999999999993, 0.4499999999999999, 0.48, 0.51, 0.54, 0.57, 0.5999999999999999, 0.63, 0.6599999999999998, 0.69, 0.72, 0.7499999999999999, 0.78, 0.8099999999999999, 0.8399999999999999, 0.87, 0.8999999999999998, 0.9299999999999998, 0.96, 0.9899999999999999, 1.02, 1.05, 1.08, 1.1099999999999999, 1.14, 1.1699999999999997, 1.1999999999999997, 1.23, 1.26, 1.29, 1.3199999999999996, 1.3499999999999999, 1.38, 1.41, 1.44, 1.4699999999999998, 1.4999999999999998, 1.5299999999999998, 1.56, 1.5899999999999999, 1.6199999999999999, 1.6499999999999997, 1.6799999999999997, 1.71, 1.74, 1.77, 1.7999999999999996, 1.8299999999999998, 1.8599999999999997, 1.89, 1.92, 1.9499999999999997, 1.9799999999999998, 2.0099999999999993, 2.04, 2.07, 2.1, 2.1299999999999994, 2.16, 2.19, 2.2199999999999998, 2.25, 2.28, 2.31, 2.3399999999999994, 2.37, 2.3999999999999995, 2.4299999999999997, 2.46, 2.4899999999999998, 2.52, 2.5499999999999994, 2.58, 2.61, 2.6399999999999992, 2.67, 2.6999999999999997, 2.73, 2.76, 2.79, 2.82, 2.8499999999999996, 2.88, 2.9099999999999997, 2.9399999999999995, 2.9699999999999998, 2.9999999999999996 ], [ 0, 0.029999999999999995, 0.05999999999999999, 0.08999999999999998, 0.11999999999999998, 0.15, 0.17999999999999997, 0.21, 0.23999999999999996, 0.27, 0.3, 0.3299999999999999, 0.35999999999999993, 0.39, 0.42, 0.44999999999999996, 0.4799999999999999, 0.51, 0.54, 0.57, 0.6, 0.6299999999999999, 0.6599999999999998, 0.69, 0.7199999999999999, 0.75, 0.78, 0.8099999999999998, 0.84, 0.87, 0.8999999999999999, 0.9299999999999999, 0.9599999999999999, 0.9899999999999999, 1.02, 1.05, 1.08, 1.1099999999999999, 1.14, 1.17, 1.2, 1.23, 1.2599999999999998, 1.29, 1.3199999999999996, 1.3499999999999999, 1.38, 1.41, 1.4399999999999997, 1.47, 1.5, 1.5299999999999998, 1.56, 1.5899999999999999, 1.6199999999999997, 1.6499999999999997, 1.68, 1.7099999999999997, 1.74, 1.77, 1.7999999999999998, 1.8299999999999998, 1.8599999999999999, 1.89, 1.9199999999999997, 1.9499999999999997, 1.9799999999999998, 2.01, 2.04, 2.07, 2.1, 2.13, 2.16, 2.19, 2.2199999999999998, 2.25, 2.28, 2.31, 2.34, 2.3699999999999997, 2.4, 2.4299999999999997, 2.46, 2.4899999999999998, 2.5199999999999996, 2.55, 2.58, 2.61, 2.6399999999999992, 2.6699999999999995, 2.6999999999999997, 2.73, 2.76, 2.7899999999999996, 2.82, 2.85, 2.8799999999999994, 2.9099999999999993, 2.94, 2.9699999999999998, 3 ], [ 0, 0.03, 0.06, 0.08999999999999998, 0.12, 0.14999999999999997, 0.17999999999999997, 0.21, 0.24, 0.27, 0.29999999999999993, 0.32999999999999996, 0.35999999999999993, 0.38999999999999996, 0.42, 0.4499999999999999, 0.48, 0.51, 0.54, 0.5699999999999998, 0.5999999999999999, 0.63, 0.6599999999999999, 0.69, 0.7199999999999999, 0.75, 0.7799999999999999, 0.8099999999999999, 0.84, 0.87, 0.8999999999999998, 0.9299999999999998, 0.96, 0.99, 1.02, 1.05, 1.08, 1.1099999999999999, 1.1399999999999997, 1.17, 1.1999999999999997, 1.23, 1.26, 1.2899999999999998, 1.3199999999999998, 1.3499999999999999, 1.38, 1.41, 1.4399999999999997, 1.47, 1.5, 1.53, 1.5599999999999998, 1.5899999999999999, 1.6199999999999999, 1.6499999999999997, 1.68, 1.71, 1.74, 1.77, 1.7999999999999996, 1.8299999999999998, 1.8599999999999997, 1.89, 1.92, 1.9499999999999997, 1.98, 2.01, 2.04, 2.07, 2.1, 2.13, 2.16, 2.19, 2.2199999999999998, 2.25, 2.2799999999999994, 2.31, 2.34, 2.37, 2.3999999999999995, 2.4299999999999997, 2.46, 2.4899999999999998, 2.52, 2.55, 2.5799999999999996, 2.61, 2.6399999999999997, 2.67, 2.6999999999999997, 2.73, 2.76, 2.79, 2.82, 2.8499999999999996, 2.8799999999999994, 2.9099999999999997, 2.94, 2.9699999999999998, 3 ], [ 0, 0.03, 0.06, 0.09, 0.12, 0.15, 0.18, 0.21, 0.24, 0.27, 0.3, 0.32999999999999996, 0.36, 0.39, 0.42, 0.44999999999999996, 0.48, 0.51, 0.54, 0.57, 0.6, 0.63, 0.6599999999999999, 0.69, 0.72, 0.7500000000000001, 0.78, 0.8099999999999999, 0.84, 0.8700000000000001, 0.8999999999999999, 0.93, 0.96, 0.9900000000000001, 1.02, 1.05, 1.08, 1.11, 1.14, 1.17, 1.2, 1.23, 1.26, 1.29, 1.3199999999999998, 1.3499999999999999, 1.38, 1.4100000000000001, 1.44, 1.47, 1.5000000000000002, 1.53, 1.56, 1.5899999999999999, 1.6199999999999999, 1.65, 1.68, 1.71, 1.7400000000000002, 1.77, 1.7999999999999998, 1.8299999999999998, 1.86, 1.89, 1.92, 1.95, 1.9800000000000002, 2.01, 2.04, 2.0700000000000003, 2.1, 2.13, 2.16, 2.19, 2.22, 2.25, 2.28, 2.31, 2.34, 2.3700000000000006, 2.4, 2.4299999999999997, 2.46, 2.4899999999999998, 2.52, 2.55, 2.58, 2.6100000000000003, 2.6399999999999997, 2.67, 2.6999999999999997, 2.73, 2.76, 2.79, 2.8200000000000003, 2.85, 2.88, 2.9099999999999997, 2.94, 2.9699999999999998, 3.0000000000000004 ], [ 0, 0.03, 0.06, 0.09000000000000001, 0.12, 0.15000000000000002, 0.18000000000000002, 0.21, 0.24, 0.27, 0.30000000000000004, 0.33, 0.36000000000000004, 0.39, 0.42, 0.45, 0.48, 0.51, 0.54, 0.57, 0.6000000000000001, 0.6300000000000001, 0.66, 0.69, 0.7200000000000001, 0.75, 0.78, 0.81, 0.84, 0.87, 0.9, 0.9299999999999999, 0.96, 0.99, 1.02, 1.05, 1.08, 1.1099999999999999, 1.14, 1.17, 1.2000000000000002, 1.23, 1.2600000000000002, 1.29, 1.32, 1.3499999999999999, 1.38, 1.41, 1.4400000000000002, 1.47, 1.5, 1.5300000000000002, 1.56, 1.5899999999999999, 1.62, 1.65, 1.68, 1.71, 1.74, 1.77, 1.8, 1.8299999999999998, 1.8599999999999999, 1.89, 1.92, 1.95, 1.98, 2.01, 2.04, 2.07, 2.1, 2.13, 2.16, 2.19, 2.2199999999999998, 2.25, 2.28, 2.31, 2.34, 2.37, 2.4000000000000004, 2.4299999999999997, 2.46, 2.4899999999999998, 2.5200000000000005, 2.55, 2.58, 2.61, 2.64, 2.67, 2.6999999999999997, 2.73, 2.76, 2.79, 2.82, 2.85, 2.8800000000000003, 2.91, 2.94, 2.9699999999999998, 3 ], [ 0, 0.03, 0.06, 0.09, 0.12, 0.15, 0.18, 0.21, 0.24, 0.27, 0.3, 0.32999999999999996, 0.36, 0.39, 0.42, 0.45, 0.48, 0.51, 0.54, 0.57, 0.6, 0.63, 0.6599999999999999, 0.69, 0.72, 0.75, 0.78, 0.8099999999999999, 0.84, 0.8700000000000001, 0.9, 0.93, 0.96, 0.99, 1.02, 1.05, 1.08, 1.1099999999999999, 1.14, 1.17, 1.2, 1.23, 1.26, 1.29, 1.3199999999999998, 1.3499999999999999, 1.38, 1.4100000000000001, 1.44, 1.4700000000000002, 1.5, 1.53, 1.56, 1.5899999999999999, 1.6199999999999999, 1.65, 1.68, 1.71, 1.7400000000000002, 1.77, 1.8, 1.8299999999999998, 1.86, 1.89, 1.92, 1.95, 1.98, 2.01, 2.04, 2.07, 2.1, 2.13, 2.16, 2.19, 2.2199999999999998, 2.25, 2.28, 2.31, 2.34, 2.37, 2.4, 2.4299999999999997, 2.46, 2.4899999999999998, 2.52, 2.55, 2.58, 2.61, 2.6399999999999997, 2.67, 2.6999999999999997, 2.73, 2.76, 2.79, 2.8200000000000003, 2.85, 2.88, 2.91, 2.9400000000000004, 2.9699999999999998, 3 ], [ 0, 0.03, 0.06, 0.09, 0.12, 0.14999999999999997, 0.18, 0.20999999999999996, 0.24, 0.27, 0.29999999999999993, 0.3299999999999999, 0.36, 0.39, 0.41999999999999993, 0.44999999999999996, 0.48, 0.51, 0.54, 0.57, 0.5999999999999999, 0.63, 0.6599999999999998, 0.69, 0.72, 0.75, 0.78, 0.8099999999999999, 0.8399999999999999, 0.87, 0.8999999999999999, 0.9299999999999999, 0.96, 0.99, 1.02, 1.05, 1.08, 1.1099999999999999, 1.14, 1.17, 1.1999999999999997, 1.23, 1.26, 1.29, 1.3199999999999996, 1.3499999999999999, 1.38, 1.41, 1.44, 1.4699999999999998, 1.5, 1.5299999999999998, 1.56, 1.5899999999999999, 1.6199999999999999, 1.65, 1.6799999999999997, 1.71, 1.74, 1.7699999999999998, 1.7999999999999998, 1.8299999999999996, 1.8599999999999999, 1.8899999999999997, 1.92, 1.9499999999999997, 1.98, 2.01, 2.04, 2.07, 2.1, 2.13, 2.16, 2.19, 2.2199999999999998, 2.25, 2.28, 2.31, 2.34, 2.3699999999999997, 2.3999999999999995, 2.4299999999999997, 2.46, 2.4899999999999998, 2.52, 2.5499999999999994, 2.58, 2.61, 2.6399999999999992, 2.67, 2.6999999999999997, 2.7299999999999995, 2.76, 2.79, 2.82, 2.8499999999999996, 2.88, 2.9099999999999997, 2.9399999999999995, 2.9699999999999998, 3 ], [ 0, 0.030000000000000002, 0.060000000000000005, 0.09, 0.12000000000000001, 0.15, 0.18, 0.21, 0.24000000000000002, 0.27, 0.3, 0.33, 0.36, 0.39000000000000007, 0.42, 0.45, 0.48000000000000004, 0.51, 0.54, 0.5700000000000001, 0.6, 0.6300000000000001, 0.66, 0.69, 0.72, 0.75, 0.7800000000000001, 0.8099999999999999, 0.84, 0.8700000000000001, 0.9, 0.9299999999999998, 0.9600000000000001, 0.99, 1.02, 1.05, 1.08, 1.1099999999999999, 1.1400000000000001, 1.17, 1.2, 1.23, 1.2600000000000002, 1.29, 1.32, 1.3499999999999999, 1.38, 1.4100000000000001, 1.44, 1.47, 1.5, 1.5300000000000002, 1.5600000000000003, 1.59, 1.6199999999999999, 1.6500000000000001, 1.68, 1.71, 1.7400000000000002, 1.77, 1.8, 1.83, 1.8599999999999997, 1.89, 1.9200000000000002, 1.95, 1.98, 2.01, 2.04, 2.07, 2.1, 2.13, 2.16, 2.1900000000000004, 2.2199999999999998, 2.25, 2.2800000000000002, 2.31, 2.34, 2.37, 2.4, 2.4299999999999997, 2.46, 2.4899999999999998, 2.5200000000000005, 2.55, 2.58, 2.6100000000000003, 2.64, 2.67, 2.6999999999999997, 2.7300000000000004, 2.76, 2.79, 2.8200000000000003, 2.8500000000000005, 2.88, 2.91, 2.94, 2.9699999999999998, 3 ], [ 0, 0.030000000000000002, 0.060000000000000005, 0.09, 0.12000000000000001, 0.15, 0.18, 0.21, 0.24000000000000002, 0.27, 0.3, 0.33, 0.36, 0.39000000000000007, 0.42, 0.45, 0.48000000000000004, 0.51, 0.54, 0.5700000000000001, 0.6, 0.6300000000000001, 0.66, 0.69, 0.72, 0.75, 0.7800000000000001, 0.8099999999999999, 0.84, 0.8700000000000001, 0.9, 0.9299999999999998, 0.9600000000000001, 0.99, 1.02, 1.05, 1.08, 1.1099999999999999, 1.1400000000000001, 1.17, 1.2, 1.23, 1.2600000000000002, 1.29, 1.32, 1.3499999999999999, 1.38, 1.4100000000000001, 1.44, 1.47, 1.5, 1.5300000000000002, 1.5600000000000003, 1.59, 1.6199999999999999, 1.6500000000000001, 1.68, 1.71, 1.7400000000000002, 1.77, 1.8, 1.83, 1.8599999999999997, 1.89, 1.9200000000000002, 1.95, 1.98, 2.01, 2.04, 2.07, 2.1, 2.13, 2.16, 2.1900000000000004, 2.2199999999999998, 2.25, 2.2800000000000002, 2.31, 2.34, 2.37, 2.4, 2.4299999999999997, 2.46, 2.4899999999999998, 2.5200000000000005, 2.55, 2.58, 2.6100000000000003, 2.64, 2.67, 2.6999999999999997, 2.7300000000000004, 2.76, 2.79, 2.8200000000000003, 2.8500000000000005, 2.88, 2.91, 2.94, 2.9699999999999998, 3 ], [ 0, 0.03, 0.06, 0.09, 0.12, 0.14999999999999997, 0.18, 0.21, 0.24, 0.27, 0.29999999999999993, 0.32999999999999996, 0.36, 0.38999999999999996, 0.42, 0.44999999999999996, 0.48, 0.51, 0.54, 0.57, 0.5999999999999999, 0.63, 0.6599999999999999, 0.69, 0.72, 0.75, 0.7799999999999999, 0.8099999999999999, 0.84, 0.87, 0.8999999999999999, 0.9299999999999999, 0.96, 0.99, 1.02, 1.05, 1.08, 1.1099999999999999, 1.14, 1.17, 1.1999999999999997, 1.23, 1.26, 1.29, 1.3199999999999998, 1.3499999999999999, 1.38, 1.41, 1.44, 1.47, 1.5, 1.53, 1.5599999999999998, 1.5899999999999999, 1.6199999999999999, 1.6499999999999997, 1.68, 1.71, 1.74, 1.7699999999999998, 1.7999999999999998, 1.8299999999999996, 1.8599999999999999, 1.8899999999999997, 1.92, 1.9499999999999997, 1.98, 2.01, 2.04, 2.07, 2.1, 2.13, 2.16, 2.19, 2.2199999999999998, 2.25, 2.28, 2.31, 2.34, 2.37, 2.3999999999999995, 2.4299999999999997, 2.46, 2.4899999999999998, 2.52, 2.55, 2.58, 2.61, 2.6399999999999997, 2.67, 2.6999999999999997, 2.73, 2.76, 2.79, 2.82, 2.85, 2.88, 2.9099999999999997, 2.94, 2.9699999999999998, 3 ], [ 0, 0.03, 0.06, 0.09, 0.12, 0.15, 0.18, 0.21, 0.24, 0.27, 0.3, 0.32999999999999996, 0.36, 0.39, 0.42, 0.45, 0.48, 0.51, 0.54, 0.57, 0.6, 0.63, 0.6599999999999999, 0.69, 0.72, 0.75, 0.78, 0.8099999999999999, 0.84, 0.8700000000000001, 0.9, 0.93, 0.96, 0.99, 1.02, 1.05, 1.08, 1.1099999999999999, 1.14, 1.17, 1.2, 1.23, 1.26, 1.29, 1.3199999999999998, 1.3499999999999999, 1.38, 1.4100000000000001, 1.44, 1.4700000000000002, 1.5, 1.53, 1.56, 1.5899999999999999, 1.6199999999999999, 1.65, 1.68, 1.71, 1.7400000000000002, 1.77, 1.8, 1.8299999999999998, 1.86, 1.89, 1.92, 1.95, 1.98, 2.01, 2.04, 2.07, 2.1, 2.13, 2.16, 2.19, 2.2199999999999998, 2.25, 2.28, 2.31, 2.34, 2.37, 2.4, 2.4299999999999997, 2.46, 2.4899999999999998, 2.52, 2.55, 2.58, 2.61, 2.6399999999999997, 2.67, 2.6999999999999997, 2.73, 2.76, 2.79, 2.8200000000000003, 2.85, 2.88, 2.91, 2.9400000000000004, 2.9699999999999998, 3 ], [ 0, 0.03, 0.06, 0.09000000000000001, 0.12, 0.15000000000000002, 0.18000000000000002, 0.21, 0.24, 0.27, 0.30000000000000004, 0.33, 0.36000000000000004, 0.39, 0.42, 0.45, 0.48, 0.51, 0.54, 0.57, 0.6000000000000001, 0.6300000000000001, 0.66, 0.69, 0.7200000000000001, 0.75, 0.78, 0.81, 0.84, 0.87, 0.9, 0.9299999999999999, 0.96, 0.99, 1.02, 1.05, 1.08, 1.1099999999999999, 1.14, 1.17, 1.2000000000000002, 1.23, 1.2600000000000002, 1.29, 1.32, 1.3499999999999999, 1.38, 1.41, 1.4400000000000002, 1.47, 1.5, 1.5300000000000002, 1.56, 1.5899999999999999, 1.62, 1.65, 1.68, 1.71, 1.74, 1.77, 1.8, 1.8299999999999998, 1.8599999999999999, 1.89, 1.92, 1.95, 1.98, 2.01, 2.04, 2.07, 2.1, 2.13, 2.16, 2.19, 2.2199999999999998, 2.25, 2.28, 2.31, 2.34, 2.37, 2.4000000000000004, 2.4299999999999997, 2.46, 2.4899999999999998, 2.5200000000000005, 2.55, 2.58, 2.61, 2.64, 2.67, 2.6999999999999997, 2.73, 2.76, 2.79, 2.82, 2.85, 2.8800000000000003, 2.91, 2.94, 2.9699999999999998, 3 ], [ 0, 0.030000000000000002, 0.060000000000000005, 0.09000000000000001, 0.12000000000000001, 0.15000000000000002, 0.18000000000000002, 0.21, 0.24000000000000002, 0.27, 0.30000000000000004, 0.33, 0.36000000000000004, 0.39, 0.42, 0.44999999999999996, 0.48000000000000004, 0.51, 0.54, 0.57, 0.6000000000000001, 0.63, 0.66, 0.6900000000000001, 0.7200000000000001, 0.7500000000000001, 0.78, 0.81, 0.84, 0.8700000000000001, 0.8999999999999999, 0.93, 0.9600000000000001, 0.9900000000000001, 1.02, 1.05, 1.08, 1.11, 1.14, 1.17, 1.2000000000000002, 1.2300000000000002, 1.26, 1.29, 1.32, 1.3499999999999999, 1.3800000000000001, 1.41, 1.4400000000000002, 1.4700000000000002, 1.5000000000000002, 1.53, 1.56, 1.59, 1.62, 1.65, 1.68, 1.71, 1.7400000000000002, 1.77, 1.7999999999999998, 1.83, 1.86, 1.89, 1.9200000000000002, 1.9500000000000002, 1.9800000000000002, 2.01, 2.04, 2.0700000000000003, 2.1, 2.13, 2.16, 2.1900000000000004, 2.22, 2.25, 2.28, 2.31, 2.34, 2.37, 2.4000000000000004, 2.4299999999999997, 2.4600000000000004, 2.49, 2.52, 2.55, 2.58, 2.61, 2.64, 2.67, 2.6999999999999997, 2.7300000000000004, 2.7600000000000002, 2.79, 2.82, 2.85, 2.8800000000000003, 2.91, 2.9400000000000004, 2.97, 3.0000000000000004 ], [ 0, 0.03, 0.06, 0.09, 0.12, 0.15000000000000002, 0.18, 0.21, 0.24, 0.27, 0.30000000000000004, 0.33, 0.36, 0.39, 0.42, 0.44999999999999996, 0.48, 0.51, 0.54, 0.57, 0.6000000000000001, 0.6300000000000001, 0.66, 0.69, 0.72, 0.75, 0.78, 0.81, 0.84, 0.87, 0.8999999999999999, 0.93, 0.96, 0.9900000000000001, 1.02, 1.05, 1.08, 1.11, 1.14, 1.1699999999999997, 1.2000000000000002, 1.23, 1.2600000000000002, 1.29, 1.32, 1.3499999999999999, 1.38, 1.41, 1.44, 1.4700000000000002, 1.5, 1.53, 1.56, 1.5899999999999999, 1.62, 1.65, 1.68, 1.71, 1.74, 1.7700000000000002, 1.7999999999999998, 1.83, 1.86, 1.8900000000000001, 1.92, 1.95, 1.9800000000000002, 2.01, 2.04, 2.07, 2.1, 2.13, 2.16, 2.19, 2.22, 2.25, 2.28, 2.31, 2.3399999999999994, 2.37, 2.4000000000000004, 2.4299999999999997, 2.46, 2.4899999999999998, 2.5200000000000005, 2.55, 2.58, 2.61, 2.64, 2.6700000000000004, 2.6999999999999997, 2.73, 2.76, 2.7900000000000005, 2.82, 2.8500000000000005, 2.88, 2.9099999999999997, 2.9400000000000004, 2.97, 3 ], [ 0, 0.03, 0.06, 0.09, 0.12, 0.15, 0.18, 0.21000000000000002, 0.24, 0.27, 0.3, 0.32999999999999996, 0.36, 0.39, 0.42000000000000004, 0.45, 0.48, 0.51, 0.54, 0.57, 0.6, 0.63, 0.6599999999999999, 0.69, 0.72, 0.75, 0.78, 0.81, 0.8400000000000001, 0.8700000000000001, 0.9, 0.9299999999999998, 0.96, 0.99, 1.02, 1.05, 1.08, 1.1099999999999999, 1.14, 1.1700000000000002, 1.2, 1.23, 1.26, 1.29, 1.3199999999999998, 1.3499999999999999, 1.38, 1.41, 1.44, 1.47, 1.5, 1.53, 1.56, 1.5899999999999999, 1.62, 1.65, 1.6800000000000002, 1.71, 1.7400000000000002, 1.77, 1.8, 1.8299999999999998, 1.8599999999999997, 1.89, 1.92, 1.95, 1.98, 2.01, 2.04, 2.07, 2.1, 2.13, 2.16, 2.19, 2.2199999999999998, 2.25, 2.28, 2.31, 2.3400000000000003, 2.37, 2.4, 2.4299999999999997, 2.46, 2.4899999999999998, 2.52, 2.55, 2.58, 2.6100000000000003, 2.6399999999999997, 2.67, 2.6999999999999997, 2.73, 2.76, 2.79, 2.82, 2.8500000000000005, 2.88, 2.91, 2.94, 2.97, 3 ], [ 0, 0.03, 0.06, 0.09, 0.12, 0.15, 0.18, 0.21, 0.24, 0.27, 0.3, 0.32999999999999996, 0.36, 0.39, 0.42, 0.4499999999999999, 0.48, 0.51, 0.54, 0.57, 0.6, 0.63, 0.6599999999999999, 0.69, 0.72, 0.7499999999999999, 0.78, 0.8099999999999999, 0.84, 0.87, 0.8999999999999998, 0.9299999999999998, 0.96, 0.9899999999999999, 1.02, 1.05, 1.08, 1.1099999999999999, 1.14, 1.17, 1.2, 1.23, 1.26, 1.29, 1.3199999999999998, 1.3499999999999999, 1.38, 1.41, 1.44, 1.47, 1.4999999999999998, 1.5299999999999998, 1.56, 1.5899999999999999, 1.6199999999999999, 1.65, 1.68, 1.71, 1.74, 1.77, 1.7999999999999996, 1.8299999999999998, 1.8599999999999997, 1.89, 1.92, 1.9499999999999997, 1.9799999999999998, 2.01, 2.04, 2.07, 2.1, 2.13, 2.16, 2.19, 2.2199999999999998, 2.25, 2.28, 2.31, 2.34, 2.37, 2.4, 2.4299999999999997, 2.46, 2.4899999999999998, 2.52, 2.5499999999999994, 2.58, 2.61, 2.6399999999999997, 2.67, 2.6999999999999997, 2.73, 2.76, 2.79, 2.82, 2.8499999999999996, 2.88, 2.9099999999999997, 2.94, 2.9699999999999998, 2.9999999999999996 ], [ 0, 0.029999999999999995, 0.05999999999999999, 0.08999999999999998, 0.11999999999999998, 0.15, 0.17999999999999997, 0.21, 0.23999999999999996, 0.27, 0.3, 0.32999999999999996, 0.35999999999999993, 0.39, 0.42, 0.4499999999999999, 0.4799999999999999, 0.51, 0.54, 0.5699999999999998, 0.6, 0.6299999999999999, 0.6599999999999999, 0.69, 0.7199999999999999, 0.75, 0.78, 0.8099999999999998, 0.84, 0.87, 0.8999999999999998, 0.9299999999999999, 0.9599999999999999, 0.9899999999999999, 1.02, 1.05, 1.08, 1.1099999999999999, 1.1399999999999997, 1.1699999999999997, 1.2, 1.23, 1.2599999999999998, 1.29, 1.3199999999999998, 1.3499999999999999, 1.38, 1.41, 1.4399999999999997, 1.47, 1.5, 1.5299999999999998, 1.56, 1.5899999999999999, 1.6199999999999997, 1.65, 1.68, 1.7099999999999997, 1.74, 1.77, 1.7999999999999996, 1.8299999999999998, 1.8599999999999999, 1.8899999999999997, 1.9199999999999997, 1.95, 1.9799999999999998, 2.0099999999999993, 2.04, 2.07, 2.1, 2.1299999999999994, 2.16, 2.19, 2.2199999999999998, 2.25, 2.2799999999999994, 2.3099999999999996, 2.3399999999999994, 2.37, 2.4, 2.4299999999999997, 2.46, 2.4899999999999993, 2.5199999999999996, 2.55, 2.58, 2.61, 2.6399999999999997, 2.67, 2.6999999999999997, 2.73, 2.76, 2.79, 2.82, 2.8499999999999996, 2.8799999999999994, 2.9099999999999997, 2.94, 2.9699999999999998, 3 ], [ 0, 0.03, 0.06, 0.09, 0.12, 0.15, 0.18, 0.21, 0.24, 0.27, 0.3, 0.32999999999999996, 0.36, 0.39, 0.42, 0.44999999999999996, 0.48, 0.51, 0.54, 0.5700000000000001, 0.6, 0.6300000000000001, 0.6599999999999999, 0.69, 0.72, 0.75, 0.78, 0.8099999999999999, 0.84, 0.87, 0.8999999999999999, 0.93, 0.96, 0.99, 1.02, 1.0499999999999998, 1.08, 1.1099999999999999, 1.1400000000000001, 1.17, 1.2, 1.23, 1.2600000000000002, 1.29, 1.3199999999999998, 1.3499999999999999, 1.38, 1.41, 1.44, 1.47, 1.5, 1.5300000000000002, 1.56, 1.5899999999999999, 1.6199999999999999, 1.6500000000000001, 1.68, 1.7100000000000002, 1.74, 1.77, 1.7999999999999998, 1.8299999999999998, 1.86, 1.89, 1.92, 1.95, 1.98, 2.01, 2.04, 2.07, 2.0999999999999996, 2.13, 2.16, 2.19, 2.2199999999999998, 2.25, 2.2800000000000002, 2.31, 2.34, 2.3699999999999997, 2.4, 2.4299999999999997, 2.46, 2.4899999999999998, 2.5200000000000005, 2.5500000000000003, 2.58, 2.61, 2.6399999999999997, 2.67, 2.6999999999999997, 2.73, 2.76, 2.79, 2.82, 2.85, 2.88, 2.91, 2.94, 2.97, 3 ], [ 0, 0.030000000000000002, 0.060000000000000005, 0.09000000000000001, 0.12000000000000001, 0.15, 0.18000000000000002, 0.21, 0.24000000000000002, 0.27, 0.3, 0.32999999999999996, 0.36000000000000004, 0.39, 0.42, 0.45, 0.48000000000000004, 0.51, 0.54, 0.57, 0.6, 0.6300000000000001, 0.6599999999999999, 0.69, 0.7200000000000001, 0.75, 0.78, 0.81, 0.84, 0.87, 0.9, 0.93, 0.9600000000000001, 0.99, 1.02, 1.05, 1.08, 1.1099999999999999, 1.14, 1.17, 1.2, 1.23, 1.2600000000000002, 1.29, 1.3199999999999998, 1.35, 1.38, 1.41, 1.4400000000000002, 1.4700000000000002, 1.5, 1.5300000000000002, 1.56, 1.5899999999999999, 1.62, 1.65, 1.68, 1.7100000000000002, 1.74, 1.77, 1.8, 1.83, 1.86, 1.89, 1.9200000000000002, 1.95, 1.98, 2.01, 2.04, 2.0700000000000003, 2.1, 2.13, 2.16, 2.19, 2.2199999999999998, 2.25, 2.28, 2.31, 2.34, 2.37, 2.4, 2.4299999999999997, 2.46, 2.4899999999999998, 2.5200000000000005, 2.55, 2.58, 2.6100000000000003, 2.6399999999999997, 2.67, 2.7, 2.73, 2.76, 2.7900000000000005, 2.82, 2.85, 2.8800000000000003, 2.9099999999999997, 2.9400000000000004, 2.97, 3 ], [ 0, 0.03, 0.06, 0.09, 0.12, 0.15, 0.18, 0.21, 0.24, 0.27, 0.3, 0.32999999999999996, 0.36, 0.39, 0.42, 0.4499999999999999, 0.48, 0.5099999999999999, 0.54, 0.57, 0.6, 0.63, 0.6599999999999999, 0.69, 0.72, 0.7499999999999999, 0.78, 0.8099999999999999, 0.84, 0.8700000000000001, 0.8999999999999998, 0.9299999999999998, 0.96, 0.99, 1.0199999999999998, 1.05, 1.08, 1.1099999999999999, 1.14, 1.17, 1.2, 1.23, 1.26, 1.29, 1.3199999999999998, 1.3499999999999999, 1.38, 1.41, 1.44, 1.4700000000000002, 1.4999999999999998, 1.5299999999999998, 1.56, 1.5899999999999999, 1.6199999999999999, 1.65, 1.68, 1.71, 1.7400000000000002, 1.77, 1.7999999999999996, 1.8299999999999998, 1.8599999999999997, 1.89, 1.92, 1.95, 1.98, 2.0099999999999993, 2.0399999999999996, 2.07, 2.1, 2.13, 2.16, 2.19, 2.2199999999999998, 2.25, 2.28, 2.31, 2.34, 2.37, 2.4, 2.4299999999999997, 2.46, 2.4899999999999993, 2.52, 2.5499999999999994, 2.58, 2.61, 2.6399999999999997, 2.67, 2.6999999999999997, 2.73, 2.76, 2.79, 2.82, 2.8500000000000005, 2.88, 2.91, 2.9400000000000004, 2.97, 2.9999999999999996 ], [ 0, 0.03, 0.06, 0.09, 0.12, 0.15, 0.18, 0.21, 0.24, 0.27, 0.3, 0.32999999999999996, 0.36, 0.39, 0.42, 0.44999999999999996, 0.48, 0.51, 0.54, 0.57, 0.6, 0.63, 0.6599999999999999, 0.69, 0.72, 0.75, 0.78, 0.8099999999999999, 0.84, 0.87, 0.8999999999999999, 0.9299999999999999, 0.96, 0.99, 1.02, 1.05, 1.08, 1.1099999999999999, 1.14, 1.17, 1.2, 1.23, 1.26, 1.29, 1.3199999999999998, 1.3499999999999999, 1.38, 1.41, 1.44, 1.47, 1.5, 1.53, 1.56, 1.5899999999999999, 1.6199999999999999, 1.65, 1.68, 1.71, 1.74, 1.77, 1.7999999999999998, 1.8299999999999998, 1.8599999999999999, 1.89, 1.92, 1.95, 1.98, 2.01, 2.04, 2.07, 2.1, 2.13, 2.16, 2.19, 2.2199999999999998, 2.25, 2.28, 2.31, 2.34, 2.37, 2.4, 2.4299999999999997, 2.46, 2.4899999999999998, 2.52, 2.55, 2.58, 2.61, 2.6399999999999997, 2.67, 2.6999999999999997, 2.73, 2.76, 2.79, 2.82, 2.85, 2.88, 2.9099999999999997, 2.94, 2.9699999999999998, 3 ], [ 0, 0.03, 0.06, 0.09, 0.12, 0.15, 0.18, 0.21, 0.24, 0.27, 0.3, 0.32999999999999996, 0.36, 0.39, 0.42, 0.4499999999999999, 0.48, 0.5099999999999999, 0.54, 0.57, 0.6, 0.63, 0.6599999999999999, 0.69, 0.72, 0.7499999999999999, 0.78, 0.8099999999999999, 0.84, 0.8700000000000001, 0.8999999999999998, 0.9299999999999998, 0.96, 0.99, 1.0199999999999998, 1.05, 1.08, 1.1099999999999999, 1.14, 1.17, 1.2, 1.23, 1.26, 1.29, 1.3199999999999998, 1.3499999999999999, 1.38, 1.41, 1.44, 1.4700000000000002, 1.4999999999999998, 1.5299999999999998, 1.56, 1.5899999999999999, 1.6199999999999999, 1.65, 1.68, 1.71, 1.7400000000000002, 1.77, 1.7999999999999996, 1.8299999999999998, 1.8599999999999997, 1.89, 1.92, 1.95, 1.98, 2.0099999999999993, 2.0399999999999996, 2.07, 2.1, 2.13, 2.16, 2.19, 2.2199999999999998, 2.25, 2.28, 2.31, 2.34, 2.3700000000000006, 2.4, 2.4299999999999997, 2.46, 2.4899999999999993, 2.52, 2.5499999999999994, 2.58, 2.61, 2.6399999999999997, 2.67, 2.6999999999999997, 2.73, 2.76, 2.79, 2.82, 2.8500000000000005, 2.88, 2.91, 2.9400000000000004, 2.97, 2.9999999999999996 ], [ 0, 0.03, 0.06, 0.08999999999999998, 0.12, 0.14999999999999997, 0.17999999999999997, 0.20999999999999996, 0.24, 0.26999999999999996, 0.29999999999999993, 0.32999999999999996, 0.35999999999999993, 0.38999999999999996, 0.41999999999999993, 0.44999999999999996, 0.48, 0.51, 0.5399999999999999, 0.57, 0.5999999999999999, 0.6299999999999999, 0.6599999999999999, 0.69, 0.7199999999999999, 0.75, 0.7799999999999999, 0.8099999999999998, 0.8399999999999999, 0.87, 0.8999999999999999, 0.9299999999999998, 0.96, 0.9899999999999999, 1.02, 1.05, 1.0799999999999998, 1.1099999999999997, 1.14, 1.17, 1.1999999999999997, 1.23, 1.2599999999999998, 1.2899999999999998, 1.3199999999999998, 1.3499999999999999, 1.38, 1.41, 1.4399999999999997, 1.4699999999999998, 1.5, 1.53, 1.5599999999999998, 1.5899999999999999, 1.6199999999999997, 1.65, 1.6799999999999997, 1.71, 1.74, 1.7699999999999998, 1.7999999999999998, 1.8299999999999998, 1.8599999999999997, 1.89, 1.92, 1.9499999999999997, 1.9799999999999998, 2.01, 2.04, 2.0699999999999994, 2.1, 2.13, 2.1599999999999997, 2.19, 2.2199999999999993, 2.25, 2.28, 2.3099999999999996, 2.34, 2.37, 2.3999999999999995, 2.4299999999999997, 2.46, 2.4899999999999993, 2.5199999999999996, 2.55, 2.5799999999999996, 2.61, 2.6399999999999997, 2.6699999999999995, 2.6999999999999997, 2.73, 2.76, 2.79, 2.82, 2.8499999999999996, 2.8799999999999994, 2.9099999999999997, 2.9399999999999995, 2.9699999999999998, 3 ], [ 0, 0.03, 0.06, 0.09, 0.12, 0.15, 0.18, 0.20999999999999996, 0.24, 0.27, 0.3, 0.32999999999999996, 0.36, 0.38999999999999996, 0.41999999999999993, 0.44999999999999996, 0.48, 0.5099999999999999, 0.54, 0.57, 0.6, 0.63, 0.6599999999999999, 0.69, 0.72, 0.7499999999999999, 0.7799999999999999, 0.8099999999999998, 0.8399999999999999, 0.87, 0.8999999999999999, 0.9299999999999999, 0.96, 0.99, 1.0199999999999998, 1.05, 1.08, 1.1099999999999999, 1.14, 1.17, 1.2, 1.23, 1.26, 1.2899999999999998, 1.3199999999999998, 1.3499999999999999, 1.38, 1.41, 1.44, 1.47, 1.4999999999999998, 1.53, 1.5599999999999998, 1.5899999999999999, 1.6199999999999997, 1.65, 1.6799999999999997, 1.71, 1.74, 1.77, 1.7999999999999998, 1.8299999999999996, 1.8599999999999999, 1.8899999999999997, 1.92, 1.9499999999999997, 1.98, 2.01, 2.0399999999999996, 2.0700000000000003, 2.1, 2.13, 2.16, 2.19, 2.2199999999999998, 2.25, 2.28, 2.3099999999999996, 2.34, 2.37, 2.4, 2.4299999999999997, 2.46, 2.4899999999999998, 2.52, 2.5499999999999994, 2.5799999999999996, 2.61, 2.6399999999999997, 2.67, 2.6999999999999997, 2.7299999999999995, 2.76, 2.79, 2.82, 2.8499999999999996, 2.88, 2.9099999999999997, 2.94, 2.9699999999999993, 2.9999999999999996 ], [ 0, 0.029999999999999995, 0.05999999999999999, 0.08999999999999998, 0.11999999999999998, 0.15, 0.17999999999999997, 0.21, 0.23999999999999996, 0.27, 0.3, 0.32999999999999996, 0.35999999999999993, 0.39, 0.42, 0.4499999999999999, 0.4799999999999999, 0.51, 0.54, 0.57, 0.6, 0.6299999999999999, 0.6599999999999999, 0.69, 0.7199999999999999, 0.75, 0.78, 0.8099999999999999, 0.84, 0.87, 0.8999999999999998, 0.9299999999999999, 0.9599999999999999, 0.99, 1.02, 1.05, 1.08, 1.1099999999999999, 1.14, 1.1699999999999997, 1.2, 1.23, 1.2599999999999998, 1.29, 1.3199999999999998, 1.3499999999999999, 1.38, 1.41, 1.4399999999999997, 1.47, 1.5, 1.5299999999999998, 1.56, 1.5899999999999999, 1.6199999999999999, 1.6500000000000001, 1.68, 1.71, 1.74, 1.77, 1.7999999999999996, 1.83, 1.8599999999999999, 1.89, 1.9199999999999997, 1.95, 1.98, 2.0099999999999993, 2.04, 2.07, 2.1, 2.1299999999999994, 2.16, 2.19, 2.2199999999999998, 2.25, 2.28, 2.31, 2.3399999999999994, 2.37, 2.4, 2.4299999999999997, 2.46, 2.4899999999999993, 2.5199999999999996, 2.5500000000000003, 2.58, 2.61, 2.6399999999999997, 2.67, 2.6999999999999997, 2.73, 2.76, 2.79, 2.82, 2.8499999999999996, 2.8799999999999994, 2.9099999999999997, 2.94, 2.9699999999999998, 3 ], [ 0, 0.03, 0.06, 0.09, 0.12, 0.15, 0.18, 0.21, 0.24, 0.27, 0.3, 0.32999999999999996, 0.36, 0.39, 0.42, 0.44999999999999996, 0.48, 0.51, 0.54, 0.57, 0.6, 0.63, 0.6599999999999999, 0.69, 0.72, 0.7499999999999999, 0.78, 0.8099999999999999, 0.84, 0.87, 0.8999999999999999, 0.9299999999999998, 0.96, 0.99, 1.02, 1.05, 1.08, 1.11, 1.14, 1.17, 1.2, 1.23, 1.26, 1.29, 1.3199999999999998, 1.3499999999999999, 1.38, 1.41, 1.44, 1.47, 1.4999999999999998, 1.5299999999999998, 1.56, 1.5899999999999999, 1.6199999999999999, 1.65, 1.68, 1.71, 1.74, 1.77, 1.7999999999999998, 1.8299999999999998, 1.8599999999999997, 1.89, 1.92, 1.95, 1.98, 2.01, 2.04, 2.07, 2.1, 2.13, 2.16, 2.19, 2.22, 2.25, 2.28, 2.31, 2.34, 2.37, 2.4, 2.4299999999999997, 2.46, 2.4899999999999998, 2.52, 2.5499999999999994, 2.58, 2.6100000000000003, 2.6399999999999997, 2.67, 2.6999999999999997, 2.73, 2.76, 2.79, 2.82, 2.85, 2.88, 2.91, 2.94, 2.97, 2.9999999999999996 ], [ 0, 0.029999999999999995, 0.05999999999999999, 0.08999999999999998, 0.11999999999999998, 0.15, 0.17999999999999997, 0.21, 0.23999999999999996, 0.27, 0.3, 0.3299999999999999, 0.35999999999999993, 0.39, 0.42, 0.44999999999999996, 0.4799999999999999, 0.51, 0.54, 0.57, 0.6, 0.6299999999999999, 0.6599999999999998, 0.69, 0.7199999999999999, 0.75, 0.78, 0.8099999999999998, 0.84, 0.87, 0.8999999999999999, 0.9299999999999999, 0.9599999999999999, 0.9899999999999999, 1.02, 1.05, 1.08, 1.1099999999999999, 1.14, 1.17, 1.2, 1.23, 1.2599999999999998, 1.29, 1.3199999999999996, 1.3499999999999999, 1.38, 1.41, 1.4399999999999997, 1.47, 1.5, 1.5299999999999998, 1.56, 1.5899999999999999, 1.6199999999999997, 1.6499999999999997, 1.68, 1.7099999999999997, 1.74, 1.77, 1.7999999999999998, 1.8299999999999998, 1.8599999999999999, 1.89, 1.9199999999999997, 1.9499999999999997, 1.9799999999999998, 2.01, 2.04, 2.07, 2.1, 2.13, 2.16, 2.19, 2.2199999999999998, 2.25, 2.28, 2.31, 2.34, 2.3699999999999997, 2.4, 2.4299999999999997, 2.46, 2.4899999999999998, 2.5199999999999996, 2.55, 2.58, 2.61, 2.6399999999999992, 2.6699999999999995, 2.6999999999999997, 2.73, 2.76, 2.7899999999999996, 2.82, 2.85, 2.8799999999999994, 2.9099999999999993, 2.94, 2.9699999999999998, 3 ], [ 0, 0.03, 0.06, 0.08999999999999998, 0.12, 0.15, 0.17999999999999997, 0.21, 0.24, 0.27, 0.3, 0.32999999999999996, 0.35999999999999993, 0.39, 0.42, 0.4499999999999999, 0.48, 0.51, 0.54, 0.5699999999999998, 0.6, 0.63, 0.6599999999999999, 0.69, 0.7199999999999999, 0.75, 0.78, 0.8099999999999999, 0.84, 0.87, 0.8999999999999998, 0.9299999999999999, 0.96, 0.99, 1.02, 1.05, 1.08, 1.1099999999999999, 1.1399999999999997, 1.17, 1.2, 1.23, 1.26, 1.29, 1.3199999999999998, 1.3499999999999999, 1.38, 1.41, 1.4399999999999997, 1.47, 1.5, 1.5300000000000002, 1.56, 1.5899999999999999, 1.6199999999999999, 1.65, 1.68, 1.71, 1.74, 1.77, 1.7999999999999996, 1.8299999999999998, 1.8599999999999999, 1.89, 1.92, 1.9499999999999997, 1.98, 2.01, 2.04, 2.07, 2.1, 2.13, 2.16, 2.19, 2.2199999999999998, 2.25, 2.2799999999999994, 2.31, 2.34, 2.37, 2.4, 2.4299999999999997, 2.46, 2.4899999999999998, 2.52, 2.55, 2.58, 2.61, 2.6399999999999997, 2.67, 2.6999999999999997, 2.73, 2.76, 2.79, 2.82, 2.8499999999999996, 2.8799999999999994, 2.91, 2.94, 2.9699999999999998, 3 ], [ 0, 0.029999999999999995, 0.05999999999999999, 0.09, 0.11999999999999998, 0.15, 0.18, 0.20999999999999996, 0.23999999999999996, 0.27, 0.3, 0.32999999999999996, 0.36, 0.38999999999999996, 0.41999999999999993, 0.4499999999999999, 0.4799999999999999, 0.51, 0.54, 0.57, 0.6, 0.6299999999999999, 0.6599999999999999, 0.69, 0.72, 0.7499999999999999, 0.7799999999999999, 0.8099999999999999, 0.8399999999999999, 0.87, 0.8999999999999998, 0.9299999999999999, 0.9599999999999999, 0.99, 1.02, 1.05, 1.08, 1.1099999999999999, 1.14, 1.1699999999999997, 1.2, 1.23, 1.2599999999999998, 1.29, 1.3199999999999998, 1.3499999999999999, 1.38, 1.41, 1.44, 1.47, 1.4999999999999998, 1.5300000000000002, 1.5599999999999998, 1.5899999999999999, 1.6199999999999999, 1.65, 1.6799999999999997, 1.7099999999999997, 1.74, 1.7699999999999998, 1.7999999999999996, 1.8299999999999998, 1.8599999999999999, 1.8899999999999997, 1.9199999999999997, 1.95, 1.98, 2.0099999999999993, 2.04, 2.07, 2.1, 2.1299999999999994, 2.16, 2.19, 2.2199999999999998, 2.25, 2.28, 2.31, 2.3399999999999994, 2.37, 2.4, 2.4299999999999997, 2.46, 2.4899999999999993, 2.5199999999999996, 2.55, 2.58, 2.61, 2.6399999999999997, 2.67, 2.6999999999999997, 2.73, 2.76, 2.79, 2.82, 2.85, 2.88, 2.9099999999999997, 2.94, 2.9699999999999993, 2.9999999999999996 ], [ 0, 0.03, 0.06, 0.09, 0.12, 0.15, 0.18, 0.21, 0.24, 0.27, 0.3, 0.32999999999999996, 0.36, 0.39, 0.42, 0.44999999999999996, 0.48, 0.51, 0.54, 0.57, 0.6, 0.63, 0.6599999999999999, 0.69, 0.72, 0.75, 0.78, 0.8099999999999999, 0.84, 0.87, 0.8999999999999999, 0.93, 0.96, 0.9899999999999999, 1.02, 1.05, 1.08, 1.1099999999999999, 1.14, 1.17, 1.2, 1.23, 1.26, 1.29, 1.3199999999999998, 1.3499999999999999, 1.38, 1.41, 1.44, 1.47, 1.5, 1.5299999999999998, 1.56, 1.5899999999999999, 1.6199999999999999, 1.65, 1.68, 1.71, 1.74, 1.77, 1.7999999999999998, 1.8299999999999998, 1.86, 1.89, 1.92, 1.95, 1.9799999999999998, 2.01, 2.04, 2.07, 2.1, 2.13, 2.16, 2.19, 2.2199999999999998, 2.25, 2.28, 2.31, 2.34, 2.37, 2.4, 2.4299999999999997, 2.46, 2.4899999999999998, 2.52, 2.55, 2.58, 2.61, 2.6399999999999997, 2.67, 2.6999999999999997, 2.7300000000000004, 2.76, 2.79, 2.82, 2.8500000000000005, 2.88, 2.9099999999999997, 2.94, 2.9699999999999998, 3 ], [ 0, 0.029999999999999995, 0.05999999999999999, 0.09, 0.11999999999999998, 0.15, 0.18, 0.21, 0.23999999999999996, 0.26999999999999996, 0.3, 0.3299999999999999, 0.36, 0.39, 0.42, 0.44999999999999996, 0.4799999999999999, 0.51, 0.5399999999999999, 0.57, 0.6, 0.6299999999999999, 0.6599999999999998, 0.69, 0.72, 0.75, 0.78, 0.8099999999999999, 0.84, 0.87, 0.8999999999999999, 0.9299999999999998, 0.9599999999999999, 0.99, 1.02, 1.05, 1.0799999999999998, 1.1099999999999999, 1.14, 1.17, 1.2, 1.23, 1.2599999999999998, 1.29, 1.3199999999999996, 1.3499999999999999, 1.38, 1.41, 1.44, 1.47, 1.5, 1.5299999999999998, 1.56, 1.5899999999999999, 1.6199999999999999, 1.6499999999999997, 1.68, 1.7099999999999997, 1.74, 1.77, 1.7999999999999998, 1.8299999999999996, 1.8599999999999997, 1.89, 1.9199999999999997, 1.9499999999999997, 1.98, 2.01, 2.04, 2.07, 2.1, 2.13, 2.1599999999999997, 2.19, 2.2199999999999998, 2.25, 2.28, 2.31, 2.34, 2.37, 2.4, 2.4299999999999997, 2.46, 2.4899999999999998, 2.5199999999999996, 2.55, 2.58, 2.61, 2.6399999999999992, 2.67, 2.6999999999999997, 2.73, 2.76, 2.79, 2.82, 2.85, 2.88, 2.9099999999999997, 2.94, 2.9699999999999993, 3 ], [ 0, 0.03, 0.06, 0.09, 0.12, 0.14999999999999997, 0.18, 0.21, 0.24, 0.27, 0.29999999999999993, 0.32999999999999996, 0.36, 0.38999999999999996, 0.42, 0.44999999999999996, 0.48, 0.51, 0.54, 0.57, 0.5999999999999999, 0.63, 0.6599999999999999, 0.69, 0.72, 0.75, 0.7799999999999999, 0.8099999999999999, 0.84, 0.87, 0.8999999999999999, 0.9299999999999999, 0.96, 0.99, 1.02, 1.05, 1.08, 1.1099999999999999, 1.14, 1.17, 1.1999999999999997, 1.23, 1.26, 1.29, 1.3199999999999998, 1.3499999999999999, 1.38, 1.41, 1.44, 1.47, 1.5, 1.53, 1.5599999999999998, 1.5899999999999999, 1.6199999999999999, 1.6499999999999997, 1.68, 1.71, 1.74, 1.7699999999999998, 1.7999999999999998, 1.8299999999999996, 1.8599999999999999, 1.8899999999999997, 1.92, 1.9499999999999997, 1.98, 2.01, 2.04, 2.07, 2.1, 2.13, 2.16, 2.19, 2.2199999999999998, 2.25, 2.28, 2.31, 2.34, 2.37, 2.3999999999999995, 2.4299999999999997, 2.46, 2.4899999999999998, 2.52, 2.55, 2.58, 2.61, 2.6399999999999997, 2.67, 2.6999999999999997, 2.73, 2.76, 2.79, 2.82, 2.85, 2.88, 2.9099999999999997, 2.94, 2.9699999999999998, 3 ], [ 0, 0.030000000000000002, 0.060000000000000005, 0.09, 0.12000000000000001, 0.15, 0.18, 0.21, 0.24000000000000002, 0.27, 0.3, 0.33, 0.36, 0.39000000000000007, 0.42, 0.45, 0.48000000000000004, 0.51, 0.54, 0.5700000000000001, 0.6, 0.6300000000000001, 0.66, 0.69, 0.72, 0.75, 0.7800000000000001, 0.8099999999999999, 0.84, 0.8700000000000001, 0.9, 0.9299999999999998, 0.9600000000000001, 0.99, 1.02, 1.05, 1.08, 1.1099999999999999, 1.1400000000000001, 1.17, 1.2, 1.23, 1.2600000000000002, 1.29, 1.32, 1.3499999999999999, 1.38, 1.4100000000000001, 1.44, 1.47, 1.5, 1.5300000000000002, 1.5600000000000003, 1.59, 1.6199999999999999, 1.6500000000000001, 1.68, 1.71, 1.7400000000000002, 1.77, 1.8, 1.83, 1.8599999999999997, 1.89, 1.9200000000000002, 1.95, 1.98, 2.01, 2.04, 2.07, 2.1, 2.13, 2.16, 2.1900000000000004, 2.2199999999999998, 2.25, 2.2800000000000002, 2.31, 2.34, 2.37, 2.4, 2.4299999999999997, 2.46, 2.4899999999999998, 2.5200000000000005, 2.55, 2.58, 2.6100000000000003, 2.64, 2.67, 2.6999999999999997, 2.7300000000000004, 2.76, 2.79, 2.8200000000000003, 2.8500000000000005, 2.88, 2.91, 2.94, 2.9699999999999998, 3 ], [ 0, 0.03, 0.06, 0.09, 0.12, 0.15, 0.18, 0.21, 0.24, 0.27, 0.3, 0.32999999999999996, 0.36, 0.39, 0.42, 0.44999999999999996, 0.48, 0.51, 0.54, 0.57, 0.6, 0.63, 0.6599999999999999, 0.69, 0.72, 0.75, 0.78, 0.8099999999999999, 0.84, 0.8700000000000001, 0.8999999999999999, 0.9299999999999998, 0.96, 0.99, 1.02, 1.05, 1.08, 1.11, 1.14, 1.17, 1.2, 1.23, 1.26, 1.29, 1.3199999999999998, 1.3499999999999999, 1.38, 1.41, 1.44, 1.47, 1.5, 1.5299999999999998, 1.56, 1.5899999999999999, 1.6199999999999999, 1.65, 1.68, 1.71, 1.7400000000000002, 1.77, 1.7999999999999998, 1.8299999999999998, 1.8599999999999997, 1.89, 1.92, 1.9499999999999997, 1.98, 2.01, 2.04, 2.07, 2.1, 2.13, 2.16, 2.19, 2.22, 2.25, 2.28, 2.31, 2.34, 2.37, 2.4, 2.4299999999999997, 2.46, 2.4899999999999998, 2.52, 2.55, 2.58, 2.61, 2.6399999999999997, 2.67, 2.6999999999999997, 2.73, 2.76, 2.79, 2.82, 2.85, 2.88, 2.9099999999999997, 2.94, 2.9699999999999998, 3 ], [ 0, 0.030000000000000002, 0.060000000000000005, 0.09, 0.12000000000000001, 0.15000000000000002, 0.18, 0.21000000000000002, 0.24000000000000002, 0.27, 0.30000000000000004, 0.32999999999999996, 0.36, 0.39000000000000007, 0.42000000000000004, 0.45, 0.48000000000000004, 0.51, 0.54, 0.57, 0.6000000000000001, 0.6300000000000001, 0.6599999999999999, 0.69, 0.72, 0.7500000000000001, 0.7800000000000001, 0.81, 0.8400000000000001, 0.8700000000000001, 0.9, 0.93, 0.9600000000000001, 0.99, 1.02, 1.0500000000000003, 1.08, 1.1099999999999999, 1.14, 1.17, 1.2000000000000002, 1.23, 1.2600000000000002, 1.29, 1.3199999999999998, 1.3499999999999999, 1.38, 1.4100000000000001, 1.44, 1.4700000000000002, 1.5000000000000002, 1.5300000000000002, 1.5600000000000003, 1.59, 1.62, 1.65, 1.6800000000000002, 1.71, 1.7400000000000002, 1.77, 1.8, 1.8299999999999998, 1.86, 1.89, 1.9200000000000002, 1.95, 1.98, 2.01, 2.04, 2.07, 2.1000000000000005, 2.13, 2.16, 2.1900000000000004, 2.2199999999999998, 2.25, 2.28, 2.31, 2.34, 2.37, 2.4000000000000004, 2.4299999999999997, 2.46, 2.4899999999999998, 2.5200000000000005, 2.55, 2.58, 2.6100000000000003, 2.6399999999999997, 2.6700000000000004, 2.6999999999999997, 2.73, 2.76, 2.79, 2.8200000000000003, 2.85, 2.88, 2.91, 2.9400000000000004, 2.9699999999999998, 3.0000000000000004 ], [ 0, 0.03, 0.06, 0.09, 0.12, 0.15, 0.18, 0.21, 0.24, 0.27, 0.3, 0.33, 0.36, 0.39, 0.42, 0.4499999999999999, 0.48, 0.51, 0.54, 0.57, 0.6, 0.63, 0.66, 0.69, 0.72, 0.7499999999999999, 0.78, 0.8099999999999999, 0.84, 0.87, 0.8999999999999998, 0.9299999999999999, 0.96, 0.99, 1.02, 1.05, 1.08, 1.1099999999999999, 1.14, 1.1699999999999997, 1.2, 1.23, 1.26, 1.29, 1.32, 1.3499999999999999, 1.38, 1.41, 1.44, 1.47, 1.4999999999999998, 1.5300000000000002, 1.56, 1.5899999999999999, 1.6199999999999999, 1.6500000000000001, 1.68, 1.71, 1.74, 1.7699999999999998, 1.7999999999999996, 1.8299999999999998, 1.8599999999999999, 1.89, 1.92, 1.9499999999999997, 1.98, 2.01, 2.04, 2.07, 2.1, 2.13, 2.16, 2.19, 2.2199999999999998, 2.25, 2.28, 2.31, 2.3399999999999994, 2.37, 2.4, 2.4299999999999997, 2.46, 2.4899999999999998, 2.52, 2.5499999999999994, 2.58, 2.61, 2.64, 2.67, 2.6999999999999997, 2.73, 2.76, 2.79, 2.82, 2.8500000000000005, 2.88, 2.9099999999999997, 2.94, 2.9699999999999998, 2.9999999999999996 ], [ 0, 0.03, 0.06, 0.09000000000000001, 0.12, 0.15000000000000002, 0.18000000000000002, 0.21, 0.24, 0.27, 0.30000000000000004, 0.33, 0.36000000000000004, 0.39, 0.42, 0.45, 0.48, 0.51, 0.54, 0.57, 0.6000000000000001, 0.6300000000000001, 0.66, 0.69, 0.7200000000000001, 0.75, 0.78, 0.81, 0.84, 0.87, 0.9, 0.9299999999999999, 0.96, 0.99, 1.02, 1.05, 1.08, 1.1099999999999999, 1.14, 1.17, 1.2000000000000002, 1.23, 1.2600000000000002, 1.29, 1.32, 1.3499999999999999, 1.38, 1.41, 1.4400000000000002, 1.47, 1.5, 1.5300000000000002, 1.56, 1.5899999999999999, 1.62, 1.65, 1.68, 1.71, 1.74, 1.77, 1.8, 1.8299999999999998, 1.8599999999999999, 1.89, 1.92, 1.95, 1.98, 2.01, 2.04, 2.07, 2.1, 2.13, 2.16, 2.19, 2.2199999999999998, 2.25, 2.28, 2.31, 2.34, 2.37, 2.4000000000000004, 2.4299999999999997, 2.46, 2.4899999999999998, 2.5200000000000005, 2.55, 2.58, 2.61, 2.64, 2.67, 2.6999999999999997, 2.73, 2.76, 2.79, 2.82, 2.85, 2.8800000000000003, 2.91, 2.94, 2.9699999999999998, 3 ], [ 0, 0.030000000000000002, 0.060000000000000005, 0.09000000000000001, 0.12000000000000001, 0.15000000000000002, 0.18000000000000002, 0.21, 0.24000000000000002, 0.27, 0.30000000000000004, 0.33, 0.36000000000000004, 0.39, 0.42, 0.44999999999999996, 0.48000000000000004, 0.51, 0.54, 0.57, 0.6000000000000001, 0.63, 0.66, 0.6900000000000001, 0.7200000000000001, 0.7500000000000001, 0.78, 0.81, 0.84, 0.8700000000000001, 0.8999999999999999, 0.93, 0.9600000000000001, 0.9900000000000001, 1.02, 1.05, 1.08, 1.11, 1.14, 1.17, 1.2000000000000002, 1.2300000000000002, 1.26, 1.29, 1.32, 1.3499999999999999, 1.3800000000000001, 1.41, 1.4400000000000002, 1.4700000000000002, 1.5000000000000002, 1.53, 1.56, 1.59, 1.62, 1.65, 1.68, 1.71, 1.7400000000000002, 1.77, 1.7999999999999998, 1.83, 1.86, 1.89, 1.9200000000000002, 1.9500000000000002, 1.9800000000000002, 2.01, 2.04, 2.0700000000000003, 2.1, 2.13, 2.16, 2.1900000000000004, 2.22, 2.25, 2.28, 2.31, 2.34, 2.37, 2.4000000000000004, 2.4299999999999997, 2.4600000000000004, 2.49, 2.52, 2.55, 2.58, 2.61, 2.64, 2.67, 2.6999999999999997, 2.7300000000000004, 2.7600000000000002, 2.79, 2.82, 2.85, 2.8800000000000003, 2.91, 2.9400000000000004, 2.97, 3.0000000000000004 ], [ 0, 0.03, 0.06, 0.08999999999999998, 0.12, 0.15, 0.17999999999999997, 0.21, 0.24, 0.27, 0.3, 0.32999999999999996, 0.35999999999999993, 0.39, 0.42, 0.44999999999999996, 0.48, 0.51, 0.54, 0.57, 0.6, 0.63, 0.6599999999999999, 0.69, 0.7199999999999999, 0.75, 0.78, 0.8099999999999999, 0.84, 0.87, 0.8999999999999999, 0.9299999999999999, 0.96, 0.99, 1.02, 1.05, 1.08, 1.11, 1.14, 1.1699999999999997, 1.2, 1.23, 1.26, 1.29, 1.3199999999999998, 1.3499999999999999, 1.38, 1.41, 1.4399999999999997, 1.4700000000000002, 1.5, 1.5299999999999998, 1.56, 1.5899999999999999, 1.6199999999999999, 1.6499999999999997, 1.68, 1.71, 1.74, 1.77, 1.7999999999999998, 1.8299999999999998, 1.8599999999999999, 1.89, 1.92, 1.9499999999999997, 1.98, 2.01, 2.04, 2.07, 2.1, 2.13, 2.16, 2.19, 2.22, 2.25, 2.28, 2.31, 2.3399999999999994, 2.37, 2.4, 2.4299999999999997, 2.46, 2.4899999999999998, 2.52, 2.55, 2.58, 2.61, 2.6399999999999997, 2.67, 2.6999999999999997, 2.73, 2.76, 2.79, 2.82, 2.85, 2.8799999999999994, 2.9099999999999993, 2.9400000000000004, 2.9699999999999998, 3 ], [ 0, 0.029999999999999995, 0.05999999999999999, 0.08999999999999998, 0.11999999999999998, 0.15, 0.17999999999999997, 0.21, 0.23999999999999996, 0.26999999999999996, 0.3, 0.3299999999999999, 0.35999999999999993, 0.39, 0.42, 0.44999999999999996, 0.4799999999999999, 0.51, 0.5399999999999999, 0.57, 0.6, 0.63, 0.6599999999999998, 0.69, 0.7199999999999999, 0.75, 0.78, 0.8099999999999999, 0.84, 0.87, 0.8999999999999999, 0.9299999999999998, 0.9599999999999999, 0.9899999999999999, 1.02, 1.05, 1.0799999999999998, 1.1099999999999999, 1.14, 1.17, 1.2, 1.2299999999999998, 1.26, 1.2899999999999998, 1.3199999999999996, 1.3499999999999999, 1.38, 1.4099999999999997, 1.4399999999999997, 1.4699999999999998, 1.5, 1.5299999999999998, 1.56, 1.5899999999999996, 1.6199999999999999, 1.6499999999999997, 1.68, 1.71, 1.74, 1.77, 1.7999999999999998, 1.8299999999999998, 1.8599999999999997, 1.89, 1.9199999999999997, 1.9499999999999997, 1.9799999999999998, 2.01, 2.04, 2.07, 2.1, 2.13, 2.1599999999999997, 2.19, 2.2199999999999998, 2.25, 2.28, 2.3099999999999996, 2.34, 2.37, 2.4, 2.4299999999999997, 2.4599999999999995, 2.4899999999999998, 2.52, 2.55, 2.5799999999999996, 2.61, 2.6399999999999992, 2.67, 2.6999999999999997, 2.73, 2.76, 2.79, 2.8199999999999994, 2.85, 2.8799999999999994, 2.9099999999999997, 2.9399999999999995, 2.9699999999999998, 3 ], [ 0, 0.03, 0.06, 0.09, 0.12, 0.15, 0.18, 0.20999999999999996, 0.24, 0.26999999999999996, 0.3, 0.32999999999999996, 0.36, 0.38999999999999996, 0.41999999999999993, 0.44999999999999996, 0.48, 0.51, 0.5399999999999999, 0.57, 0.6, 0.6299999999999999, 0.6599999999999999, 0.69, 0.72, 0.75, 0.7799999999999999, 0.8099999999999999, 0.8399999999999999, 0.87, 0.8999999999999999, 0.9299999999999998, 0.96, 0.9899999999999999, 1.02, 1.05, 1.0799999999999998, 1.1099999999999999, 1.14, 1.17, 1.2, 1.23, 1.2599999999999998, 1.29, 1.3199999999999998, 1.3499999999999999, 1.38, 1.4099999999999997, 1.44, 1.47, 1.5, 1.53, 1.5599999999999998, 1.5899999999999999, 1.6199999999999999, 1.6499999999999997, 1.6799999999999997, 1.7099999999999997, 1.74, 1.77, 1.7999999999999998, 1.8299999999999996, 1.8599999999999997, 1.89, 1.92, 1.95, 1.9799999999999998, 2.01, 2.04, 2.07, 2.1, 2.13, 2.1599999999999997, 2.19, 2.2199999999999998, 2.25, 2.28, 2.3099999999999996, 2.34, 2.3699999999999997, 2.4, 2.4299999999999997, 2.46, 2.4899999999999998, 2.5199999999999996, 2.55, 2.58, 2.61, 2.6399999999999997, 2.6699999999999995, 2.6999999999999997, 2.73, 2.76, 2.79, 2.8199999999999994, 2.85, 2.88, 2.9099999999999993, 2.94, 2.9699999999999998, 3 ], [ 0, 0.029999999999999995, 0.05999999999999999, 0.08999999999999998, 0.11999999999999998, 0.15, 0.17999999999999997, 0.21, 0.23999999999999996, 0.27, 0.3, 0.32999999999999996, 0.35999999999999993, 0.39, 0.42, 0.4499999999999999, 0.4799999999999999, 0.51, 0.54, 0.5699999999999998, 0.6, 0.6299999999999999, 0.6599999999999999, 0.69, 0.7199999999999999, 0.75, 0.78, 0.8099999999999998, 0.84, 0.87, 0.8999999999999998, 0.9299999999999999, 0.9599999999999999, 0.9899999999999999, 1.02, 1.05, 1.08, 1.1099999999999999, 1.1399999999999997, 1.1699999999999997, 1.2, 1.23, 1.2599999999999998, 1.29, 1.3199999999999998, 1.3499999999999999, 1.38, 1.41, 1.4399999999999997, 1.47, 1.5, 1.5299999999999998, 1.56, 1.5899999999999999, 1.6199999999999997, 1.6500000000000001, 1.68, 1.7099999999999997, 1.74, 1.77, 1.7999999999999996, 1.8299999999999998, 1.8599999999999999, 1.89, 1.9199999999999997, 1.95, 1.9799999999999998, 2.0099999999999993, 2.04, 2.07, 2.1, 2.1299999999999994, 2.16, 2.19, 2.2199999999999998, 2.25, 2.2799999999999994, 2.3099999999999996, 2.3399999999999994, 2.37, 2.4, 2.4299999999999997, 2.46, 2.4899999999999993, 2.5199999999999996, 2.55, 2.58, 2.61, 2.6399999999999997, 2.67, 2.6999999999999997, 2.73, 2.76, 2.79, 2.82, 2.8499999999999996, 2.8799999999999994, 2.9099999999999997, 2.94, 2.9699999999999998, 3 ], [ 0, 0.03, 0.06, 0.09, 0.12, 0.15000000000000002, 0.18, 0.21, 0.24, 0.27, 0.30000000000000004, 0.32999999999999996, 0.36, 0.39, 0.42, 0.44999999999999996, 0.48, 0.51, 0.54, 0.5700000000000001, 0.6000000000000001, 0.6300000000000001, 0.6599999999999999, 0.69, 0.72, 0.75, 0.78, 0.8099999999999999, 0.84, 0.87, 0.8999999999999999, 0.93, 0.96, 0.99, 1.02, 1.05, 1.08, 1.1099999999999999, 1.1400000000000001, 1.17, 1.2000000000000002, 1.23, 1.2600000000000002, 1.29, 1.3199999999999998, 1.3499999999999999, 1.38, 1.41, 1.44, 1.47, 1.5, 1.5300000000000002, 1.56, 1.5899999999999999, 1.6199999999999999, 1.6500000000000001, 1.68, 1.7100000000000002, 1.74, 1.77, 1.7999999999999998, 1.8299999999999998, 1.86, 1.89, 1.92, 1.95, 1.98, 2.01, 2.04, 2.07, 2.1, 2.13, 2.16, 2.19, 2.2199999999999998, 2.25, 2.2800000000000002, 2.31, 2.34, 2.3699999999999997, 2.4000000000000004, 2.4299999999999997, 2.46, 2.4899999999999998, 2.5200000000000005, 2.5500000000000003, 2.58, 2.61, 2.6399999999999997, 2.67, 2.6999999999999997, 2.73, 2.76, 2.79, 2.82, 2.85, 2.88, 2.91, 2.94, 2.97, 3 ], [ 0, 0.030000000000000002, 0.060000000000000005, 0.09000000000000001, 0.12000000000000001, 0.15, 0.18000000000000002, 0.21, 0.24000000000000002, 0.27, 0.3, 0.32999999999999996, 0.36000000000000004, 0.39, 0.42, 0.45, 0.48000000000000004, 0.51, 0.54, 0.57, 0.6, 0.6300000000000001, 0.6599999999999999, 0.69, 0.7200000000000001, 0.75, 0.78, 0.81, 0.84, 0.87, 0.9, 0.93, 0.9600000000000001, 0.99, 1.02, 1.0499999999999998, 1.08, 1.1099999999999999, 1.14, 1.17, 1.2, 1.23, 1.2600000000000002, 1.29, 1.3199999999999998, 1.3499999999999999, 1.38, 1.41, 1.4400000000000002, 1.4700000000000002, 1.5, 1.5300000000000002, 1.56, 1.5899999999999999, 1.62, 1.65, 1.68, 1.7100000000000002, 1.74, 1.77, 1.8, 1.8299999999999998, 1.86, 1.89, 1.9200000000000002, 1.95, 1.98, 2.01, 2.04, 2.0700000000000003, 2.0999999999999996, 2.13, 2.16, 2.19, 2.2199999999999998, 2.25, 2.28, 2.31, 2.34, 2.37, 2.4, 2.4299999999999997, 2.46, 2.4899999999999998, 2.5200000000000005, 2.55, 2.58, 2.61, 2.6399999999999997, 2.67, 2.6999999999999997, 2.73, 2.76, 2.79, 2.82, 2.85, 2.8800000000000003, 2.9099999999999997, 2.9400000000000004, 2.97, 3 ], [ 0, 0.03, 0.06, 0.09, 0.12, 0.15, 0.18, 0.21, 0.24, 0.27, 0.3, 0.32999999999999996, 0.36, 0.39, 0.42, 0.4499999999999999, 0.48, 0.5099999999999999, 0.54, 0.57, 0.6, 0.63, 0.6599999999999999, 0.69, 0.72, 0.7499999999999999, 0.78, 0.8099999999999999, 0.84, 0.87, 0.8999999999999998, 0.9299999999999998, 0.96, 0.99, 1.0199999999999998, 1.05, 1.08, 1.1099999999999999, 1.14, 1.17, 1.2, 1.23, 1.26, 1.2899999999999998, 1.3199999999999998, 1.3499999999999999, 1.38, 1.41, 1.44, 1.4700000000000002, 1.4999999999999998, 1.5299999999999998, 1.56, 1.5899999999999999, 1.6199999999999999, 1.65, 1.68, 1.71, 1.74, 1.77, 1.7999999999999996, 1.8299999999999998, 1.8599999999999997, 1.89, 1.92, 1.95, 1.98, 2.0099999999999993, 2.0399999999999996, 2.07, 2.1, 2.13, 2.16, 2.19, 2.2199999999999998, 2.25, 2.28, 2.31, 2.34, 2.37, 2.4, 2.4299999999999997, 2.46, 2.4899999999999993, 2.52, 2.5499999999999994, 2.5799999999999996, 2.61, 2.6399999999999997, 2.67, 2.6999999999999997, 2.73, 2.76, 2.79, 2.82, 2.8500000000000005, 2.88, 2.91, 2.9400000000000004, 2.97, 2.9999999999999996 ], [ 0, 0.03, 0.06, 0.09, 0.12, 0.15, 0.18, 0.21, 0.24, 0.27, 0.3, 0.32999999999999996, 0.36, 0.39, 0.42, 0.44999999999999996, 0.48, 0.51, 0.54, 0.57, 0.6, 0.63, 0.6599999999999999, 0.69, 0.72, 0.75, 0.78, 0.8099999999999999, 0.84, 0.87, 0.8999999999999999, 0.9299999999999999, 0.96, 0.99, 1.02, 1.05, 1.08, 1.1099999999999999, 1.14, 1.17, 1.2, 1.23, 1.26, 1.29, 1.3199999999999998, 1.3499999999999999, 1.38, 1.41, 1.44, 1.47, 1.5, 1.53, 1.56, 1.5899999999999999, 1.6199999999999999, 1.65, 1.68, 1.71, 1.74, 1.77, 1.7999999999999998, 1.8299999999999998, 1.8599999999999999, 1.89, 1.92, 1.95, 1.98, 2.01, 2.04, 2.07, 2.1, 2.13, 2.16, 2.19, 2.2199999999999998, 2.25, 2.28, 2.31, 2.34, 2.37, 2.4, 2.4299999999999997, 2.46, 2.4899999999999998, 2.52, 2.55, 2.58, 2.61, 2.6399999999999997, 2.67, 2.6999999999999997, 2.73, 2.76, 2.79, 2.82, 2.85, 2.88, 2.9099999999999997, 2.94, 2.9699999999999998, 3 ], [ 0, 0.03, 0.06, 0.09, 0.12, 0.15, 0.18, 0.21, 0.24, 0.27, 0.3, 0.32999999999999996, 0.36, 0.39, 0.42, 0.4499999999999999, 0.48, 0.5099999999999999, 0.54, 0.57, 0.6, 0.63, 0.6599999999999999, 0.69, 0.72, 0.7499999999999999, 0.78, 0.8099999999999999, 0.84, 0.8700000000000001, 0.8999999999999998, 0.9299999999999998, 0.96, 0.99, 1.0199999999999998, 1.05, 1.08, 1.1099999999999999, 1.14, 1.17, 1.2, 1.23, 1.26, 1.29, 1.3199999999999998, 1.3499999999999999, 1.38, 1.41, 1.44, 1.4700000000000002, 1.4999999999999998, 1.5299999999999998, 1.56, 1.5899999999999999, 1.6199999999999999, 1.65, 1.68, 1.71, 1.7400000000000002, 1.77, 1.7999999999999996, 1.8299999999999998, 1.8599999999999997, 1.89, 1.92, 1.95, 1.98, 2.0099999999999993, 2.0399999999999996, 2.07, 2.1, 2.13, 2.16, 2.19, 2.2199999999999998, 2.25, 2.28, 2.31, 2.34, 2.37, 2.4, 2.4299999999999997, 2.46, 2.4899999999999993, 2.52, 2.5499999999999994, 2.58, 2.61, 2.6399999999999997, 2.67, 2.6999999999999997, 2.73, 2.76, 2.79, 2.82, 2.8500000000000005, 2.88, 2.91, 2.9400000000000004, 2.97, 2.9999999999999996 ], [ 0, 0.030000000000000002, 0.060000000000000005, 0.09000000000000001, 0.12000000000000001, 0.15, 0.18000000000000002, 0.21, 0.24000000000000002, 0.27, 0.3, 0.32999999999999996, 0.36000000000000004, 0.39, 0.42, 0.45, 0.48000000000000004, 0.51, 0.54, 0.57, 0.6, 0.6300000000000001, 0.6599999999999999, 0.6900000000000001, 0.7200000000000001, 0.75, 0.78, 0.81, 0.84, 0.87, 0.9, 0.93, 0.9600000000000001, 0.99, 1.02, 1.05, 1.08, 1.1099999999999999, 1.14, 1.17, 1.2, 1.23, 1.2600000000000002, 1.29, 1.3199999999999998, 1.35, 1.3800000000000001, 1.41, 1.4400000000000002, 1.4700000000000002, 1.5, 1.5300000000000002, 1.56, 1.5899999999999999, 1.62, 1.65, 1.68, 1.7100000000000002, 1.74, 1.77, 1.8, 1.83, 1.86, 1.89, 1.9200000000000002, 1.95, 1.98, 2.01, 2.04, 2.0700000000000003, 2.1, 2.13, 2.16, 2.19, 2.2199999999999998, 2.25, 2.28, 2.31, 2.34, 2.37, 2.4, 2.4299999999999997, 2.46, 2.4899999999999998, 2.5200000000000005, 2.55, 2.58, 2.6100000000000003, 2.6399999999999997, 2.67, 2.7, 2.73, 2.7600000000000002, 2.7900000000000005, 2.82, 2.8500000000000005, 2.8800000000000003, 2.9099999999999997, 2.9400000000000004, 2.97, 3 ], [ 0, 0.03, 0.06, 0.09, 0.12, 0.15, 0.18, 0.20999999999999996, 0.24, 0.27, 0.3, 0.32999999999999996, 0.36, 0.38999999999999996, 0.41999999999999993, 0.44999999999999996, 0.48, 0.5099999999999999, 0.54, 0.57, 0.6, 0.63, 0.6599999999999999, 0.69, 0.72, 0.7499999999999999, 0.7799999999999999, 0.8099999999999998, 0.8399999999999999, 0.87, 0.8999999999999999, 0.9299999999999999, 0.96, 0.99, 1.0199999999999998, 1.05, 1.08, 1.1099999999999999, 1.14, 1.17, 1.2, 1.2299999999999998, 1.26, 1.2899999999999998, 1.3199999999999998, 1.3499999999999999, 1.38, 1.41, 1.44, 1.47, 1.4999999999999998, 1.53, 1.5599999999999998, 1.5899999999999999, 1.6199999999999997, 1.65, 1.6799999999999997, 1.71, 1.74, 1.77, 1.7999999999999998, 1.8299999999999996, 1.8599999999999999, 1.8899999999999997, 1.92, 1.9499999999999997, 1.98, 2.01, 2.0399999999999996, 2.0700000000000003, 2.1, 2.13, 2.16, 2.19, 2.2199999999999998, 2.25, 2.28, 2.3099999999999996, 2.34, 2.37, 2.4, 2.4299999999999997, 2.4599999999999995, 2.4899999999999998, 2.52, 2.5499999999999994, 2.5799999999999996, 2.61, 2.6399999999999997, 2.67, 2.6999999999999997, 2.7299999999999995, 2.76, 2.79, 2.82, 2.8499999999999996, 2.88, 2.9099999999999997, 2.94, 2.9699999999999993, 2.9999999999999996 ], [ 0, 0.03, 0.06, 0.09, 0.12, 0.15, 0.18, 0.21, 0.24, 0.27, 0.3, 0.32999999999999996, 0.36, 0.39, 0.42, 0.44999999999999996, 0.48, 0.51, 0.54, 0.57, 0.6, 0.63, 0.6599999999999999, 0.69, 0.72, 0.75, 0.78, 0.8099999999999999, 0.84, 0.8700000000000001, 0.8999999999999999, 0.93, 0.96, 0.99, 1.02, 1.05, 1.08, 1.11, 1.14, 1.1699999999999997, 1.2, 1.23, 1.26, 1.29, 1.3199999999999998, 1.3499999999999999, 1.38, 1.41, 1.44, 1.47, 1.5, 1.53, 1.56, 1.5899999999999999, 1.6199999999999999, 1.6500000000000001, 1.68, 1.71, 1.7400000000000002, 1.77, 1.7999999999999998, 1.83, 1.86, 1.89, 1.92, 1.95, 1.98, 2.01, 2.04, 2.07, 2.1, 2.13, 2.16, 2.19, 2.22, 2.25, 2.28, 2.31, 2.3399999999999994, 2.37, 2.4, 2.4299999999999997, 2.46, 2.4899999999999998, 2.52, 2.5500000000000003, 2.58, 2.61, 2.6399999999999997, 2.67, 2.6999999999999997, 2.73, 2.76, 2.79, 2.82, 2.85, 2.88, 2.91, 2.94, 2.97, 3 ], [ 0, 0.03, 0.06, 0.08999999999999998, 0.12, 0.15, 0.17999999999999997, 0.21, 0.24, 0.26999999999999996, 0.3, 0.32999999999999996, 0.35999999999999993, 0.39, 0.42, 0.4499999999999999, 0.48, 0.51, 0.5399999999999999, 0.5699999999999998, 0.6, 0.63, 0.6599999999999999, 0.69, 0.7199999999999999, 0.7499999999999999, 0.78, 0.8099999999999998, 0.84, 0.87, 0.8999999999999998, 0.9299999999999998, 0.96, 0.9899999999999999, 1.02, 1.05, 1.0799999999999998, 1.1099999999999999, 1.1399999999999997, 1.17, 1.2, 1.23, 1.26, 1.29, 1.3199999999999998, 1.3499999999999999, 1.38, 1.41, 1.4399999999999997, 1.47, 1.4999999999999998, 1.53, 1.56, 1.5899999999999996, 1.6199999999999997, 1.6499999999999997, 1.68, 1.71, 1.74, 1.77, 1.7999999999999996, 1.8299999999999996, 1.8599999999999997, 1.89, 1.92, 1.9499999999999997, 1.9799999999999998, 2.0099999999999993, 2.04, 2.07, 2.1, 2.13, 2.1599999999999997, 2.19, 2.2199999999999998, 2.25, 2.2799999999999994, 2.3099999999999996, 2.34, 2.3699999999999997, 2.4, 2.4299999999999993, 2.46, 2.4899999999999998, 2.52, 2.55, 2.58, 2.61, 2.6399999999999997, 2.67, 2.6999999999999997, 2.73, 2.76, 2.7899999999999996, 2.82, 2.85, 2.8799999999999994, 2.9099999999999997, 2.94, 2.9699999999999998, 2.9999999999999996 ], [ 0, 0.03, 0.06, 0.08999999999999998, 0.12, 0.15, 0.17999999999999997, 0.21, 0.24, 0.27, 0.3, 0.32999999999999996, 0.35999999999999993, 0.38999999999999996, 0.42, 0.44999999999999996, 0.48, 0.51, 0.54, 0.57, 0.6, 0.63, 0.6599999999999999, 0.69, 0.7199999999999999, 0.7499999999999999, 0.7799999999999999, 0.8099999999999999, 0.84, 0.87, 0.8999999999999999, 0.9299999999999999, 0.96, 0.9899999999999999, 1.02, 1.05, 1.08, 1.1099999999999999, 1.14, 1.17, 1.2, 1.23, 1.26, 1.29, 1.3199999999999998, 1.3499999999999999, 1.38, 1.41, 1.4399999999999997, 1.47, 1.4999999999999998, 1.5300000000000002, 1.5599999999999998, 1.5899999999999999, 1.6199999999999999, 1.6500000000000001, 1.68, 1.7099999999999997, 1.74, 1.77, 1.7999999999999998, 1.8299999999999998, 1.8599999999999999, 1.89, 1.92, 1.95, 1.9799999999999998, 2.01, 2.04, 2.07, 2.1, 2.13, 2.16, 2.19, 2.2199999999999998, 2.25, 2.28, 2.31, 2.34, 2.37, 2.4, 2.4299999999999997, 2.46, 2.4899999999999998, 2.52, 2.55, 2.58, 2.61, 2.6399999999999997, 2.67, 2.6999999999999997, 2.73, 2.76, 2.79, 2.82, 2.85, 2.8799999999999994, 2.91, 2.94, 2.9699999999999998, 2.9999999999999996 ], [ 0, 0.03, 0.06, 0.08999999999999998, 0.12, 0.14999999999999997, 0.17999999999999997, 0.21, 0.24, 0.27, 0.29999999999999993, 0.32999999999999996, 0.35999999999999993, 0.38999999999999996, 0.42, 0.4499999999999999, 0.48, 0.51, 0.54, 0.5699999999999998, 0.5999999999999999, 0.63, 0.6599999999999999, 0.69, 0.7199999999999999, 0.75, 0.7799999999999999, 0.8099999999999999, 0.84, 0.87, 0.8999999999999998, 0.9299999999999998, 0.96, 0.99, 1.02, 1.05, 1.08, 1.1099999999999999, 1.1399999999999997, 1.17, 1.1999999999999997, 1.23, 1.26, 1.2899999999999998, 1.3199999999999998, 1.3499999999999999, 1.38, 1.41, 1.4399999999999997, 1.47, 1.5, 1.53, 1.5599999999999998, 1.5899999999999999, 1.6199999999999999, 1.6499999999999997, 1.68, 1.71, 1.74, 1.77, 1.7999999999999996, 1.8299999999999998, 1.8599999999999997, 1.89, 1.92, 1.9499999999999997, 1.98, 2.01, 2.04, 2.07, 2.1, 2.13, 2.16, 2.19, 2.2199999999999998, 2.25, 2.2799999999999994, 2.31, 2.34, 2.37, 2.3999999999999995, 2.4299999999999997, 2.46, 2.4899999999999998, 2.52, 2.55, 2.5799999999999996, 2.61, 2.6399999999999997, 2.67, 2.6999999999999997, 2.73, 2.76, 2.79, 2.82, 2.8499999999999996, 2.8799999999999994, 2.9099999999999997, 2.94, 2.9699999999999998, 3 ], [ 0, 0.029999999999999995, 0.05999999999999999, 0.09, 0.11999999999999998, 0.15, 0.18, 0.20999999999999996, 0.23999999999999996, 0.27, 0.3, 0.32999999999999996, 0.36, 0.38999999999999996, 0.41999999999999993, 0.4499999999999999, 0.4799999999999999, 0.51, 0.54, 0.57, 0.6, 0.6299999999999999, 0.6599999999999999, 0.69, 0.72, 0.7499999999999999, 0.7799999999999999, 0.8099999999999999, 0.8399999999999999, 0.87, 0.8999999999999998, 0.9299999999999999, 0.9599999999999999, 0.99, 1.02, 1.05, 1.08, 1.1099999999999999, 1.14, 1.1699999999999997, 1.2, 1.23, 1.2599999999999998, 1.29, 1.3199999999999998, 1.3499999999999999, 1.38, 1.41, 1.44, 1.47, 1.4999999999999998, 1.5300000000000002, 1.5599999999999998, 1.5899999999999999, 1.6199999999999999, 1.65, 1.6799999999999997, 1.7099999999999997, 1.74, 1.7699999999999998, 1.7999999999999996, 1.8299999999999998, 1.8599999999999999, 1.8899999999999997, 1.9199999999999997, 1.95, 1.98, 2.0099999999999993, 2.04, 2.07, 2.1, 2.1299999999999994, 2.16, 2.19, 2.2199999999999998, 2.25, 2.28, 2.31, 2.3399999999999994, 2.37, 2.4, 2.4299999999999997, 2.46, 2.4899999999999993, 2.5199999999999996, 2.55, 2.58, 2.61, 2.6399999999999997, 2.67, 2.6999999999999997, 2.73, 2.76, 2.79, 2.82, 2.85, 2.88, 2.9099999999999997, 2.94, 2.9699999999999993, 2.9999999999999996 ], [ 0, 0.03, 0.06, 0.09, 0.12, 0.15, 0.18, 0.21, 0.24, 0.27, 0.3, 0.32999999999999996, 0.36, 0.39, 0.42, 0.44999999999999996, 0.48, 0.51, 0.54, 0.57, 0.6, 0.63, 0.6599999999999999, 0.69, 0.72, 0.75, 0.78, 0.8099999999999999, 0.84, 0.87, 0.8999999999999999, 0.93, 0.96, 0.9899999999999999, 1.02, 1.05, 1.08, 1.1099999999999999, 1.14, 1.17, 1.2, 1.23, 1.26, 1.29, 1.3199999999999998, 1.3499999999999999, 1.38, 1.41, 1.44, 1.47, 1.5, 1.5299999999999998, 1.56, 1.5899999999999999, 1.6199999999999999, 1.65, 1.68, 1.71, 1.74, 1.77, 1.7999999999999998, 1.8299999999999998, 1.86, 1.89, 1.92, 1.95, 1.9799999999999998, 2.01, 2.04, 2.07, 2.1, 2.13, 2.16, 2.19, 2.2199999999999998, 2.25, 2.28, 2.31, 2.34, 2.37, 2.4, 2.4299999999999997, 2.46, 2.4899999999999998, 2.52, 2.55, 2.58, 2.61, 2.6399999999999997, 2.67, 2.6999999999999997, 2.7300000000000004, 2.76, 2.79, 2.82, 2.8500000000000005, 2.88, 2.9099999999999997, 2.94, 2.9699999999999998, 3 ], [ 0, 0.03, 0.06, 0.09, 0.12, 0.15, 0.18, 0.21, 0.24, 0.26999999999999996, 0.3, 0.32999999999999996, 0.36, 0.38999999999999996, 0.42, 0.4499999999999999, 0.48, 0.51, 0.5399999999999999, 0.57, 0.6, 0.63, 0.6599999999999999, 0.69, 0.72, 0.7499999999999999, 0.7799999999999999, 0.8099999999999999, 0.84, 0.87, 0.8999999999999998, 0.9299999999999999, 0.96, 0.99, 1.02, 1.05, 1.0799999999999998, 1.11, 1.14, 1.17, 1.2, 1.23, 1.26, 1.29, 1.3199999999999998, 1.3499999999999999, 1.38, 1.4099999999999997, 1.44, 1.47, 1.4999999999999998, 1.53, 1.5599999999999998, 1.5899999999999999, 1.6199999999999999, 1.65, 1.68, 1.71, 1.74, 1.77, 1.7999999999999996, 1.8299999999999998, 1.8599999999999999, 1.89, 1.92, 1.9499999999999997, 1.98, 2.01, 2.04, 2.07, 2.1, 2.13, 2.1599999999999997, 2.19, 2.22, 2.25, 2.28, 2.31, 2.34, 2.3699999999999997, 2.4, 2.4299999999999997, 2.46, 2.4899999999999998, 2.52, 2.55, 2.58, 2.61, 2.6399999999999997, 2.67, 2.6999999999999997, 2.73, 2.76, 2.7899999999999996, 2.8199999999999994, 2.85, 2.88, 2.9099999999999993, 2.94, 2.9699999999999998, 2.9999999999999996 ], [ 0, 0.030000000000000002, 0.060000000000000005, 0.09, 0.12000000000000001, 0.15, 0.18, 0.21, 0.24000000000000002, 0.27, 0.3, 0.32999999999999996, 0.36, 0.39, 0.42, 0.44999999999999996, 0.48000000000000004, 0.51, 0.54, 0.57, 0.6, 0.63, 0.6599999999999999, 0.6900000000000001, 0.72, 0.75, 0.78, 0.8099999999999999, 0.84, 0.87, 0.8999999999999999, 0.93, 0.9600000000000001, 0.99, 1.02, 1.05, 1.08, 1.11, 1.14, 1.17, 1.2, 1.23, 1.26, 1.29, 1.3199999999999998, 1.3499999999999999, 1.3800000000000001, 1.41, 1.44, 1.4700000000000002, 1.5, 1.53, 1.56, 1.5899999999999999, 1.6199999999999999, 1.6499999999999997, 1.68, 1.71, 1.74, 1.7700000000000002, 1.7999999999999998, 1.8299999999999998, 1.86, 1.89, 1.9200000000000002, 1.95, 1.98, 2.01, 2.04, 2.0700000000000003, 2.1, 2.13, 2.16, 2.19, 2.22, 2.25, 2.28, 2.31, 2.34, 2.37, 2.4, 2.4299999999999997, 2.46, 2.4899999999999998, 2.52, 2.55, 2.58, 2.61, 2.6399999999999997, 2.67, 2.6999999999999997, 2.7300000000000004, 2.7600000000000002, 2.79, 2.82, 2.8499999999999996, 2.88, 2.9099999999999997, 2.9400000000000004, 2.9699999999999998, 3 ], [ 0, 0.030000000000000002, 0.060000000000000005, 0.09, 0.12000000000000001, 0.15, 0.18, 0.21, 0.24000000000000002, 0.27, 0.3, 0.33, 0.36, 0.39000000000000007, 0.42, 0.45, 0.48000000000000004, 0.51, 0.54, 0.5700000000000001, 0.6, 0.6300000000000001, 0.66, 0.69, 0.72, 0.75, 0.7800000000000001, 0.8099999999999999, 0.84, 0.8700000000000001, 0.9, 0.9299999999999998, 0.9600000000000001, 0.99, 1.02, 1.05, 1.08, 1.1099999999999999, 1.1400000000000001, 1.17, 1.2, 1.23, 1.2600000000000002, 1.29, 1.32, 1.3499999999999999, 1.38, 1.4100000000000001, 1.44, 1.47, 1.5, 1.5300000000000002, 1.5600000000000003, 1.59, 1.6199999999999999, 1.6500000000000001, 1.68, 1.71, 1.7400000000000002, 1.77, 1.8, 1.83, 1.8599999999999997, 1.89, 1.9200000000000002, 1.95, 1.98, 2.01, 2.04, 2.07, 2.1, 2.13, 2.16, 2.1900000000000004, 2.2199999999999998, 2.25, 2.2800000000000002, 2.31, 2.34, 2.37, 2.4, 2.4299999999999997, 2.46, 2.4899999999999998, 2.5200000000000005, 2.55, 2.58, 2.6100000000000003, 2.64, 2.67, 2.6999999999999997, 2.7300000000000004, 2.76, 2.79, 2.8200000000000003, 2.8500000000000005, 2.88, 2.91, 2.94, 2.9699999999999998, 3 ], [ 0, 0.03, 0.06, 0.09, 0.12, 0.15, 0.18, 0.20999999999999996, 0.24, 0.27, 0.3, 0.32999999999999996, 0.36, 0.39, 0.41999999999999993, 0.44999999999999996, 0.48, 0.51, 0.54, 0.57, 0.6, 0.63, 0.6599999999999999, 0.69, 0.72, 0.7499999999999999, 0.78, 0.8099999999999998, 0.8399999999999999, 0.87, 0.8999999999999999, 0.9299999999999998, 0.96, 0.99, 1.02, 1.05, 1.08, 1.1099999999999999, 1.14, 1.1699999999999997, 1.2, 1.23, 1.26, 1.29, 1.3199999999999998, 1.3499999999999999, 1.38, 1.41, 1.44, 1.47, 1.4999999999999998, 1.5299999999999998, 1.56, 1.5899999999999999, 1.6199999999999997, 1.65, 1.6799999999999997, 1.71, 1.74, 1.77, 1.7999999999999998, 1.8299999999999998, 1.8599999999999997, 1.8899999999999997, 1.92, 1.9499999999999997, 1.98, 2.01, 2.04, 2.07, 2.1, 2.13, 2.16, 2.1900000000000004, 2.2199999999999998, 2.25, 2.28, 2.31, 2.3399999999999994, 2.37, 2.4, 2.4299999999999997, 2.46, 2.4899999999999998, 2.52, 2.55, 2.58, 2.61, 2.6399999999999997, 2.67, 2.6999999999999997, 2.73, 2.76, 2.79, 2.82, 2.85, 2.88, 2.9099999999999997, 2.94, 2.9699999999999998, 2.9999999999999996 ], [ 0, 0.030000000000000002, 0.060000000000000005, 0.09, 0.12000000000000001, 0.15, 0.18, 0.21, 0.24000000000000002, 0.27, 0.3, 0.33, 0.36, 0.39, 0.42, 0.45, 0.48000000000000004, 0.51, 0.54, 0.57, 0.6, 0.63, 0.66, 0.6900000000000001, 0.72, 0.75, 0.78, 0.8099999999999999, 0.84, 0.87, 0.9, 0.93, 0.9600000000000001, 0.9900000000000001, 1.02, 1.05, 1.08, 1.11, 1.14, 1.17, 1.2, 1.23, 1.26, 1.29, 1.32, 1.3499999999999999, 1.3800000000000001, 1.41, 1.44, 1.4700000000000002, 1.5, 1.5300000000000002, 1.56, 1.59, 1.6199999999999999, 1.6500000000000001, 1.68, 1.71, 1.74, 1.77, 1.8, 1.8299999999999998, 1.86, 1.89, 1.9200000000000002, 1.95, 1.9800000000000002, 2.01, 2.04, 2.07, 2.1, 2.13, 2.16, 2.1900000000000004, 2.22, 2.25, 2.28, 2.31, 2.34, 2.37, 2.4, 2.4299999999999997, 2.46, 2.4899999999999998, 2.52, 2.55, 2.58, 2.6100000000000003, 2.64, 2.67, 2.6999999999999997, 2.7300000000000004, 2.7600000000000002, 2.79, 2.82, 2.8500000000000005, 2.88, 2.9099999999999997, 2.9400000000000004, 2.97, 3 ], [ 0, 0.03, 0.06, 0.09, 0.12, 0.15, 0.18, 0.21, 0.24, 0.27, 0.3, 0.33, 0.36, 0.39, 0.42, 0.4499999999999999, 0.48, 0.51, 0.54, 0.57, 0.6, 0.63, 0.66, 0.69, 0.72, 0.7499999999999999, 0.78, 0.8099999999999999, 0.84, 0.87, 0.8999999999999998, 0.9299999999999999, 0.96, 0.99, 1.02, 1.05, 1.08, 1.1099999999999999, 1.14, 1.1699999999999997, 1.2, 1.23, 1.26, 1.29, 1.32, 1.3499999999999999, 1.38, 1.41, 1.44, 1.47, 1.4999999999999998, 1.5300000000000002, 1.56, 1.5899999999999999, 1.6199999999999999, 1.6500000000000001, 1.68, 1.71, 1.74, 1.7699999999999998, 1.7999999999999996, 1.8299999999999998, 1.8599999999999999, 1.89, 1.92, 1.9499999999999997, 1.98, 2.01, 2.04, 2.07, 2.1, 2.13, 2.16, 2.19, 2.2199999999999998, 2.25, 2.28, 2.31, 2.3399999999999994, 2.37, 2.4, 2.4299999999999997, 2.46, 2.4899999999999998, 2.52, 2.5499999999999994, 2.58, 2.61, 2.64, 2.67, 2.6999999999999997, 2.73, 2.76, 2.79, 2.82, 2.8500000000000005, 2.88, 2.9099999999999997, 2.94, 2.9699999999999998, 2.9999999999999996 ], [ 0, 0.03, 0.06, 0.09, 0.12, 0.15000000000000002, 0.18, 0.21, 0.24, 0.27, 0.30000000000000004, 0.33, 0.36, 0.39, 0.42, 0.44999999999999996, 0.48, 0.51, 0.54, 0.57, 0.6000000000000001, 0.63, 0.66, 0.69, 0.72, 0.75, 0.78, 0.8099999999999999, 0.84, 0.87, 0.8999999999999999, 0.9299999999999999, 0.96, 0.99, 1.02, 1.05, 1.08, 1.11, 1.14, 1.17, 1.2000000000000002, 1.23, 1.26, 1.29, 1.32, 1.3499999999999999, 1.38, 1.4099999999999997, 1.44, 1.47, 1.5, 1.5300000000000002, 1.56, 1.5899999999999999, 1.6199999999999999, 1.6500000000000001, 1.68, 1.71, 1.74, 1.7700000000000002, 1.7999999999999998, 1.8299999999999998, 1.8599999999999999, 1.89, 1.92, 1.95, 1.98, 2.01, 2.04, 2.07, 2.1, 2.13, 2.16, 2.19, 2.22, 2.25, 2.28, 2.31, 2.34, 2.3700000000000006, 2.4000000000000004, 2.4299999999999997, 2.46, 2.4899999999999998, 2.52, 2.55, 2.58, 2.61, 2.64, 2.67, 2.6999999999999997, 2.7300000000000004, 2.76, 2.79, 2.8199999999999994, 2.8500000000000005, 2.88, 2.9099999999999997, 2.94, 2.97, 3 ], [ 0, 0.03, 0.06, 0.09, 0.12, 0.15, 0.18, 0.21, 0.24, 0.27, 0.3, 0.32999999999999996, 0.36, 0.39, 0.42, 0.44999999999999996, 0.48, 0.51, 0.54, 0.57, 0.6, 0.63, 0.6599999999999999, 0.69, 0.72, 0.7500000000000001, 0.78, 0.8099999999999999, 0.84, 0.8700000000000001, 0.8999999999999999, 0.93, 0.96, 0.9900000000000001, 1.02, 1.05, 1.08, 1.11, 1.14, 1.17, 1.2, 1.23, 1.26, 1.29, 1.3199999999999998, 1.3499999999999999, 1.38, 1.4100000000000001, 1.44, 1.47, 1.5000000000000002, 1.53, 1.56, 1.5899999999999999, 1.6199999999999999, 1.65, 1.68, 1.71, 1.7400000000000002, 1.77, 1.7999999999999998, 1.8299999999999998, 1.86, 1.89, 1.92, 1.95, 1.9800000000000002, 2.01, 2.04, 2.0700000000000003, 2.1, 2.13, 2.16, 2.19, 2.22, 2.25, 2.28, 2.31, 2.34, 2.3700000000000006, 2.4, 2.4299999999999997, 2.46, 2.4899999999999998, 2.52, 2.55, 2.58, 2.6100000000000003, 2.6399999999999997, 2.67, 2.6999999999999997, 2.73, 2.76, 2.79, 2.8200000000000003, 2.85, 2.88, 2.9099999999999997, 2.94, 2.9699999999999998, 3.0000000000000004 ], [ 0, 0.03, 0.06, 0.09000000000000001, 0.12, 0.15, 0.18000000000000002, 0.21, 0.24, 0.27, 0.3, 0.33, 0.36000000000000004, 0.39, 0.42, 0.44999999999999996, 0.48, 0.51, 0.54, 0.57, 0.6, 0.63, 0.66, 0.69, 0.7200000000000001, 0.75, 0.78, 0.8099999999999999, 0.84, 0.8700000000000001, 0.8999999999999999, 0.9299999999999999, 0.96, 0.99, 1.02, 1.05, 1.08, 1.11, 1.14, 1.1700000000000002, 1.2, 1.23, 1.26, 1.29, 1.32, 1.3499999999999999, 1.38, 1.41, 1.4400000000000002, 1.47, 1.5, 1.5300000000000002, 1.56, 1.5899999999999999, 1.6199999999999999, 1.6500000000000001, 1.68, 1.71, 1.7400000000000002, 1.7700000000000002, 1.7999999999999998, 1.83, 1.8599999999999999, 1.89, 1.92, 1.95, 1.98, 2.01, 2.04, 2.07, 2.1, 2.13, 2.16, 2.19, 2.22, 2.25, 2.28, 2.31, 2.3400000000000003, 2.37, 2.4, 2.4299999999999997, 2.46, 2.4899999999999998, 2.52, 2.55, 2.58, 2.6100000000000003, 2.64, 2.67, 2.6999999999999997, 2.73, 2.76, 2.79, 2.82, 2.85, 2.8800000000000003, 2.91, 2.94, 2.9699999999999998, 3 ], [ 0, 0.029999999999999995, 0.05999999999999999, 0.09, 0.11999999999999998, 0.15, 0.18, 0.21, 0.23999999999999996, 0.26999999999999996, 0.3, 0.32999999999999996, 0.36, 0.39, 0.42, 0.44999999999999996, 0.4799999999999999, 0.51, 0.5399999999999999, 0.57, 0.6, 0.63, 0.6599999999999999, 0.69, 0.72, 0.75, 0.78, 0.8099999999999999, 0.84, 0.87, 0.8999999999999999, 0.9299999999999998, 0.9599999999999999, 0.9899999999999999, 1.02, 1.05, 1.0799999999999998, 1.1099999999999999, 1.14, 1.17, 1.2, 1.23, 1.26, 1.2899999999999998, 1.3199999999999998, 1.3499999999999999, 1.38, 1.4099999999999997, 1.44, 1.47, 1.5, 1.53, 1.56, 1.5899999999999996, 1.6199999999999999, 1.65, 1.68, 1.71, 1.74, 1.77, 1.7999999999999998, 1.8299999999999998, 1.8599999999999997, 1.89, 1.9199999999999997, 1.9499999999999997, 1.9799999999999998, 2.01, 2.04, 2.07, 2.1, 2.13, 2.1599999999999997, 2.19, 2.2199999999999998, 2.25, 2.28, 2.3099999999999996, 2.34, 2.37, 2.4, 2.4299999999999997, 2.46, 2.4899999999999998, 2.52, 2.55, 2.5799999999999996, 2.61, 2.6399999999999997, 2.67, 2.6999999999999997, 2.73, 2.76, 2.79, 2.8199999999999994, 2.85, 2.88, 2.9099999999999997, 2.94, 2.97, 3 ], [ 0, 0.03, 0.06, 0.09, 0.12, 0.15, 0.18, 0.21, 0.24, 0.27, 0.3, 0.32999999999999996, 0.36, 0.39, 0.42, 0.44999999999999996, 0.48, 0.51, 0.54, 0.57, 0.6, 0.63, 0.6599999999999999, 0.69, 0.72, 0.7499999999999999, 0.78, 0.8099999999999999, 0.84, 0.87, 0.8999999999999999, 0.9299999999999998, 0.96, 0.99, 1.02, 1.05, 1.08, 1.11, 1.14, 1.17, 1.2, 1.23, 1.26, 1.29, 1.3199999999999998, 1.3499999999999999, 1.38, 1.41, 1.44, 1.47, 1.4999999999999998, 1.5299999999999998, 1.56, 1.5899999999999999, 1.6199999999999999, 1.65, 1.68, 1.71, 1.74, 1.77, 1.7999999999999998, 1.8299999999999998, 1.8599999999999997, 1.89, 1.92, 1.95, 1.98, 2.01, 2.04, 2.07, 2.1, 2.13, 2.16, 2.19, 2.22, 2.25, 2.28, 2.31, 2.34, 2.37, 2.4, 2.4299999999999997, 2.46, 2.4899999999999998, 2.52, 2.5499999999999994, 2.58, 2.6100000000000003, 2.6399999999999997, 2.67, 2.6999999999999997, 2.73, 2.76, 2.79, 2.82, 2.85, 2.88, 2.91, 2.94, 2.97, 2.9999999999999996 ], [ 0, 0.029999999999999995, 0.05999999999999999, 0.09, 0.11999999999999998, 0.14999999999999997, 0.18, 0.21, 0.23999999999999996, 0.26999999999999996, 0.29999999999999993, 0.32999999999999996, 0.36, 0.38999999999999996, 0.42, 0.44999999999999996, 0.4799999999999999, 0.51, 0.5399999999999999, 0.57, 0.5999999999999999, 0.63, 0.6599999999999999, 0.69, 0.72, 0.75, 0.7799999999999999, 0.8099999999999998, 0.84, 0.87, 0.8999999999999999, 0.9299999999999999, 0.9599999999999999, 0.99, 1.02, 1.05, 1.0799999999999998, 1.1099999999999999, 1.14, 1.17, 1.1999999999999997, 1.23, 1.26, 1.29, 1.3199999999999998, 1.3499999999999999, 1.38, 1.41, 1.44, 1.4699999999999998, 1.5, 1.53, 1.5599999999999998, 1.5899999999999999, 1.6199999999999997, 1.6499999999999997, 1.68, 1.71, 1.74, 1.77, 1.7999999999999998, 1.8299999999999998, 1.8599999999999999, 1.89, 1.9199999999999997, 1.95, 1.98, 2.01, 2.04, 2.0700000000000003, 2.1, 2.13, 2.1599999999999997, 2.19, 2.2199999999999998, 2.25, 2.28, 2.31, 2.34, 2.3699999999999997, 2.3999999999999995, 2.4299999999999997, 2.46, 2.4899999999999998, 2.52, 2.55, 2.58, 2.61, 2.6399999999999997, 2.67, 2.6999999999999997, 2.73, 2.76, 2.79, 2.82, 2.85, 2.88, 2.9099999999999993, 2.9399999999999995, 2.9699999999999998, 3 ], [ 0, 0.03, 0.06, 0.09, 0.12, 0.15000000000000002, 0.18, 0.21, 0.24, 0.2700000000000001, 0.30000000000000004, 0.32999999999999996, 0.36, 0.39, 0.42, 0.44999999999999996, 0.48, 0.51, 0.5400000000000001, 0.5700000000000001, 0.6000000000000001, 0.6300000000000001, 0.6599999999999999, 0.69, 0.72, 0.75, 0.78, 0.8099999999999999, 0.84, 0.87, 0.8999999999999999, 0.93, 0.96, 0.99, 1.02, 1.05, 1.0800000000000003, 1.1099999999999999, 1.1400000000000001, 1.17, 1.2000000000000002, 1.23, 1.2600000000000002, 1.29, 1.3199999999999998, 1.3499999999999999, 1.38, 1.41, 1.44, 1.47, 1.5, 1.5300000000000002, 1.56, 1.5899999999999999, 1.6199999999999999, 1.6500000000000001, 1.68, 1.7100000000000002, 1.74, 1.77, 1.7999999999999998, 1.8299999999999998, 1.86, 1.89, 1.92, 1.95, 1.98, 2.01, 2.04, 2.0700000000000003, 2.1, 2.1300000000000003, 2.1600000000000006, 2.19, 2.2199999999999998, 2.25, 2.2800000000000002, 2.31, 2.34, 2.37, 2.4000000000000004, 2.4299999999999997, 2.46, 2.4899999999999998, 2.5200000000000005, 2.5500000000000003, 2.58, 2.61, 2.6399999999999997, 2.6700000000000004, 2.6999999999999997, 2.73, 2.76, 2.79, 2.82, 2.85, 2.88, 2.91, 2.94, 2.97, 3 ], [ 0, 0.03, 0.06, 0.09000000000000001, 0.12, 0.15, 0.18000000000000002, 0.21, 0.24, 0.27, 0.3, 0.32999999999999996, 0.36000000000000004, 0.39, 0.42, 0.44999999999999996, 0.48, 0.51, 0.54, 0.57, 0.6, 0.63, 0.6599999999999999, 0.69, 0.7200000000000001, 0.7499999999999999, 0.78, 0.8099999999999999, 0.84, 0.87, 0.8999999999999999, 0.9299999999999999, 0.96, 0.9899999999999999, 1.02, 1.0499999999999998, 1.08, 1.1099999999999999, 1.14, 1.17, 1.2, 1.23, 1.26, 1.29, 1.3199999999999998, 1.3499999999999999, 1.38, 1.41, 1.4400000000000002, 1.47, 1.4999999999999998, 1.5300000000000002, 1.56, 1.5899999999999999, 1.6199999999999999, 1.65, 1.68, 1.71, 1.74, 1.77, 1.7999999999999998, 1.8299999999999998, 1.8599999999999999, 1.8899999999999997, 1.92, 1.9499999999999997, 1.9799999999999998, 2.01, 2.04, 2.0700000000000003, 2.0999999999999996, 2.13, 2.16, 2.19, 2.2199999999999998, 2.25, 2.28, 2.31, 2.34, 2.3699999999999997, 2.4, 2.4299999999999997, 2.46, 2.4899999999999998, 2.52, 2.5499999999999994, 2.58, 2.61, 2.6399999999999997, 2.67, 2.6999999999999997, 2.73, 2.76, 2.79, 2.82, 2.85, 2.8800000000000003, 2.9099999999999993, 2.94, 2.97, 2.9999999999999996 ], [ 0, 0.03, 0.06, 0.09, 0.12, 0.15, 0.18, 0.21, 0.24, 0.27, 0.3, 0.32999999999999996, 0.36, 0.39, 0.42, 0.4499999999999999, 0.48, 0.5099999999999999, 0.54, 0.57, 0.6, 0.63, 0.6599999999999999, 0.69, 0.72, 0.7499999999999999, 0.78, 0.8099999999999999, 0.84, 0.87, 0.8999999999999998, 0.9299999999999998, 0.96, 0.99, 1.0199999999999998, 1.05, 1.08, 1.1099999999999999, 1.14, 1.17, 1.2, 1.23, 1.26, 1.2899999999999998, 1.3199999999999998, 1.3499999999999999, 1.38, 1.41, 1.44, 1.4700000000000002, 1.4999999999999998, 1.5299999999999998, 1.56, 1.5899999999999999, 1.6199999999999999, 1.65, 1.68, 1.71, 1.74, 1.77, 1.7999999999999996, 1.8299999999999998, 1.8599999999999997, 1.89, 1.92, 1.95, 1.98, 2.0099999999999993, 2.0399999999999996, 2.07, 2.1, 2.13, 2.16, 2.19, 2.2199999999999998, 2.25, 2.28, 2.31, 2.34, 2.37, 2.4, 2.4299999999999997, 2.46, 2.4899999999999993, 2.52, 2.5499999999999994, 2.5799999999999996, 2.61, 2.6399999999999997, 2.67, 2.6999999999999997, 2.73, 2.76, 2.79, 2.82, 2.8500000000000005, 2.88, 2.91, 2.9400000000000004, 2.97, 2.9999999999999996 ], [ 0, 0.03, 0.06, 0.09, 0.12, 0.15, 0.18, 0.21, 0.24, 0.27, 0.3, 0.32999999999999996, 0.36, 0.39, 0.42, 0.44999999999999996, 0.48, 0.51, 0.54, 0.57, 0.6, 0.63, 0.6599999999999999, 0.69, 0.72, 0.75, 0.78, 0.8099999999999999, 0.84, 0.87, 0.8999999999999999, 0.9299999999999999, 0.96, 0.99, 1.02, 1.05, 1.08, 1.1099999999999999, 1.14, 1.17, 1.2, 1.23, 1.26, 1.29, 1.3199999999999998, 1.3499999999999999, 1.38, 1.41, 1.44, 1.47, 1.5, 1.53, 1.56, 1.5899999999999999, 1.6199999999999999, 1.65, 1.68, 1.71, 1.74, 1.77, 1.7999999999999998, 1.8299999999999998, 1.8599999999999999, 1.89, 1.92, 1.95, 1.98, 2.01, 2.04, 2.07, 2.1, 2.13, 2.16, 2.19, 2.2199999999999998, 2.25, 2.28, 2.31, 2.34, 2.37, 2.4, 2.4299999999999997, 2.46, 2.4899999999999998, 2.52, 2.55, 2.58, 2.61, 2.6399999999999997, 2.67, 2.6999999999999997, 2.73, 2.76, 2.79, 2.82, 2.85, 2.88, 2.9099999999999997, 2.94, 2.9699999999999998, 3 ], [ 0, 0.03, 0.06, 0.09, 0.12, 0.15000000000000002, 0.18, 0.21, 0.24, 0.27, 0.30000000000000004, 0.32999999999999996, 0.36, 0.39, 0.42, 0.44999999999999996, 0.48, 0.5099999999999999, 0.54, 0.5700000000000001, 0.6000000000000001, 0.63, 0.6599999999999999, 0.69, 0.72, 0.7499999999999999, 0.78, 0.8099999999999999, 0.84, 0.8700000000000001, 0.8999999999999999, 0.9299999999999999, 0.96, 0.99, 1.0199999999999998, 1.05, 1.08, 1.1099999999999999, 1.1400000000000001, 1.17, 1.2000000000000002, 1.23, 1.26, 1.29, 1.3199999999999998, 1.3499999999999999, 1.38, 1.41, 1.44, 1.4700000000000002, 1.4999999999999998, 1.5299999999999998, 1.56, 1.5899999999999999, 1.6199999999999999, 1.65, 1.68, 1.71, 1.7400000000000002, 1.77, 1.7999999999999998, 1.8299999999999998, 1.8599999999999999, 1.89, 1.92, 1.95, 1.98, 2.0099999999999993, 2.0399999999999996, 2.07, 2.1, 2.13, 2.16, 2.19, 2.2199999999999998, 2.25, 2.2800000000000002, 2.31, 2.34, 2.3700000000000006, 2.4000000000000004, 2.4299999999999997, 2.46, 2.4899999999999998, 2.52, 2.55, 2.58, 2.61, 2.6399999999999997, 2.67, 2.6999999999999997, 2.73, 2.76, 2.79, 2.82, 2.8500000000000005, 2.88, 2.91, 2.9400000000000004, 2.97, 2.9999999999999996 ], [ 0, 0.030000000000000002, 0.060000000000000005, 0.09000000000000001, 0.12000000000000001, 0.15, 0.18000000000000002, 0.21, 0.24000000000000002, 0.27, 0.3, 0.32999999999999996, 0.36000000000000004, 0.39, 0.42, 0.45, 0.48000000000000004, 0.51, 0.54, 0.57, 0.6, 0.6300000000000001, 0.6599999999999999, 0.69, 0.7200000000000001, 0.75, 0.78, 0.81, 0.84, 0.87, 0.9, 0.93, 0.9600000000000001, 0.99, 1.02, 1.05, 1.08, 1.1099999999999999, 1.14, 1.17, 1.2, 1.23, 1.2600000000000002, 1.29, 1.3199999999999998, 1.35, 1.38, 1.41, 1.4400000000000002, 1.4700000000000002, 1.5, 1.5300000000000002, 1.56, 1.5899999999999999, 1.62, 1.65, 1.68, 1.7100000000000002, 1.74, 1.77, 1.8, 1.83, 1.86, 1.89, 1.9200000000000002, 1.95, 1.98, 2.01, 2.04, 2.0700000000000003, 2.1, 2.13, 2.16, 2.19, 2.2199999999999998, 2.25, 2.28, 2.31, 2.34, 2.37, 2.4, 2.4299999999999997, 2.46, 2.4899999999999998, 2.5200000000000005, 2.55, 2.58, 2.6100000000000003, 2.6399999999999997, 2.67, 2.7, 2.73, 2.76, 2.7900000000000005, 2.82, 2.85, 2.8800000000000003, 2.9099999999999997, 2.9400000000000004, 2.97, 3 ], [ 0, 0.03, 0.06, 0.09000000000000001, 0.12, 0.15, 0.18000000000000002, 0.21, 0.24, 0.27, 0.3, 0.32999999999999996, 0.36000000000000004, 0.39, 0.42, 0.45, 0.48, 0.5099999999999999, 0.54, 0.57, 0.6, 0.63, 0.6599999999999999, 0.69, 0.7200000000000001, 0.75, 0.78, 0.8099999999999999, 0.84, 0.87, 0.9, 0.93, 0.96, 0.99, 1.0199999999999998, 1.05, 1.08, 1.11, 1.14, 1.17, 1.2, 1.23, 1.26, 1.29, 1.3199999999999998, 1.3499999999999999, 1.38, 1.41, 1.4400000000000002, 1.47, 1.5, 1.5300000000000002, 1.56, 1.5899999999999999, 1.6199999999999999, 1.6500000000000001, 1.68, 1.71, 1.74, 1.7700000000000002, 1.8, 1.8299999999999998, 1.86, 1.89, 1.92, 1.95, 1.98, 2.01, 2.0399999999999996, 2.0700000000000003, 2.1, 2.13, 2.16, 2.19, 2.22, 2.25, 2.28, 2.31, 2.34, 2.37, 2.4, 2.4299999999999997, 2.46, 2.4899999999999998, 2.52, 2.55, 2.58, 2.6100000000000003, 2.6399999999999997, 2.67, 2.6999999999999997, 2.73, 2.76, 2.79, 2.82, 2.8499999999999996, 2.8800000000000003, 2.9099999999999997, 2.94, 2.9699999999999998, 3 ], [ 0, 0.029999999999999995, 0.05999999999999999, 0.08999999999999998, 0.11999999999999998, 0.15, 0.17999999999999997, 0.21, 0.23999999999999996, 0.27, 0.3, 0.32999999999999996, 0.35999999999999993, 0.39, 0.42, 0.44999999999999996, 0.4799999999999999, 0.51, 0.54, 0.57, 0.6, 0.6299999999999999, 0.6599999999999999, 0.69, 0.7199999999999999, 0.75, 0.78, 0.8099999999999999, 0.84, 0.8700000000000001, 0.8999999999999999, 0.93, 0.9599999999999999, 0.99, 1.02, 1.05, 1.08, 1.11, 1.14, 1.1699999999999997, 1.2, 1.23, 1.2599999999999998, 1.29, 1.3199999999999998, 1.3499999999999999, 1.38, 1.41, 1.4399999999999997, 1.47, 1.5, 1.53, 1.56, 1.5899999999999999, 1.6199999999999999, 1.6500000000000001, 1.68, 1.71, 1.7400000000000002, 1.77, 1.7999999999999998, 1.83, 1.86, 1.89, 1.9199999999999997, 1.95, 1.98, 2.01, 2.04, 2.07, 2.1, 2.1299999999999994, 2.16, 2.19, 2.22, 2.25, 2.28, 2.31, 2.3399999999999994, 2.37, 2.4, 2.4299999999999997, 2.46, 2.4899999999999998, 2.5199999999999996, 2.5500000000000003, 2.58, 2.61, 2.6399999999999997, 2.67, 2.6999999999999997, 2.73, 2.76, 2.79, 2.82, 2.85, 2.8799999999999994, 2.91, 2.94, 2.97, 3 ], [ 0, 0.03, 0.06, 0.09, 0.12, 0.15, 0.18, 0.21, 0.24, 0.27, 0.3, 0.32999999999999996, 0.36, 0.38999999999999996, 0.42, 0.44999999999999996, 0.48, 0.51, 0.54, 0.57, 0.6, 0.63, 0.6599999999999999, 0.69, 0.72, 0.75, 0.7799999999999999, 0.8099999999999999, 0.84, 0.87, 0.8999999999999999, 0.9299999999999998, 0.96, 0.99, 1.02, 1.05, 1.08, 1.1099999999999999, 1.14, 1.17, 1.2, 1.23, 1.26, 1.29, 1.3199999999999998, 1.3499999999999999, 1.38, 1.41, 1.44, 1.47, 1.5, 1.53, 1.5599999999999998, 1.59, 1.6199999999999999, 1.65, 1.68, 1.71, 1.74, 1.77, 1.7999999999999998, 1.8299999999999998, 1.8599999999999997, 1.89, 1.92, 1.95, 1.98, 2.01, 2.04, 2.07, 2.1, 2.13, 2.16, 2.19, 2.2199999999999998, 2.25, 2.28, 2.31, 2.34, 2.3699999999999997, 2.4, 2.4299999999999997, 2.46, 2.4899999999999998, 2.52, 2.55, 2.58, 2.61, 2.6399999999999997, 2.67, 2.6999999999999997, 2.7300000000000004, 2.76, 2.79, 2.82, 2.85, 2.88, 2.9099999999999997, 2.94, 2.9699999999999998, 3 ], [ 0, 0.03, 0.06, 0.08999999999999998, 0.12, 0.15, 0.17999999999999997, 0.21, 0.24, 0.27, 0.3, 0.32999999999999996, 0.35999999999999993, 0.38999999999999996, 0.42, 0.44999999999999996, 0.48, 0.51, 0.54, 0.57, 0.6, 0.63, 0.6599999999999999, 0.69, 0.7199999999999999, 0.7499999999999999, 0.7799999999999999, 0.8099999999999999, 0.84, 0.87, 0.8999999999999999, 0.9299999999999999, 0.96, 0.9899999999999999, 1.02, 1.05, 1.08, 1.1099999999999999, 1.14, 1.1699999999999997, 1.2, 1.23, 1.26, 1.29, 1.3199999999999998, 1.3499999999999999, 1.38, 1.41, 1.4399999999999997, 1.47, 1.4999999999999998, 1.5300000000000002, 1.5599999999999998, 1.5899999999999999, 1.6199999999999999, 1.65, 1.68, 1.7099999999999997, 1.74, 1.77, 1.7999999999999998, 1.8299999999999996, 1.8599999999999999, 1.89, 1.92, 1.95, 1.9799999999999998, 2.01, 2.04, 2.07, 2.1, 2.13, 2.16, 2.19, 2.2199999999999998, 2.25, 2.28, 2.31, 2.3399999999999994, 2.37, 2.4, 2.4299999999999997, 2.46, 2.4899999999999998, 2.52, 2.55, 2.58, 2.61, 2.6399999999999997, 2.67, 2.6999999999999997, 2.73, 2.76, 2.79, 2.82, 2.85, 2.8799999999999994, 2.9099999999999997, 2.94, 2.9699999999999998, 2.9999999999999996 ], [ 0, 0.03, 0.06, 0.09, 0.12, 0.15000000000000002, 0.18, 0.21, 0.24, 0.27, 0.30000000000000004, 0.33, 0.36, 0.39, 0.42, 0.44999999999999996, 0.48, 0.51, 0.54, 0.57, 0.6000000000000001, 0.6300000000000001, 0.66, 0.69, 0.72, 0.75, 0.78, 0.81, 0.84, 0.87, 0.8999999999999999, 0.93, 0.96, 0.9900000000000001, 1.02, 1.05, 1.08, 1.11, 1.14, 1.1699999999999997, 1.2000000000000002, 1.23, 1.2600000000000002, 1.29, 1.32, 1.3499999999999999, 1.38, 1.41, 1.44, 1.4700000000000002, 1.5, 1.53, 1.56, 1.5899999999999999, 1.62, 1.65, 1.68, 1.71, 1.74, 1.7700000000000002, 1.7999999999999998, 1.83, 1.86, 1.8900000000000001, 1.92, 1.95, 1.9800000000000002, 2.01, 2.04, 2.07, 2.1, 2.13, 2.16, 2.19, 2.22, 2.25, 2.28, 2.31, 2.3399999999999994, 2.37, 2.4000000000000004, 2.4299999999999997, 2.46, 2.4899999999999998, 2.5200000000000005, 2.55, 2.58, 2.61, 2.64, 2.6700000000000004, 2.6999999999999997, 2.73, 2.76, 2.7900000000000005, 2.82, 2.8500000000000005, 2.88, 2.9099999999999997, 2.9400000000000004, 2.97, 3 ], [ 0, 0.03, 0.06, 0.09, 0.12, 0.15, 0.18, 0.21, 0.24, 0.27, 0.3, 0.32999999999999996, 0.36, 0.39, 0.42, 0.44999999999999996, 0.48, 0.51, 0.54, 0.57, 0.6, 0.63, 0.6599999999999999, 0.69, 0.72, 0.75, 0.78, 0.8099999999999999, 0.84, 0.8700000000000001, 0.8999999999999999, 0.9299999999999999, 0.96, 0.99, 1.02, 1.05, 1.08, 1.1099999999999999, 1.14, 1.1700000000000002, 1.2, 1.23, 1.26, 1.29, 1.3199999999999998, 1.3499999999999999, 1.38, 1.41, 1.44, 1.4700000000000002, 1.5, 1.5300000000000002, 1.56, 1.5899999999999999, 1.6199999999999999, 1.65, 1.68, 1.71, 1.7400000000000002, 1.77, 1.7999999999999998, 1.8299999999999998, 1.8599999999999999, 1.89, 1.92, 1.95, 1.98, 2.01, 2.04, 2.07, 2.1, 2.13, 2.16, 2.19, 2.2199999999999998, 2.25, 2.28, 2.31, 2.3400000000000003, 2.37, 2.4, 2.4299999999999997, 2.46, 2.4899999999999998, 2.52, 2.5500000000000003, 2.58, 2.61, 2.6399999999999997, 2.67, 2.6999999999999997, 2.73, 2.76, 2.79, 2.82, 2.85, 2.88, 2.91, 2.9400000000000004, 2.9699999999999998, 3 ], [ 0, 0.03, 0.06, 0.09000000000000001, 0.12, 0.15000000000000002, 0.18000000000000002, 0.21, 0.24, 0.27, 0.30000000000000004, 0.33, 0.36000000000000004, 0.39, 0.42, 0.45, 0.48, 0.51, 0.54, 0.57, 0.6000000000000001, 0.6300000000000001, 0.66, 0.69, 0.7200000000000001, 0.75, 0.78, 0.81, 0.84, 0.87, 0.9, 0.9299999999999999, 0.96, 0.99, 1.02, 1.05, 1.08, 1.1099999999999999, 1.14, 1.17, 1.2000000000000002, 1.23, 1.2600000000000002, 1.29, 1.32, 1.3499999999999999, 1.38, 1.41, 1.4400000000000002, 1.47, 1.5, 1.5300000000000002, 1.56, 1.5899999999999999, 1.62, 1.65, 1.68, 1.71, 1.74, 1.77, 1.8, 1.8299999999999998, 1.8599999999999999, 1.89, 1.92, 1.95, 1.98, 2.01, 2.04, 2.07, 2.1, 2.13, 2.16, 2.19, 2.2199999999999998, 2.25, 2.28, 2.31, 2.34, 2.37, 2.4000000000000004, 2.4299999999999997, 2.46, 2.4899999999999998, 2.5200000000000005, 2.55, 2.58, 2.61, 2.64, 2.67, 2.6999999999999997, 2.73, 2.76, 2.79, 2.82, 2.85, 2.8800000000000003, 2.91, 2.94, 2.9699999999999998, 3 ], [ 0, 0.03, 0.06, 0.08999999999999998, 0.12, 0.15, 0.17999999999999997, 0.21, 0.24, 0.27, 0.3, 0.33, 0.35999999999999993, 0.39, 0.42, 0.44999999999999996, 0.48, 0.51, 0.54, 0.57, 0.6, 0.63, 0.66, 0.69, 0.7199999999999999, 0.75, 0.78, 0.8099999999999999, 0.84, 0.87, 0.8999999999999999, 0.93, 0.96, 0.99, 1.02, 1.05, 1.08, 1.1099999999999999, 1.14, 1.17, 1.2, 1.23, 1.26, 1.29, 1.32, 1.3499999999999999, 1.38, 1.41, 1.4399999999999997, 1.4700000000000002, 1.5, 1.5299999999999998, 1.56, 1.5899999999999999, 1.6199999999999999, 1.65, 1.68, 1.71, 1.74, 1.77, 1.7999999999999998, 1.8299999999999998, 1.86, 1.89, 1.92, 1.9499999999999997, 1.98, 2.01, 2.04, 2.07, 2.1, 2.13, 2.16, 2.19, 2.2199999999999998, 2.25, 2.28, 2.31, 2.34, 2.37, 2.4, 2.4299999999999997, 2.46, 2.4899999999999998, 2.52, 2.5499999999999994, 2.58, 2.61, 2.64, 2.67, 2.6999999999999997, 2.7300000000000004, 2.76, 2.79, 2.82, 2.8499999999999996, 2.8799999999999994, 2.9099999999999997, 2.9400000000000004, 2.9699999999999998, 3 ], [ 0, 0.030000000000000002, 0.060000000000000005, 0.09, 0.12000000000000001, 0.15, 0.18, 0.21, 0.24000000000000002, 0.27, 0.3, 0.32999999999999996, 0.36, 0.39, 0.42, 0.44999999999999996, 0.48000000000000004, 0.51, 0.54, 0.57, 0.6, 0.63, 0.6599999999999999, 0.6900000000000001, 0.72, 0.75, 0.78, 0.8099999999999999, 0.84, 0.87, 0.8999999999999999, 0.93, 0.9600000000000001, 0.99, 1.02, 1.05, 1.08, 1.11, 1.14, 1.17, 1.2, 1.23, 1.26, 1.29, 1.3199999999999998, 1.3499999999999999, 1.3800000000000001, 1.41, 1.44, 1.4700000000000002, 1.5, 1.53, 1.56, 1.5899999999999999, 1.6199999999999999, 1.6499999999999997, 1.68, 1.71, 1.74, 1.7700000000000002, 1.7999999999999998, 1.8299999999999998, 1.86, 1.89, 1.9200000000000002, 1.95, 1.98, 2.01, 2.04, 2.0700000000000003, 2.1, 2.13, 2.16, 2.19, 2.22, 2.25, 2.28, 2.31, 2.34, 2.37, 2.4, 2.4299999999999997, 2.46, 2.4899999999999998, 2.52, 2.55, 2.58, 2.61, 2.6399999999999997, 2.67, 2.6999999999999997, 2.7300000000000004, 2.7600000000000002, 2.79, 2.82, 2.8499999999999996, 2.88, 2.9099999999999997, 2.9400000000000004, 2.9699999999999998, 3 ], [ 0, 0.03, 0.06, 0.09, 0.12, 0.15, 0.18, 0.20999999999999996, 0.24, 0.27, 0.3, 0.32999999999999996, 0.36, 0.39, 0.41999999999999993, 0.44999999999999996, 0.48, 0.51, 0.54, 0.57, 0.6, 0.63, 0.6599999999999999, 0.69, 0.72, 0.7499999999999999, 0.78, 0.8099999999999998, 0.8399999999999999, 0.87, 0.8999999999999999, 0.9299999999999998, 0.96, 0.99, 1.02, 1.05, 1.08, 1.1099999999999999, 1.14, 1.1699999999999997, 1.2, 1.23, 1.26, 1.29, 1.3199999999999998, 1.3499999999999999, 1.38, 1.41, 1.44, 1.47, 1.4999999999999998, 1.5299999999999998, 1.56, 1.5899999999999999, 1.6199999999999997, 1.65, 1.6799999999999997, 1.71, 1.74, 1.77, 1.7999999999999998, 1.8299999999999998, 1.8599999999999997, 1.8899999999999997, 1.92, 1.9499999999999997, 1.98, 2.01, 2.04, 2.07, 2.1, 2.13, 2.16, 2.1900000000000004, 2.2199999999999998, 2.25, 2.28, 2.31, 2.3399999999999994, 2.37, 2.4, 2.4299999999999997, 2.46, 2.4899999999999998, 2.52, 2.55, 2.58, 2.61, 2.6399999999999997, 2.67, 2.6999999999999997, 2.73, 2.76, 2.79, 2.82, 2.85, 2.88, 2.9099999999999997, 2.94, 2.9699999999999998, 2.9999999999999996 ], [ 0, 0.03, 0.06, 0.09, 0.12, 0.15, 0.18, 0.21, 0.24, 0.27, 0.3, 0.32999999999999996, 0.36, 0.39, 0.42, 0.44999999999999996, 0.48, 0.51, 0.54, 0.57, 0.6, 0.63, 0.6599999999999999, 0.69, 0.72, 0.75, 0.78, 0.81, 0.84, 0.87, 0.8999999999999999, 0.9299999999999998, 0.96, 0.99, 1.02, 1.05, 1.08, 1.1099999999999999, 1.14, 1.17, 1.2, 1.23, 1.26, 1.29, 1.3199999999999998, 1.3499999999999999, 1.38, 1.41, 1.44, 1.47, 1.5, 1.53, 1.56, 1.5899999999999999, 1.62, 1.65, 1.68, 1.71, 1.74, 1.77, 1.7999999999999998, 1.8299999999999998, 1.8599999999999997, 1.89, 1.92, 1.9499999999999997, 1.98, 2.01, 2.04, 2.07, 2.1, 2.13, 2.16, 2.19, 2.2199999999999998, 2.25, 2.28, 2.31, 2.34, 2.37, 2.4, 2.4299999999999997, 2.46, 2.4899999999999998, 2.52, 2.55, 2.58, 2.61, 2.6399999999999997, 2.67, 2.6999999999999997, 2.73, 2.76, 2.79, 2.82, 2.85, 2.88, 2.9099999999999997, 2.94, 2.97, 3 ], [ 0, 0.030000000000000002, 0.060000000000000005, 0.09, 0.12000000000000001, 0.15, 0.18, 0.21, 0.24000000000000002, 0.27, 0.3, 0.33, 0.36, 0.39, 0.42, 0.45, 0.48000000000000004, 0.51, 0.54, 0.57, 0.6, 0.63, 0.66, 0.6900000000000001, 0.72, 0.75, 0.78, 0.8099999999999999, 0.84, 0.87, 0.9, 0.93, 0.9600000000000001, 0.9900000000000001, 1.02, 1.05, 1.08, 1.11, 1.14, 1.17, 1.2, 1.23, 1.26, 1.29, 1.32, 1.3499999999999999, 1.3800000000000001, 1.41, 1.44, 1.4700000000000002, 1.5, 1.5300000000000002, 1.56, 1.59, 1.6199999999999999, 1.6500000000000001, 1.68, 1.71, 1.74, 1.77, 1.8, 1.8299999999999998, 1.86, 1.89, 1.9200000000000002, 1.95, 1.9800000000000002, 2.01, 2.04, 2.07, 2.1, 2.13, 2.16, 2.1900000000000004, 2.22, 2.25, 2.28, 2.31, 2.34, 2.37, 2.4, 2.4299999999999997, 2.46, 2.4899999999999998, 2.52, 2.55, 2.58, 2.6100000000000003, 2.64, 2.67, 2.6999999999999997, 2.7300000000000004, 2.7600000000000002, 2.79, 2.82, 2.8500000000000005, 2.88, 2.9099999999999997, 2.9400000000000004, 2.97, 3 ], [ 0, 0.030000000000000002, 0.060000000000000005, 0.09, 0.12000000000000001, 0.15, 0.18, 0.21, 0.24000000000000002, 0.27, 0.3, 0.32999999999999996, 0.36, 0.39, 0.42, 0.45, 0.48000000000000004, 0.51, 0.54, 0.57, 0.6, 0.63, 0.6599999999999999, 0.69, 0.72, 0.7500000000000001, 0.78, 0.8099999999999999, 0.84, 0.87, 0.9, 0.93, 0.9600000000000001, 0.9900000000000001, 1.02, 1.05, 1.08, 1.1099999999999999, 1.14, 1.17, 1.2, 1.23, 1.26, 1.29, 1.3199999999999998, 1.3499999999999999, 1.38, 1.41, 1.44, 1.47, 1.5000000000000002, 1.5300000000000002, 1.56, 1.59, 1.6199999999999999, 1.6500000000000001, 1.68, 1.71, 1.74, 1.77, 1.8, 1.83, 1.86, 1.89, 1.9200000000000002, 1.95, 1.9800000000000002, 2.01, 2.04, 2.07, 2.1, 2.13, 2.16, 2.19, 2.2199999999999998, 2.25, 2.28, 2.31, 2.34, 2.3700000000000006, 2.4, 2.4299999999999997, 2.46, 2.4899999999999998, 2.52, 2.5500000000000003, 2.58, 2.61, 2.6399999999999997, 2.6700000000000004, 2.6999999999999997, 2.73, 2.76, 2.7900000000000005, 2.82, 2.8500000000000005, 2.88, 2.91, 2.94, 2.9699999999999998, 3.0000000000000004 ], [ 0, 0.03, 0.06, 0.09, 0.12, 0.15000000000000002, 0.18, 0.21, 0.24, 0.27, 0.30000000000000004, 0.33, 0.36, 0.39, 0.42, 0.44999999999999996, 0.48, 0.51, 0.54, 0.57, 0.6000000000000001, 0.63, 0.66, 0.69, 0.72, 0.75, 0.78, 0.8099999999999999, 0.84, 0.87, 0.8999999999999999, 0.9299999999999999, 0.96, 0.99, 1.02, 1.05, 1.08, 1.11, 1.14, 1.17, 1.2000000000000002, 1.23, 1.26, 1.29, 1.32, 1.3499999999999999, 1.38, 1.4099999999999997, 1.44, 1.47, 1.5, 1.5300000000000002, 1.56, 1.5899999999999999, 1.6199999999999999, 1.6500000000000001, 1.68, 1.71, 1.74, 1.7700000000000002, 1.7999999999999998, 1.8299999999999998, 1.8599999999999999, 1.89, 1.92, 1.95, 1.98, 2.01, 2.04, 2.07, 2.1, 2.13, 2.16, 2.19, 2.22, 2.25, 2.28, 2.31, 2.34, 2.3700000000000006, 2.4000000000000004, 2.4299999999999997, 2.46, 2.4899999999999998, 2.52, 2.55, 2.58, 2.61, 2.64, 2.67, 2.6999999999999997, 2.7300000000000004, 2.76, 2.79, 2.8199999999999994, 2.8500000000000005, 2.88, 2.9099999999999997, 2.94, 2.97, 3 ], [ 0, 0.03, 0.06, 0.09, 0.12, 0.15, 0.18, 0.21, 0.24, 0.27, 0.3, 0.32999999999999996, 0.36, 0.39, 0.42, 0.44999999999999996, 0.48, 0.51, 0.54, 0.57, 0.6, 0.63, 0.6599999999999999, 0.69, 0.72, 0.7500000000000001, 0.78, 0.8099999999999999, 0.84, 0.8700000000000001, 0.8999999999999999, 0.93, 0.96, 0.9900000000000001, 1.02, 1.05, 1.08, 1.11, 1.14, 1.17, 1.2, 1.23, 1.26, 1.29, 1.3199999999999998, 1.3499999999999999, 1.38, 1.4100000000000001, 1.44, 1.47, 1.5000000000000002, 1.53, 1.56, 1.5899999999999999, 1.6199999999999999, 1.65, 1.68, 1.71, 1.7400000000000002, 1.77, 1.7999999999999998, 1.8299999999999998, 1.86, 1.89, 1.92, 1.95, 1.9800000000000002, 2.01, 2.04, 2.0700000000000003, 2.1, 2.13, 2.16, 2.19, 2.22, 2.25, 2.28, 2.31, 2.34, 2.3700000000000006, 2.4, 2.4299999999999997, 2.46, 2.4899999999999998, 2.52, 2.55, 2.58, 2.6100000000000003, 2.6399999999999997, 2.67, 2.6999999999999997, 2.73, 2.76, 2.79, 2.8200000000000003, 2.85, 2.88, 2.9099999999999997, 2.94, 2.9699999999999998, 3.0000000000000004 ], [ 0, 0.03, 0.06, 0.09000000000000001, 0.12, 0.15000000000000002, 0.18000000000000002, 0.21, 0.24, 0.27, 0.30000000000000004, 0.33, 0.36000000000000004, 0.39, 0.42, 0.44999999999999996, 0.48, 0.51, 0.54, 0.57, 0.6000000000000001, 0.63, 0.66, 0.6900000000000001, 0.7200000000000001, 0.75, 0.78, 0.8099999999999999, 0.84, 0.8700000000000001, 0.8999999999999999, 0.93, 0.96, 0.9900000000000001, 1.02, 1.05, 1.08, 1.11, 1.14, 1.1700000000000002, 1.2000000000000002, 1.23, 1.26, 1.29, 1.32, 1.3499999999999999, 1.3800000000000001, 1.41, 1.4400000000000002, 1.4700000000000002, 1.5, 1.5300000000000002, 1.56, 1.5899999999999999, 1.6199999999999999, 1.6500000000000001, 1.68, 1.71, 1.7400000000000002, 1.7700000000000002, 1.7999999999999998, 1.83, 1.86, 1.89, 1.92, 1.95, 1.9800000000000002, 2.01, 2.04, 2.07, 2.1, 2.13, 2.16, 2.19, 2.22, 2.25, 2.28, 2.31, 2.3400000000000003, 2.37, 2.4000000000000004, 2.4299999999999997, 2.46, 2.4899999999999998, 2.52, 2.55, 2.58, 2.6100000000000003, 2.64, 2.67, 2.6999999999999997, 2.73, 2.7600000000000002, 2.79, 2.82, 2.85, 2.8800000000000003, 2.91, 2.9400000000000004, 2.97, 3 ], [ 0, 0.029999999999999995, 0.05999999999999999, 0.09, 0.11999999999999998, 0.15, 0.18, 0.21, 0.23999999999999996, 0.27, 0.3, 0.32999999999999996, 0.36, 0.39, 0.42, 0.44999999999999996, 0.4799999999999999, 0.51, 0.54, 0.57, 0.6, 0.63, 0.6599999999999999, 0.69, 0.72, 0.75, 0.78, 0.8099999999999999, 0.84, 0.87, 0.8999999999999999, 0.9299999999999998, 0.9599999999999999, 0.9899999999999999, 1.02, 1.05, 1.08, 1.1099999999999999, 1.14, 1.17, 1.2, 1.23, 1.26, 1.29, 1.3199999999999998, 1.3499999999999999, 1.38, 1.41, 1.44, 1.47, 1.5, 1.53, 1.56, 1.5899999999999999, 1.6199999999999999, 1.65, 1.68, 1.71, 1.74, 1.77, 1.7999999999999998, 1.8299999999999998, 1.8599999999999997, 1.89, 1.9199999999999997, 1.95, 1.9799999999999998, 2.01, 2.04, 2.07, 2.1, 2.13, 2.16, 2.19, 2.2199999999999998, 2.25, 2.28, 2.31, 2.34, 2.37, 2.4, 2.4299999999999997, 2.46, 2.4899999999999998, 2.52, 2.55, 2.58, 2.61, 2.6399999999999997, 2.67, 2.6999999999999997, 2.73, 2.76, 2.79, 2.82, 2.8500000000000005, 2.88, 2.9099999999999997, 2.94, 2.97, 3 ], [ 0, 0.03, 0.06, 0.08999999999999998, 0.12, 0.15, 0.17999999999999997, 0.21, 0.24, 0.27, 0.3, 0.3299999999999999, 0.35999999999999993, 0.39, 0.42, 0.44999999999999996, 0.48, 0.51, 0.54, 0.57, 0.6, 0.63, 0.6599999999999998, 0.69, 0.7199999999999999, 0.75, 0.78, 0.8099999999999999, 0.84, 0.8699999999999999, 0.8999999999999999, 0.9299999999999998, 0.96, 0.99, 1.02, 1.05, 1.08, 1.1099999999999999, 1.14, 1.1699999999999997, 1.2, 1.23, 1.26, 1.2899999999999998, 1.3199999999999996, 1.3499999999999999, 1.38, 1.41, 1.4399999999999997, 1.4700000000000002, 1.5, 1.53, 1.56, 1.5899999999999999, 1.6199999999999999, 1.65, 1.68, 1.71, 1.7399999999999998, 1.77, 1.7999999999999998, 1.8299999999999998, 1.8599999999999997, 1.89, 1.92, 1.95, 1.98, 2.01, 2.04, 2.07, 2.1, 2.13, 2.16, 2.19, 2.2199999999999998, 2.25, 2.28, 2.31, 2.3399999999999994, 2.37, 2.4, 2.4299999999999997, 2.46, 2.4899999999999993, 2.52, 2.55, 2.5799999999999996, 2.61, 2.6399999999999992, 2.67, 2.6999999999999997, 2.73, 2.76, 2.7899999999999996, 2.82, 2.85, 2.8799999999999994, 2.9099999999999997, 2.9400000000000004, 2.9699999999999998, 3 ], [ 0, 0.029999999999999995, 0.05999999999999999, 0.09, 0.11999999999999998, 0.14999999999999997, 0.18, 0.21, 0.23999999999999996, 0.27, 0.29999999999999993, 0.32999999999999996, 0.36, 0.38999999999999996, 0.42, 0.44999999999999996, 0.4799999999999999, 0.51, 0.54, 0.57, 0.5999999999999999, 0.63, 0.6599999999999999, 0.69, 0.72, 0.75, 0.7799999999999999, 0.8099999999999998, 0.84, 0.87, 0.8999999999999999, 0.9299999999999999, 0.9599999999999999, 0.99, 1.02, 1.05, 1.08, 1.1099999999999999, 1.14, 1.17, 1.1999999999999997, 1.23, 1.26, 1.29, 1.3199999999999998, 1.3499999999999999, 1.38, 1.41, 1.44, 1.4699999999999998, 1.5, 1.53, 1.5599999999999998, 1.5899999999999999, 1.6199999999999997, 1.6499999999999997, 1.68, 1.71, 1.74, 1.77, 1.7999999999999998, 1.8299999999999998, 1.8599999999999999, 1.89, 1.9199999999999997, 1.95, 1.98, 2.01, 2.04, 2.0700000000000003, 2.1, 2.13, 2.16, 2.19, 2.2199999999999998, 2.25, 2.28, 2.31, 2.34, 2.3699999999999997, 2.3999999999999995, 2.4299999999999997, 2.46, 2.4899999999999998, 2.52, 2.55, 2.58, 2.61, 2.6399999999999997, 2.67, 2.6999999999999997, 2.73, 2.76, 2.79, 2.82, 2.85, 2.88, 2.9099999999999997, 2.9399999999999995, 2.97, 3 ], [ 0, 0.030000000000000002, 0.060000000000000005, 0.09, 0.12000000000000001, 0.15000000000000002, 0.18, 0.21, 0.24000000000000002, 0.2700000000000001, 0.30000000000000004, 0.32999999999999996, 0.36, 0.39, 0.42, 0.44999999999999996, 0.48000000000000004, 0.51, 0.5400000000000001, 0.5700000000000001, 0.6000000000000001, 0.6300000000000001, 0.6599999999999999, 0.69, 0.72, 0.75, 0.78, 0.8099999999999999, 0.84, 0.87, 0.8999999999999999, 0.93, 0.9600000000000001, 0.99, 1.02, 1.05, 1.0800000000000003, 1.1099999999999999, 1.1400000000000001, 1.17, 1.2000000000000002, 1.23, 1.2600000000000002, 1.29, 1.3199999999999998, 1.3499999999999999, 1.38, 1.41, 1.44, 1.4700000000000002, 1.5, 1.5300000000000002, 1.56, 1.59, 1.6199999999999999, 1.6500000000000001, 1.68, 1.7100000000000002, 1.74, 1.77, 1.7999999999999998, 1.8299999999999998, 1.86, 1.89, 1.9200000000000002, 1.95, 1.98, 2.01, 2.04, 2.0700000000000003, 2.1, 2.1300000000000003, 2.1600000000000006, 2.19, 2.2199999999999998, 2.25, 2.2800000000000002, 2.31, 2.34, 2.37, 2.4000000000000004, 2.4299999999999997, 2.46, 2.4899999999999998, 2.5200000000000005, 2.5500000000000003, 2.58, 2.61, 2.6399999999999997, 2.6700000000000004, 2.6999999999999997, 2.73, 2.76, 2.79, 2.82, 2.85, 2.88, 2.91, 2.9400000000000004, 2.97, 3 ], [ 0, 0.03, 0.06, 0.09000000000000001, 0.12, 0.15, 0.18000000000000002, 0.21, 0.24, 0.27, 0.3, 0.32999999999999996, 0.36000000000000004, 0.39, 0.42, 0.44999999999999996, 0.48, 0.51, 0.54, 0.57, 0.6, 0.63, 0.6599999999999999, 0.69, 0.7200000000000001, 0.7499999999999999, 0.78, 0.81, 0.84, 0.87, 0.8999999999999999, 0.9299999999999999, 0.96, 0.9899999999999999, 1.02, 1.0499999999999998, 1.08, 1.1099999999999999, 1.14, 1.17, 1.2, 1.23, 1.26, 1.29, 1.3199999999999998, 1.3499999999999999, 1.38, 1.41, 1.4400000000000002, 1.47, 1.4999999999999998, 1.5300000000000002, 1.56, 1.5899999999999999, 1.62, 1.65, 1.68, 1.71, 1.74, 1.77, 1.7999999999999998, 1.8299999999999998, 1.8599999999999999, 1.8899999999999997, 1.92, 1.9499999999999997, 1.9799999999999998, 2.01, 2.04, 2.0700000000000003, 2.0999999999999996, 2.13, 2.16, 2.19, 2.2199999999999998, 2.25, 2.28, 2.31, 2.34, 2.3699999999999997, 2.4, 2.4299999999999997, 2.46, 2.4899999999999998, 2.52, 2.5499999999999994, 2.58, 2.61, 2.6399999999999997, 2.67, 2.6999999999999997, 2.73, 2.76, 2.79, 2.82, 2.85, 2.8800000000000003, 2.9099999999999993, 2.94, 2.97, 2.9999999999999996 ], [ 0, 0.03, 0.06, 0.09, 0.12, 0.15, 0.18, 0.21, 0.24, 0.27, 0.3, 0.32999999999999996, 0.36, 0.39, 0.42, 0.4499999999999999, 0.48, 0.5099999999999999, 0.54, 0.57, 0.6, 0.63, 0.6599999999999999, 0.69, 0.72, 0.7499999999999999, 0.78, 0.8099999999999999, 0.84, 0.8700000000000001, 0.8999999999999998, 0.9299999999999998, 0.96, 0.99, 1.0199999999999998, 1.05, 1.08, 1.1099999999999999, 1.14, 1.17, 1.2, 1.23, 1.26, 1.2899999999999998, 1.3199999999999998, 1.3499999999999999, 1.38, 1.41, 1.44, 1.4700000000000002, 1.4999999999999998, 1.5299999999999998, 1.56, 1.5899999999999999, 1.6199999999999999, 1.65, 1.68, 1.71, 1.7400000000000002, 1.77, 1.7999999999999996, 1.8299999999999998, 1.8599999999999997, 1.89, 1.92, 1.95, 1.98, 2.0099999999999993, 2.0399999999999996, 2.07, 2.1, 2.13, 2.16, 2.19, 2.2199999999999998, 2.25, 2.28, 2.31, 2.34, 2.37, 2.4, 2.4299999999999997, 2.46, 2.4899999999999993, 2.52, 2.5499999999999994, 2.5799999999999996, 2.61, 2.6399999999999997, 2.67, 2.6999999999999997, 2.73, 2.76, 2.79, 2.82, 2.8500000000000005, 2.88, 2.91, 2.9400000000000004, 2.97, 2.9999999999999996 ], [ 0, 0.03, 0.06, 0.09, 0.12, 0.15, 0.18, 0.21, 0.24, 0.27, 0.3, 0.32999999999999996, 0.36, 0.39, 0.42, 0.44999999999999996, 0.48, 0.51, 0.54, 0.57, 0.6, 0.63, 0.6599999999999999, 0.69, 0.72, 0.75, 0.78, 0.8099999999999999, 0.84, 0.87, 0.8999999999999999, 0.9299999999999999, 0.96, 0.99, 1.02, 1.05, 1.08, 1.1099999999999999, 1.14, 1.17, 1.2, 1.23, 1.26, 1.29, 1.3199999999999998, 1.3499999999999999, 1.38, 1.41, 1.44, 1.47, 1.5, 1.53, 1.56, 1.5899999999999999, 1.6199999999999999, 1.65, 1.68, 1.71, 1.74, 1.77, 1.7999999999999998, 1.8299999999999998, 1.8599999999999999, 1.89, 1.92, 1.95, 1.98, 2.01, 2.04, 2.07, 2.1, 2.13, 2.16, 2.19, 2.2199999999999998, 2.25, 2.28, 2.31, 2.34, 2.37, 2.4, 2.4299999999999997, 2.46, 2.4899999999999998, 2.52, 2.55, 2.58, 2.61, 2.6399999999999997, 2.67, 2.6999999999999997, 2.73, 2.76, 2.79, 2.82, 2.85, 2.88, 2.9099999999999997, 2.94, 2.9699999999999998, 3 ] ] } ], "layout": {} }, "text/html": [ "
" ] }, "metadata": {}, "output_type": "display_data" } ], "source": [ "import plotly.plotly as py\n", "from plotly.offline import download_plotlyjs, init_notebook_mode, plot, iplot,iplot_mpl\n", "import plotly.graph_objs as go\n", "init_notebook_mode()\n", "\n", "import numpy as np\n", "\n", "r=np.linspace(0,3,101)\n", "t=np.linspace(0,2*np.pi,101)\n", "\n", "r,t=np.meshgrid(r,t)\n", "x=r*np.cos(t)\n", "y=r*np.sin(t)\n", "\n", "f= np.sqrt(x**2+y**2)\n", "\n", "surface = go.Surface(x=x, y=y, z=f,opacity=0.9)\n", "data = [surface]\n", "\n", "fig = go.Figure(data=data)\n", "iplot(fig)" ] }, { "cell_type": "markdown", "metadata": {}, "source": [ "Example\n", "---\n", " Find the both absolute extrema values of $f (x, y) =2x^2+y^2-4x-2y+3$ on the rectangle:\n", " $$ D=\\{0\\le x\\le3,0\\le y\\le2\\}$$\n", "1. critical value:\n", "\\begin{eqnarray*}\n", " & f_x = 4x-4 = 0 \\text{ and } f_y = 2y-2 = 0 & \\\\\n", " \\Longrightarrow & (x, y) = (1, 1) & \n", " \\end{eqnarray*}\n", "i.e. there are only one critical value, $(x,y) = (1,1)$ with $f(x,y)|_{(x,y)=(1,1)}=0$.\n", "- Let the boundary of $D$ be:\n", "$$L_1\\cup L_2\\cup L3\\cup L_4=\\{y=0\\}\\cup\\{x=3\\}\\cup\\{y=2\\}\\cup\\{x=0\\}$$\n", " - $L_1=\\{y=0\\}$:\n", " \\begin{eqnarray}\n", " f_{L_1}&=&f(x,0)\\text{ and } 0\\le x\\le3\\\\\n", " &=& 2x^2-4x\\\\\n", " f_{L_1}'=0&\\Rightarrow&4x-4=0\\to x=1 \\text{ and } x=3\\\\\n", " &\\rightarrow& f(1,0)=1,f(3,0)=9 \n", " \\end{eqnarray}\n", " - $L_2=\\{x=3\\}$:\n", " \\begin{eqnarray}\n", " f_{L_2}&=&f(y,3)\\text{ and } 0\\le y\\le2\\\\\n", " &=& y^2-2y+9\\\\\n", " f_{L_2}'=0&\\Rightarrow&2y-2=0\\to y=1 \\text{ and } y=0,2\\\\\n", " &\\rightarrow& f(3,1)=8,f(3,0)=f(3,2)=9 \n", " \\end{eqnarray} \n", " - $L_3=\\{y=2\\}$:\n", " \\begin{eqnarray}\n", " f_{L_3}&=&f(x,0)\\text{ and } 0\\le x\\le3\\\\\n", " &=& 2x^2-4x+3\\\\\n", " f_{L_3}'=0&\\Rightarrow&4x-4=0\\to x=1 \\text{ and } x=3\\\\\n", " &\\rightarrow& f(1,2)=1,f(3,2)=9 \n", " \\end{eqnarray} \n", " - $L_4=\\{x=0\\}$:\n", " \\begin{eqnarray}\n", " f_{L_4}&=&f(y,0)\\text{ and }0\\le y\\le2\\\\\n", " &=& y^2-2y+3\\\\\n", " f_{L_4}'=0&\\Rightarrow&2y-2=0\\to y=1 \\text{ and } y=0,2\\\\\n", " &\\rightarrow& f(0,1)=2,f(0,0)=f(0,2)=3\n", " \\end{eqnarray} \n", "Thus the absolute maximum is 9 and mabsolute minimum is 0. " ] }, { "cell_type": "markdown", "metadata": {}, "source": [ "Example\n", "---\n", " Suppose that $f (x, y) = 4 x - 2 y - x^2 - 2 y^2 + 2 x y - 10.$ Then the\n", " critical value comes from the following relations:\n", "\n", "\\begin{eqnarray*}\n", " & f_1 = 4 - 2 x + 2 y = 0 \\text{ and } f_2 = - 2 - 4 y + 2 x = 0 & \\\\\n", " \\Longrightarrow & (x, y) = (3, 1) & \n", " \\end{eqnarray*}\n", " i.e. only one critical value $(x, y) = (3, 1)$. By the way, we also have\n", "\n", "\\begin{eqnarray*}\n", " & H_2 = \\left(\\begin{array}{cc}\n", " f_{11} & f_{12}\\\\\n", " f_{21} & f_{22}\n", " \\end{array}\\right) = \\left(\\begin{array}{cc}\n", " - 2 & 2\\\\\n", " 2 & - 4\n", " \\end{array}\\right) & \\\\\n", " \\Longrightarrow & f_{11} = - 2 < 0 \\text{ and } D = f_{11} f_{22} -\n", " (f_{12})^2 = 4 > 0 & \n", " \\end{eqnarray*}\n", "\n", "Therefore $f (3, 1) = - 5$ is relative maximum." ] }, { "cell_type": "markdown", "metadata": {}, "source": [ "Example\n", "---\n", " $f (x, y) = x^4 + y^4 - 4 x y$,Then the critical values come from the\n", " following relations:\n", "\n", "\\begin{eqnarray*}\n", " & f_1 = 4 x^3 - 4 y = 0 \\text{ and } f_2 = 4 y^3 - 4 x = 0 & \\\\\n", " \\Longrightarrow & x = y^3 \\text{ and } y = x^3 (\\text{i.e. } x = x^9) & \\\\\n", " \\Longrightarrow & (x, y) = (0, 0) \\text{ or } (\\pm 1, \\pm 1) & \n", " \\end{eqnarray*}\n", " And the Hessian matrix is:\n", "\n", "$$ H = \\left(\\begin{array}{cc}\n", " 12 x^2 & - 4\\\\\n", " - 4 & 12 y^2\n", " \\end{array}\\right) $$\n", "\n", "**1.** at $(x, y) = (0, 0)$,\n", "\n", "$$ D = \\left| \\left(\\begin{array}{cc}\n", " 0 & - 4\\\\\n", " - 4 & 0\n", " \\end{array}\\right) \\right| = - 16 < 0 $$\n", " saddle point.\n", "\n", "**2.** at $(x, y) = (\\pm 1, \\pm 1)$:\n", "\n", "$$ D = \\left| \\left(\\begin{array}{cc}\n", " 12 & - 4\\\\\n", " - 4 & 12\n", " \\end{array}\\right) \\right| = 128 > 0 $$\n", "\n", "with $f_{11} (\\pm 1, \\pm 1) = 12 > 0$. Then $f (- 1, - 1) = - 2$ is a\n", " relative minimum and $f (1, 1) = - 128$ is also a relative minimum. " ] }, { "cell_type": "markdown", "metadata": {}, "source": [ "Example\n", "---\n", " Revisit the previous example $f (x, y, z) = 2 x^2 + y^2 + 4 z^2 - x - 2 y$.\n", " We have:\n", "\n", "\\begin{eqnarray*}\n", " f_1 & = & 4 x - 1\\\\\n", " f_2 & = & 2 y - 2\\\\\n", " f_3 & = & 8 z\\\\\n", " H & = & \\left(\\begin{array}{ccc}\n", " f_{11} & f_{12} & f_{13}\\\\\n", " f_{21} & f_{22} & f_{23}\\\\\n", " f_{31} & f_{32} & f_{33}\n", " \\end{array}\\right)\\\\\n", " & = & \\left(\\begin{array}{ccc}\n", " 4 & 0 & 0\\\\\n", " 0 & 2 & 0\\\\\n", " 0 & 0 & 8\n", " \\end{array}\\right)\n", " \\end{eqnarray*}\n", " From the last result, we have:\n", "\n", "\\begin{eqnarray*}\n", " |H| & = & 64 > 0\\\\\n", " |H_1 | & = & 4 > 0\\\\\n", " |H_2 | & = & 4 \\cdot 2 = 8 > 0\\\\\n", " |H_3 | & = & |H| > 0\n", " \\end{eqnarray*}\n", "\n", "Therefore, $f (x, y, z)$ actually attains its relative minimum at the\n", " critical point, $(1 / 4, 1, 0)$." ] }, { "cell_type": "code", "execution_count": 14, "metadata": {}, "outputs": [ { "name": "stdout", "output_type": "stream", "text": [ "the critical point is (1/4,1,0) and det(H3)=64, det(H2)=8\n" ] } ], "source": [ "from sympy import symbols,diff,solve,hessian\n", "\n", "f=2*x**2+y**2+4*z**2-x-2*y\n", "\n", "cpts=solve(grad(f,[x,y,z]),[x,y,z])\n", "H=hessian(f,[x,y,z]);H2=hessian(f,[x,y]) \n", "H_det=H.det();\n", "H2_det=H2.det() \n", "print(\"the critical point is (%s,%s,%s) and det(H3)=%s, det(H2)=%s\" %(cpts[x],cpts[y],cpts[z],H_det,H2_det)) " ] }, { "cell_type": "markdown", "metadata": {}, "source": [ "Example\n", "---\n", " If a company produces two products, $A$ and $B$, with prices 100 and 300\n", " respectively. The total cost in producing $x$ units of $A$ and $y$ units of\n", " $B$ is\n", "\n", "$$ C (x, y) = 2000 + 50 x + 80 y + x^2 + 2 y^2 $$\n", " Revenue $R = 100 x + 300 y$ and\n", "\n", "\\begin{eqnarray*}\n", " P (x, y) & = & R (x, y) - C (x, y)\\\\\n", " & = & - 2000 + 50 x + 220 y - x^2 - 2 y^2\n", " \\end{eqnarray*}\n", " with $o \\leqslant x, y$. First we want to find the critical point:\n", "\n", "\\begin{eqnarray*}\n", " \\nabla P = \\vec{0} & \\Longrightarrow & \\left( \\frac{\\partial P}{\\partial\n", " x}, \\frac{\\partial P}{\\partial y} \\right) = \\vec{0}\\\\\n", " & \\Longrightarrow & 50 - 2 x = 0 \\text{and} 220 - 4 y = 0\n", " \\end{eqnarray*}\n", " i.e. only one critical point $(x, y) = (25, 55)$. Also the Hessian matrix is\n", " as follows:\n", "\n", "$$ H = \\left(\\begin{array}{cc}\n", " - 2 & 0\\\\\n", " 0 & - 4\n", " \\end{array}\\right) $$\n", " $P_1 = - 2 < 0$ but $|H| = 8 > 0$. This means that $P (x, y)$ attains its\n", " relative maximum at $(25, 55)$. At this point, $P (x, y)$ attains its\n", " maximum too." ] }, { "cell_type": "markdown", "metadata": {}, "source": [ "Exercise\n", "---\n", " Find the relative extrema of $f (x, y) = x^3 + y^3 - 3 x y$ if any.\n", "\n", "**1.** Find the critical point(s) as follows:\n", "\n", "\\begin{eqnarray*}\n", " \\nabla f = \\vec{0} & \\Longrightarrow & (3 x^2 - 3 y, 3 y^2 - 3 x) = (0,0)\\\\\n", " & \\Longrightarrow & y = x^2 \\text{ and } x = y^2 (i.e. x = x^4)\\\\\n", " & \\Longrightarrow & (x, y) = (0, 0) \\text{ or } (1, 1)\n", " \\end{eqnarray*}\n", " There exist two critical points.\n", "**2.** Find Hessian matrix:\n", "\n", "$$ \\begin{array}{lll}\n", " H & = & \\left(\\begin{array}{cc}\n", " 6 x & - 3\\\\\n", " - 3 & 6 y\n", " \\end{array}\\right)\n", " \\end{array} $$\n", " - $(x, y) = (0, 0)$: $|H| = - 9 < 0 \\Rightarrow f (0, 0)$ is a saddle\n", " point.\n", " - $(x, y) = (1, 1)$: $|H| = 36 - 9 > 0$ and $f_{11} = 6 > 0\n", " \\Rightarrow f (1, 1)$ is a relative minimum.\n", "\n", "\n", "Exercise\n", "---\n", "\n", " Find the relative extrema of $f (x, y) = \\exp (- x^2 - y^2)$ if any.\n", "\n", "**1.** Find the critical point(s) as follows:\n", "\n", "\\begin{eqnarray*}\n", " \\nabla f = \\vec{0} & \\Longrightarrow & (- 2 x e^{- x^2 - y^2}, - 2 y e^{-\n", " x^2 - y^2}) = (0, 0)\\\\\n", " \\ & \\Longrightarrow & (x, y) = (0, 0)\n", " \\end{eqnarray*}\n", " There exists one critical point.\n", "\n", "**2.** Find Hessian matrix:\n", "\n", "$$ \\begin{array}{lll}\n", " H & = & \\left(\\begin{array}{cc}\n", " (4 x^2 - 2) e^{- x^2 - y^2} & 4 x y e^{- x^2 - y^2}\\\\\n", " 4 x y e^{- x^2 - y^2} & (4 y^2 - 2) e^{- x^2 - y^2}\n", " \\end{array}\\right)\n", " \\end{array} $$\n", "\n", "$(x, y) = (0, 0)$: $|H| = 4 > 0$ and $f_{11} (0, 0) = - 2 < 0\\Longrightarrow f (0, 0)$ is a relative maximum.\n", "\n", "\n" ] }, { "cell_type": "markdown", "metadata": {}, "source": [ "In general, it is not difficulty to find extrema for functions without any\n", "restriction. The following theorem is an extension in the case of all\n", "variables defined within intervals, i.e. $ x^i \\in [a^i, b^i]$,\n", "for all $1 \\le i \\le n$:\n", "\n", "Theorem\n", "---\n", " Suppose that\n", "\n", "$$ \\text{Domain} (f) = \\left\\{ \\mathbf{x} | x^i \\in\n", " [a^i, b^i], \\text{ for all } 1 \\leqslant i \\leqslant n \\} \n", " \\right. $$\n", " and $f (\\mathbf{x})$ is smooth with all the first order derivatives in\n", " domain. Suppose that $f (\\mathbf{x})$ attains it global maximum at\n", " $\\mathbf{x}=\\mathbf{x}^{\\ast}$, then one or both following condition(s)\n", " must hold:\n", "1. $f_i (\\mathbf{x}^{\\ast}) \\le 0$ and $((x^i)^{\\ast} - a^i_{})\n", " f_i (\\mathbf{x}^{\\ast}) = 0$;\n", "- $f_i (\\mathbf{x}^{\\ast}) \\ge 0$ and $(b^i - (x^i)^{\\ast})\n", " f_i (\\mathbf{x}^{\\ast}) = 0$.\n", " for all $1 \\leqslant i \\leqslant n$.\n", "\n", "Proof\n", "---\n", " \n", "\n", "Since the domain is in the rectangular form, there are only three\n", " possibilities in each direction:\n", "1. $\\mathbf{x}^{\\ast}$ is an interior point of domain, then $f_i(\\mathbf{x}^{\\ast}) = 0$ for any $i$ since it is smooth.\n", "- It is at boundary and suppose that it is increasing, then $(x^i)^{\\ast} = b^i$ and $f_i (\\mathbf{x}^{\\ast}) > 0$.\n", "- It is at boundary and suppose that it is decreasing, then $(x^i)^{\\ast} = a^i_{}$ and $f_i (\\mathbf{x}^{\\ast}) < 0$.\n", "\n", "And these prove this theorem. " ] }, { "cell_type": "markdown", "metadata": {}, "source": [ "Similarly, we have the following theorem to describe the behavior as minimum\n", "of function within bounded region:\n", "\n", "Theorem\n", "---\n", " Suppose that\n", "$$ \\text{Domain} (f) = \\left\\{ \\vec{x} | x_i \\in [a_i,\n", " b_i], \\text{ for all $1 \\leqslant i \\leqslant n$:} \\} \\right.\n", " $$\n", " and $f (\\mathbf{x})$ is smooth with all its first order derivatives.\n", " Suppose that $f (\\mathbf{x})$ attains it global minimum at\n", " $\\mathbf{x}=\\mathbf{x}^{\\ast}$, then one or both following condition(s)\n", " must hold:\n", "1. $f_i (\\mathbf{x}^{\\ast}) \\geqslant 0$ and $((x^i)^{\\ast} - a^i_{})\n", " f_i (\\mathbf{x}^{\\ast}) = 0$;\n", "- $f_i (\\mathbf{x}^{\\ast}) \\leqslant 0$ and $(b^i - (x^i)^{\\ast})\n", " f_i (\\mathbf{x}^{\\ast}) = 0$\n", "\n", " for all $1 \\leqslant i \\leqslant n$.\n", "\n", " \n", "**Proof**\n", "\n", " \n", "\n", "There are only three possibilities in each direction:\n", "1. $\\mathbf{x}^{\\ast}$ is an interior point of domain, then $f_i\n", " (\\mathbf{x}^{\\ast}) = 0$ for any $i$ since it is smooth.\n", "- It is at boundary and suppose that it is decreasing, then $(x^i)^{\\ast} = b^i$ and $f_i (\\mathbf{x}^{\\ast}) < 0$.\n", "- It is at boundary and suppose that it is increasing, then $(x^i)^{\\ast} = a^i_{}$ and $f_i (\\mathbf{x}^{\\ast}) > 0$.\n", "\n", " And these prove this theorem.\n", "\n", "\n" ] }, { "cell_type": "markdown", "metadata": {}, "source": [ "Example\n", "---\n", " Suppose that one factory inputs its goods from two different supply plants,\n", " $A$ and $B$, with different costs, $4$ and $6$ each respective. And suppose\n", " the price function in the market is the same and is decided as $p (x, y) =\n", " 100 - x - y$ where $x$ and $y$ are the demand functions. Discuss the the\n", " maxima problem due to the following situations:\n", "1. $0 \\leqslant x, y$\n", "- $0 \\leqslant x \\leqslant 30$, $0 \\leqslant y \\leqslant 30$\n", "\n", "**Solve**\n", " \n", "**1.** The profit function is\n", "\n", "\\begin{eqnarray*}\n", " P (x, y) & = & p (x, y) (x + y) - 4 x - 6 y\\\\\n", " & = & 100 (x + y) - (x + y)^2 - 4 x - 6 y\n", " \\end{eqnarray*}\n", " with all its first derivative:\n", "\n", "\\begin{eqnarray*}\n", " P_x (x, y) & = & 100 - 2 (x + y) - 4\\\\\n", " P_y (x, y) & = & 100 - 2 (x + y) - 6\n", " \\end{eqnarray*}\n", " Obviously, it is impossible to get the equations simultaneously:\n", "\n", "\\begin{eqnarray*}\n", " P_x (x, y) = 0 & \\Longrightarrow & 100 - 2 (x + y) - 4 = 0\\\\\n", " P_y (x, y) = 0 & \\Longrightarrow & 100 - 2 (x + y) - 6 = 0\n", " \\end{eqnarray*}\n", "\n", "since it is absolutely inconsistent. Does it imply no maximum for $P (x,y)$? Certainly not. Think about it: if you can input more cheaper goods,\n", "$4$ each, why not do so? The first derivative test can only be used to\n", "confirm that the maximum does not occur at the point, i.e. satisfying $P_x= P_y = 0$.\n", " \n", "\n", "In this case, we always choose to input from $A$ with cost $4$ each but not from $B$. This just implies:\n", "\n", "\\begin{eqnarray*}\n", " y = 0 & \\text{ and } & P (x, y) = 100 x - x^2 - 4 x\n", " \\end{eqnarray*}\n", "\n", "Then $P (x, y)$ attains its maximum at $(x, y) = (48, 0)$. Note that we have\n", "\n", "\\begin{eqnarray*}\n", " \\vec{x}^{\\ast} = (x^{\\ast}, y^{\\ast}) = (48, 0) & \\Longrightarrow &\n", " (x^{\\ast} - 0) P_x (\\vec{x}^{\\ast}) = 0 \\text{ and } P (\\vec{x}^{\\ast}) =\n", " 0\\\\\n", " & \\Longrightarrow & P_y (x^{\\ast}, y^{\\ast}) = - 2 - 2 y^{\\ast} < 0\n", " \\text{ and } (y^{\\ast} - 0) P_x (\\vec{x}^{\\ast}) = 0\n", " \\end{eqnarray*}\n", "\n", "**2.** Since the it is beneficial for us to input from $A$, certainly $x$ is equal to $30$, the maximal output from $A$. This concludes\n", "\n", "$$ P (x, y) = P (30, y) = 100 (30 + y) - (30 + y)^2 - 10 - 6 y $$\n", "and\n", "\n", "\\begin{eqnarray*}\n", " P_x (x, y) = 0 & \\Longrightarrow & 100 - 2 (x + y) - 4 = 0\\\\\n", " P_y (x, y) = 0 & \\Longrightarrow & 100 - 2 (x + y) - 6 = 0\n", " \\end{eqnarray*}\n", " \n", " \n", "By the first derivative test, $P (x, y)$ will attains its maxima at $x =30$ (boundary point) and $y = 17$, coming from the fact, $P_y (30, y) =0$. In this case,\n", "\n", "\\begin{eqnarray*}\n", " \\vec{x}^{\\ast} = (x^{\\ast}, y^{\\ast}) = (30, 17) & \\Longrightarrow & (30\n", " - x^{\\ast}) P_x (\\vec{x}^{\\ast}) = 0 \\text{ and } P_x (\\vec{x}^{\\ast}) <\n", " 0\\\\\n", " & \\Longrightarrow & P_y (\\vec{x}^{\\ast}) = 0\n", " \\end{eqnarray*}\n" ] }, { "cell_type": "markdown", "metadata": {}, "source": [ "Example\n", "---\n", "Two kinds of eggs, white and brown, are sold. The daily sales for white eggs\n", " will be $W (x, y) = 30 - 15 x + 3 y$ and daily sales for brown eggs will be\n", " $B (x, y) = 20 - 12 y + 2 x$ where $x, y$ are the sale prices for white and\n", " brown eggs respectively. Then the revenue is\n", "\n", "\\begin{eqnarray*}\n", " R (x, y) & = & x W (x, y) + y B (x, y)\\\\\n", " & = & x (30 - 15 x + 3 y) + y (20 - 12 y + 2 x)\\\\\n", " & = & 30 x + 20 y - 15 x^2 + 5 x y - 12 y^2\n", " \\end{eqnarray*}\n", " The critical point is calculated as follows:\n", "\n", "\\begin{eqnarray*}\n", " \\nabla R = \\vec{0} & \\Longrightarrow & (30 - 30 x + 5 y, 20 + 5 x - 24 y)\n", " = (0, 0)\\\\\n", " & \\Longrightarrow & (x, y) = \\left( \\frac{164}{139}, \\frac{150}{139}\n", " \\right)\n", " \\end{eqnarray*}\n", " Also the Hessian matrix is as follows:\n", "\n", "\\begin{eqnarray*}\n", " H & = & \\left(\\begin{array}{cc}\n", " - 30 & 5\\\\\n", " 5 & - 24\n", " \\end{array}\\right)\n", " \\end{eqnarray*}\n", " $R_1 = - 30 < 0$ and $|H| > 0$. Then at $(x, y) = \\left(\\frac{164}{139},\n", " \\frac{150}{139}\\right)$, $R (x, y)$ attains its relative maximum and maximum too." ] }, { "cell_type": "markdown", "metadata": {}, "source": [ "Example\n", "---\n", "\n", "Suppose that a firm produces one kind of output, $y$, by two inputs, $K$,\n", " called capital, and $L$, called labor. Each unit of capital costs $u$ and\n", " each unit of labor costs $v$. And the production function follows the\n", " Cobb-Douglas relation:\n", "\n", "$$ P (K, L) = A K^{\\alpha} L^{\\beta} \\text{ where } A, \\alpha \\text{ and } \\beta\n", " > 0 $$\n", "If the price function is constant $p$, find the condition at which the\n", " profit function attains its extremum and determine the condition at which\n", " profit function actually attains its maximum at the critical point.\n", " \n", "**Sol**\n", " \n", "\n", " The profit function, by definition, is\n", "\n", "\\begin{eqnarray*}\n", " P (K, L) & = & R (K, L) - C (K, L)\\\\\n", " & = & p A K^{\\alpha} L^{\\beta} - u K - v L\n", " \\end{eqnarray*}\n", "By the first order condition, the extremum occurs at which\n", "\n", "\\begin{eqnarray*}\n", " 0 = \\frac{\\partial P}{\\partial K} & = p \\alpha A K^{\\alpha - 1}\n", " L^{\\beta} - u & \\\\\n", " 0 = \\frac{\\partial P}{\\partial L} & = p \\beta A K^{\\alpha} L^{\\beta - 1}\n", " - v & \n", " \\end{eqnarray*}\n", "Then\n", "\n", "\\begin{eqnarray*}\n", " & \\frac{L}{K} & = \\frac{u \\beta}{v \\alpha}\\\\\n", " \\Longrightarrow & p \\alpha A K^{\\alpha - 1} \\left( \\frac{u \\beta}{v\\alpha} K \\right)^{\\beta} & = u\\\\\n", " \\Longrightarrow & \\hat{K} & = \\left( \\frac{v^{\\beta} \\alpha^{\\beta -\n", " 1}}{p A u^{\\beta - 1} \\beta^{\\beta}} \\right)^{\\frac{1}{\\alpha + \\beta -1}}\\\\\n", " \\Longrightarrow & \\hat{L} & = \\left( \\frac{u^{\\alpha} \\beta^{\\alpha -\n", " 1}}{p A v^{\\alpha - 1} \\alpha^{\\alpha}} \\right)^{\\frac{1}{\\alpha + \\beta - 1}}\n", "\\end{eqnarray*}\n", "\n", "If $P (K, L)$ attains its maxima at $(\\hat{K}, \\hat{L})$, then\n", "\n", "\\begin{eqnarray*}\n", " & |H| > 0 \\text{ and } |H_1 | < 0 & \\\\\n", " \\Longrightarrow & \\frac{\\partial^2 P}{\\partial L^2} = p \\alpha (\\alpha -1) A K^{\\alpha - 2} L^{\\beta} < 0 & \\\\\n", " & |H| = (p A)^2 \\alpha (\\alpha - 1) \\beta (\\beta - 1) K^{\\alpha + \\beta - 2} L^{\\alpha + \\beta - 2} & \\\\\n", " & - (p A \\alpha \\beta)^2 K^{\\alpha + \\beta - 2} L^{\\alpha + \\beta - 2}< 0 & \\\\\n", " & & \\\\\n", " \\Longrightarrow & \\alpha < 1 \\text{ and } \\alpha + \\beta < 1 & \n", "\\end{eqnarray*}" ] }, { "cell_type": "code", "execution_count": null, "metadata": { "collapsed": true }, "outputs": [], "source": [] }, { "cell_type": "code", "execution_count": 7, "metadata": { "collapsed": true }, "outputs": [], "source": [ "def plot3d(x,y,z):\n", " fig = plt.figure()\n", " ax = Axes3D(fig)\n", " ax.plot_surface(x, y, z, rstride=1, cstride=1, cmap=plt.cm.binary,alpha=0.9)\n", " #ax.contour(x, y, z, lw=3, cmap=\"autumn_r\", linestyles=\"solid\", zdir='z',offset=0)\n", " ax.set_xlabel('X')\n", " ax.set_ylabel('Y')\n", " ax.set_zlabel('Z')\n", " ax.set_zlim(-7, 25)\n", " #ax.scatter3D([1],[1],[0],color=(0,0,0));\n", " ax.arrow(x=1,y=1,dx=0.1,dy=0.1)\n", " t=np.linspace(0,2*np.pi,101)\n", " xt=np.cos(t)\n", " yt=np.sin(t)\n", " zt=xt**2-xt*yt+yt**2-xt+yt-6\n", " ax.plot3D(xt,yt,zt)\n", " #ax.plot3D(xt,yt,np.sqrt(xt)+np.sqrt(yt))" ] }, { "cell_type": "code", "execution_count": 53, "metadata": {}, "outputs": [ { "data": { "image/png": "iVBORw0KGgoAAAANSUhEUgAAAcUAAAE1CAYAAACWU/udAAAABHNCSVQICAgIfAhkiAAAAAlwSFlz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\nKAIA4BCKAAA43yi/FQldfza9AAAAAElFTkSuQmCC\n", "text/plain": [ "" ] }, "metadata": {}, "output_type": "display_data" } ], "source": [ "x=np.linspace(-3,3,101)\n", "y=np.linspace(-3,3,101)\n", "\n", "x,y=np.meshgrid(x,y)\n", "f= x**2-x*y+y**2-x+y-6\n", "plot3d(x,y,f)" ] }, { "cell_type": "code", "execution_count": 10, "metadata": {}, "outputs": [ { "ename": "NameError", "evalue": "name 'iplot' is not defined", "output_type": "error", "traceback": [ "\u001b[0;31m---------------------------------------------------------------------------\u001b[0m", "\u001b[0;31mNameError\u001b[0m Traceback (most recent call last)", "\u001b[0;32m\u001b[0m in \u001b[0;36m\u001b[0;34m()\u001b[0m\n\u001b[1;32m 23\u001b[0m \u001b[0;34m\u001b[0m\u001b[0m\n\u001b[1;32m 24\u001b[0m \u001b[0mfig\u001b[0m \u001b[0;34m=\u001b[0m \u001b[0mgo\u001b[0m\u001b[0;34m.\u001b[0m\u001b[0mFigure\u001b[0m\u001b[0;34m(\u001b[0m\u001b[0mdata\u001b[0m\u001b[0;34m=\u001b[0m\u001b[0mdata\u001b[0m\u001b[0;34m)\u001b[0m\u001b[0;34m\u001b[0m\u001b[0m\n\u001b[0;32m---> 25\u001b[0;31m \u001b[0miplot\u001b[0m\u001b[0;34m(\u001b[0m\u001b[0mfig\u001b[0m\u001b[0;34m)\u001b[0m\u001b[0;34m\u001b[0m\u001b[0m\n\u001b[0m", "\u001b[0;31mNameError\u001b[0m: name 'iplot' is not defined" ] } ], "source": [ "import plotly.graph_objs as go\n", "\n", "x=np.linspace(-3,3,101)\n", "y=np.linspace(-3,3,101)\n", "\n", "x,y=np.meshgrid(x,y)\n", "f= x**2-x*y+y**2-x+y-6\n", "g=x**2-x*y+y**2\n", "t=np.linspace(0,2*np.pi,101)\n", "xt=np.cos(t)\n", "yt=np.sin(t)\n", "zt=xt**2-xt*yt+yt**2-xt+yt-6\n", "surface = go.Surface(x=x, y=y, z=f,opacity=0.9)\n", "surface1 = go.Scatter3d(x=xt, y=yt, z=zt,\n", " mode = \"lines\",\n", " line = dict(color='orange',width = 5)\n", " ) \n", "surface2 = go.Scatter3d(x=xt, y=yt, z=0*zt,\n", " mode = \"lines\",\n", " line = dict(color='red',width = 5)\n", " )\n", "data = [surface,surface1,surface2]\n", "\n", "fig = go.Figure(data=data)\n", "iplot(fig)" ] }, { "cell_type": "markdown", "metadata": {}, "source": [ "Sometimes, what the range of domain does influences the extrema.\n", "\n", "Example\n", "---\n", " Find the extrema of $f (x, y) = x^2 - x y + y^2 - x + y - 6$ for $(x, y)$\n", " within the following region $\\Omega$ respectively:\n", "1. $\\Omega = \\{(x, y) | x^2 + y^2 \\le 1\\}$;\n", "- $\\Omega = \\{(x, y) | - 2 \\le x, y \\le 2\\}$;\n", "- $\\Omega = \\{(x, y) | 0 \\le x \\le 1, 0 \\le y\n", " \\leqslant x\\}$.\n", " \n", " \n", "**Sol:** First we want to find the critical point(s) as follows:\n", "\n", "\\begin{eqnarray*}\n", " \\vec{0} & = & (f_1, f_2)\\\\\n", " & = & (2 x - y - 1, 2 y - x + 1)\\\\\n", " \\Longrightarrow & & x = 1 / 3 \\text{ and } y = - 1 / 3\n", " \\end{eqnarray*}\n", "\n", "**1.** Critical point is within this region and $f (1 / 3, - 1 / 3) = - 6\n", " \\frac{1}{3}$. And the possible positions for $f (x, y)$ attaining its\n", " extrema is at the boundary, $\\partial \\Omega = \\{(x, y) | x^2 + y^2 =\n", " 1\\}$, i.e. $x = \\cos \\theta$ and $y = \\sin \\theta$, $0 \\le \\theta\n", " \\le 2 \\pi$. On the boundary,\n", " \n", "\\begin{eqnarray*}\n", " f (x, y) & = & f (\\cos \\theta, \\sin \\theta)\\\\\n", " & = & \\cos^2 \\theta - \\sin \\theta \\cos \\theta + \\sin^2 \\theta - \\cos\n", " \\theta + \\sin \\theta - 6\\\\\n", " & = & - \\sin \\theta \\cos \\theta - \\cos \\theta + \\sin \\theta - 5\\\\\n", " \\frac{d f}{d \\theta} = 0 & \\Longrightarrow & \\sin^2 \\theta - \\cos^2\n", " \\theta + \\sin \\theta + \\cos \\theta = 0\\\\\n", " & \\Longrightarrow & (\\sin \\theta + \\cos \\theta) (\\sin \\theta - \\cos\n", " \\theta + 1) = 0\n", " \\end{eqnarray*}\n", "   **a.** $\\sin \\theta + \\cos \\theta = 0$: $\\theta = 3 \\pi / 4$ and $7 \\pi /\n", " 4$, this implies\n", "\n", "\\begin{eqnarray*}\n", " f \\left( \\cos \\frac{3 \\pi}{4}, \\sin \\frac{3 \\pi}{4} \\right) & = & -\n", " \\sin \\frac{3 \\pi}{4} \\cos \\frac{3 \\pi}{4} - \\cos \\frac{3 \\pi}{4} +\n", " \\sin \\frac{3 \\pi}{4} - 5 = - 4 \\frac{1}{2}\\\\\n", " f \\left( \\cos \\frac{7 \\pi}{4}, \\sin \\frac{7 \\pi}{4} \\right) & = & - 4\n", " \\frac{1}{2}\n", " \\end{eqnarray*}\n", "   **b.** $\\sin \\theta - \\cos \\theta + 1 = 0$: $\\theta = 0$ and $3 \\pi / 2$,\n", " this implies\n", "\n", "\\begin{eqnarray*}\n", " f (\\cos 0, \\sin 0) & = & - 6\\\\\n", " f \\left(\\cos \\frac{3 \\pi}{2}, \\sin \\frac{3 \\pi}{2}\\right) & = & - 6\n", " \\end{eqnarray*}\n", "These conclude: maximum is $- 4 \\frac{1}{2}$ and minimum is $- 6\n", " \\frac{1}{3}$ at $(x, y) = (1 / 3, - 1 / 3)$.\n", "\n", "**2.** In this square case, $\\partial \\Omega$ contains four components:
\n", "  **a.** $I_1 = \\{(x, y) | - 2 \\leqslant x \\leqslant 2, y = - 2\\}$;
\n", "  **b.** $I_2 = \\{(x, y) | - 2 \\leqslant y \\leqslant 2, x = 2\\}$;
\n", "  **c.** $I_3 = \\{(x, y) | - 2 \\leqslant x \\leqslant 2, y = 2\\}$;
\n", "  **d.** $I_4 = \\{(x, y) | - 2 \\leqslant y \\leqslant 2, x = - 2\\}$;
\n", "\n", " - For $f (x, y) |_{I_1} = f (x, - 2) = x^2 + x - 4, - 2 \\leqslant x \\leqslant 2$:\n", " - maximum: $2$ at $(x, y) = (2, - 2)$;\n", " - minimum: $- 4 \\frac{1}{4}$ at $(x, y) = (- 1 / 2, - 2)$.\n", " - $f (x, y) |_{I_2} = f (2, y) = y^2 - y - 4^{}, - 2\\leqslant y \\leqslant 2$:\n", " - maximum: 2 at $(x, y) = (2, - 2)$;\n", " - minimum: $- 4 \\frac{1}{4}$ at $(x, y) = (2, 1 / 2)$.\n", " - $f (x, y) |_{I_3} = f (x, 2) = x^2 - 3 x, - 2 \\leqslant x\\leqslant 2$:\n", " - maximum: 10 at $(x, y) = (- 2, 2)$;\n", " - minimum: $- 9 / 4$ at $(x, y) = (3 / 2, 2)$.\n", " - $f (x, y) |_{I_4} = f (- 2, y) = y^2 + 3 y, - 2 \\leqslant y \\leqslant 2$:\n", " - maximum: 10 at $(x, y) = (- 2, 2)$;\n", " - minimum: $- 9 / 4$ at $(x, y) = (- 2, - 3 / 2)$.\n", "\n", "These conclude: maximum is $10$ at $(-2,2)$ and minimum is $- 6\n", " \\frac{1}{3}$ at $(x, y) = (1 / 3, - 1 / 3)$.\n", "\n", "**3.** the last case, we have these conclusions: maximum is $10$ at $(x, y) = (- 2,\n", " 2)$ and minimum is $- 6 \\frac{1}{3}$ at $(x, y) = (1 / 3, - 1 / 3)$.\n", "Since critical point is not included. Then the extrema might\n", " attain at the boundary $\\partial \\Omega$ as follows:\n", " - $I_1 = \\{(x, y) | 0 \\leqslant x \\leqslant 1, y = 0\\}$;\n", " - $I_2 = \\{(x, y) | 0 \\leqslant y \\leqslant 1, x = 1\\}$;\n", " - $I_3 = \\{(x, y) | 0 \\leqslant x \\leqslant 1, y = x\\}$;\n", "\n", "and\n", "\n", " - $f (x, y) |_{I_1} = f (x, 0) = x^2 - x - 6, 0 \\leqslant x\\leqslant 1$:\n", " - maximum : - 6 at $(x, y) = (0, 0)$ ;\n", " - minimum : $- 6 \\frac{1}{4}$ at $(x, y) = (1 / 2, 0)$.\n", " - $f (x, y) |_{I_2} = f (1, y) = y^2 - 6, 0 \\leqslant y\\leqslant 1$:\n", " - maximum: - 5 at $(x, y) = (1, 1)$;\n", " - minimum: - 6 at $(x, y) = (0, 1)$.\n", " - $f (x, y) |_{I_3} = f (x, x) = x^2 - 6, 0 \\leqslant x \\leqslant 1$:\n", " - maximum: - 5 at $(x, y) = (1, 1)$;\n", " - minimum: - 6 at }$(x, y) = (0, 0)$.\n", "\n", "These conclude:\n", " maximum is $- 5$ at $(x, y) = (1, 1)$ and minimum is $- 6 \\frac{1}{4}$ at\n", " $(x, y) = (1 / 2, 0)$ .\n" ] }, { "cell_type": "markdown", "metadata": {}, "source": [ "Example\n", "---\n", "Find the extrema of $f (x, y, z) = (x - 1)^2 + (y - 1)^2 + (z - 1)^2$ for $x+ y + z \\le 4$ and $0 \\leqslant x, y, z$.\n", " \n", " The critical point is $(1, 1, 1)$ (with $f (1, 1, 1) = 0$) since\n", " $$ \\nabla f = 2 (x - 1, y - 1, z - 1) = \\vec{0} $$\n", " and it is within the domain. Since $f (x, y, z) \\geqslant 0$, then $f (x, y,\n", " z)$ attain its minimum at $(1, 1, 1)$. Another positions at which $f (x, y,\n", " z)$ possibly attains its extrema (or maximum) are on the boundary:\n", "- $I_1 : \\{(x, y, 0) | x + y \\leqslant 4, x, y \\geqslant 0\\}$;\n", "- $I_2 : \\{(x, 0, z) | x + z \\leqslant 4, x, z \\geqslant 0\\}$;\n", "- $I_3 : \\{(0, y, z) | y + z \\leqslant 4, y, z \\geqslant 0\\}$;\n", "- $I_4 : \\{(x, y, z) | x + y + z = 4, x, y, z \\geqslant 0\\}$\n", "\n", " \n", " \n", "By the symmetry, the maximum in $I_1, I_2, I_3$ are the same. Therefore only\n", " $I_1$ and $I_4$ are necessarily to be considered:\n", "1. On $I_1$: $g (x, y) = f (x, y, 0) = (x - 1)^2 + (y - 1)^2 + 1$\n", " - critical point is $(1, 1)$ (with $g (1, 1) = 1$) since\n", " $$ \\nabla g = 2 (x - 1, y - 1) = \\vec{0} $$\n", " - boundary points:\n", " - $0 \\leqslant x \\leqslant 4, y = 0$: $g (x, y) = (x - 1)^2 + 2$\n", " implies that maximum is 11 at $x = 4$.\n", " - $0 \\leqslant y \\leqslant 4, x = 0$: $g (x, y) = (y - 1)^2 + 2$\n", " implies that maximum is 11 at $y = 4$.\n", " - $x + y = 4, x, y \\geqslant 0$: $g (x, y) = 2 x^2 - 8 x + 11$, $0\n", " \\leqslant x \\leqslant 4$ implies that maximum is 11 at $x = 0$ or 4.\n", "- On $I_4$: $f (x, y, z) = (x - 1)^2 + (y - 1)^2 + (z - 1)^2$ with $x\n", " + y + z = 4, x, y, z \\geqslant 0$. This can be solved the following\n", " technique, Lagrange multiplier, introduced in the following section:\n", "\n", "Let\n", "\n", "\\begin{eqnarray*}\n", " L (x, y, z, \\lambda) & = & f (x, y, z) + \\lambda (4 - x - y - z)\\\\\n", " & = & (x - 1)^2 + (y - 1)^2 + (z - 1)^2 + \\lambda (4 - x - y - z)\n", " \\end{eqnarray*}\n", "and\n", "\n", "\\begin{eqnarray*}\n", " \\nabla L = \\vec{0} & \\Longrightarrow & 2 (x - 1) = 2 (y - 1) = 2 (z -\n", " 1) = \\lambda\n", " \\end{eqnarray*}\n", " By symmetry, $x = y = z$, then\n", "\n", "\\begin{eqnarray*}\n", " x + y + z = 4 & \\Rightarrow & x = y = z = 4 / 3\n", " \\end{eqnarray*}\n", " This implies $f (x, y, z)$ attains its minimum $1 / 3$ (why?) at this point $(4 / 3, 4 / 3, 4 / 3)$, but not maximum.\n", "\n", "\n", "Therefore the maximum is 11 and minimum is 0.\n", " \n", " " ] }, { "cell_type": "code", "execution_count": 87, "metadata": {}, "outputs": [ { "data": { "text/plain": [ "{x: l/2 + 1, z: l/2 + 1, y: l/2 + 1}" ] }, "execution_count": 87, "metadata": {}, "output_type": "execute_result" } ], "source": [ "x,y,z,l=symbols('x y z l')\n", "f=(x-1)**2+(y-1)**2+(z-1)**2\n", "cond=4-x-y-z\n", "F=f+l*cond\n", "solve(grad(F,[x,y,z]),[x,y,z])" ] }, { "cell_type": "markdown", "metadata": {}, "source": [ "Exercise\n", "---\n", "Suppose that $f (x, y) = 3 x + 4 y$. Find the extrema of $f (x, y)$ in the\n", " following regions $\\Omega$ respectively:\n", "1. $\\Omega = \\{(x, y) | - 2 \\leqslant x, y \\leqslant 2\\}$;\n", "- $\\Omega = \\{(x, y) | x^2 + y^2 \\leqslant 2^2 \\}$;\n", "\n", " \n", "1.\n", "\\begin{eqnarray*}\n", " \\partial \\Omega & = & \\{y = - 2\\} \\cup \\{x = 2\\} \\cup \\{y = 2\\} \\cup \\{x =\n", " - 2\\}\\\\\n", " & = & I_1 \\cup I_2 \\cup I_3 \\cup I_4\n", " \\end{eqnarray*}\n", " And the extrema appear at the ends, i.e. the four corners of the region, ($-\n", " 2, - 2), (2, - 2), (2, 2)$ and (-2,2). And the functions values at these\n", " points are $- 14, - 2, 14$ and $2$. Therefore, the maximum is $14$ and\n", " minimum is $- 14$\n", "- Since $\\partial \\Omega = \\{x^2 + y^2 = 4\\}$, consider the Lagrangian\n", " function:\n", " $$ L (x, y, \\lambda) = f (x, y) + \\lambda (4 - x^2 - y^2) $$\n", " Then\n", "\n", "\\begin{eqnarray*}\n", " \\nabla L = \\vec{0} & \\Longrightarrow & (3 - 2 \\lambda x, 4 - 2 \\lambda y,\n", " 4 - x^2 - y^2) = \\vec{0}\\\\\n", " & \\Longrightarrow & x = \\frac{3}{2 \\lambda}, y = \\frac{2}{\\lambda}\\\\\n", " & \\Longrightarrow & y = \\frac{4}{3} x\\\\\n", " & \\Longrightarrow & (x, y) = \\left( \\pm \\frac{6}{5}, \\pm \\frac{8}{5}\n", " \\right)\n", " \\end{eqnarray*}\n", " Then the maximum is $f (6 / 5, 8 / 5) = 10$ and minimum is $f (- 6 / 5, - 8\n", " / 5) = - 10$.\n", "\n", "\n", "Exercise\n", "---\n", " Suppose that $f (x, y) = x^2 + y^2$. Find the extrema of $f (x, y)$ in the\n", " following regions $\\Omega$ respectively:\n", "1. $\\Omega = \\{(x, y) | - 2 \\leqslant x, y \\leqslant 2\\}$;\n", "- $\\Omega = \\{(x, y) | 3 x + 4 y \\leqslant 5, 0 \\leqslant x, y\\}$;\n", "\n", "**1.**\n", "\n", "\\begin{eqnarray*}\n", " \\partial \\Omega & = & \\{y = - 2\\} \\cup \\{x = 2\\} \\cup \\{y = 2\\} \\cup \\{x =\n", " - 2\\}\\\\\n", " & = & I_1 \\cup I_2 \\cup I_3 \\cup I_4\n", " \\end{eqnarray*}\n", " And the extrema appear at the ends, i.e. the four corners of the region, ($-\n", " 2, - 2), (2, - 2), (2, 2)$ and (-2,2). The functions values at these points\n", " are all 8 and $0 \\leqslant x^2 + y^2$. Therefore, the maximum is $8$ and\n", " minimum is $0$.\n", "\n", "**2.** Since\n", "\n", "\\begin{eqnarray*}\n", " \\partial \\Omega & = & \\{y = 0, 0 \\leqslant x \\leqslant 5 / 3\\} \\cup \\{x =\n", " 0, 0 \\leqslant y \\leqslant 5 / 4\\} \\cup \\{3 x + 4 y = 5\\}\\\\\n", " & = & I_1 \\cup I_2 \\cup I_3\n", " \\end{eqnarray*}\n", " Consider the Lagrangian function:\n", " $$ L (x, y, \\lambda) = f (x, y) + \\lambda (5 - 3 x - 4 y) $$\n", " Then\n", " 1. $(x, y) \\in I_1$: $f (x, y) = x^2$ and $0 \\leqslant x \\leqslant 5 /\n", " 3$. Then maximum is $25 / 9$ and minimum is 0.\n", " - $(x, y) \\in I_2$: $f (x, y) = y^2$ and $0 \\leqslant y \\leqslant 5 /\n", " 4$. Then maximum is $25 / 16$ and minimum is 0.\n", " - $(x, y) \\in I_3$\n", " \n", "\\begin{eqnarray*}\n", " \\nabla L = \\vec{0} & \\Longrightarrow & (2 x - 3 \\lambda x, 2 y - 4 \\lambda\n", " y, 5 - 3 x - 4 y) = \\vec{0}\\\\\n", " & \\Longrightarrow & x = \\frac{2}{3 \\lambda}, y = \\frac{1}{2 \\lambda}\\\\\n", " & \\Longrightarrow & y = \\frac{3}{4} x\\\\\n", " & \\Longrightarrow & (x, y) = \\left( \\frac{5}{6}, \\frac{5}{8} \\right)\n", " \\end{eqnarray*}\n", " Then the value of $f (x, y)$ is $625 / 576$ and is the minimum on $I_3$\n", " (why?). These conclude that minimum is 0 and maximum is 25/9.\n" ] }, { "cell_type": "markdown", "metadata": {}, "source": [ "P.1125\n", "---\n", "Classify the types of extrema or saddle point of $f(x,y)$:
\n", "**14. ** $f(x,y)=xy((3-x-y)$" ] }, { "cell_type": "code", "execution_count": 12, "metadata": {}, "outputs": [ { "data": { "text/plain": [ "" ] }, "execution_count": 12, "metadata": {}, "output_type": "execute_result" }, { "data": { "image/png": "iVBORw0KGgoAAAANSUhEUgAAArMAAAFbCAYAAADGGAZpAAAABHNCSVQICAgIfAhkiAAAAAlwSFlz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\n", "text/plain": [ "" ] }, "metadata": {}, "output_type": "display_data" } ], "source": [ "fig = plt.figure(figsize=(12,6))\n", "ax = fig.add_subplot(1, 1, 1, projection='3d')\n", "\n", "x=np.linspace(-1,3.1,101)\n", "y=np.linspace(-1,3.1,101)\n", "f=x*y*(3-x-y)\n", "x,y=np.meshgrid(x,y)\n", "ax.plot_surface(x,y,f)\n", "ax.set_title(\"$f(x,y)=x y (3-x-y)$\")" ] }, { "cell_type": "code", "execution_count": 6, "metadata": {}, "outputs": [ { "name": "stdout", "output_type": "stream", "text": [ "(0, 0)\n", "the critical point is (0,0) and det(H2)=-9\n", "(0, 3)\n", "the critical point is (0,3) and det(H2)=-9\n", "(1, 1)\n", "the critical point is (1,1) and det(H2)=3\n", "(3, 0)\n", "the critical point is (3,0) and det(H2)=-9\n" ] } ], "source": [ "from sympy import hessian,symbols,solve,diff,pprint\n", "\n", "grad = lambda func, vars :[diff(func,var) for var in vars]\n", "\n", "x,y,z=symbols(\"x y z\",real=True)\n", "f=x*y*(3-x-y)\n", "cpts=solve(grad(f,[x,y]),[x,y])\n", "H=hessian(f,[x,y]);\n", "#H2=hessian(f,[x,y]) \n", "H_det=H.det();\n", "#H2_det=H2.det() \n", "#print(cpts,H_det,H2_det) \n", "for cpt in cpts:\n", " print(cpt)\n", " det_H_val=H_det.subs({x:cpt[0],y:cpt[1]})\n", " #det_H2_val=H2_det.subs({x:cpt[0],y:cpt[1]})\n", " print(\"the critical point is (%s,%s) and det(H2)=%s\" %(cpt[0],cpt[1],det_H_val)) " ] }, { "cell_type": "code", "execution_count": 7, "metadata": { "collapsed": true }, "outputs": [], "source": [ "def criticaltype(f):\n", " cpts=solve(grad(f,[x,y]),[x,y])\n", " H=hessian(f,[x,y]);\n", " H_det=H.det();\n", " print(\"Hessian Matrix\\n---\")\n", " pprint(H)\n", " #H2_det=H.det()\n", " num=1\n", " if len(cpts)==0:\n", " print(\" no critical point!\") \n", " elif (type(cpts)==dict):\n", " \"\"\"\n", " If only one critical point, return {x:a,y:b} --- dict,\n", " if more than one point return {(a,b),(c,d),...} --- list\n", " \"\"\" \n", " cx=cpts[x]\n", " cy=cpts[y]\n", " print(\"only one critical (x,y)=(%s,%s)\" %(cx,cy))\n", " delta2=H_det.subs({x:cx,y:cy}) \n", " if delta2<0:\n", " print(\" |H|=%s<0: Saddle point here.\" %delta2)\n", " elif delta2==0:\n", " print(\" |H|=0: No conclusion.\") \n", " else:\n", " f1=diff(f,x,2).subs({x:cx,y:cy})\n", " if f1>0:\n", " print(\" |H|=%s>0, fxx=%s>0: local minimum here.\" %(delta2,f1))\n", " else:\n", " print(\" |H|=%s>0, fxx=%s<0: local maximum here.\" %(delta2,f1))\n", " else:\n", " for i in cpts: \n", " cx=i[0]\n", " cy=i[1]\n", " print(\"%d. critical (x,y)=(%s,%s)\" %(num,cx,cy))\n", " delta2=H_det.subs({x:cx,y:cy}) \n", " if delta2<0:\n", " print(\" |H|=%s<0: Saddle point here.\" %delta2)\n", " elif delta2==0:\n", " print(\" |H|=0: No conclusion.\") \n", " else:\n", " f1=diff(f,x,2).subs({x:cx,y:cy})\n", " if f1>0:\n", " print(\" |H|=%s>0, fxx=%s>0: local minimum here.\" %(delta2,f1))\n", " else:\n", " print(\" |H|=%s>0, fxx=%s<0: local maximum here.\" %(delta2,f1))\n", " #print(H_det)\n", " num+=1\n", " \n", " " ] }, { "cell_type": "code", "execution_count": 28, "metadata": {}, "outputs": [ { "name": "stdout", "output_type": "stream", "text": [ "Hessian Matrix\n", "---\n", "⎡ -2⋅y -2⋅x - 2⋅y + 3⎤\n", "⎢ ⎥\n", "⎣-2⋅x - 2⋅y + 3 -2⋅x ⎦\n", "1. critical (x,y)=(0,0)\n", " |H|=-9<0: Saddle point here.\n", "2. critical (x,y)=(0,3)\n", " |H|=-9<0: Saddle point here.\n", "3. critical (x,y)=(1,1)\n", " |H|=3>0, fxx=-2<0: local maximum here.\n", "4. critical (x,y)=(3,0)\n", " |H|=-9<0: Saddle point here.\n" ] } ], "source": [ "f=x*y*(3-x-y)\n", "\n", "criticaltype(f)" ] }, { "cell_type": "markdown", "metadata": {}, "source": [ "1. $\\nabla f =(y(3-x-y)-xy,x(3-x-y)-xy)=(y(3-2x-y),x(3-x-2y))=(0,0)$\n", "- Hessian matrix\n", "\\begin{eqnarray*}\n", " H & = & \\left(\\begin{array}{cc}\n", " - 2y & 3-2x-2y\\\\\n", " 3-2x-2y & - 2x\n", " \\end{array}\\right)\n", " \\end{eqnarray*}\n", "- critical values:
\n", " a. $y=0$ implies $x=0$ or $3-x=0$, i.e. $(0,0),(3,0)$; ($H<0$)
\n", " b. $3-2x-y=0$ implies \n", " - $x=0$ and $y=3$, ($H<0$)\n", " - $3-x-2y=0$ implies $(x,y)=(1,1)$ ($H>0,a<0$)\n", " " ] }, { "cell_type": "markdown", "metadata": {}, "source": [ "**16. ** $f(x,y)=4y/(x^2+y^2+1)$" ] }, { "cell_type": "code", "execution_count": 10, "metadata": {}, "outputs": [ { "data": { "text/plain": [ "" ] }, "execution_count": 10, "metadata": {}, "output_type": "execute_result" }, { "data": { "image/png": "iVBORw0KGgoAAAANSUhEUgAAArMAAAFbCAYAAADGGAZpAAAABHNCSVQICAgIfAhkiAAAAAlwSFlz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\nRK5CYII=\n", "text/plain": [ "" ] }, "metadata": {}, "output_type": "display_data" } ], "source": [ "fig = plt.figure(figsize=(12,6))\n", "ax = fig.add_subplot(1, 1, 1, projection='3d')\n", "\n", "x=np.linspace(-2,2,101)\n", "y=np.linspace(-2,2,101)\n", "f=4*y/(1+x*x+y*y)\n", "x,y=np.meshgrid(x,y)\n", "ax.plot_surface(x,y,f,alpha=0.6)\n", "ax.set_title(\"$f(x,y)=4y/(1+x^2+y^2)$\")" ] }, { "cell_type": "code", "execution_count": 29, "metadata": {}, "outputs": [ { "name": "stdout", "output_type": "stream", "text": [ "Hessian Matrix\n", "---\n", "⎡ 2 2 ⎤\n", "⎢ 32⋅x ⋅y 8⋅y 32⋅x⋅y 8⋅x ⎥\n", "⎢────────────── - ────────────── ────────────── - ──────────────⎥\n", "⎢ 3 2 3 2⎥\n", "⎢⎛ 2 2 ⎞ ⎛ 2 2 ⎞ ⎛ 2 2 ⎞ ⎛ 2 2 ⎞ ⎥\n", "⎢⎝x + y + 1⎠ ⎝x + y + 1⎠ ⎝x + y + 1⎠ ⎝x + y + 1⎠ ⎥\n", "⎢ ⎥\n", "⎢ 2 3 ⎥\n", "⎢ 32⋅x⋅y 8⋅x 32⋅y 24⋅y ⎥\n", "⎢────────────── - ────────────── ────────────── - ──────────────⎥\n", "⎢ 3 2 3 2⎥\n", "⎢⎛ 2 2 ⎞ ⎛ 2 2 ⎞ ⎛ 2 2 ⎞ ⎛ 2 2 ⎞ ⎥\n", "⎣⎝x + y + 1⎠ ⎝x + y + 1⎠ ⎝x + y + 1⎠ ⎝x + y + 1⎠ ⎦\n", "1. critical (x,y)=(0,-1)\n", " |H|=4>0, fxx=2>0: local minimum here.\n", "2. critical (x,y)=(0,1)\n", " |H|=4>0, fxx=-2<0: local maximum here.\n" ] } ], "source": [ "f=4*y/(x**2+y**2+1)\n", "criticaltype(f)" ] }, { "cell_type": "markdown", "metadata": {}, "source": [ "1. $\\nabla f =(-8xy/(x^2+y^2+1)^2,(4x^2-4y^2+4)/(x^2+y^2+1)^2)=(0,0)$\n", "- Hessian matrix\n", "\\begin{eqnarray*}\n", " H & = & \\frac{8}{(3x^2+y^2+1)^3}\\left(\\begin{array}{cc}\n", " y(x^2-1-y^2) & x(3y^2-1-x^2)\\\\\n", " x(3y^2-x^2-1) & y(x^2-3y^2+1)\n", " \\end{array}\\right)\n", " \\end{eqnarray*}\n", "- critical values: $x=0$ implies $y=\\pm1$, i.e. $(0,1),(0,-1)$; ($H>0$)
\n", " a. $(x,y)=(0,1)$: $H>0,a<0$\n", " b. $(x,y)=(0,-1)$: $H>0,a>0$\n", " " ] }, { "cell_type": "markdown", "metadata": {}, "source": [ "**36. ** Find the absolute extrema of $f(x,y)=3x^2+2xy+y^2$ on the triangle with vertices $(-2,-1),(1,-1),(1,2)$." ] }, { "cell_type": "code", "execution_count": 4, "metadata": {}, "outputs": [ { "data": { "text/plain": [ "" ] }, "execution_count": 4, "metadata": {}, "output_type": "execute_result" }, { "data": { "image/png": "iVBORw0KGgoAAAANSUhEUgAAAcUAAAHFCAYAAACDweKEAAAABHNCSVQICAgIfAhkiAAAAAlwSFlz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\nKhQKhUJRQomiQqFQKBQl/j8s6rfdYWq++gAAAABJRU5ErkJggg==\n", "text/plain": [ "" ] }, "metadata": {}, "output_type": "display_data" } ], "source": [ "fig = plt.figure(figsize=(6,6))\n", "ax = Axes3D(fig)\n", "\n", "X = np.linspace(-2, 1, 30)\n", "X1 = np.linspace(-2, 1, 100)\n", "X2 = np.linspace(-2, 1, 100)\n", "Y2 = X2+1\n", "\n", "Y = np.linspace(-1, 2, 30)\n", "Y1 = np.linspace(-1, 2, 100)\n", "\n", "Zs = 3*X1**2-2*X1+1\n", "Zt = 3+2*Y1+Y1**2\n", "\n", "X,Y=np.meshgrid(X,Y)\n", "func= 3*X*X+2*X*Y+Y*Y\n", "base=0*X\n", "\n", "Z2 = 3*X2*X2+2*X2*Y2+Y2*Y2\n", "\n", "\n", "ax.scatter(X1,0*X1-1,Zs, color=\"red\",alpha=0.9)\n", "ax.scatter(0*Y1+1,Y1,Zt,color=\"red\",alpha=0.9)\n", "ax.scatter(X2,Y2, Z2,color=\"red\",alpha=0.9)\n", "\n", "ax.plot_surface(X,Y, func, rstride=1, cstride=1, cmap=cm.jet,alpha=0.4)\n", "\n", "ax.plot_surface(X,Y, base, rstride=1, cstride=1, cmap=cm.jet,alpha=0.2)\n" ] }, { "cell_type": "code", "execution_count": 30, "metadata": {}, "outputs": [ { "name": "stdout", "output_type": "stream", "text": [ "Hessian Matrix\n", "---\n", "⎡6 2⎤\n", "⎢ ⎥\n", "⎣2 2⎦\n", "only one critical (x,y)=(0,0)\n", " |H|=8>0, fxx=6>0: local minimum here.\n" ] } ], "source": [ "f=3*x**2+2*x*y+y**2\n", "criticaltype(f)" ] }, { "cell_type": "code", "execution_count": 30, "metadata": {}, "outputs": [ { "data": { "text/plain": [ "0" ] }, "execution_count": 30, "metadata": {}, "output_type": "execute_result" } ], "source": [ "f.subs({x:0,y:0})" ] }, { "cell_type": "code", "execution_count": 31, "metadata": {}, "outputs": [ { "data": { "text/plain": [ "[1/3]" ] }, "execution_count": 31, "metadata": {}, "output_type": "execute_result" } ], "source": [ "#L1: (-2,-1) to (1,-1), x in [-2,1]\n", "f1=f.subs({y:-1})\n", "solve(diff(f1,x),x)" ] }, { "cell_type": "code", "execution_count": 32, "metadata": {}, "outputs": [ { "name": "stdout", "output_type": "stream", "text": [ "17\n", "0.666666666666667\n", "2\n" ] } ], "source": [ "for i in [-2,1/3,1]:\n", " print(f1.subs({x:i}))" ] }, { "cell_type": "code", "execution_count": 19, "metadata": {}, "outputs": [ { "data": { "text/plain": [ "[-1]" ] }, "execution_count": 19, "metadata": {}, "output_type": "execute_result" } ], "source": [ "#L2: (1,-1) to (1,2), y in [-1,2]\n", "f2=f.subs({x:1})\n", "solve(diff(f2,y),y)" ] }, { "cell_type": "code", "execution_count": 20, "metadata": {}, "outputs": [ { "name": "stdout", "output_type": "stream", "text": [ "2\n", "11\n" ] } ], "source": [ "for i in [-1,2]:\n", " print(f2.subs({y:i}))" ] }, { "cell_type": "code", "execution_count": 21, "metadata": {}, "outputs": [ { "data": { "text/plain": [ "[-1/3]" ] }, "execution_count": 21, "metadata": {}, "output_type": "execute_result" } ], "source": [ "#L3: (1,2) to (-2,-1), y-x=1 for x in [-2,1]\n", "f3=f.subs({x:x,y:x+1})\n", "solve(diff(f3,x),x)" ] }, { "cell_type": "code", "execution_count": 22, "metadata": {}, "outputs": [ { "name": "stdout", "output_type": "stream", "text": [ "17\n", "0.333333333333333\n", "11\n" ] } ], "source": [ "for i in [-2,-1/3,1]:\n", " print(f3.subs({x:i}))" ] }, { "cell_type": "markdown", "metadata": {}, "source": [ "These conclude: maximum is $17$, munimum is $0$." ] }, { "cell_type": "code", "execution_count": null, "metadata": { "collapsed": true }, "outputs": [], "source": [] }, { "cell_type": "markdown", "metadata": {}, "source": [ "**40. ** Find the absolute extrema of $f(x,y)=4x^2+2x+y^2-y$ on the elipse $4x^2+y^2\\le1$." ] }, { "cell_type": "code", "execution_count": 8, "metadata": {}, "outputs": [ { "data": { "image/png": "iVBORw0KGgoAAAANSUhEUgAAAcUAAAHFCAYAAACDweKEAAAABHNCSVQICAgIfAhkiAAAAAlwSFlz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\n", "text/plain": [ "" ] }, "metadata": {}, "output_type": "display_data" } ], "source": [ "fig = plt.figure(figsize=(6,6))\n", "ax = Axes3D(fig)\n", "\n", "X = np.linspace(-1, 1, 100)\n", "t = np.linspace(0, 2*np.pi, 100)\n", "Xt = np.cos(t)/2\n", "Yt = np.sin(t)\n", "\n", "Y = np.linspace(-1, 1, 30)\n", "Y1 = np.linspace(-1, 1, 100)\n", "\n", "\n", "X,Y=np.meshgrid(X,Y)\n", "func= 4*X*X+2*X+Y*Y-Y\n", "\n", "base=0*X\n", "\n", "ax.scatter(Xt,Yt,4*Xt**2+2*Xt+Yt**2-Yt, color=\"red\",alpha=0.9)\n", "\n", "\n", "ax.plot_surface(X,Y, func, rstride=1, cstride=1, cmap=cm.jet,alpha=0.4)\n", "\n", "ax.plot_surface(X,Y, base, rstride=1, cstride=1, cmap=cm.jet,alpha=0.2)\n", "ax.view_init(azim=-110,elev=25)" ] }, { "cell_type": "code", "execution_count": 31, "metadata": {}, "outputs": [ { "name": "stdout", "output_type": "stream", "text": [ "Hessian Matrix\n", "---\n", "⎡8 0⎤\n", "⎢ ⎥\n", "⎣0 2⎦\n", "only one critical (x,y)=(-1/4,1/2)\n", " |H|=16>0, fxx=8>0: local minimum here.\n" ] } ], "source": [ "f=4*x**2+y**2+2*x-y\n", "criticaltype(f)" ] }, { "cell_type": "code", "execution_count": 34, "metadata": {}, "outputs": [ { "data": { "text/plain": [ "-0.500000000000000" ] }, "execution_count": 34, "metadata": {}, "output_type": "execute_result" } ], "source": [ "f.subs({x:-1/4,y:1/2})" ] }, { "cell_type": "code", "execution_count": 35, "metadata": { "collapsed": true }, "outputs": [], "source": [ "# boundary\n", "from sympy import sin,cos,pi\n", "t=symbols(\"t\",real=True)\n", "ft=f.subs({x:cos(t)/2,y:sin(t)})" ] }, { "cell_type": "code", "execution_count": 36, "metadata": {}, "outputs": [ { "name": "stdout", "output_type": "stream", "text": [ "f(sqrt(2)/4,-sqrt(2)/2)=1 + sqrt(2)\n", "f(-sqrt(2)/4,sqrt(2)/2)=-sqrt(2) + 1\n" ] } ], "source": [ "cpts=solve(diff(ft,t),t)\n", "for cpt in cpts:\n", " fval=ft.subs({t:cpt})\n", " xt=cos(cpt)/2\n", " yt=sin(cpt)\n", " print(\"f(%s,%s)=%s\" %(xt,yt,fval))" ] }, { "cell_type": "markdown", "metadata": {}, "source": [ "These conclude: maximum is $1+\\sqrt2$, munimum is $-0.5$." ] }, { "cell_type": "markdown", "metadata": {}, "source": [ "**44.** Find the point on the surface $xy^2z=4$ that are closest to the origin and what is the shortest distance between them?" ] }, { "cell_type": "code", "execution_count": 32, "metadata": {}, "outputs": [ { "name": "stdout", "output_type": "stream", "text": [ "Hessian Matrix\n", "---\n", "⎡ 320 128 ⎤\n", "⎢2 + ───── ───── ⎥\n", "⎢ 6 2 5 3 ⎥\n", "⎢ x ⋅y x ⋅y ⎥\n", "⎢ ⎥\n", "⎢ 128 96 ⎥\n", "⎢ ───── 2 + ─────⎥\n", "⎢ 5 3 4 4⎥\n", "⎣ x ⋅y x ⋅y ⎦\n", "1. critical (x,y)=(-2**(3/4),-2**(1/4))\n", " |H|=64>0, fxx=12>0: local minimum here.\n", "2. critical (x,y)=(-2**(3/4),2**(1/4))\n", " |H|=64>0, fxx=12>0: local minimum here.\n", "3. critical (x,y)=(2**(1/4)*sqrt(2),-2**(1/4))\n", " |H|=64>0, fxx=12>0: local minimum here.\n", "4. critical (x,y)=(2**(1/4)*sqrt(2),2**(1/4))\n", " |H|=64>0, fxx=12>0: local minimum here.\n" ] } ], "source": [ "f=x**2+y**2+(4/x**2/y)**2\n", "criticaltype(f)" ] }, { "cell_type": "markdown", "metadata": {}, "source": [ "**62.** Let $f(x,y)=x^2-y^2+2xy+2$.\n", "1. no extrema since $f(x,y)\\to\\infty$, if $x\\to\\infty,y=0$, and $f(x,y)\\to-\\infty$, if $y\\to\\infty,x=0$.\n", "- Find extrema on $D=\\{x^2+4y^2\\le4\\}$" ] }, { "cell_type": "code", "execution_count": 34, "metadata": {}, "outputs": [ { "name": "stdout", "output_type": "stream", "text": [ "Hessian Matrix\n", "---\n", "⎡2 2 ⎤\n", "⎢ ⎥\n", "⎣2 -2⎦\n", "only one critical (x,y)=(0,0)\n", " |H|=-8<0: Saddle point here.\n" ] } ], "source": [ "f=x**2-y**2+2*x*y+2\n", "criticaltype(f)" ] }, { "cell_type": "code", "execution_count": 37, "metadata": {}, "outputs": [ { "name": "stdout", "output_type": "stream", "text": [ "[2*atan(-5/4 + sqrt(2)*sqrt(-5*sqrt(41) + 41)/4 + sqrt(41)/4), -2*atan(5/4 + sqrt(41)/4 + sqrt(2)*sqrt(5*sqrt(41) + 41)/4), -2*atan(-sqrt(41)/4 + sqrt(2)*sqrt(-5*sqrt(41) + 41)/4 + 5/4), -2*atan(-sqrt(2)*sqrt(5*sqrt(41) + 41)/4 + 5/4 + sqrt(41)/4)]\n", "f(2*cos(2*atan(-5/4 + sqrt(2)*sqrt(-5*sqrt(41) + 41)/4 + sqrt(41)/4)),sin(2*atan(-5/4 + sqrt(2)*sqrt(-5*sqrt(41) + 41)/4 + sqrt(41)/4)))=4*sin(2*atan(-5/4 + sqrt(2)*sqrt(-5*sqrt(41) + 41)/4 + sqrt(41)/4))*cos(2*atan(-5/4 + sqrt(2)*sqrt(-5*sqrt(41) + 41)/4 + sqrt(41)/4)) - sin(2*atan(-5/4 + sqrt(2)*sqrt(-5*sqrt(41) + 41)/4 + sqrt(41)/4))**2 + 4*cos(2*atan(-5/4 + sqrt(2)*sqrt(-5*sqrt(41) + 41)/4 + sqrt(41)/4))**2 + 2\n", "f(2*cos(2*atan(5/4 + sqrt(41)/4 + sqrt(2)*sqrt(5*sqrt(41) + 41)/4)),-sin(2*atan(5/4 + sqrt(41)/4 + sqrt(2)*sqrt(5*sqrt(41) + 41)/4)))=-sin(2*atan(5/4 + sqrt(41)/4 + sqrt(2)*sqrt(5*sqrt(41) + 41)/4))**2 - 4*sin(2*atan(5/4 + sqrt(41)/4 + sqrt(2)*sqrt(5*sqrt(41) + 41)/4))*cos(2*atan(5/4 + sqrt(41)/4 + sqrt(2)*sqrt(5*sqrt(41) + 41)/4)) + 2 + 4*cos(2*atan(5/4 + sqrt(41)/4 + sqrt(2)*sqrt(5*sqrt(41) + 41)/4))**2\n", "f(2*cos(2*atan(-sqrt(41)/4 + sqrt(2)*sqrt(-5*sqrt(41) + 41)/4 + 5/4)),-sin(2*atan(-sqrt(41)/4 + sqrt(2)*sqrt(-5*sqrt(41) + 41)/4 + 5/4)))=-4*sin(2*atan(-sqrt(41)/4 + sqrt(2)*sqrt(-5*sqrt(41) + 41)/4 + 5/4))*cos(2*atan(-sqrt(41)/4 + sqrt(2)*sqrt(-5*sqrt(41) + 41)/4 + 5/4)) - sin(2*atan(-sqrt(41)/4 + sqrt(2)*sqrt(-5*sqrt(41) + 41)/4 + 5/4))**2 + 4*cos(2*atan(-sqrt(41)/4 + sqrt(2)*sqrt(-5*sqrt(41) + 41)/4 + 5/4))**2 + 2\n", "f(2*cos(2*atan(-sqrt(2)*sqrt(5*sqrt(41) + 41)/4 + 5/4 + sqrt(41)/4)),-sin(2*atan(-sqrt(2)*sqrt(5*sqrt(41) + 41)/4 + 5/4 + sqrt(41)/4)))=-sin(2*atan(-sqrt(2)*sqrt(5*sqrt(41) + 41)/4 + 5/4 + sqrt(41)/4))**2 - 4*sin(2*atan(-sqrt(2)*sqrt(5*sqrt(41) + 41)/4 + 5/4 + sqrt(41)/4))*cos(2*atan(-sqrt(2)*sqrt(5*sqrt(41) + 41)/4 + 5/4 + sqrt(41)/4)) + 2 + 4*cos(2*atan(-sqrt(2)*sqrt(5*sqrt(41) + 41)/4 + 5/4 + sqrt(41)/4))**2\n" ] } ], "source": [ "ft=f.subs({x:2*cos(t),y:sin(t)})\n", "cpts=solve(diff(ft,t),t)\n", "print(cpts)\n", "for cpt in cpts:\n", " fval=ft.subs({t:cpt})\n", " xt=2*cos(cpt)\n", " yt=sin(cpt)\n", " print(\"f(%s,%s)=%s\" %(xt,yt,fval))" ] }, { "cell_type": "markdown", "metadata": {}, "source": [ "Note\n", "---\n", "For $(x,y)\\in\\partial D$,\n", "\\begin{eqnarray}\n", " f(x,y)&=&f(2\\cos t,\\sin t)\\\\\n", " &=&4\\cos^2t-\\sin^2t +4\\sin t\\cos t+2 \\\\\n", " &=& 2(1+\\cos 2t)-\\frac{1}{2}(1-\\cos2t)+2\\sin2t+2\\\\\n", " &=&\\frac{5}{2}\\cos2t+2\\sin2t-\\frac{3}{2}\\\\\n", " \\Rightarrow && -\\sqrt{(5/2)^2+2^2}+2\\le f\\le\\sqrt{(5/2)^2+2^2}+2\n", "\\end{eqnarray}" ] }, { "cell_type": "markdown", "metadata": {}, "source": [ "Optimization Problem with Constraints\n", "---\n", "\n", "Theorem\n", "---\n", " If a relative extrema of $f (x, y)$ and $g (x, y) = 0$ occurs at $(x, y)$,\n", " then there exists a $\\lambda$ for which $(x, y, \\lambda)$ is the critical\n", " point, $(a,b)$, of $L = f (x, y) + \\lambda g (x, y)$, i.e.:\n", "$$ \\nabla f(a,b)+\\lambda\\nabla g(a,b)=\\mathbf{0}$$\n", "\n", " This function is called\n", " **Lagrangian** of $f (x)$ and $g (x)$.\n", "\n", " \n", "\n", "In real world, Lagrangian is more realistic than other kinds of optimization\n", "problem: finding extrema under resources limited.\n", "\n", "For general case, $\\mathbf{x}\\in \\mathbb{R}^n$, the extrema of $f(\\mathbf{x}$ with constraint $g(\\mathbf{x}=0$ at $\\mathbf{x}_0$ should satisfy the following:\n", "$$ \\nabla f(\\mathbf{x}_0)+\\lambda\\nabla g(\\mathbf{x}_0)=\\mathbf{0}$$\n", "where $\\lambda\\ne0$ and $\\nabla g(\\mathbf{x}_0)\\ne\\mathbf{0}$." ] }, { "cell_type": "markdown", "metadata": {}, "source": [ "Example\n", "---\n", " Suppose that the Cobb-Douglas function is\n", " $$ P (x, y) = 100 x^{1 / 4} y^{3 / 4} $$\n", " where $x$ is the unit of labor and $y$ is the unit of capital. Each unit of\n", " labor costs $200$ and each of capital costs 100. If the total of 800 worth\n", " of labor and capital is to be used, Find the maximum of $P (x, y)$.\n", " \n", "First note:\n", "\n", "the critical point at which $P (x, y)$ attains it maximum is also the\n", " critical point for $\\ln P (x, y)$ that attains its maximum.\n", "\n", " \n", " Since the constraint is\n", "\n", "$$ 200 x + 100 y = 800 $$\n", " consider the Lagrangian function:\n", "\n", "\\begin{eqnarray*}\n", " L (x, y, \\lambda) & = & \\ln P (x, y) + \\lambda (800 - 200 x - 100 y)\\\\\n", " & = & \\ln 100 + \\frac{1}{4} \\ln x + \\frac{3}{4} \\ln y + \\lambda (800 -\n", " 200 x - 100 y)\n", " \\end{eqnarray*}\n", "\n", "Note: The critical value of $P (x, y)$ is\n", " also critical value of $\\ln P (x, y)$!}\n", "\n", "Now the critical point(s) is as follows:\n", "\n", "\\begin{eqnarray*}\n", " \\vec{0} & = & \\nabla L (x, y, \\lambda)\\\\\n", " & = & \\left( \\frac{\\partial L}{\\partial x}, \\frac{\\partial L}{\\partial\n", " y}, \\frac{\\partial L}{\\partial \\lambda} \\right)\\\\\n", " & = & \\left( \\frac{1}{4 x} - 200 \\lambda, \\frac{3}{4 y} - 100 \\lambda,\n", " 800 - 200 x - 100 y \\right)\\\\\n", " \\Longrightarrow & & x = \\frac{1}{800 \\lambda}, y = \\frac{3}{400 \\lambda} \n", " (\\text{ i.e. } y = 6 x)\\\\\n", " \\Longrightarrow & & x = 1 \\text{ and } y = 6\n", " \\end{eqnarray*}\n", "\n", "$(x, y) = (1, 6)$ is the only one critical point. Since $0 \\leqslant x, y\\le 8$, $P (x, y)$ has to be a maximum in such closed region.\n", " Therefore, $P (x, y)$ attains it maximum at $(1, 6)$.\n", " \n", " " ] }, { "cell_type": "code", "execution_count": 112, "metadata": {}, "outputs": [ { "name": "stdout", "output_type": "stream", "text": [ "Function of 100 x**(1/4)y**(3/4) with constraint -200*x - 100*y + 800:\n", "\n", "critical point (x,y) is (1,6)\n" ] } ], "source": [ "x,y,z,l=symbols('x y z l')\n", "expf=100\n", "f=log(100)+log(x)/4+3*log(y)/4\n", "cond=800-200*x-100*y\n", "F=f+l*cond\n", "cpts=solve(grad(F,[x,y,z]),[x,y,z])\n", "l0=solve(cond.subs({x:cpts[x],y:cpts[y]}),l)\n", "print('Function of %s with constraint %s:\\n' %('100 x**(1/4)y**(3/4)',cond))\n", "print('critical point (x,y) is (%s,%s)' %(cpts[x].subs(l,l0[0]),cpts[y].subs(l,l0[0])))\n" ] }, { "cell_type": "markdown", "metadata": {}, "source": [ "Implement Python Lagrange's Multiplier solver\n", "---" ] }, { "cell_type": "code", "execution_count": 8, "metadata": { "collapsed": true }, "outputs": [], "source": [ "def lagrangian(func,X,conditions,solvable=True):\n", " l,m=symbols(\"lambda mu\")\n", " if len(X)==2 and len(conditions)==1:\n", " L=func+l*conditions[0]\n", " cpts=solve([diff(L,x),diff(L,y),conditions[0]],[x,y,l])\n", " print(\"Function, %s, subject to %s=0\\n---\" %(func,conditions[0])) \n", " elif len(X)==3 and len(conditions)==1: \n", " L=func+l*conditions[0]\n", " cpts=solve([diff(L,x),diff(L,y),diff(L,z),conditions[0]],[x,y,z,l]) \n", " print(\"Function, %s, subject to %s=0\\n---\" %(func,conditions[0])) \n", " else:\n", " L=func+l*conditions[0]+m* conditions[1]\n", " cpts=solve([diff(L,x),diff(L,y),diff(L,z),conditions[0],conditions[1]],[x,y,z,l,m]) \n", " print(\"Function, %s, subject to %s=0 and %s=0\\n---\" %(func,conditions[0],conditions[1]))\n", " i=1\n", " vals=[]\n", " if solvable:\n", " for cpt in cpts: \n", " if len(X)==2:\n", " funcVal=func.subs({x:cpt[0],y:cpt[1]})\n", " print(\"%d֯ ). f = %s = %s at critical value (x,y)=(%s,%s)\" %(i,func,funcVal,cpt[0],cpt[1]))\n", " else:\n", " funcVal=func.subs({x:cpt[0],y:cpt[1],z:cpt[2]})\n", " print(\"%d֯ ). f = %s = %s at critical value (x,y,z)=(%s,%s,%s)\" %(i,func, funcVal,cpt[0],cpt[1],cpt[2]))\n", " vals.append(funcVal)\n", " i+=1\n", " print(\"---\\n\") \n", " print(\"Maximum on the boundary is %s\" %max(vals))\n", " print(\"Minimum on the boundary is %s\" %min(vals)) \n", " else:\n", " print(cpts) " ] }, { "cell_type": "markdown", "metadata": {}, "source": [ "Example\n", "---\n", " Suppose you wishes to allocate your available time of $16$ hours in this\n", " week between quizzes, English and Calculus, held in the next week. What\n", " would you do in such way to maximize your grade average?\n", "\n", "**Solution**\n", " \n", "Suppose that\n", "1. $f (t_1) = 20 + 20 \\sqrt{t_1} $: the time will be spent for\n", "English course with $t_1$ hour per week,\n", "- $g (t_2) = 50 + 3 t$ : the time will be spent for Calculus course\n", " with $t_2$ hour per week.\n", "\n", "Then the problem is turned to be:\n", "\n", "\\begin{eqnarray*}\n", " \\text{Maximize } S (t_1, t_2) & = & \\frac{f (t_1) + g (t_2)}{2}\\\\\n", " \\text{ subject to } & & t_1 + t_2 = 16\n", " \\end{eqnarray*}\n", "Consider the Lagrangian:\n", " $$ L (t_1, t_2) = 20 + 20 \\sqrt{t_1} + 50 + 3 t_2 + \\lambda (16 - t_1 -\n", " t_2) $$\n", "The extremum occurs at the place that satisfies:\n", "\n", "\\begin{eqnarray*}\n", " \\frac{\\partial L}{\\partial t_1} = 0, & \\frac{\\partial L}{\\partial t_2} =\n", " 0, & \\frac{\\partial L}{\\partial \\lambda} = 0.\n", " \\end{eqnarray*}\n", "And these imply:\n", "\n", "\\begin{eqnarray*}\n", " & \\frac{10}{\\sqrt{t_1}} - \\lambda = 0, & 3 - \\lambda = 0\\\\\n", " \\Longrightarrow & t_1 = \\left( \\frac{10}{3} \\right)^2 & \\text{ and } t_2 =\n", " 16 - \\left( \\frac{10}{3} \\right)^2\n", " \\end{eqnarray*}\n", "How do I know at which the maximum occurs? Since $S (t_1, t_2)$ is\n", "continuous for both $t_1$ and $t_2$ are in bounded intervals, the function\n", "obtains its maximum and minimum. Comparing with the function values at\n", "boundary, $S (t_1, t_2)$ will obtains its maximum at $(t_1, t_2) = (1,15)$ with the constraint $t_1 + t_2 = 16$.\n", "\n", "Theorem (Heron's formulae)\n", "---\n", "Suppose that The lengths of sides of triangle are $x, y$\n", "and $z$ respectively and $x + y + z = l$. Then the area of this triangle is\n", " $\\sqrt{s (s - x) (s - y) (s - z)}$ where $s = l / 2$.\n", "\n", "\n", "Example\n", "---\n", " Suppose that the perimeter is $24$. Find the dimension of this triangle such\n", " that it owns maximum area.\n", " \n", " Suppose $x, y, z$ are the lengths of sides of this triangle and let $A$ be\n", " its area. Then\n", "\n", "\\begin{eqnarray*}\n", " & \\text{Maximize} & A = \\sqrt{12 (12 - x) (12 - y) (12 - z)} \\text{ with }\n", " x + y + z = 24\\\\\n", " \\Longrightarrow & \\text{Maximize } & 12 (12 - x) (12 - y) (12 - z)\n", " \\text{ with } x + y + z = 24\\\\\n", " \\Longrightarrow & \\text{Maximize } & L (x, y, z, \\lambda) = \\ln (12 (12 -\n", " x) (12 - y) (12 - z)) + \\lambda (24 - x - y - z)\n", " \\end{eqnarray*}\n", "Since\n", "$$ L (x, y, z, \\lambda) = \\ln 12 + \\ln (12 - x) + \\ln (12 - y) + \\ln (12 -\n", " z) + \\lambda (24 - x - y - z) $$\n", " then the critical value of $L (x, y, z, \\lambda)$ can be found by the\n", " following steps:\n", "\n", "\\begin{eqnarray*}\n", " \\vec{0} & = & \\nabla L (x, y, z, \\lambda)\\\\\n", " & = & \\left( \\frac{\\partial L}{\\partial x}, \\frac{\\partial L}{\\partial\n", " y}, \\frac{\\partial L}{\\partial z}, \\frac{\\partial L}{\\partial \\lambda}\n", " \\right)\\\\\n", " & = & \\left( - \\frac{1}{12 - x} - 1, - \\frac{1}{12 - y} - 1, -\n", " \\frac{1}{12 - z} - 1, 24 - x - y - z \\right)\n", " \\end{eqnarray*}\n", " This implies that $x = y = z = 8$, i.e. this triangle is equilateral. " ] }, { "cell_type": "markdown", "metadata": {}, "source": [ "Exercise\n", "---\n", "Assume that $P (x, y) = 100 x^{1 / 4} y^{3 / 4}$. Then $\\frac{\\partial\n", " P}{\\partial x} = \\frac{25 y^{\\frac{3}{4}}}{x^{\\frac{3}{4}}}$ and\n", " $\\frac{\\partial P}{\\partial y} = \\frac{75\n", " x^{\\frac{1}{4}}}{y^{\\frac{1}{4}}}$.\n", "1. $\\frac{\\partial P}{\\partial x} (1, 6) = 25 \\sqrt[4]{6^3}$\n", "- $\\frac{\\partial P}{\\partial y} (1, 6) = 75 / \\sqrt[4]{6^{}}$\n", "- $\\frac{\\partial P}{\\partial x} (1, 6) / \\frac{\\partial P}{\\partial\n", " y} (1, 6) = 25 \\sqrt[4]{6^3} / (75 / \\sqrt[4]{6^{}}) = 6 / 3 = 2$\n", "- Let $F (x, y) = P (x, y) + \\lambda (80 - 20 x - 10 y)$. Then\n", "\n", "$$\\text{ } 0 = \\frac{\\partial F}{\\partial x} = \\frac{25\n", " y^{\\frac{3}{4}}}{x^{\\frac{3}{4}}} - 20 \\lambda \\text{ and } 0 =\n", " \\frac{\\partial F}{\\partial y} = \\frac{75\n", " x^{\\frac{1}{4}}}{y^{\\frac{1}{4}}} - 10 \\lambda $$\n", "These imply $y = 6 x. \\text{Therefore}$ $F (x, y)$ attains its maximum\n", " under the constraint $20 x + 10 y = 80$ as $x = 10$ and $y = 60$." ] }, { "cell_type": "markdown", "metadata": {}, "source": [ "Example\n", "---\n", "Find the extrema of $f(x,y)=x^2-2y$ subject to $x^2+y^2=9$.\n", "\n", "**1.** Let $L=f+\\lambda(9-x^2-y^2)=x^2-2y+\\lambda(9-x^2-y^2)$.
\n", "**2.**\n", "\\begin{eqnarray}\n", " \\vec{0} & = & \\nabla L (x, y, \\lambda)\\\\\n", " & = & \\left( \\frac{\\partial L}{\\partial x}, \\frac{\\partial L}{\\partial\n", " y}, \\frac{\\partial L}{\\partial \\lambda}\n", " \\right)\\\\\n", " & = & \\left( 2x-2\\lambda x, -2 - 2\\lambda y , 9-x^2-y^2 \\right)\n", "\\end{eqnarray}\n", "**3.**\n", " - $x=0\\to 0^2+y^2=9\\to y=\\pm3$\n", " - $\\lambda=1\\to y=-1\\to x=\\pm2\\sqrt2$\n", " - $f(0,-3)=6,f(0,3)=-6 (\\min),f(\\pm2\\sqrt2,-1)=10 (\\max)$" ] }, { "cell_type": "markdown", "metadata": {}, "source": [ "Example\n", "---\n", "Find the extrema of $f(x,y)=2x^2+y^2-2y+1$ subject to $x^2+y^2\\le4$.\n", "\n", "**0.** critical point:\n", "$$\\nabla f=(0,0)\\to(x,y)=(0,1)$$\n", "and $f(0,1)=0$.\n", "\n", "**1.** Let $L=f+\\lambda(4-x^2-y^2)=2x^2+y^2-2y+1+\\lambda(4-x^2-y^2)$.
\n", "**2.**\n", "\\begin{eqnarray}\n", " \\vec{0} & = & \\nabla L (x, y, \\lambda)\\\\\n", " & = & \\left( \\frac{\\partial L}{\\partial x}, \\frac{\\partial L}{\\partial\n", " y}, \\frac{\\partial L}{\\partial \\lambda}\n", " \\right)\\\\\n", " & = & \\left( 4x-2\\lambda x, 2y-2 - 2\\lambda y , 4-x^2-y^2 \\right)\n", "\\end{eqnarray}\n", "**3.**\n", " - $x=0\\to 0^2+y^2=4\\to y=\\pm2$\n", " - $\\lambda=2\\to y=-1\\to x=\\pm\\sqrt3$\n", " - $f(0,1)=0$ (minimum), $f(0,-2)=9,f(0,2)=1,f(\\pm\\sqrt3,-1)=10$ (maximum)." ] }, { "cell_type": "markdown", "metadata": {}, "source": [ "Example (With two constraints)\n", "---\n", "Find the extrema of $f(x,y,z)=3x+2y+4z$ subject to $x-y+2z=1$ and $x^2+y^2=4$.\n", "\n", "**1.** Let $L=f+\\lambda(1-x+y-2z)+\\mu(4-x^2-y^2)=3x+2y+4z+\\lambda(1-x+y-2z)+\\mu(4-x^2-y^2)$.
\n", "**2.** Find critical point(s):\n", "\\begin{eqnarray}\n", " \\vec{0\\!} & = & \\nabla L (x, y, z,\\lambda,\\mu)\\\\\n", " & = & \\left( \\frac{\\partial L}{\\partial x}, \\frac{\\partial L}{\\partial y}, \n", " \\frac{\\partial L}{\\partial z}, \\frac{\\partial L}{\\partial \\lambda},\n", " \\frac{\\partial L}{\\partial\\mu}\n", " \\right)\\\\\n", " & = & \\left( 3-\\lambda -2\\mu x, 2+\\lambda-2\\mu y,4-2\\lambda , ...,...\\right)\n", "\\end{eqnarray}\n", "**3.**\n", " - $\\lambda=2\\to x=1/(2\\mu) \\text{ and }y=2/\\mu$\n", " - $x^2+y^2=4\\to \\mu=\\pm\\sqrt{17}/4\\to(x,y,z)=\\left(\\pm2/\\sqrt{17},\\pm8/\\sqrt{17},\\frac{1}{2}\\left(1\\pm\\frac{6}{\\sqrt{17}}\\right)\\right)$\n", " - Maximum $f\\left(2/\\sqrt{17},8/\\sqrt{17},\\frac{1}{2}\\left(1+\\frac{6}{\\sqrt{17}}\\right)\\right)=2(1+\\sqrt{17})$,\n", " - Minimum $f\\left(-2/\\sqrt{17},-8/\\sqrt{17},\\frac{1}{2}\\left(1-\\frac{6}{\\sqrt{17}}\\right)\\right)=2(1-\\sqrt{17})$.\n", " " ] }, { "cell_type": "code", "execution_count": null, "metadata": { "collapsed": true }, "outputs": [], "source": [] }, { "cell_type": "markdown", "metadata": {}, "source": [ "Exercise\n", "---\n", "1. Find the extrema of $f (x, y) = x y^2$ with $2 x^2 + y^2 = 12$.\n", "- Find the extrema of $f (x, y, z) = 10 x + 2 y + 6 z$ with $x^2 + y^2+ z^2 = 35$." ] }, { "cell_type": "code", "execution_count": 117, "metadata": {}, "outputs": [ { "name": "stdout", "output_type": "stream", "text": [ "8*sqrt(2)\n" ] } ], "source": [ "#from sympy import jacobian\n", "#grad = lambda func, vars : Matrix(1,1,[func]).jacobian(vars)\n", "f1=x*y*y\n", "cond=12-2*x*x-y*y\n", "L=f1+l*cond\n", "cpts=solve(grad(L,[x,y,l]),[x,y,l])\n", "print(max([p[0]*p[1]*p[1] for p in cpts]))" ] }, { "cell_type": "code", "execution_count": 186, "metadata": {}, "outputs": [ { "name": "stdout", "output_type": "stream", "text": [ "Function, x*y**2, subject to -2*x**2 - y**2 + 12=0\n", "---\n", "1֯ ). f = x*y**2 = -8*sqrt(2) at critical value (x,y)=(-sqrt(2),-2*sqrt(2))\n", "2֯ ). f = x*y**2 = -8*sqrt(2) at critical value (x,y)=(-sqrt(2),2*sqrt(2))\n", "3֯ ). f = x*y**2 = 8*sqrt(2) at critical value (x,y)=(sqrt(2),-2*sqrt(2))\n", "4֯ ). f = x*y**2 = 8*sqrt(2) at critical value (x,y)=(sqrt(2),2*sqrt(2))\n", "5֯ ). f = x*y**2 = 0 at critical value (x,y)=(-sqrt(6),0)\n", "6֯ ). f = x*y**2 = 0 at critical value (x,y)=(sqrt(6),0)\n", "---\n", "\n", "Maximum on the boundary is 8*sqrt(2)\n", "Minimum on the boundary is -8*sqrt(2)\n" ] } ], "source": [ "f=x*y*y\n", "cond=12-2*x*x-y*y\n", "lagrangian(f,[x,y],[cond])" ] }, { "cell_type": "code", "execution_count": 187, "metadata": {}, "outputs": [ { "name": "stdout", "output_type": "stream", "text": [ "Function, 10*x + 2*y + 6*z, subject to -x**2 - y**2 - z**2 + 35=0\n", "---\n", "1֯ ). f = 10*x + 2*y + 6*z = -70 at critical value (x,y,z)=(-5,-1,-3)\n", "2֯ ). f = 10*x + 2*y + 6*z = 70 at critical value (x,y,z)=(5,1,3)\n", "---\n", "\n", "Maximum on the boundary is 70\n", "Minimum on the boundary is -70\n" ] } ], "source": [ "f=10*x+2*y+6*z\n", "cond=35-x*x-y*y-z*z\n", "lagrangian(f,[x,y,z],[cond])" ] }, { "cell_type": "markdown", "metadata": {}, "source": [ "Exercise p1138\n", "---\n", "**10.** Find the extrema of $f(x,y)=x^2+y^2$ with $x^4+y^4=1$." ] }, { "cell_type": "code", "execution_count": 84, "metadata": { "collapsed": true }, "outputs": [], "source": [ "from sympy import symbols,solve\n", "x,y,z,l,m=symbols(\"x y z l m\",real=True)" ] }, { "cell_type": "code", "execution_count": 51, "metadata": {}, "outputs": [ { "name": "stdout", "output_type": "stream", "text": [ "Maximum is sqrt(2)\n", "Minimum is 1\n" ] } ], "source": [ "f1=x*x+y*y\n", "cond=1-x**4-y**4\n", "L=f1+l*cond\n", "cpts=solve(grad(L,[x,y,l]),[x,y,l])\n", "print(\"Maximum is %s\" %max([p[0]*p[0]+p[1]*p[1] for p in cpts]))\n", "print(\"Minimum is %s\" %min([p[0]*p[0]+p[1]*p[1] for p in cpts]))" ] }, { "cell_type": "code", "execution_count": 184, "metadata": {}, "outputs": [ { "name": "stdout", "output_type": "stream", "text": [ "Function, x**2 + y**2, subject to -x**4 - y**4 + 1=0\n", "---\n", "1֯ ). f = x**2 + y**2 = 1 at critical value (x,y)=(-1,0)\n", "2֯ ). f = x**2 + y**2 = 1 at critical value (x,y)=(0,-1)\n", "3֯ ). f = x**2 + y**2 = 1 at critical value (x,y)=(0,1)\n", "4֯ ). f = x**2 + y**2 = 1 at critical value (x,y)=(1,0)\n", "5֯ ). f = x**2 + y**2 = sqrt(2) at critical value (x,y)=(-2**(3/4)/2,-2**(3/4)/2)\n", "6֯ ). f = x**2 + y**2 = sqrt(2) at critical value (x,y)=(-2**(3/4)/2,2**(1/4)*sqrt(2)/2)\n", "7֯ ). f = x**2 + y**2 = sqrt(2) at critical value (x,y)=(2**(1/4)*sqrt(2)/2,-2**(3/4)/2)\n", "8֯ ). f = x**2 + y**2 = sqrt(2) at critical value (x,y)=(2**(1/4)*sqrt(2)/2,2**(1/4)*sqrt(2)/2)\n", "---\n", "\n", "Maximum on the boundary is sqrt(2)\n", "Minimum on the boundary is 1\n" ] } ], "source": [ "f1=x*x+y*y\n", "cond=1-x**4-y**4\n", "lagrangian(f1,[x,y],[cond])" ] }, { "cell_type": "markdown", "metadata": {}, "source": [ "\n", "**14.** Find the extrema of $f(x,y)=x^2+y^2+z^2$ with $y-x=1$." ] }, { "cell_type": "code", "execution_count": 66, "metadata": {}, "outputs": [ { "data": { "text/plain": [ "{l: 1, x: -1/2, y: 1/2, z: 0}" ] }, "execution_count": 66, "metadata": {}, "output_type": "execute_result" } ], "source": [ "f2=x*x+y*y+z*z\n", "cond2=1+x-y\n", "L2=f2+l*cond2\n", "cpts=solve(grad(L2,[x,y,z,l]),[x,y,z,l])\n", "#print(\"Maximum is %s\" %max([p[0]*p[0]+p[1]*p[1]+p[2]*p[2] for p in cpts]))\n", "#print(\"Minimum is %s\" %min([p[0]*p[0]+p[1]*p[1]+p[2]*p[2] for p in cpts]))\n", "cpts" ] }, { "cell_type": "code", "execution_count": 183, "metadata": {}, "outputs": [ { "name": "stdout", "output_type": "stream", "text": [ "Function, x**2 + y**2 + z**2, subject to x - y + 1=0\n", "---\n", "{x: -1/2, y: 1/2, z: 0, lambda: 1}\n" ] } ], "source": [ "f2=x*x+y*y+z*z\n", "cond2=1+x-y\n", "\n", "lagrangian(f2,[x,y,z],[cond2],solvable=False)" ] }, { "cell_type": "markdown", "metadata": {}, "source": [ "Since $x^2+y^2+z^2\\ge0$, only minimum attains and at $(x,y,z)=(-1/2,1/2,0)$, i.e. min=$1/2$." ] }, { "cell_type": "markdown", "metadata": {}, "source": [ "\n", "**18.** Find the extrema of $f(x,y)=x+y+z$ with $x^2+y^2=1$ with $x+z=2$." ] }, { "cell_type": "code", "execution_count": 70, "metadata": {}, "outputs": [ { "name": "stdout", "output_type": "stream", "text": [ "Maximum is 3\n", "Minimum is 1\n" ] } ], "source": [ "f3=x+y+z\n", "cond31=1-x**2-y**2\n", "cond32=2-x-z\n", "L3=f3+l*cond31+m*cond32\n", "cpts=solve(grad(L3,[x,y,z,l,m]),[x,y,z,l,m])\n", "print(\"Maximum is %s\" %max([p[0]+p[1]+p[2] for p in cpts]))\n", "print(\"Minimum is %s\" %min([p[0]+p[1]+p[2] for p in cpts]))\n" ] }, { "cell_type": "code", "execution_count": 188, "metadata": {}, "outputs": [ { "name": "stdout", "output_type": "stream", "text": [ "Function, x + y + z, subject to -x**2 - y**2 + 1=0 and -x - z + 2=0\n", "---\n", "1֯ ). f = x + y + z = 1 at critical value (x,y,z)=(0,-1,2)\n", "2֯ ). f = x + y + z = 3 at critical value (x,y,z)=(0,1,2)\n", "---\n", "\n", "Maximum on the boundary is 3\n", "Minimum on the boundary is 1\n" ] } ], "source": [ "f3=x+y+z\n", "cond31=1-x**2-y**2\n", "cond32=2-x-z\n", "\n", "lagrangian(f3,[x,y,z],[cond31,cond32])" ] }, { "cell_type": "markdown", "metadata": {}, "source": [ "\n", "**22.** Find the extrema of $f(x,y)=x^2y$ with $4x^2+y^2\\le4$." ] }, { "cell_type": "code", "execution_count": 140, "metadata": {}, "outputs": [ { "data": { "text/plain": [ "[(0, 0)]" ] }, "execution_count": 140, "metadata": {}, "output_type": "execute_result" } ], "source": [ "f=x**2*y\n", "solve(grad(f,[x,y]),[x,y])" ] }, { "cell_type": "markdown", "metadata": {}, "source": [ "Actually, set of critical points is $(x,y)=\\{(0,y),y\\in(-2,2)\\}$, infinite points included." ] }, { "cell_type": "code", "execution_count": 9, "metadata": {}, "outputs": [ { "name": "stdout", "output_type": "stream", "text": [ "Function, x**2*y, subject to -4*x**2 - y**2 + 4=0\n", "---\n", "1֯ ). f = x**2*y = 0 at critical value (x,y)=(0,-2)\n", "2֯ ). f = x**2*y = 0 at critical value (x,y)=(0,2)\n", "3֯ ). f = x**2*y = -4*sqrt(3)/9 at critical value (x,y)=(-sqrt(6)/3,-2*sqrt(3)/3)\n", "4֯ ). f = x**2*y = 4*sqrt(3)/9 at critical value (x,y)=(-sqrt(6)/3,2*sqrt(3)/3)\n", "5֯ ). f = x**2*y = -4*sqrt(3)/9 at critical value (x,y)=(sqrt(6)/3,-2*sqrt(3)/3)\n", "6֯ ). f = x**2*y = 4*sqrt(3)/9 at critical value (x,y)=(sqrt(6)/3,2*sqrt(3)/3)\n", "---\n", "\n", "Maximum on the boundary is 4*sqrt(3)/9\n", "Minimum on the boundary is -4*sqrt(3)/9\n" ] } ], "source": [ "f=x*x*y\n", "cond=4-4*x**2-y**2\n", "lagrangian(f,[x,y],[cond])" ] }, { "cell_type": "markdown", "metadata": {}, "source": [ "After all, Maximum is $4\\sqrt3/9$, and Minimum is $-4\\sqrt3/9$.\n" ] }, { "cell_type": "code", "execution_count": null, "metadata": { "collapsed": true }, "outputs": [], "source": [] }, { "cell_type": "markdown", "metadata": {}, "source": [ "The Method of Least Squares\n", "---\n", "\n", "Suppose that there are $n$ paired data, $(x_1, y_1), (x_2, y_2), \\cdots,\n", "(x_n, y_n)$, that were observed in one experiment. Can we find the approximate\n", "relation between $X = x_i$ and $Y = y_i$?\n", "\n", "\n", " \n" ] }, { "cell_type": "markdown", "metadata": {}, "source": [ "The real relation between $X$ and $Y$ can not be found out even by rigours\n", "theory. Thus the approximation will be a good replaced solution. There are\n", "still some problems that have to be considered:\n", "1. Which relation do we want? The simple linear relation, i.e. $Y = m X +\n", " b$, is a good suggestion;\n", "- Since the relation is an approximation, the error can not be ignored.\n", " As shown in the last picture:\n", "$$l_1, l_2, \\cdots, l_5 \\text{ errors}$$\n", "\n", "The linear approximation will be a good candidate if the sum of errors\n", "\n", "$$ E = l_1 + \\cdots + l_5 $$\n", " is minimum! But different signs of errors will reduce the sum of errors.\n", " This means that the sum of errors is very small but the differences between\n", " exact values and observed data are very large. Therefore we can consider to\n", " minimize the sum of the square of errors:\n", "\n", "$$ \\color{brown}{E' = l_1^2 + \\cdots + l_5^2} $$\n", "\n", "The following famous theorem given by Gauss describes how to predict the\n", "relation from the data come from the real world:\n", "\n" ] }, { "cell_type": "markdown", "metadata": {}, "source": [ "Theorem\n", "---\n", " The line $l = m x + b$ that best fits the data points $(x_1, y_1), (x_2,\n", " y_2), \\cdots, (x_n, y_n)$ is the line for which sum of the sum of square\n", " errors\n", " $$ E_1 + E_2 + \\cdots + E_n, \\text{ where } E_i = (y_i - m x_i - b)^2 $$\n", " is a minimum and\n", "\n", "$$ m = \\frac{\\overline{x y} - \\bar{x} \\bar{y}}{\\overline{x^2} - \\bar{x}^2},\n", " b = \\frac{\\overline{x^2} \\bar{y} - \\bar{x} \\overline{x y}}{\\overline{x^2}\n", " - \\bar{x}^2} $$\n", " Here, we use the notation of \"average of variable\",\n", " $\\overline{\\color{brown}{\\cdot}}$, to represent the sum of relative\n", " variables, for example,\n", "\n", "$$ \\overline{x y} = \\frac{1}{n} \\sum_{k = 1}^n x_k y_k $$\n", "\n", "\n", "**Proof**\n", "\n", " Since the sum of total error, $E_i$, is sum of (positive)\n", " square of errors, it must attain its minimum, i.e.\n", "\n", "$$ E = \\sum_{k = 1}^n E_i = \\sum_{k = 1}^n (y_i - \\color{brown}{m} x_i -\n", " \\color{brown}{b})^2 $$\n", " $\\min E$ exists since the last sum is sum of positive squares. Then the\n", " critical value satisfies\n", "\n", "$$ 0 = \\frac{\\partial E}{\\partial m} = - 2 \\sum_{k = 1}^n x_i (y_i - m x_i -\n", " b) $$ \n", "\n", "$$ 0 = \\frac{\\partial E}{\\partial b} = - 2 \\sum_{k = 1}^n (y_i - m x_i - b)$$\n", " And these can be reduced into the following linear system equations:\n", "\n", "$$ m \\sum_{k = 1}^n x_k^2 + b \\sum_{k = 1}^n x_k = \\sum_{k = 1}^n y_k x_k\n", " \\longrightarrow \\color{brown}{m} \\overline{x^2} + \\color{brown}{b}\n", " \\bar{x} = \\overline{x y} $$\n", "\n", "$$m \\sum_{k = 1}^n x_k^{} + n b = \\sum_{k = 1}^n y_k \\longrightarrow\n", " \\color{brown}{m} \\overline{x^{}} + \\color{brown}{b} = \\overline{y}$$\n", " \n", " And by the general procedure of solving linear system of equations:\n", "\n", "\\begin{eqnarray*}\n", " m & = & \\frac{\\left|\\begin{array}{cc}\n", " \\overline{x y} & \\bar{x}\\\\\n", " \\overline{y} & 1\n", " \\end{array}\\right|}{\\left|\\begin{array}{cc}\n", " \\overline{x^2 } & \\bar{x}\\\\\n", " \\overline{x} & 1\n", " \\end{array}\\right|}\\\\\n", " & = & \\frac{\\overline{x y} - \\bar{x} \\bar{y}}{\\overline{x^2} - \\bar{x}^2}\n", " \\\\\n", " b & = & \\frac{\\left|\\begin{array}{cc}\n", " \\overline{x^2} & \\overline{x y}\\\\\n", " \\overline{x} & \\bar{y}\n", " \\end{array}\\right|}{\\left|\\begin{array}{cc}\n", " \\overline{x^2 } & \\bar{x}\\\\\n", " \\overline{x} & 1\n", " \\end{array}\\right|}\\\\\n", " & = & \\frac{\\overline{x^2} \\bar{y} - \\bar{x} \\overline{x\n", " y}}{\\overline{x^2} - \\bar{x}^2}\n", " \\end{eqnarray*}\n", " \n", " \n", " we have the result.\n", "\n", "\n" ] }, { "cell_type": "markdown", "metadata": {}, "source": [ "Example\n", "---\n", " The discount rate (in $\\%$) for 5 months beginning in June of a given year\n", " in U.S.A. is as following table:\n", " \n", "\n", " June July August September October\n", " 14 13 10 10 9\n", "\n", "1. Find the linear approximation for discount rate with respect to time\n", " (month), i.e. the least square regression.\n", "- Predict the discount rates in November and December.\n", "\n", "**Sol:** Let $x_i$ be the number of $i$-months after June with $y_i$\n", "be the discount rate during this month, then we\n", "have\n", "\n", "$$ \\sum_{i = 1}^5 x_i = 10 ; \\sum_{i = 1}^5 y_i = 56 ; \\sum_{i = 1}^5 x_i^2 =\n", " 30 ; \\sum_{i = 1}^5 x_i y_i = 99. $$\n", "\n", "\n", "**1. **\n", "\n", "$$y = m x + b = \\frac{5 \\cdot 99 - 10 \\cdot 56}{5 \\cdot 30^{} - 10^2} x\n", " + \\frac{30 \\cdot 56 - 10 \\cdot 99}{5 \\cdot 30^{} - 10^2} = - \\frac{13}{10} x\n", " + \\frac{69}{5}$$\n", "\n", "**2.** Since Nov. and Dec. $x'$s are 5 and 6 respectively, we have\n", "\n", "$$ y_{\\text{Nov}} = - \\frac{13}{10} 5 + \\frac{69}{5} = 7.3\\%;\\\\ y_{\\text{Dec}}\n", " = - \\frac{13}{10} 6 + \\frac{69}{5} = 6\\% $$" ] }, { "cell_type": "code", "execution_count": 126, "metadata": {}, "outputs": [ { "data": { "image/png": "iVBORw0KGgoAAAANSUhEUgAAAgAAAAFkCAYAAABW9YMrAAAABHNCSVQICAgIfAhkiAAAAAlwSFlz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\n2WIM2h0AAAAASUVORK5CYII=\n", "text/plain": [ "" ] }, "metadata": {}, "output_type": "display_data" } ], "source": [ "x=np.arange(0,5,1)\n", "y=[14, 13, 10, 10, 9]\n", "A=np.column_stack([x,np.ones_like(x)])\n", "c,resid,rank,sigma=np.linalg.lstsq(A,y)\n", "z=c[0]*x+c[1]*np.ones(len(x))\n", "plt.plot(x,z)\n", "plt.scatter(x,y)\n", "for i in range(len(x)):\n", " plt.plot([x[i],x[i]],[z[i],y[i]],'r--')" ] }, { "cell_type": "markdown", "metadata": {}, "source": [ "Example\n", "---\n", " Suppose the last 7 records of average week stock price for certain a company\n", " (PRIME VIEW INTERNATIONAL CO., LTD.) in Taiwan is listed as follows:\n", " $$ \\begin{array}{llllllll}\n", " \\text{Week} & 3 / 26 \\sim 3 / 30 & 4 / 2 \\sim 4 / 6 & 4 / 9 \\sim 4 / 13\n", " & 4 / 16 \\sim 4 / 20 & 4 / 23 \\sim 4 / 27 & 4 / 30 \\sim 5 / 4 & 5 / 7\n", " \\sim 5 / 11\\\\\n", " \\text{Price} & 17.8 & 19.6 & 21.9 & 20.85 & 23.0 & 23.6 & 24.4\n", " \\end{array} $$\n", "1. Find the linear approximation of the relation between time (in week)\n", " and stock price.\n", "- Predict the stock price that will be in the next week (i.e. $5 / 14\n", " \\sim 5 / 18$). (The real value is 25.85)\n", "\n", "**Sol:**\n", " \n", " Suppose that $(x_i, y_i)$ be the paired (week,price) record for $i = 1, 2,\n", " 3, 4, 5, 6, 7$. Then\n", "$$ \\sum_{i = 1}^7 x_i = 28, \\sum_{i = 1}^7 y_i = 151.15, \\sum_{i = 1}^7\n", " x_i^2 = 140, \\sum_{i = 1}^7 x_i y_i = 642.5 $$\n", "\n", "**1.**\n", "\n", "\\begin{eqnarray*}\n", " m & = & \\frac{\\overline{x y} - \\bar{x} \\bar{y}}{\\overline{x^2} -\n", " \\bar{x}^2}\\\\\n", " & \\sim & 1.032\\\\\n", " b & = & \\frac{\\overline{x^2} \\bar{y} - \\bar{x} \\overline{x\n", " y}}{\\overline{x^2} - \\bar{x}^2}\\\\\n", " & \\sim & 17.46\n", " \\end{eqnarray*}\n", "\n", "This means\n", "$$ y = 1.032 x + 17.46 $$\n", "\n", "**2.** The price in the next week will be approximately estimated as:\n", "\n", "$$ 1.032 \\cdot 8 + 17.46 = 25.72 $$\n", "\n", "Comparing with the real value $(25.85)$, the error is within $1\\%$!\n" ] }, { "cell_type": "markdown", "metadata": {}, "source": [ "Exercise\n", "---\n", " Find the least square approximations for the following data:\n", "1. $(x_{i,} y_i) = (0, 20), (2, 24), (4, 25), (6, 32), (8, 34)$ for\n", " i=1,2,3,4,5\n", "- $(x_{i,} y_i) = (0, 160), (1, 164), (2, 168), (3, 171), (4, 175)$\n", " for i=1,2,3,4,5\n", "\n", "**Answer**\n", "\n", "1.$$\\sum_{i = 1}^5 x_i = 20 ; \\sum_{i = 1}^5 y_i = 135 ; \\sum_{i = 1}^5\n", " x_i^2 = 120 ; \\sum_{i = 1}^5 x_i y_i = 612. $$\n", " Therefore $$y = m x + b = \\frac{9}{5} x + \\frac{99}{5}\\text{ and }y_{1994} =\n", " \\frac{189}{5}y_{2000} = \\frac{243}{5}$$\n", "**2.** $\\sum_{i = 1}^5 x_i = 10 ; \\sum_{i = 1}^5 y_i = 838 ; \\sum_{i = 1}^5\n", " x_i^2 = 30 ; \\sum_{i = 1}^5 x_i y_i = 1713$. \n", " Therefore $y = m x + b =\n", " \\frac{37}{10} x + \\frac{801}{5}$ and $y_{1995} =\\left. y\\right|_{x = 5} = 178.7$\n", " $y_{1998} = \\left.y\\right|_{x = 5.6} = 180.92$\n" ] }, { "cell_type": "code", "execution_count": 1, "metadata": {}, "outputs": [ { "name": "stdout", "output_type": "stream", "text": [ "[NbConvertApp] Converting notebook 6 Multi-variable Calculus Differentiation-3.ipynb to html\n", "[NbConvertApp] Writing 2913867 bytes to 6 Multi-variable Calculus Differentiation-3.html\n" ] } ], "source": [ "!jupyter nbconvert 6*-3.ipynb" ] }, { "cell_type": "code", "execution_count": null, "metadata": { "collapsed": true }, "outputs": [], "source": [] } ], "metadata": { "kernelspec": { "display_name": "VPython", "language": "python", "name": "vpython" }, "language_info": { "codemirror_mode": { "name": "ipython", "version": 3 }, "file_extension": ".py", "mimetype": "text/x-python", "name": "python", "nbconvert_exporter": "python", "pygments_lexer": "ipython3", "version": "3.6.0" }, "widgets": { "state": {}, "version": "2.0.10" } }, "nbformat": 4, "nbformat_minor": 1 }